Sie sind auf Seite 1von 598

V

V O
O L
L U
U M
M E
E N
N U
U N
N O
O

Matemáticas
un enfoque de resolución de problemas
para Maestros de Educación Básica
d é c i m a e d i c i ó n

BILLSTEIN • LIBESKIND • LOTT


vers ión e n español
M A N U E L L Ó P E Z M AT E O S
© Lopez Mateos Editores. ISBN 978-607-95583-2-1, obra completa, versión electrónica, ISBN 978-607-95583-3-8, volumen 1, versión electrónica. Ejemplar asignado a: Helecto Villarroel gutierrez -
helecto@gmail.com. Fecha: 27 de octubre de 2014. Prohibida su modificación, copia o distribución.
Matemáticas, un enfoque de resolución de problemas, décima edición
Este libro está diseñado para cubrir tus requerimientos
educativos como futuro maestro de educación básica o
media. Para ello, hemos incluido algunas características
clave a fin de prepararte mejor para cuando seas un
maestro con tu propio salón de clase.

Páginas de muestra de libros de texto a las que se hace


referencia a lo largo del libro para ilustrar cómo se expli-
can en realidad las matemáticas a los alumnos de K-8.

Actividades de laboratorio integradas a lo largo del libro para pro-


porcionar ejercicios prácticos.

© Lopez Mateos Editores. ISBN 978-607-95583-2-1, obra completa, versión electrónica, ISBN 978-607-95583-3-8, volumen 1, versión electrónica. Ejemplar asignado a: Helecto Villarroel gutierrez -
helecto@gmail.com. Fecha: 27 de octubre de 2014. Prohibida su modificación, copia o distribución.
Ahora intenta éste: Son proble-
mas que aparecen a lo largo de
cada capítulo. Esta característica lo
ayuda a participar de manera activa
en su aprendizaje, a desarrollar ha-
bilidades para resolver problemas,
y a estimular las discusiones.

Conjuntos de problemas: Hay cinco tipos diferentes de proble-


mas para repasar la comprensión matemática y para desarrollar la
habilidad de explicar la matemática a otras personas. Los tipos de
problemas son:
(1) Comunicación
(2) Respuesta abierta
(3) Aprendizaje colectivo
(4) Preguntas del salón de clase
(5) Preguntas de repaso.
También se incluyen ejercicios del Third International Mathemat-
ics and Science Study (TIMSS) (Tercer Estudio Internacional sobre
las Matemáticas y la Ciencia) y del
National Assessment of Educational
Progress (NAEP) (Evaluación Na-
cional del Progreso Educativo).

© Lopez Mateos Editores. ISBN 978-607-95583-2-1, obra completa, versión electrónica, ISBN 978-607-95583-3-8, volumen 1, versión electrónica. Ejemplar asignado a: Helecto Villarroel gutierrez -
helecto@gmail.com. Fecha: 27 de octubre de 2014. Prohibida su modificación, copia o distribución.
Formas ocultas
Estudia la figura que se muestra a continuación, o la de la tapa
del libro, para ver si puedes hallar las formas siguientes:
1. Un cubo
2. Un cilindro
3. Una pirámide cuadrada
4. Una estrella de 5 picos
5. Un prisma rectangular
6. Un cono

Las respuestas se muestran en la tapa posterior del libro.

© Lopez Mateos Editores. ISBN 978-607-95583-2-1, obra completa, versión electrónica, ISBN 978-607-95583-3-8, volumen 1, versión electrónica. Ejemplar asignado a: Helecto Villarroel gutierrez -
helecto@gmail.com. Fecha: 27 de octubre de 2014. Prohibida su modificación, copia o distribución.
V O L U M E N U N O

Matemáticas
un enfoque de resolución de problemas
para Maestros de Educación Básica
D É C I M A E D I C I Ó N

RICK BILLSTEIN
Un ivers idad de Montana

SHLOMO LIBESKIND
Un ivers idad de Ore gon

J O H N N Y W. L O T T
Un ivers idad de Miss iss ippi

vers ión e n español


M A N U E L L Ó P E Z M AT E O S
c o n la c olaborac ión de
L O U R D E S C L A U D I A PA T I Ñ O R O M Á N
J U L I O C É SAR SALA Z AR GARC Í A
editores

López
Mateos
© Lopez Mateos Editores. ISBN 978-607-95583-2-1, obra completa, versión electrónica, ISBN 978-607-95583-3-8, volumen 1, versión electrónica. Ejemplar asignado a: Helecto Villarroel gutierrez -
helecto@gmail.com. Fecha: 27 de octubre de 2014. Prohibida su modificación, copia o distribución.
Authorized translation from the English language edition, entitled PROBLEM SOLVING APPROACH TO MATHEMATICS
FOR ELEMENTARY SCHOOL TEACHERS, A, 10th Edition by RICK BILLSTEIN; SHLOMO LIBESKIND; JOHNNY
LOTT, published by Pearson Education, Inc., publishing as Addison-Wesley, Copyright © 2010 Pearson Education, Inc.

Traducción autorizada de la edición en inglés titulada PROBLEM SOLVING APPROACH TO MATHEMATICS FOR ELE-
MENTARY SCHOOL TEACHERS, A, décima edición por RICK BILLSTEIN; SHLOMO LIBESKIND; JOHNNY LOTT,
publicada por Pearson Education, Inc., bajo Addison-Wesley Higher Education, Copyright © 2010 Pearson Education, Inc.

Traducción Manuel López Mateos


Corrección del texto José María Fábregas Puig
Corrección técnica Lourdes Claudia Patiño Román, Julio César Salazar García
Formación Constancio Hernández García
Formación de las páginas de muestra Víctor Andrés Hernández Patiño
Revisión de páginas finales Libia López Mateos Cortés

En la página 515, que es parte de esta página legal, se agradece gentilmente a los propietarios de los derechos el permiso para
usar su material registrado y se da el crédito correspondiente a los diseñadores de la edición en inglés.

Décima edición, 2012


© 2012 López Mateos Editores, s.a. de c.v.
Ave. Insurgentes Sur 1863-301
Guadalupe Inn
Álvaro Obregón, D. F.
C.P. 01020
México

ISBN 978-607-95583-2-1. Obra completa, versión electrónica.

ISBN 978-607-95583-3-8. Volumen uno, versión electrónica.

Información para catalogación bibliográfica:


Billstein, Rick.
MATEMÁTICAS: Un enfoque de resolución de problemas para maestros de educación básica, Vol. I /
Rick Billstein, Shlomo Libeskind, Johnny W. Lott / Manuel López Mateos Tr.—10a ed.
xii–520 p. 20.2x25.4cm.
ISBN 978-607-95583-2-1. Obra completa, versión electrónica. ISBN 978-607-95583-3-8. Volumen uno, versión electrónica.
1. Matemáticas—Aprendizaje y enseñanza (básica) 2. Resolución de problemas—Aprendizaje y enseñanza (básica) 3. For-
mación de maestros—Actualización 4. Educación básica I. Libeskind, Shlomo. II. Lott, Johnny W., 1944- III. Título.

All rights reserved. No part of this book may be reproduced or transmitted in any form or by any means, electronic or mecha-
nical, including photocopying, recording, or by any information storage retrieval system, without permission from Pearson
Education, Inc.

Electronic SPANISH language edition published by López Mateos Editores. Copyright © 2012.

Todos los derechos reservados. Queda prohibido reproducir o transmitir todo o parte de este libro, en cualquier forma o por
cualquier medio, electrónico o mecánico, incluyendo fotocopia, grabado o cualquier sistema de almacenamiento y recuperación
de información, sin permiso de Pearson Education, Inc.

Edición electrónica en ESPAÑOL publicada por López Mateos Editores. Copyright © 2012.

Producido en México.
editores

López
Mateos ISBN 978-607-95583-2-1. Obra completa, versión electrónica.
ISBN 978-607-95583-3-8. Volumen uno, versión electrónica.
www.lopezmateos.mx

© Lopez Mateos Editores. ISBN 978-607-95583-2-1, obra completa, versión electrónica, ISBN 978-607-95583-3-8, volumen 1, versión electrónica. Ejemplar asignado a: Helecto Villarroel gutierrez -
helecto@gmail.com. Fecha: 27 de octubre de 2014. Prohibida su modificación, copia o distribución.
Para todos los estudiantes y maestros que han usado
este libro desde su origen—RWB, SL y JWL

Para Jane, por su paciencia durante estas 10 ediciones—RB

A la memoria de mi amado abuelo Itzhak Bial/owa˛s y mi querido


tío Marian Bial/owa˛s—SL

Para la siguiente generación de estudiantes de matemáticas, incluyendo a Hamilton Grey Lott,


William Thomas Falk y Grant Warren Falk—JWL

© Lopez Mateos Editores. ISBN 978-607-95583-2-1, obra completa, versión electrónica, ISBN 978-607-95583-3-8, volumen 1, versión electrónica. Ejemplar asignado a: Helecto Villarroel gutierrez -
helecto@gmail.com. Fecha: 27 de octubre de 2014. Prohibida su modificación, copia o distribución.
Contenido
Prefacio a la edición en español vii
Prefacio viii
Agradecimientos xii

CAPÍTULO 1 Una introducción a la resolución de problemas 1


1-1 Matemáticas y resolución de problemas 3
1-2 Exploración con patrones 22
*1-3 Razonamiento y lógica: una introducción 42
Resumen y revisión del capítulo 56

CAPÍTULO 2 Sistemas de numeración y conjuntos 61


2-1 Sistemas de numeración 62
2-2 Descripción de conjuntos 78
2-3 Otras operaciones entre conjuntos y sus propiedades 93
Resumen y revisión del capítulo 106

CAPÍTULO 3 Números completos y sus operaciones 110


3-1 Suma y resta de números completos 111
3-2 Algoritmos para la suma y la resta de números completos 127
3-3 Multiplicación y división de números completos 142
3-4 Algoritmos para multiplicar y dividir números completos 161
3-5 Matemática mental y estimación 178
Resumen y revisión del capítulo 189

CAPÍTULO 4 Razonamiento algebraico 194


4-1 Variables 197
4-2 Ecuaciones 206
4-3 Funciones 220
Resumen y revisión del capítulo 244

CAPÍTULO 5 Enteros y teoría de números 248


5-1 Los enteros y las operaciones de suma y resta 250
5-2 Multiplicación y división de enteros 269
5-3 Divisibilidad 285
5-4 Números primos y compuestos 300

* Sección optativa
iv
© Lopez Mateos Editores. ISBN 978-607-95583-2-1, obra completa, versión electrónica, ISBN 978-607-95583-3-8, volumen 1, versión electrónica. Ejemplar asignado a: Helecto Villarroel gutierrez -
helecto@gmail.com. Fecha: 27 de octubre de 2014. Prohibida su modificación, copia o distribución.
Contenido v

5-5 Máximo divisor común y mínimo múltiplo común 315


*5-6 Aritmética de reloj y modular 329
Resumen y revisión del capítulo 336

CAPÍTULO 6 Los números racionales como fracciones 340


6-1 El conjunto de los números racionales 342
6-2 Suma, resta y estimación con números racionales 363
6-3 Multiplicación y división de números racionales 381
Resumen y revisión del capítulo 405

CAPÍTULO 7 Decimales y números reales 409


7-1 Introducción a los decimales 411
7-2 Operaciones con decimales 421
7-3 Decimales que no terminan 439
7-4 Números reales 450
*7-5 Uso de los números reales en ecuaciones 461
Resumen y revisión del capítulo 472

CAPÍTULO 8 Razonamiento proporcional, porcentajes y aplicaciones 476


8-1 Razones, proporciones y razonamiento proporcional 477
8-2 Porcentajes 489
*8-3 Cálculo de interés 505
Resumen y revisión del capítulo 512

CAPÍTULO 9 Probabilidad 515 V O L U M E N D O S


9-1 Cómo se determinan las probabilidades 517
9-2 Experimentos multietapa con diagramas de árbol
y probabilidades geométricas 534
9-3 Uso de simulaciones en probabilidad 555
9-4 Momios, probabilidad condicional y valor esperado 564
9-5 Uso de permutaciones y combinaciones en probabilidad 575
Resumen y revisión del capítulo 587

CAPÍTULO 10 Análisis de datos/Estadística: una introducción 592


10-1 Presentación de datos: Parte I 594
10-2 Presentación de datos: Parte II 615
10-3 Medición de la tendencia central y la variación 630
10-4 Abusos de la estadística 658
~10-5 Diseño de experimentos/Recolección de datos
Resumen y revisión del capítulo 671

* Sección optativa
~ Sección disponible en www.lopezmateos.mx/Billstein10einfo

© Lopez Mateos Editores. ISBN 978-607-95583-2-1, obra completa, versión electrónica, ISBN 978-607-95583-3-8, volumen 1, versión electrónica. Ejemplar asignado a: Helecto Villarroel gutierrez -
helecto@gmail.com. Fecha: 27 de octubre de 2014. Prohibida su modificación, copia o distribución.
vi Contenido

CAPÍTULO 11 Introducción a la geometría 678


11-1 Nociones básicas 680
11-2 Polígonos 697
11-3 Más acerca de ángulos 710
11-4 Geometría en tres dimensiones 726
~11-5 Redes
Resumen y revisión del capítulo 741

CAPÍTULO 12 Construcciones, congruencia y semejanza 746


12-1 De congruencia a construcciones 747
12-2 Otras propiedades de la congruencia 768
12-3 Otras construcciones 780
12-4 Triángulos semejantes y figuras semejantes 793
12-5 Rectas y ecuaciones lineales en un sistema coordenado cartesiano 810
~12-6 Razones trigonométricas vía semejanza
Resumen y revisión del capítulo 832

CAPÍTULO 13 Conceptos de medición 837


13-1 Medición lineal 838
13-2 Área de polígonos y círculos 854
13-3 El teorema de Pitágoras, la fórmula de la distancia
y la ecuación de un círculo 876
13-4 Área de superficie 894
13-5 Volumen, masa y temperatura 906
Resumen y revisión del capítulo 928

CAPÍTULO 14 Geometría del movimiento y embaldosados 935


14-1 Traslaciones y rotaciones 937
14-2 Reflexiones y reflexiones deslizadas 954
14-3 Homotecias 967
14-4 Simetrías 978
14-5 Embaldosados del plano 990
Resumen y revisión del capítulo 1003

Continuación de la página legal 515

Respuesta a los problemas R-1

Índice I-1

~ Sección disponible en www.lopezmateos.mx/Billstein10einfo

© Lopez Mateos Editores. ISBN 978-607-95583-2-1, obra completa, versión electrónica, ISBN 978-607-95583-3-8, volumen 1, versión electrónica. Ejemplar asignado a: Helecto Villarroel gutierrez -
helecto@gmail.com. Fecha: 27 de octubre de 2014. Prohibida su modificación, copia o distribución.
Prefacio a la edición en español
La pertinencia de la versión en español de este libro, presentado en dos volúmenes, que es uno de los
más populares en su materia en Estados Unidos, se debe a la preocupante carencia de textos para la for-
mación de profesores de matemáticas en el ámbito de habla hispana. Al cubrir los contenidos de mate-
máticas de la currícula de la educación básica, se convierte en el libro de texto ideal para la formación de
maestros; pero no sólo eso, también se convierte en el soporte adecuado para el proceso de actualización
de maestros de educación básica en servicio, para que, con un conocimiento sólido de los contenidos
académicos de matemáticas, los maestros adquieran confianza y seguridad en los cursos que imparten,
mejoren su metodología y capacidad didáctica y, finalmente, estén en óptimas condiciones para acoplarse
a la inevitable evolución de los planes y programas de estudio.

OBSERVACIONES
En aras de tener una versión en español apegada al espíritu de la edición original, se ha mantenido el
diseño gráfico, traduciendo el contenido de las páginas de libros de texto estadounidenses de educa-
ción básica incluidas como muestra. Dichas obras no existen en español. Asimismo, se ha preservado
la diversidad empleada por los autores en el uso de unidades en ejemplos y ejercicios, así como las
fuentes originales de los datos utilizados en el manejo de la estadística y la probabilidad. Los maestros
podrán sugerir como actividad la búsqueda de bases de datos locales para ilustrar ciertos temas.

Se ha respetado la denominación de los conjuntos de números usada por los autores en la edición origi-
nal, en la que introducen el término de números completos para los enteros no negativos (es decir, los
naturales junto con el cero). Así, los conjuntos de números usados son los números naturales: 1, 2,
3,…, los números completos: 0, 1, 2, 3,… , y los números enteros: Á , - 3, - 2, - 1, 0, 1, 2, 3, Á .

Para que el lector de habla hispana se ubique en el contexto de los niveles de educación básica emplea-
dos por los autores y referidos al sistema educativo de Estados Unidos, presentamos la siguiente tabla
de equivalencias:

Edad 3 4 5 6 7 8 9 10 11 12 13 14 15 16 17

México Pre1 Pre2 Pre3 1 2 3 4 5 6 1S 2S 3S 1B 2B 3B

EUA PreK K 1 2 3 4 5 6 7 8 9 10 11 12

La referencia en todo el libro es al sistema educativo de Estados Unidos, es decir, a los grados de preK a
12. En casi todo el ámbito iberoamericano la educación básica se divide en dos o tres años de educación
preescolar (de 3 a 5 años), equivalente a preK (prekindergarten) y K (kindergarten); seis años de educa-
ción primaria, que coinciden con los grados 1-6 de Estados Unidos; tres años de educación secundaria,
que coinciden con los grados 7-9; y tres años de bachillerato, equivalentes a los grados 10-12.

Para esta edición, contamos con la invaluable colaboración profesional del Mtro. José María Fábregas
Puig en la corrección del texto, de Julio César Salazar García en la revisión técnica y del Dr. Constancio
Hernández García en la formación.

M.L.M.

vii
© Lopez Mateos Editores. ISBN 978-607-95583-2-1, obra completa, versión electrónica, ISBN 978-607-95583-3-8, volumen 1, versión electrónica. Ejemplar asignado a: Helecto Villarroel gutierrez -
helecto@gmail.com. Fecha: 27 de octubre de 2014. Prohibida su modificación, copia o distribución.
viii Prefacio

Prefacio
La décima edición de MATEMÁTICAS: Un enfoque de resolución de problemas para maestros de educación
básica está diseñada para cubrir las necesidades de capacitación de los prospectos de maestros de educa-
ción básica, quienes serán los mentores de alta calidad en el futuro. Esta edición mantiene su
orientación de basarse fuertemente en el desarrollo de conceptos y habilidades, con un nuevo énfasis en
el aprendizaje activo y colectivo. Se revisó y actualizó el contenido a fin de preparar a los estudiantes
para cuando ocupen, como maestros, su propio salón de clase.

OBJETIVOS DEL NCTM


• Principios y objetivos Nos enfocamos en la publicación del National Council of Teachers of Mathe-
matics (Consejo Nacional de Maestros de Matemáticas de Estados Unidos) (NCTM), Principles and
Standards of School Mathematics (Principios y objetivos para matemáticas escolares) (2000) (referidos
de ahora en adelante como Principios y objetivos).
¡Nuevo! • Puntos focales en el currículo El National Council of Teachers of Mathematics (Consejo Nacional de
Maestros de Matemáticas de Estados Unidos) publicó en 2006 Curriculum Focal Points for Pre-kinder-
garten through Grade 8 Mathematics (Puntos focales en el currículo de matemáticas, de preescolar al
grado 8), donde describe los conceptos y habilidades matemáticos esenciales con los que se relacio-
nan las matemáticas de cada capítulo. En todo el texto hacemos referencia a los Puntos focales.
El texto completo de NCTM Principles and Standards y de Curriculum Focal Points se puede encontrar
en Internet, en www.nctm.org.

NUESTROS OBJETIVOS
• Presentar las matemáticas apropiadas de manera intelectualmente honesta y
matemáticamente correcta.
• Usar la resolución de problemas como parte integral de las matemáticas.
• Presentar las matemáticas en un orden tal que inspiren confianza al estudiante y al mismo tiempo
signifiquen un reto para él.
• Presentar formas alternativas de enseñanza y aprendizaje.
• Presentar problemas que deban exponerse para desarrollar la habilidad en la expresión escrita y
permitan que los estudiantes expliquen en voz alta.
• Estimular la incorporación de herramientas tecnológicas.
• Presentar aspectos centrales de las matemáticas a los prospectos de maestros de educación básica y
media de manera que les intrigue y se pregunten por qué las matemáticas se hacen como se hacen.
• Proporcionar aspectos centrales de las matemáticas que permitan a los maestros usar métodos inte-
grados con contenido.
• Ayudar a los futuros maestros a conectar las matemáticas, sus ideas y sus aplicaciones.
La décima edición permite que los maestros utilicen diversos métodos de enseñanza, estimula la dis-
cusión y la colaboración entre los estudiantes y entre éstos y sus maestros, y permite incorporar pro-
yectos de investigación al currículo. Lo más importante es que promueve el descubrimiento y el
aprendizaje activo, tanto para estudiantes como para maestros.

LO NUEVO EN ESTA EDICIÓN


• Como el razonamiento algebraico es tan importante en todos los niveles, incluimos un nuevo capítulo
separado sobre el tema, el capítulo 4 “Razonamiento algebraico”, continuando así la integración del
álgebra a lo largo del libro.
• Se añadió un capítulo aparte, el capítulo 8 “Razonamiento proporcional, porcentajes y
aplicaciones”, para satisfacer más ampliamente las necesidades de los futuros maestros de enseñanza
media.
• Las evaluaciones están mejor organizadas, de manera más lógica y fácilmente accesibles. En el texto
se da la respuesta a los problemas en la Evaluación A de manera que los estudiantes puedan revisar
su trabajo. En la Evaluación B hay problemas similares a los de la Evaluación A, pero no se dan las
respuestas. Al crear conjuntos paralelos de ejercicios incrementamos el número de problemas y da-
mos más oportunidad de escoger a los maestros.
© Lopez Mateos Editores. ISBN 978-607-95583-2-1, obra completa, versión electrónica, ISBN 978-607-95583-3-8, volumen 1, versión electrónica. Ejemplar asignado a: Helecto Villarroel gutierrez -
helecto@gmail.com. Fecha: 27 de octubre de 2014. Prohibida su modificación, copia o distribución.
Prefacio ix

Los problemas de conexiones matemáticas se colocaron aparte pues suelen tener soluciones abiertas y
permiten a los alumnos y al maestro trabajar solos o en grupo para hallar posibles soluciones. Están
divididos en las siguientes categorías: Comunicación, Solución abierta, Aprendizaje colectivo, Preguntas del
salón de clase y Repaso. Los conjuntos de problemas también incluyen ejemplos de preguntas de las
pruebas TIMMS y NAEP, de modo que los futuros maestros puedan examinar el tipo de preguntas
que se plantean a los estudiantes en los exámenes nacionales (de Estados Unidos) e internacionales.
• Se actualizó la parte de análisis de datos y razonamiento probabilístico —se amplió el material y se
incluyó más contenido sobre poblaciones, muestreo y encuestas.

ASPECTOS DEL CONTENIDO


Volumen I
Capítulo 1 Una introducción a la resolución de problemas
Al reorganizar este capítulo colocamos primero el tema de matemáticas y la resolución de problemas,
seguido de una sección ampliada sobre exploración de patrones. Se añadieron nuevos problemas y pá-
ginas de muestra, así como una nueva sección de sucesiones de Fibonacci. Se incluye la sección final
sobre razonamiento y lógica para quienes quieran seguir estos temas durante el curso.

Capítulo 2 Sistemas de numeración y conjuntos


Este capítulo se abrevió y reorganizó. El desarrollo de los sistemas de numeración está ahora en la
primera sección debido al desarrollo histórico de los sistemas, que existieron mucho antes de que se
desarrollaran conceptos más formales de conjuntos. El capítulo incluye más adelante todos los
conceptos tradicionales de conjuntos.

Capítulo 3 Números completos y sus operaciones


Este capítulo explora los números completos y las operaciones entre ellos. Varios algoritmos se anali-
zan y explican en detalle. Se destacan la matemática mental y la estimación con números completos.

Capítulo 4 Razonamiento algebraico


En respuesta al gran énfasis puesto en el aprendizaje y enseñanza del álgebra a lo largo del currículo
de la escuela elemental, se añadió un nuevo capítulo sobre razonamiento algebraico. Sólo se usan nú-
meros completos, pero en cada capítulo subsecuente se refuerza el razonamiento algebraico cuando
se introducen los números enteros, los racionales y finalmente los números reales. También se
refuerza el razonamiento algebraico en el capítulo sobre probabilidad y estadística, así como en los
capítulos sobre geometría.

Capítulo 5 Enteros y teoría de números


Este capítulo trata con enteros y las operaciones entre ellos. Se introducen con explicaciones nuevos
modelos para operaciones y algoritmos con enteros. La divisibilidad y los números primos se estudian
junto con explicaciones acerca de por qué funcionan las reglas de la divisibilidad. Se presentan el
máximo divisor común y el mínimo múltiplo común. Hay una sección optativa sobre aritmética del re-
loj, o modular, dedicada a quienes quieran examinar la manera en que funciona un sistema numérico di-
ferente.

Capítulo 6 Números racionales como fracciones


Nuevos ejemplos en este capítulo hacen énfasis en las habilidades algebraicas por medio de la simpli-
ficación de expresiones algebraicas y la resolución de ecuaciones y de problemas planteados mediante
alguna situación. Se resalta el concepto de división mediante explicaciones y ejemplos mejor trabaja-
dos. Se repasan las funciones con dominio en los números racionales.

Capítulo 7 Decimales y números reales


Este capítulo se abrevió al añadir un nuevo capítulo, el 8. Se añadieron más páginas de muestra; una
nueva sección optativa, “Uso de números reales en ecuaciones”, agrega un énfasis algebraico a este
reorganizado capítulo.

Capítulo 8 Razonamiento proporcional, porcentajes y aplicaciones


Debido a que el razonamiento proporcional y los porcentajes son tan importantes en la enseñanza me-
dia, se dedica todo un capítulo al tema. El capítulo incluye una explicación de por qué la relación entre
© Lopez Mateos Editores. ISBN 978-607-95583-2-1, obra completa, versión electrónica, ISBN 978-607-95583-3-8, volumen 1, versión electrónica. Ejemplar asignado a: Helecto Villarroel gutierrez -
helecto@gmail.com. Fecha: 27 de octubre de 2014. Prohibida su modificación, copia o distribución.
x Prefacio

dos razones es multiplicativa en lugar de aditiva, y por qué esto es importante. Se amplía el trabajo con
porcentajes y se incluyen las barras de porcentajes y estimaciones con porcentajes. Se incluye una sec-
ción optativa sobre cálculo de intereses para ilustrar una aplicación de los porcentajes.

Volumen II

Capítulo 9 Probabilidad
El problema preliminar, que incluye una obra de François Morellet, da indicios de que la probabi-
lidad se usa en el mundo real y en el mundo que los alumnos experimentan. Se añadieron páginas de
muestra para ilustrar cómo aparecen los conceptos en cada grado; los conceptos se ilustran con dibu-
jos, tiras cómicas y diagramas.

Capítulo 10 Análisis de datos/Estadística: una introducción


Se ha hecho énfasis en las Indicaciones para la evaluación e instrucción para la educación en estadís-
tica: Un marco curricular de Pre K a 12 (Guidelines for Assessment and Instruction in Statistics Education
(GAISE) Report: A Pre-K–12 Curriculum Framework) de la Asociación Estadística de Estados Unidos (
the American Statistical Association) (2005). Se desarrolla una sección, “Diseño de experimentos y reco-
lección de datos”, basada en este marco estadístico, con acceso mediante Internet. Se agregan muchos
nuevos problemas y se utilizan nociones algebraicas en el desarrollo del capítulo.

Capítulo 11 Introducción a la geometría


Los variados conceptos de geometría se explican de manera más minuciosa y hay un tratamiento más
detallado de los ángulos interiores y exteriores de polígonos convexos. A lo largo del capítulo se des-
taca el pensamiento algebraico.

Capítulo 12 Construcciones, congruencia y semejanza


El estudio sobre la congruencia y no congruencia de triángulos se amplió para incluir el caso
ambiguo LLA; también se añadió el tema de la congruencia de cuadriláteros. El estudio de los siste-
mas de ecuaciones lineales se amplió para incluir una explicación algebraica acerca de cuándo un sis-
tema de dos ecuaciones con dos incógnitas no tiene solución y cuándo tiene infinidad de soluciones.

Capítulo 13 Conceptos de medición


En este capítulo se trabaja tanto con el sistema inglés como con el sistema métrico, junto con conver-
siones dentro de los sistemas y entre ellos. Se incluyen mediciones lineales, de área, de volumen, de
masa y de temperatura. Se deducen fórmulas para calcular mediciones ilustrando de dónde vienen. El
teorema de Pitágoras y la fórmula de la distancia se desarrollan a lo largo de una nueva sección sobre
la ecuación del círculo.

Capítulo 14 Geometría del movimiento y embaldosados


Aunque se mantiene la mayoría de las características de la pasada edición, en la nueva edición de este
capítulo hay muchos más dibujos y más referencias a páginas de muestra que antes. Tratamos de
construir lo que los futuros maestros necesitan saber, que es más de lo que sus futuros alumnos
podrían necesitar. Este capítulo ofrece una visión de lo divertida e interesante que puede ser la
geometría del movimiento.

Uso de calculadoras
Como se afirma en los Principios y objetivos, es necesario y oportuno trabajar con calculadoras. Los
usos de calculadoras graficadoras se presentan cuando es relevante, en el Rincón de la tecnología.
Además, en los conjuntos de problemas aparece el uso de calculadoras científicas/fraccionales y
graficadoras.

CARACTERÍSTICAS
Seguimos incorporando ayudas y características que facilitan el aprendizaje.

© Lopez Mateos Editores. ISBN 978-607-95583-2-1, obra completa, versión electrónica, ISBN 978-607-95583-3-8, volumen 1, versión electrónica. Ejemplar asignado a: Helecto Villarroel gutierrez -
helecto@gmail.com. Fecha: 27 de octubre de 2014. Prohibida su modificación, copia o distribución.
Prefacio xi

Desarrollo profesional
¡Nuevo y • Se incluyen Páginas de muestra de libros de texto actualizadas para ilustrar cómo se presentan en la
mejorado! rea lidad las matemáticas a los alumnos de K a 8 y se hace referencia a ellas a lo largo del libro. Se
pide a los alumnos completar varias actividades de las páginas de muestra de manera que perciban
lo que van a ver en las escuelas básicas.
• Se presentan Notas de investigación en los márgenes, donde se exponen varios proyectos actuales de
investigación en matemáticas y en matemática educativa, relacionados con el contexto.
• Las Notas históricas agregan contexto y humanizan las matemáticas.
• Se incorporan a lo largo del libro citas importantes de los Principios y objetivos y de los Puntos focales
del NCTM.
• Preguntas del salón de clase presenta dudas que podrían tener los alumnos de K-8. Se añade un
número importante de estas dudas y preguntas. Ahora aparecen al final de cada sección como parte
de las Conexiones matemáticas.

Aprendizaje activo
• Los Rompecabezas proporcionan un camino diferente para resolver problemas. Se pueden usar como
reto para los alumnos.
• Las Actividades de laboratorio están integradas a lo largo del libro para proporcionar ejercicios de
aprendizaje por medio de actividades.
• Ahora intenta éste, son actividades que aparecen a lo largo de cada capítulo que están diseñadas para
que los alumnos se involucren de manera activa en su aprendizaje, facilitando así el desarrollo e in-
cremento de su razonamiento crítico y habilidad para resolver problemas, y estimulando las discu-
siones tanto dentro como fuera del salón de clases. Al final del libro aparecen las respuestas.
• En el Rincón de la tecnología se incluye el uso de hojas de cálculo, calculadoras graficadoras y científi-
cas, el programa The Geometer’s Sketchpad y actividades con computadoras.

Herramientas pedagógicas
• Las definiciones, propiedades y teoremas se resaltan en el texto para un rápido repaso.
• Las estrategias para resolver problemas se resaltan en cursivas, y en las cajas azules de Resolución de
problemas se usan estas estrategias.
• Las tiras cómicas enseñan o hacen énfasis en material importante y amenizan el contenido.
• En el Esbozo del capítulo al final de cada capítulo se ayuda a los alumnos a revisarlo.
• El Resumen del capítulo al final de cada uno permite a los alumnos autoevaluarse de manera efectiva
como preparación para un examen.
• La Bibliografía seleccionada al final de cada capítulo, se actualizó y revisó.

¡Nuevo y Evaluación
de problemas: Se revisaron minuciosamente y se reorganizaron en Evaluación A, B y
mejorado! • Conjuntos
Conexiones matemáticas. Los problemas en la Evaluación A tienen la respuesta al final del libro de
modo que los alumnos puedan verificar sus resultados. La Evaluación B contiene problemas simila-
res a los de la Evaluación A, pero no se dan las respuestas. Las Conexiones matemáticas se dividen
en las siguientes categorías de problemas: Comunicación, Respuesta abierta, Aprendizaje colectivo,
Preguntas del salón de clase y Problemas de repaso. Al final del libro se incluyen las respuestas a los
ejercicios impares.
• Los problemas reales y de importancia son más accesibles y atractivos para estudiantes de los más
diversos antecedentes.

© Lopez Mateos Editores. ISBN 978-607-95583-2-1, obra completa, versión electrónica, ISBN 978-607-95583-3-8, volumen 1, versión electrónica. Ejemplar asignado a: Helecto Villarroel gutierrez -
helecto@gmail.com. Fecha: 27 de octubre de 2014. Prohibida su modificación, copia o distribución.
xii Prefacio

Agradecimientos
Muchos ilustres y famosos educadores en matemáticas y matemáticos han revisado las anteriores ediciones
de este libro. Para honrar su trabajo, así como el de los revisores de la actual edición, hemos nombrado a
todos, pero señalamos con un asterisco a los revisores de esta edición. Queremos agradecer a Jerrold
Grossman su minuciosa revisión de este libro.

Leon J. Ablon Diane Ginsbach Linda Padilla


Paul Ache Elizabeth Gray Dennis Parker
G.L. Alexanderson * Jerrold Grossman Clyde Paul
Haldon Anderson Alice Guckin Keith Peck
Bernadette Antkoviak Jennifer Hegeman Barbara Pence
Richard Avery Joan Henn Glen L. Pfeifer
Sue H. Baker Boyd Henry Debra Pharo
Jane Barnard Linda Hintzman Jack Porter
Joann Becker Alan Hoffer Edward Rathnell
Cindy Bernlohr E. John Hornsby, Jr. Sandra Rucker
James Bierden * Patricia A. Jaberg Jennifer Rutherford
Jackie Blagg Judith E. Jacobs Helen R. Santiz
Jim Boone Donald James Sherry Scarborough
Sue Boren Thomas R. Jay Jane Schielack
Barbara Britton * Jeff Johannes Barbara Shabell
Beverly R. Broomell Jerry Johnson M. Geralda Shaefer
Anne Brown Wilburn C. Jones Nancy Shell
* Jane Buerger Robert Kalin Wade H. Sherard
Maurice Burke Sarah Kennedy Gwen Shufelt
David Bush Steven D. Kerr Julie Sliva
Laura Cameron Leland Knauf Ron Smit
Louis J. Chatterley Margret F. Kothmann Joe K. Smith
Phyllis Chinn Kathryn E. Lenz William Sparks
Donald J. Dessart Hester Lewellen Virginia Strawderman
Ronald Dettmers Ralph A. Liguori Mary M. Sullivan
Jackie Dewar * Richard Little Viji Sundar
* Nicole Duvernoy * Susan B. Lloyd Sharon Taylor
Amy Edwards Don Loftsgaarden Jo Temple
Lauri Edwards Sharon Louvier C. Ralph Verno
Margaret Ehringer Stanley Lukawecki Hubert Voltz
* Rita Eisele * Lou Ann Martin John Wagner
Albert Filano Judith Merlau Edward Wallace
Marjorie Fitting Barbara Moses Virginia Warfield
Michael Flom Cynthia Naples Lettie Watford
Martha Gady Charles Nelson Mark F. Weiner
Edward A. Gallo Glenn Nelson Grayson Wheatley
Dwight Galster Kathy Nickell Jim Williamson
Sandy Geiger * Bethany Noblitt Ken Yoder
Glenadine Gibb Dale Oliver Jerry L. Young
Don Gilmore Mark Oursland Deborah Zopf

© Lopez Mateos Editores. ISBN 978-607-95583-2-1, obra completa, versión electrónica, ISBN 978-607-95583-3-8, volumen 1, versión electrónica. Ejemplar asignado a: Helecto Villarroel gutierrez -
helecto@gmail.com. Fecha: 27 de octubre de 2014. Prohibida su modificación, copia o distribución.
Una introducción a la
resolución de problemas
CAPÍTULO

Problema preliminar
Hay tres platos de fruta en un estante tan alto que no los puedes ver. Un plato contiene sólo
manzanas, otro plato contiene sólo naranjas y otro plato contiene manzanas y naranjas. Cada
plato tiene visible uno de los siguientes rótulos: MANZANAS, NARANJAS, o MANZANAS Y
NARANJAS. Sin embargo, cada plato tiene el rótulo equivocado. Tu misión es seleccionar un
plato, alcanzarlo y tomar una fruta. Al hacer esto y con la información anterior, ¿puedes rotular
correctamente cada plato? Explica tu respuesta.

1
© Lopez Mateos Editores. ISBN 978-607-95583-2-1, obra completa, versión electrónica, ISBN 978-607-95583-3-8, volumen 1, versión electrónica. Ejemplar asignado a: Helecto Villarroel gutierrez -
helecto@gmail.com. Fecha: 27 de octubre de 2014. Prohibida su modificación, copia o distribución.
2 Una introducción a la resolución de problemas

R esolver problemas se ha reconocido, desde hace mucho tiempo, como una caracterís-
tica relevante de las matemáticas. ¿Qué significa resolver problemas? George Pólya
(1887–1985), uno de los más grandes matemáticos y maestros del siglo xx, señaló que “re-
solver un problema significa hallar una manera de superar una dificultad, o rodear un obs-
táculo, para lograr un objetivo que no podía obtenerse de inmediato” (Pólya 1981, p. ix).
En los Principles and Standards for School Mathematics PSSM (Principios y objetivos para
matemáticas escolares), publicado por el NCTM, National Council of Teachers of Mathe-
matics (Consejo Nacional de Maestros de Matemáticas) de Estados Unidos en el año 2000,
se afirma que:
Resolver un problema significa emprender una tarea para la cual no se conoce de antemano el método
de solución. Para encontrar una solución, los estudiantes deben producir conocimiento, y en ese pro-
ceso desarrollarán una mayor comprensión matemática. Resolver problemas no es sólo un objetivo de
aprender matemáticas, sino el mejor medio de hacerlo. Los estudiantes deberán tener oportunidades
frecuentes para formular, enfrentar y resolver problemas complejos que requieran una cantidad signi-
ficativa de esfuerzo, lo cual se plasmará en una mayor capacidad de razonar. (p. 52)
Más aún, hallamos que
Los programas desde preescolar hasta el grado 12 capacitarán a los estudiantes para:
• crear nuevo conocimiento matemático mediante la resolución de problemas;
• resolver problemas que surjan en matemáticas y en otros contextos;
• aplicar y adaptar diversas estrategias para resolver problemas;
• revisar y meditar acerca del proceso de resolución matemática de problemas. (p. 52)
Los estudiantes aprenden matemáticas como resultado de resolver problemas. Los ejerci-
cios, que son las prácticas rutinarias para adquirir habilidades tienen un propósito en el
aprendizaje de las matemáticas, pero la resolución de problemas debe ser el centro de aten-
ción de las matemáticas escolares. Como se señala en la Nota de investigación, una cantidad
razonable de tensión e incomodidad mejora el desempeño de los estudiantes para resolver
problemas. Tu experiencia matemática te ayudará a identificar cuándo una situación es un
problema o cuándo se trata de un ejercicio.

Nota de Una cantidad razonable de tensión e incomodidad mejora el desempeño de los estudiantes
investigación para resolver problemas. La motivación es deshacerse de la tensión una vez resuelto el pro-
blema. Si no está presente la tensión, el problema es un ejercicio o los estudiantes
“generalmente no tienen el deseo de atacar el problema con seriedad” (Bloom y Broder
1950; McLeod 1985). ◆

La experiencia matemática de los estudiantes de nivel elemental deberá alimentarse con


problemas interesantes, que valgan la pena, no sólo con problemas de rutina. Para involu-
crar a los estudiantes en tareas que valgan la pena, los problemas deben estar inmersos en
un contexto familiar o conocido, como se ve en la tira cómica.

© Lopez Mateos Editores. ISBN 978-607-95583-2-1, obra completa, versión electrónica, ISBN 978-607-95583-3-8, volumen 1, versión electrónica. Ejemplar asignado a: Helecto Villarroel gutierrez -
helecto@gmail.com. Fecha: 27 de octubre de 2014. Prohibida su modificación, copia o distribución.
Sección 1-1 Matemáticas y resolución de problemas 3

La buena experiencia de resolver problemas matemáticos ocurre cuando se da lo si-


guiente:
1. Se presenta a los estudiantes una situación que comprenden, pero ignoran cómo proce-
der directamente para obtener una solución.
2. Los estudiantes están interesados en obtener la solución y lo intentan.
3. Los estudiantes deben usar ideas matemáticas para resolver el problema.
En este libro de texto tendrás múltiples oportunidades para resolver problemas. Cada ca-
pítulo comienza con un problema que puede resolverse usando los conceptos desarrollados
en ese capítulo. Al final de cada capítulo se da una sugerencia para la solución del problema.
A lo largo del texto se encuentran numerosos problemas resueltos por el procedimiento de
los cuatro pasos y otros resueltos por medio de diferentes formatos.

◆ Nota de Los estudiantes que explican sus soluciones a otros estudiantes, principalmente si están en
investigación desacuerdo, obtendrán una mejor comprensión matemática. El análisis de los diferentes
puntos de vista es parte importante del aprendizaje. Así se aprende el lenguaje matemático
y se valora la necesidad de precisión en el lenguaje (Hatano e Ingaki 1991). ◆

Como lo indica la Nota de investigación, trabajar con otros estudiantes para resolver
problemas mejora tanto tu capacidad para solucionarlos como tus habilidades de comunica-
ción. Recomendamos el aprendizaje colectivo y sugerimos a los estudiantes que trabajen en
grupo lo más posible. Para impulsar el trabajo en grupo e identificar cuándo conviene usar
el aprendizaje colectivo, hemos ubicado actividades donde puede ser útil contar con varias
personas para recolectar datos, o el problema puede ser tal que la discusión en grupo con-
duzca a encontrar estrategias para resolver el problema.

1-1 Matemáticas y resolución de problemas

Si enfocas la resolución de problemas de una sola manera, corres el riesgo de emplear ideas
preconcebidas. Por ejemplo, deletrea la palabra ropa tres veces en voz alta: “¡R-O-P-A! ¡R-O-
P-A! ¡R-O-P-A!” Ahora responde la pregunta: “¿Qué haces cuando llegas a un semáforo en
verde?” Escribe tu respuesta. Si respondiste “Paro”, se te puede acusar de tener una idea pre-
concebida. Uno no para con la luz verde.
Considera el siguiente problema: “Un pastor tenía 36 ovejas. Todas murieron, excepto 10.
¿Cuántas quedaron vivas?” ¿Tu respuesta fue “10”? Si así fue, ya estás entendiendo y estás
preparado para intentar resolver algunos problemas. Si tu respuesta no fue “10”, entonces
no entendiste la pregunta. El primer paso en el proceso de cuatro pasos desarrollado por
George Polya es entender el problema. Usar el proceso de cuatro pasos para resolver proble-
mas no garantiza que hallemos la solución, sino que nos proporciona una manera sistemá-
tica de atacarlos.

◆ Nota George Pólya (1887–1985) nació en Hungría y recibió su doctorado en la Universidad de


histórica Budapest. Se mudó a Estados Unidos en 1940 y, después de una breve estancia en la Uni-
versidad de Brown, formó parte del personal docente de la Universidad de Stanford.
Además de ser un eminente matemático, se ocupó de la importancia fundamental de la edu-
cación matemática. En Standford publicó 10 libros, incluyendo How to Solve It (Cómo
plantear y resolver problemas) (1945), que se ha traducido a 23 idiomas. ◆

© Lopez Mateos Editores. ISBN 978-607-95583-2-1, obra completa, versión electrónica, ISBN 978-607-95583-3-8, volumen 1, versión electrónica. Ejemplar asignado a: Helecto Villarroel gutierrez -
helecto@gmail.com. Fecha: 27 de octubre de 2014. Prohibida su modificación, copia o distribución.
4 Una introducción a la resolución de problemas

Proceso de cuatro pasos para resolver problemas


1. Entender el problema
a. ¿Puedes enunciar el problema con tus propias palabras?
b. ¿Qué tratas de hallar o de hacer?
c. ¿Cuáles son las incógnitas?
d. ¿De qué información dispones?
e. ¿Qué información, si es el caso, falta o cuál no se necesita?
2. Trazar un plan
La siguiente lista de estrategias, aunque no es completa, resulta muy útil:
a. Buscar un patrón.
b. Examinar problemas relacionados y determinar si las técnicas aplicadas para
resolverlos se pueden aplicar en este caso.
c. Examinar un caso más sencillo, o un caso particular del problema, para comprender
mejor la solución del problema original.
d. Hacer una tabla o lista.
e. Hacer un diagrama.
f. Plantear una ecuación.
g. Proponer y verificar.
h. Trabajar regresivamente.
i. Identificar un objetivo parcial.
j. Usar razonamiento indirecto.
k. Usar razonamiento directo.
3. Realizar el plan
a. Llevar a cabo la estrategia o estrategias del paso 2 y efectuar las acciones y los cálcu-
los necesarios.
b. Verificar cada paso del plan conforme se avanza. La verificación puede ser intuitiva
o una demostración formal de cada paso.
c. Llevar un registro preciso del trabajo.
4. Revisar
a. Verificar los resultados en el problema original. (En algunos casos se requerirá una
demostración.)
b. Intepretar la solución en términos del problema original. ¿Tiene sentido tu
respuesta?, ¿es razonable?, ¿responde la pregunta hecha originalmente?
c. Averiguar si hay otro método para hallar la solución.
d. Si es posible, determinar otros problemas relacionados, o más generales, para los
cuales funcione la técnica usada.
¿Cuál es el papel que debería jugar el proceso de resolver problemas de Pólya en la ense-
ñanza de las matemáticas elementales? Esto se responde en los Principios y objetivos de la si-
guiente manera:

Una pregunta obvia es ¿Cómo deberían enseñarse estas estrategias? ¿Deberían recibir una atención
explícita, y cómo deberían integrarse al currículo matemático? Como cualquier otra componente
de las herramientas matemáticas, debe darse la debida importancia a la enseñanza de las estrategias
si se espera que los estudiantes las aprendan. En los grados inferiores los maestros pueden ayudar a
los niños a expresar, categorizar y comparar sus estrategias. La oportunidad de usar estrategias debe
incluirse de manera natural en el currículo, a lo largo del contenido de las diferentes áreas. Cuando
los estudiantes lleguen a los grados medios ya deberían ser hábiles para reconocer cuándo son apro-
piadas diversas estrategias y ser capaces de decidir cuándo y cómo usarlas. (p. 54)

© Lopez Mateos Editores. ISBN 978-607-95583-2-1, obra completa, versión electrónica, ISBN 978-607-95583-3-8, volumen 1, versión electrónica. Ejemplar asignado a: Helecto Villarroel gutierrez -
helecto@gmail.com. Fecha: 27 de octubre de 2014. Prohibida su modificación, copia o distribución.
Sección 1-1 Matemáticas y resolución de problemas 5

◆ Nota de La habilidad para resolver problemas se desarrolla lentamente, quizá debido a que la com-
investigación prensión y los recursos necesarios para resolver problemas se desarrollan a diferentes rit-
mos. Un elemento clave para desarrollar habilidades en la resolución de problemas es tener
experiencia múltiple y continua para resolver problemas en diferentes contextos y con dis-
tintos niveles de dificultad (Kantowski 1981). ◆

Estrategias para resolver problemas


A continuación presentamos una variedad de problemas en diferentes contextos para que
puedas obtener experiencia en resolver problemas, como se mencionó en la Nota de inves-
tigación. Con frecuencia es necesario emplear varias estrategias para resolver éstos y otros
problemas.
Las estrategias son herramientas que puedes usar para descubrir o construir los medios
que te permitan alcanzar un objetivo. Para cada estrategia descrita a continuación, damos
un problema que puede resolverse usándola. Es frecuente que los problemas se puedan re-
solver en más de una manera, como se ilustra en la caricatura. Puedes diseñar una estrategia
diferente para resolver los problemas de muestra. No existe una estrategia que sea la mejor.

SOLUCIÓN NO TRADICIONAL

◆ Nota Carl Gauss (1777–1855) está considerado como el más grande matemático del siglo die-
histórica cinueve y uno de los más prominentes de todos los tiempos. Nacido de padres pobres en
Brunswick, Alemania, fue un niño prodigio; se dice que a la edad de tres años corrigió un
error cometido en la contabilidad de su padre. Gauss realizó contribuciones en las áreas
de astronomía, geodesia y electricidad. Después de su muerte, el rey de Hanover ordenó
acuñar una medalla conmemorativa en su honor. En la medalla se inscribió la frase, refe-
rida a Gauss, de “Príncipe de las Matemáticas”, título que ha permanecido junto con su
nombre. ◆

© Lopez Mateos Editores. ISBN 978-607-95583-2-1, obra completa, versión electrónica, ISBN 978-607-95583-3-8, volumen 1, versión electrónica. Ejemplar asignado a: Helecto Villarroel gutierrez -
helecto@gmail.com. Fecha: 27 de octubre de 2014. Prohibida su modificación, copia o distribución.
6 Una introducción a la resolución de problemas

Estrategia: Buscar un patrón

Resolver problemas Problema de Gauss


Cuando Carl Gauss era niño, su maestro pidió a los alumnos que hallaran la suma de los
primeros 100 números naturales, esperando así mantener a la clase ocupada un buen rato.
Gauss dio la respuesta casi de inmediato. ¿Puedes hacerlo tú?

Comprender el problema Los números naturales son 1, 2, 3, 4, Á . Así, el problema es ha-


llar la suma 1 + 2 + 3 + 4 + Á + 100.

Trazar un plan Aquí es útil la estrategia buscar un patrón. Una versión de la historia acerca del
joven Gauss dice que listó los números según se muestra en la figura 1-1.
Sea S = 1 + 2 + 3 + 4 + 5 + Á + 98 + 99 + 100. Entonces,

S = 1 + 2 + 3 + 4 + 5 + Á + 98 + 99 + 100
S = 100 + 99 + 98 + 97 + 96 + Á + 3 + 2 + 1
2S = 101 + 101 + 101 + 101 + 101 + Á + 101 + 101 + 101

Figura 1-1
Para descubrir la suma original, Gauss dividió entre 2 la suma 2S de la figura 1-1.

# 100 # 101
Realizar el plan Hay 100 sumas de 101. Así, 2S = 100 101 y S = , ó 5050.
2
Revisar El método es matemáticamente correcto pues la suma se puede efectuar en cual-
quier orden, y la multiplicación es una suma repetida. Además, la suma en cada par siempre
es 101 pues al movernos de un par al siguiente, sumamos 1 al de arriba y restamos 1 al de
abajo, lo cual no cambia la suma; por ejemplo, 2 + 99 = (1 + 1) + (100 - 1) = 1 + 100,
3 + 98 = (2 + 1) + (99 - 1) = 2 + 99 = 101, y así sucesivamente.
Un problema más general es hallar la suma de los primeros n números naturales,
1 + 2 + 3 + 4 + 5 + 6 + Á + n. Usamos el mismo plan que antes y notamos la relación
en la figura 1-2. Hay n sumas de n + 1 que dan un total de n(n + 1). Por lo tanto,
n(n + 1)
2S = n(n + 1) y S = .
2

S = 1 + 2 + 3 + 4 + Á + n
S = n + (n - 1) + (n - 2) + (n - 3) + Á + 1
2S = (n + 1) + (n + 1) + (n + 1) + (n + 1) + Á + (n + 1)

Figura 1-2

Una estrategia diferente para hallar la suma 1 + 2 + 3 + Á + n consiste en hacer un dia-


grama y pensar la suma de manera geométrica como una pila de bloques. Para hallar la
suma, considera la pila en la figura 1-3(a) y la pila del mismo tamaño pero colocada de ma-
nera diferente, como en la figura 1-3(b). El número total de bloques en la pila de la figura
1-3(b) es n(n + 1), que es el doble de la suma deseada. Entonces la suma deseada es
n(n + 1)>2.

© Lopez Mateos Editores. ISBN 978-607-95583-2-1, obra completa, versión electrónica, ISBN 978-607-95583-3-8, volumen 1, versión electrónica. Ejemplar asignado a: Helecto Villarroel gutierrez -
helecto@gmail.com. Fecha: 27 de octubre de 2014. Prohibida su modificación, copia o distribución.
Sección 1-1 Matemáticas y resolución de problemas 7

n n

n n 1
(a) (b)

◆ Figura 1-3

n(n + 1)
O B S E R VA C I Ó N La suma 1 + 2 + 3 + 4 + 5 + Á + n = se analizará de
2
nuevo en la siguiente sección, cuando estudiemos sucesiones aritméticas.

AHORA INTENTA ÉSTE 1-1 Un corte en un tronco produce dos piezas, dos cortes producen tres
piezas y tres cortes producen cuatro piezas. ¿Cuántas piezas se producen con diez cortes? Supón que los
cortes se realizan de la misma manera que los tres primeros. ¿Cuántas piezas se producen con n cortes?

Estrategia: Examinar un problema relacionado

Resolver problemas Suma de números naturales pares


Halla la suma de los números naturales pares menores o iguales a 100. Diseña una estrate-
gia para hallar esa suma y generaliza el resultado.

Comprender el problema Los números naturales pares son 2, 4, 6, 8, 10, Á . El problema es


obtener la suma de los números naturales pares 2 + 4 + 6 + 8 + Á + 100.

Trazar un plan Reconocer que la suma se puede separar en dos partes más sencillas relacio-
nadas con el problema original de Gauss, nos ayuda a trazar un plan. Considera lo siguiente:
2 + 4 + 6 + 8 + Á + 100 = 2 # 1 + 2 # 2 + 2 # 3 + 2 # 4 + Á + 2 # 50
= 2(1 + 2 + 3 + 4 + Á + 50)
Así, podemos usar el método de Gauss para hallar la suma de los primeros 50 números na-
turales y después tomar el doble.

© Lopez Mateos Editores. ISBN 978-607-95583-2-1, obra completa, versión electrónica, ISBN 978-607-95583-3-8, volumen 1, versión electrónica. Ejemplar asignado a: Helecto Villarroel gutierrez -
helecto@gmail.com. Fecha: 27 de octubre de 2014. Prohibida su modificación, copia o distribución.
8 Una introducción a la resolución de problemas

Realizar el plan Realizamos el plan como sigue:


2 + 4 + 6 + 8 + Á + 100 = 2(1 + 2 + 3 + 4 + Á + 50)
= 2 # [50(50 + 1)>2]
= 2550
Así, la suma es 2550.

Revisar Otra manera de considerar el problema es comprender que hay 25 sumas de 102,
según se ve en la figura 1-4.

102
102
102
102
2 + 4 + 6 + 8 + . . . + 94 + 96 + 98 + 100

Figura 1-4

◆Así, la suma es 25 # 102, ó 2550.

AHORA INTENTA ÉSTE 1-2


a. Halla la suma de los números naturales impares menores que 100.
b. Sea a1, a2, a3, a4, Á , an cualquier sucesión de n términos, donde a2 - a1 = a3 - a2 = a4 - a3 =
. . . = an - an - 1 = d, donde d es un número fijo. Escribe una expresión para la suma de los términos
de esta sucesión, expresada en términos de a1, an y n.

Estrategia: Examinar un caso más sencillo


Una estrategia para resolver un problema complejo es examinar un caso más sencillo del pro-
blema y después considerar otras partes del problema complejo. En la siguiente página se
muestra un ejemplo.

AHORA INTENTA ÉSTE 1-3 Dieciséis personas participaron en un torneo de frontenis de todos con-
tra todos, es decir, cada persona juega contra cada uno de los otros participantes. ¿Cuántos partidos se ju-
garon?

Estrategia: Hacer una tabla


Una estrategia que se usa a menudo en la escuela primaria es hacer una tabla. Se puede usar
una tabla para buscar patrones que emerjan en el problema y que a su vez puedan con-
ducirnos a una solución. En la página 10 vemos un ejemplo de esta estrategia. ¿Realmente
el Plan II paga $128?

AHORA INTENTA ÉSTE 1-4 Mónica y Carla se iniciaron en un nuevo empleo el mismo día. Después de
comenzar, Mónica debe visitar la oficina central cada 15 días y Carla debe ir a la oficina central cada 18 días.
¿Cuántos días van a transcurrir antes de que vayan el mismo día a la oficina central?

© Lopez Mateos Editores. ISBN 978-607-95583-2-1, obra completa, versión electrónica, ISBN 978-607-95583-3-8, volumen 1, versión electrónica. Ejemplar asignado a: Helecto Villarroel gutierrez -
helecto@gmail.com. Fecha: 27 de octubre de 2014. Prohibida su modificación, copia o distribución.
Sección 1-1 Matemáticas y resolución de problemas 9

Página de un libro de texto Resolver un problema más sencillo

Estrategia para resolver problemas


Aprovechar lo que sabes
Idea clave
Aprender cómo y Resolver un problema usar la estrategia de:
cuándo resolver un
problema más más sencillo Resolver un problema más sencillo.

sencillo te puede
ayudar a resolver
problemas.
APRENDE
¿Cómo resuelves un problema más sencillo?
Trenes de triángulos Cada lado de cada
triángulo de la figura de la derecha mide
una pulgada. Si hay 12 triángulos en fila,
¿cuál es el perímetro de la figura?

Los triángulos están conectados. Cada


lado de cada triángulo mide una pulgada.
Hallar el perímetro de la figura con 12
triángulos

é Estrategia: Resolver un problema más sencillo


Puedo ver 1 triángulo, después 2
triángulos y después 3 triángulos.
Paso 1 Divide o cambia el
problema por uno que sea
más fácil de resolver.
perímetro = 3 pulgadas
Paso 2 Resuelve el problema más sencillo.
Paso 3 Usa las respuestas del perímetro = 4 pulgadas
problema más sencillo para
resolver el problema original.
perímetro = 5 pulgadas
Respuesta: El perímetro es 2 más que el
número de triángulos. Para 12 triángulos
el perímetro es de 14 pulgadas.

Revisa y verifica
Sí, ubiqué un patrón correcto.

Tema de plática
¿Cómo se dividió en problemas más sencillos?
Describe el patrón en los problemas más sencillos.

Fuente: Scott Foresman-Addison Wesley, Grade 4, 2008 (p. 648).

© Lopez Mateos Editores. ISBN 978-607-95583-2-1, obra completa, versión electrónica, ISBN 978-607-95583-3-8, volumen 1, versión electrónica. Ejemplar asignado a: Helecto Villarroel gutierrez -
helecto@gmail.com. Fecha: 27 de octubre de 2014. Prohibida su modificación, copia o distribución.
10 Una introducción a la resolución de problemas

Página de un libro de texto Hacer una tabla

Estrategia para resolver problemas

Idea clave
Aprender cómo y
cuándo hacer una usar la estrategia de:
tabla te puede APRENDE Hacer una tabla.
ayudar a resolver
problemas. ¿Cómo puedes hacer y usar una
tabla para resolver un problema?
Cuidado de bebés A Carolina le ofrecieron un empleo de cuidado
de bebés durante la tarde, por 10 días. Los padres que la quieren
contratar le ofrecieron dos planes de pago. ¿Cuál de ellos deberá
aceptar Carolina?
Plan I: Un pago único de $100 por los 10 días de trabajo.
Plan II: El pago por el primer día de trabajo será de $0.25. Después,
por cada día de trabajo se doblará el pago.

Hay dos planes diferentes.


Hallar el pago total, por los 10 días
del Plan II.

Hacer una tabla


Días
Cantidad

Días
Cantidad
Cómo hacer una tabla
Paso 1 Construye la tabla con Días
las etiquetas correctas. Cantidad
Paso 2 Registra en la tabla Días
los datos conocidos.
Paso 3 Busca un patrón, Cantidad
amplía la tabla. Respuesta: Carolina debe aceptar el Plan II que paga $128.
Paso 4 Halla la respuesta en
la tabla.
Revisa y verifica

Sí, la respuesta debe ser un


número par pues las cantidades
en la tabla se duplicaron.

Fuente: Scott Foresman-Addison Wesley, Grade 6, 2008 (p. 156).

© Lopez Mateos Editores. ISBN 978-607-95583-2-1, obra completa, versión electrónica, ISBN 978-607-95583-3-8, volumen 1, versión electrónica. Ejemplar asignado a: Helecto Villarroel gutierrez -
helecto@gmail.com. Fecha: 27 de octubre de 2014. Prohibida su modificación, copia o distribución.
Sección 1-1 Matemáticas y resolución de problemas 11

Estrategia: Identificar un objetivo parcial


Al intentar trazar un plan para resolver algunos problemas, es posible tener la sensación de
que el problema se podría resolver si pudiéramos hallar la solución de un problema algo
más fácil o familiar. Hallar la solución de ese problema más fácil puede convertirse en un
objetivo parcial del objetivo principal de resolver el problema original. El siguiente problema
de cuadrados mágicos muestra un ejemplo de esta situación.

Resolver problemas Cuadrados mágicos


Arregla los números del 1 al 9 en un cuadrado subdividido en nueve cuadrados menores
como el mostrado en la figura 1-5, de manera que cada renglón, cada columna y cada dia-
gonal principal sume lo mismo. (El resultado se llama cuadrado mágico.)

Comprender el problema Necesitamos colocar cada uno de los nueve números 1, 2, 3, Á , 9


en los cuadrados pequeños, un número diferente en cada cuadrado, de manera que la suma
Figura 1-5 de los números en cada renglón, columna y diagonal principal sea la misma.

Trazar un plan Si conociéramos el número fijo que deben sumar los renglones, las columnas
y las diagonales, tendríamos una mejor idea de qué números deben ir juntos en un renglón,
columna o diagonal. Así, nuestro objetivo parcial es hallar esa suma fija. La suma de los
nueve números, 1 + 2 + 3 + Á + 9, es igual a 3 veces la suma en un renglón (¿por qué?).
En consecuencia, la suma fija se obtiene al dividir 1 + 2 + 3 + Á + 9, entre 3. Usando el
procedimiento desarrollado por Gauss, tenemos (1 + 2 + 3 + Á + 9) , 3 =
9 # 10
a b , 3, ó 45 , 3 = 15, de modo que la suma en cada renglón, columna y diagonal
2
debe ser 15. A continuación, necesitamos decidir qué números podrían ocupar qué lugares.
El número en el centro debe aparecer en cuatro sumas de 15 (en dos diagonales, en el se-
gundo renglón y en la segunda columna). Cada número en las esquinas debe aparecer en tres
sumas de 15. (¿Puedes ver por qué?) Si escribimos el 15 como suma de tres números dife-
rentes del 1 al 9 de todas las maneras posibles, podríamos contar, para cada número del 1 al
9, cuántas sumas lo contienen. Los números que aparezcan en al menos cuatro sumas son
candidatos para ocupar el cuadrado del centro, mientras que los números que aparezcan en
al menos tres sumas son candidatos para los cuadrados de las esquinas. Nuestro nuevo
objetivo parcial es escribir el número 15 de todas las maneras posibles, como suma de tres nú-
meros diferentes tomados del conjunto {1, 2, 3, Á , 9}.

Realizar el plan Las sumas de 15 se pueden escribir, de manera sistemática, como sigue:

9 + 5 + 1
9 + 4 + 2
8 + 6 + 1
8 + 5 + 2
8 + 4 + 3
7 + 6 + 2
7 + 5 + 3
6 + 5 + 4

© Lopez Mateos Editores. ISBN 978-607-95583-2-1, obra completa, versión electrónica, ISBN 978-607-95583-3-8, volumen 1, versión electrónica. Ejemplar asignado a: Helecto Villarroel gutierrez -
helecto@gmail.com. Fecha: 27 de octubre de 2014. Prohibida su modificación, copia o distribución.
12 Una introducción a la resolución de problemas

Nota que 1 + 5 + 9 y 5 + 1 + 9, por ejemplo, se cuentan una sola vez. Nota que el 1 apa-
rece sólo en dos sumas, el 2 aparece en tres sumas, el 3 aparece en dos sumas, y así sucesiva-
mente. En la tabla 1-1 se resume el patrón.

Tabla 1-1
Número 1 2 3 4 5 6 7 8 9

Número de sumas que contienen al número 2 3 2 3 4 3 2 3 2

El único número que aparece en cuatro sumas es el 5; por lo tanto, el 5 debe estar en el
centro del cuadrado. (¿Puedes ver por qué?) Como 2, 4, 6 y 8 aparecen tres veces cada uno,
deben ir en las esquinas. Supongamos que escogemos el 2 para la esquina superior izquierda.
Entonces debemos colocar el 8 en la esquina inferior derecha. (¿Por qué?) Observa la figura
1-6(a). Ahora podemos colocar el 6 en la esquina inferior izquierda o en la esquina superior
derecha. Si escogemos la esquina superior derecha, obtenemos el resultado mostrado en la
figura 1-6(b). El cuadrado mágico se puede completar como se muestra en la figura 1-6(c).

2 2 6 2 7 6

5 5 9 5 1

8 8 4 3 8

(a) (b) (c)

Figura 1-6

Revisar Hemos visto que el 5 fue el único número, de los dados, que podía ocupar el cen-
tro. Sin embargo, tuvimos varios candidatos para las esquinas y, por lo tanto, parece que el
cuadrado mágico que hallamos no es el único posible. ¿Puedes encontrar los demás?
Otra manera de ver que el 5 debe estar en el centro es considerar las sumas
1 + 9, 2 + 8, 3 + 7, 4 + 6, como se muestra en la figura 1-7. Podemos sumar 5 a cada una
para obtener 15.

1 2 3 4 5 6 7 8 9

10
10
10
10
◆ Figura 1-7

AHORA INTENTA ÉSTE 1-5 Cinco amigos decidieron hacer una fiesta y compartir los gastos en partes
iguales. Alberto gastó $47.50 en invitaciones, Beti gastó $120 en bebidas y $52.50 en verduras, Carlos gastó
$240 en comida, Daniel gastó $60 en platos y servilletas, y Elena gastó $130 en decorados. Averigua quién
le debe dinero a quién y cómo se puede pagar.

© Lopez Mateos Editores. ISBN 978-607-95583-2-1, obra completa, versión electrónica, ISBN 978-607-95583-3-8, volumen 1, versión electrónica. Ejemplar asignado a: Helecto Villarroel gutierrez -
helecto@gmail.com. Fecha: 27 de octubre de 2014. Prohibida su modificación, copia o distribución.
Sección 1-1 Matemáticas y resolución de problemas 13

Estrategia: Hacer un diagrama


Se ha dicho a menudo que una imagen vale lo que mil palabras. Esto es particularmente
cierto en la resolución de problemas. En el problema siguiente, hacer un diagrama nos ayuda
a entender el problema y a trabajar para encontrar la solución.

Resolver problemas Problema de la carrera de 50 m


Beto y Juan compitieron 3 veces en una carrera de 50 m. La velocidad de los corredores no
varió. En la primera carrera, Juan iba en el metro 45 cuando Beto estaba cruzando la meta.
a. En la segunda carrera, para que fuera más pareja Juan comenzó 5 m adelante de Beto,
quien se colocó en la línea de salida. ¿Quién ganará ésta?
b. En la tercera carrera, Juan comienza en la línea de salida y Beto comienza 5 m atrás.
¿Quién ganará la carrera?

Comprender el problema Cuando Beto y Juan corren 50 m, Beto gana por 5 metros; cada vez
que Beto cubre 50 m, en ese mismo tiempo Juan cubre sólo 45 m. Si Beto comienza en la lí-
nea de salida y da a Juan una ventaja de 5 metros, debemos determinar quién gana la ca-
rrera. Si Juan comienza en la línea de salida y Beto 5 metros atrás, determinaremos quién
va a ganar.

Trazar un plan Una estrategia para determinar al ganador en cada una de las condiciones es
dibujar un diagrama. En la figura 1-8(a) damos un diagrama para la primera carrera de 50 m.
En este caso Beto gana por 5m. En la segunda carrera Juan tiene 5m de ventaja y cuando Beto
corre los 50 m que lo separan de la meta, Juan corre sólo 45 m. Como Juan está a 45 m de la
meta, llega al mismo tiempo que Beto. Esto se muestra en la figura 1-8(b). En la tercera ca-
rrera, como Beto comienza 5 m atrás, usamos la figura 1-8(a) y movemos a Beto 5 m como se
muestra en la figura 1-8(c). Del diagrama podemos determinar los resultados en cada caso.
Comienzo Final
0 50 m

45 m Beto
(a)
Juan
5m

0 50 m
Beto
(b) 45 m
Juan
5m

0 50 m
Beto
(c)
45 m 5m
Juan
5m 5m
Figura 1-8

© Lopez Mateos Editores. ISBN 978-607-95583-2-1, obra completa, versión electrónica, ISBN 978-607-95583-3-8, volumen 1, versión electrónica. Ejemplar asignado a: Helecto Villarroel gutierrez -
helecto@gmail.com. Fecha: 27 de octubre de 2014. Prohibida su modificación, copia o distribución.
14 Una introducción a la resolución de problemas

Realizar el plan De la figura 1-8(b) vemos que si Juan recibe 5 m de ventaja, entonces la ca-
rrera se empata. Si Beto comienza 5 m detrás de Juan, entonces estarán empatados a los
45 m. Como Beto es más veloz que Juan, Beto recorrerá los últimos 5 m más rápido que
Juan y ganará la carrera.

Revisar Los diagramas muestran que la solución tiene sentido y es apropiada. Se pueden in-
vestigar otros problemas relacionados con carreras y ventajas. Por ejemplo, si Beto y Juan co-
rren en una pista ovalada de 50 m, ¿cuántas vueltas requerirá Beto para aventajar a Juan una
◆ vuelta completa? (Supón que las velocidades son las anteriores.)

O B S E R VA C I Ó N En muchas ocasiones las soluciones de los estudiantes pueden


incluir procesos que ocurren simultáneamente: pensar en el problema y apoyar ese
razonamiento haciendo un diagrama.

AHORA INTENTA ÉSTE 1-6  Un elevador se detiene en el piso de en medio de un edificio. Después se
mueve 4 pisos hacia arriba y se detiene. Luego se mueve hacia abajo 6 pisos y se detiene. A continuación se
mueve 10 pisos hacia arriba y se detiene. El elevador está ahora a 3 pisos del piso más alto. ¿Cuántos pisos
tiene el edificio?

Estrategia: Proponer y verificar


En la estrategia de proponer y verificar, primero proponemos una solución “al tanteo” usando
un tanteo lo más razonable posible. A continuación, verificamos si la propuesta fue correcta.
De no ser así, el paso siguiente es aprender lo más posible acerca de la solución basados en la
propuesta anterior, antes de hacer una nueva propuesta. Esta estrategia se puede considerar
una forma de ensayo y error, donde la información acerca del error nos ayuda a escoger el
siguiente ensayo. La estrategia de proponer y verificar es utilizada con frecuencia por los
alumnos que no saben resolver el problema de manera más eficiente o que no tienen aún las
herramientas para resolver el problema con más rapidez. Vean en la página del libro de texto
de la página 15 cómo se benefician los estudiantes al observar los “errores”, como se men-
ciona en la Nota de investigación.

◆ Nota de Los estudiantes de los grados 1 a 3 usan principalmente la estrategia de proponer y verificar
investigación cuando encuentran un problema matemático, y conforme llegan a los grados de 6 a 12 esta
tendencia decrece. Los estudiantes mayores se benefician más de los “errores” observados
después de una primera propuesta al formular un nuevo “intento” (Lester 1975). ◆

AHORA INTENTA ÉSTE 1-7 Un criptarritmo es una colección de palabras donde cada letra representa
un número único. Halla los dígitos que pueden substituirse en lo siguiente:
TIN
+ PIN
TOMA

© Lopez Mateos Editores. ISBN 978-607-95583-2-1, obra completa, versión electrónica, ISBN 978-607-95583-3-8, volumen 1, versión electrónica. Ejemplar asignado a: Helecto Villarroel gutierrez -
helecto@gmail.com. Fecha: 27 de octubre de 2014. Prohibida su modificación, copia o distribución.
Sección 1-1 Matemáticas y resolución de problemas 15

Página de un libro de texto Proponer y verificar

Estrategia de solución de problemas

Idea clave
La estrategia “Intenta, usar la estrategia de:
verifica y revisa” te Intentar, verificar y revisar.
puede ayudar a
resolver problemas.

¿Qué estás
buscando?

Intentar, verificar y revisar

Dos camas es demasiado. Intentaré con una.


Después trataré de añadir 2 artículos
pequeños. Lo intentaré primero con las correas.

Revisa y verifica

Fuente: Scott Foresman-Addison Wesley, Grade 4, 2005 ( p. 278).

© Lopez Mateos Editores. ISBN 978-607-95583-2-1, obra completa, versión electrónica, ISBN 978-607-95583-3-8, volumen 1, versión electrónica. Ejemplar asignado a: Helecto Villarroel gutierrez -
helecto@gmail.com. Fecha: 27 de octubre de 2014. Prohibida su modificación, copia o distribución.
16 Una introducción a la resolución de problemas

Estrategia: Trabajar regresivamente


En algunos problemas es mejor comenzar por el resultado y trabajar hacia atrás (regre-
sivamente), situación que ilustramos en la Página de un libro de texto siguiente. Nota que
también se usa la estrategia de hacer un diagrama.

AHORA INTENTA ÉSTE 1-8 Luisa tiene un promedio (media) de 80 en sus 11 exámenes de matemáticas.
Su maestra le dice que va a eliminar la calificación más baja, 50. ¿Cuál es su nuevo promedio?

Estrategia: Usar razonamiento indirecto


Para mostrar que una afirmación o proposición es verdadera, con frecuencia es más fácil
mostrar que es imposible que la afirmación sea falsa. Esto puede lograrse mostrando que si
la afirmación fuera falsa, implicaría algo contradictorio o imposible. Este enfoque es útil
cuando se dificulta comenzar con un argumento directo y cuando negar la afirmación dada
nos proporciona algo tangible para trabajar. Veamos un ejemplo.

Resolver problemas Problema del tablero de ajedrez


En la figura 1-9 vemos un tablero de ajedrez donde eliminamos dos esquinas opuestas. Te-
nemos un conjunto de fichas de dominó de tal forma que cada una cubre 2 cuadros adya-
centes del tablero. ¿Se pueden arreglar las fichas de dominó de manera que los cuadros
restantes en el tablero queden cubiertos sin que haya fichas encimadas o colgando fuera?
De no ser posible, ¿por qué?

Figura 1-9

Comprender el problema Se eliminaron dos espacios rojos en esquinas opuestas del tablero
de ajedrez, según se muestra en la figura 1-9. Se nos pregunta si es posible cubrir los 62
cuadros restantes con fichas de dominó del tamaño de 2 cuadros.

Trazar un plan Si tratamos de cubrir el tablero de la figura 1-9 con fichas, veremos que éstas
no encajan y que algunos cuadros quedan sin cubrir. Para mostrar que no hay manera de
cubrir el tablero con fichas, usamos el razonamiento indirecto. Si los 62 cuadros de la figura
1-9 se pudieran cubrir con fichas de dominó sin que se encimen o salgan del tablero, se
requerirían 31 fichas. Queremos mostrar que esto implica algo imposible.

© Lopez Mateos Editores. ISBN 978-607-95583-2-1, obra completa, versión electrónica, ISBN 978-607-95583-3-8, volumen 1, versión electrónica. Ejemplar asignado a: Helecto Villarroel gutierrez -
helecto@gmail.com. Fecha: 27 de octubre de 2014. Prohibida su modificación, copia o distribución.
Sección 1-1 Matemáticas y resolución de problemas 17

Página de un libro de texto Trabajar hacia atrás

Estrategia para resolver problemas


Identificar pasos en el proceso
Idea clave
Aprender cómo y Trabajar hacia atrás usar la estrategia de:
cuándo trabajar Trabajar hacia atrás.
hacia atrás te APRENDE
puede ayudar a
resolver problemas.
¿Cómo puedes trabajar hacia
atrás para resolver un problema?
Durante la cuarta semana los
Construcción de túnel A los obreros
obreros excavaron 21s millas. La
les tomó 5 semanas excavar un túnel
semana siguiente excavaron 11f
de 10 millas de largo.
para terminar el túnel.
¿Cuánto habían avanzado los obreros
después de 3 semanas de excavar?

Los obreros terminaron un túnel de 10 millas en 5


semanas. Durante la semana 4 excavaron 21s millas.
Durante la semana 5 excavaron 11f millas.
¿Qué estás buscando? ¿Cuántas millas del túnel excavaron los obreros en las
primeras 3 semanas?

é Estrategia: Trabajar hacia atrás


No conocemos el número de millas que excavaron
durante las 3 primeras semanas.
Cómo trabajar hacia atrás
distancia excavada en las primeras 3 semanas = n millas
Paso 1
millas túnel de diez millas 10 millas
Paso 2 semana semana
semanas

Paso 3 millas millas millas

Respuesta: Los obreros cavaron 6 1f millas del túnel durante las


primeras 3 semanas.

Revisa y verifica
¿Tu respuesta es Sí, pues al trabajar hacia adelante, partiendo de la
razonable? cantidad inicial, obtengo el resultado final.
61f millas + 21s millas + 11f millas = 10 millas

Fuente: Scott Foresman-Addison Wesley, Grade 5, 2008 ( p. 484).

© Lopez Mateos Editores. ISBN 978-607-95583-2-1, obra completa, versión electrónica, ISBN 978-607-95583-3-8, volumen 1, versión electrónica. Ejemplar asignado a: Helecto Villarroel gutierrez -
helecto@gmail.com. Fecha: 27 de octubre de 2014. Prohibida su modificación, copia o distribución.
18 Una introducción a la resolución de problemas

Realizar el plan Cada ficha de dominó debe cubrir 1 cuadro negro y 1 cuadro rojo. Por lo
tanto, 31 fichas deberían cubrir 31 cuadros rojos y 31 cuadros negros. Esto es imposible
pues el tablero de la figura 1-9 tiene 30 cuadros rojos y 32 cuadros negros. En consecuen-
cia, nuestra hipótesis de que el tablero de la figura 1-9 se podía cubrir con fichas de dominó
está equivocada.
Revisar Del conteo de los cuadros negros y rojos vemos que si eliminamos cualquier nú-
mero de cuadros de un tablero de ajedrez de manera que el número de los cuadros rojos res-
tantes difiera del número de los cuadros negros restantes, el tablero no se podrá cubrir con
fichas de dominó. (¿Puedes ver por qué?) También podríamos investigar lo que sucede
cuando se eliminan dos cuadrados del mismo color de un tablero de 8 por 7 o de tableros
de otras medidas. Podríamos investigar, además, si siempre es posible cubrir el tablero res-
◆ tante cuando se eliminan dos cuadros de color opuesto.

AHORA INTENTA ÉSTE 1-9 Ale, Beto, Cali y Dani participan en exactamente un deporte ya sea natación,
beisbol, baloncesto o tenis. Beto juega beisbol. Ale no puede nadar. Cali juega baloncesto. ¿En qué deportes
participa cada persona?

Estrategia: Usar razonamiento directo


Resolver problemas Juego de damas
Dos personas jugaron damas entre sí y cada una ganó tres partidas. ¿Es posible que sólo hayan
jugado cinco partidas?

Solución Sabemos que cada persona ganó tres partidas. Razonando de manera directa, vemos
que si cada una ganó tres partidas y jugaron entre ellas, entonces se tuvieron que jugar seis
partidas. De otra forma no podrían haber jugado entre sí y tener tres victorias cada una.
¿Podría tener cada una tres victorias luego de jugar un total de cinco partidas, habiéndose
◆ enfrentado entre sí? La respuesta es no, y la situación es imposible.
Estrategia: Plantear una ecuación
Una estrategia para resolver problemas usada en el razonamiento algebraico es plantear una
ecuación. Esta estrategia es muy importante y la veremos en el capítulo 4, “Razonamiento
algebraico”.

Evaluación 1-1A

1. Usa el enfoque del problema de Gauss para hallar las su- tros. Hay un descanso a la mitad del camino entre El Ca-
mas siguientes (no uses fórmulas): marón y Tehuantepec. ¿A qué distancia de Oaxaca está el
a. 1 + 2 + 3 + 4 + Á + 99 punto de descanso?
b. 1 + 3 + 5 + 7 + Á + 1001
5. Yolanda, Chocolata, Trueno y Marisolita están en una
2. Halla la suma 36 + 37 + 38 + 39 + Á + 146 + 147.
carrera de caballos. Chocolata es la más lenta, Trueno es
más veloz que Yolanda pero más lento que Marisolita.
3. Las galletas se venden solas o en paquetes de dos o de
Da el orden de llegada de los caballos.
seis. ¿De cuántas maneras puedes comprar una docena
de galletas? 6. Pancho y Juanito comienzan a leer una novela el mismo
4. Acabas de salir de Oaxaca hacia el Istmo. El Camarón día. Pancho lee 8 páginas diarias y Juanito 5 páginas diarias.
está a 120 kilómetros y Tehuantepec está a 200 kilóme- Si Pancho va en la página 72, ¿en qué página va Juanito?

© Lopez Mateos Editores. ISBN 978-607-95583-2-1, obra completa, versión electrónica, ISBN 978-607-95583-3-8, volumen 1, versión electrónica. Ejemplar asignado a: Helecto Villarroel gutierrez -
helecto@gmail.com. Fecha: 27 de octubre de 2014. Prohibida su modificación, copia o distribución.
Sección 1-1 Matemáticas y resolución de problemas 19

7. ¿Cuál es la mayor suma de dinero —en monedas comu- 夝 13. Se pegan cubos del mismo tamaño para construir una suce-
nes y corrientes— que puedes llevar en el bolsillo sin que sión de sólidos con forma de escalera, como se muestra:
puedas dar cambio de un billete de cien pesos, ni uno de
cincuenta, ni 25, ni una moneda de diez pesos ni una de
cinco?
8. a. Coloca los dígitos 1, 2, 4, 5 y 7 en los cuadros siguien- , ,
tes de manera que en (i) se obtenga el mayor pro-
Todas las caras de los cubos que no están pegadas requie-
ducto y en (ii) se obtenga el mayor cociente:
ren pintarse. ¿Cuántos cuadrados necesitarán pintarse
(a) en el 100-ésimo sólido y (b) en el n-ésimo sólido?
(i) (ii)
14. Un granjero necesita cercar un terreno rectangular y
× quiere que la longitud del campo sea 80 metros mayor que
el ancho. Si tiene 1080 metros de material para la cerca,
¿cuáles deberán ser la longitud y el ancho del campo?
b. Usa los mismos dígitos que en (a) para obtener (i) el 15. En una noche de invierno la temperatura descendió 2°C
menor producto y (ii) el menor cociente. entre la medianoche y las 7 a.m. A las 11 a.m. la tempera-
9. Supón que puedes gastar $10 cada minuto, día y noche. tura era el doble que la de las 7 a.m.. Para medio día se
¿Cuánto podrías gastar en un año (de 365 días)? elevó 2°C para llegar a 16°C. ¿Cuál era la temperatura a la
10. ¿Cuántos números de cuatro dígitos tienen los mismos medianoche?
dígitos que 1993? 16. Alicia, Beti, Carlos y Daniel nacieron en diferente esta-
11. Un compás y una regla, juntos, cuestan $40. El compás ción. Alicia nació en febrero. Beti no nació en otoño.
cuesta $9 más que la regla. ¿Cuánto cuesta el compás? Carlos nació en primavera. Determina en qué estación
12. Cata está parada a la mitad de una escalera. Sube tres es- nació cada persona.
calones, baja cinco y luego sube siete escalones. Por úl- 17. En los cuadros a continuación se escriben los 14 dígitos
timo, sube los restantes seis escalones para llegar al final de una tarjeta de crédito. Si la suma de tres dígitos con-
de la escalera. ¿Cuántos peldaños tiene la escalera? secutivos cualesquiera es 20, ¿cuál es el valor de A?

A 7 7 4

Evaluación 1-1B

1. Usa el enfoque del problema de Gauss para hallar las b. Si P = 1 + 3 + 5 + 7 + Á + 99 , ¿cuál es el más
sumas siguientes (no uses fórmulas): grande, E o P, y por cuánto?
a. 1 + 2 + 3 + 4 + Á + 49 6. a. Coloca los dígitos 4, 5, 6, 7 y 9 en los cuadros siguien-
b. 1 + 3 + 5 + 7 + Á + 2009 tes de manera que en (i) se obtenga el mayor pro-
2. Halla la suma de 58 + 59 + 60 + 61 + Á + 203. ducto y en (ii) se obtenga el mayor cociente.
3. ¿De cuántas maneras se puede fraccionar un billete de
$50 usando billetes de $5, $10 y $20?
(i) (ii)
4. ¿Cuántos cuadrados diferentes hay en la siguiente fi-
gura? ×

b. Usa los mismos dígitos que en (a) para obtener (i) el


menor producto y (ii) el menor cociente.
7. Marta va a la tienda y lleva $1 en cambio. Tiene al me-
nos una de cada moneda menor de 50¢, pero no tiene
moneda de 50¢.
5. a. Sin calcular cada suma, encuentra cuál es el más gran- a. ¿Cuál es el mínimo número de monedas que puede
de, O o E, y por cuánto. tener?
b. ¿Cuál es el máximo número de monedas que puede
O = 1 + 3 + 5 + 7 + Á + 97 tener?
E = 2 + 4 + 6 + 8 + Á + 98

© Lopez Mateos Editores. ISBN 978-607-95583-2-1, obra completa, versión electrónica, ISBN 978-607-95583-3-8, volumen 1, versión electrónica. Ejemplar asignado a: Helecto Villarroel gutierrez -
helecto@gmail.com. Fecha: 27 de octubre de 2014. Prohibida su modificación, copia o distribución.
20 Una introducción a la resolución de problemas

8. Halla un cuadrado mágico de 3 por 3 usando los núme- 12. Supón que arrojas tres dardos al blanco ilustrado a con-
ros 3, 5, 7, 9, 11, 13, 15, 17 y 19. tinuación. Todos los dardos dan en el tablero. ¿Cuáles
9. Tenemos ocho canicas de igual apariencia, pero una es son las puntuaciones posibles?
un poco más pesada que las otras. Usando una balanza,
explica cómo se puede descubrir la canica más pesada en P u n t os
10
exactamente Punt o
50

s
a. tres pesadas.
b. dos pesadas. 100
Puntos
10. a. Halla la suma de todos los números en el arreglo si-
guiente:

1 2 3 4 5 6 ... 100
2 4 6 8 10 12 ... 200
13. El siguiente es un cuadrado mágico (todos los renglones,
3 6 9 12 15 18 ... 300
columnas y diagonales suman lo mismo). Halla el valor
o o o o o o o o de cada variable.
100 200 300 400 500 600 Á 100 # 100
17 a 7
b. Generaliza la parte (a) para un arreglo similar en donde
cada renglón tenga n números y haya n renglones. 12 22 b
11. a. Usando las rectas existentes en el tablero de ajedrez
que se muestra, ¿cuántos cuadrados diferentes hay? c d 27

14. Hay dos cartas sobre una mesa. En una está escrito el
número 12 y en la otra el 9. Cada carta tiene un número
escrito en el reverso. Al voltear una carta, las dos cartas o
ninguna carta, y sumando los dos números, se obtienen
las sumas de 15, 16, 20 y 21. ¿Qué número está escrito
en el reverso de cada carta?
15. Supón que vas a comprar merienda para el club de mate-
máticas. Tienes dinero suficiente para comprar 20 ensala-
das o 15 emparedados. El grupo quiere 12 emparedados.
¿Cuántas ensaladas puedes comprar?
b. Si se duplica el número de renglones y de columnas 16. a. Supón que tienes monedas de 25¢, 10¢ y 1¢ que suman
del tablero, ¿se duplica también el número de cuadra- un total de $1.19. ¿Cuántas monedas de cada una puedes
dos diferentes? Justifica la respuesta. tener de manera que no puedas cambiar $1.00?
b. Di por qué la combinación de monedas que tienes en
la parte (a) es la mayor cantidad de dinero que puedes
tener sin cambiar $1.00.
17. Tienes dos recipientes. En uno caben 7 tazas y en el otro
4 tazas. ¿Cómo puedes medir exactamente 5 tazas de
agua si dispones de una cantidad ilimitada de agua para
empezar?

Conexiones matemáticas 1-1

Comunicación jetivos de la NCTM que aparecen en la tapa posterior del


1. ¿Por qué la enseñanza de la resolución de problemas es libro.
parte importante de las matemáticas? 3. Explica cómo puedes usar la estrategia de proponer y ve-
2. Analiza cómo se relaciona el proceso de cuatro pasos de rificar.
resolución de problemas de Polya con los dos últimos ob-

© Lopez Mateos Editores. ISBN 978-607-95583-2-1, obra completa, versión electrónica, ISBN 978-607-95583-3-8, volumen 1, versión electrónica. Ejemplar asignado a: Helecto Villarroel gutierrez -
helecto@gmail.com. Fecha: 27 de octubre de 2014. Prohibida su modificación, copia o distribución.
Sección 1-1 Mátematicas y resolución de problemas 21

Solución abierta para hacer un libro de 8 páginas y éstas se numeran, pre-


4. Usa exactamente cuatro dígitos 4 y cualquier operación dice la suma de los números en cada lado de cada hoja y
matemática para obtener los números del 1 al 20 inclu- la suma de todos los números de las páginas. Haz tu li-
sive; por ejemplo, 4>4 + 4>4 = 2 y 4 * 4 + 4 - bro para ver si estabas en lo correcto. Ensaya lo mismo
14 = 18. con 3 hojas.
5. Elige una estrategia para resolver problemas y elabora un a. Supón que vas a hacer un libro de 100 páginas; ¿cuán-
problema en que pueda usarse esta estrategia. Escribe la tas hojas vas a necesitar?
solución usando el enfoque de los cuatro pasos de Pólya. b. ¿Cuál es la suma de dos números de página colocados
en el mismo lado de la hoja?
Aprendizaje colectivo c. ¿Cuál es la suma de todos los números de página del
6. Que cada persona de tu grupo trabaje con el siguiente libro?
problema: si 8 personas se dan la mano entre sí, ¿cuántos d. Supón que tienes n hojas de papel. Generaliza para
apretones hubo? hallar el número de páginas del libro, la suma de los
a. Comparen sus estrategias para resolver el problema. números colocados en el mismo lado de la hoja, y la
¿En qué se parecen? ¿En qué difieren? suma de todos los números de página del libro.
b. Hallen la mayor cantidad posible de maneras para re-
Preguntas del salón de clase
solver el problema.
c. Generalicen la solución para n personas. 10. Ana te pregunta “qué es la resolución de problemas” y si
7. La distancia alrededor del mundo es cercana a los 3 * 8 es un problema. ¿Qué le dices?
40,000 km. ¿Aproximadamente cuántas personas de ta- 11. Juanito pregunta por qué el último paso del proceso de
maño promedio de tu grupo se requerirían para rodear cuatro pasos de Pólya para resolver problemas, revisar, es
el mundo tomadas de la mano? necesario si ya se dio la respuesta. ¿Qué le puedes decir?
8. Trabajen en parejas en la siguiente versión de un juego lla- 12. Una estudiante pregunta por qué no puede simple-
mado NIM. Se necesita una calculadora para cada pareja. mente realizar una “propuesta al azar” en lugar de una
a. El jugador 1 presiona 1 y + ó 2 y + . El juga- “propuesta inteligente” cuando se usa la estrategia de
dor 2 hace lo mismo. “proponer y verificar” para resolver problemas. ¿Qué le
Juegan de manera alternada hasta que se llega a 21. El respondes?
primer jugador que llega a 21 gana. Determinen una 13. Beto dice que sí es posible crear un cuadrado mágico con
estrategia que decida quién gana siempre. los números 1, 3, 4, 5, 6, 7, 8, 9 y 10. ¿Cómo le respondes?
b. Jueguen NIM usando los dígitos 1, 2, 3 y 4, con la Pregunta del Third International Mathematics and
meta de 104. El primer jugador que llegue a 104 gana. Science Study (TIMSS) (Tercer Estudio Internacional
¿Cuál es la estrategia ganadora? sobre las Matemáticas y la Ciencia)
c. Jueguen NIM usando los dígitos 3, 5 y 7, con la meta
de 73. El primer jugador que rebase 73 pierde. ¿Cuál
es la estrategia ganadora? 4 11 6
d. Ahora jueguen NIM inverso con las teclas 1 y 2 .
En lugar de + usen - . Coloquen el número 21 en la
9 5
pantalla. La meta es 0. Determinen una estrategia para
8 3 10
ganar en NIM inverso.
e. Jueguen NIM inverso usando los dígitos 1, 2 y 3 y co-
miencen con 24 en la pantalla. La meta es 0. ¿Cuál es La regla para construir la tabla es que los números de
la estrategia ganadora? cada renglón y columna deben sumar lo mismo. ¿Qué
f. Jueguen NIM inverso usando los dígitos 3, 5 y 7 co- número va en el centro de la tabla?
menzando con el 73 en la pantalla. El primer jugador a. 1 b. 2
que obtenga un número negativo pierde. ¿Cuál es la c. 7 d. 12
estrategia ganadora? TIMSS 2003, Grado 4
9. Cuando se imprime un libro, se pasan pliegos por una im-
presora y después se doblan para formar el libro. Para ver Pregunta del National Assessment of Educational Progress
cómo funciona esto comencemos con un libro sencillo, (NAEP) (Evaluación Nacional del Progreso Educativo)
formado por una hoja de tamaño carta de 812 * 11pulg. Habrá 58 personas en un desayuno y cada una comerá 2
Dobla la hoja a la mitad, a lo largo, forma un libro y nu- huevos. Hay 12 huevos en cada cartón. ¿Cuántos carto-
mera sus páginas de 1 a 4. Cuando abres la hoja de papel, nes de huevo se necesitarán para el desayuno?
los números 2 y 3 están en un lado de la hoja y los núme- a. 9 b. 10
ros 1 y 4 están en el otro lado. La suma de los números c. 72 d. 116
en cada lado de la hoja de papel es 5 y la suma de los nú- NAEP 2007, Grado 4
meros de las páginas es 10. Si se usan dos hojas de papel

© Lopez Mateos Editores. ISBN 978-607-95583-2-1, obra completa, versión electrónica, ISBN 978-607-95583-3-8, volumen 1, versión electrónica. Ejemplar asignado a: Helecto Villarroel gutierrez -
helecto@gmail.com. Fecha: 27 de octubre de 2014. Prohibida su modificación, copia o distribución.
22 Una introducción a la resolución de problemas

ROMPECABEZAS Diez mujeres están pescando sentadas en fila en un bote. El asiento del centro está
vacío. Las cinco mujeres sentadas al frente quieren cambiar de asiento con las cinco sentadas atrás. Una
persona se puede mover de su asiento al siguiente que esté vacío o puede pasar sobre otra persona sin
que zozobre el bote. ¿Cuál es el número mínimo de movimientos necesarios para que las cinco mujeres
sentadas al frente cambien de lugar con las cinco sentadas atrás?

ACTIVIDAD DE LABORATORIO Coloca una moneda de $20, una de $10 y una de $5 en la posición
A mostrada en la figura 1-10. Trata de mover estas monedas, una por una, a la posición C. En ningún mo-
mento se permite colocar una moneda mayor sobre otra menor. Las monedas se pueden colocar en la
posición B. ¿Cuántos movimientos se necesitan para llevarlas a la posición C? Añade ahora una moneda de
$2 y observa cuántos movimientos son necesarios. Éste es un caso particular del famoso problema de las
Torres de Hanoi, en el cual se pide a los ancianos sacerdotes brahamanes que muevan una pila de 64 dis-
cos de tamaño decreciente, después de lo cual el mundo acabará. ¿Cuánto tiempo tardarán si efectúan
un movimiento por segundo?

ST
RU
BERT

LI
T

L I

BE Y 199
GOD WE

RTY
IN

4
IN GOD
WE
TRUST

1 9 9 4

A B C

Figura 1-10

1-2 Exploración con patrones


Las matemáticas se han descrito como el estudio de los patrones. Hay patrones donde sea
—en papel tapiz, mosaicos, tráfico y aun en los horarios de la televisión. Cuando se come-
ten crímenes en serie, los investigadores policíacos estudian los archivos de cada caso en
busca del modus operandi, o patrón de operación. Los científicos buscan patrones para aislar
variables de manera que se logren conclusiones válidas en su investigación. En los Principios
y objetivos hallamos lo siguiente:

. . . los estudiantes deberán investigar patrones numéricos y geométricos y expresarlos matemáti-


camente en palabras o en símbolos. Deberán analizar la estructura del patrón y cómo crece o
cambia, organizar sistemáticamente dicha información y usar su análisis para desarrollar generali-
zaciones acerca de las relaciones matemáticas en el patrón. (p. 159)

Los patrones no necesariamente son numéricos, como se muestra en la actividad Ahora


intenta éste 1-10.

© Lopez Mateos Editores. ISBN 978-607-95583-2-1, obra completa, versión electrónica, ISBN 978-607-95583-3-8, volumen 1, versión electrónica. Ejemplar asignado a: Helecto Villarroel gutierrez -
helecto@gmail.com. Fecha: 27 de octubre de 2014. Prohibida su modificación, copia o distribución.
Sección 1-2 Exploración con patrones 23

AHORA INTENTA ÉSTE 1-10


a. Halla tres términos más de manera que se continúe un patrón:
o, ^, ^, o, ^, ^, o ___, ___, ___
b. Describe con palabras el patrón hallado en la parte (a).

Los patrones pueden ser sorprendentes. Considera el ejemplo 1-1.

Ejemplo 1-1 a. Describe los patrones que ves en lo siguiente:










1 + 0#9 = 1
2 + 1 # 9 = 11
3 + 12 # 9 = 111
4 + 123 # 9 = 1111
5 + 1234 # 9 = 11111

b. ¿Continúa el patrón anterior? Expresa por qué sí o por qué no.

Solución a. Hay varios patrones posibles. Por ejemplo, los números en el extremo izquierdo
son números naturales, esto es, números del conjunto {1, 2, 3, 4, 5, Á }. El pa-
trón comienza con 1 y continúa al siguiente número natural mayor en cada línea
sucesiva. Los números de “en medio” son el producto de dos números, el se-
gundo de los cuales es 9. El primer número en el primer producto es 0; después
ese primer número se forma usando números naturales y añadiendo uno más en
cada línea sucesiva. Los números resultantes del lado derecho se forman usando
números 1 y añadiendo un 1 en cada línea sucesiva.
b. El patrón en la ecuación completa parece continuar para varios casos más,
pero no continúa en general; por ejemplo,

13 + 123456789101112 # 9 = 1,111,111,101,910,021

El patrón se rompe cuando el número multiplicado por 9 contiene dígitos


usados previamente.

Como vimos en el ejemplo 1-1, no es confiable determinar un patrón basados en unos


cuantos casos. Cuando hallemos patrones debemos, una de dos, encontrar un contraejem-
plo que muestre que el patrón no es válido en general o explicar por qué el patrón siempre
funciona.
En los Principios y objetivos hallamos lo siguiente:

Cuando los alumnos realizan un descubrimiento o determinan un hecho, en lugar de decirles si es


válido para todos los números o si es correcto, el maestro deberá ayudarlos a que lo determinen
por sí mismos. Los maestros deberían hacer preguntas como “¿Por qué es cierto?”, “¿Cómo lo sa-
bes?” y también deberían mostrar caminos para que los alumnos puedan determinar cuándo una
afirmación es verdadera, una generalización es válida o una respuesta es correcta, y hacerlo por sí
mismos en lugar de depender de la autoridad del maestro o del libro. (p. 126)

© Lopez Mateos Editores. ISBN 978-607-95583-2-1, obra completa, versión electrónica, ISBN 978-607-95583-3-8, volumen 1, versión electrónica. Ejemplar asignado a: Helecto Villarroel gutierrez -
helecto@gmail.com. Fecha: 27 de octubre de 2014. Prohibida su modificación, copia o distribución.
24 Una introducción a la resolución de problemas

◆ Nota de Razonamiento inductivo


investigación Los científicos realizan observaciones y proponen leyes generales basados en patrones. Los
estudiosos de la estadística usan patrones cuando llegan a conclusiones basados en los datos
En un estudio acerca
de la comprensión de
recolectados. Este proceso, el razonamiento inductivo, es el método de hacer generaliza-
demostraciones ma- ciones con base en observaciones y patrones. Aunque el razonamiento inductivo puede con-
temáticas, se halló ducir a descubrimientos, su debilidad consiste en que las conclusiones se obtienen sólo de las
que el 80% de los es- evidencias recolectadas. Si no se han verificado todos los casos, existe la posibilidad de que
tudiantes de grado en algún otro caso la conclusión obtenida sea falsa. En matemáticas, el razonamiento induc-
11 no comprendía el tivo nos puede conducir a emitir una conjetura, una afirmación que pensamos es verdadera
concepto de contra- pero que no se ha demostrado si es verdadera o falsa. Por ejemplo, basados únicamente en
ejemplo, y más del que 02 = 0 y que 12 = 1, podríamos emitir la conjetura de que cualquier número elevado al
70% del grupo no
cuadrado es igual a sí mismo. Cuando hallamos un ejemplo que contradice la conjetura, hemos
podía diferenciar en-
tre razonamiento in-
exhibido un contraejemplo y hemos demostrado que la conjetura es falsa en general. A los
ductivo y deductivo, estudiantes se les dificulta comprender el concepto de contraejemplo, como se señala en la
lo cual incluía no es- Nota de investigación. Para mostrar que la conjetura anterior es falsa, es suficiente exhibir al
tar consciente de que menos un contraejemplo, digamos 22 = 4. A veces es difícil hallar un contraejemplo, pero el
el razonamiento in- hecho de no poder hallar uno no significa que la conjetura sea verdadera.
ductivo no demues- A continuación vemos un patrón que sí funciona y nos ayuda a resolver un problema.
tra nada (Williams ¿Cómo puedes hallar la suma de tres números naturales consecutivos sin efectuar la opera-
1980). ◆ ción? Damos varios ejemplos. Busca un patrón en estos ejemplos.
14 + 15 + 16 1452
19 + 20 + 21 1602
99 + 100 + 101 13002
Después de estudiar las sumas, se revela el patrón de multiplicar por 3 el número de en me-
dio. Se pueden probar otros números para ver si podemos exhibir un contraejemplo. El pa-
trón sugiere otros planteamientos matemáticos a considerar. Por ejemplo,
1. ¿Esto funciona con cualesquier tres números naturales consecutivos?
2. ¿Cómo puedes hallar la suma de un número impar de números naturales consecutivos?
3. ¿Qué sucede si hay un número par de números naturales consecutivos?

Para responder a la pregunta (1), demostramos que la suma de tres números naturales con-
◆ Nota de secutivos es igual a 3 por el número de en medio.
investigación Demostración
Al comparar solucio- Sea n el primero de tres números naturales consecutivos. Entonces los tres números son n,
nes y cuestionar el n + 1 y n + 2 . La suma de estos tres números es n + 1n + 12 + 1n + 22 = 3n +
razonamiento del
otro, los estudiantes
3 = 31n + 12. Por lo tanto, la suma de los tres números naturales consecutivos es 3 veces
comienzan a apren- el número de en medio.
der a describir rela-
ciones válidas en El peligro de hacer conjeturas basados en unos cuantos casos
muchos casos y a de- En los Principios y objetivos hallamos lo siguiente:
sarrollar y defender
argumentos acerca
Durante los grados 3–5, los estudiantes deberán avanzar hacia un razonamiento que dependa de
de por qué esas rela-
ciones se pueden ge- relaciones y propiedades. Es necesario plantear retos a los estudiantes con preguntas como ¿Qué
neralizar y a qué pasaría si te diera veinte problemas más como éste? ¿Los trabajarías todos de la misma manera?
casos se aplican ¿Cómo lo sabes? (p. 190)
(Maher y Martino
1996). ◆ En la Nota de investigación se hace mayor énfasis en este concepto .

© Lopez Mateos Editores. ISBN 978-607-95583-2-1, obra completa, versión electrónica, ISBN 978-607-95583-3-8, volumen 1, versión electrónica. Ejemplar asignado a: Helecto Villarroel gutierrez -
helecto@gmail.com. Fecha: 27 de octubre de 2014. Prohibida su modificación, copia o distribución.
Sección 1-2 Exploración con patrones 25

A continuación ilustraremos el peligro de emitir una conjetura basados en unos cuantos


casos. En la figura 1-11, escogemos puntos en un círculo y los conectamos para formar re-
giones distintas, que no se traslapen. En la figura, 2 puntos determinan 2 regiones, 3 puntos
determinan 4 regiones y 4 puntos determinan 8 regiones. ¿Cuál es el máximo número de
regiones que podrían determinarse con 10 puntos?

2
1
7
2 1
6
4 2 8 3
5
1 , 3 ,
4
(a) 2 puntos, 2 regiones (b) 3 puntos, 4 regiones (c) 4 puntos, 8 regiones

Figura 1-11

Los datos de la figura 1-11 se registran en la tabla 1-2. Parece que cada vez que agrega-
mos un punto se duplica el número de regiones. Si fuera cierto, para 5 puntos tendríamos 2
veces el número de regiones que con 4 puntos, ó 2 # 8 = 16 = 24, y así sucesivamente. Si
basamos nuestra conjetura en este patrón, podríamos creer que para 10 puntos tendríamos
29, ó 512 regiones. (¿Por qué?)

Tabla 1-2
Número de puntos 2 3 4 5 6 Á 10

Máximo número de regiones 2 4 8 ?

1 Una verificación inicial de esta conjetura es ver si obtenemos 16 regiones para 5 puntos.
8 9 2 Obtenemos una figura similar a la figura 1-12, donde se verifica nuestra suposición de las
10
7 16 regiones. El patrón predice que para 6 puntos tendremos 32 regiones. Traza un círculo y
5 6 16 11 escoge los puntos de modo que no estén arreglados simétricamente ni estén igualmente es-
15 paciados, y cuenta con cuidado las regiones. Obtendrás 31 regiones, no 32 como se predijo.
12
13 3 No importa cómo se localicen los puntos en el círculo, la suposición de 32 regiones no es
14
correcta. El contraejemplo nos dice que no es correcto el patrón de duplicar las regiones;
4
nota que no nos dice si hay o no 512 regiones para 10 puntos, sino sólo que el patrón no se
comporta como conjeturamos.
Figura 1-12
En este ejemplo se sugiere el patrón de apariencia natural 2, 4, 8, 16, Á pero el patrón
no continúa, como se muestra cuando se trazan las figuras. Si vemos sólo los primeros cua-
tro términos de la sucesión 2, 4, 8, 16 fuera de contexto, el patrón de ir duplicando la canti-
dad es lógico. En el contexto de contar el número de regiones de un círculo, el patrón es
incorrecto.

© Lopez Mateos Editores. ISBN 978-607-95583-2-1, obra completa, versión electrónica, ISBN 978-607-95583-3-8, volumen 1, versión electrónica. Ejemplar asignado a: Helecto Villarroel gutierrez -
helecto@gmail.com. Fecha: 27 de octubre de 2014. Prohibida su modificación, copia o distribución.
26 Una introducción a la resolución de problemas

AHORA INTENTA ÉSTE 1-11 Un número primo es un número natural con exactamente dos números
positivos, distintos, que lo dividen: 1 y el número mismo; por ejemplo, 2, 3, 5, 7, 11, 13 son primos. Un día Ana
emitió la conjetura de que la fórmula y = x 2 + x + 11 produciría sólo números primos si en el lugar de x
substituía los números naturales 1, 2, 3, 4, 5. . . . . Ella colocó su trabajo en la tabla 1-3 para x = 1, 2, 3, 4.

Tabla 1-3
x 1 2 3 4
y 13 17 23 31

a. ¿Qué tipo de razonamiento está usando Ana?


b. Prueba con los números que siguen, a ver cómo funciona.
c. ¿Puedes exhibir un contraejemplo para mostrar que la conjetura de Ana es falsa?

Sucesiones aritméticas
Una sucesión es un arreglo ordenado de números, figuras u objetos. Una sucesión tiene
términos identificados como el 1º, 2º, 3º, y así sucesivamente. A veces es posible clasificar
las sucesiones por medio de sus propiedades. Por ejemplo, ¿qué propiedad tienen las pri-
meras tres sucesiones que no tiene la cuarta?
a. 1, 2, 3, 4, 5, 6, Á
b. 0, 5, 10, 15, 20, 25, Á
c. 2, 6, 10, 14, 18, 22, Á
d. 1, 11, 111, 1111, 11111, 111111, Á
En cada una de las primeras tres sucesiones, cada término —comenzando desde el se-
gundo— se obtiene del anterior sumando un número fijo llamado diferencia común o
diferencia. En la parte (a) la diferencia es 1, en la parte (b) la diferencia es 5 y en la parte (c)
la diferencia es 4. Sucesiones como las tres primeras son sucesiones aritméticas. Una
sucesión aritmética es aquella en que cada término se obtiene del anterior mediante la
suma o resta de un número fijo. La sucesión en la parte (d) no es aritmética pues no existe
un número fijo que puedas sumar o restar del término anterior para obtener el siguiente.
También se pueden generar sucesiones aritméticas a partir de objetos, como se muestra
en el ejemplo 1-2.

Ejemplo 1-2 Halla un patrón en el número de cerillos requeridos para continuar el patrón mostrado en
la figura 1-13.








, , ,

Figura 1-13

Solución    Supón que los cerillos se han arreglado de modo que cada figura tiene un
cuadrado más a la derecha que la figura anterior. Nota que añadir un cuadrado a un arreglo
requiere la adición de tres cerillos. Así, el patrón numérico obtenido es 4, 7, 10, 13, 16,
19, Á , una sucesión aritmética con diferencia 3.

© Lopez Mateos Editores. ISBN 978-607-95583-2-1, obra completa, versión electrónica, ISBN 978-607-95583-3-8, volumen 1, versión electrónica. Ejemplar asignado a: Helecto Villarroel gutierrez -
helecto@gmail.com. Fecha: 27 de octubre de 2014. Prohibida su modificación, copia o distribución.
Sección 1-2 Exploración con patrones 27

Se puede describir informalmente una sucesión aritmética mediante el patrón “sumar d”,
donde d es la diferencia común. En el ejemplo 1-2, d = 3. En lenguaje infantil el patrón del
ejemplo 1-2 es “sumar 3”. Éste es un ejemplo de patrón recursivo. En un patrón recur-
sivo, después de uno o más términos consecutivos que se dan para comenzar, cada término
sucesivo se obtiene a partir del término o términos anteriores. Por ejemplo, 3, 6, 9 , Á es
otra sucesión de “sumar 3” que comienza con 3, y 1, 2, 3, 5, 8, 13, Á es un patrón recur-
sivo en el cual el término siguiente (a partir del tercero) se obtiene al sumar los dos térmi-
nos anteriores.
Se usan patrones recursivos en una hoja de cálculo, como vemos en la tabla 1-4, donde
en la columna A se registra el orden de los términos; los encabezados de las columnas son
A, B, etc. El primer registro en la columna B (en la celda B1) es 4; y para hallar el término
de la celda B2 usamos el número de la celda B1 y le sumamos 3. Una vez hallado el regis-
tro de la celda B2, se continúa el patrón por medio del comando Llenar Abajo. En lenguaje
de hoja de cálculo, la fórmula = B1 + 3 halla cualquier término después del primero, su-
mando 3 al término anterior. La fórmula está basada en un patrón recursivo; es una fór-
mula recursiva. (Para instrucciones más detalladas acerca de cómo usar una hoja de
cálculo, ver el Manual de tecnología.)

Tabla 1-4

Si quieres hallar el número de cerillos en la figura número 100 del ejemplo 1-2, puedes
usar una hoja de cálculo o hallar un tipo diferente de regla general para encontrar el nú-
mero de cerillos dado el número del término. Aquí, de nuevo, es útil la estrategia de hacer
una tabla para resolver problemas.
La hoja de cálculo de la tabla 1-4 proporciona una manera fácil de hacer una tabla. La co-
lumna A da la numeración de los términos y la columna B da los términos de la sucesión. Si
se construye dicha tabla sin usar una hoja de cálculo, podría verse como la tabla 1-5. Las
elipsis, denotadas con tres puntos, indican que la sucesión sigue de la misma manera. Nota
que cada término es una suma de 4 más cierta cantidad de veces 3. Vemos que la cantidad
de veces 3 es 1 menos que el número del término. Este patrón deberá continuar pues el pri-
mer término es 4 + 0 # 3 y cada vez que incrementamos en 1 el número del término, añadi-
mos un 3 más. Así, se ve que el término 100-ésimo es 4 + 1100 - 123, y, en general, el
término n-ésimo, an, es 4 + 1n - 123. Escribimos esto como an = 4 + 1n - 123. Nota
que 4 + 1n - 123 se puede escribir como 3n + 1.

© Lopez Mateos Editores. ISBN 978-607-95583-2-1, obra completa, versión electrónica, ISBN 978-607-95583-3-8, volumen 1, versión electrónica. Ejemplar asignado a: Helecto Villarroel gutierrez -
helecto@gmail.com. Fecha: 27 de octubre de 2014. Prohibida su modificación, copia o distribución.
28 Una introducción a la resolución de problemas

Tabla 1-5

Número de término Término


1 4
2 7 = 4 + 3 = 4 + 1#3
3 10 = 14 + 1 # 32 + 3 = 4 + 2 # 3
4 13 = 14 + 2 # 32 + 3 = 4 + 3 # 3
# #
# #
# #
n 4 + 1n - 123

Incluso podemos usar otro enfoque para obtener el número de cerillos en el término número
cien de la figura 1-13. Procedamos así: si la figura formada por cerillos tiene 100 cuadrados,
podríamos hallar el número total de cerillos sumando el número de cerillos horizontales y el
de cerillos verticales. Hay 2 # 100 cerillos colocados horizontalmente (¿puedes ver por qué?).
Nota que en la primera figura hay 2 cerillos colocados verticalmente, en la segunda hay 3 y en
la tercera hay 4. En la figura número 100 deberá haber 100 + 1 cerillos verticales. En total
tendremos 2 # 100 + 1100 + 12, ó 301, cerillos en la figura número cien. De manera análoga,
en la figura n-ésima habría 2n cerillos horizontales y 1n + 12 verticales, para dar un total de
3n + 1. Resumimos esto en la tabla 1-6.

Tabla 1-6
Número de cerillos Número de cerillos
Número de término horizontales Verticales Total

1 2 2 4
2 4 3 7
3 6 4 10
4 8 5 13
# # # #
# # # #
# # # #
100 200 101 301
# # # #
# # # #
# # # #
n 2n n + 1 2n + 1n + 12 = 3n + 1

Si nos dieran el valor de un término, podríamos usar la fórmula del término n-ésimo de la
tabla 1-6 para trabajar hacia atrás, o de manera regresiva, para hallar el número de término.
Por ejemplo, dado el término 1798, sabemos que 3n + 1 = 1798. Por lo tanto, 3n = 1797 y
n = 599. En consecuencia, el término número 599 es 1798. Obtendríamos la misma respuesta
despejando n en 4 + 1n - 123 = 1798.
En el problema de los cerillos hallamos el término n-ésimo de una sucesión. Si nos dan el
término n-ésimo podemos hallar cualquier término de la sucesión, como se muestra en el
ejemplo 1-3.
Ejemplo 1-3 Halla los primeros cuatro términos de una sucesión cuyo término n-ésimo está dado, y di
en qué caso la sucesión es aritmética:








a. an = 4n + 3 b. an = n2 - 1

© Lopez Mateos Editores. ISBN 978-607-95583-2-1, obra completa, versión electrónica, ISBN 978-607-95583-3-8, volumen 1, versión electrónica. Ejemplar asignado a: Helecto Villarroel gutierrez -
helecto@gmail.com. Fecha: 27 de octubre de 2014. Prohibida su modificación, copia o distribución.
Sección 1-2 Exploración con patrones 29

Solución a. Número de término Término


1 4#1 + 3 = 7
2 4#2 + 3 = 11
3 4#3 + 3 = 15
4 4#4 + 3 = 19
Así, los primeros cuatro términos de la sucesión son 7, 11, 15, 19. Esta suce-
sión es aritmética, con diferencia 4.

b. Número de término Término


1 12 - 1 = 0
2 22 - 1 = 3
3 32 - 1 = 8
4 42 - 1 = 15
Así, los primeros cuatro términos de la sucesión son 0, 3, 8, 15. Esta sucesión
no es aritmética ya que no hay diferencia común.

Generalización de sucesiones aritméticas


Para generalizar nuestro trabajo con sucesiones aritméticas, supongamos que el primer tér-
mino en una sucesión aritmética es a1 y que la diferencia es d. Se puede usar la estrategia de
hacer una tabla para investigar el término general de la sucesión a1, a1 + d, a1 + 2d,
a1 + 3d, Á como se muestra en la tabla 1-7. El n-ésimo término de cualquier sucesión con pri-
mer término a1 y diferencia d está dado por an = a1 + 1n - 12d. Por ejemplo, en la sucesión
aritmética 5, 9, 13, 17, 21, 25, Á , el primer término es 5 y la diferencia es 4. Así, el término
n-ésimo está dado por a1 + 1n - 12d = 5 + 1n - 124. Simplificando algebraicamente ob-
tenemos 5 + 1n - 124 = 5 + 4n - 4 = 4n + 1. Verifica para ver si 4n + 1 genera la su-
cesión 5, 9, 13, 17, 21, Á .

Tabla 1-7

Número de término Término


1 a1
2 a1 + d
3 a1 + 2d
4 a1 + 3d
5 a1 + 4d
# #
# #
# #
n a1 + 1n - 12d

O B S E R VA C I Ó N El término n-ésimo de cualquier sucesión con primer término a1 y


diferencia d está dado por an = a1 + 1n - 12d, donde n es un número natural pero no
hay restricciones para d.

AHORA INTENTA ÉSTE 1-12 En una sucesión aritmética con segundo término 11 y quinto término 23,
halla el término cien.

© Lopez Mateos Editores. ISBN 978-607-95583-2-1, obra completa, versión electrónica, ISBN 978-607-95583-3-8, volumen 1, versión electrónica. Ejemplar asignado a: Helecto Villarroel gutierrez -
helecto@gmail.com. Fecha: 27 de octubre de 2014. Prohibida su modificación, copia o distribución.
30 Una introducción a la resolución de problemas

Ejemplo 1-4 Los diagramas de la figura 1-14 muestran la estructura molecular de los alcanos, una clase
de hidrocarburos. C representa un átomo de carbono y H un átomo de hidrógeno. El









segmento que los une muestra un enlace químico. (Observación: CH4 significa C1H4.)

H H H H H H
u u u u u u
H C H , H C C H , H C C C H

u
u
u
u

u
u

u
u
u u u u u u
H H H H H H
metano (CH4) etano (C2H6) propano (C3H8)

Figura 1-14
a. El hectano es un alcano con 100 átomos de carbono. ¿Cuántos átomos de hidrógeno
tiene?
b. Escribe una regla general para los alcanos CnHm que muestre la relación entre m y n.

Solución a. Para determinar la relación entre el número de átomos de carbono y de hidró-


geno, hay que estudiar la figura de los alcanos y no tomar en cuenta los áto-
mos de hidrógeno que están en los extremos izquierdo y derecho. Con esta
restricción podemos ver que por cada átomo de carbono hay dos átomos de
hidrógeno. Por lo tanto, hay el doble de átomos de hidrógeno que de carbo-
no, más los dos átomos de hidrógeno de los extremos. Por ejemplo, cuando
hay 3 átomos de carbono hay 12 # 32 + 2, u 8, átomos de hidrógeno. Esto se
resume en la tabla 1-8. Si extendemos la tabla para 4 átomos de carbono, ob-
tendremos 12 # 42 + 2, ó 10, átomos de hidrógeno. Para 100 átomos de car-
bono hay 12 # 1002 + 2, ó 202, átomos de hidrógeno.
b. En general, para n átomos de carbono se tendrían n átomos de hidrógeno por
arriba, n por debajo y 2 más a los lados. Entonces, el total de número de áto-
mos de hidrógeno sería 2n + 2. Como se designó con m al número de átomos
de hidrógeno, se sigue que m = 2n + 2.

Tabla 1-8
Núm. de átomos de carbono Núm. de átomos de hidrógeno

1 4
2 6
3 8
# #
# #
# #
100 ?
# #
# #
# #
n m

Ejemplo 1-5 Un teatro está construido de manera que hay 20 asientos en la primera fila y 4 asientos
adicionales en cada fila consecutiva. La última fila tiene 144 asientos. ¿Cuántas filas tiene el








teatro?
Solución Como en cada fila consecutiva se añaden 4 asientos, el número de asientos en
una fila forma una sucesión aritmética. El primer término, a1, de la sucesión es 20 y la
© Lopez Mateos Editores. ISBN 978-607-95583-2-1, obra completa, versión electrónica, ISBN 978-607-95583-3-8, volumen 1, versión electrónica. Ejemplar asignado a: Helecto Villarroel gutierrez -
helecto@gmail.com. Fecha: 27 de octubre de 2014. Prohibida su modificación, copia o distribución.
Sección 1-2 Exploración con patrones 31

diferencia d es 4. El último término de la sucesión es 144. Se podría usar una hoja de cálculo
computarizada para contar el número de términos en la sucesión 20, 24, 28, Á , 144. Sin
embargo, sin tecnología podemos contar los términos de la manera que sigue: en una
sucesión aritmética, el término, an = a1 + 1n - 12d, donde a1 es el primer término, d es la
diferencia y n es el número del término. En este caso, a1 = 20 y d = 4. Por lo tanto,

an = a1 + 1n - 12d = 20 + 1n - 124

Queremos ahora hallar el número del término cuando an = 20 + 1n - 124 es igual a 144.
Entonces,
20 + 1n - 124 = 144
1n - 124 = 124
n - 1 = 31
n = 32

Esto muestra que hay 32 filas en el teatro.


Sucesión de Fibonacci
El popular libro El Código Da Vinci ha renovado el interés por una de las más famosas suce-
siones de todos los tiempos, la sucesión de Fibonacci. Se habla de la sucesión de Fibonacci
en la siguiente tira cómica. ¿Puedes dar una regla para obtener dicha sucesión?

ESTAMOS
¿ES LA
¡CORRECTO! CANSADOS DE
SERIE DE
FIBONACCI? ¡TOUCHDOWN QUE EL FÚTBOL
MARCUS! FAVOREZCA A
lOS FORTA-
CHONES.

◆ Nota Leonardo de Pisa nació en alrededor de 1170. Su apellido real era Bonaccio, pero él
histórica prefirió el alias de Fibonacci, derivado del latín filius Bonacci que significa “hijo de Bo-
nacci”. En sus viajes, Leonardo aprendió el sistema numérico indoarábigo con los mo-
ros. En su libro Liber Abaci (1202), describió los trabajos del sistema indoarábigo. Uno
de los problemas incluidos en su libro fue el ahora famoso problema de los conejos,
cuya solución es la sucesión 1, 1, 2, 3, 5, 8, 13, 21, Á , que se conoció como sucesión de
Fibonacci ◆

© Lopez Mateos Editores. ISBN 978-607-95583-2-1, obra completa, versión electrónica, ISBN 978-607-95583-3-8, volumen 1, versión electrónica. Ejemplar asignado a: Helecto Villarroel gutierrez -
helecto@gmail.com. Fecha: 27 de octubre de 2014. Prohibida su modificación, copia o distribución.
32 Una introducción a la resolución de problemas

En la tira cómica, la sucesión de Fibonacci tiene al 0 como término de inicio. Usualmente


la sucesión es como sigue:

1, 1, 2, 3, 5, 8, 13, 21, 34, 55, 89, 144, Á

La sucesión se llama así en honor del italiano Leonardo de Pisa, mejor conocido como
Fibonacci. Esta sucesión no es aritmética pues no hay una diferencia fija d.
La manera matemática convencional de representar un número de Fibonacci es F1 para
el primer término, F2 para el segundo término, F3 para el tercer término y, en general, Fn
para el n-ésimo término. Si queremos indicar los números de Fibonacci que vienen después
de Fn, los escribimos como Fn + 1, Fn + 2, y así sucesivamente. El número que viene antes de
Fn es Fn - 1. Con esta notación, la regla para generar la sucesión de Fibonacci se puede escri-
bir como

Fn = Fn - 1 + Fn - 2, para n = 3, 4, 5, Á

Nota que esta regla no se puede aplicar a los dos primeros números de Fibonacci. Como
F1 = 1 y F2 = 1, entonces F3 = 1 + 1 = 2. Las semillas F1 = 1 y F2 = 1 y la regla Fn =
Fn - 1 + Fn - 2 dan otro ejemplo de una definición recursiva pues la regla en la sucesión de-
fine un número usando números anteriores en la misma sucesión. Usando las semillas y
la regla podemos hallar cualquier número de Fibonacci. Para hallar F100 , con lo que
sabemos hasta ahora, deberíamos conocer F98 y F99 . Con una hoja de cálculo se puede
generar fácilmente esta sucesión.

AHORA INTENTA ÉSTE 1-13


a. Suma los primeros tres números de Fibonacci.
b. Suma los primeros cuatro números de Fibonacci.
c. Suma los primeros cinco números de Fibonacci.
d. Suma los primeros seis números de Fibonacci.
e. Suma los primeros siete números de Fibonacci.
f. ¿Qué patrón hay en las sumas de las partes (a)–(e) y cualquiera de los números restantes en la sucesión
de Fibonacci?
g. Escribe una regla para el patrón que obtuviste en la parte (f) usando la notación para números de Fibonacci.

Sucesiones geométricas
Una niña tiene 2 padres biológicos (su mamá y su papá), 4 abuelos, 8 bisabuelos, 16 tatara-
buelos, y así sucesivamente. El número de ancestros forma la sucesión geométrica 2, 4, 8,
16, 32, Á . Cada término de una sucesión geométrica se obtiene a partir de su predecesor al
multiplicarlo por un número fijo, la razón. En este ejemplo, tanto el primer término como
la razón son iguales a 2. (La razón es 2 porque cada persona tiene dos padres.) Para hallar el
término n-ésimo, an, examina el patrón de la tabla 1-9.
En la tabla 1-9, cuando el término dado se escribe como potencia de 2, el número del tér-
mino es el exponente. Siguiendo este patrón, el término 10, a10, es 210, ó 1024, el término

© Lopez Mateos Editores. ISBN 978-607-95583-2-1, obra completa, versión electrónica, ISBN 978-607-95583-3-8, volumen 1, versión electrónica. Ejemplar asignado a: Helecto Villarroel gutierrez -
helecto@gmail.com. Fecha: 27 de octubre de 2014. Prohibida su modificación, copia o distribución.
Sección 1-2 Exploración con patrones 33

Tabla 1-9
Número de término Término
1 2 = 21
2 4 = 2 # 2 = 22
3 8 = 12 # 22 # 2 = 23
4 16 = 12 # 2 # 22 # 2 = 24
5 32 = 12 # 2 # 2 # 22 # 2 = 25
# #
# #
# #

100, a100, es 2100, y el término n-ésimo, an, es 2n. Así, el número de ancestros en la n-ésima
generación anterior es 2n. La notación usada en la tabla 1-9 se puede generalizar de la si-
guiente manera:

Definición
n factores

Si n es un número natural, entonces an = a # a # a # Á # a. Si n = 0 y a Z 0, entonces a0 = 1.

Las sucesiones geométricas juegan un papel muy importante en la vida cotidiana. Por
ejemplo, supón que tienes $1000 en un banco que paga 5% de interés anual. Si no depositas
o retiras dinero, entonces al final del primer año tendrás el dinero inicial más el 5%, esto es,
Año 1: $1000 + 0.051$10002 = $100011 + 0.052 = $100011.052 = $1050
Si no depositas o retiras dinero, entonces al final del segundo año tendrás 5% más dinero
que el año anterior.
Año 2: $1050 + 0.051$10502 = $105011 + 0.052 = $105011.052 = $1102.50
La cantidad de dinero en la cuenta después de cualquier número de años se puede hallar al
notar que cada peso invertido durante un año se convierte en 1 + 0.05 # 1, ó 1.05 pesos. Por
lo tanto, la cantidad de cada año se obtiene multiplicando la cantidad del año anterior por
1.05. Las cantidades en el banco después de transcurrido cada año forman una sucesión geo-
métrica pues la cantidad de cada año (comenzando con el año 2) se obtiene multiplicando la
cantidad del año anterior por el mismo número. Lo anterior se resume en la tabla 1-10.

Tabla 1-10

Número de término (Año) Término (Cantidad al principio de cada año)

1 $1000
2 $100011.0521 = $1050.00
3 $100011.0522 = $1102.50
4 $100011.0523 = $1157.63
# #
# #
# #
n $100011.052n - 1

© Lopez Mateos Editores. ISBN 978-607-95583-2-1, obra completa, versión electrónica, ISBN 978-607-95583-3-8, volumen 1, versión electrónica. Ejemplar asignado a: Helecto Villarroel gutierrez -
helecto@gmail.com. Fecha: 27 de octubre de 2014. Prohibida su modificación, copia o distribución.
34 Una introducción a la resolución de problemas

Tabla 1-11 Cómo hallar el término n-ésimo de una sucesión geométrica


Número de Es posible hallar el término n-ésimo, an, de cualquier sucesión geométrica dados el
término Término primer término y la razón. Si el primer término es a1 y la razón es r, entonces los tér-
minos son como se indica en la tabla 1-11. Nota que el segundo término es a1r , el
1 a1
tercer término es a1r 2 y el cuarto término es a1r 3 . La potencia de r en cada término
2 a1r
es 1 menos que el número del término. Este patrón continúa puesto que multiplica-
3 a1r 2
mos por r para obtener el término siguiente. Así, el término n-ésimo, an , es a1r n - 1 .
4 a1r 3
Para n = 1 , tenemos a1r1 - 1 = a1r0 . Como el primer término es a1 , entonces
5 a1r 4
a1r0 = a1 . Para todos los números r Z 0 , tenemos que r0 = 1 . Para la sucesión geo-
# #
métrica 3, 12, 48, 192, Á , el primer término es 3 y la razón es 4, y así, el término n-
# #
# ésimo, an , está dado por an = a1r n - 1 = 3 # 4n - 1.
#
n a1r n - 1
O B S E R VA C I Ó N El término n-ésimo de una sucesión geométrica con primer término a1
y razón r es an = a1 # r n - 1, donde n es un número natural y r Z 0.

AHORA INTENTA ÉSTE 1-14


a. Hay dos bacterias en un plato. El número de bacterias se triplica cada hora. Siguiendo este patrón, halla
el número de bacterias en el plato al cabo de 10 horas y al cabo de n horas.
b. Supón que en lugar de crecer geométricamente, como en la parte (a), el número de bacterias crece
aritméticamente en 3 cada hora. Compara el crecimiento al cabo de 10 horas y al cabo de n horas. Haz
un comentario sobre la diferencia en el crecimiento de una sucesión geométrica versus una sucesión
aritmética.

Otras sucesiones
Los números figurados son ejemplos de sucesiones que no son ni aritméticas ni geométri-
cas. Dichos números pueden representarse con puntos arreglados en la forma de algunas fi-
guras geométricas. El número 1 es el comienzo de la mayoría de los patrones con números
figurados. El arreglo en la figura 1-15 representa los primeros cuatro términos de la suce-
sión de números triangulares.

, , ,
1 punto 3 puntos 6 puntos 10 puntos

Figura 1-15

Los números triangulares se pueden escribir, numéricamente, como 1, 3, 6, 10, 15, Á . Esta
sucesión no es aritmética pues no hay una diferencia común, como lo muestra la figura 1-16.
No es una sucesión geométrica porque no hay una razón común. Tampoco es una sucesión de
Fibonacci.
1 3 6 10 15
(Primera diferencia) 2 3 4 5

Figura 1-16

© Lopez Mateos Editores. ISBN 978-607-95583-2-1, obra completa, versión electrónica, ISBN 978-607-95583-3-8, volumen 1, versión electrónica. Ejemplar asignado a: Helecto Villarroel gutierrez -
helecto@gmail.com. Fecha: 27 de octubre de 2014. Prohibida su modificación, copia o distribución.
Sección 1-2 Exploración con patrones 35

Sin embargo, la sucesión de las diferencias 2, 3, 4, 5, Á , es una sucesión aritmética con di-
ferencia 1, como lo muestra la figura 1-17. Los siguientes términos sucesivos de la sucesión
original se muestran en color en la figura 1-17.

1 3 6 10 15 21 28
(Primera diferencia) 2 3 4 5 6 7
(Segunda diferencia) 1 1 1 1 1

Figura 1-17

La tabla 1-12 sugiere un patrón para hallar los términos siguientes y el término n-ésimo
de los números triangulares. El segundo término se obtuvo sumándole 2 al primero; el ter-
cer término se obtuvo sumándole 3 al segundo; y así sucesivamente.

Tabla 1-12

Número de término Término

1 1
2 3 = 1 + 2
3 6 = 1 + 2 + 3
4 10 = 1 + 2 + 3 + 4
5 15 = 1 + 2 + 3 + 4 + 5
# #
# #
# #
10 55 = 1 + 2 + 3 + 4 + 5 + 6 + 7 + 8 + 9 + 10

En general, debido a que el n-ésimo número triangular tiene n puntos en la n-ésima fila, es igual
a la suma de los puntos en el número triangular anterior (el 1n - 12-ésimo) más los n puntos de
la n-ésima fila. Siguiendo este patrón, el término décimo es 1 + 2 + 3 + 4 + 5 + 6 +
7 + 8 + 9 + 10, ó 55, y el término n-ésimo, an, es 1 + 2 + 3 + 4 + 5 + Á + 1n - 12 + n.
Este problema es similar al Problema de Gauss de la sección 1-1. Debido al trabajo realizado en
la sección 1-1, sabemos que
n1n + 12
an =
2
A continuación consideremos los primeros cuatro números cuadrados de la figura 1-18.
Estos números cuadrados, 1, 4, 9, 16, Á , se pueden escribir como 12, 22, 32, 42, y así sucesi-
vamente. El número de puntos en el arreglo 10-ésimo es 102, el número de puntos en el
arreglo 100-ésimo es 1002 y el número de puntos en el arreglo n-ésimo es n2. La sucesión
de números cuadrados no es aritmética ni geométrica. Averigua si la sucesión de las prime-
ras diferencias es una sucesión aritmética y di por qué.

, , ,
1 punto 4 puntos 9 puntos 16 puntos

Figura 1-18

© Lopez Mateos Editores. ISBN 978-607-95583-2-1, obra completa, versión electrónica, ISBN 978-607-95583-3-8, volumen 1, versión electrónica. Ejemplar asignado a: Helecto Villarroel gutierrez -
helecto@gmail.com. Fecha: 27 de octubre de 2014. Prohibida su modificación, copia o distribución.
36 Una introducción a la resolución de problemas

Ejemplo 1-6 Usa diferencias para encontrar un patrón. Después, suponiendo que continúe el patrón









descubierto, halla el séptimo término en cada una de las sucesiones siguientes:
a. 5, 6, 14, 29, 51, 80, Á
b. 2, 3, 9, 23, 48, 87, Á

Solución a. Vemos a continuación la sucesión de las primeras diferencias:

(Primera diferencia)

Para descubrir un patrón para la sucesión original, tratamos de hallar un


patrón para la sucesión de diferencias 1, 8, 15, 22, 29, . . . . Esta sucesión es
aritmética con diferencia fija 7:

(Primera diferencia)
(Segunda diferencia)

Así, el sexto término en la primera diferencia es 29 + 7 , ó 36, y el séptimo


término de la sucesión original es 80 + 36 , ó 116. ¿Qué número sigue al 116?
b. Como la segunda diferencia no es un número fijo, seguimos hasta la tercera
diferencia, como se muestra:

(Primera diferencia)
(Segunda diferencia)
(Tercera diferencia)

La tercera diferencia es un número fijo; por lo tanto, la segunda diferencia es


una sucesión aritmética. El quinto término de la sucesión “segunda
diferencia” es 14 + 3, ó 17; el sexto término de la sucesión “primera dife-
rencia” es 39 + 17, ó 56; el séptimo término en la sucesión original es
87 + 56, ó 143.

© Lopez Mateos Editores. ISBN 978-607-95583-2-1, obra completa, versión electrónica, ISBN 978-607-95583-3-8, volumen 1, versión electrónica. Ejemplar asignado a: Helecto Villarroel gutierrez -
helecto@gmail.com. Fecha: 27 de octubre de 2014. Prohibida su modificación, copia o distribución.
Sección 1-2 Exploración con patrones 37

AHORA INTENTA ÉSTE 1-15 En la figura 1-19 se muestran las tres primeras figuras de arreglos de palillos.

, ,
a15 4 a25 12 a35 24

Figura 1-19

a. Traza el siguiente arreglo de palillos.


b. Construye una tabla que muestre el número de término y el número de palillos para n = 1, 2, 3, 4.
c. Usa diferencias para predecir el número de palillos para n = 5, 6, 7.
d. ¿Hallar diferencias es el mejor camino para determinar cuántos palillos hay en a100 ? Di cómo
obtendrías a100 y an.

Cuando se te pida hallar un patrón para una sucesión dada busca, en primer lugar, un pa-
trón que sea fácilmente identificable y determina si la sucesión es aritmética o geométrica.
Si el patrón sigue confuso, puede ser útil tomar diferencias sucesivas. Es posible que ninguno
de los métodos descritos revele un patrón.

Evaluación 1-2 A

1. Para cada una de las siguientes sucesiones de figuras, de- 3. Halla el 100-ésimo y el n-ésimo términos de cada una
termina un patrón posible y traza la que seguiría, de de las sucesiones del problema 2 anterior.
acuerdo con el patrón: 4. Usa una carátula de reloj tradicional para averiguar
a. , , , cuáles son los siguientes tres términos en la sucesión:
1, 6, 11, 4, 9, Á
, , ,
b.
5. En el patrón, 8, 16, 14, 10, Á , se puede usar la suma de
c.
los dígitos para crear el número siguiente. En este caso
cada número sucesor es el doble de la suma de los dígi-
, , tos del número previo.
2. En cada uno de los incisos siguientes, lista términos que a. Halla los tres números siguientes en la sucesión des-
continúen un posible patrón. ¿Cuáles de las siguientes su- crita.
cesiones son aritméticas, cuáles son geométricas y cuáles b. Halla los tres números siguientes en la sucesión 4,
no son ni una ni otra? 16, 49, 169, 256, _____, _____, _____. Describe la
a. 1, 3, 5, 7, 9 regla que usaste.
b. 0, 50, 100, 150, 200 夝c. Halla los tres números siguientes en la sucesión 4, 16,
c. 3, 6, 12, 24, 48 37, 58, 89, 145, 42, 20, _____, _____, _____. Describe
d. 10, 100, 1,000, 10,000, 100,000 la regla que usaste.
e. 9, 13, 17, 21, 25, 29 d. ¿Qué sucederá si la sucesión de la parte (c) se conti-
f. 1, 8, 27, 64, 125 núa indefinidamente?

© Lopez Mateos Editores. ISBN 978-607-95583-2-1, obra completa, versión electrónica, ISBN 978-607-95583-3-8, volumen 1, versión electrónica. Ejemplar asignado a: Helecto Villarroel gutierrez -
helecto@gmail.com. Fecha: 27 de octubre de 2014. Prohibida su modificación, copia o distribución.
38 Una introducción a la resolución de problemas

6. Los siguientes arreglos geométricos sugieren una suce- 13. ¿Cuántos términos hay en cada una de las siguientes
sión de números: sucesiones?
a. 51, 52, 53, 54, Á , 151
b. 1, 2, 22, 23, Á , 260
c. 10, 20, 30, 40, Á , 2000
d. 1, 2, 4, 8, 16, 32, Á , 1024
, , , 14. Halla los primeros cinco términos de las sucesiones cu-
2 6 12 20 yos términos n-ésimos son:
a. an = n2 + 2
b. an = 5n - 1
a. Halla los siguientes tres términos. c. an = 10n - 1
b. Halla el término 100-ésimo. d. an = 3n + 2
c. Halla el término n-ésimo. 15. Exhibe un contraejemplo para cada caso:
7. El primer juego de aspas consta de 5 cuadrados, el se- a. Si x es un número natural, entonces 1x + 52>5 = x + 1.
gundo requiere 9 cuadrados y el tercero lleva 13 cuadra- b. Si x es un número natural, entonces 1x + 422 = x + 16.
dos, según se muestra en la figura. ¿Cuántos cuadrados se 16. Supón que continúa el siguiente patrón de mosaicos, 1n2,
necesitan para construir (a) el 10-ésimo juego de aspas? y responde las preguntas.
(b) el n-ésimo? (c) ¿Cuántos palillos se necesitarán para
construir el n-ésimo juego de aspas?

, , , , , , , ...
a1 a2 a3 a4 a5
a. ¿Cuántos mosaicos hay en la sexta figura, a6?
b. ¿Cuántos mosaicos hay en la n-ésima figura, an?
c. ¿Existe una figura que tenga exactamente 1259 mosai-
cos? De ser así, ¿cuál es?
8. Las figuras de la siguiente sucesión están formadas por
17. Halla los términos tercero, cuarto y quinto de la sucesión
cubos pegados. Si se quiere pintar la superficie expuesta,
si a1 = 2, a2 = 5, y an = 2an - 1 - an - 2.
¿cuántos cuadrados se pintarán en (a) la 10-ésima figura?
18. Considera las siguientes sucesiones:
(b) la n-ésima figura?
300, 500, 700, 900, 1100, 1300, Á
2, 4, 8, 16, 32, 64, Á
, ,
Halla el número del primer término en el cual la sucesión
geométrica es mayor que la sucesión aritmética.
9. En cierto plantel se predijo que la población escolar se 19. Comienza con un trozo de papel. Corta esa pieza de pa-
incrementaría en 50 estudiantes al año durante los 10 pel en cinco trozos. Toma cualquiera de los trozos y cór-
años siguientes. Si la matrícula actual es de 700 estudian- talos de nuevo en cinco piezas, y así sucesivamente.
tes, ¿cuál será la matrícula al cabo de 10 años? a. ¿Qué cantidad de trozos se puede obtener de esta manera?
10. El ingreso anual de Pepe se ha incrementado cada año b. ¿Qué cantidad de trozos se obtuvo en el n-ésimo
en la misma cantidad. En el primer año su ingreso fue de corte?
$24,000 y en el noveno año fue de $31,680. ¿En qué año 20. La sucesión 32, a, b, c, 512, Á es una sucesión geométrica.
tuvo un ingreso de $45,120? Halla a, b, c.
11. La primera diferencia de una sucesión es 2, 4, 6, 8, 21. Supón que continúa el siguiente patrón de puntos:
10, . Á Halla los primeros seis términos de la sucesión a. ¿Cuántos puntos hay en a6?
original en cada uno de los casos siguientes: b. ¿Cuántos puntos hay en la n-ésima figura, an?
a. El primer término de la sucesión original es 3.
b La suma de los primeros dos términos de la sucesión
original es 10.
c. El quinto término de la sucesión original es 35.
12. Lista los siguientes tres términos para continuar con el
patrón en cada uno de los siguientes incisos. (Puede ser
útil hallar diferencias.)
a. 5, 6, 14, 32, 64, 115, 191 , , , , …
b. 0, 2, 6, 12, 20, 30, 42 a1 a2 a3 a4
© Lopez Mateos Editores. ISBN 978-607-95583-2-1, obra completa, versión electrónica, ISBN 978-607-95583-3-8, volumen 1, versión electrónica. Ejemplar asignado a: Helecto Villarroel gutierrez -
helecto@gmail.com. Fecha: 27 de octubre de 2014. Prohibida su modificación, copia o distribución.
Sección 1-2 Exploración con patrones 39

Evaluación 1-2 B

1. Para cada una de las siguientes sucesiones de figuras, de- 7. Al final del día, un tanque contiene 15,360 L de agua. Al
termina un patrón posible y traza la que seguiría, de final de cada día subsecuente queda la mitad del agua, y
acuerdo con el patrón: no se repone. ¿Cuánta agua quedará en el tanque al cabo
a. de 10 días?
, , , ,... 8. Cada lado de cada uno de los pentágonos de abajo mide
b. 1 unidad de longitud.

, , ,...
, , ,...
c.
a. Traza la siguiente figura de la sucesión.
b. ¿Cuál es el perímetro (la distancia alrededor) de cada
, , , ,... una de las primeras cuatro figuras?
c. ¿Cuál es el perímetro de la 100-ésima figura?
2. En cada uno de los incisos siguientes, lista términos que d. ¿Cuál es el perímetro de la n-ésima figura?
continúen un posible patrón. ¿Cuáles de las sucesiones 9. Una escuela secundaria tiene un horario que forma una su-
son aritméticas, cuáles son geométricas y cuáles no son cesión aritmética. Cada periodo de clase dura lo mismo e
ni una ni otra? incluye un 4º periodo para almorzar. Los primeros tres pe-
a. 8, 11, 14, 17, 20, Á riodos comienzan a las 8:10 A.M., 9:00 A.M. y 9:50 A.M., res-
b. 1, 16, 81, 256, 625, Á pectivamente. ¿A qué hora comienza el octavo periodo?
c. 5, 15, 45, 135, 405, Á 10. La primera diferencia de una sucesión es 3, 6, 9, 12,
d. 2, 7, 12, 17, 22, Á 15, Á Halla los primeros seis términos de la sucesión
1 1 1 1 original en cada uno de los casos siguientes:
e. 1, , , , , Á
2 4 8 16 a. El primer término de la sucesión original es 3.
3. Halla el 100-ésimo y el n-ésimo términos de cada una b. La suma de los primeros dos términos de la sucesión
de las sucesiones del problema 2 anterior. original es 7.
4. Observa el siguiente patrón: c. El quinto término de la sucesión original es 34.
11. Lista los siguientes tres términos para continuar con el
1 + 3 = 22, patrón en cada uno de los siguientes incisos. (Puede ser
1 + 3 + 5 = 32, útil hallar diferencias.)
1 + 3 + 5 + 7 = 42 a. 3, 8, 15, 24, 35, 48, Á
b. 1, 7, 18, 37, 67, 111, Á
a. Establece una generalización basada en este patrón. 12. ¿Cuántos términos hay en cada una de las siguientes su-
b. Con base en la generalización enunciada en (a), halla cesiones?
1 + 3 + 5 + 7 + Á + 35 a. 1, 2, 22, 23, Á , 260
b. 9, 13, 17, 21, 25, Á , 353
5. En el siguiente patrón se necesitan 6 palillos para formar c. 38, 39, 40, 41, Á , 198
un hexágono, 11 palillos para formar dos hexágonos y así 13. Halla los primeros cinco términos de las sucesiones cu-
sucesivamente. ¿Cuántos palillos se necesitarían para yos términos n-ésimos son:
construir (a) 10 hexágonos? (b) n hexágonos? a. an = 5n + 6
b. an = 6n - 2
c. an = 5n + 1
, , d. an = 3n - 3
14. Exhibe un contraejemplo para cada caso:
6. Cada una de las figuras siguientes está formada por trián- a. Si x es un número natural, entonces 13 + x2>3 = x.
b. Si x es un número natural, entonces 1x - 222 = x2 - 22.
gulos pequeños, como el primero en la sucesión. (La se-
gunda figura está formada por 4 triángulos.) Emite una
15. Supón que continúa el siguiente patrón de mosaicos,
1n2, y responde las preguntas.
conjetura respecto al número de triángulos pequeños que
se necesitan para construir (a) la 100-ésima figura y (b) la
n-ésima figura.

, , ,... , , , ,...
a a a
© Lopez Mateos Editores. ISBN 978-607-95583-2-1, obra completa, versión electrónica, ISBN 978-607-95583-3-8, volumen 1,1versión 2electrónica. Ejemplar
3 a
4
asignado a: Helecto Villarroel gutierrez -
helecto@gmail.com. Fecha: 27 de octubre de 2014. Prohibida su modificación, copia o distribución.
40 Una introducción a la resolución de problemas

a. ¿Cuántos mosaicos hay en la sexta figura, a6? 18. Considera las siguientes sucesiones:
b. ¿Cuántos mosaicos hay en la n-ésima figura, an?
200, 500, 800, 1100, 1400, 1700, Á
c. ¿Existe una figura que tenga exactamente 449 mosai-
cos? De ser así, ¿cuál es? 1, 3, 9, 27, 81, 243, Á
16. Escribe números impares consecutivos en forma trian- Halla el número del primer término a partir del cual la
gular, como se muestra. sucesión geométrica es mayor que la sucesión aritmética.
1 19. La sucesión 17, a, b, c, 1377, Á es una sucesión geomé-
trica. Halla a, b, c.
3 5
20. Halla la suma de los primeros 43 términos de una suce-
7 9 11 sión aritmética en la que el término número 11 es 83 y el
13 15 17 19 término número 62 es 440.
y así sucesivamente 21. Las abejas hembra nacen de huevos fertilizados y las abe-
jas macho nacen de huevos no fertilizados. Esto significa
que una abeja macho tiene sólo madre, mientras que una
a. Halla la suma de cada uno de los primeros cinco ren- abeja hembra tiene madre y padre. Si se rastrean los an-
glones. cestros de una abeja macho hasta 10 generaciones atrás,
b. ¿Notas algún patrón? ¿cuántas abejas hay en esas 10 generaciones? (Sugerencia:
17. Halla los términos tercero, cuarto y quinto de la suce- La sucesión de Fibonacci puede ayudar.) Explica cómo
sión si a1 = 3, a2 = 6, y an = 3an - 1 - 2an - 2. llegaste a tu respuesta.

Conexiones matemáticas 1-2

Comunicación Solución-abierta
1. Explica en qué sentido las dos sucesiones de cada caso 5. Se pueden usar patrones para contar el número de pun-
son la misma, y en qué sentido son diferentes. tos en un tablero de damas chinas. Se muestran dos pa-
a. 2, 4, 6, 8, 10, Á y 2, 4, 8, 16, 32, Á trones. Determina varios patrones más para contar los
b. 2, 4, 6, 8, 10, Á y 3, 5, 7, 9, 11, Á puntos.
c. 5, 10, 15, 20, 25, Á y 50, 100, 150, 200, 250, Á
2. Da dos ejemplos de cómo puedes usar el razonamiento
inductivo en tu vida cotidiana. ¿Es cierta una conclusión
basada en el razonamiento inductivo?
3. a. Si se suma un número fijo a cada término de una su-
cesión aritmética, ¿el resultado es una sucesión arit-
mética? Justifica la respuesta. ,
b. Si cada término de una sucesión aritmética se multi-
plica por un número fijo, ¿el resultado será siempre
una sucesión aritmética? Justifica la respuesta. 1 + 2 + 3 + . . . + 13 + 3(10) 1 + 3 + 5 + 7 + . . . + 17 + 4(10)
c. Si se suman los términos correspondientes de dos su-
cesiones aritméticas, ¿el resultado es una sucesión 6. Construye un patrón con números figurados y halla una
aritmética? fórmula para el término 100-ésimo. Describe el patrón
4. Una estudiante dice que leyó que Thomas Robert usado y la manera de hallar el término 100-ésimo.
Malthus (1766–1834), un renombrado economista y 7. Una sucesión que siga el mismo patrón que la sucesión
demógrafo británico, aseguraba que el incremento de de Fibonacci pero que sus dos primeros términos no
población, de no controlarlo, se daría en sucesión geo- sean 1, sino números cualesquiera, se llama sucesión tipo
métrica, mientras que la producción de alimentos crece- Fibonacci. Escoge algunas de dichas sucesiones y res-
ría sólo en sucesión aritmética. Esta teoría implica que la ponde las preguntas de Ahora intenta éste 1-13. ¿Se com-
población crece más rápido que la producción de ali- portan de la misma manera estas sucesiones?
mentos. La estudiante se pregunta por qué. ¿Qué le res-
pondes?

© Lopez Mateos Editores. ISBN 978-607-95583-2-1, obra completa, versión electrónica, ISBN 978-607-95583-3-8, volumen 1, versión electrónica. Ejemplar asignado a: Helecto Villarroel gutierrez -
helecto@gmail.com. Fecha: 27 de octubre de 2014. Prohibida su modificación, copia o distribución.
Sección 1-2 Exploración con patrones 41

Aprendizaje colectivo 13. Un estudiante afirma que si el numerador y el denomi-


8. El patrón siguiente se llama triángulo de Pascal en honor nador de una fracción son mayores, respectivamente,
del matemático Blaise Pascal (1623–1662). que el numerador y el denominador de otra fracción, en-
tonces la primera fracción deberá ser la mayor. ¿Cómo le
1 respondes?
1 1 14. Una estudiante afirma que la sucesión 6, 6, 6, 6, 6, Á
1 2 1 nunca cambia, de modo que no es aritmética ni geomé-
1 3 3 1 trica. ¿Cómo le respondes?
1 4 6 4 1 15. Un estudiante afirma que dos términos son suficientes
1 5 10 10 5 1 para determinar cualquier sucesión. Por ejemplo, 3, 6, Á
1 6 15 20 15 6 1 significa que la sucesión es 3, 6, 9, 12, 15, Á . ¿Cómo le
1 7 21 35 35 21 7 1 respondes?
a. Se pide que cada persona del grupo halle cuatro dife- 16. Lisa afirma que al substraer el primer término del último
rentes patrones en el triángulo y luego los intercambie término y dividiendo entre la diferencia común, puede
con el resto del grupo. decir cuál es el número de términos en cualquier sucesión
b. Suma los números de cada renglón. Analiza el patrón aritmética finita. ¿Cómo le respondes?
presentado. Problemas de repaso
c. Usa la experiencia obtenida en el punto (b) para hallar
17. En una liga de beisbol formada por 10 equipos, cada
la suma en el renglón 16.
equipo juega dos veces con cada uno de los demás. ¿Cuán-
d. ¿Cuál es la suma de los números en el n-ésimo ren-
tos juegos se efectuarán?
glón?
18. ¿De cuántas maneras puedes cambiar una ficha de 40¢
9. Si el patrón mostrado en la figura continúa indefinida-
usando sólo fichas de 5, 10 y 25?
mente (lo cual quiere decir que continúa por siempre), la
19. Hay tiendas para 2, 3, 5, 6 ó 12 personas. ¿Qué combi-
figura resultante se llama triángulo de Sierpinski.
naciones de tiendas son posibles para que duerman 26
personas si sólo se usa una tienda de 12 personas?
Preguntas del Third International Mathematics and
Science Study (TIMSS) (Tercer Estudio Internacional
, , , sobre las Matemáticas y la Ciencia)
En grupo, contesten las preguntas siguientes. Analicen di- Los números de la sucesión 7, 11, 15, 19, 23, Á
ferentes estrategias para contar. crecen de cuatro en cuatro. Los números de la sucesión
a. ¿Cuántos triángulos negros habría en la quinta figura? 1, 10, 19, 28, 37, Á crecen de nueve en nueve. El nú-
b. ¿Cuántos triángulos blancos habría en la quinta figura? mero 19 está en ambas sucesiones. Si se continúan las
c. Si el patrón continúa hasta n figuras, ¿cuántos triángulos dos sucesiones, ¿cuál es el siguiente número que estará
negros habrá? en AMBAS sucesiones?
d. Si el patrón continúa hasta n figuras, ¿cuántos triángulos TIMSS 2003, Grado 8
blancos habrá?
Las tres figuras están divididas en pequeños triángulos
10. Crea una sucesión de números que siga un patrón. Mués-
congruentes.
trala a tus compañeras de clase. En caso de que no puedan
determinar la regla que sigue el patrón, explícala.
Preguntas del salón de clase
1 3
11. Pepe dijo que como 4, 24, 44 y 64 dejan residuo 0 al divi-
2 4
dirlos entre 4, entonces todos los números que terminan
en 4 deben dejar residuo 0 al dividirlos entre 4. ¿Cómo le 1 5 7
2 6 8
respondes?
12. Se pidió a Alicia y a Beti que extendieran la sucesión 2, 4, Figura 1 Figura 2 Figura 3
8, . Á Alicia dijo que su respuesta de 2, 4, 8, 16, 32,
64, Á era la correcta. Beti dijo que Alicia estaba equivo-
cada y que debería ser 2, 4, 8, 14, 22, 32, 44, . Á ¿Qué les
dices a las estudiantes?

© Lopez Mateos Editores. ISBN 978-607-95583-2-1, obra completa, versión electrónica, ISBN 978-607-95583-3-8, volumen 1, versión electrónica. Ejemplar asignado a: Helecto Villarroel gutierrez -
helecto@gmail.com. Fecha: 27 de octubre de 2014. Prohibida su modificación, copia o distribución.
42 Una introducción a la resolución de problemas

a. Completa la tabla que sigue. Primero, di cuántos b. Extendemos la sucesión hasta la séptima figura.
triángulos pequeños forman la figura 3. Después, ha- ¿Cuántos triángulos pequeños se necesitarían para la
lla el número de triángulos pequeños que se requeri- figura 7?
rían para la cuarta figura si se extendiera la sucesión c. Extendemos la sucesión hasta la figura 50. Explica
de las figuras. una manera de hallar el número de triángulos peque-
ños en la figura 50 sin dibujar ni contar el número de
Número de triángulos.
Figura triángulos pequeños
TIMSS, Grado 8
1 2
2 8
3
4

ROMPECABEZAS Halla el renglón que sigue en el patrón que aparece a continuación y explica tu
patrón:
1
1    1
2    1
1    2    1    1
1    1    1    2    2    1

*1-3 Razonamiento y lógica: una introducción

La lógica es una herramienta utilizada en el razonamiento matemático y para resolver


problemas. Es indispensable para razonar y, como se señala en la Nota de investigación,
◆ Nota de no se puede enseñar en una sola unidad sobre lógica. Sin embargo, en esta sección pre-
investigación sentamos un breve resumen de los elementos. En lógica, una proposición es una frase
que es verdadera o falsa, pero no ambas. Las siguientes expresiones no son proposiciones
No se puede enseñar
a razonar y demostrar pues no es posible determinar sus valores de verdad sin disponer de mayor información:
en una sola unidad
1. Ella tiene ojos azules. 5. ¿Cómo estás?
sobre lógica, por
ejemplo, o por medio 2. x + 7 = 18. 6. ¡Cuidado!
de “hacer demostra- 3. 2y + 7 7 1. 7. Pedro Infante fue el mejor cantante.
ciones”. Demostrar 4. 2 + 3
es una tarea muy difí-
cil para los estudian- Las expresiones (1), (2) y (3) pueden convertirse en proposiciones si para (1) identifica-
tes. Quizá a los mos quién es “ella” y para (2) y (3) asignamos valores a x y y, respectivamente. Sin embargo,
estudiantes del nivel una expresión que incluya a él o ella, o a x o y puede ser una proposición. Por ejemplo, “Si
bachillerato les pare- él mide más de 210 cm, entonces pasa de los 2 m de altura” y “21x + y2 = 2x + 2y” son
cen tan difíciles las proposiciones verdaderas sin importar quién sea él o qué valores numéricos tengan x y y.
demostraciones por-
que sólo tienen la ex- Negación y cuantificadores
periencia de escribir
demostraciones en la Dada una proposición, es posible crear una nueva formando su negación. La negación de
geometría de secun- una proposición es una proposición con valor de verdad opuesto al de la proposición dada. Si una
daria, de modo que proposición es verdadera su negación es falsa, y si una proposición es falsa su negación es
tienen una pers- verdadera. Considera la proposición “Está lloviendo”. La negación puede expresarse, sim-
pectiva limitada plemente, como “no está lloviendo”.
(Moore 1994). ◆

© Lopez Mateos Editores. ISBN 978-607-95583-2-1, obra completa, versión electrónica, ISBN 978-607-95583-3-8, volumen 1, versión electrónica. Ejemplar asignado a: Helecto Villarroel gutierrez -
helecto@gmail.com. Fecha: 27 de octubre de 2014. Prohibida su modificación, copia o distribución.
Sección 1-3 Razonamiento y lógica: una introducción 43

Ejemplo 1-7 Niega cada una de las siguientes proposiciones:











a. 2 + 3 = 5.
b. Un hexágono tiene 6 lados.

Solución a. 2 + 3 Z 5.
b. Un hexágono no tiene 6 lados.

Frases como “La camisa es azul” y “La camisa es verde” son proposiciones si las coloca-
mos en contexto. Sin embargo, no son negación una de la otra. Una proposición y su nega-
ción deben tener valores de verdad opuestos. Si sucede que la camisa es roja, entonces
ambas frases anteriores serán falsas y, por lo tanto, no pueden ser negaciones una de otra.
Sin embargo, las proposiciones “La camisa es azul” y “La camisa no es azul” son negaciones
una de otra pues tienen valores de verdad opuestos, sin importar de qué color sea la camisa.
Algunas proposiciones incluyen cuantificadores, y negarlas es más complicado. Los
cuantificadores incluyen palabras como todo, alguno, cada y existe.

• Los cuantificadores todo, cada y ningún se refieren a todos y cada uno de los elementos de
un conjunto, y se llaman cuantificadores universales.
• Los cuantificadores alguno y existe al menos uno se refieren a uno o más, quizás a todos, los
elementos de un conjunto, y se llaman cuantificadores existenciales.
• Todo, cada y para cada tienen el mismo significado matemático. De manera análoga, algún,
alguno y existe al menos uno tienen el mismo significado.
Considera la siguiente proposición, que incluye el cuantificador existencial algún y que
sabemos es verdadera: “Algunos profesores de la Universidad de Sonora miden más de
1.70 m”. Esto significa que al menos un profesor de la Universidad de Sonora mide más
de 1.70 m. No insinúa la posibilidad de que todos los profesores de la Universidad de So-
nora midan más de 1.70 m, o de que algunos profesores de la Universidad de Sonora no
midan más de 1.70 m. Como la negación de una proposición verdadera es falsa, ninguna
de las proposiciones “Algunos profesores de la Universidad de Sonora no miden más de
1.70 m” y “Todos los profesores de la Universidad de Sonora miden 1.70 m” es una nega-
ción de la proposición original. Una posible negación de la proposición original es “Nin-
gún profesor de la Universidad de Sonora mide más de 1.70 m”.
Para saber si una proposición es negación de otra, usamos argumentos similares al del pá-
rrafo anterior y averiguamos si tienen valores de verdad opuestos en todos los casos posibles.
A continuación presentamos algunas proposiciones cuantificadas, junto con sus negaciones:

Proposición Negación
Algunos a son b. Ningún a es b.
Algunos a no son b. Todos los a son b.
Todos los a son b. Algunos a no son b.
Ningún a es b. Algunos a son b.

Ejemplo 1-8 Niega cada una de las siguientes proposiciones sin importar su valor de verdad:








a. A todos los estudiantes les gustan los tacos.


b. A algunas personas les gustan las matemáticas.
c. Existe un número natural x tal que 3x = 6.
d. Para todos los números naturales, 3x = 3x.

© Lopez Mateos Editores. ISBN 978-607-95583-2-1, obra completa, versión electrónica, ISBN 978-607-95583-3-8, volumen 1, versión electrónica. Ejemplar asignado a: Helecto Villarroel gutierrez -
helecto@gmail.com. Fecha: 27 de octubre de 2014. Prohibida su modificación, copia o distribución.
44 Una introducción a la resolución de problemas

Solución a. A algunos estudiantes no les gustan los tacos.


b. A ninguna persona le gustan las matemáticas.
c. Para todos los números naturales x, 3x Z 6.
d. Existe un número natural x tal que 3x Z 3x.

▲ ▲
Tablas de verdad y proposiciones compuestas
Para determinar la veracidad de una proposición, considera el siguiente acertijo propuesto
por uno de los más importantes escritores actuales de acertijos lógicos, Raymond Smull-
yan. Ha escrito varios libros sobre lógica, incluido ¿La dama o el tigre? El título está to-
mado de un cuento de Frank Stockton acerca de un prisionero que debe escoger entre
dos puertas: detrás de una está un tigre hambriento y detrás de la otra está una bella dama.
La propuesta de Smullyan es que cada puerta tiene un letrero y el prisionero sabe que
sólo un letrero dice la verdad. En la Puerta 1 se lee:

EN ESTA HABITACIÓN HAY UNA DAMA Y


EN LA OTRA HABITACIÓN HAY UN TIGRE.

En la Puerta 2 se lee:

EN UNA DE ESTAS HABITACIONES HAY UNA DAMA Y


EN UNA DE ESTAS HABITACIONES HAY UN TIGRE.

Con esta información la persona puede escoger la puerta correcta. Analiza este problema y
trata de hallar una solución antes de seguir leyendo.

Solución Si el letrero de la Puerta 1 es verdadero, entonces el letrero de la Puerta 2 debe ser


verdadero. Como esto no puede suceder, el letrero de la Puerta 2 debe ser el verdadero, lo cual hace
que el letrero de la Puerta 1 sea falso. Como el letrero de la Puerta 1 es falso, la dama no puede
estar en la Habitación 1, y debe estar en la Habitación 2.
Hay un sistema simbólico definido para ayudar en el estudio de la lógica. Si p representa
una proposición, la negación de la proposición p se denota con ¬ p que se lee “no p”. Las
tablas de verdad se usan para mostrar todos los patrones posibles de verdad o falsedad de
las proposiciones. En la tabla 1-13 presentamos la tabla de verdad para p y ¬ p.
Tabla 1-13 Dadas dos proposiciones, podemos crear una nueva proposición compuesta usando un
conectivo como y. Se puede formar una proposición compuesta combinando dos o más
Proposición Negación
proposiciones. Por ejemplo, “Está nevando” y “la pista de esquiar está abierta” junto con
p ¬p
y dan “Está nevando y la pista de esquiar está abierta”. Otra proposición compuesta se
V F obtiene usando el conectivo o. Por ejemplo, “Está nevando o la pista de esquiar está
F V abierta”. Se usan los símbolos ¿ y ¡ para representar los conectivos y y o, respectiva-
mente. Por ejemplo, si p representa “Está nevando” y q representa “la pista de esquiar está
abierta”, entonces “Está nevando y la pista de esquiar está abierta” se denota con p ¿ q. De
manera análoga, “Está nevando o la pista de esquiar está abierta” se denota con p ¡ q.
El valor de verdad de cualquier proposición compuesta, como p ¿ q, se define a partir del
valor de verdad de cada proposición simple. Como cada una de las proposiciones p y q
puede ser verdadera o falsa, hay cuatro posibilidades para el valor de verdad de p ¿ q, como

© Lopez Mateos Editores. ISBN 978-607-95583-2-1, obra completa, versión electrónica, ISBN 978-607-95583-3-8, volumen 1, versión electrónica. Ejemplar asignado a: Helecto Villarroel gutierrez -
helecto@gmail.com. Fecha: 27 de octubre de 2014. Prohibida su modificación, copia o distribución.
Sección 1-3 Razonamiento y lógica: una introducción 45

se muestra en la tabla 1-14. La proposición compuesta es la conjunción de p y q y, por


definición, es verdadera si, y sólo si, p y q son verdaderas. De no suceder así, es falsa.

Tabla 1-14    Tabla 1-15   


Conjunción Disyunción
p q p ¿ q p q p ¡ q

V V V V V V
V F F V F V
F V F F V V
F F F F F F

La proposición compuesta p ¡ q —esto es, p o q — es una disyunción. En lenguaje


cotidiano, no siempre se interpreta o de la misma manera. En lógica usamos un o in-
clusivo. La proposición “Iré al cine o leeré un libro” significa que voy a ir al cine o voy
a leer un libro, o voy a hacer ambas cosas. Por lo tanto, en lógica p o q, simbolizado
por p ¡ q, es, por definición, falsa si tanto p como q son falsas, y es verdadera en los
demás casos. Resumimos esto en la tabla 1-15.

Ejemplo 1-9 Clasifica cada una de las siguientes proposiciones como verdadera o falsa.








p: 2 + 3 = 5 q: 2 # 3 = 6 r: 5 + 3 = 9
a. p ¿ q c. ¬ p ¡ r e. ¬ 1p ¿ q2
b. q ¡ r d. ¬ p ¿ ¬ q f. 1p ¿ q) ¡ ¬ r
Solución a. p es verdadera y q es verdadera, de modo que p ¿ q es verdadera.
b. q es verdadera y r es falsa, de modo que q ¡ r es verdadera.
c. ¬ p es falsa y r es falsa, de modo que ¬ p ¡ r es falsa.
d. ¬ p es falsa y ¬ q es falsa, de modo que ¬ p ¿ ¬ q es falsa.
e. p ¿ q es verdadera, de modo que ¬ 1p ¿ q2 es falsa.
f. p ¿ q es verdadera y ¬ r es verdadera, de modo que 1p ¿ q) ¡ ¬ r es verdadera.

Las tablas de verdad se usan no sólo para resumir los valores de verdad de proposiciones
compuestas; también se usan para determinar si dos proposiciones son lógicamente equiva-
lentes. Dos proposiciones son lógicamente equivalentes si, y sólo si, tienen los mismos
valores de verdad. Si p y q son lógicamente equivalentes, lo escribimos p K q.

◆ Nota George Boole (1815–1864) nació en Lincoln, Inglaterra, y fue llamado “el padre de la
histórica lógica”. A la edad de 15 años comenzó su carrera de maestro. En 1849 fue nombrado pro-
fesor en Queens College, en Cork, Irlanda. En su trabajo empleó símbolos para repre-
sentar conceptos y desarrolló un sistema de manipulaciones algebraicas que acompañaban
a los símbolos. Su trabajo fue la unión de la lógica y las matemáticas. Muchas de las ideas
de Boole, como el álgebra booleana, tienen aplicaciones en ciencias de la computación y
en el diseño de aparatos conmutadores para telefonía. ◆

© Lopez Mateos Editores. ISBN 978-607-95583-2-1, obra completa, versión electrónica, ISBN 978-607-95583-3-8, volumen 1, versión electrónica. Ejemplar asignado a: Helecto Villarroel gutierrez -
helecto@gmail.com. Fecha: 27 de octubre de 2014. Prohibida su modificación, copia o distribución.
46 Una introducción a la resolución de problemas

Ejemplo 1-10 Muestra que ¬ 1p ¿ q) K ¬ p ¡ ¬ q.











Solución Dos proposiciones son lógicamente equivalentes si tienen los mismos valores de
verdad. En las tablas 1-16 y 1-17 damos las tablas de verdad de estas proposiciones.

Tabla 1-16 Tabla 1-17

p q p ¿ q ¬ 1p ¿ q2 p q ¬p ¬q ¬ p ¡ ¬q

V V V F V V F F F
V F F V V F F V V
F V F V F V V F V
F F F V F F V V V

Como las dos proposiciones tienen los mismos valores de verdad, vemos que ¬ 1p ¿ q) K
¬ p ¡ ¬ q.

AHORA INTENTA ÉSTE 1-16  En el ejemplo 1-10 se muestra que ¬ 1p ¿ q) K ¬ p ¡ ¬ q. De la misma


manera podemos demostrar que ¬ 1p ¡ q) K ¬ p ¿ ¬ q. Estas equivalencias se llaman Leyes de De Mor-
gan. Confirma que se cumple la segunda ley de De Morgan empleando tablas de verdad.

Condicionales y bicondicionales
Las proposiciones expresadas en la forma “si p, entonces q” se llaman condicionales, o
implicaciones, y se denotan con p : q. También se pueden leer como “p implica q”. La
parte “si” de una condicional es la hipótesis de la implicación y la parte “entonces” es la
conclusión. Muchas proposiciones se pueden enunciar en la forma “si-entonces”. Demos
un ejemplo:
Proposición: Todos los triángulos equiláteros tienen ángulos agudos.
Forma “si-entonces”: Si un triángulo es equilátero, entonces tiene ángulos agudos.
Hipótesis Conclusión
También se puede pensar en una implicación como una promesa. Supón que Beti hace la
promesa, “Si logro un aumento, entonces te invitaré a cenar”. Si Beti mantiene su pro-
mesa, la implicación es verdadera; si Beti rompe su promesa, la implicación es falsa. Con-
sidera las siguientes cuatro posibilidades:

p q
(1) V V Beti obtiene el aumento; te invita a cenar.
(2) V F Beti obtiene el aumento; no te invita a cenar.
(3) F V Beti no obtiene el aumento; te invita a cenar.
(4) F F Beti no obtiene el aumento; no te invita a cenar.

El único caso en el que Beti rompe su promesa es cuando obtiene su aumento y no te


invita a cenar, el caso (2). Si ella no obtiene el aumento, puede o no invitarte a cenar sin
romper su promesa. La definición de la implicación está resumida en la tabla 1-18.

© Lopez Mateos Editores. ISBN 978-607-95583-2-1, obra completa, versión electrónica, ISBN 978-607-95583-3-8, volumen 1, versión electrónica. Ejemplar asignado a: Helecto Villarroel gutierrez -
helecto@gmail.com. Fecha: 27 de octubre de 2014. Prohibida su modificación, copia o distribución.
Sección 1-3 Razonamiento y lógica: una introducción 47

Tabla 1-18 Observa que el único caso en que la implicación es falsa es cuando p es verdadera y q es
falsa.
Implicación
q p:q Una implicación se puede expresar de distintas maneras en palabras:
p

V V V 1. Si hay sol, entonces la piscina abre. (Si p, entonces q.)


V F F 2. Si hay sol, la piscina abre. (Si p, q.)
F V V 3. La piscina abre si hay sol. (q si p.)
F F V 4. Que haya sol implica que la piscina abra. ( p implica q.)
5. Hay sol sólo si la piscina abre. ( p sólo si q.)
6. Que haya sol es condición suficiente para que abra la piscina.
( p es condición suficiente para q.)
7. Que la piscina abra es condición necesaria para que haya sol.
(q es una condición necesaria para p.)

Cualquier implicación p : q tiene tres proposiciones relacionadas:

Proposición: Si p, entonces q. p:q


Recíproca: Si q, entonces p. q:p
Inversa: Si no p, entonces no q. ¬p : ¬q
Contrapositiva: Si no q, entonces no p. ¬q : ¬p

Ejemplo 1-11 Escribe la recíproca, la inversa y la contrapositiva de la siguiente proposición:










Si estoy en Pachuca, entonces estoy en Hidalgo.

Solución Recíproca: Si estoy en Hidalgo, entonces estoy en Pachuca.


Inversa: Si no estoy en Pachuca, entonces no estoy en Hidalgo.
Contrapositiva: Si no estoy en Hidalgo, entonces no estoy en Pachuca.

El ejemplo 1-11 se puede usar para mostrar que si una proposición es verdadera, su recí-
proca y su inversa no necesariamente son verdaderas. Sin embargo, la contrapositiva es ver-
dadera. Verifiquemos estas observaciones en la siguiente proposición: Si un número es
natural, el número no es 0. El conjunto de los números naturales es N = {1, 2, 3, 4, 5, 6, Á }.
Verifiquemos la veracidad de la recíproca, la inversa y la contrapositiva.

Inversa: Si un número no es natural, entonces es 0. Esto es falso pues - 6 no es natural pero


tampoco es 0.
Recíproca: Si un número no es 0, entonces es natural. Esto es falso pues -6 no es 0 pero tam-
poco es un número natural.
Contrapositiva: Si un número es 0, entonces no es un número natural. Esto es cierto pues
N = {1, 2, 3, 4, 5, 6, Á }.

La contrapositiva de la última proposición es la proposición original. Por lo tanto, lo an-


terior sugiere que si p : q es verdadera, su contrapositiva ¬ q : ¬ p también es verdadera;
y si la contrapositiva es verdadera, la proposición original debe ser verdadera. Se sigue de
aquí que una proposición y su contrapositiva no pueden tener valores de verdad opuestos.
Resumimos lo anterior en el siguiente teorema.

© Lopez Mateos Editores. ISBN 978-607-95583-2-1, obra completa, versión electrónica, ISBN 978-607-95583-3-8, volumen 1, versión electrónica. Ejemplar asignado a: Helecto Villarroel gutierrez -
helecto@gmail.com. Fecha: 27 de octubre de 2014. Prohibida su modificación, copia o distribución.
48 Una introducción a la resolución de problemas

Teorema 1–1: Equivalencia de una proposición y su contrapositiva


La implicación p : q y su contrapositiva ¬ q : ¬ p son lógicamente equivalentes.

Ejemplo 1-12 Usa tablas de verdad para mostrar que p : q K ¬ q : ¬ p.










Solución Las tablas de verdad para estas proposiciones están dadas en las tablas 1-19 y 1-20.

Tabla 1-19 Tabla 1-20


p q p:q p q ¬q ¬p ¬q : ¬p

V V V V V F F V
V F F V F V F F
F V V F V F V V
F F V F F V V V

Como estas dos proposiciones tienen los mismos valores de verdad, vemos que
p : q K ¬ q : ¬ p.

AHORA INTENTA ÉSTE 1-17 Anteriormente usamos la implicación “Si Beti obtiene un aumento (p),
entonces te invita a cenar (q)” como motivación para construir la tabla de verdad de p : q. Dijimos que el
único caso en que Beti rompe su promesa es cuando obtiene el aumento y no te invita a cenar, esto es,
p ¿ ¬ q. Por lo tanto, p ¿ ¬ q es un buen candidato para la negación de p : q. Usa tablas de verdad y
verifica que ¬ 1p : q) K p ¿ ¬ q para averiguar si p ¿ ¬ q es la negación de p : q.

Al conectar una proposición y su recíproca mediante el conectivo y obtenemos


1 p : q2 ¿ 1q : p2. Esta proposición compuesta se puede escribir como p 4 q y usual-
mente se lee “p si, y sólo si, q”. La proposición “p si, y sólo si, q” es una bicondicional.

AHORA INTENTA ÉSTE 1-18  Construye una tabla de verdad para determinar cuándo una proposición
bicondicional es verdadera.

Razonamiento válido
Al resolver problemas decimos que un razonamiento es válido si la conclusión se sigue de
manera inevitable partiendo de hipótesis verdaderas. Así, en todos los argumentos presen-
tados en esta sección suponemos que las hipótesis son verdaderas. Consideremos los ejem-
plos siguientes:
Hipótesis: Todos los perros son animales.
Tintín es un perro.
Conclusión: Por lo tanto, Tintín es un animal.

© Lopez Mateos Editores. ISBN 978-607-95583-2-1, obra completa, versión electrónica, ISBN 978-607-95583-3-8, volumen 1, versión electrónica. Ejemplar asignado a: Helecto Villarroel gutierrez -
helecto@gmail.com. Fecha: 27 de octubre de 2014. Prohibida su modificación, copia o distribución.
Sección 1-3 Razonamiento y lógica: una introducción 49

La proposición “Todos los perros son animales” se puede ilustrar con el diagrama de Euler
de la figura 1-20(a).

Animales Animales

Perros
Perros Tintín

(a) (b)

Figura 1-20

La información “Tintín es un perro” implica que Tintín debe pertenecer al círculo que
contiene los perros, como se ilustra en la figura 1-20(b). Tintín también debe pertenecer
al círculo que contiene los animales. Así, el razonamiento es válido porque es imposible
trazar una figura que satisfaga la hipótesis y contradiga la conclusión.
Considera el siguiente argumento.
Hipótesis: Todos los maestros de educación básica saben matemáticas.
Algunas personas que saben matemáticas no son niños.
Conclusión: Por lo tanto, ningún maestro de educación básica es niño.
Sea B el conjunto de los maestros de educación básica, M el conjunto de las personas que
saben matemáticas y N el conjunto de los niños. Entonces, la proposición “Todos los maes-
tros de educación básica saben matemáticas” se puede ilustrar como en la figura 1-21(a). La
proposición “Algunas personas que saben matemáticas no son niños” se puede ilustrar de
varias maneras; listamos tres de ellas en la figura 1-21, de la (b) a la (d). De acuerdo con la
figura 1-21(d), es posible que algunos maestros de educación básica sean niños y, así, se sa-
tisfacen las proposiciones dadas. Por lo tanto, la conclusión de que “Ningún maestro de
educación básica es niño” no se sigue de las hipótesis dadas. En consecuencia, el razona-
miento no es válido.

M M M M

B B B B N
N N

(a) (b) (c) (d)

Figura 1-21

Basta con que se pueda trazar una figura que satisfaga las hipótesis de un argumento y
contradiga la conclusión, para que el argumento no sea válido. Sin embargo, para mostrar
que un argumento es válido, todas las figuras posibles deben mostrar que no hay contradic-
ciones. No debe haber manera de satisfacer las hipótesis y contradecir la conclusión si el ar-
gumento es válido.

© Lopez Mateos Editores. ISBN 978-607-95583-2-1, obra completa, versión electrónica, ISBN 978-607-95583-3-8, volumen 1, versión electrónica. Ejemplar asignado a: Helecto Villarroel gutierrez -
helecto@gmail.com. Fecha: 27 de octubre de 2014. Prohibida su modificación, copia o distribución.
50 Una introducción a la resolución de problemas

Ejemplo 1-13 Determinar si es válido el argumento siguiente:











Hipótesis: En el DF todos los gestores usan lentes obscuros.
Nadie en el DF que mida más de 1.80 m usa lentes obscuros.
Conclusión: Las personas que miden más de 1.80 m no son gestores en el DF.

Solución Si G representa a los gestores en el DF y L a las personas que usan lentes


obscuros, la primera hipótesis se ilustra en la figura 1-22(a). Si D representa a las personas
en el DF que miden más de 1.80 m, la segunda hipótesis se ilustra en la figura 1-22(b).
Como las personas que miden más de 1.80 m están fuera del círculo que representa a los
que usan lentes obscuros y los gestores están en el círculo L, la conclusión es válida y
ninguna persona que mida más de 1.80 m es gestor en el DF.

L L

G G D

(a) (b)

Figura 1-22

Un método diferente para determinar si un argumento es válido usa el razonamiento


directo y una forma de argumento llamada ley de encadenamiento hacia adelante (o
modus ponens). Por ejemplo, supongamos que las siguientes proposiciones son verdaderas:
Si sale el sol, entonces saldremos de viaje.
Salió el sol.
Usando estas dos proposiciones, podemos concluir que saldremos de viaje. En general, la
ley de encadenamiento hacia adelante (o modus ponens) se expresa como:

Si la proposición “si p, entonces q” es verdadera, y si p es verdadera, entonces q debe ser verdadera.

Ejemplo 1-14 Determina si el siguiente argumento es válido si las hipótesis son verdaderas y x es un
número natural:








Hipótesis: Si x 7 2, entonces x2 7 4.
x 7 2.
Conclusión: Por lo tanto, x 2 7 4.

Solución Usando el razonamiento directo, modus ponens, vemos que la conclusión es


válida.

Ejemplo 1-15 Muestra que 3( p : q) ¿ p4 : q siempre es verdadera.










© Lopez Mateos Editores. ISBN 978-607-95583-2-1, obra completa, versión electrónica, ISBN 978-607-95583-3-8, volumen 1, versión electrónica. Ejemplar asignado a: Helecto Villarroel gutierrez -
helecto@gmail.com. Fecha: 27 de octubre de 2014. Prohibida su modificación, copia o distribución.
Sección 1-3 Razonamiento y lógica: una introducción 51

Solución    En la tabla 1-21 damos una tabla de verdad para estas implicaciones.

Tabla 1-21

p q p:q ( p : q) ¿ p [( p : q) ¿ p ] : q

V V V V V
V F F F V
F V V F V
F F V F V


O B S E R VA C I Ó N La proposición 3( p : q) ¿ p4 : q es una tautología; es decir, una
proposición que siempre es verdadera.

Un tipo diferente de razonamiento, el razonamiento indirecto, usa otra forma de argu-


mentación llamada modus tollens. Por ejemplo, considera las siguientes proposiciones ver-
daderas:

Si una figura es un cuadrado, entonces es un rectángulo.


La figura no es un rectángulo.

La conclusión es que la figura no puede ser un cuadrado. El modus tollens se puede interpretar
como sigue:

Si tenemos una condicional aceptada como verdadera y sabemos que la conclusión es falsa, en-
tonces la hipótesis debe ser falsa.

Ejemplo 1-16 Obtén conclusiones de cada uno de los siguientes pares de proposiciones verdaderas:








a. Si una persona vive en Jalapa, entonces la persona vive en Veracruz. Juana no vive en
Veracruz.
b. Si x = 3, entonces 2x Z 7. Sabemos que 2x = 7.

Solución a. Juana no vive en Jalapa (modus tollens).


b. x Z 3 (modus tollens).

La argumentación final para razonar que vamos a ver aquí incluye la regla de la cadena
(transitividad). Considera las siguientes proposiciones:

Si ahorro, me retiraré pronto.


Si me retiro pronto, jugaré golf.

¿Cuál es la conclusión? La conclusión es que si ahorro, jugaré golf. En general, la regla de


la cadena se puede expresar como sigue:

Si “si p, entonces q” y “si q, entonces r” son verdaderas, entonces “si p, entonces r” es verdadera.

Muchas personas suelen llegar a conclusiones inválidas basadas en anuncios u otra infor-
mación. Considera, por ejemplo, la proposición válida “Las personas sanas comen el cereal
Súper”. ¿Son válidas las conclusiones siguientes?

© Lopez Mateos Editores. ISBN 978-607-95583-2-1, obra completa, versión electrónica, ISBN 978-607-95583-3-8, volumen 1, versión electrónica. Ejemplar asignado a: Helecto Villarroel gutierrez -
helecto@gmail.com. Fecha: 27 de octubre de 2014. Prohibida su modificación, copia o distribución.
52 Una introducción a la resolución de problemas

Si una persona come cereal Súper, entonces la persona es sana.


Si la persona no es sana, la persona no come cereal Súper.
Si denotamos con p : q la proposición original, donde p es “una persona es sana” y q es
“una persona come cereal Súper”, entonces la primera conclusión es el recíproco de p : q,
es decir, q : p, y la segunda conclusión es la inversa de p : q, esto es, ¬ p : ¬ q. Por lo
tanto, ninguna es válida.

Ejemplo 1-17 Determina conclusiones para las siguientes proposiciones verdaderas:










a. Si un triángulo es equilátero, entonces es isósceles. Si un triángulo es isósceles, tiene al


menos dos lados congruentes.
b. Si un número es un número completo, entonces el número es un entero. Si un número es
entero, entonces el número es un número racional. Si un número es un número racional,
entonces el número es un número real.

Solución a. Si un triángulo es equilátero, entonces tiene al menos dos lados congruentes.


b. Si un número es un número completo, entonces es un número real.

Evaluación 1-3A

1. Determina cuáles de las siguientes son proposiciones y 6. Completa cada una de las siguientes tablas de verdad:
clasifica cada una como verdadera o falsa: a. p ¬ p ¬ ( ¬ p)
a. 2 + 4 = 8.
b. Jalapa es un estado. V
c. ¿Qué hora es? F
d. 3 # 2 = 6.
e. Esta proposición es falsa.
2. Usa cuantificadores para hacer verdadera cada una de las b. p ¬p p ¡ ¬p p ¿ ¬p
siguientes proposiciones, donde x es un número natural:
a. x + 8 = 11. b. x 2 = 4. V
c. x + 3 = 3 + x. d. 5x + 4x = 9x. F
3. Usa cuantificadores para hacer que cada ecuación del
problema 2 sea falsa.
c. Con base en la parte (a), ¿es p lógicamente equiva-
lente a ¬ 1 ¬ p2?
4. Escribe la negación de cada una de las siguientes propo-
d. Con base en la parte (b), ¿es p ¡ ¬ p lógicamente
siciones:
equivalente a p ¿ ¬ p?
a. Este libro tiene 500 páginas.
b. 3 # 5 = 15.
7. Si q significa “El curso es fácil” y r significa “Los flojos no
c. Todos los perros tienen cuatro patas.
estudian”, escribe cada expresión en forma simbólica:
d. Algunos rectángulos son cuadrados.
a. El curso es fácil y los flojos no estudian.
e. No todos los rectángulos son cuadrados.
b. Los flojos no estudian o el curso no es fácil.
f. Ningún perro tiene pulgas.
c. Es falso tanto que el curso sea fácil como que los flo-
5. En cada caso, di si es verdadero o falso:
jos no estudian.
a. Para algunos números naturales x, x 6 6 y x 7 3.
d. El curso no es fácil.
b. Para algunos números naturales x, x 7 0 o x 6 5.

© Lopez Mateos Editores. ISBN 978-607-95583-2-1, obra completa, versión electrónica, ISBN 978-607-95583-3-8, volumen 1, versión electrónica. Ejemplar asignado a: Helecto Villarroel gutierrez -
helecto@gmail.com. Fecha: 27 de octubre de 2014. Prohibida su modificación, copia o distribución.
Sección 1-3 Razonamiento y lógica: una introducción 53

8. Si p es falsa y q es verdadera, halla los valores de verdad a. Si todo dígito de un número no es 6, entonces el nú-
para lo siguiente: mero no es divisible entre 3.
a. p ¿ q b. ¬ p b. Si un número no es divisible entre 3, entonces algún
c. ¬ 1 ¬ p2 d. p ¿ ¬ q dígito del número no es 6.
e. ¬ 1 ¬ p ¿ q2 c. Si un número es divisible entre 3, entonces todo dí-
9. Halla el valor de verdad de cada proposición del pro- gito del número es 6.
blema 8 si p es falsa y q es falsa. 15. Escribe una proposición lógicamente equivalente a la
10. Di qué par de proposiciones son lógicamente equiva- proposición “Si un número es múltiplo de 8, entonces es
lentes. múltiplo de 4”.
a. ¬ 1p ¡ q2 y ¬ p ¡ ¬ q 16. Investiga la validez de cada una de las siguientes argu-
b. ¬ 1p ¿ q2 y ¬ p ¿ ¬ q mentaciones:
11. Completa la siguiente tabla de verdad: a. Todos los cuadrados son cuadriláteros.
Todos los cuadriláteros son polígonos.
p q ¬p ¬p ¿ q Por lo tanto, todos los cuadrados son polígonos.
b. Todas las maestras son inteligentes.
V V
Algunas maestras son ricas.
V F Por lo tanto, alguna persona inteligente es rica.
F V c. Si un estudiante está en primer año, entonces cursa
F F matemáticas.
Juana está en segundo año.
12. Escribe lo que sigue en forma simbólica si p es la proposi- Por lo tanto, Juana no cursa matemáticas.
ción “Está lloviendo” y q es la proposición “La hierba está 17. Forma una conclusión que se siga lógicamente de las
húmeda”. proposiciones dadas:
a. Si está lloviendo, entonces la hierba está húmeda. a. A algunas estudiantes de primero les gustan las mate-
b. Si no está lloviendo, entonces la hierba está húmeda. máticas.
c. Si está lloviendo, entonces la hierba no está húmeda. Todas las personas a las que les gustan las matemáticas
d. La hierba está húmeda si está lloviendo. son inteligentes.
e. Que la hierba no esté húmeda implica que no está llo- b. Si estudio para el examen final, entonces lo pasaré.
viendo. Si paso el examen final, entonces pasaré el curso.
f. La hierba está húmeda si, y sólo si, está lloviendo. Si paso el curso, entonces buscaré trabajo como maestra.
13. Para cada una de las implicaciones siguientes escribe la c. Todo triángulo equilátero es isósceles.
recíproca, la inversa y la contrapositiva: Existen triángulos que son equiláteros.
a. Si x = 5, entonces 2x = 10. 18. Escribe en forma si-entonces:
b. Si no te gusta este libro, entonces no te gustan las ma- a. Toda figura que es un cuadrado es un rectángulo.
temáticas. b. Todos los enteros son números racionales.
c. Si no usas la pasta dental Ultra, entonces tienes caries. c. Los polígonos con exactamente 3 lados son triángulos.
d. Si eres bueno en lógica, entonces tus calificaciones 19. Usa las leyes de DeMorgan del Ahora intenta éste 1-16
son altas. para escribir una negación de cada caso:
14. Considera la proposición “Si todo dígito de un número es a. 3 # 2 = 6 y 1 + 1 Z 3.
6, entonces el número es divisible entre 3”. ¿Cuáles expre- b. Me puedes pagar ahora o me puedes pagar después.
siones son lógicamente equivalentes a la proposición?

Evaluación 1-3B

1. Determina cuáles de las siguientes son proposiciones y 2. Usa cuantificadores para hacer verdadera cada una de las
clasifica cada proposición como verdadera o falsa: siguientes proposiciones, donde x es un número natural:
a. Cierra la ventana. a. x + 0 = x
b. Él está en la ciudad. b. x + 1 = x + 2
c. 2 # 2 = 2 + 2. c. 31x + 22 = 12
d. 2 + 3 = 8. d. x 3 = 8
e. ¡Quieto!

© Lopez Mateos Editores. ISBN 978-607-95583-2-1, obra completa, versión electrónica, ISBN 978-607-95583-3-8, volumen 1, versión electrónica. Ejemplar asignado a: Helecto Villarroel gutierrez -
helecto@gmail.com. Fecha: 27 de octubre de 2014. Prohibida su modificación, copia o distribución.
54 Una introducción a la resolución de problemas

3. Usa cuantificadores para hacer que cada ecuación del 13. Escribe lo siguiente en forma simbólica si p es la propo-
problema 2 sea falsa. sición “Tú lo construiste” y q es la proposición “Ellos
4. Escribe la negación de cada una de las siguientes propo- vendrán”:
siciones: a. Si tú lo construiste, ellos vendrán.
a. Seis es menor que 8. b. Si tú no lo construiste, entonces ellos vendrán.
b. Algunos gatos no tienen nueve vidas. c. Si tú lo construiste, ellos no vendrán.
c. Todos los cuadrados son rectángulos. d. Ellos vendrán si tú lo construiste.
d. No todos los números son positivos. e. Si tú no lo construiste, entonces ellos no vendrán.
e. Algunas personas tienen el cabello castaño. f. Si ellos no vienen, entonces tú no lo construiste.
5. En cada caso, di si es verdadero o falso: 14. Para cada una de las implicaciones siguientes escribe la
a. Para algunos números naturales x, x 7 5 y x 7 2. recíproca, la inversa y la contrapositiva:
b. Para algunos números naturales x, x 7 5 ó x 6 5. a. Si x = 3, entonces x2 = 9.
6. Si sabes que p es verdadera, ¿qué puedes concluir acerca b. Si nieva, entonces se suspenden las clases.
del valor de verdad de p ¡ q, aunque no conozcas el va- 15. Iris realiza la proposición verdadera: “Si llueve, entonces
lor de verdad de q? voy al cine”. ¿Se sigue lógicamente que si no llueve, en-
7. Si sabes que p es falsa, ¿qué puedes concluir acerca del tonces Iris no va al cine?
valor de verdad de p : q, aun si no conoces el valor de 16. Investiga la validez de cada una de las siguientes argu-
verdad de q? mentaciones:
8. Si q es “Tú dijiste adiós” y r es “Yo dije hola”, escribe lo a. Todas las mujeres son mortales.
siguiente en forma simbólica: Hipatia era mujer.
a. Tú dijiste adiós y yo dije hola. Por lo tanto, Hipatia era mortal.
b. Tú dijiste adiós y yo no dije hola. b. Todos los días lluviosos están nublados.
c. Yo no dije hola o tú no dijiste adiós. Hoy no está nublado.
d. Es falso que tú dijiste adiós y yo dije hola. Por lo tanto, hoy no está lluvioso.
9. Si p es falsa y q es verdadera, halla los valores de verdad c. A algunos estudiantes les gusta esquiar.
para lo siguiente: Alicia es una estudiante.
a. p ¡ q b. ¬ q Por lo tanto, a Alicia le gusta esquiar.
c. ¬ p ¡ q d. ¬ 1p ¡ q2 17. Forma una conclusión que se siga lógicamente de las
e. ¬ q ¿ ¬ p proposiciones dadas:
10. Halla el valor de verdad de cada proposición del pro- a. Todos los estudiantes de bachillerato son pobres.
blema 9 si p es falsa y q es falsa. Elena es estudiante de bachillerato.
11. Di qué par de proposiciones son lógicamente equivalentes. b. Todos los ingenieros necesitan matemáticas.
a. ¬ 1p ¡ q2 y ¬ p ¿ ¬ q Daniel no necesita matemáticas.
b. ¬ 1p ¿ q2 y ¬ p ¡ ¬ q c. Todas las bicicletas tienen ruedas.
12. Completa la siguiente tabla de verdad: Todas las ruedas usan caucho.
18. Escribe en forma si-entonces:
a. Todos los números naturales son números reales.
p q ¬q p ¡ ¬q b. Cada círculo es una figura cerrada.
19. Usa las leyes de De Morgan de Ahora intenta éste 1-16
V V para escribir una negación de lo siguiente:
V F a. 3 + 5 Z 9 y 3 # 5 = 15.
F V b. Yo voy o ella va.
F F

Conexiones matemáticas 1-3

Comunicación 3. Describe lo que significa una proposición compuesta.


1. Explica por qué las órdenes, preguntas y opiniones no 4. a. Describe en qué condiciones una disyunción es verda-
son proposiciones. dera.
b. Describe en qué condiciones una implicación es ver-
2. Explica cómo escribir la negación de una proposición cuan-
dadera.
tificada de la forma “Algunos A son B”. Da un ejemplo. 5. ¿Qué significa el uso del o “inclusivo”?

© Lopez Mateos Editores. ISBN 978-607-95583-2-1, obra completa, versión electrónica, ISBN 978-607-95583-3-8, volumen 1, versión electrónica. Ejemplar asignado a: Helecto Villarroel gutierrez -
helecto@gmail.com. Fecha: 27 de octubre de 2014. Prohibida su modificación, copia o distribución.
Sección 1-3 Razonamiento y lógica: una introducción 55

6. Explica cómo determinar si dos proposiciones son lógi- Conéctate a Internet y explora los conectivos AND, OR
camente equivalentes. y NOT. Explica tus resultados.
7. Describe lo más posible al Dr. No.
Solución abierta
10. Da dos ejemplos de matemáticas para lo siguiente:
Espías a. Una proposición y su recíproca son verdaderas.
b. Una proposición es verdadera, pero su recíproca es
Mujeres falsa.
espías c. Una proposición verdadera del tipo “si y sólo si”.
Pobres d. Una proposición falsa del tipo “si y sólo si”.
Aprendizaje colectivo
Altos
11. Cada uno de los miembros de un grupo realiza cinco
proposiciones similares a las del ejemplo 1-13 pero
acerca de objetos matemáticos, cada una con conclusio-
8. Considera el poema:
nes válidas o inválidas. Las proposiciones deben ser lo
Por buscar un clavo se perdió el zapato. más variadas posible. Cada integrante del grupo inter-
Por buscar un zapato se perdió el caballo. cambia sus proposiciones con otra persona —sin revelar
Por buscar un caballo se perdió el jinete. cuáles son válidas y cuáles no— y determina cuáles de las
Por buscar un jinete se perdió la batalla. proposiciones recibidas de la otra persona son válidas y
Por buscar la batalla se perdió la guerra. cuáles no. Los dos miembros del grupo comparan sus
Por lo tanto, por buscar un clavo se perdió la guerra. respuestas y discuten sus discrepancias.
12. Discute la paradoja que surge de lo siguiente:
a. Escribe cada renglón en forma de una proposición si-
a. Este libro tiene 1000 páginas.
entonces.
b. El autor de este libro de texto es Dante.
b. ¿Se sigue lógicamente la conclusión? ¿Por qué?
c. Las proposiciones (a), (b) y (c) son falsas.
9. Hoy día muchos estudiantes usan buscadores en Internet,
como Yahoo o Google. Para usar de manera eficiente di- Preguntas del salón de clase
chos buscadores hay que saber algo de los conectivos 13. Una estudiante dice que no entiende la diferencia entre
AND (Y), OR (O) y NOT (NO). Un tipo común de bús- ¬ ( p ¿ q) y ¬ p ¿ q. ¿Cómo se lo explicas?
queda avanzada se llama búsqueda booleana (ver la Nota 14. Una estudiante dice que no ve cómo una proposición
histórica). Con una búsqueda booleana incrementas la compuesta formada por dos frases sencillas que son falsas,
precisión de la búsqueda al especificar relaciones entre
puede ser verdadera. ¿Cómo le respondes?
frases y palabras clave. El operador AND dice al busca-
15. Un estudiante dice que si la hipótesis es falsa, un razona-
dor que presente todos los documentos que contengan
miento no puede ser válido. ¿Cómo le respondes?
ambas palabras, por ejemplo, “deportes AND beisbol”.

Sugerencia para resolver el problema preliminar


Aquí puede ser útil la estrategia de proponer y verificar. Por ejemplo, determinar qué
sucedería si seleccionas un plato en particular con su rótulo incorrecto. ¿Qué fruta po-
drías sacar de ese plato, y basado en esa información, qué cambios de letrero habría que
hacer? ¿Hay alguna selección que junto con razonamiento lógico conduzca a la rotula-
ción correcta? ¿Hay sólo un punto inicial posible que conduzca a la rotulación correcta?

© Lopez Mateos Editores. ISBN 978-607-95583-2-1, obra completa, versión electrónica, ISBN 978-607-95583-3-8, volumen 1, versión electrónica. Ejemplar asignado a: Helecto Villarroel gutierrez -
helecto@gmail.com. Fecha: 27 de octubre de 2014. Prohibida su modificación, copia o distribución.
56 Una introducción a la resolución de problemas

Resumen del capítulo

I. Resolución de problemas 3. En un patrón recursivo, después de dar


A. La resolución de problemas puede guiarse por uno o más terminos para comenzar la suce-
el siguiente procedimiento de cuatro pasos: sión, cada término sucesivo se obtiene de
1. Entender el problema. los términos previos. La sucesión de Fi-
2. Trazar un plan. bonacci, 1, 1, 2, 3, 5, 8, 13, 21, Á , es un
3. Realizar el plan. ejemplo de una sucesión recursiva, donde
4. Revisar. F1 = 1, F2 = 1, Fn + 2 = Fn + Fn + 1 .
B. Entre las estrategias importantes para resolver 4. an = a # a # a # a # a # Á # a, donde n Z 0.
problemas se incluyen las siguientes: n factores
1. Buscar un patrón. 5. a0 = 1, donde a Z 0.
2. Hacer una tabla. 6. Hallar las diferencias en una sucesión es una
3. Examinar un caso más sencillo o un caso técnica para hallar los términos siguientes.
particular del problema para acercarse a la III. Razonamiento y lógica
solución del caso más general.
*
A. Una proposición es una frase que es verda-
4. Identificar un objetivo parcial. dera o falsa, pero no ambas.
5. Examinar problemas relacionados y deter- B. La negación de una proposición es una propo-
minar si se puede aplicar la misma técnica. sición cuyo valor de verdad es opuesto al de la
6. Trabajar regresivamente. proposición dada. La negación de p se denota
7. Plantear una ecuación. con ¬ p.
8. Trazar un diagrama. C. La proposición compuesta p ¿ q es la con-
9. Proponer y verificar. junción de p y q y, por definición, es verdadera
10. Usar razonamiento indirecto. si, y sólo si, p y q son verdaderas.
11. Usar razonamiento directo. D. La proposición compuesta p ¡ q es la dis-
C. ¡Cuidado con los prejuicios! yunción de p y q, y es verdadera si p o q o am-
II. Patrones matemáticos bas son verdaderas.
A. Los patrones son parte importante de la resolu- E. Las proposiciones de la forma “si p, entonces
ción de problemas. q” son condicionales o implicaciones y son
B. Los patrones se usan en el razonamiento in- falsas sólo si p es verdadera y q es falsa.
ductivo para formar conjeturas. El razona- F. Dada la condicional p : q , se pueden hallar
miento inductivo es un método que consiste en las siguientes:
realizar generalizaciones basadas en observacio- 1. Recíproca: q : p
nes y patrones. Una conjetura es una proposi- 2. Inversa: ¬ p : ¬ q
ción que se piensa verdadera, pero que no se ha 3. Contrapositiva: ¬ q : ¬ p
demostrado si es verdadera o falsa. Una manera G. Si p : q es verdadera, entonces la recíproca y
de probar que una proposición es falsa es exhi- la inversa no necesariamente son verdaderas,
biendo un contraejemplo. pero la contrapositiva es verdadera.
C. Una sucesión es un grupo de términos en un H. Dos proposiciones son lógicamente equiva-
orden definido. lentes si, y sólo si, tienen el mismo valor de
1. Sucesión aritmética: Cada término suce- verdad. Una implicación y su contrapositiva
sivo se obtiene del anterior sumando un son lógicamente equivalentes.
número fijo llamado diferencia. El tér- I. La proposición “p : q y q : p” se escribe
mino n-ésimo está dado por an = a1 + p 4 q, es una bicondicional, y nos referimos
1n - 12d , donde a1 es el primer término y a ella como “p si y sólo si q.”
d es la diferencia. J. Las leyes para determinar la validez de una ar-
2. Sucesión geométrica: Cada término su- gumentación incluyen el modus ponens, el
cesivo se obtiene de su predecesor multi- modus tollens y la regla de la cadena.
plicándolo por un número fijo llamado
razón. El término n-ésimo está dado por
a1r n - 1 , donde a1 es el primer término y r
es la razón.

© Lopez Mateos Editores. ISBN 978-607-95583-2-1, obra completa, versión electrónica, ISBN 978-607-95583-3-8, volumen 1, versión electrónica. Ejemplar asignado a: Helecto Villarroel gutierrez -
helecto@gmail.com. Fecha: 27 de octubre de 2014. Prohibida su modificación, copia o distribución.
Revisión del capítulo 57

Revisión del capítulo

1. Lista tres términos más para completar el patrón: los restantes 128 jugadores juegan en 64 partidos.
a. 0, 1, 3, 6, 10, ____, ____, ____ Así, 65 jugadores entran a la segunda ronda. ¿Cuán-
b. 52, 47, 42, 37, ____, ____, ____ tos partidos deberán jugarse para determinar quién
c. 6400, 3200, 1600, 800, ____, ____, ____ es el campeón del torneo?
d. 1, 2, 3, 5, 8, 13, ____, ____, ____ 12. a. Usa patrones para predecir los dos renglones siguientes.
e. 2, 5, 8, 11, 14, ____, ____, ____
3#2
f. 1, 4, 16, 64, ____, ____, ____ 3 =
g. 0, 4, 8, 12, ____, ____, ____ 2
h. 1, 8, 27, 64, ____, ____, ____ 6#3
3 + 6 =
2. Clasifica cada sucesión del problema 1 como arit- 2
mética, geométrica o ninguna de las dos. 9#4
3 + 6 + 9 =
3. Halla un posible término n-ésimo para: 2
a. 5, 8, 11, 14, Á 12 # 5
3 + 6 + 9 + 12 =
b. 0, 7, 26, 63, Á 2
c. 3, 9, 27, 81, 243, Á
4. Halla los primeros cinco términos de las sucesiones b. Muestra que este patrón funciona en general al
cuyo término n-ésimo es: añadir múltiplos consecutivos de 3.
a. 3n - 2 13. Si una vuelta completa de una llanta de automóvil lo
b. n2 + n mueve 6 pies hacia adelante, ¿cuántas vueltas de
c. 4n - 1 llanta se darán antes de agotar la garantía de 50,000
5. Halla las sumas siguientes: millas?
a. 2 + 4 + 6 + 8 + 10 + Á + 200 14. Los alumnos de la clase de la maestra Dolores se pa-
b. 51 + 52 + 53 + 54 + Á + 151 ran formando un círculo, guardando la misma dis-
6. Construye un contraejemplo, si es posible, para re- tancia entre sí y numerados en orden. El estudiante
futar cada una de las siguientes expresiones: con el número 7 está parado directamente enfrente
a. Si se suman dos números impares, entonces la del estudiante número 17. ¿Cuántos estudiantes hay
suma es impar. en la clase?
b. Si un número es impar, entonces termina en 1 o en 3. 15. Un carpintero tiene tres cajas grandes. Dentro de
c. Si se suman dos números pares, entonces la suma cada caja grande hay dos cajas medianas. Dentro
es par. de cada caja mediana hay cinco cajas pequeñas.
7. Completa el siguiente cuadrado mágico, esto es, ¿Cuántas cajas hay en total?
completa el cuadrado de manera que la suma de 16. ¿Cuántos triángulos diferentes hay en la siguiente
cada renglón, columna y diagonal sea la misma. figura? Explica tu razonamiento:

16 3 2 13
10

9 7 12

4 14

8. ¿Cuántas personas pueden sentarse en 12 mesas


cuadradas alineadas una tras otra si cada mesa da ca-
bida a cuatro personas?
17. María viajó desde su casa en bicicleta, cuesta arriba,
9. Una camisa y una corbata se venden en $95. La ca-
a casa de Luis, a un promedio de 16 km/h. De re-
misa cuesta $55 más que la corbata. ¿Cuánto cuesta
greso por la misma ruta, venía a un promedio de 20
la corbata?
km/h. Si tardó 4 horas en hacer el trayecto de re-
10. Si se van a colocar postes en fila cada 5 m para una
greso, ¿cuánto tiempo duró todo el viaje?
cerca, ¿cuántos postes se necesitan para 100 m de cerca?
18. Usa diferencias para hallar el siguiente término del
11. Hay un total de 129 jugadores en un torneo de eli-
patrón:
minación simple de frontón a mano limpia. En la
primera ronda, el jugador mejor clasificado pasa y 5, 15, 37, 77, 141, _____
© Lopez Mateos Editores. ISBN 978-607-95583-2-1, obra completa, versión electrónica, ISBN 978-607-95583-3-8, volumen 1, versión electrónica. Ejemplar asignado a: Helecto Villarroel gutierrez -
helecto@gmail.com. Fecha: 27 de octubre de 2014. Prohibida su modificación, copia o distribución.
58 Una introducción a la resolución de problemas

19. Una granja de hormigas puede contener un total de * 27. Usa tablas de verdad para mostrar que p : ¬ q K
100,000 hormigas. Si la granja tiene 1500 hormigas q : ¬ p.
el primer día, 3000 hormigas en el segundo día, * 28. Construye tablas de verdad para cada caso:
6000 hormigas el tercer día y así sucesivamente, ¿en a. ( p ¿ ¬ q) ¡ ( p ¿ q)
cuánto tiempo estará llena la granja? b. [( p ¡ q) ¿ ¬ p] : q
20. El equipo de Lipa participó en un concurso de ma- * 29. Obtén conclusiones válidas de las siguientes hipóte-
temáticas en el que los equipos compiten respon- sis verdaderas:
diendo preguntas que valen 3 ó 5 puntos. No se da a. Todos los mexicanos adoran a Juan Gabriel y el mole.
crédito parcial. El equipo de Lipa obtuvo 44 puntos Chucho González es mexicano.
en 12 preguntas. ¿Cuántas preguntas de 5 puntos b. El acero finalmente se oxida.
contestó correctamente el equipo? La estatua de la Libertad tiene una estructura de acero.
21. Se requieren tres piezas de madera para un trabajo. c. Albertina pasó el curso de matemáticas o lo
Se van a obtener de una pieza de 90 cm de longitud. abandonó.
La pieza más larga debe tener 3 veces la longitud de Albertina no lo abandonó.
la mediana y la más corta debe tener 10 cm menos * 30. Escribe las siguientes argumentaciones de manera
que la mediana. ¿Se puede hacer esto en dos cortes? simbólica y determina su validez:
De ser así, di por qué Si tienes la piel delicada, te vas a quemar con el sol.
22. Estoy pensando un número. Si lo duplico, elevo el re- Si te quemas, no irás al baile.
sultado al cuadrado y después lo divido entre 2 y le Si no vas al baile, tus padres querrán saber por qué.
sumo 8, obtengo 40. ¿En qué número estoy pensando? Tus padres no quieren saber por qué no fuiste al
* 23. Explica la diferencia entre las proposiciones siguien- baile.
tes: (i) Todos los estudiantes pasaron el examen final. Por lo tanto, no tienes la piel delicada.
(ii) Algunos estudiantes pasaron el examen final. * 31. Averigua, en cada caso, si la conclusión es verdadera
* 24. ¿Cuáles de las siguientes expresiones son proposiciones? o falsa, y di por qué.
a. La luna no está habitada. a. Si Beto obtiene 80 en el examen final, pasará el
b. 3 + 5 = 8. curso.
c. x + 7 = 15. Beto no pasó el curso.
d. Algunas mujeres tienen doctorado en matemáticas. Por lo tanto, Beto no obtuvo 80 en el final.
* 25. Niega las siguientes proposiciones: b. Si lo construyes, vendrán.
a. Algunas mujeres fuman. Lo construiste.
b. 3 + 5 = 8. Por lo tanto, vendrán.
c. La música de mariachi es ruidosa.
d. Beethoven escribió sólo música clásica.
* 26. Escribe la recíproca, inversa y contrapositiva de lo
siguiente: “Si hay un concierto de rock, alguien se
va a desmayar”.

Bibliografía seleccionada

Ameis, J. “Stories Invite Children to Solve Mathemati- . “Children Who Enjoy Problem-Solving.”
cal Problems.” Teaching Children Mathematics 8 Teaching Children Mathematics 9 (May 2003): 539–544.
( January 2002): 260–264. ———“Teaching Problem Solving in Mathematics.”
Artz, S., and E. Armour-Thomas. “Development of a Teaching Children Mathematics 10 (February 2004):
Cognitive-Metacognitive Framework for Protocol 302–309.
Analysis of Mathematical Problem Solving in Small Buyea, R. “Problem Solving in a Structured Mathemat-
Groups.” Cognition and Instruction 9 (1992): 137–175. ics Program.” Teaching Children Mathematics 13
Bloom, B., and L. Broder. Problem Solving Processes of (February 2007): 300–307.
College Students. Chicago, IL: University of Chicago Clement, L., and J. Bernhard. “A Problem-Solving
Press, 1950. Alternative to Using Key Words.” Mathematics
Buschman, L. “Becoming a Problem-Solver.” Teaching Teaching in the Middle School 10 (March 2005):
Children Mathematics 9 (October 2002): 98–103. 360–365.

© Lopez Mateos Editores. ISBN 978-607-95583-2-1, obra completa, versión electrónica, ISBN 978-607-95583-3-8, volumen 1, versión electrónica. Ejemplar asignado a: Helecto Villarroel gutierrez -
helecto@gmail.com. Fecha: 27 de octubre de 2014. Prohibida su modificación, copia o distribución.
Bibliografía seleccionada 59

Crespo, C., and A. Kyriakides. “To Draw or Not to O’Donnell, B. “On Becoming a Better Problem-Solving
Draw: Exploring Children’s Drawings for Solving Teacher.” Teaching Children Mathematics 12 (March
Mathematics Problems.” Teaching Children Mathe- 2006): 346–351.
matics 14 (September 2007): 118–125. Pólya, G. How to Solve It. Princeton, NJ: Princeton
Dugdale, S., J. Matthews, and S. Guerro. “The Art of University Press, 1945.
Posing Problems and Guiding Investigations.” ———. Mathematical Discovery, Combined Edition. New
Mathematics Teaching in the Middle School 10 (Octo- York: John Wiley & Sons, Inc., 1981.
ber 2004): 140–147. Reeves, A., and R. Gleichowski. “Engaging Contexts
Ferrucci, B., B. Yeap, and J. Carter. “A Modeling for the Game of Nim.” Mathematics Teaching in the
Approach for Enhancing Problem-Solving in the Middle School 12 (December 2006/January 2007):
Middle Grades.” Mathematics Teaching in the Middle 251–255.
School 8 (May 2003): 470–475. Reid, D. “Describing Reasoning in Early Elementary
Hatano, G., and K. Ingaki. “Sharing Cognition School Mathematics.” Teaching Children Mathematics
through Collective Comprehension Activity.” In 9 (December 2002): 234–237.
Perspectives on Socially Shared Cognition, edited by Rigelman, N. “Fostering Mathematical Thinking and
L. Resnick, J. Levine, and S. Teasley. Washington, Problem Solving: The Teacher’s Role.” Teaching
D.C.: American Psychological Association, 1991, Children Mathematics 13 (February 2007): 308–314.
pp. 331–348. Rivera, F., and J. Becker. “Figural and Numerical
Hylton-Lindsay, A. “Problem-Solving, Patterns, Proba- Modes of Generalizing in Algebra.” Mathematics
bility, Pascal, and Palindromes.” Mathematics Teach- Teaching in the Middle School 11 (November 2005):
ing in the Middle School 8 (February 2003): 288–293. 198–203.
Hoosain, E., and R. Chance. “Problem-Solving Strate- Rubenstein, R. “Building Explicit and Recursive Forms
gies of First Graders.” Teaching Children Mathemat- of Patterns with the Function Game.” Mathematics
ics 10 (May 2004): 474–479. Teaching in the Middle School 7 (April 2002):
Kantowski, M. “Problem Solving.” In Mathematics Edu- 426–431.
cation Research: Implications for the 80s, edited by Siegel, M. “The Sum of Cubes: An Activity Review and
E. Fennema. Alexandria, VA: ASCD, 1981. Conjecture.” Mathematics Teaching in the Middle
Krebs, A. “Studying Students’ Reasoning in Writing School 10 (March 2005): 356–359.
Generalizations.” Mathematics Teaching in the Middle Smith, M., A. Hillen, and C. Catania. “Using Pattern
School 10 (February 2005): 284–287. Tasks to Develop Mathematical Understandings
Lee, L., and V. Freiman. “Developing Algebraic Think- and Set Classroom Norms.” Mathematics Teaching in
ing through Pattern Exploration.” Mathematics the Middle School 13 (August 2007): 38–44.
Teaching in the Middle School 11 (May 2006): Steele, D. “Understanding Students’ Problem-Solving
428–433. Knowledge Through Their Writing.” Mathematics
Lester, F. “Developmental Aspects of Children’s Ability Teaching in the Middle School 13 (September 2007):
to Understand Mathematical Proof.” Journal for 102–109.
Research in Mathematics Education 6 (1975): 14–25. Strutchens, M. “Multicultural Literature as a Context
Maher, C., and A. Martino. “The Development of the for Problem Solving: Children and Parents Learn-
Idea of Mathematical Proof: A Five-Year Case ing Together.” Teaching Children Mathematics 8
Study.” Journal for Research in Mathematics Education (April 2002): 448–454.
27 (March 1996): 194–214. Thomas, K. “Students THINK: A Framework for
McLeod, D. “Affective Issues in Research on Teaching Improving Problem Solving.” Teaching Children
Mathematical Problem Solving.” In Teaching and Mathematics 13 (September 2006): 86–95.
Learning Mathematical Problem Solving: Multiple Turner, E., and B. Strawhun. “Posing Problems That
Research Perspectives, edited by E. Silver. Hillsdale, Matter: Investigating School Overcrowding.” Teaching
NJ: LEA, 1985. Children Mathematics 13 (May 2007): 457–463.
Martinez-Cruz, A., and E. Barger. “Adding a la Gauss.” Van de Walle, J. Elementary and Middle School Mathe-
Mathematics Teaching in the Middle School 10 (October matics: Teaching Developmentally. New York: Addison
2004): 152–155. Wesley Longman, 2001.
Moore, R. C. “Making the Transition to Formal Proof.” Van Reeuwijk, M., and M. Wijers. “Students’ Construc-
Educational Studies in Mathematics 27, no. 3 (1994): tion of Formulas in Context.” Mathematics Teaching
249–266. in the Middle School 2 (February 1997): 230–236.
Moran, G. “X-tending the Fibonacci Sequence.” Verzoni, K. “Turning Students into Problem Solvers.”
Mathematics Teaching in the Middle School 7 (April Mathematics Teaching in the Middle School 3 (October
2002): 452–454. 1997): 102–107.

© Lopez Mateos Editores. ISBN 978-607-95583-2-1, obra completa, versión electrónica, ISBN 978-607-95583-3-8, volumen 1, versión electrónica. Ejemplar asignado a: Helecto Villarroel gutierrez -
helecto@gmail.com. Fecha: 27 de octubre de 2014. Prohibida su modificación, copia o distribución.
60 Una introducción a la resolución de problemas

Wallace, A. “Anticipating Student Responses to Im- Wilburne, J. “Preparing Preservice Elementary Teachers
prove Problem Solving.” Mathematics Teaching in the to Teach Problem Solving.” Teaching Children Mathe-
Middle School 12 (May 2007): 504–511. matics 12 (May 2006): 454–463.
Wells, P., and D. Coffey. “Are They Wrong? Or Did Williams, E. “An Investigation of Senior High School
They Just Answer a Different Question?” Teach- Students’ Understanding of Mathematical Proof.”
ing Children Mathematics 12 (November 2005): Journal for Research in Mathematics Education 11 (May
202–207. 1980): 165–166.
Whitin, D. “Problem Posing in the Elementary Class- Yolles, A. “Using Friday Puzzles to Discover Arithmetic
room.” Teaching Children Mathematics 13 (August Sequences.” Mathematics Teaching in the Middle
2006): 14–18. School 9 (November 2003): 180–185.

© Lopez Mateos Editores. ISBN 978-607-95583-2-1, obra completa, versión electrónica, ISBN 978-607-95583-3-8, volumen 1, versión electrónica. Ejemplar asignado a: Helecto Villarroel gutierrez -
helecto@gmail.com. Fecha: 27 de octubre de 2014. Prohibida su modificación, copia o distribución.
Sistemas de numeración
y conjuntos
CAPÍTULO

Problema preliminar
En un viaje, varios egiptólogos/guías platicaban acerca de las personas que asistieron a la última
conferencia sobre educación matemática, todas ellas de Mississippi o Tennessee. Los guías no
podían recordar el número total de personas en el grupo; sin embargo, registraron los siguientes
datos: el grupo contenía 26 mujeres de Mississippi, 17 señoras de Tennessee, 17 hombres de Ten-
nessee, 29 niñas, 44 residentes de Mississippi, 29 señoras, y 24 adultos de Mississippi. Halla el nú-
mero total de personas en el grupo.

61
© Lopez Mateos Editores. ISBN 978-607-95583-2-1, obra completa, versión electrónica, ISBN 978-607-95583-3-8, volumen 1, versión electrónica. Ejemplar asignado a: Helecto Villarroel gutierrez -
helecto@gmail.com. Fecha: 27 de octubre de 2014. Prohibida su modificación, copia o distribución.
62 Sistemas de numeración y conjuntos

E l National Council of Teachers of Mathematics (NCTM), Consejo Nacional de Maestros


de Matemáticas, de Estados Unidos, reconoció en 2006 la necesidad de una mayor
coherencia en el currículo de matemáticas de los grados elementales. En su documento
Curriculum Focal Points for Prekindergarten through Grade 8 Mathematics: A Quest for Coherence
(Puntos focales en el currículo de matemáticas, de las preescolares al grado 8: Una búsqueda
de coherencia), el Consejo sugiere temas específicos que deben enseñarse desde preescolar
hasta el grado 8. Ya desde preescolar, en ese documento vemos que:
Niñas y niños desarrollan una comprensión de los significados de los números completos [0, 1, 2,
3, Á ] y reconocen la cantidad de objetos en grupos pequeños sin contar —el primero y más ele-
mental algoritmo matemático. Entienden que los nombres de los números se refieren a cantida-
des. Usan correspondencia biunívoca (uno a uno) para resolver problemas identificando conjuntos
entre sí y comparando cantidades, así como contando objetos hasta 10 y más. (p. 11)

En este capítulo presentamos varios sistemas de numeración antiguos. Después presenta-


mos el desarrollo histórico matemático realizado por Georg Cantor, que dotó de una es-
tructura al sistema numérico y proporcionó métodos para tratarlo teóricamente.

2-1 Sistemas de numeración

En esta sección introducimos varios sistemas numéricos y los comparamos con el sistema
de numeración indoarábigo que usamos hoy día. Al comparar nuestro actual sistema con
sistemas antiguos que usaban otras bases, podemos percibir con mayor claridad los núme-
ros. Nuestro sistema está basado en 10 dígitos —0, 1, 2, 3, 4, 5, 6, 7, 8 y 9. Los símbolos es-
critos de los dígitos, como el 2 o el 5, son los numerales. A lo largo de los años, diferentes
culturas desarrollaron distintos numerales para representar los números. La tabla 2-1 mues-
tra otras representaciones y su relación con los dígitos del 0 al 9 y el número 10.

Tabla 2-1

Babilonio

Egipcio

Maya

Griego α β γ δ ∈ z ζ η υ ι

Romano I II III IV V VI VII VIII IX X

Hindú

Árabe

Indoarábigo 0 1 2 3 4 5 6 7 8 9 10

En la tabla 2-1 se muestran elementos de los distintos conjuntos de números. Un sistema


de numeración es una colección de propiedades y símbolos acordados para representar
números de manera sistemática. Mediante el estudio de diversos sistemas de numeración,
exploramos la evolución de nuestro sistema actual, el sistema indoarábigo.

© Lopez Mateos Editores. ISBN 978-607-95583-2-1, obra completa, versión electrónica, ISBN 978-607-95583-3-8, volumen 1, versión electrónica. Ejemplar asignado a: Helecto Villarroel gutierrez -
helecto@gmail.com. Fecha: 27 de octubre de 2014. Prohibida su modificación, copia o distribución.
Sección 2-1 Sistemas de numeración y conjuntos 63

Sistema de numeración indoarábigo


El sistema de numeración indoarábigo que usamos actualmente fue desarrollado por los
hindúes y transladado a Europa por los árabes, de ahí el nombre indoarábigo. El sistema in-
doarábigo se basa en las propiedades siguientes:
1. Todos los numerales se construyen a partir de los 10 dígitos —0, 1, 2, 3, 4, 5, 6, 7, 8 y 9.
2. El valor posicional está basado en las potencias de 10, el número base del sistema.
Debido a que el sistema indoarábigo está basado en potencias de 10, a veces se le llama
sistema de base diez, o decimal. El valor posicional asigna un valor a un dígito depen-
diendo de su colocación en un numeral. Para hallar el valor de un dígito en un número
completo, multiplicamos el valor posicional del dígito por su valor nominal, donde el va-
lor nominal es un dígito. Por ejemplo, en el numeral 5984 el 5 tiene el valor posicional
de “millares”, el 9 tiene el valor posicional de “centenas”, el 8 tiene el valor posicional de
“decenas” y el 4 tiene el valor posicional de “unidades”, como se ve en la figura 2-1.

3)
0
r (1
lla 2) 1)
mi 10 (10 s (1)
de s(
des e na enas dade
ida n t c i
Un Ce De Un
5 9 8 4

Figura 2-1

Podríamos escribir 5984 en forma expandida como 5 # 103 + 9 # 102 + 8 # 10 + 4 # 1. En


la forma expandida de 5984 hemos usado exponentes. Por ejemplo 1000, ó 10 # 10 # 10, se
escribe como 103. En este caso, 10 es un factor del producto. En general, tenemos la si-
guiente definición.

Definición de a n
Si a es cualquier número y n es cualquier número natural, entonces
an = a # a # a # Á # a.

n factores

El conjunto de cubos de base diez mostrado en la figura 2-2 consta de unidades, barras, lo-
sas y bloques que representan 1, 10, 100 y 1000, respectivamente. Estos conjuntos de base
▲ diez, un subconjunto de los conjuntos multibase, se pueden usar para enseñar el valor posi-

Nota de cional.
investigación
1
Los cubos de base unidad 10 unidades = 1 barra
diez ayudan a los es-
tudiantes a entender
el valor posicional, a
hacer cálculos preci-
sos en problemas con
sumas y restas de va-
rios dígitos, y a
incluir reagrupa-
100 unidades = 1 losa 1000 unidades = 1 bloque
miento o
intercambio. (Fuson Figura 2-2
1992). ▲▲

© Lopez Mateos Editores. ISBN 978-607-95583-2-1, obra completa, versión electrónica, ISBN 978-607-95583-3-8, volumen 1, versión electrónica. Ejemplar asignado a: Helecto Villarroel gutierrez -
helecto@gmail.com. Fecha: 27 de octubre de 2014. Prohibida su modificación, copia o distribución.
64 Sistemas de numeración y conjuntos

1 barra : 101 = 1 fila de 10 unidades


1 losa : 102 = 1 fila de 10 barras, ó 100 unidades
1 bloque : 103 = 1 fila de 10 losas, ó 100 barras, ó 1000 unidades
Los alumnos manipulan los cubos reagrupándolos. Esto es, toman un conjunto de cubos
de base diez, que representa un número, y lo van reagrupando hasta que tienen la menor
cantidad de piezas posible representando el mismo número. Por ejemplo, supón que tie-
nes 58 unidades y quieres cambiarlas por cubos de base diez. Primero substituye las unida-
des por tantas barras como sea posible. Cinco conjuntos de 10 unidades cada uno se
pueden cambiar por 5 barras. Así, 58 unidades se pueden cambiar de modo que tengan
ahora 5 barras y 8 unidades. En términos de números, esto es análogo a reescribir 58 como
5 # 10 + 8. Se ha demostrado que el uso de objetos manipulables ayuda al estudiante a
comprender, como se ve en la Nota de investigación de la página 63.

Ejemplo 2-1 ¿Cuál es el menor número de piezas que puedes recibir en un intercambio justo por 11








losas, 17 barras y 16 unidades?

Solución 11 losas 17 barras 16 unidades (16 unidades = 1 barra y 6 unidades)

M
1 barra 6 unidades (Intercambio)
11 losas 18 barras 6 unidades (Después del primer intercambio)

11 losas 18 barras 6 unidades (18 barras = 1 losa y 8 barras)

M
1 losa 8 barras (Intercambio)
12 losas 8 barras 6 unidades (Después del segundo intercambio)

12 losas 8 barras 6 unidades (12 losas = 1 bloque y 2 losas)

M
1 bloque 2 losas (Intercambio)
1 bloque 2 losas 8 barras 6 unidades (Después del tercer intercambio)

Por lo tanto, el menor número de piezas es 1 + 2 + 8 + 6 = 17. Este intercambio es aná-


logo a reescribir 11 # 100 + 17 # 10 + 16 como 1 # 103 + 2 # 102 + 8 # 10 + 6, lo cual im-
plica que hay 1286 unidades.

◆ Nota La invención del sistema de numeración indoarábigo está considerada como uno de los
histórica acontecimientos más importantes en matemáticas. Primero se introdujo el sistema en la In-
dia y después se transmitió, por medio de los árabes, al norte de África y España, y de ahí al
resto de Europa. Los historiadores registran el uso del cero como un lugar vacío hasta el si-
glo cuarto a.c. Los matemáticos árabes extendieron el sistema decimal para incluir las frac-
ciones. El matemático italiano Leonardo de Pisa (1170–1250), también conocido como
Fibonacci, estudió en Argelia y a su regreso trajo consigo el nuevo sistema de numeración,
el cual describió y usó en un libro publicado en 1202. ◆

© Lopez Mateos Editores. ISBN 978-607-95583-2-1, obra completa, versión electrónica, ISBN 978-607-95583-3-8, volumen 1, versión electrónica. Ejemplar asignado a: Helecto Villarroel gutierrez -
helecto@gmail.com. Fecha: 27 de octubre de 2014. Prohibida su modificación, copia o distribución.
Sección 2-1 Sistemas de numeración y conjuntos 65

AHORA INTENTA ÉSTE 2-1 Mediante intercambios con cubos de base diez (mostrados en la Figura 2-2)
escribe 3 bloques, 12 losas, 11 barras y 17 unidades en el sistema de numeración indoarábigo.

A continuación trataremos otros sistemas de numeración. El estudio de dichos sistemas


nos dará una perspectiva histórica del desarrollo de los sistemas numéricos y nos ayudará a
comprender mejor nuestro propio sistema.

Sistema de numeración de muescas


El sistema de numeración de muescas usa rayas o muescas para representar cada objeto
contado; por ejemplo, los numerales que representan los diez primeros números son

ƒ, ƒ ƒ, ƒ ƒ ƒ, ƒ ƒ ƒ ƒ, ƒ ƒ ƒ ƒ ƒ, ƒ ƒ ƒ ƒ ƒ ƒ, ƒ ƒ ƒ ƒ ƒ ƒ ƒ, ƒ ƒ ƒ ƒ ƒ ƒ ƒ ƒ, ƒ ƒ ƒ ƒ ƒ ƒ ƒ ƒ ƒ, ƒ ƒ ƒ ƒ ƒ ƒ ƒ ƒ ƒ ƒ

En un sistema de muescas hay una correspondencia entre las marcas y los objetos conta-
dos. El sistema es sencillo pero requiere muchos símbolos, principalmente cuando los nú-
meros son mayores. Además, a medida que los números son más grandes, es más difícil leer
los numerales.
Como vemos en la tira cómica de “Barney Google y Snuffy Smith”, el sistema de muescas
puede mejorarse agrupando. Vemos que las rayas se agrupan de cinco en cinco al trazar una
diagonal cruzando cuatro rayas para formar, así, un grupo de cinco. Al agrupar se facilita la
lectura del numeral.

Pa, ya vas Sí, pero ¿Qué todo tiene que


con la ¿quién está ser educativo para
séptima contando? el niño?
. . . ¡¡ je !! ¡¡ je !!

Sistema de numeración egipcio


El sistema de numeración egipcio, que data de alrededor del 3400 a.c., usaba rayas. En
los primeros nueve numerales de dicho sistema, presentados en la tabla 2-1, se nota el
uso de rayas o muescas. Los egipcios mejoraron el sistema basado solamente en muescas
al desarrollar un sistema de agrupación para representar ciertos conjuntos de números.
Esto facilita la escritura del número. Por ejemplo, los egipcios usaban el símbolo del
hueso del talón, x , para representar diez muescas agrupadas.

© Lopez Mateos Editores. ISBN 978-607-95583-2-1, obra completa, versión electrónica, ISBN 978-607-95583-3-8, volumen 1, versión electrónica. Ejemplar asignado a: Helecto Villarroel gutierrez -
helecto@gmail.com. Fecha: 27 de octubre de 2014. Prohibida su modificación, copia o distribución.
66 Sistemas de numeración y conjuntos

En la tabla 2-2 se muestran otros numerales usados por los egipcios en su sistema, y en la
figura 2-3 algunos de los símbolos del templo de Karnak en Luxor.

Tabla 2-2
Equivalente
Numeral egipcio Descripción indoarábigo
Raya vertical 1
Hueso de talón 10
Rollo 100
Flor de loto 1,000
Dedo señalando 10,000
Pez 100,000
Hombre sorprendido 1,000,000

Figura 2-3

Nota que en la figura 2-3 el símbolo para 100 está labrado en una dirección diferente de
como aparece en la tabla 2-2.
En su forma más sencilla, el sistema egipcio incluye una propiedad aditiva, a saber: el
valor de un número era la suma de los valores nominales de los numerales. Los egipcios es-
cribían los numerales en orden decreciente de izquierda a derecha, como en . El
número se puede convertir a base diez como se muestra a continuación:

representa 100,000
representa 300 1100 + 100 + 1002
representa 20 110 + 102
representa 2 11 + 12
representa 100,322

© Lopez Mateos Editores. ISBN 978-607-95583-2-1, obra completa, versión electrónica, ISBN 978-607-95583-3-8, volumen 1, versión electrónica. Ejemplar asignado a: Helecto Villarroel gutierrez -
helecto@gmail.com. Fecha: 27 de octubre de 2014. Prohibida su modificación, copia o distribución.
Sección 2-1 Sistemas de numeración y conjuntos 67

AHORA INTENTA ÉSTE 2-2


a. Usa el sistema egipcio para representar 1,312,322.
b. Usa el sistema indoarábigo para representar .
c. ¿Qué desventajas ves en el sistema egipcio comparado con el sistema indoarábigo?

Sistema de numeración babilonio


El sistema de numeración babilonio se desarrolló casi al mismo tiempo que el sistema egip-
cio. Los símbolos de la tabla 2-3 se trazaron usando una mediacaña, aplicada vertical u hori-
zontalmente, en tabletas de arcilla.

Tabla 2-3
Numeral babilonio Equivalente indoarábigo

1
10

Los numerales babilonios del 1 al 59 eran similares a los numerales egipcios, pero la raya
vertical y el hueso del talón se reemplazaron por los símbolos mostrados en la tabla 2-3. Por
ejemplo, representa 22.
El sistema de numeración babilonio utilizó un sistema de valor posicional. Los números ma-
yores que 59 se representaban repitiendo grupos de 60, así como ahora usamos grupos de
10. Por ejemplo, representaría 2 # 60 + 20, ó 140. El espacio indica que repre-
senta 2 # 60 en lugar de 2. Los numerales situados inmediatamente a la izquierda del se-
gundo espacio tienen un valor de 60 # 60 multiplicado por el valor nominal, y así
sucesivamente.
representa 20 # 60 + 1, ó 1201
representa 11 # 60 # 60 + 11 # 60 + 1, ó 11 # 602 + 11 # 60 + 1,
ó 40,261
representa 1 # 60 # 60 # 60 + 11 # 60 # 60 + 11 # 60 + 1, ó
1 # 603 + 11 # 602 + 11 # 60 + 1, ó 256,261
Según los cánones actuales, el sistema babilonio inicial sería inadecuado. Por ejemplo, el
símbolo podría representar 2 ó 2 # 60 . Posteriormente, los babilonios introdujeron el
símbolo para indicar ausencia de valor en una posición. Usando el símbolo,
representa 10 # 60 + 21 y representa 10 # 602 + 0 # 60 + 21. En este sentido,
representa al 0.

AHORA INTENTA ÉSTE 2-3


a. Usa el sistema babilonio para representar 12,321.
b. Usa el sistema indoarábigo para representar .
c. ¿Qué ventajas tiene el sistema indoarábigo sobre el sistema babilonio?

© Lopez Mateos Editores. ISBN 978-607-95583-2-1, obra completa, versión electrónica, ISBN 978-607-95583-3-8, volumen 1, versión electrónica. Ejemplar asignado a: Helecto Villarroel gutierrez -
helecto@gmail.com. Fecha: 27 de octubre de 2014. Prohibida su modificación, copia o distribución.
68 Sistemas de numeración y conjuntos

Sistema de numeración maya


En el desarrollo temprano de los sistemas de numeración las personas usaban, con fre-
cuencia, las partes del cuerpo para contar. Los dedos se podían hacer corresponder con
objetos y representar uno, dos, tres, cuatro o cinco objetos. Entonces, con las dos manos
se podían representar hasta diez objetos. En climas cálidos, con los pies descubiertos,
también se podían usar, para contar, los dedos de los pies además de los de las manos. Los
mayas introdujeron un atributo que no estaba presente en el sistema egipcio o en el ba-
bilonio temprano, a saber, un símbolo para el cero. Los mayas usaban sólo tres símbolos,
que se muestran en la tabla 2-4, y basaron su sistema en el 20 con agrupación vertical.
Tabla 2-4
Numeral maya Equivalente indoarábigo
1
5
0

Los símbolos para los primeros diez numerales del sistema maya se mostraron en la tabla 2-1.
Nota los agrupamientos de cinco, donde cada barra horizontal representa un grupo de cinco.
Así, el símbolo para 19 era , o tres cincos y cuatro unos. El símbolo para 20 era , que
representa un grupo de veinte más cero unos. En la figura 2-4(a) tenemos 2 # 5 + 3 # 1, ó 13
grupos de veinte, más 2 # 5 + 1 # 1, u once unos, lo que hace un total de 271. En la figura 2-4(b)
tenemos 3 # 5 + 1 # 1, ó 16, grupos de 20 y cero unos, lo que hace un total de 320.

(2 ? 5 + 3)20 13 ? 20 (3 ? 5 + 1)20 16 ? 20
(2 ? 5 + 1)1 + 11 ? 1 0?1 + 0
271 320
(a) (b)

Figura 2-4

En un verdadero sistema de base veinte el valor posicional de la tercera posición vertical, a


partir de abajo, debería ser 202, ó 400. Sin embargo, el sistema maya usaba 20 # 18, ó 360, en lu-
gar de 400. (El número 360 es una aproximación de la longitud de un año, que constaba de 18
meses de 20 días cada uno, más 5 días de “mala suerte”.) Así, en lugar de los valores posicionales
de 1, 20, 202, 203, 204, y así sucesivamente, los mayas usaron 1, 20, 20 # 18, 202 # 18, 203 # 18, y así
sucesivamente. Por ejemplo, en la figura 2-5(a) tenemos 5 + 1 (ó 6) grupos de 360, más
2 # 5 + 2 (ó 12) grupos de 20, más 5 + 4 (ó 9) grupos de 1, lo que hace un total de 2409. En la fi-
gura 2-5(b) tenemos 2 # 5 (ó 10) grupos de 360, más 0 grupos de 20, más dos 1, lo que hace un
total de 3602. Los espacios son importantes en el sistema maya. Por ejemplo, si se colocan dos
barras horizontales juntas, como en , los símbolos representan 5 + 5 = 10. Pero si están
espaciadas, como en , entonces el valor es 5 # 20 + 5 # 1 = 105.

(1 ? 5 + 1)20 ? 18 6 ? 360 2160 (2 ? 5)20 ? 18 10 ? 360 3600

(2 ? 5 + 2)20 12 ? 20 240 0 ? 20 0 ? 20 0

(1 ? 5 + 4)1 9?1 + 9 2?1 2 + 2


2409 3602
(a) (b)

Figura 2-5

© Lopez Mateos Editores. ISBN 978-607-95583-2-1, obra completa, versión electrónica, ISBN 978-607-95583-3-8, volumen 1, versión electrónica. Ejemplar asignado a: Helecto Villarroel gutierrez -
helecto@gmail.com. Fecha: 27 de octubre de 2014. Prohibida su modificación, copia o distribución.
Sección 2-1 Sistemas de numeración y conjuntos 69

Sistema de numeración romano


El sistema de numeración romano se usó en Europa, en su forma temprana, desde el tercer
siglo a.c. Todavía se usa, como se puede ver en piedras conmemorativas, en las primeras pá-
ginas de libros y en las carátulas de algunos relojes. El sistema romano usa sólo algunos sím-
bolos, como se muestra en la tabla 2-5.
Tabla 2-5

Numeral romano Equivalente indoarábigo

i 1
v 5
x 10
l 50
c 100
d 500
m 1000

Los numerales romanos se pueden combinar usando una propiedad aditiva. Por ejemplo,
mdclxvi representa 1000 + 500 + 100 + 50 + 10 + 5 + 1 = 1666; cccxxviii representa
328, y vi representa 6.
Para evitar repetir un símbolo más de tres veces, como en iiii, se introdujo en la Edad
Media una propiedad substractiva. Por ejemplo, i es menor que v, así que si se coloca a la
izquierda de v, se resta. Entonces, iv tiene el valor de 5 - 1, ó 4, y xc representa 100 - 10,
ó 90. Algunas extensiones de la propiedad substractiva podrían conducir a resultados ambi-
guos. Por ejemplo, ixc podría ser 91 u 89. Por costumbre, 91 se escribe xci y 89 se escribe
lxxxix. En general, sólo un símbolo de número menor se coloca a la izquierda de un sím-
bolo de número mayor, y la pareja debe ser una de las listadas en la tabla 2-6.

Tabla 2-6

Numeral romano Equivalente indoarábigo

iv 5 – 1, ó 4
ix 10 – 1, ó 9
xl 50 – 10, ó 40
xc 100 – 10, ó 90
cd 500 – 100, ó 400
cm 1000 – 100, ó 900

En la Edad Media se colocó una barra sobre un número romano para multiplicarlo por
1000. El uso de barras se basa en una propiedad multiplicativa. Por ejemplo, v representa
5 # 1000, ó 5000, y cdx representa 410 # 1000, ó 410,000. Para indicar números aún mayores,
se usan más barras. Por ejemplo, v representa 15 # 100021000, ó 5,000,000; cxi representa
111 # 10003, ó 111,000,000,000; y cxi representa 110 # 1000 + 1, ó 110,001.
Se pueden usar varias propiedades para representar algunos números, por ejemplo:
dclix = (500 # 1000) + (100 + 50) + (10 - 1) = 500,159
Multiplicativa Aditiva Substractiva
Aditiva

© Lopez Mateos Editores. ISBN 978-607-95583-2-1, obra completa, versión electrónica, ISBN 978-607-95583-3-8, volumen 1, versión electrónica. Ejemplar asignado a: Helecto Villarroel gutierrez -
helecto@gmail.com. Fecha: 27 de octubre de 2014. Prohibida su modificación, copia o distribución.
70 Sistemas de numeración y conjuntos

Además, el sistema romano evolucionó al transcurrir el tiempo, de modo que existen ejem-
plos en los cuales no se siguen todas las reglas.

Sistemas de numeración con otras bases


Para comprender mejor nuestro sistema de base diez, y para investigar algunos de los pro-
blemas que pudieran tener los alumnos cuando están aprendiendo el sistema indoarábigo,
buscamos sistemas similares pero con diferente base numérica.

Base cinco
Las personas de Luo, en Kenia, usaban un sistema quintario, o de base cinco. Un sistema de
este tipo se puede modelar contando sólo con una mano. Los dígitos disponibles para con-
tar son 0, 1, 2, 3 y 4. En el “sistema de una mano”, o sistema de base cinco, se cuenta 1, 2,
3, 4, 10, donde 10 representa una mano, sin dedos adicionales. Para contar en base cinco se pro-
cede según se muestra en la figura 2-6. Escribimos el número “cinco” en letras pequeñas
debajo del numeral para recordar que el número está escrito en base cinco. Si no se escribe
un número, entonces suponemos que está en base diez. Nota, además, que 1, 2, 3, 4 son
iguales y tienen el mismo significado en ambas bases, cinco y diez.

Sistema de una mano Símbolo en base cinco Cubos en base cinco

0 dedos 0cinco

1 dedo 1cinco

2 dedos 2cinco

3 dedos 3cinco

4 dedos 4cinco

1mano y 0 dedos 10cinco

1 mano y 1 dedo 11cinco

1 mano y 2 dedos 12cinco

1 mano y 3 dedos 13cinco

1 mano y 4 dedos 14cinco

2 manos y 0 dedos 20cinco

2 manos y 1 dedo 21cinco

Figura 2-6
Contar en base cinco es similar a contar en base diez. Como sólo tenemos cinco dígitos
(0cinco, 1cinco, 2cinco, 3cinco y 4cinco), 4cinco juega el papel del 9 en base diez. En la figura 2-7 se
muestra cómo podemos hallar el número que sigue a 34cinco usando cubos de base cinco.

+1 Se cambian por una barra


+1
34cinco 34cinco + 1cinco 40cinco
Figura 2-7

© Lopez Mateos Editores. ISBN 978-607-95583-2-1, obra completa, versión electrónica, ISBN 978-607-95583-3-8, volumen 1, versión electrónica. Ejemplar asignado a: Helecto Villarroel gutierrez -
helecto@gmail.com. Fecha: 27 de octubre de 2014. Prohibida su modificación, copia o distribución.
Sección 2-1 Sistemas de numeración y conjuntos 71

¿Qué número sigue al 44cinco? Ya no hay más números de dos dígitos en el sistema,
después de 44cinco. En base diez ocurre lo mismo con el 99. Usamos el 100 para repre-
sentar diez números 10, o un 100. En el sistema de base cinco necesitamos un símbolo
para representar cinco 5. Continuando con la analogía con la base diez, usamos 100cinco
para representar un grupo de cinco 5, cero grupos de cinco, y cero unidades. Para dis-
tinguirlo del “cien” en base diez, el nombre de 100cinco se lee “uno-cero-cero base
cinco”. El número 100 significa 1 # 102 + 0 # 101 + 0, mientras que el número 100cinco
significa (1 # 102 + 0 # 101 + 0)cinco, ó 1 # 52 + 0 # 51 + 0, ó 25.
En la figura 2-8 presentamos ejemplos de numerales en base cinco junto con su repre-
sentación de cubos en base cinco y conversiones a base diez. A lo largo del libro usaremos
cubos multibase para ilustrar varios conceptos.
Numeral en base cinco Cubos en base cinco Numeral en base diez
14cinco 1 ?5 1 4 5 9

124cinco 1?52 1 2?5 1 4 5 39

1030cinco 1?53 1 0?52 1 3?5 1 0?1 5 140

Figura 2-8

Ejemplo 2-2 Convierte 11244cinco a base diez.










Solución 11244cinco = 1 # 54 + 1 # 53 + 2 # 52 + 4 # 51 + 4 # 1
= 1 # 625 + 1 # 125 + 2 # 25 + 4 # 5 + 4 # 1
= 625 + 125 + 50 + 20 + 4
= 824

El ejemplo 2-2 sugiere un método para cambiar un número en base diez a un número en
base cinco, usando potencias de 5. Para convertir 824 a base cinco, dividimos entre poten-
cias sucesivas de cinco. A continuación presentamos un método abreviado para ilustrar la
conversión mencionada:

54 = 625 : 625 824 1 ¿Cuántos grupos de 625 hay en 824?


- 625
53 = 125 : 125 199 1 ¿Cuántos grupos de 125 hay en 199?
- 125
52 = 25 : 25 74 2 ¿Cuántos grupos de 25 hay en 74?
- 50
51 = 5 : 5 24 4 ¿Cuántos grupos de 5 hay en 24?
-20
50 = 1 : 1 4 4 ¿Cuántos 1 hay en 4?
-4
0
Así, 824 = 11244cinco.

© Lopez Mateos Editores. ISBN 978-607-95583-2-1, obra completa, versión electrónica, ISBN 978-607-95583-3-8, volumen 1, versión electrónica. Ejemplar asignado a: Helecto Villarroel gutierrez -
helecto@gmail.com. Fecha: 27 de octubre de 2014. Prohibida su modificación, copia o distribución.
72 Sistemas de numeración y conjuntos

AHORA INTENTA ÉSTE 2-4  Se muestra un método diferente de convertir 824 a 5 824
base cinco por medio de divisiones sucesivas entre 5. En cada caso el cociente se 5 164 4
coloca debajo del dividendo y el residuo se coloca a la derecha, en el mismo renglón 5 32 4
que el cociente. La respuesta se lee de abajo hacia arriba, esto es, 11244cinco. 5 6 2
1 1
a. ¿Por qué funciona el método?
b. Usa el método anterior para convertir 728 a base cinco.

Se pueden usar calculadoras con la característica de división entera — INT, en una cal-
culadora Texas Instruments o ,R en una Casio— para cambiar números de base diez a di-
ferentes bases numéricas. Por ejemplo, para convertir 8 a base cinco tecleamos 8 INT,
1 3
5 = y obtenemos Q R . Esto implica que 8 = 13cinco. ¿Funcionará esta técnica
para convertir 34 a base cinco? ¿por qué sí o por qué no?

Base dos
Se relatan historias de tribus antiguas que usaban la base dos. Algunas tribus aborígenes aún
cuentan “uno, dos, dos y uno, dos dos, dos dos y uno, … .” Como la base dos tiene sólo dos
dígitos, se llama sistema binario. La base dos es particularmente importante debido a su
uso en computadoras. Uno de los dos dígitos se representa con la presencia de una señal
eléctrica y el otro con la ausencia de una señal eléctrica. Aunque la base dos funciona bien
para algunos propósitos, es ineficiente para el uso diario pues al contar con este sistema se
alcanzan rápidamente números de multiples dígitos. En la siguiente tira cómica vemos a un
bebé trabajando con el sistema binario.

Lo que me
¡Qué! ¿¡Escribiendo
permite
un nuevo programa!?
este sistema
operativo.

Las conversiones de base dos a base diez y viceversa se pueden efectuar de manera similar
a la usada para base cinco.

Ejemplo 2-3 a. Convierte 10111dos a base diez.










b. Convierte 27 a base dos.

Solución a. 10111dos = 1 # 24 + 0 # 23 + 1 # 22 + 1 # 21 + 1
= 16 + 0 + 4 + 2 + 1
= 23

© Lopez Mateos Editores. ISBN 978-607-95583-2-1, obra completa, versión electrónica, ISBN 978-607-95583-3-8, volumen 1, versión electrónica. Ejemplar asignado a: Helecto Villarroel gutierrez -
helecto@gmail.com. Fecha: 27 de octubre de 2014. Prohibida su modificación, copia o distribución.
Sección 2-1 Sistemas de numeración y conjuntos 73

b. 16 27 1 ¿Cuántos grupos de 16 hay en 27? Solución alternativa:


-16 2 27
8 11 1 ¿Cuántos grupos de 8 hay en 11? 2 13 1
-8 2 6 1
2 3 0
4 3 0 ¿Cuántos grupos de 4 hay en 3?
1 1
-0
2 3 1 ¿Cuántos grupos de 2 hay en 3?
-2
1 1 1 ¿Cuántos 1 hay en 1?
-1
0

Así, 27 es equivalente a 11011dos.


Base doce
Otro sistema de numeración usado comúnmente es el de base doce, o sistema duodecimal
(las “docenas”). Los huevos se compran por docena y los lápices se compran por gruesa
(una docena de docenas). En base doce hay doce dígitos, así como hay diez dígitos en la base
diez, cinco dígitos en la base cinco y dos dígitos en la base dos. En la base doce necesitamos
nuevos símbolos para representar los siguientes grupos de x:
10 x 11 x
y
xxxxxxxxxx xxxxxxxxxxx
Usamos D y O, respectivamente, de modo que los dígitos en base doce son 0, 1, 2, 3, 4, 5, 6,
7, 8, 9, D y O. Así, en base doce contamos “1, 2, 3, 4, 5, 6, 7, 8, 9, D, O, 10, 11, 12, Á , 17,
18, 19, 1D, 1O, 20, 21, 22, Á , 28, 29, 2D, 2O, 30, Á .”

Ejemplo 2-4 a. Convierte O2Ddoce a base diez.










b. Convierte 1277 a base doce.

Solución a. O2Ddoce = 11 # 122 + 2 # 121 + 10 # 1


= 11 # 144 + 24 + 10
= 1584 + 24 + 10
= 1618
b. 144 1277 8 ¿Cuántos grupos de 144 hay en 1277?
-1152
12 125 D ¿Cuántos grupos de 12 hay en 125?
-120
1 5 5 ¿Cuántos 1 hay en 5?
-5
0

Así, 1277 = 8D5doce.


© Lopez Mateos Editores. ISBN 978-607-95583-2-1, obra completa, versión electrónica, ISBN 978-607-95583-3-8, volumen 1, versión electrónica. Ejemplar asignado a: Helecto Villarroel gutierrez -
helecto@gmail.com. Fecha: 27 de octubre de 2014. Prohibida su modificación, copia o distribución.
74 Sistemas de numeración y conjuntos

Ejemplo 2-5 Roberto usó la base doce para escribir lo siguiente:











g36doce = 1050diez
¿Cuál es el valor de g?

Solución Usando la forma expandida, podemos escribir las ecuaciones siguientes:


g # 122 + 3 # 12 + 6 # 1 = 1050
144g + 36 + 6 = 1050
144g + 42 = 1050
144g = 1008
g = 7

Evaluación 2-1A

1. En cada caso, di cuál numeral representa el mayor nú- 8. Un cierto número completo de tres dígitos tiene las pro-
mero y por qué: piedades siguientes: el dígito de las centenas es mayor
a. mcdxxiv y mcdxxiv que 7, el dígito de las decenas es un número impar y la
b. 4632 y 46,032 suma de los dígitos es 10. ¿Qué número podría ser?
c. y 9. Estudia el siguiente marco de conteo. En el marco, el
d. y valor de cada punto está representado por el número en
e. y la caja ubicado debajo del punto. Por ejemplo, la figura
2. En cada caso, menciona el número subsecuente y el pre- siguiente representa el número 154:
cedente (uno más y uno menos):
a. mcmxlix •• ••• ••
b. 64 8 1
c.
d. ¿Qué números están representados en los marcos (a) y (b)?
3. Si en una piedra conmemorativa de un edificio se labra a. ••• •• •
el año en que éste se construyó, y en la piedra se lee
mcmxxii, ¿cuándo se construyó el edificio? 25 5 1
4. Escribe los números siguientes en símbolos romanos: b. • • •
a. 121 b. 42 8 4 2 1
5. Completa la tabla siguiente, la cual compara símbolos de
números en diferentes sistemas de numeración: 10. Escribe el numeral en base cuatro para los cubos en base
cuatro dados a continuación.
Indo-
arábigo Babilonio Egipcio Romano Maya
a. 72
b.
c.
6. Para cada uno de lo numerales decimales siguientes, da
el valor posicional del numeral subrayado: 11. Escribe los primeros 15 números para cada una de las si-
a. 827,367 guientes bases:
b. 8,421,000 a. base dos b. base cuatro
7. Reescribe cada caso como un numeral en base diez: 12. ¿Cuántos dígitos diferentes se necesitan para la base
a. 3 # 106 + 4 # 103 + 5 veinte?
b. 2 # 104 + 1 13. Escribe 2032cuatro en notación expandida.

© Lopez Mateos Editores. ISBN 978-607-95583-2-1, obra completa, versión electrónica, ISBN 978-607-95583-3-8, volumen 1, versión electrónica. Ejemplar asignado a: Helecto Villarroel gutierrez -
helecto@gmail.com. Fecha: 27 de octubre de 2014. Prohibida su modificación, copia o distribución.
Sección 2-1 Sistemas de numeración y conjuntos 75

14. Determina el mayor número de tres dígitos en cada una $900 de manera que des el menor número de premios?
de las bases siguientes: 24. Efectúa las conversiones siguientes:
a. base dos b. base doce a. 58 días a semanas y días
15. Halla los números precedente y sucesor: b. 29 horas a días y horas
a. OO0doce 25. En cada caso halla b, de ser posible. De no ser posible, di
b. 100000dos por qué.
c. 555seis a. b2siete = 44diez b. 5b2doce = 734diez
16. ¿Cuál es el error, de existir, con los numerales siguientes? 26. El ábaco chino, según está presentado, muestra el nú-
a. 204cuatro mero 5857. (Sugerencia: Las cuentas sobre la barra repre-
b. 607cinco sentan cantidades de 5, de 50, de 500 y de 5000.)
17. El menor número de cubos de base cuatro necesarios
para representar 214 es bloque(s) losa(s)
barra(s) unidad(es).
18. Dibuja cubos multibase para representar 231cinco.
19. Una introducción a la base cinco es particularmente ade-
cuada para el aprendizaje inicial en la escuela elemental,
cuando el niño puede pensar en cambiar monedas de
centavo, cinco y veinticinco (llamadas pesetas). Usa sólo
estas monedas para responder lo siguiente:
a. ¿Cuál es el menor número de monedas de veinti-
cinco, de cinco y de a centavo que puedes recibir en Explica cómo se representa el número 5857 y muestra
un intercambio justo por dos de veinticinco, nueve de cómo se representa el número 4869.
cinco y ocho centavos? 27. En una calculadora, usando sólo las teclas diferentes del
b. ¿Cómo podrías usar el enfoque de (a) para escribir 73 cero, llena la pantalla de la calculadora para mostrar el
en base cinco? mayor número posible si cada tecla se puede usar una
20. Recuerda que con cubos de base diez, 1 barra = 10 sola vez.
unidades, 1 losa = 10 barras y 1 bloque = 10 losas (ver la 28. En un juego llamado BARRER, debemos “barrer” dígi-
Figura 2-2). En los siguientes conjuntos de piezas multi- tos de la pantalla de una calculadora sin cambiar ninguno
base, realiza todos los intercambios posibles para obtener de los otros dígitos. En este caso, “barrer” significa re-
el menor número de piezas y escribe el numeral corres- emplazar con un 0 el o los dígitos escogidos. Por ejem-
pondiente en la base dada. plo, si el número inicial es 54,321 y vamos a barrer el 4,
a. Diez losas en base diez podemos restar 4000 para obtener 50,321. Completa los
b. Veinte losas en base doce dos problemas siguientes y después trata con otros nú-
21. Cambia 42ocho a base dos sin cambiar primero a base meros, o reta a otra persona para barrer un dígito de un
diez. número que hayas colocado en la pantalla:
22. Escribe cada caso en base diez: a. Barre los 2 de 32,420.
a. 432cinco b. 101101dos b. Barre el 5 de 67,357.
c. 92Odoce
23. Te piden distribuir $900 en premios. Los premios son de
$625, $125, $25, $5 y $1. ¿Cómo deberás repartir los

Evaluación 2-1B

1. En cada caso di cuál numeral representa el mayor nú- a. mi


mero, y por qué: b. cmxcix
a. mdcxxiv y mcdxxiv c.
b. 3456 y 30,456 d.
c. y e.
d. y 3. Escribe los números siguientes en símbolos romanos:
e. y a. 89
2. En cada caso, menciona el número subsecuente y el pre- b. 5202
cedente (uno más y uno menos):

© Lopez Mateos Editores. ISBN 978-607-95583-2-1, obra completa, versión electrónica, ISBN 978-607-95583-3-8, volumen 1, versión electrónica. Ejemplar asignado a: Helecto Villarroel gutierrez -
helecto@gmail.com. Fecha: 27 de octubre de 2014. Prohibida su modificación, copia o distribución.
76 Sistemas de numeración y conjuntos

4. Completa la tabla siguiente, la cual compara símbolos de 15. ¿Cuál es el error, de existir, en los numerales siguientes?
números en diferentes sistemas de numeración: a. 306cuatro
Indo- b. 1023dos
arábigo Babilonio Egipcio Romano Maya 16. El menor número de cubos de base tres necesarios para
representar 79 es bloque(s) losa(s)
a. 78 barra(s) unidad(es).
b. 17. Dibuja cubos multibase para representar 1001dos.
c. 18. Usando un sistema numérico basado en docenas y grue-
sas, ¿cómo describirías la representación para 277?
5. Para cada uno de lo numerales decimales siguientes, da 19. Sin cambiar cada número a base diez, di cuál es el menor
el valor posicional del numeral subrayado: de cada uno de los siguientes pares de números:
a. 97, 998 a. OOD9Odoce ó O0D9Odoce
b. 810, 485 b. 1011011dos ó 101011dos
6. Reescribe cada caso como un numeral en base diez: c. 50555seis ó 51000seis
a. 3 # 103 + 5 # 102 + 6 # 10 20. ¿Cuál es el menor número de piezas de bloques multi-
b. 9 # 106 + 9 # 10 + 9 base que se pueden usar para escribir el numeral corres-
7. Cierto número de dos dígitos tiene la propiedad de que el pondiente en la base dada?
dígito de las unidades es 4 menos que el dígito de las dece- a. 10 barras en base cuatro
nas y el dígito de las decenas es el doble del dígito de las b. 10 barras en base tres
unidades. ¿Qué número podría ser? 21. Convierte cada número en base diez a números en la
8. En un marco de conteo se representa el número siguiente. base indicada:
¿Qué número es? Explica tu razonamiento. a. 234 a base cuatro
b. 1876 a base doce
• •• •• c. 303 a base tres
27 9 1 d. 22 a base dos
9. Escribe el numeral en base tres de la representación en 22. Escribe cada caso en base diez:
base tres mostrada. a. 432seis
b. 11011dos
c. O29doce
23. ¿Quién quiere dinero? es el nombre de un programa de te-
levisión que distribuye premios que son potencias de 2.
10. Escribe los primeros 10 números de cada una de las ba-
¿Cuál es el mínimo número de premios que se pueden
ses siguientes:
distribuir con $900?
a. base tres
24. Una cafetería vendió 1 taza, 1 pinta (2 tazas) y 1 cuarto
b. base ocho
(2 pintas) de café. Expresa en base dos el número de ta-
11. ¿Cuántos dígitos diferentes se necesitan para la base
zas vendidas.
dieciocho?
25. En cada caso halla b, de ser posible. De no ser posible, di
12. Escribe 2022tres en forma expandida.
por qué.
13. Determina cuál es el mayor número de tres dígitos en
a. b3ocho - 31diez
cada una de las siguientes bases:
b. 1b2doce = 1534seis
a. base tres b. base doce
26. En una calculadora usa sólo las teclas numéricas y llena la
14. Halla los números precedente y sucesor:
pantalla con el mayor número posible de cuatro dígitos, si
a. 100siete
cada tecla se puede usar una sola vez.
b. 10000dos
c. 101dos

Conexiones matemáticas 2-1

Comunicación 2. ¿Cuáles son las mayores desventajas de cada uno de los


1. Benjamín afirma que cero es lo mismo que nada. Explica sistemas siguientes?
cómo responderías a la afirmación de Benjamín si fueras a. Egipcio b. Babilonio c. Romano
su maestra.

© Lopez Mateos Editores. ISBN 978-607-95583-2-1, obra completa, versión electrónica, ISBN 978-607-95583-3-8, volumen 1, versión electrónica. Ejemplar asignado a: Helecto Villarroel gutierrez -
helecto@gmail.com. Fecha: 27 de octubre de 2014. Prohibida su modificación, copia o distribución.
Sección 2-1 Sistemas de numeración y conjuntos 77

3. a. ¿Por qué en México se escriben comas para separar Preguntas del salón de clase
números grandes en grupos de tres? 7. Al estudiar varias bases numéricas, una estudiante pre-
b. Halla ejemplos de países donde no se use la coma para gunta si es posible tener un número negativo como base.
separar grupos de tres dígitos. ¿Qué le dices a esta estudiante?
4. Marta asegura que si realizas una serie de actividades y 8. Un estudiante afirma que el sistema romano es un sis-
cálculos matemáticos, ella te puede decir cuál es tu mas- tema de base diez pues tiene símbolos para 10, 100 y
cota de la suerte. Primero debes hallar tu número espe- 1000. ¿Qué le respondes?
cial de la siguiente manera. 9. Al usar numerales romanos, una estudiante pregunta si
Toma el número de tu mes de nacimiento. es correcto escribir ii , así como mi, para 1001. ¿Cómo
Súmale 24. respondes?
Súmale la diferencia obtenida cuando restas el número de 10. Un padre de familia se queja del uso de objetos manipula-
tu mes de nacimiento de 12. bles en el salón de clase y prefiere que se usen los dedos
Divide entre 3. para contar. ¿Qué le dices?
Réstale 3.
El resultado es tu número especial. Preguntas del Third International Mathematics and
Ahora asigna a cada letra del alfabeto el número de su Science Study (TIMSS) (Tercer Estudio Internacional
orden alfabético, esto es, a = 1, b = 2,c = 3, d = 4 y así sobre las Matemáticas y la Ciencia)
sucesivamente. Halla la letra que corresponda a tu nú- ¿Cuál es el dígito que está en el lugar de las centenas en 2345?
mero especial. A continuación, escribe el nombre de un a. 2 b. 3 c. 4 d. 5
animal que comience con esa letra. ¿Cuál fue la mascota TIMSS, 2003, Grado 4
de la suerte que te dijo Marta? ¿Cómo funciona esto? ¿Cuál es el nombre de 9740?
Solución abierta a. Nueve mil setenta y cuatro
b. Nueve mil setecientos cuarenta
5. Un inspector de pesas y medidas usa un conjunto espe-
cial de pesas para verificar la precisión de las básculas. Se c. Nueve mil setecientos cuatro
colocan varias pesas en una báscula para verificar la pre- d. Novecientos setenta y cuatro mil
cisión de cualquier cantidad, de 1 oz a 15 oz. ¿Cuál es el TIMSS, 2003, Grado 4
menor número de pesas que necesita el inspector? ¿Qué
Pregunta del National Assessment of Educational Progress
pesas se necesitan para verificar la precisión de básculas
(NAEP) (Evaluación Nacional del Progreso Educativo)
de 1 oz a 15 oz? ¿Y de 1 oz a 31 oz?
Aprendizaje colectivo
6. a. Crea un sistema de numeración con tus propios sím-
bolos y escribe un párrafo explicando las propiedades
del sistema.
b. Completa la tabla siguiente usando el sistema:

Numeral Numeral en
indoarábigo tu sistema
1 1 cuarto = 2 pintas
5
El señor Haro compró 6 pintas de leche. ¿A cuántos
10 cuartos de leche equivalen?
50 a. 3 b. 4 c. 6 d. 12
100
NAEP, 2007, Grado 4
5,000
10,000
115,280

© Lopez Mateos Editores. ISBN 978-607-95583-2-1, obra completa, versión electrónica, ISBN 978-607-95583-3-8, volumen 1, versión electrónica. Ejemplar asignado a: Helecto Villarroel gutierrez -
helecto@gmail.com. Fecha: 27 de octubre de 2014. Prohibida su modificación, copia o distribución.
78 Sistemas de numeración y conjuntos

2-2 Descripción de conjuntos

Una vez presentados distintos sistemas de numeración y examinados algunos aspectos del
sistema indoarábigo que usamos actualmente, ha llegado el momento de considerar una de
las principales aportaciones surgidas casi al principio del siglo veinte, que dotó de una base
teórica al sistema numérico que ya estábamos acostumbrados a usar. En los años de 1871 a
1884 Georg Cantor creó la teoría de conjuntos, que tuvo un profundo impacto en la investi-
gación y enseñanza de las matemáticas.
Los conjuntos y las relaciones entre ellos forman una base para que los niños aprendan
los conceptos de números completos y de “menor que”, así como los de suma, resta y multi-
plicación de números completos. Introducimos la notación de conjuntos, las relaciones en-
tre conjuntos, las operaciones entre conjuntos y sus propiedades. El concepto de conjunto
es útil para definir relaciones y funciones (ver el Capítulo 4).
En los Principios y objetivos (2000) de la NCTM vemos que:
Los programas desde preescolar hasta el grado 12 capacitarán a todos los estudiantes para
• entender los números, maneras de representar números, relaciones entre números, y sistemas
numéricos;
• comprender significados de las operaciones y cómo se relacionan entre sí Á . (p. 32)
La comprensión que tengan los maestros de los números y operaciones puede aumentar si
se tiene una comprensión profunda de las matemáticas que hay detrás del sistema numé-
rico. Una parte de dicha comprensión incluye la teoría de conjuntos.

El lenguaje de los conjuntos


Un conjunto se entiende como una colección de objetos. Los objetos individuales de un
conjunto son sus elementos o miembros. Por ejemplo, cada letra es un elemento del con-
junto de las letras del idioma español. El conjunto A de las letras minúsculas del alfabeto
español se puede escribir de la siguiente manera en notación de conjuntos:
A = 5a, b, c, d, e, f, g, h, i, j, k, l, m, n, ñ, o, p, q, r, s, t, u, v, w, x, y, z6
El orden en que se escriben los elementos no establece diferencia alguna, y cada elemento se lista una
sola vez. Así, 5l, e, r6 y 5r, e, l6 se consideran el mismo conjunto.
La pertenencia de un elemento a un conjunto se representa por medio del símbolo 僆.
Por ejemplo, b 僆 A. Si un elemento no pertenece a un conjunto, se usa el símbolo 僆. Por
ejemplo, 3 僆 A.

Nota

Georg Cantor (1845–1918) nació en San Petersburgo, Rusia. Su familia se mudó a Frank-
histórica furt cuando él tenía 11 años. En contra de los consejos de su padre de estudiar ingeniería,
Cantor siguió la carrera de matemáticas y obtuvo su doctorado en Berlín a la edad de 22
años. La mayor parte de su carrera académica la realizó en la Universidad de Halle. Su de-
seo de ser profesor en la Universidad de Berlín no se materializó, pues su trabajo ganó poco
reconocimiento durante su vida.
Sin embargo, después de su muerte el trabajo de Cantor se ha elogiado como un “asom-
broso producto del pensamiento matemático y una de las más bellas realizaciones de la ac-
tividad humana”. ◆

© Lopez Mateos Editores. ISBN 978-607-95583-2-1, obra completa, versión electrónica, ISBN 978-607-95583-3-8, volumen 1, versión electrónica. Ejemplar asignado a: Helecto Villarroel gutierrez -
helecto@gmail.com. Fecha: 27 de octubre de 2014. Prohibida su modificación, copia o distribución.
Sección 2-2 Descripción de conjuntos 79

O B S E R VA C I Ó N En matemáticas no se pueden intercambiar libremente una letra ma-


yúscula y una minúscula. Por ejemplo, en el mencionado conjunto A tenemos que b 僆 A
pero B 僆 A.

Para que un conjunto pueda ser así llamado en matemáticas, debe estar bien definido;
esto es, si nos dan un conjunto y algún objeto particular, debemos poder decir si el objeto
pertenece o no al conjunto. Por ejemplo, el conjunto de los habitantes de la ciudad de Ve-
racruz que comieron arroz el 1º de enero del 2010 está bien definido. Podemos no saber si
un habitante particular de Veracruz comió o no arroz, pero ese habitante pertenece o no
pertenece al conjunto. Por otro lado, el conjunto de la gente alta no está bien definido pues
no existe una manera precisa de cómo calificar a una persona como “alta”.
Podemos usar conjuntos para definir términos matemáticos. Por ejemplo, el conjunto N
de los números naturales se define como
N = 51, 2, 3, 4, Á 6
Una elipsis (tres puntos) indica que la sucesión continúa de la misma manera.
Dos métodos comunes para describir los conjuntos son el listado y la notación cons-
tructora de conjuntos, como vemos en los ejemplos:
Método de listado: C = 51, 2, 3, 46
Notación constructora de conjuntos: C = 5x ƒ x 僆 N donde x 6 56
Esta notación se lee como sigue:
C = { x ƒ x僆N donde x 6 5}
Conjunto C es el todos tales x es un donde x es menor
igual conjunto los elementos que número que 5
a de x natural

Cuando los elementos individuales de un conjunto no se conocen o son demasiados para


listarlos, se usa la notación constructora de conjuntos. Por ejemplo, el conjunto de decima-
les entre 0 y 1 se puede escribir como
D = 5x ƒ x es un decimal entre 0 y 16
que se lee “D es el conjunto de todos los elementos x tales que x es un decimal entre 0 y 1”. Se-
ría imposible listar todos los elementos de D. Por lo tanto, es indispensable aquí la notación
constructora de conjuntos.
Ejemplo 2-6 Escribe los siguientes conjuntos usando la notación constructora de conjuntos:
a. 52, 4, 6, 8, 10, Á 6 b. 51, 3, 5, 7, Á 6








Solución a. 5x ƒ x es un número natural par6. O, como todo número natural par se puede
escribir como 2 multiplicado por algún número natural, este conjunto se
puede escribir como 5x ƒ x = 2n, donde n 僆 N6; o bien, en una forma más
sencilla, como 52n ƒ n 僆 N6.
b. 5x ƒ x es un número natural impar6. O, como todo número natural impar se
puede escribir como algún número par menos uno, este conjunto se puede es-
cribir como 5x ƒ x = 2n - 1, donde n 僆 N6 ó 52n - 1 ƒ n 僆 N6.

© Lopez Mateos Editores. ISBN 978-607-95583-2-1, obra completa, versión electrónica, ISBN 978-607-95583-3-8, volumen 1, versión electrónica. Ejemplar asignado a: Helecto Villarroel gutierrez -
helecto@gmail.com. Fecha: 27 de octubre de 2014. Prohibida su modificación, copia o distribución.
80 Sistemas de numeración y conjuntos

Ejemplo 2-7 Cada uno de los conjuntos siguientes se describe mediante la notación constructora de con-
juntos. Escribe cada conjunto listando sus elementos.









a. A = 52k + 1 ƒ k = 3, 4, 56
b. B = 5a2 + b2 ƒ a = 2 ó 3 y b = 2, 3 ó 46

Solución a. Substituimos k = 3, 4, 5 en 2k + 1 y obtenemos los valores correspondientes


mostrados en la tabla 2-7. Así, A = 57, 9, 116.
Tabla 2-7
k 2k ⴙ 1
3 2#3 + 1 = 7
4 2#4 + 1 = 9
5 2 # 5 + 1 = 11
b. Aquí a = 2 ó 3 y b = 2, 3 ó 4. En la tabla 2-8 se muestran todas las combinacio-
nes posibles de a y b y los valores correspondientes de a2 + b2. Así, B = 58,
13, 20, 18, 256. Nota que el 13 aparece dos veces en la tabla, pero sólo una vez
en el conjunto; ¿por qué?
Tabla 2-8
b
a 2 3 4
2 2 2 2
2 2 + 2 = 8 2 + 3 = 13 2 + 42 = 20
2

3 32 + 22 = 13 32 + 32 = 18 32 + 42 = 25


Como se mencionó anteriormente, no importa el orden en que se listen. Si A y B son
iguales, lo cual se escribe A = B, entonces todo elemento de A es un elemento de B y todo
elemento de B es un elemento de A. Si A no es igual a B, lo escribimos A Z B.

Definición de conjuntos iguales


Dos conjuntos son iguales si, y sólo si, contienen exactamente los mismos elementos.

Correspondencia biunívoca, o uno a uno


Una de las herramientas más útiles en matemáticas es establecer una correspondencia biu-
nívoca, o uno a uno, entre dos conjuntos. Los conjuntos pueden ser iguales o no. Por ejem-
plo, considera el conjunto de personas P = 5Tomás, Daniel, Mari6 y el conjunto de los
carriles de natación C = 51, 2, 36. Supón que cada persona en P va a nadar en un carril
numerado 1, 2 ó 3, de manera que dos personas no pueden ocupar el mismo carril. Este pareo
entre personas y carriles es una correspondencia biunívoca, también llamada correspondencia
uno a uno. En la figura 2-9 se muestra una manera de exhibir una correspondencia mediante
flechas que conectan los elementos correspondientes.
P C

Tomás 1
Daniel 2
Mari 3

Figura 2-9

© Lopez Mateos Editores. ISBN 978-607-95583-2-1, obra completa, versión electrónica, ISBN 978-607-95583-3-8, volumen 1, versión electrónica. Ejemplar asignado a: Helecto Villarroel gutierrez -
helecto@gmail.com. Fecha: 27 de octubre de 2014. Prohibida su modificación, copia o distribución.
Sección 2-2 Descripción de conjuntos 81

Existen otras posibles correspondencias biunívocas entre los conjuntos P y C. Hay varias
maneras de exhibirlas. Por ejemplo, listamos a continuación las seis correspondencias biu-
nívocas entre los conjuntos P y C:

1. Tomás 4 1 2. Tomás 4 1 3. Tomás 4 2


Daniel 4 2 Daniel 4 3 Daniel 4 1
Mari 4 3 Mari 4 2 Mari 4 3
4. Tomás 4 2 5. Tomás 4 3 6. Tomás 4 3
Daniel 4 3 Daniel 4 1 Daniel 4 2
Mari 4 1 Mari 4 2 Mari 4 1

Nota que el listado en (1) y la figura 2-9 representan una sola correspondencia biunívoca
entre los conjuntos P y C. La correspondencia Tomás 4 1 también puede ser una corres-
pondencia biunívoca pero entre otros dos conjuntos, a saber, {Tomás} y {1}. El conjunto de
todas las correspondencias biunívocas entre los conjuntos P y C se muestra en la tabla 2-9.

Tabla 2-9
Carriles
Pareos 1 2 3

1. Tomás Daniel Mari


2. Tomás Mari Daniel
3. Daniel Tomás Mari
4. Daniel Mari Tomás
5. Mari Tomás Daniel
6. Mari Daniel Tomás

A continuación, la definición general de correspondencia biunívoca:

Definición de correspondencia biunívoca


Si los elementos de los conjuntos P y C se pueden parear de manera que a cada elemento de P le
corresponda exactamente un elemento de C y a cada elemento de C le corresponda exactamente
un elemento de P, entonces se dice que los dos conjuntos, P y C, están en correspondencia bi-
unívoca.

AHORA INTENTA ÉSTE 2-5  Considera un conjunto de cuatro personas 5A, B, C, D6 y un conjunto de
cuatro carriles para nadar 51, 2, 3, 46.
a. Exhibe todas las correspondencias biunívocas entre los dos conjuntos.
b. ¿Cuántas de dichas correspondencias biunívocas hay?
c. Halla el número de correspondencias biunívocas entre dos conjuntos con cinco elementos cada uno, y
explica tu razonamiento.

También podemos usar un diagrama de árbol para listar las posibles correspondencias
biunívocas, como se muestra en la figura 2-10. Para leer el diagrama de árbol y ver la co-
rrespondencia biunívoca, seguimos cada rama. La persona que ocupa un carril específico en

© Lopez Mateos Editores. ISBN 978-607-95583-2-1, obra completa, versión electrónica, ISBN 978-607-95583-3-8, volumen 1, versión electrónica. Ejemplar asignado a: Helecto Villarroel gutierrez -
helecto@gmail.com. Fecha: 27 de octubre de 2014. Prohibida su modificación, copia o distribución.
82 Sistemas de numeración y conjuntos

una correspondencia se lista debajo del número del carril. Por ejemplo, la rama superior
ilustra el pareo (Tomás, 1), (Daniel, 2) y (Mari, 3).
Carril Carril Carril
1 2 3
Daniel Mari

Tomás

Mari Daniel

Tomás Mari

Daniel

Mari Tomás

Tomás Daniel

Mari

Daniel Tomás

Figura 2-10

Observa en la figura 2-10 que al asignar a un nadador el carril 1, podemos colocar a alguna
de las tres personas: Tomás, Daniel o Mari. Si ponemos a Tomás en el carril 1, entonces él
no podrá estar en el carril 2 y, en consecuencia, el segundo carril deberá estar ocupado ya sea
por Daniel o por Mari. Asimismo, vemos que si Daniel está en el carril 1, entonces hay dos
maneras de escoger a alguien para el carril 2: Tomás o Mari. De manera análoga, si Mari está
en el carril 1 hay, de nuevo, dos maneras de escoger a alguien para el segundo carril: Tomás
o Daniel. Así, para cada una de las tres maneras en que podemos ocupar el primer carril, hay
dos maneras subsecuentes de ocupar el segundo carril y hay, por lo tanto, 2 + 2 + 2, ó 3 # 2,
ó 6 maneras de colocar a los nadadores en los primeros dos carriles. Nota que para cada uno
de los arreglos de los nadadores en los dos primeros carriles, sólo queda un posible nadador
para ocupar el tercer carril. Esto es, si Mari ocupa el primer carril y Daniel el segundo, en-
tonces Tomás debe ocupar el tercero. Así, el número total de arreglos para los tres nadadores
es igual a 3 # 2, ó 6.
Se puede usar un razonamiento similar para hallar cuántos arreglos de sabores de helados
es posible colocar en un barquillo donde quepan dos bolas si disponemos de diez sabores.
Ahora, si consideramos que chocolate y vainilla (primero se coloca la bola de chocolate y
encima la de vainilla) es diferente de vainilla y chocolate (primero se coloca la bola de vaini-
lla y encima la de chocolate) y permitimos que haya dos bolas del mismo sabor, podemos
proceder como sigue. Hay diez maneras de escoger la primera bola de helado, y para cada
selección hay 10 maneras subsecuentes de escoger la segunda bola. Así, el número total de
arreglos es de 10 # 10, ó 100.
El argumento usado para hallar el número de las posibles correspondencias biunívocas en-
tre el conjunto de nadadores y el conjunto de carriles, así como el problema anterior de los
arreglos de las bolas de helado, son ejemplos del Principio Fundamental del Conteo.

Teorema 2–1:    Principio Fundamental del Conteo


Si un evento M puede ocurrir de m maneras y, después de ocurrido, el evento N puede ocurrir
de n maneras, entonces el evento M seguido del evento N puede ocurrir de mn maneras.

© Lopez Mateos Editores. ISBN 978-607-95583-2-1, obra completa, versión electrónica, ISBN 978-607-95583-3-8, volumen 1, versión electrónica. Ejemplar asignado a: Helecto Villarroel gutierrez -
helecto@gmail.com. Fecha: 27 de octubre de 2014. Prohibida su modificación, copia o distribución.
Sección 2-2 Descripción de conjuntos 83

AHORA INTENTA ÉSTE 2-6 Explica cómo es posible extender el Principio Fundamental del Conteo a
cualquier número de eventos.

Conjuntos equivalentes
Estrechamente relacionado con las correspondencias biunívocas está el concepto de con-
juntos equivalentes. Por ejemplo, supón que hay 20 sillas en una habitación y un estu-
diante está sentado en cada una, sin que nadie quede de pie. Hay una correspondencia
biunívoca entre el conjunto de sillas y el conjunto de los estudiantes en la habitación. En
este caso el conjunto de sillas y el conjunto de estudiantes son conjuntos equivalentes.

Definición de conjuntos equivalentes


Dos conjuntos A y B son equivalentes —se escribe A ' B— si y sólo si existe una
correspondencia biunívoca entre los conjuntos.

No debemos confundir el término equivalente con el término igual. En el ejemplo 2-8 se


verá la diferencia.

Ejemplo 2-8 Sea


A = 5p, q, r, s6, B = 5a, b, c6, C = 5x, y, z6, y D = 5b, a, c6.








Compara los conjuntos usando los términos igual y equivalente.

Solución Cada conjunto es igual, y es equivalente, a sí mismo.


Los conjuntos A y B no son equivalentes 1A ⬃ B2 y no son iguales 1A Z B2.
Los conjuntos A y C no son equivalentes 1A ⬃ C2 y no son iguales 1A Z C2.
Los conjuntos A y D no son equivalentes 1A ⬃ D2 y no son iguales 1A Z D2.
Los conjuntos B y C son equivalentes 1B ' C2 pero no iguales 1B Z C2.
Los conjuntos B y D son equivalentes 1B ' D2 e iguales 1B = D2.
Los conjuntos C y D son equivalentes 1C ' D2 pero no iguales 1C Z D2.

AHORA INTENTA ÉSTE 2-7


a. Si dos conjuntos son equivalentes, ¿necesariamente son iguales? Explica por qué sí o por qué no.
b. Si dos conjuntos son iguales, ¿necesariamente son equivalentes? Explica por qué sí o por qué no.

Números cardinales
El concepto de correspondencia biunívoca se puede usar para introducir el concepto de dos
conjuntos con el mismo número de elementos. Sin saber contar, un niño podría compren-
der que tiene tantos dedos en la mano izquierda como en la otra mano colocando los dedos
de una mano sobre los de la otra, como en la figura 2-11. Al colocar de manera natural los
dedos de modo que el pulgar izquierdo toque el pulgar derecho, el índice izquierdo toque
© Lopez Mateos Editores. ISBN 978-607-95583-2-1, obra completa, versión electrónica, ISBN 978-607-95583-3-8, volumen 1, versión electrónica. Ejemplar asignado a: Helecto Villarroel gutierrez -
helecto@gmail.com. Fecha: 27 de octubre de 2014. Prohibida su modificación, copia o distribución.
84 Sistemas de numeración y conjuntos

el índice derecho y así sucesivamente, exhibimos una correspondencia biunívoca entre los
dedos de las dos manos. De manera análoga, sin contar, los niños comprenden que si todos
los alumnos de un grupo se sientan en una silla y no hay sillas vacías, entonces hay tantas si-
llas como alumnos y viceversa.

Figura 2-11

La correspondencia biunívoca entre conjuntos ayuda a explicar el concepto de número.


Considera los cinco conjuntos 5a, b6, 5p, q6, 5x, y6, 5b, a6 y {*, #}; son conjuntos equivalen-
tes entre sí, comparten la propiedad de “ser dos”; es decir, estos conjuntos tienen el mismo
número cardinal, a saber, 2. El número cardinal de un conjunto C, denotado con n1C2 , in-
dica el número de elementos que están en el conjunto C. Si C = 5a, b6, el número cardinal
de C es 2 y se escribe como n1C2 = 2. Si dos conjuntos, A y B, son equivalentes, entonces A y
B tienen el mismo número cardinal, esto es, n1A2 = n1B2.
Un conjunto que no tiene elementos tiene número cardinal 0 y es un conjunto vacío o
nulo. El conjunto vacío se designa por medio del símbolo ⭋ o 5 6. A continuación presen-
tamos dos ejemplos de conjuntos sin elementos:
C = 5x ƒ x era un estado de México antes de 1200 a.c.6
D = 5x ƒ x es un número natural menor que 16

O B S E R VA C I Ó N El conjunto vacío a menudo se escribe, de manera incorrecta, como


5⭋6. Este conjunto no es vacío pues contiene un elemento. Asimismo, 506 no repre-
senta al conjunto vacío. ¿Por qué?
Un conjunto es finito si su número cardinal es cero o un número natural. El conjunto N
de los números naturales es un conjunto infinito; no es finito. El conjunto E que contiene
todos los números naturales y el 0 es el de los números completos E = 50, 1, 2, 3, Á 6. E
es un conjunto infinito.
La siguiente tira cómica de “Peanuts” ilustra cómo se relacionan los conceptos de la
teoría de conjuntos con la suma, aunque no debe esperarse que un niño sepa estos concep-
tos para poder sumar 2 más 2.

“CONJUNTOS"..... “CONJUNTOS EQUIVALENTES”.... “RENOMBRANDO DOS”... TODO LO QUE QUIERO


“CONJUNTOS NO EQUIVALENTES”.... “SUBCONJUNTOS”....“UNIÓN DE
“CORRESPONDEN- SABER ES ¿CUÁNTO ES DOS
“CONJUNTOS DE UNO”...“CONJUNTOS CONJUNTOS”....“EXPRESIONES
CIA UNO A MÁS DOS?
DE DOS”.. NUMÉRICAS”... “VARIABLES”...
UNO"..

© Lopez Mateos Editores. ISBN 978-607-95583-2-1, obra completa, versión electrónica, ISBN 978-607-95583-3-8, volumen 1, versión electrónica. Ejemplar asignado a: Helecto Villarroel gutierrez -
helecto@gmail.com. Fecha: 27 de octubre de 2014. Prohibida su modificación, copia o distribución.
Sección 2-2 Descripción de conjuntos 85

AHORA INTENTA ÉSTE 2-8 Usa el razonamiento para explicar por qué no puede haber un número na-
tural que sea el mayor. Esto es, explica por qué el conjunto de números naturales no es un conjunto finito,
como lo sugiere Dolly en la tira cómica a continuación.

El CIRCO FAMILIAR Por Bil Keaene

“El alfabeto termina en la ‘Z’, pero los


números continúan por siempre.”

Más acerca de conjuntos


El conjunto universal o universo, denotado con U, es el conjunto que contiene todos
los elementos considerados en una situación determinada. Supón que U = 5x ƒ x es una
persona que vive en Caracas6 y M = 5x ƒ x es una mujer que vive en Caracas6. El conjun-
to universal, U, y el conjunto M se pueden representar por medio de un diagrama, como en
la figura 2-12(a). El conjunto universal se representa con un rectángulo grande y M se in-
dica por medio de un círculo dentro del rectángulo, como se muestra en la figura 2-12(a).
Esta figura es un ejemplo de diagrama de Venn, llamado así en honor del inglés John
Venn (1834–1923), quien usó dichos diagramas para ilustrar ideas en lógica. El conjunto de
los elementos en el universo que no están en M, denotado con M, es el conjunto de los
varones que viven en Caracas, y es el complemento de M. En la figura 2-12(b) se repre-
senta por medio de la región sombreada .

U U

M M M

(a) (b)

Figura 2-12

Definición de complemento de un conjunto


El complemento de un conjunto M, que escribimos como M, es el conjunto de todos los
elementos del conjunto universal U que no están en M; esto es, M = 5x ƒ x 僆 U y x 僆 M6.

© Lopez Mateos Editores. ISBN 978-607-95583-2-1, obra completa, versión electrónica, ISBN 978-607-95583-3-8, volumen 1, versión electrónica. Ejemplar asignado a: Helecto Villarroel gutierrez -
helecto@gmail.com. Fecha: 27 de octubre de 2014. Prohibida su modificación, copia o distribución.
86 Sistemas de numeración y conjuntos

Ejemplo 2-9 a. Si U = 5a, b, c, d6 y B = 5c, d6, halla (i) B; (ii) U; (iii) ⭋.


b. Si U = 5x ƒ x es un animal del zoológico6 y S = 5x ƒ x es una serpiente del zoológico6,









pp describe S.
c. Si U = N, P = 52, 4, 6, 8, Á 6, e I = 51, 3, 5, 7, Á 6, halla (i) P y (ii) I.

Solución a. (i) B = 5a, b6; (ii) U = ⭋; (iii) ⭋ = U


b. Como no sabemos cuáles son los animales del zoológico, S debe describirse
usando la notación constructora de conjuntos:
S = 5x ƒ x es un animal del zoológico que no es una serpiente6
c. (i) P = I; (ii) I = P

Subconjuntos
Considera los conjuntos A = 51, 2, 3, 4, 5, 66 y B = 52, 4, 66. Todos los elementos de B
están contenidos en A. Decimos entonces que B es un subconjunto de A y lo escribimos
B 8 A. En general, tenemos la siguiente definición:

Definición de subconjunto
B es un subconjunto de A, se escribe B 8 A, si y sólo si todo elemento de B es un elemento de A.

Esta definición permite que B sea igual a A. La definición se escribió con la frase “si y sólo
si”, que significa que “si B es un subconjunto de A, entonces todo elemento de B es un ele-
mento de A, y si todo elemento de B es un elemento de A, entonces B es un subconjunto de
A”. Si suceden tanto A 8 B como B 8 A, entonces A = B.
Cuando un conjunto A no es subconjunto de otro conjunto B, escribimos A h B. Para
mostrar que A h B, debemos hallar al menos un elemento de A que no esté en B. Si
A = 51, 3, 56 y B = 51, 2, 36, entonces A no es un subconjunto de B pues 5 es un ele-
mento de A que no está en B. Asimismo, B h A pues 2 pertenece a B pero no está en A.
No es obvio que el conjunto vacío cumpla la definición de subconjunto pues no hay
elementos del conjunto vacío que sean elementos de otro conjunto. Para analizar este
problema usamos las estrategias de razonamiento indirecto y examen de un caso particular.
Para el conjunto 51, 26, ha de suceder que ⭋ 8 51, 26 ó ⭋ h 51, 26. Supón que
⭋ h 51, 26. Entonces debe haber algún elemento de ⭋ que no esté en 51, 26. Como el con-
junto vacío no tiene elementos, no puede haber un elemento en el conjunto vacío que no esté
en 51, 26. En consecuencia, ⭋ h 51, 26 es falso. Por lo tanto la única otra posibilidad,
⭋ 8 51, 26, es verdadera. Se puede aplicar el mismo razonamiento en el caso del conjunto
vacío o de cualquier otro conjunto.
Si B es un subconjunto de A y B no es igual a A, entonces B es un subconjunto propio
de A, que se escribe B ( A. Esto significa que todo elemento de B está contenido en A y que
hay al menos un elemento de A que no está en B. A veces se usa un diagrama de Venn para
indicar un subconjunto propio, como en la figura 2-13, mostrando un punto (un elemento)
de A que no está en B.
U

A
B

Figura 2-13

© Lopez Mateos Editores. ISBN 978-607-95583-2-1, obra completa, versión electrónica, ISBN 978-607-95583-3-8, volumen 1, versión electrónica. Ejemplar asignado a: Helecto Villarroel gutierrez -
helecto@gmail.com. Fecha: 27 de octubre de 2014. Prohibida su modificación, copia o distribución.
Sección 2-2 Descripción de conjuntos 87

Ejemplo 2-10 Dados A = 51, 2, 3, 4, 56, B = 51, 36, P = 5x ƒ x = 2n - 1, donde n 僆 N6:











a. ¿Cuáles conjuntos son subconjuntos de cuáles?
b. ¿Cuáles subconjuntos son subconjuntos propios de cuáles?
c. Si C = 52k ƒ k 僆 N6 y D = 54k ƒ k 僆 N6, muestra que uno de los conjuntos es
subconjunto del otro.

Solución a. Como 21 - 1 = 1, 22 - 1 = 3, 23 - 1 = 7, 24 - 1 = 15, 25 - 1 = 31, y así


sucesivamente, P = 51, 3, 7, 15, 31, Á 6. Así, B 8 P. También B 8 A, A 8 A,
B 8 B y P 8 P.
b. B ( A y B ( P
c. Como 4k = 212k2, cada elemento de D es un elemento de C. Así, D 8 C.

AHORA INTENTA ÉSTE 2-9


a. Supón que A ( B. ¿Podemos concluir siempre que A 8 B?
b. Si A 8 B, ¿se sigue que A ( B?

A menudo se confunden los subconjuntos y los elementos de un conjunto. Decimos


que 2 僆 51, 2, 36. Pero como 2 no es un conjunto, no podemos substituir el símbolo 8
en lugar de 僆. Sin embargo, 526 8 51, 2, 36 y 526 ( 51, 2, 36. Nota que el símbolo 僆 no
puede colocarse entre 526 y 51, 2, 36 y obtener una proposición verdadera.

AHORA INTENTA ÉSTE 2-10 Convence a una compañera de clase de que lo siguiente es verdadero:
a. El conjunto vacío es un subconjunto de sí mismo.
b. El conjunto vacío no es un subconjunto propio de sí mismo.

Desigualdades: aplicación de los conceptos de conjuntos


El concepto de subconjunto propio y de correspondencia biunívoca se pueden usar para defi-
nir el concepto de “menor que” entre los números naturales. El conjunto 5a, b, c6 tiene me-
nos elementos que el conjunto 5w, x, y, z6 pues al tratar de parear los elementos de los dos
conjuntos, como en
5a, b, c6

5x, y, z, w6
vemos que hay un elemento del segundo conjunto al que no le corresponde uno del primer
conjunto. El conjunto 5a, b, c6 es equivalente a un subconjunto propio de 5x, y, z, w6.
En general, si A y B son conjuntos finitos, A tiene menos elementos que B si A es equivalente a
un subconjunto propio de B. Decimos entonces que n1A2 es menor que n1B2 y lo escribimos
n1A2 6 n1B2. Decimos que a es mayor que b, y lo escribimos a 7 b, si y sólo si, b 6 a. El
concepto de “menor o igual que” se define de manera análoga y se explora en la Evaluación
2-2A y 2-2B.
Hemos visto que si A y B son conjuntos finitos y A ( B, entonces A tiene menos elemen-
tos que B y no es posible hallar una correspondencia biunívoca entre los conjuntos. En con-
secuencia, A y B no son equivalentes. Sin embargo, cuando ambos conjuntos son infinitos y
A ( B, los conjuntos podrían ser equivalentes. Por ejemplo, considera el conjunto N de nú-
© Lopez Mateos Editores. ISBN 978-607-95583-2-1, obra completa, versión electrónica, ISBN 978-607-95583-3-8, volumen 1, versión electrónica. Ejemplar asignado a: Helecto Villarroel gutierrez -
helecto@gmail.com. Fecha: 27 de octubre de 2014. Prohibida su modificación, copia o distribución.
88 Sistemas de numeración y conjuntos

meros naturales y el conjunto P de números naturales pares. Tenemos que P ( N, pero, aun
◆ Nota así, podemos hallar una correspondencia biunívoca entre los conjuntos. Para ello, hacemos
de investigación corresponder cada número en el conjunto N con el doble del número en el conjunto P. Esto
Tsamir y Triosh es, a n 僆 N le corresponde 2n 僆 P, como se muestra a continuación:
N = 51, 2, 3, 4, 5, Á , n, Á 6
(1999) reportaron que
con conjuntos infini-
tos, los estudiantes a

P = 52, 4, 6, 8, 10, Á , 2n, Á 6


menudo se alteran
con conclusiones con-
tradictorias y no usan Nota que en la correspondencia anterior, a cada elemento de N le corresponde un solo ele-
correspondencias bi- mento de P y, recíprocamente, a cada elemento de P le corresponde un solo elemento de N.
unívocas. Estos resul- Por ejemplo, al 11 en N le corresponde el 2 # 11, ó 22, en P. Y a 100 en P le corresponde el
tados no fueron 100 , 2, ó 50, en N. Así, N ' P; Esto es, N y P son equivalentes.
inconsistentes con el A menudo los estudiantes tienen dificultades con los conjuntos infinitos y especialmente con
razonamiento sus números cardinales, llamados números transfinitos. Como se mostró anteriormente, P es
matemático de antes un subconjunto propio de N, pero como se pueden poner en correspondencia biunívoca, son
del trabajo de equivalentes y tienen el mismo número cardinal. Tsamir y Triosh hallaron que las representa-
Cantor. ◆ ciones de los conjuntos infinitos causaron problemas, como se ve en la Nota de investigación.

Resolver problemas Aprobación de una medida


Una comisión de senadores está formada por Arroyo, Barragán, Cruz y Díaz. Supón que
cada miembro de la comisión tiene un voto y que sólo se requiere mayoría simple para
aprobar o rechazar una medida. Una medida que no pasa ni se rechaza se considera blo-
queada y deberá votarse de nuevo. Determina el número de maneras en que una medida
podría aprobarse o rechazarse, y el número de maneras en que una medida puede
bloquearse.

Comprender el problema Se nos pide determinar de cuántas maneras un comité de cuatro


miembros puede aprobar o rechazar una propuesta y de cuántas maneras ese comité puede
bloquear una propuesta. Para aprobar o rechazar una propuesta se requiere una coalición
ganadora, esto es, un grupo de senadores que pueda aprobar o rechazar la propuesta inde-
pendientemente de lo que hagan los otros. Para bloquear una propuesta debe haber una
coalición bloqueadora, esto es, un grupo que impida que pase la proposición, pero que no
pueda rechazarla.

Trazar un plan Para resolver el problema podemos hacer una lista de subconjuntos del
conjunto de senadores. Cualquier subconjunto del conjunto de senadores con tres o cua-
tro elementos formará una coalición ganadora. Cualquier subconjunto de senadores con
precisamente dos elementos formará una coalición bloqueadora.

Realizar el planListamos todos los subconjuntos del conjunto S = 5Arroyo, Barragán,


Cruz, Díaz6 que tengan al menos tres elementos y todos los subconjuntos que tengan exac-
tamente dos elementos. Por conveniencia, identificamos los elementos como sigue: A—
Arroyo, B—Barragán, C—Cruz, D—Díaz. A continuación damos todos los subconjuntos:
⭋ 5A6 5A, B6 5A, B, C6 5A, B, C, D6
5B6 5A, C6 5A, B, D6
5C6 5A, D6 5A, C, D6
5D6 5B, C6 5B, C, D6
5B, D6
5C, D6
© Lopez Mateos Editores. ISBN 978-607-95583-2-1, obra completa, versión electrónica, ISBN 978-607-95583-3-8, volumen 1, versión electrónica. Ejemplar asignado a: Helecto Villarroel gutierrez -
helecto@gmail.com. Fecha: 27 de octubre de 2014. Prohibida su modificación, copia o distribución.
Sección 2-2 Descripción de conjuntos 89

Hay cinco subconjuntos con al menos tres miembros que pueden formar una coalición
ganadora y aprobar o rechazar una medida, y seis subconjuntos con exac tamente dos
miembros que pueden bloquear una medida.

Revisar Se pueden considerar otras cuestiones, como:


1. ¿Cuántas coaliciones ganadoras mínimas hay? En otras palabras, ¿cuántos subconjun-
tos hay de los cuales ningún subconjunto propio puede aprobar una medida?
2. Diseña un método para resolver este problema sin listar todos los subconjuntos.
3. En la parte de “Realizar el Plan” se listaron 16 subconjuntos de 5A, B, C, D6. Usa ese
resultado para listar, de manera sistemática, todos los subconjuntos de un comité de
cinco senadores. ¿Puedes hallar el número de subconjuntos de un comité de cinco
◆ miembros sin tener que contar los subconjuntos?

AHORA INTENTA ÉSTE 2-11 Supongamos que un comité de senadores consta de cinco miembros.
a. Compara el número de coaliciones ganadoras que tengan exactamente cuatro miembros con el
número de senadores del comité. ¿Por qué razón obtenemos este resultado?
b. Compara el número de coaliciones ganadoras de exactamente tres miembros con el número de subcon-
juntos del comité que tienen exactamente dos miembros. ¿Por qué razón obtenemos este resultado?

Número de subconjuntos de un conjunto


¿Cuántos subconjuntos se pueden formar de un conjunto que tiene n elementos? Para
obtener una fórmula general, usamos primero la estrategia de intentar un caso más sencillo.
1. Si P = 5a6, entonces P tiene dos subconjuntos, ⭋ y 5a6.
2. Si Q = 5a, b6, entonces Q tiene cuatro subconjuntos, ⭋, 5a6, 5b6 y 5a, b6.
3. Si R = 5a, b, c6, entonces R tiene ocho subconjuntos, ⭋, 5a6, 5b6, 5c6, 5a, b6,
5a, c6, 5b, c6 y 5a, b, c6.
Una estrategia alternativa para listar el número de subconjuntos de un conjunto dado
consiste en usar un diagrama de árbol. Por ejemplo, en la figura 2-14 se ilustran los diagra-
mas de árbol de los subconjuntos de Q = 5a, b6 y R = 5a, b, c6.

a∈Q b∈Q Subconjuntos a∈Q b∈Q c∈Q Subconjuntos


Sí {a, b} Sí {a, b, c}
Sí Sí
No {a, b}
No {a} Sí
Sí {a, c}
Sí {b} No
No No {a}
No Ø Sí {b, c}

No {b}
No
Sí {c}
No
No Ø
(a) (b)

Figura 2-14

Usando la información proporcionada por estos casos, hacemos una tabla para buscar patro-
nes, como en la tabla 2-10.

© Lopez Mateos Editores. ISBN 978-607-95583-2-1, obra completa, versión electrónica, ISBN 978-607-95583-3-8, volumen 1, versión electrónica. Ejemplar asignado a: Helecto Villarroel gutierrez -
helecto@gmail.com. Fecha: 27 de octubre de 2014. Prohibida su modificación, copia o distribución.
90 Sistemas de numeración y conjuntos

Tabla 2-10
Número de elementos Número de subconjuntos
1 2, ó 21
2 4, ó 22
3 8, ó 23
# #
# #
# #

La tabla 2-10 sugiere que para cuatro elementos podría haber 24, ó 16, subconjuntos. ¿Es
correcta esta suposición? Si S = 5a, b, c, d6, entonces todos los subconjuntos de
R = 5a, b, c6 también son subconjuntos de S. Se forman ocho nuevos subconjuntos al agre-
gar el elemento d a cada uno de los ocho subconjuntos de R. Los ocho nuevos subconjuntos
son 5d6, 5a, d6, 5b, d6, 5c, d6, 5a, b, d6, 5a, c, d6, 5b, c, d6 y 5a, b, c, d6. Así, hay el doble de
subconjuntos del conjunto S (con cuatro elementos) que de subconjuntos del conjunto R
(con tres elementos). En consecuencia, hay 2 # 8, ó 24, subconjuntos de un conjunto con
cuatro elementos.. Debido a que al incluir un elemento más en un conjunto finito se du-
plica el número de posibles subconjuntos del nuevo conjunto, un conjunto con 5 elementos
tendrá 2 # 24, ó 25, subconjuntos, y así sucesivamente. En cada caso, el número de elementos
y la potencia de 2 usada para obtener el número de subconjuntos son iguales. Por lo tanto, si
hay n elementos en un conjunto, se podrán formar 2n subconjuntos. Si aplicamos el resultado an-
terior al conjunto vacío —esto es, cuando n = 0— tenemos entonces que 20 = 1. El patrón
tiene sentido pues sólo hay un subconjunto del conjunto vacío: él mismo.

AHORA INTENTA ÉSTE 2-12


a. ¿Cuántos subconjuntos propios tiene un conjunto de cuatro elementos?
b. ¿Cuántos subconjuntos propios tiene un conjunto de n elementos?

Evaluación 2-2A

1. Escribe los conjuntos siguientes listando sus elementos o b. 5a, b, c, d, e, f, Á , m6 y 51, 2, 3, 4, 5, 6, Á , 136
usando la notación constructora de conjuntos: c. 5x ƒ x es una letra de la palabra matemáticas6 y
a. El conjunto de las letras en la palabra matemáticas 51, 2, 3, 4, Á , 116
b. El conjunto de los números naturales mayores que 20 4. ¿Cuántas correspondencias biunívocas hay entre dos
2. Reescribe lo siguiente usando símbolos matemáticos: conjuntos con
a. P es igual al conjunto cuyos elementos son a, b, c y d. a. 6 elementos cada uno?
b. El conjunto formado por los elementos 1 y 2 es un b. n elementos cada uno?
subconjunto propio de 51, 2, 3, 46. 5. ¿Cuántas correspondencias biunívocas hay entre los
c. El conjunto formado por los elementos 0 y 1 no es un conjuntos 5x, y, z, u, v6 y 51, 2, 3, 4, 56 si en cada co-
subconjunto propio de 51, 2, 3, 46. rrespondencia
d. 0 no es un elemento del conjunto vacío. a. x debe corresponderse con 5?
3. De los siguientes pares de conjuntos, ¿cuáles se pueden b. x debe corresponderse con 5 y y con 1?
colocar en correspondencia biunívoca? c. x, y y z deben corresponderse con números impares?
a. 51, 2, 3, 4, 56 y 5m, n, o, p, q6

© Lopez Mateos Editores. ISBN 978-607-95583-2-1, obra completa, versión electrónica, ISBN 978-607-95583-3-8, volumen 1, versión electrónica. Ejemplar asignado a: Helecto Villarroel gutierrez -
helecto@gmail.com. Fecha: 27 de octubre de 2014. Prohibida su modificación, copia o distribución.
Sección 2-2 Descripción de conjuntos 91

6. ¿Cuáles de los siguientes casos representan conjuntos 11. Indica qué símbolo, 僆 o 僆, hace que las proposiciones
iguales? siguientes sean verdaderas:
A = 5a, b, c, d6 B = 5x, y, z, w6 a. 0 ⭋
C = 5c, d, a, b6 D = 5x ƒ 1 … x … 4 donde x 僆 N6 b. 516 51, 26
E = ⭋ F = 5⭋6 c. 1024 5x ƒ x = 2n donde n 僆 N6
G = 506 H = 56 d. 3002 5x ƒ x = 3n - 1 donde n 僆 N 6
I = 5x ƒ x = 2n+1 donde n 僆 C6 y C = 50, 1, 2, 3, Á 6 12. Indica qué símbolo, 8 o h , hace que cada parte del
J = 5x ƒ x = 2n - 1 donde n 僆 N6 problema 11 sea verdadera.
7. Halla el número cardinal de cada uno de los siguientes 13. Responde lo siguiente. Si tu respuesta es no, di por qué.
conjuntos: a. Si A = B, ¿siempre podemos concluir que A 8 B?
a. 5101, 102, 103, Á , 11006 b. Si A 8 B, ¿siempre podemos concluir que A ( B?
b. 51, 3, 5, Á , 10016 c. Si A ( B, ¿siempre podemos concluir que A 8 B?
c. 51, 2, 4, 8, 16, Á , 10246 d. Si A 8 B, ¿siempre podemos concluir que A = B?
d. 5x ƒ x = k2 donde k = 1, 2, 3, Á , ó 1006 14. Usa la definición de menor que para demostrar lo si-
e. 5i + j ƒ i 僆 51, 2, 36 y j 僆 51, 2, 366 guiente:
8. Si U es el conjunto de todos los estudiantes de secundaria y a. 3 6 100
A es el conjunto de los estudiantes de secundaria con pro- b. 0 6 3
medio de 10, describe A. 15. En cierto comité del senado hay siete senadores: Arana,
9. Supón que B es un subconjunto propio de C. Bedolla, Cuevas, Domínguez, Estrada, Fabela y García.
a. Si n1C2 = 8, ¿cuál es el máximo número de elementos Van a citar a tres de ellos para formar un subcomité.
en B? ¿Cuántos subcomités posibles hay?
b. ¿Cuál es el menor número posible de elementos en B? 16. ¿Cuántos números de dos dígitos en base diez pueden
10. Supón que C es un subconjunto de D y D es un subcon- formarse de modo que el dígito de las decenas no sea 0
junto de C. y ningún dígito se repita?
a. Si n1C2 = 5, halla n1D2.
b. ¿Qué otra relación existe entre los conjuntos C y D?

Evaluación 2-2B

1. Escribe los conjuntos siguientes listando sus elementos o 5. ¿Cuántas correspondencias biunívocas hay entre los
usando la notación constructora de conjuntos: conjuntos 5a, b, c, d6 y 51, 2, 3, 46 si en cada correspon-
a. El conjunto de las letras en la palabra geometría dencia
b. El conjunto de los números naturales mayores que 7 a. b debe corresponderse con 3?
2. Reescribe lo siguiente usando símbolos matemáticos: b. b debe corresponderse con 3 y d con 4?
a. Q es igual al conjunto cuyos elementos son a, b y c. c. a y c deben corresponderse con números pares?
b. El conjunto formado por los elementos 1 y 3 es un 6. ¿Cuáles conjuntos representan conjuntos diferentes?
subconjunto propio de los números naturales. A = 5a, b, c, d6 B = 5x, y, z, w6
c. El conjunto formado por los elementos 1 y 3 no es un C = 5c, d, a, b6 D = 5x ƒ 1 … x … 4 donde x 僆 N6
subconjunto propio de 51, 4, 6, 76 E = ⭋ F = 5⭋6
d. El conjunto vacío no contiene al 0 como elemento. G = 506 H = 56
3. De los siguientes pares de conjuntos, ¿cuáles se pueden I = 5x ƒ x = 2n + 1 donde n 僆 W6, y
colocar en correspondencia biunívoca? W = 50, 1, 2, 3, Á 6
a. 51, 2, 3, 46 y 5w, c, y, z6 J = 5x ƒ x = 2n - 1 donde n 僆 N6
b. 51, 2, 3, Á , 256 y 5a, b, c, d, Á , x, y6 7. Halla el número cardinal de cada uno de los siguientes
c. 5x ƒ x es una letra de la palabra geometría6 y conjuntos:
51, 2, 3, 4, 5, 6, 7, 86 a. 59, 10, 11, Á , 996
4. ¿Cuántas correspondencias biunívocas hay entre dos b. 52, 4, 6, 8, Á , 20026
conjuntos con c. 50, 1, 3, 7, 15, Á , 10236
a. 8 elementos cada uno? d. 5x2 ƒ x = 1, 3, 5, 7, Á , ó 996
b. n - 1 elementos cada uno? e. 5i # j ƒ i 僆 51, 2, 36 y j 僆 51, 2, 366

© Lopez Mateos Editores. ISBN 978-607-95583-2-1, obra completa, versión electrónica, ISBN 978-607-95583-3-8, volumen 1, versión electrónica. Ejemplar asignado a: Helecto Villarroel gutierrez -
helecto@gmail.com. Fecha: 27 de octubre de 2014. Prohibida su modificación, copia o distribución.
92 Sistemas de numeración y conjuntos

8. Si U es el conjunto de todas las mujeres y M es el con- 13. Responde lo siguiente. Si tu respuesta es no, di por qué.
junto de las alumnas de la Universidad Nacional Autó- a. Si A 8 B, ¿siempre podemos concluir que A = B?
noma de México, describe M . b. Si A ( B, ¿podemos concluir que A = B?
9. Supón que A 8 B. c. Si A y B se pueden colocar en correspondencia biuní-
a. ¿Cuál es el número mínimo de elementos que puede voca, ¿entonces necesariamente A = B?
haber en el conjunto A? d. Si A y B se pueden colocar en correspondencia biuní-
b. ¿Es posible que B sea el conjunto vacío? De ser así, da voca, ¿entonces necesariamente A 8 B?
un ejemplo de conjuntos A y B que satisfagan esto. 14. Usa la definición de menor que para demostrar lo siguiente:
En caso contrario, explica por qué. a. 0 6 2
10. Si dos conjuntos son subconjuntos uno de otro, ¿qué b. 99 6 100
otra relación hay, necesariamente, entre ellos? 15. ¿Cuántas maneras hay de servir un cono de helado con 4
11. Indica qué símbolo, 僆 o 僆, hace que las proposiciones bolas si se puede escoger entre
siguientes sean verdaderas: a. vainilla, chocolate, pistache y fresa, y cada bola debe
a. ⭋ ____ ⭋ ser diferente?
b. 526 ____ 53, 2, 16 b. vainilla, chocolate, pistache y fresa, y no hay la res-
c. 1022 ____ 5s ƒ s = 2n - 2 donde n es un elemento tricción de que las bolas deben ser diferentes?
de N6 16. ¿Cuántos números de teléfono de siete dígitos hay que
d. 3004 ____ 5x ƒ x = 3n + 1 donde n es un número no empiecen con 0 ó 1?
natural6
12. Indica qué símbolo, 8 ó h , hace que cada parte del
problema 11 sea verdadera.

Conexiones matemáticas 2-2

Comunicación 9. Halla un conjunto infinito A tal que


1. Explica la diferencia entre un conjunto bien definido y a. A es finito.
otro que no lo está. Da ejemplos. b. A es infinito.
2. ¿Cuál de los siguientes conjuntos no está bien definido? 10. Describe dos conjuntos partiendo de situaciones de la vida
Explica. real de manera que quede claro, a partir del uso de la co-
a. El conjunto de maestras de escuela ricas rrespondencia biunívoca y no de contar, que un conjunto
b. El conjunto de grandes libros tiene menos elementos que el otro.
c. El conjunto de números naturales mayores que 100 Aprendizaje colectivo
d. El conjunto de subconjuntos de 51, 2, 3, 4, 5, 66 11. a. Usa una calculadora, si es necesario, para estimar el
e. El conjunto 5x ƒ x Z x y x 僆 N6 tiempo en años que tardaría una computadora en lis-
3. ¿Es ⭋ un subconjunto propio de cualquier conjunto no tar todos los subconjuntos de 51, 2, 3, Á , 646. Su-
vacío? Explica tu razonamiento. pón que la computadora más rápida puede listar un
4. Explica por qué ⭋ es un elemento de {⭋} y también es un subconjunto en aproximadamente 1 microsegundo
subconjunto. (una millonésima de segundo).
5. Explica cómo demostrarías que A h B . b. Estima el tiempo en años que le llevaría a la computa-
6. Explica por qué todo conjunto es subconjunto de sí mismo. dora exhibir todas las correspondencias biunívocas en-
7. Define menor o igual que de manera análoga a la defini- tre los conjuntos 51, 2, 3, Á , 646 y 565, 66, 67,
ción de menor que. Á , 1286.
Solución abierta 12. Coloca en fila a algunos compañeros de clase y deter-
mina el número posible de diferentes arreglos de 1, 2, 3,
8. a. Da tres ejemplos de conjuntos A y B, y un conjunto
4 y 5 personas. Usa tu modelo para validar el Principio
universal U, tales que A ( B; halla A y B. Fundamental del Conteo.
b. Con base en tus observaciones, emite una conjetura
acerca de la relación entre B y A. Preguntas del salón de clase
c. Justifica tu conjetura en (b) por medio de un diagrama 13. Un estudiante asegura que 5⭋6 es la notación adecuada
de Venn. para el conjunto vacío. ¿Qué le respondes?

© Lopez Mateos Editores. ISBN 978-607-95583-2-1, obra completa, versión electrónica, ISBN 978-607-95583-3-8, volumen 1, versión electrónica. Ejemplar asignado a: Helecto Villarroel gutierrez -
helecto@gmail.com. Fecha: 27 de octubre de 2014. Prohibida su modificación, copia o distribución.
Sección 2-3 Otras operaciones entre conjuntos y sus propiedades 93

14. Una estudiante afirma que un conjunto finito es cualquier b. Magallanes tardó 1126 días en dar la vuelta al
conjunto que tiene un elemento mayor. ¿Estás de acuerdo? mundo. ¿Cuántos años son?
15. Un estudiante alega que A = 51, 5166 tiene sólo un ele- c. ¿Cuál es tu edad en segundos?
mento. ¿Cómo le respondes? d. ¿Aproximadamente cuántas veces late tu corazón en
16. Una estudiante asegura que A 8 B ó B 8 A. ¿Tiene razón 1 año?
la estudiante? Pregunta del National Assessment of Educational Progress
Problemas de repaso (NAEP) (Evaluación Nacional del Progreso Educativo)
17. Investiga cuáles son las medidas de longitud en el sis- Cuatro personas —A, X, Y y Z— van al cine y se colocan
tema métrico decimal. Diseña un plan para usar el valor en asientos adyacentes. Si A se sienta junto al pasillo,
posicional con las longitudes para convertir valores en- lista todos los posibles arreglos de las otras tres personas.
tre las diferentes unidades métricas. A continuación se muestra uno de los arreglos.
18. Escribe 5280 en forma expandida.
19. ¿Cuál es el valor de mcdx en numerales indoarábigos?
20. Covierte cada caso a base diez: Pasillo
a. O0Ddoce A X Y Z
b. 1011dos
c. 43cinco
21. Si 1 mes tiene aproximadamente 4 semanas y 1 año tiene
aproximadamente 365 días, ó 52 semanas, responde lo si-
guiente:
a. Lewis y Clark pasaron aproximadamente 2 años, 4
meses y 9 días explorando el noroeste de Estados NAEP, 1996, Grado 12
Unidos. ¿Cuánto tiempo es en semanas?

ROMPECABEZAS En una escuela secundaria, los grupos del maestro González y de la maestra Salas
tienen 24 y 25 alumnos, respectivamente. Linda, una alumna del maestro González, asegura que el
número de comités escolares que se pueden formar de manera que contengan al menos un estudiante
de cada grupo es mayor que el número de personas en el mundo. Suponiendo que un comité puede te-
ner hasta 49 estudiantes, halla el número de comités y determina si Linda tiene razón.

2-3 Otras operaciones entre conjuntos y sus propiedades

Hallar el complemento de un conjunto es una operación que actúa sólo en un conjunto a la


vez. En esta sección consideramos operaciones que actúan en dos conjuntos a la vez.

U
Intersección de conjuntos
Supongamos que en el trimestre de otoño, en una escuela quieren enviar por correo una
encuesta a los estudiantes inscritos tanto en cursos de arte como de biología. Para ello, las
A B autoridades de la escuela deben identificar a los alumnos que toman ambas materias. Si A y
B son, respectivamente, el conjunto de alumnos que toman arte y el conjunto de alumnos
que toman biología durante el trimestre de otoño, respectivamente, entonces el conjunto
A B de estudiantes buscado está formado por los que tienen en común A y B, o la intersección
de A y B. La intersección de los conjuntos A y B es la región sombreada en la figura 2-15.
Figura 2-15
OBSERVACIÓN La figura 2-15 describe la posibilidad de que A y B contengan elementos
comunes. La intersección podría no contener elementos.

© Lopez Mateos Editores. ISBN 978-607-95583-2-1, obra completa, versión electrónica, ISBN 978-607-95583-3-8, volumen 1, versión electrónica. Ejemplar asignado a: Helecto Villarroel gutierrez -
helecto@gmail.com. Fecha: 27 de octubre de 2014. Prohibida su modificación, copia o distribución.
94 Sistemas de numeración y conjuntos

Definición de intersección de conjuntos


La intersección de dos conjuntos A y B, que se escribe A ¨ B, es el conjunto de todos los
elementos comunes a A y a B, A ¨ B = 5x ƒ x 僆 A y x 僆 B6.

La palabra clave en la definición de intersección es “y” (ver el Capítulo 1). Como en el len-
guaje cotidiano, en matemáticas “y” significa que se deben cumplir ambas condiciones. En
el ejemplo anterior, el conjunto deseado es el de los estudiantes inscritos en ambas mate-
rias, arte y biología.
Si los conjuntos A y B no tuvieran elementos en común, entonces se trataría de conjun-
tos ajenos. En otras palabras, dos conjuntos A y B son ajenos si, y sólo si, A ¨ B = ⭋. Por
ejemplo, el conjunto de varones que toman biología y el conjunto de mujeres que toman
biología son ajenos.

Ejemplo 2-11 Halla A ¨ B


a. A = 51, 2, 3, 46, B = 53, 4, 5, 66








b. A = 50, 2, 4, 6, Á 6, B = 51, 3, 5, 7, Á 6
c. A = 52, 4, 6, 8, Á 6, B = 51, 2, 3, 4, Á 6

Solución a. A ¨ B = 53, 46.


b. A ¨ B = ⭋; por lo tanto, A y B son ajenos.
c. A ¨ B = A pues todos los elementos de A también están en B.

U Si A representa a todos los estudiantes que toman clases de arte y B a todos los estudian-
tes que toman clases de biología, podemos usar un diagrama de Venn tomando en cuenta
que algunos estudiantes están inscritos en ambas materias. Si sabemos que hay 100 estu-
A B
80 20 180 diantes en arte y 200 en biología, y que 20 de ellos toman tanto arte como biología, enton-
ces 100 - 20, u 80, estudiantes están inscritos en arte pero no en biología, y 200 - 20, ó
180, están inscritos en biología pero no en arte. Podemos registrar esta información como
en la figura 2-16. Nota que el número total de estudiantes en el conjunto A es 100 y que el
Figura 2-16
total en el conjunto B es 200.

Unión de conjuntos
Si A es el conjunto de los estudiantes que cursan arte durante el trimestre de otoño y B es el
conjunto de los estudiantes que cursan biología, entonces el conjunto de estudiantes que
U cursan arte o biología, o ambos, durante el trimestre de otoño, es la unión de los conjuntos
A y B. La unión de los conjuntos A y B se representa gráficamente en la figura 2-17.

A B
Definición de unión de conjuntos
La unión de dos conjuntos A y B, que se escribe A ´ B, es el conjunto de todos los elementos
A B que están en A o en B, A ´ B = 5x ƒ x 僆 A o x 僆 B6.
Figura 2-17

La palabra clave en la definición de unión es “o”. En matemáticas, “o” usualmente signi-


fica “uno u otro o ambos”. Se conoce como “o inclusivo”.

© Lopez Mateos Editores. ISBN 978-607-95583-2-1, obra completa, versión electrónica, ISBN 978-607-95583-3-8, volumen 1, versión electrónica. Ejemplar asignado a: Helecto Villarroel gutierrez -
helecto@gmail.com. Fecha: 27 de octubre de 2014. Prohibida su modificación, copia o distribución.
Sección 2-3 Otras operaciones entre conjuntos y sus propiedades 95

Ejemplo 2-12 Halla A ´ B en cada uno de los siguientes casos:


a. A = 51, 2, 3, 46, B = 53, 4, 5, 66









b. A = 50, 2, 4, 6, Á 6, B = 51, 3, 5, 7, Á 6
c. A = 52, 4, 6, 8, Á 6, B = 51, 2, 3, 4, Á 6

Solución a. A ´ B = 51, 2, 3, 4, 5, 66.


b. A ´ B = 50, 1, 2, 3, 4, Á 6.
c. Como todo elemento de A ya está en B, tenemos que A ´ B = B.

AHORA INTENTA ÉSTE 2-13 Nota que en la figura 2-16, n(A ´ B) = 80 + 20 + 180 = 280, pero que
n(A) + n(B) = 100 + 200 = 300; por lo tanto, en general, n(A ´ B) Z n(A) + n(B). Usa el concepto de in-
tersección de conjuntos a fin de obtener una fórmula para n(A ´ B).

Diferencia de conjuntos
Si A es el conjunto de los estudiantes que cursan arte durante el trimestre de otoño y B es el
conjunto de estudiantes que cursan biología, entonces el conjunto de todos los estudiantes
que cursan biología pero no arte se llama complemento de A respecto a B o diferencia
de conjuntos de B y A.

Definición de complemento relativo


El complemento de A respecto a B, que se escribe B - A, es el conjunto de todos los elemen-
tos en B que no están en A; B - A = 5x ƒ x 僆 B y x 僆 A6.

O B E R VA C I Ó N Nota que B - A no se lee como “B menos A”. Menos es una operación


entre números y diferencia de conjuntos es una operación entre conjuntos.

En la figura 2-18(a) se muestra un diagrama de Venn que representa B - A. La región som-


breada representa todos los elementos que están en B pero no en A. En la figura 2-18(b) pre-
sentamos un diagrama de Venn para B ¨ A. La región sombreada representa todos los
elementos que están en B y en A. Nota que B ¨ A = B - A pues B ¨ A es por definición de
intersección y de complemento, el conjunto de todos los elementos en B que no están en A.

U U

A B A B

B–A B A
(a) (b)

Figura 2-18

© Lopez Mateos Editores. ISBN 978-607-95583-2-1, obra completa, versión electrónica, ISBN 978-607-95583-3-8, volumen 1, versión electrónica. Ejemplar asignado a: Helecto Villarroel gutierrez -
helecto@gmail.com. Fecha: 27 de octubre de 2014. Prohibida su modificación, copia o distribución.
96 Sistemas de numeración y conjuntos

Ejemplo 2-13 Si A = 5d, e, f 6, B = 5a, b, c, d, e, f 6 y C = 5a, b, c6, halla:











a. A - B
b. B - A
c. B - C
d. C - B
e. ¿Es importante saber cuál es el conjunto universo para responder las partes (a)–(d)?
Solución a. A - B = ⭋
b. B - A = 5a, b, c6
c. B - C = 5d, e, f6
d. C - B = ⭋
e. Las respuestas de las partes (a)–(d) son independientes del conjunto universo.
La definición de diferencia de conjuntos relaciona un conjunto con otro, sin im-
portar cuál sea el conjunto universo.

Propiedades de las operaciones entre conjuntos


Debido a que no importa el orden de los elementos en un conjunto, A ´ B es igual a B ´ A.
Ésta es la propiedad conmutativa de la unión de conjuntos. No importa en qué orden se
escriban los conjuntos cuando se trata de su unión. De manera análoga, A ¨ B = B ¨ A.
Ésta es la propiedad conmutativa de la intersección de conjuntos.

AHORA INTENTA ÉSTE 2-14 Usa diagramas de Venn y otros medios para ver en qué casos es
importante la manera en que se agrupan los términos cuando se trata de la misma operación. Por ejemplo,
¿es siempre cierto que A ¨ (B ¨ C ) = (A ¨ B) ¨ C ? Investiga cuestiones semejantes respecto a la unión y
la diferencia.

Quizá al responder Ahora intenta éste 2-14 hayas descubierto las propiedades siguientes:

Teorema 2–2:    Propiedad asociativa de la intersección y de la unión de


conjuntos
La propiedad A ¨ 1B ¨ C2 = 1A ¨ B2 ¨ C es la propiedad asociativa de la intersección de
conjuntos. De manera análoga, A ´ 1B ´ C2 = 1A ´ B2 ´ C es la propiedad asociativa de la
unión de conjuntos.

Ejemplo 2-14 ¿Es importante la manera en que se agrupan los términos cuando se trata de dos operacio-
nes? Por ejemplo, ¿es siempre cierto que A ¨ 1B ´ C2 = 1A ¨ B2 ´ C?








Solución Para investigar la situación, tomamos A = 5a, b, c, d6, B = 5c, d, e6 y C = 5d, e, f, g6.
Entonces
A ¨ 1B ´ C2 = 5a, b, c, d6 ¨ (5c, d, e6 ´ 5d, e, f, g6)
= 5a, b, c, d6 ¨ 5c, d, e, f, g6
= 5c, d6
1A ¨ B2 ´ C = 15a, b, c, d6 ¨ 5c, d, e62 ´ 5d, e, f, g6
= 5c, d6 ´ 5d, e, f, g6
= 5c, d, e, f, g6
En este caso, A ¨ 1B ´ C2 Z 1A ¨ B2 ´ C. Así, hemos hallado un contraejemplo, esto es,
un ejemplo que ilustra que la proposición no siempre es verdadera.

© Lopez Mateos Editores. ISBN 978-607-95583-2-1, obra completa, versión electrónica, ISBN 978-607-95583-3-8, volumen 1, versión electrónica. Ejemplar asignado a: Helecto Villarroel gutierrez -
helecto@gmail.com. Fecha: 27 de octubre de 2014. Prohibida su modificación, copia o distribución.
Sección 2-3 Otras operaciones entre conjuntos y sus propiedades 97

U Para descubrir una expresión que sea igual a A ¨ 1B ´ C2, considera el diagramas de
Venn mostrado en la figura 2-19. donde la región sombreada representa A ¨ 1B ´ C2. En
A B
la figura, A ¨ C y A ¨ B son subconjuntos de la región sombreada. La unión de A ¨ C y
A ¨ B es toda la región sombreada. Luego entonces, A ¨ 1B ´ C2 = 1A ¨ B2 ´ 1A ¨ C2 . A
C
continuación enunciamos formalmente esta propiedad.

Figura 2-19 Teorema 2–3:    Propiedad distributiva de la intersección sobre la unión


Para conjuntos cualesquiera A, B y C,
A ¨ 1B ´ C2 = 1A ¨ B2 ´ 1A ¨ C2

AHORA INTENTA ÉSTE 2-15 ¿Será verdadera la propiedad obtenida al intercambiar los símbolos ¨ y
´ en ambos lados de la ecuación de la propiedad distributiva de la intersección sobre la unión ? Explica por
qué. ¿Cómo debería llamarse la propiedad?

Ejemplo 2-15 Usa notación de conjuntos para describir las partes sombreadas de los diagramas de Venn
de la figura 2-20.








U U

A B A B C

(a) (b)

U U

A B A B

C C

(c) (d)

Figura 2-20

Solución La solución se puede escribir de maneras distintas pero equivalentes. Las si-
guientes son posibles respuestas:
a. 1A ´ B2 - 1A ¨ B2, 1A ´ B2 ¨ 1A ¨ B2 o 1A - B2 ´ 1B - A2
b. 1A ¨ B2 ´ 1B ¨ C2 o B ¨ 1A ´ C2
c. 1A - B2 - C, A - 1B ´ C2 o 1A - 1A ¨ B22 - 1A ¨ C2
d. 11A ´ C2 - B2 ´ 1A ¨ B ¨ C2 o 1A - 1B ´ C22 ´ 1C - 1A ´ B22 ´ 1A ¨ C2

Diagramas de Venn como herramienta para resolver problemas


Los diagramas de Venn se pueden usar como una herramienta para modelar información al
resolver problemas, como se muestra en los ejemplos siguientes.
© Lopez Mateos Editores. ISBN 978-607-95583-2-1, obra completa, versión electrónica, ISBN 978-607-95583-3-8, volumen 1, versión electrónica. Ejemplar asignado a: Helecto Villarroel gutierrez -
helecto@gmail.com. Fecha: 27 de octubre de 2014. Prohibida su modificación, copia o distribución.
98 Sistemas de numeración y conjuntos

Ejemplo 2-16 Supón que M es el conjunto de todos los estudiantes que toman matemáticas y E es el con-









junto de quienes toman español. Identifica a los estudiantes descritos en cada región de la
figura 2-21.
U
Solución
M E
(a) (b) (c) La región (a) contiene a todos los estudiantes que toman matemáticas pero no español.
La región (b) contiene a todos los estudiantes que toman tanto matemáticas como español.
(d)
La región (c) contiene a todos los estudiantes que toman español pero no matemáticas.
Figura 2-21 La región (d) contiene a todos los estudiantes que no toman ni matemáticas ni español.

AHORA INTENTA ÉSTE 2-16 En la siguiente Página de un libro de texto aparece otro ejemplo donde
se usan diagramas de Venn para modelar información. Responde la pregunta 27.

Ejemplo 2-17 Se aplicó una encuesta a 110 alumnos de primer ingreso de la universidad para investigar su
formación en el bachillerato, y se reunió la información siguiente:








25 cursaron física.
45 cursaron biología.
48 cursaron matemáticas.
10 cursaron física y matemáticas.
8 cursaron biología y matemáticas.
6 cursaron física y biología.
5 cursaron todas las materias.
a. ¿Cuántos estudiantes cursaron biología pero no física ni matemáticas?
b. ¿Cuántos cursaron física, biología o matemáticas?
c. ¿Cuántos no cursaron ninguna de las tres materias?
Solución  Para resolver este problema construimos un modelo usando conjuntos. Como hay tres
U
materias distintas, deberemos usar tres círculos. El máximo número de regiones de un dia-
grama de Venn determinado por tres círculos es ocho. En la figura 2-22 representamos con
F 1 B F el conjunto de estudiantes que cursaron física, con B el conjunto que cursó biología y con
14 5 36 M el conjunto que cursó matemáticas. La región sombreada representa a los cinco estudian-
5 3
tes que cursaron las tres materias. La región rayada representa a los estudiantes que cursaron
35 M
11 física y matemáticas pero no biología.
En la parte (a) se nos pide hallar el número de estudiantes en el subconjunto de B que no
Figura 2-22 tiene elementos en común con F o con M. Esto es, B - 1F ´ M2. En la parte (b) se nos pide
el número de elementos en F ´ B ´ M. Finalmente, en la parte (c) se nos pide el número de
elementos en F ´ B ´ M, o U - 1F ´ B ´ M2. Nuestra estrategia es hallar el número de es-
tudiantes en cada una de las ocho regiones, sin traslapo.
Un prejuicio del cual debemos cuidarnos en este problema es pensar que los 25 que cur-
saron, por ejemplo, física, sólo cursaron física. Eso no necesariamente es así. Si esos estu-
diantes sólo hubieran cursado física, nos lo debieron haber dicho.
a. Como un total de 10 estudiantes cursaron física y matemáticas, y 5 de ellos también cur-
saron biología, 10 - 5 , ó 5, estudiantes cursaron física y matemáticas pero no biología.

© Lopez Mateos Editores. ISBN 978-607-95583-2-1, obra completa, versión electrónica, ISBN 978-607-95583-3-8, volumen 1, versión electrónica. Ejemplar asignado a: Helecto Villarroel gutierrez -
helecto@gmail.com. Fecha: 27 de octubre de 2014. Prohibida su modificación, copia o distribución.
Sección 2-3 Otras operaciones entre conjuntos y sus propiedades 99

Página de un libro de texto DIAGRAMAS DE V E N N

Para un trabajo de clase, los estudiantes recopilan datos acerca del número de
niñas y niños que hay en las familias de sus compañeros. Usa la tabla siguiente
para responder a los ejercicios 27 a 30

Número de niñas Número de niños


Nombre
en la familia en la familia
Ana 2 0
Beto 0 8

Conexiones
Cali 2 1
Diana 1 0
Elsa 1 1
Félix 0 2
Gloria 2 0
Hugo 2 1
Iván 1 1
Jorge 1 4

27. Ana quiere hacer un diagrama de Venn con los grupos “Tiene niñas
en la familia” y “Tiene niños en la familia”. Ella comienza colocándose
a sí misma en el diagrama. Copia y completa su diagrama.

Tiene niños Tiene niñas


en la familia en la familia

Ana

28. Construye una gráfica de barras que muestre que el número usual de
menores en la familia es dos y que la familia de Beto no es usual.
29. Cali quiere construir una gráfica circular. ¿Cómo se pueden nombrar las
partes del círculo?
30. Construye una gráfica para ver si existe alguna relación entre el número
de niñas en una familia y el número de niños.

Investigación 4 Relacionar dos variables 77

Fuente: Connected Mathematics 2, 2006 (p. 77).

© Lopez Mateos Editores. ISBN 978-607-95583-2-1, obra completa, versión electrónica, ISBN 978-607-95583-3-8, volumen 1, versión electrónica. Ejemplar asignado a: Helecto Villarroel gutierrez -
helecto@gmail.com. Fecha: 27 de octubre de 2014. Prohibida su modificación, copia o distribución.
100 Sistemas de numeración y conjuntos

De manera análoga, como 8 estudiantes cursaron biología y matemáticas, y 5 cursaron


las tres materias, 8 - 5, ó 3, cursaron biología y matemáticas pero no física. Además
6 - 5, ó 1, estudiante cursó física y biología pero no matemáticas. Para hallar el número
de estudiantes que cursaron biología pero no física ni matemáticas, restamos de 45 (el nú-
mero total de quienes cursaron biología) el número de aquellos que están en las regiones
que incluyen biología y las otras materias, esto es, 1 + 5 + 3, ó 9. Como 45 - 9 = 36,
sabemos que 36 estudiantes cursaron biología pero no física ni matemáticas.
b. Para hallar el número de estudiantes en las distintas regiones de F, M o B, procedemos
como sigue. El número de estudiantes que cursó física pero no matemáticas ni biología
es 25 - 11 + 5 + 52, ó 14. El número de estudiantes que cursó matemáticas pero no
física ni biología es 48 - 15 + 5 + 32, ó 35. Así, el número de estudiantes que cursó
matemáticas, física o biología es 35 + 14 + 36 + 3 + 5 + 5 + 1, ó 99.
c. Como el número total de estudiantes es 110, el número de aquellos que no cursaron nin-
guna de las materias mencionadas es 110 - 99, u 11.

Productos cartesianos
Otra manera de obtener un conjunto a partir de dos conjuntos dados es construir su
producto cartesiano. Se forman parejas relacionando los elementos de un conjunto con
los elementos del otro, de una manera específica. Supongamos que una persona tiene tres
pantalones, P = 5azul,blanco, verde6 y dos camisas, C = 5azul, roja6 . De acuerdo con el
Principio Fundamental del Conteo, hay 3 # 2 , ó 6, posibles pares diferentes de pantalón y
camisa, como se muestra en la figura 2-23.
Camisas (C)

Roja (Azul, Roja) (Blanco, Roja) (Verde, Roja)

Azul (Azul, Azul) (Blanco, Azul) (Verde, Azul)

Azul Blanco Verde


Pantalones (P)

Figura 2-23

Los pares de pantalones y camisas forman el conjunto de todos los pares posibles en
donde el primer miembro del par es un elemento del conjunto P y el segundo miembro
es un elemento del conjunto C. En la figura 2-23 se da el conjunto de todos los pares po-
sibles. Como la primera componente de cada par representa pantalones y la segunda
componente de cada par representa camisas, es importante el orden en que se escriban
las componentes. Así, (verde, azul) representa pantalón verde y camisa azul, mientras
que (azul, verde) representa pantalón azul y camisa verde. Por lo tanto, los dos pares re-
presentan una vestimenta distinta. Debido a que el orden en cada par es importante, los
pares son pares ordenados. Las posiciones que ocupen los pares dentro del conjunto de
ves ti mentas no tiene la menor importancia. Lo que es importante es el orden de las
componentes en cada par.
La pareja pantalón-camisa sugiere la siguiente definición de igualdad de pares ordena-
dos: 1x, y2 = 1m, n2 si, y sólo si, las primeras componentes son iguales y las segundas componentes
son iguales. Un conjunto formado de pares ordenados es un ejemplo de producto carte-
siano. A continuación se presenta una definición formal.

© Lopez Mateos Editores. ISBN 978-607-95583-2-1, obra completa, versión electrónica, ISBN 978-607-95583-3-8, volumen 1, versión electrónica. Ejemplar asignado a: Helecto Villarroel gutierrez -
helecto@gmail.com. Fecha: 27 de octubre de 2014. Prohibida su modificación, copia o distribución.
Sección 2-3 Otras operaciones entre conjuntos y sus propiedades 101

Definición de producto cartesiano


Para dos conjuntos cualesquiera A y B, el producto cartesiano de A y B, que se escribe A * B,
es el conjunto de todos los pares ordenados tales que la primera componente de cada par es un
elemento de A y la segunda componente de cada par es un elemento de B.
A * B = 51x, y2 ƒ x 僆 A y y 僆 B6

O B S E R VA C I Ó N A * B comúnmente se lee como “A cruz B” y nunca debe leerse como


“A por B ”.

Ejemplo 2-18 Si A = 5a, b, c6 y B = 51, 2, 36, halla cada uno de los siguientes conjuntos:








a. A * B b. B * A c. A * A
Solución a. A * B = 51a, 12, 1a, 22, 1a, 32, 1b, 12, 1b, 22, 1b, 32, 1c, 12, 1c, 22, 1c, 326
b. B * A = 511, a2, 11, b2, 11, c2, 12, a2, 12, b2, 12, c2, 13, a2, 13, b2, 13, c26
c. A * A = {1a, a2, 1a, b2, 1a, c2, 1b, a2, 1b, b2, 1b, c2, 1c, a2, 1c, b2, 1c, c26

Es posible formar un producto cartesiano que incluya el conjunto vacío. Supongamos que
A = 51, 26. Como no hay elementos en ⭋, no es posible formar pares ordenados (x, y) con
x 僆 A y y 僆 ⭋, así que A * ⭋ = ⭋. Esto es válido para todo conjunto A. De manera aná-
loga, ⭋ * A = ⭋ para todo conjunto A. Hay cierta analogía entre la última ecuación y el
hecho de que 0 # a = 0, donde a es un número natural. En el capítulo 3 usamos el concepto
de producto cartesiano para definir la multiplicación de números naturales.

Evaluación 2-3A

1. Si N = 51, 2, 3, 4, Á 6, A = 5x ƒ x = 2n - 1 donde 5. En las figuras que siguen, sombrea la parte del diagrama
n 僆 N6, B = 5x ƒ x = 2n donde n 僆 N6 y de Venn que represente el conjunto dado:
C = 5x ƒ x = 2n + 1 donde n = 0 o n 僆 N6, halla las a. A ´ B b. A ¨ B
expresiones más sencillas para: c. 1A ¨ B2 ´ 1A ¨ C2 d. 1A ´ B2 ¨ C
a. A ´ C b. A ´ B c. A ¨ B e. 1A ¨ B2 ´ C
2. Di cuáles de los siguientes pares de conjuntos siempre
son iguales: U
a. A ¨ B y B ¨ A
A B
b. A ´ B y B ´ A
c. A ´ 1B ´ C2 y 1A ´ B2 ´ C
d. A ´ A y A ´ ⭋ C
3. Di si lo mencionado a continuación es verdadero para todos
los conjuntos A y B. De ser falso, exhibe un contraejemplo. 6. Si S es un subconjunto del universo U, halla:
a. A ´ ⭋ = A a. S ´ S b. U
b. A - B = B - A c. S ¨ S d. ⭋ ¨ S
c. A ¨ B = A ¨ B 7. Halla A - B en las condiciones siguientes:
d. 1A ´ B2 - A = B a. A ¨ B = ⭋
e. 1A - B2 ´ A = 1A - B2 ´ 1B - A2 b. B = U
4. Si B 8 A, halla una expresión más sencilla para: 8. Si sabemos que para los conjuntos A y B sucede que
a. A ¨ B b. A ´ B A - B = ⭋ , ¿es necesariamente verdadero que A 8 B?
Justifica la respuesta.

© Lopez Mateos Editores. ISBN 978-607-95583-2-1, obra completa, versión electrónica, ISBN 978-607-95583-3-8, volumen 1, versión electrónica. Ejemplar asignado a: Helecto Villarroel gutierrez -
helecto@gmail.com. Fecha: 27 de octubre de 2014. Prohibida su modificación, copia o distribución.
102 Sistemas de numeración y conjuntos

9. Usa notación de conjuntos para identificar cada una de 15. Dado el universo como el conjunto de todos los seres
las regiones sombreadas: humanos, B = 5x ƒ x es un jugador de baloncesto uni-
versitario6 y E = 5x ƒ x es un estudiante universitario
que mide más de 200 cm6, describe con palabras los
U

conjuntos siguientes:
A B
a. B ¨ E b. E c. B ´ E
d. B ´ E e. B ¨ E f. B ¨ E
16. De los alumnos de segundo grado de una escuela secun-
daria, 7 jugaron baloncesto, 9 jugaron voleibol, 10 juga-
a. ron futbol, 1 jugó sólo baloncesto y voleibol, 1 jugó sólo
baloncesto y futbol, 2 jugaron sólo voleibol y futbol, y 2
U jugaron voleibol, baloncesto y futbol. ¿Cuántos jugaron
uno o más de los tres deportes mencionados?
17. En una asociación estudiantil con 30 miembros, 18 cursan
A B matemáticas, 5 cursan matemáticas y biología, y 8 no cur-
san ni matemáticas ni biología. ¿Cuántos cursan biología
pero no matemáticas?
b. 18. En la tienda de bicicletas de Pablo se revisaron 40 bici-
cletas. Si 20 necesitaban llantas nuevas y 30 necesitaban
U reparación de frenos, responde lo siguiente:
a. ¿Cuál es el mayor número de bicicletas que requieren ambos?
A B
b. ¿Cuál es el menor número de bicicletas que podrían
requerir de ambos?
c. ¿Cuál es el mayor número de bicicletas que no reque-
C
rirían reparación?
19. La Cruz Roja busca la presencia de tres tipos de antígenos
c. en los análisis de sangre: A, B y Rh. Cuando se presenta el
10. En la figura que sigue, sombrea la parte del diagrama de antígeno A o B se lista, pero si ambos antígenos están au-
Venn que represente el conjunto dado: sentes la sangre es de tipo O. Si está presente el antígeno
Rh la sangre es positiva; de no ser así, es negativa. Si los
U
técnicos del laboratorio reportan el siguiente resultado
después de analizar las muestras de sangre de 100 perso-
A B nas, ¿cuántas fueron clasificadas como O negativo? Ex-
plica tu razonamiento.
Número de muestras Antígeno en sangre
40 A
11. Usa diagramas de Venn para determinar si cada caso es 18 B
verdadero: 82 Rh
a. A ´ 1B ¨ C2 = 1A ´ B2 ¨ C 5 AyB
b. A - 1B - C2 = 1A - B2 - C 31 A y Rh
12. Para cada uno de los pares siguientes de conjuntos, ex- 11 B y Rh
plica cuál es subconjunto del otro. Si sucede que nin-
4 A, B y Rh
guno es subconjunto del otro, explica por qué.
a. A ¨ B y A ¨ B ¨ C 20. Clasifica cada una de las afirmaciones siguientes como
verdadera o falsa. Si es falsa, exhibe un contraejemplo.
b. A ´ B y A ´ B ´ C
Supón que A y B son conjuntos finitos.
13. a. Si A tiene tres elementos y B tiene dos elementos, ¿cuál
a. Si n1A2 = n1B2, entonces A = B.
es el mayor número de elementos posible en (i) A ´ B?
b. Si A - B = ⭋ , entonces A = B.
(ii) A ¨ B? (iii) B - A? (iv) A - B?
c. Si A ( B, entonces n1A2 6 n1B2.
b. Si A tiene n elementos y B tiene m elementos, ¿cuál es
21. Tres cronistas intentan predecir quiénes van a ser los ga-
el mayor número de elementos posible en (i) A ´ B?
nadores del futbol del domingo. El único equipo que
(ii) A ¨ B? (iii) B - A? (iv) A - B?
juega el domingo y no eligieron fue el Necaxa. Las opi-
14. Si n1A2 = 4, n1B2 = 5 y n1C2 = 6, ¿cuál es el mayor y
niones fueron:
cuál es el menor número de elementos posible en
Felipe: Cruz Azul, Guadalajara, unam, Atlas
a. A ´ B ´ C?
Paula: Guadalajara, Pachuca, Cruz Azul, América
b. A ¨ B ¨ C?
Ramón: América, unam, Santos, Cruz Azul
© Lopez Mateos Editores. ISBN 978-607-95583-2-1, obra completa, versión electrónica, ISBN 978-607-95583-3-8, volumen 1, versión electrónica. Ejemplar asignado a: Helecto Villarroel gutierrez -
helecto@gmail.com. Fecha: 27 de octubre de 2014. Prohibida su modificación, copia o distribución.
Sección 2-3 Otras operaciones entre conjuntos y sus propiedades 103

Si los equipos que juegan el domingo son únicamente los 23. Halla C y D si el producto cartesiano C * D es:
mencionados, ¿quién juega contra quién? a. 51a, b2, 1a, c2, 1a, d2, 1a, e26
22. Sean A = 5x, y6 y B = 5a, b, c6. Halla: b. 511, 12, 11, 22, 11, 32, 12, 12, 12, 22, 12, 326
a. A * B c. 510, 12, 10, 02, 11, 12, 11, 026
b. B * A

Evaluación 2-3B

1. Si C = 50, 1, 2, 3, Á 6, A = 5x ƒ x = 2n + 1 donde U U
n 僆 C6, B = 5x ƒ x = 2n donde n 僆 C6, y N =
51, 2, 3, Á 6, halla las expresiones más sencillas para: A B AA BB

a. C - A b. A ¨ B c. C ¨ N
2. Di cuáles de los siguientes pares de conjuntos siempre C C
son iguales.
a. X ¨ Y y Y ¨ X a. b.
b. X ´ Y y Y ´ X
c. A ¨ 1B ¨ C2 y 1A ¨ B2 ¨ C U
d. B ´ ⭋ y B ¨ B
3. Di si lo mencionado a continuación es verdadero o falso, A B
para todos los conjuntos A, B o C. De ser falso, exhibe un
contraejemplo.
C
a. A - B = A - ⭋
b. A ´ B = A ´ B
c. A ¨ 1B ´ C2 = 1A ¨ B2 ´ C c.
d. 1A - B2 ¨ A = A
e. A - 1B ¨ C2 = 1A - B2 ¨ 1A - C2
10. En la figura que sigue, sombrea la parte del diagrama de
Venn que represente el conjunto dado:
4. Si X 8 Y, halla una expresión más sencilla para: U
a. X - Y b. X ¨ Y
5. En los casos siguientes, sombrea la porción del diagrama
de Venn que ilustre el conjunto: A B
a. A ¨ C b. A ´ B
c. 1A ¨ B2 ´ 1B ¨ C2 d. A ´ 1B ¨ C2
e. A ´ 1B ¨ C2
A¨B

U 11. Usa diagramas de Venn para determinar si cada caso es


verdadero:
A a. A - 1B ¨ C2 = 1A - B2 ¨ 1A - C2
b. A - 1B ´ C2 = 1A - B2 ´ 1A - C2
C

12. Para cada uno de los pares siguientes de conjuntos, ex-


B plica cuál es subconjunto del otro. Si sucede que nin-
guno es subconjunto del otro, explica por qué.
a. A - B y A - 1B - C2
6. Si A es un subconjunto del universo U, halla: b. A ´ B y 1A ´ B2 - ⭋
a. A ´ U b. U - A 13. a. Si n1A ´ B2 = 22, n1A ¨ B2 = 8 y n1B2 = 12, halla
c. A - ⭋ d. ⭋ ¨ A n1A2.
7. Halla B - A en las condiciones siguientes: b. Si n1A2 = 8, n1B2 = 14 y n1A ¨ B2 = 5, halla
a. A = B b. B 8 A n1A ´ B2.
8. Da dos ejemplos de conjuntos A y B para los cuales 14. La ecuación A ´ B = A ¨ B y una ecuación análoga para
B - A = ⭋ . Muestra que en cada ejemplo B 8 A. A ¨ B se conocen como leyes de De Morgan en honor del
9. Usa notación de conjuntos para identificar cada una de famoso matemático británico que las descubrió.
las regiones sombreadas: a. Usa diagramas de Venn para mostrar que
A ´ B = A ¨ B.
© Lopez Mateos Editores. ISBN 978-607-95583-2-1, obra completa, versión electrónica, ISBN 978-607-95583-3-8, volumen 1, versión electrónica. Ejemplar asignado a: Helecto Villarroel gutierrez -
helecto@gmail.com. Fecha: 27 de octubre de 2014. Prohibida su modificación, copia o distribución.
104 Sistemas de numeración y conjuntos

b. Descubre una ecuación análoga que incluya a 18. Cuando se intersecan tres conjuntos A, B y C, como en
A ¨ B, A y B . Usa diagramas de Venn para mostrar el diagrama del problema 17, se crean ocho regiones sin
que la ecuación es válida. traslapo. Describe cada una de las regiones usando nota-
c. Verifica las ecuaciones (a) y (b) para conjuntos espe- ción de conjuntos.
cíficos. 19. Una encuestadora entrevistó a 500 estudiantes universi-
15. Supongamos que E es el conjunto de todos los estudian- tarios avanzados que tenían tarjeta de crédito. Reportó
tes de segundo grado en una escuela secundaria. B es el que 240 tenían TarjetaOro, 290 tenían SuperTarjeta y
conjunto de los estudiantes que tocan en una banda y C 270 tenían GranTarjeta. De esos estudiantes el reporte
el de los estudiantes que participan en un coro. Identifica afirmaba que 80 tenían sólo TarjetaOro y SuperTarjeta,
con palabras a los estudiantes descritos por cada región 70 tenían sólo TarjetaOro y GranTarjeta, 60 tenían sólo
de la figura siguiente: SuperTarjeta y GranTarjeta, y 50 tenían las tres tarjetas.
E Cuando enviaron el reporte para publicación en el pe-
riódico universitario, el editor lo rechazó asegurando
B C que la encuesta no era correcta. ¿Tuvo razón el editor?
a. b. c. ¿Por qué sí o por qué no?
20. El manejador de un equipo de beisbol revisó su lista de
d. jugadores y se percató de lo siguiente.
16. Llena los diagramas de Venn con los números apropia- • Todos los jardineros fueron bateadores ambidiestros.
dos, con base en la información siguiente: • Un tercio del cuadro fueron bateadores ambidiestros.
n1A2 = 26 n1B ¨ C2 = 12 • La mitad de los bateadores ambidiestros fueron jardineros.
n1B2 = 32 n1A ¨ C2 = 8 • Hay 12 jugadores de cuadro y 8 jardineros, y ninguna
n1C2 = 23 n1A ¨ B ¨ C2 = 3 persona jugó en ambas posiciones.
n1A ¨ B2 = 10 n1U2 = 65 ¿Cuántos bateadores ambidiestros no fueron jugadores
de cuadro ni jardineros?
U 21. En el primer día para la selección de la Miniliga, se pre-
sentaron 128 niños de edades: 10 (D), 11 (O) y 12 (C). Se
les preguntó qué posición querrían jugar, aparte de lanza-
dor: cuadro (K), jardín (J) o receptor (R). Los resultados
A B se muestran en la tabla siguiente:

K J R Total
C
10 (D) 28 14 12 54
11 (O) 18 20 8 46
17. Escribe las letras en la sección apropiada del siguiente 12 (C) 10 12 6 28
diagrama de Venn, usando la información siguiente: Total 56 46 26 128

A es el conjunto de las letras de la palabra Iowa. Di en palabras lo que significa lo siguiente junto con el
B es el conjunto de las letras de la palabra Hawaii. número de niños indicados en cada parte:
C es el conjunto de las letras de la palabra Ohio. a. K ¨ C
El conjunto universal U es el conjunto de las letras de la b. R ¨ 1D ´ O2
palabra Washington. c. 1K ´ J2 ¨ D
d. 1D ´ O2 ¨ J .
U 22. Di, en cada caso, si lo siguiente es verdadero o falso, y
por qué.
a. 12, 52 = 15, 22 b. 12, 52 = 52, 56
23. Responde lo siguiente:
A B
a. Si A tiene cinco elementos y B tiene cuatro elemen-
tos, ¿cuántos elementos hay en A * B?
b. Si A tiene m elementos y B tiene n elementos, ¿cuán-
C tos elementos hay en A * B?
c. Si A tiene m elementos, B tiene n elementos y C tiene
p elementos, ¿cuántos elementos hay en 1A * B2 * C?

© Lopez Mateos Editores. ISBN 978-607-95583-2-1, obra completa, versión electrónica, ISBN 978-607-95583-3-8, volumen 1, versión electrónica. Ejemplar asignado a: Helecto Villarroel gutierrez -
helecto@gmail.com. Fecha: 27 de octubre de 2014. Prohibida su modificación, copia o distribución.
Sección 2-3 Otras operaciones entre conjuntos y sus propiedades 105

Conexiones matemáticas 2-3

Comunicación y A ´ B = A ´ C, entonces B = C. ¿Qué le respondes?


1. Responde lo siguiente y justifica la respuesta: 10. Un estudiante afirma que se puede “romper” la barra so-
a. Si a 僆 A ¨ B, ¿es verdad que a 僆 A ´ B? bre la operación de intersección; esto es, que
b. Si a 僆 A ´ B, ¿es verdad que a 僆 A ¨ B? A ¨ B = A ¨ B. ¿Qué le respondes?
2. Explica cómo se relaciona A con U - A. 11. Se le pide a un estudiante que encuentre todas las corres-
3. ¿Es conmutativa la operación de formar productos carte- pondencias biunívocas posibles entre dos conjuntos da-
sianos? Explica por qué sí o por qué no. dos. Él obtiene el producto cartesiano de los conjuntos y
4. Si A y B son conjuntos, ¿es cierto siempre que afirma que ésa es la respuesta correcta pues tiene todos
n1A - B2 = n1A2 - n1B2? Explica. los posibles pareos entre los elementos de los conjuntos.
¿Qué le respondes?
Solución abierta 12. Una estudiante asegura que es lo mismo sumar dos con-
5. Redacta y resuelve una situación acerca de ciertos conjun- juntos A y B, o A + B , que tomar la unión, A ´ B .
tos A, B y C para los cuales se conozca n1A ´ B ´ C2 y se ¿Cómo le respondes?
requiera hallar n1A2, n1B2 y n1C2.
Problemas de repaso
6. Describe una situación de la vida real que se pueda repre-
sentar con: 13. ¿Existe el número “dos” en base dos? Explica tu razona-
a. A ¨ B miento.
b. A ¨ B ¨ C 14. ¿Cómo escribirías 81 en base tres? ¿Y cualquier potencia
c. A - 1B ´ C2 de 3 en base diez?
15. a. Escribe 54, 5, 6, 7, 8, 96 usando la notación generadora
Aprendizaje colectivo de conjuntos.
7. Usa operaciones de conjuntos como unión, intersección, b. Escribe 5x ƒ x = 5n, donde n = 3, 6 ó 96 como lista.
complemento y diferencia para describir, en la mayor can- 16. Halla el número de elementos en los conjuntos siguientes:
tidad de maneras posible, la región sombreada de la figura a. 5x ƒ x es una letra de abandonados6
siguiente. Compara tus expresiones con las de otros gru- b. El conjunto de letras que aparecen en la palabra atrac-
pos para ver quién tiene más. ¿Cuál es el número total de tiva
expresiones diferentes halladas entre todos los grupos? 17. Si A = 51, 2, 3, 46 y B = 51, 2, 3, 4, 56, responde:
¿Qué expresiones aparecieron en todos los grupos? a. ¿Cuántos subconjuntos de A no contienen el elemento 1?
U
b. ¿Cuántos subconjuntos de A contienen el elemento 1?
c. ¿Cuántos subconjuntos de A contienen el elemento 1
ó 2?
d. ¿Cuántos subconjuntos de A no contienen los elemen-
A tos 1 ni 2?
e. ¿Cuántos subconjuntos de B contienen el elemento 5 y
B cuántos no?
f. Si conocemos todos los subconjuntos de A, ¿cómo po-
demos listar, de manera sistemática, todos los subcon-
juntos de B? ¿Cuántos subconjuntos de B hay?
C
18. a. ¿Cuáles de los conjuntos siguientes son iguales?
b. ¿Cuáles son subconjuntos propios de otro?
A = 52, 4, 6, 8, 10, Á 6
Preguntas del salón de clase B = 5x ƒ x = 2n + 2 donde n = 0, 1, 2, 3, 4, Á 6
8. Un estudiante pregunta, “¿Si A = 5a, b, c6 y B = 5b, c, d6, C = 5x ƒ x = 4n donde n 僆 N6
por qué no es cierto que A ´ B = 5a, b, c, b, c, d6?” ¿Qué 19. Da ejemplos de la vida real para:
le respondes? a. Una correspondencia biunívoca entre dos conjuntos
9. Una estudiante dice que ella puede demostrar que si b. Una correspondencia entre dos conjuntos que no sea
A ¨ B = A ¨ C, entonces B = C no necesariamente es biunívoca
cierto; pero piensa que en el caso de que A ¨ B = A ¨ C

© Lopez Mateos Editores. ISBN 978-607-95583-2-1, obra completa, versión electrónica, ISBN 978-607-95583-3-8, volumen 1, versión electrónica. Ejemplar asignado a: Helecto Villarroel gutierrez -
helecto@gmail.com. Fecha: 27 de octubre de 2014. Prohibida su modificación, copia o distribución.
106 Sistemas de numeración y conjuntos

20. Si hay seis equipos en la liga Alfa y cinco equipos en la Pregunta del National Assessment of Educational Progress
liga Beta y si cada equipo de una liga juega contra cada (NAEP) (Evaluación Nacional del Progreso Educativo)
equipo de la otra liga una sola vez, ¿cuántos partidos se
jugaron?
21. José tiene cuatro pantalones, cinco camisas y tres suéteres.
Si cada día escoge un pantalón, una camisa y un suéter,
¿cuántas combinaciones puede hacer?
A B C D
Melissa escogió una de las figuras anteriores.
• La figura que escogió estaba sombreada.
• La figura que escogió no fue un triángulo.
¿Qué figura escogió?
a. A b. B c. C d. D
NAEP, 2007, Grado 4

ACTIVIDAD DE LABORATORIO Un juego de figuras y atributos consta de 32 piezas. Cada pieza está
identificada por su forma, tamaño y color. Las 4 formas son cuadrado, triángulo, rombo y círculo; los 4 co-
lores son rojo, amarillo, azul y verde; los 2 tamaños son grande y chico. Además de las piezas, hay un gru-
po de 20 cartas. Diez de las cartas especifican uno de los atributos (por ejemplo rojo, grande, cuadrado).
Las otras 10 cartas son cartas de negación y especifican la ausencia de un atributo (por ejemplo no verde,
no círculo). Con este material se pueden estudiar muchos problemas de conjuntos . Por ejemplo, sea A el
conjunto de todas las piezas verdes y B el conjunto de todas las piezas grandes. Usando todas las piezas
como el conjunto universal, describe los elementos de cada uno de los conjuntos listados a continuación
y determina cuáles son iguales:

1. A ´ B; B ´ A
2. A ¨ B; A ¨ B
3. A ¨ B; A ´ B
4. A - B; A ¨ B

Sugerencia para resolver el problema preliminar


El diagrama de Venn que vimos en este capítulo es una buena herramienta para orde-
nar datos. Trata de ubicar varios conjuntos ajenos de personas. Por ejemplo, considera
un círculo con adultos, otro con los ciudadanos de Mississippi y otro con personas del
sexo femenino. Al ir ordenando la información con estos círculos encontrarás el camino
para hallar la solución. Recuerda ubicar qué tipo de personas está en las intersecciones y
cuáles son los complementos de los conjuntos.

Resumen del capítulo

I. Sistemas de numeración 4. Propiedad multiplicativa


A. Un sistema de numeración está formado por 5. El valor posicional asigna un valor a un dí-
un conjunto de símbolos con operaciones y gito dependiendo de su colocación en un
propiedades para presentar los números de ma- numeral. El valor de un dígito es el pro-
nera sistemática. ducto de su valor posicional por su valor
B. Las propiedades de los sistemas de numeración nominal.
dan la estructura básica de los sistemas. C. El sistema de numeración indoarábigo es un
1. Propiedad aditiva sistema de base diez que usa los dígitos 0, 1, 2,
2. Propiedad del valor posicional 3, 4, 5, 6, 7, 8 y 9.
3. Propiedad substractiva o de la resta

© Lopez Mateos Editores. ISBN 978-607-95583-2-1, obra completa, versión electrónica, ISBN 978-607-95583-3-8, volumen 1, versión electrónica. Ejemplar asignado a: Helecto Villarroel gutierrez -
helecto@gmail.com. Fecha: 27 de octubre de 2014. Prohibida su modificación, copia o distribución.
Revisión del capítulo 107

II. Exponentes de B que no está en A (se escribe A ( B ).


A. Para cualquier número completo a y cualquier F. Un conjunto con n elementos tiene 2n subconjuntos.
número natural n, G. La unión de dos conjuntos A y B es el conjunto
de todos los elementos que están en A, en B o
an = a # a # a # Á # a. en ambos (se escribe A ´ B ).
n factores H. La intersección de dos conjuntos A y B es el
conjunto de todos los elementos que pertene-
donde a es la base y n es el exponente. cen tanto a A como a B (se escribe A ¨ B).
B. a0 = 1 donde a 僆 N I. El número cardinal de un conjunto finito C, n1C2
III. Definiciones y notación de conjuntos indica el número de elementos del conjunto.
A. Un conjunto se puede describir como cual- J. Un conjunto es finito si el número de elemen-
quier colección de objetos. tos en el conjunto es cero o un número natural.
B. Los conjuntos deben estar bien definidos de De lo contrario, el conjunto es infinito.
manera que dado un objeto, éste pertenece o no K. Dos conjuntos A y B son ajenos si no tienen
pertenece al conjunto. elementos en común.
C. Un elemento es cualquier miembro de un conjunto. L. El complemento de un conjunto A es el con-
D. Los conjuntos se pueden especificar listando to- junto formado por los elementos del conjunto
dos sus elementos o usando la notación construc- universo que no están en A (se escribe A ).
tora de conjuntos. M. El complemento del conjunto A respecto al
E. El conjunto vacío, que se escribe ⭋, no tiene conjunto B (diferencia de conjuntos) es el con-
elementos. junto de todos los elementos de B que no están
F. El conjunto universo contiene todos los ele- en A (se escribe B - A).
mentos en consideración. N. El producto cartesiano de los conjuntos A y B,
IV. Relaciones y operaciones entre conjuntos que se escribe A * B, es el conjunto de todos los
A. Dos conjuntos son iguales si, y sólo si, tienen pares ordenados tales que el primer elemento de
exactamente los mismos elementos. cada par es de A y el segundo elemento de cada
B. Dos conjuntos A y B están en correspondencia par es de B.
biunívoca o correspondencia uno-a-uno si, y O. Propiedades de las operaciones de conjuntos
sólo si, a cada elemento de A le corresponde exac- 1. Propiedad conmutativa de la unión e inter-
tamente un elemento de B y a cada elemento de sección de conjuntos
B le corresponde exactamente un elemento de A. 2. Propiedad asociativa de la unión e intersec-
C. Dos conjuntos A y B son equivalentes si, y sólo ción de conjuntos
si, sus elementos se pueden colocar en corres- 3. Propiedad distributiva de la intersección so-
pondencia biunívoca (se escribe A ' B). bre la unión, y de la unión sobre la interse-
D. El conjunto A es un subconjunto del conjunto ción de conjuntos.
B si, y sólo si, todo elemento de A es un ele- P. Principio fundamental del conteo: Si un
mento de B (se escribe A 8 B). evento M puede ocurrir de m maneras y, des-
E. El conjunto A es un subconjunto propio del pués de que ocurrió, el evento N puede ocurrir
conjunto B si, y sólo si, todo elemento de A es de n maneras, entonces el evento M seguido
un elemento de B y existe al menos un elemento por el evento N puede ocurrir de mn maneras.

Revisión del capítulo


1. Para cada uno de los números siguientes en base d. 346diez a base cinco
diez, di el valor posicional de cada uno de los dígi- e. 27diez a base dos
tos en círculos: 4. Simplifica cada caso, de ser posible. Escribe la res-
a. 4 3 2 b. 3 432 puesta en forma exponencial, ab.
c. 19 3 24 a. 34 # 37 # 36 b. 210 # 211
2. Convierte cada caso a base diez: 5. Escribe el numeral en base tres de los cubos en base
a. cdxliv tres mostrados a continuación.
b. 432cinco
c. OD0doce
d. 1011dos
e. 4136siete 6. El menor número de cubos de base tres necesarios
3. Convierte cada caso al número en el sistema indicado: para representar 51 es cubos losas
a. 999 a romano barras unidades.
b. 86 a egipcio 7. Dibuja cubos multibase para representar
c. 123 a maya a. 123cuatro. b. 24cinco.

© Lopez Mateos Editores. ISBN 978-607-95583-2-1, obra completa, versión electrónica, ISBN 978-607-95583-3-8, volumen 1, versión electrónica. Ejemplar asignado a: Helecto Villarroel gutierrez -
helecto@gmail.com. Fecha: 27 de octubre de 2014. Prohibida su modificación, copia o distribución.
108 Sistemas de numeración y conjuntos

8. a. El primer dígito de la izquierda (el frontal) de un 18. a. Muestra una posible correspondencia biunívoca
numeral de base diez es 4 seguido de 10 ceros. entre los conjuntos D y E si D = 5v, o, y6 y E =
¿Cuál es el valor posicional del 4? 5f, i, n6.
b. Un número en base cinco tiene 10 dígitos. ¿Cuál es b. ¿Cuántas correspondencias biunívocas es posible
el valor posicional del segundo dígito a la izquierda? establecer entre los conjuntos D y E?
c. Un número en base dos tiene dígito frontal 1 se- 19. Usa un diagrama de Venn para ver si A ¨ 1B ´ C2 =
guido de 30 ceros y dígito de las unidades 1. 1A ¨ B2 ´ C para todos los conjuntos A, B y C.
¿Cuál es el valor posicional del dígito frontal? 20. De acuerdo con una encuesta a 16 estudiantes les gusta
9. Escribe los siguientes numerales de base diez en la la materia de historia, a 19 la de literatura, a 18 la de
base indicada, sin efectuar ninguna multiplicación: matemáticas, a 8 la de matemáticas y la de literatura, a
a. 1010 + 23 en base diez 5 la de historia y la de literatura, a 7 la de historia y la
b. 210 + 1 en base dos de matemáticas, a 3 les gustan las tres materias y a to-
c. 510 + 1 en base cinco dos los estudiantes les gusta al menos una de las mate-
d. 1010 - 1 en base diez rias. Traza un diagrama de Venn que describa esta
e. 210 - 1 en base dos información y responde las preguntas siguientes:
f. 125 - 1 en base doce a. ¿Cuántos estudiantes participaron en la encuesta?
10. Escribe un ejemplo de una base distinta de la base b. ¿A cuántos estudiantes les gusta sólo matemáticas?
diez que se use en una situación de la vida real. c. ¿A cuántos estudiantes les gusta literatura y ma-
¿Cómo se usa? temáticas pero no historia?
11. Describe las características importantes de los si- 21. Describe, usando símbolos, la porción sombreada
guientes sistemas: en cada una de las siguientes figuras:
a. Egipcio b. Babilonio U U
c. Romano d. Indoarábigo
12. Escribe 128 en cada una de las bases siguientes:
a. cinco b. dos A
c. doce C
A
13. Escribe en las bases indicadas sin multiplicar las po- C
B
tencias:
a. 4 # 56 + 11 # 54 + 9 en base cinco B
b. 210 + 23 en base dos
c. 11 # 125 + 10 # 123 + 20 en base doce
d. 9 # 85 + 8 en base ocho a. b.
14. Lista todos los subconjuntos de {m, a, t, e}.
15. Sea 22. Clasifica cada una de las siguientes afirmaciones
U = 5u, n, i, v, e, r, s, a, l6, como verdadera o falsa. De ser falsa, di por qué.
A = 5v, e, r, a6, C = 5n, e, l, i6, a. Para todos los conjuntos A y B, sucede que A 8 B
B = 5e, r, a6, D = 5s, a, l, e6. o B 8 A.
Halla: b. El conjunto vacío es un subconjunto propio de
a. A ´ B b. C ¨ D cualquier conjunto.
c. D d. A ¨ D c. Para todos los conjuntos A y B, si A ' B, enton-
e. B ´ C ces A = B.
f. 1B ´ C2 ¨ D d. El conjunto 55, 10, 15, 20, Á 6 es un conjunto finito.
g. 1A ´ B2 ¨ 1C ¨ D 2 e. Ningún conjunto es equivalente a un subconjunto
h. 1C ¨ D2 ¨ A propio de sí mismo.
i. n1C 2 j. n1C * D2 f. Si A es un conjunto infinito y B 8 A, entonces B
16. Indica los siguientes conjuntos sombreando la figura: también es un conjunto infinito.
U U g. Para todos los conjuntos finitos A y B, si
A ¨ B Z ⭋, entonces
A B A B n1A ´ B2 Z n1A2 + n1B2.
h. Si A y B son conjuntos tales que A ¨ B = ⭋,
entonces A = ⭋ o B = ⭋.
C C 23. Usa diagramas de Venn para decir cuál de las afir-
a. A ¨ 1B ´ C2 b. 1A ´ B2 ¨ C
maciones siguientes siempre es verdadera para los
conjuntos finitos A y B.
17. Supón que juegas a las palabras con siete letras distin-
a. n1A ´ B2 = n1A - B2 + n1B - A2 + n1A ¨ B2
tas. ¿Cuántas palabras de siete letras puede haber?
b. n1A ´ B2 = n1A - B2 + n1B2 =
n1B - A2 + n1A2

© Lopez Mateos Editores. ISBN 978-607-95583-2-1, obra completa, versión electrónica, ISBN 978-607-95583-3-8, volumen 1, versión electrónica. Ejemplar asignado a: Helecto Villarroel gutierrez -
helecto@gmail.com. Fecha: 27 de octubre de 2014. Prohibida su modificación, copia o distribución.
Bibliografía seleccionada 109

24. Supón que P y Q son conjuntos equivalentes y que 26. Considera el siguiente conjunto estados del norte
n1P2 = 17. de la República Mexicana {Baja California, Sonora,
a. ¿Cuál es el mínimo número de elementos en Chihuahua, Coahuila, Nuevo León, Tamaulipas,
P ´ Q? Durango, Sinaloa}. Si una persona escoge un ele-
b. ¿Cuál es el máximo número de elementos en mento, muestra que bastan tres preguntas de sí o no
P ´ Q? para determinar ese elemento.
c. ¿Cuál es el mínimo número de elementos en 27. Usando las definiciones de menor que o mayor
P ¨ Q? que, prueba que cada una de las siguientes desigual-
d. ¿Cuál es el máximo número de elementos en dades es verdadera:
P ¨ Q? a. 3 6 13 b. 12 7 9
25. En una escuela se designó para competir a 26 estu-
diantes en remo, 15 en natación y 16 en futbol. Si la 28. Nina tiene un pantalón café y uno gris; una blusa
selección consta de 46 estudiantes y sólo 2 partici- café, una amarilla y una blanca; y un suéter azul y uno
pan en todos los deportes, ¿cuántos estudiantes blanco. ¿De cuántas maneras puede vestirse si lleva
fueron seleccionados para participar en exactamente un pantalón, una blusa y un suéter?
dos de los tres deportes?

Bibliografía seleccionada
Barkley, C. “Other Ways to Count.” Student Math Notes Pickreign, J. “Alternative Base Arithmetic Activities.”
(November 2003). ON-Math 5 (2006–7).
Framer, J., and R. Powers. “Exploring Mayan Numer- Resnick, L. “From Protoquantities to Operators: Build-
als.” Teaching Children Mathematics 12 (September ing Mathematical Competence on a Foundation of
2005): 69. Everyday Knowledge.” In Analysis of Arithmetic for
Fuson, K. “Research on Learning and Teaching Addition Mathematics Teaching, edited by D. Leinhardt,
and Subtraction of Whole Numbers.” In Handbook of R. Putnam, and R. Hattrup. Hillsdale, NJ: LEA,
Research on Mathematics Teaching and Learning, edited 1992.
by D. Grouws. New York: MacMillan, 1992. Siegler, R. Emerging Minds: The Process of Change in
Ginsburg, H., A. Klein, and P. Starkey. “The Develop- Children’s Thinking. New York: Oxford University
ment of Children’s Mathematical Thinking: Con- Press, 1996.
necting Research with Practice.” In Child Psychology in Tsamir, P., and D. Triosh. “Consistency and Represen-
Practice, edited by Irving E. Sigel and K. Ann Ren- tations: The Case of Actual Infinity.” Journal for Re-
ninger, pp. 401–476, vol. 4 of Handbook of Child search in Mathematics Education 30 (March 1999):
Psychology, edited by William Damon. New York: 213–219.
John Wiley & Sons, 1998. Uy, F. “The Chinese Numeration System and Place
Klein, A., M. Beishuizen, and A. Treffers. “The Empty Value.” Teaching Children Mathematics 9 (January
Number Line in Dutch Second Grades: Realistic 2003): 243.
Versus Gradual Program Design.” Journal of Research Walmsley, A. “Math Roots: Understanding Aztec and
in Mathematics Education 29 (July 1998): 443–464. Mayan Numeration Systems.” Mathematics Teaching
Moldovan, C. “Culture in the Curriculum: Enriching in the Middle School 12 (August 2006): 55.
Numeration and Number Operations.” Teaching Zaslavsky, C. “Developing Number Sense: What Can
Children Mathematics 8 (December 2001): 238–243. Other Cultures Tell Us?” Teaching Children Mathe-
Overbay, S., and M. Brod. “Magic with Mayan Mathe- matics 7 (February 2001): 312–319.
matics.” Mathematics Teaching in the Middle School 12 Zaslavsky, C. “The Influence of Ancient Egypt on Greek
(February 2007): 340. and Other Numeration Systems.” Mathematics Teach-
ing in the Middle School 9 (November 2003): 174.

© Lopez Mateos Editores. ISBN 978-607-95583-2-1, obra completa, versión electrónica, ISBN 978-607-95583-3-8, volumen 1, versión electrónica. Ejemplar asignado a: Helecto Villarroel gutierrez -
helecto@gmail.com. Fecha: 27 de octubre de 2014. Prohibida su modificación, copia o distribución.
Números completos
y sus operaciones
CAPÍTULO

Problema preliminar
Usando precisamente cinco números 5 y sólo suma, resta, multiplicación y división, escribe una
expresión que sea igual a cada uno de los números del 1 al 10. No tienes que usar todas las opera-
ciones. Se permiten números como 55; por ejemplo, 5 se puede escribir como
5 + 315 - 52 # 554 .

110
© Lopez Mateos Editores. ISBN 978-607-95583-2-1, obra completa, versión electrónica, ISBN 978-607-95583-3-8, volumen 1, versión electrónica. Ejemplar asignado a: Helecto Villarroel gutierrez -
helecto@gmail.com. Fecha: 27 de octubre de 2014. Prohibida su modificación, copia o distribución.
Sección 3-1 Suma y resta de números completos 111

E n la sección 2-2 vimos que se puede usar el concepto de correspondencia biunívoca en-
tre conjuntos para explicar a niñas y niños el concepto de número. En Curriculum Focal
Points for Prekindergarten through Grade 8 Mathematics (Puntos focales en el currículo de ma-
temáticas de preescolar al grado 8) de la NCTM, hallamos lo siguiente:

Niñas y niños desarrollan la capacidad para comprender el significado de los números completos
y reconocen la cantidad de objetos en grupos pequeños tanto sin contar como contando —el
primer y más elemental algoritmo matemático. Entienden que los nombres de los números se re-
fieren a cantidades. Usan correspondencia biunívoca para resolver problemas al parear conjuntos
y comparar los números, así como al contar objetos hasta 10 y más. Entienden que la última pala-
bra que pronuncian al contar dice “cuántos son”, cuentan para determinar el número correspon-
diente a la cantidad y comparan las cantidades (usando expresiones como “mayor que” y “menor
que”), para finalmente ordenar los conjuntos según el número de objetos en ellos. (p. 11)

En la siguiente tira cómica de Peanuts, parece que el hermano menor de Lucy todavía no
puede asociar los nombres de los números con una colección de objetos. Pronto aprenderá
que este conjunto de dedos se puede poner en correspondencia biunívoca con muchos con-
juntos de objetos que pueden contarse. Asociará la palabra tres no sólo con los tres dedos
que le muestra Lucy, sino con otros conjuntos de objetos con el mismo número cardinal.

RERUN, ME HAN DICHO A VER, UNA PEQUEÑA ¡DEDOS!


QUE ERES BUENO CON PRUEBA. . DIME
LOS NÚMEROS. . QUÉ VES. . .

En este capítulo estudiaremos las operaciones que usan números completos. Según se
afirma en los Principios y objetivos (POME), en “Número y operación” para los grados de pre-
escolar a 2, todos los estudiantes de este nivel deberán entender el significado de las opera-
ciones y cómo se relacionan una con otra. En particular, en POME se afirma que de
preescolar al grado 2 todos los estudiantes deberán:

• comprender los diferentes significados de suma y resta de números completos, y la relación en-
tre las dos operaciones;
• entender los efectos de sumar y restar números completos;
• comprender situaciones que incluyan la multiplicación y la división, como los agrupamientos
iguales y la repartición equitativa. (p. 78)

3-1 Suma y resta de números completos

Cuando se añade el cero al conjunto de los números naturales, N = 51, 2, 3, 4, 5, Á 6,


tenemos el conjunto de números completos, denotados con E = 50, 1, 2, 3, 4, 5, Á 6. En
esta sección proporcionamos una variedad de modelos para enseñar a desarrollar habilida-
des para efectuar cuentas con ellos y damos la oportunidad de repasar las matemáticas y ob-
tener la comprensión profunda que requiere un maestro.

© Lopez Mateos Editores. ISBN 978-607-95583-2-1, obra completa, versión electrónica, ISBN 978-607-95583-3-8, volumen 1, versión electrónica. Ejemplar asignado a: Helecto Villarroel gutierrez -
helecto@gmail.com. Fecha: 27 de octubre de 2014. Prohibida su modificación, copia o distribución.
112 Números completos y sus operaciones

Suma de números completos


En sus años de preescolar, niñas y niños se encuentran con la suma al combinar objetos y pre-
guntarse cuántos objetos hay en el conjunto combinado. Pueden “continuar contando” como
◆ Nota lo sugieren Carpenter y Moser en la Nota de investigación, o pueden contar los objetos en el
conjunto combinado para hallar su número cardinal.
de investigación
Los estudiantes pue- Modelo de conjuntos
den entender mejor Un modelo de conjuntos es una manera de representar la suma de números completos. Su-
problemas sencillos de pón que Juana tiene 4 cubos en un montón y 3 en otro. Si junta los dos grupos de cubos,
suma y resta cuando ¿cuántos cubos hay en el grupo combinado? La figura 3-1 muestra la solución como podría
resuelven problemas aparecer en un libro de texto de educación básica. El conjunto combinado de cubos es la
donde hay que “aña- unión de los conjuntos ajenos de 4 cubos y 3 cubos. Después de juntar los conjuntos, los ni-
dir” o “quitar”, mode- ños cuentan los cubos y concluyen que hay 7 cubos en total. Nota la importancia de que los
lando directamente la conjuntos no tengan elementos en común. Si los conjuntos tienen elementos en común, podría-
situación o usando es- mos extraer una conclusión incorrecta.
trategias de continua-
ción del conteo o
contar hacia atrás n(A) = 4 n(B) = 3
(Carpenter y Moser A B
1984). ◆
a b
e f g
c d

a b
c d e

f g

n(A) + n(B) = 4 + 3 = 7 = n(A B)

Figura 3-1

Definimos formalmente la suma usando la terminología de conjuntos.

Definición de suma de números completos


Sean A y B dos conjuntos finitos ajenos. Si n1A2 = a y n1B2 = b, entonces a + b = n1A ´ B2.

Los números a y b en a + b son los sumandos y a + b es la suma.

AHORA INTENTA ÉSTE 3-1 Si en la definición anterior de suma de números completos los conjuntos
no son ajenos, explica por qué la definición es incorrecta.

◆ Nota Los historiadores piensan que la palabra cero viene de la palabra hindú sunya, que significa
histórica “vacío”. Después sunya se tradujo al árabe como sifr, que al traducirse al latín se convirtió
en zephirum, de la cual se deriva la palabra cero. ◆

© Lopez Mateos Editores. ISBN 978-607-95583-2-1, obra completa, versión electrónica, ISBN 978-607-95583-3-8, volumen 1, versión electrónica. Ejemplar asignado a: Helecto Villarroel gutierrez -
helecto@gmail.com. Fecha: 27 de octubre de 2014. Prohibida su modificación, copia o distribución.
Sección 3-1 Suma y resta de números completos 113

Modelo de la recta numérica (mediciones)


Para algunos problemas, el modelo de conjuntos podría no ser el mejor. Considera, por
ejemplo, las preguntas siguientes:
1. José tiene 4 metros de cinta roja y 3 metros de cinta blanca. ¿Cuántos metros de cinta
tiene en total?
2. Un día Gilda bebió 4 onzas de jugo de naranja en la mañana y 3 onzas en el almuerzo.
Si ella no tomó más jugo de naranja ese día, ¿cuántas onzas de jugo de naranja bebió en
todo el día?
Se puede usar una recta numérica para modelar la suma de números completos y respon-
der las preguntas 1 y 2. Cualquier recta marcada con dos puntos fundamentales, uno repre-
sentando 0 y otro representando 1, se puede habilitar como recta numérica. Los puntos que
representan 0 y 1 marcan los extremos de un segmento unitario. Se pueden marcar otros pun-
tos y darles nombre, como se muestra en la figura 3-2. Cualesquier dos puntos consecuti-
vos sobre la recta numérica de la figura 3-2 marcan los extremos de un segmento que tiene
la misma longitud que el segmento unitario.

4+3
4 3

0 1 2 3 4 5 6 7 8 9

Figura 3-2

Los problemas de suma se pueden modelar usando flechas dirigidas (vectores) sobre la
recta numérica. Por ejemplo, en la figura 3-2 se muestra la suma 4 + 3. Las flechas que re-
presentan los sumandos, 4 y 3, se combinan para formar una flecha que representa la suma
4 + 3. La figura 3-2 plantea un problema a los estudiantes. Si una flecha que comienza en 0
y termina en 3 representa al 3, ¿por qué va a representar al 3 una flecha que comienza en 4
y termina en 7? Los estudiantes necesitan entender que la suma representada por cuales-
quier dos flechas dirigidas se puede obtener al colocar en 0 el inicio de la primera flecha di-
rigida y después unir la flecha dirigida del segundo número, sin huecos ni traslapos. Así se
puede ver cuál es la suma de los números. Hemos descrito los sumandos como flechas (o
vectores) colocadas sobre la recta numérica, pero los estudiantes concatenan (conectan) las
flechas directamente en la recta.

AHORA INTENTA ÉSTE 3-2 Un error común es que los estudiantes representan 3 como una flecha sobre
la recta numérica que comienza en 1, como se muestra en la figura 3-3. Explica por qué esto no es apropiado.

0 1 2 3 4 5 6 7 8

Figura 3-3

◆ Nota El símbolo “ + ” apareció primero en un manuscrito de 1417 y era una forma de abre-
histórica viar la palabra en latín et, que significa “y”. La palabra minus significa “menos” en latín.
Primero se escribió como una m y después se abrevió como una barra horizontal. En
1489 Johannes Widman escribió un libro en el que usó los símbolos + y - para la suma
y la resta. ◆

© Lopez Mateos Editores. ISBN 978-607-95583-2-1, obra completa, versión electrónica, ISBN 978-607-95583-3-8, volumen 1, versión electrónica. Ejemplar asignado a: Helecto Villarroel gutierrez -
helecto@gmail.com. Fecha: 27 de octubre de 2014. Prohibida su modificación, copia o distribución.
114 Números completos y sus operaciones

Orden en los números completos


En los Puntos focales del currículo de la NCTM para el grado 1 hallamos lo siguiente:
Niñas y niños comparan y ordenan números completos (al menos hasta 100) para comprender y
resolver problemas que incluyan los tamaños relativos de estos números. Piensan los números
completos entre 10 y 100 como grupos de dieces y unos (reconociendo de manera especial los
números del 11 al 19 como el grupo 1 de diez y números particulares de unos). Entienden el or-
den secuencial de los números y de sus magnitudes relativas, y representan números sobre la
recta numérica. (p. 13)

En el capítulo 2 usamos el concepto de conjunto y el concepto de correspondencia


biunívoca para definir la relación mayor que. También podemos usar una recta numérica
horizontal para describir relaciones de mayor que y de menor que en el conjunto de los
números completos. Por ejemplo, en la figura 3-2 notamos que, en la recta numérica, el 4
está a la izquierda del 7. Decimos entonces “cuatro es menor que siete” y lo escribimos
4 6 7. Como 4 está a la izquierda de 7, hay un número natural que puede sumarse al 4
para obtener 7, a saber 3. Así, 4 6 7 porque 4 + 3 = 7. Podemos formalizar estos comen-
tarios para elaborar la siguiente definición de menor que.

Definición de menor que


Para cualesquier números completos a y b, a es menor que b, que escribimos a 6 b, si, y sólo si,
existe algún número natural k tal que a + k = b.

A veces se combina la igualdad con las desigualdades mayor que y menor que para dar las
relaciones mayor o igual que y menor o igual que, denotadas con Ú y …. Así, a … b sig-
nifica a 6 b o a = b. El énfasis respecto a estos símbolos es en el nexo “o” de modo que
3 … 5, 5 Ú 3 y 3 Ú 3 son proposiciones verdaderas.

Propiedades de la suma de números completos


Garantizamos que cada vez que se sumen dos números completos se obtendrá un número
completo. Esta propiedad suele llamarse cerradura de la suma de números completos. Decimos
que “el conjunto de los números completos es cerrado bajo la suma”.

Teorema 3–1:    Propiedad de la cerradura de la suma de números completos


Si a y b son números completos, entonces a + b es un número completo.

O B S E R VA C I Ó N La propiedad de la cerradura implica que existe la suma de dos números


completos y que esa suma es un número completo único; por ejemplo, 5 + 2 es un nú-
mero completo único e identificamos ese número como 7.

AHORA INTENTA ÉSTE 3-3 Di cuál de los conjuntos siguientes es cerrado bajo la suma:
a. E = 52, 4, 6, 8, 10, Á 6
b. F = 51, 3, 5, 7, 96

© Lopez Mateos Editores. ISBN 978-607-95583-2-1, obra completa, versión electrónica, ISBN 978-607-95583-3-8, volumen 1, versión electrónica. Ejemplar asignado a: Helecto Villarroel gutierrez -
helecto@gmail.com. Fecha: 27 de octubre de 2014. Prohibida su modificación, copia o distribución.
Sección 3-1 Suma y resta de números completos 115

En los Principios y objetivos hallamos lo siguiente:

Al comprender el significado de suma y resta con números completos, los estudiantes irán encon-
trando las propiedades de las operaciones, como la conmutatividad y la asociatividad de la suma.
Aunque hay estudiantes que descubren y usan las propiedades de manera natural, los maestros
pueden resaltar estas propiedades mediante discusiones en clase. (p. 83)

La figura 3-4(a) muestra dos sumas. Arriba de la recta numérica se ilustra 3 + 5 y debajo
de la recta numérica tenemos 5 + 3. La suma es la misma. La figura 3-4(b) muestra la
misma suma obtenida por medio de barras coloreadas, cuyo resultado es el mismo. Ambas
ilustraciones presentan la idea de que dos números completos se pueden sumar en cualquier
orden. Esta propiedad es cierta y se conoce como propiedad conmutativa de la suma de núme-
ros completos. Decimos que la “suma de números completos es conmutativa”. La palabra
conmutativa viene de conmutar, que significa “intercambiar”.

3+5
5
3 5
3

0 1 2 3 4 5 6 7 8 9 10 11 12
5 3 3
5+3 5
(a) (b)

Figura 3-4

Teorema 3–2:    Propiedad conmutativa de la suma de números completos


Si a y b son números completos cualesquiera, entonces a + b = b + a.

La propiedad conmutativa de la suma de números completos no es obvia para muchas ni-


ñas y niños. Pueden ser capaces de obtener la suma 9 + 2 y no poder hallar la suma 2 + 9.
Con la técnica de seguir contando se puede calcular 9 + 2 comenzando en el 9 y después se-
guir contando dos veces más y decir “diez” y “once”. Para calcular 2 + 9 sin la propiedad
conmutativa, seguir contando es más complicado. Los alumnos deben comprender que 2 + 9
es otra manera de llamar a 9 + 2.

AHORA INTENTA ÉSTE 3-4 Usa el modelo de conjuntos para mostrar la propiedad conmutativa en
3 + 5 = 5 + 3.

Se presenta otra propiedad de la suma cuando escogemos el orden para sumar tres o más
números. Por ejemplo, podríamos calcular 24 + 8 + 2 agrupando 24 y 8: 124 + 82 +
2 = 32 + 2 = 34. (Los paréntesis indican que los dos primeros números están agrupados.)
También podemos reconocer que es fácil sumar cualquier número más 10 y calcular
24 + 18 + 22 = 24 + 10 = 34. Este ejemplo ilustra la propiedad asociativa de la suma de nú-
meros completos. La palabra asociativa viene de asociar, que significa “unir”.

© Lopez Mateos Editores. ISBN 978-607-95583-2-1, obra completa, versión electrónica, ISBN 978-607-95583-3-8, volumen 1, versión electrónica. Ejemplar asignado a: Helecto Villarroel gutierrez -
helecto@gmail.com. Fecha: 27 de octubre de 2014. Prohibida su modificación, copia o distribución.
116 Números completos y sus operaciones

Teorema 3–3:    Propiedad asociativa de la suma de números completos


Si a, b y c son números completos, entonces 1a + b2 + c = a + 1b + c2.

Cuando se suman varios números, usualmente se omiten los paréntesis pues agruparlos no
altera el resultado.
Otra propiedad de la suma de números completos opera cuando un sumando es 0. En la fi-
gura 3-5, el conjunto A tiene 5 cubos y el conjunto B tiene 0 cubos. La unión de los conjuntos
A y B tiene sólo 5 cubos.
n(A) = 5 n(B) = 0 porque B =
A B
a b

o r

a b

o r

n(A) + n(B) = 5 + 0 = 5 = n(A B)

Figura 3-5

Este ejemplo ilustra la siguiente propiedad de los números completos:

Teorema 3–4:    Propiedad de la identidad aditiva de números completos


Existe un número completo único, el 0, llamado identidad aditiva, tal que para cualquier
número completo a, a + 0 = a = 0 + a.

Nota cómo se introducen las propiedades asociativas y de identidad en el grado 3, en la


página de muestra que sigue. Resuelve las partes 4–7.

Ejemplo 3-1 ¿Qué propiedades justifican lo siguiente?










a. 5 + 7 = 7 + 5
b. 1001 + 733 es un único número completo.
c. 13 + 52 + 7 = 15 + 32 + 7
d. 18 + 52 + 2 = 2 + 18 + 52 = 12 + 82 + 5

Solución a. Propiedad conmutativa de la suma


b. Propiedad de la cerradura de la suma
c. Propiedad conmutativa de la suma
d. Propiedades conmutativa y asociativa de la suma

© Lopez Mateos Editores. ISBN 978-607-95583-2-1, obra completa, versión electrónica, ISBN 978-607-95583-3-8, volumen 1, versión electrónica. Ejemplar asignado a: Helecto Villarroel gutierrez -
helecto@gmail.com. Fecha: 27 de octubre de 2014. Prohibida su modificación, copia o distribución.
Sección 3-1 Suma y resta de números completos 117

Página de un libro de texto PROPIEDADES DE LA SUMA

¿Cuál es la propiedad asociativa?


La propiedad asociativa (de agrupación) de la suma dice
que puedes agrupar los sumandos de cualquier manera y
la suma será la misma.

Los símbolos de agrupación, como


los paréntesis, ( ), indican qué
números sumar primero.

Así,
Tema de plática
Evaristo dice, “Puedes reescribir (5+3) + 2 como 8+2”. ¿Estás
de acuerdo? Explica.

¿Cuál es la propiedad de la identidad?


La propiedad de la identidad (el cero) de
la suma dice que la suma de cualquier
número y el cero es ese mismo número.

Tema de plática
¿Cómo puedes usar la propiedad de la identidad de la suma para obtener 536 + 0?
Ver otro ejemplo en el Conjunto 2-1, en la p. 116.
VERIFICACIÓN
Obtén cada suma.

Escribe el número faltante.

Sentido numérico ¿Qué propiedad de la suma se utiliza en la siguiente


proposición numérica? Explica. 4 +(5 +2)= (5 +2)+ 4
Fuente: Scott Foresman-Addison Wesley Math, Grade 3, 2008 (p. 67).

© Lopez Mateos Editores. ISBN 978-607-95583-2-1, obra completa, versión electrónica, ISBN 978-607-95583-3-8, volumen 1, versión electrónica. Ejemplar asignado a: Helecto Villarroel gutierrez -
helecto@gmail.com. Fecha: 27 de octubre de 2014. Prohibida su modificación, copia o distribución.
118 Números completos y sus operaciones

Perfeccionar sumas básicas


Un aspecto matemático importante son las sumas básicas. Las sumas básicas son las que in-
cluyen la suma de un dígito más otro dígito, mismas que conocemos como “tablas de su-
mar”. En la tira cómica de “Daniel el Travieso” se nota que Daniel todavía no conoce las
tablas de sumar.
DANIEL EL TRAVIESO

“DECÍDETE. ¡PRIMERO ME DICES QUE 3 MÁS 3 SON


SEIS, Y AHORA DICES QUE 4 MÁS 2 SON SEIS!”

◆ Nota de Un método para aprender sumas básicas es organizarlas de acuerdo con diferentes sumas
investigación derivadas basadas en estrategias, listadas a continuación.
Los estudiantes que
están aprendiendo las 1. Seguir contando. La estrategia de seguir contando a partir del mayor de los suman-
tablas de sumar ten- dos puede usarse en cualquier momento para sumar números completos, pero es inefi-
drían que familiari- ciente. La usamos cuando el otro sumando es 1, 2 ó 3. Por ejemplo, en la tira cómica
zarse con estrategias Daniel pudo calcular 4 + 2 comenzando en 4 y continuar contando 5, 6. Asimismo,
que incluyan sumas podemos calcular 3 + 3 comenzando en 3 y continuar contando 4, 5, 6.
derivadas. Por ejem- 2. Dobles. En la siguiente estrategia se usan los dobles. Los dobles, como el 3 + 3 de la
plo, 5 + 6 se puede tira cómica, llaman de manera especial la atención de los estudiantes. Después de
transformar en
15 + 52 + 1, que
dominar los dobles, es fácil aprender dobles + 1 y dobles + 2. Por ejemplo, si un estudi-
puede resolverse ob- ante sabe que 6 + 6 = 12, entonces 6 + 7 es 16 + 62 + 1, o uno más que el doble de
teniendo la suma más 6, ó 13. Asimismo, 7 + 9 es 17 + 72 + 2, ó 2 más que el doble de 7, ó 16.
fácil del doble 3. Completar 10. Otra estrategia es completar 10 y después sumar el resto. Por ejemplo,
5 + 5 = 10 y 1. podemos pensar 8 + 5 como se muestra en la figura 3-6. Nota que, en realidad,
Como estrategia, per-
usamos la propiedad asociativa de la suma.
mite al estudiante de-
sarrollar un sentido
numérico y relaciones 8 + 5 8 + ( 2 + 3 )
importantes entre
ciertas combinaciones
básicas de sumas; esto
+ +
( +
(
lo ayuda a recordar ( 8 + 2 ) + 3 10 + 3 = 13
las sumas y lo dota de
un mecanismo al cual
puede recurrir
(Fuson 1992). ◆
( +
( + +

Figura 3-6

© Lopez Mateos Editores. ISBN 978-607-95583-2-1, obra completa, versión electrónica, ISBN 978-607-95583-3-8, volumen 1, versión electrónica. Ejemplar asignado a: Helecto Villarroel gutierrez -
helecto@gmail.com. Fecha: 27 de octubre de 2014. Prohibida su modificación, copia o distribución.
Sección 3-1 Suma y resta de números completos 119

4. Contar hacia atrás. La estrategia de contar hacia atrás se usa cuando un número es 1 ó
2 menor que 10. Por ejemplo, como 9 es 1 menor que 10, entonces 9 + 7 es 1
menos que 10 + 7, ó 16. Usando símbolos, 9 + 7 = 110 + 72 - 1 = 17 - 1 = 16.
También, 8 + 7 = 110 + 72 - 2 = 17 - 2 = 15.
Muchas sumas básicas se pueden clasificar en más de una estrategia. Por ejemplo, pode-
mos hallar 9 + 8 completando 10 como 9 + 11 + 72 = 19 + 12 + 7 = 10 + 7 = 17, o po-
demos usar dobles + 1 como 18 + 82 + 1.

Resta de números completos


En los Puntos focales para el grado 1 hallamos lo siguiente:

Al comparar varias estrategias de solución, niñas y niños relacionan la suma y la resta como ope-
raciones inversas. (p. 13)

En la escuela básica, las operaciones que se “deshacen” entre sí se llaman operaciones in-
versas. La resta o substracción es la operación inversa de la suma. Como se ve en la siguien-
te tira cómica, a veces resulta difícil para los estudiantes entender la relación inversa entre
las dos operaciones.

¿CÓMO VA LA ESCUELA, HIJO? ¿QUÉ QUIERES AYER APRENDIMOS PARECE QUE NO


¿Y?
DECIR? A SUMAR, HOY ESTAMOS
APRENDIMOS A LLEGANDO A
RESTAR. NINGÚN LADO.

¡ES UNA
LATA!

En los Principios y objetivos hallamos lo siguiente:

Los estudiantes comprenden mejor la suma cuando resuelven problemas de sumando faltante
que surgen en relatos o en la vida real. La resta se entiende mejor por medio de situaciones en que
se necesita igualar dos colecciones o que una colección debe lograr un tamaño determinado. Al-
gunos problemas, como “Carlos tenía tres galletas. María le dio algunas más, y ahora tiene ocho.
¿Cuántas galletas le dio?”, pueden ayudar a los estudiantes a ver la relación entre la suma y la resta.
(p. 83)

La resta de números completos se puede modelar por medio de varias estrategias de solu-
ción, incluyendo el modelo de quitar elementos, el modelo del sumando faltante, el modelo de
comparación y el modelo de la recta numérica (medición).

© Lopez Mateos Editores. ISBN 978-607-95583-2-1, obra completa, versión electrónica, ISBN 978-607-95583-3-8, volumen 1, versión electrónica. Ejemplar asignado a: Helecto Villarroel gutierrez -
helecto@gmail.com. Fecha: 27 de octubre de 2014. Prohibida su modificación, copia o distribución.
120 Números completos y sus operaciones

Modelo de quitar elementos


En la suma, imaginamos un segundo conjunto de objetos añadidos a un primer conjunto,
pero en la resta imaginamos un segundo conjunto de elementos como algo que se va a qui-
tar del primero. Por ejemplo, supongamos que tenemos 8 cubos y quitamos 3. Lo ilustra-
mos en la figura 3-7 y registramos este proceso como 8 - 3 = 5.

8 cubos Quitamos
3 cubos

8 – 3 = 5 ; quedan 5 cubos

Figura 3-7

AHORA INTENTA ÉSTE 3-5 Recuerda que la suma de números completos se definió usando el con-
cepto de unión de dos conjuntos ajenos. De manera análoga, plantea una definición de resta de números
completos usando los conceptos de subconjuntos y de diferencia de conjuntos.

Modelo del sumando faltante


Un segundo modelo para la resta, el modelo del sumando faltante, relaciona la resta y la su-
ma. En la figura 3-7 representamos 8 - 3 como 8 cubos de los cuales “quitamos” 3 cubos.
El número de cubos restantes es el número 8 - 3, ó 5. Esto también puede pensarse como
el número de cubos que deberíamos añadir a 3 cubos para obtener 8 cubos, esto es,
3 + 8 - 3 = 8
El número 8 - 3, ó 5, es el sumando faltante en la ecuación
3 + n = 8.
También podemos relacionar el enfoque del sumando faltante con los conjuntos o con una
recta numérica. La resta 8 - 3 se ilustra en la figura 3-8(a) usando conjuntos y en la figura
3-8(b) usando la recta numérica.
(a) n(A) = 8 (b) 3 ?
A 8
¿Cuántos?
0 1 2 3 4 5 6 7 8 9 10

? 8 - 3 = ?
si, y sólo si,
3 + ? = 8

Figura 3-8

El modelo del sumando faltante proporciona una oportunidad para que los estudiantes de
la escuela básica comiencen a practicar el razonamiento algebraico. Una incógnita es una
parte principal del problema de hallar la diferencia 8 - 3.
© Lopez Mateos Editores. ISBN 978-607-95583-2-1, obra completa, versión electrónica, ISBN 978-607-95583-3-8, volumen 1, versión electrónica. Ejemplar asignado a: Helecto Villarroel gutierrez -
helecto@gmail.com. Fecha: 27 de octubre de 2014. Prohibida su modificación, copia o distribución.
Sección 3-1 Suma y resta de números completos 121

Los cajeros a veces usan el modelo del sumando faltante. Por ejemplo, si la entrada al cine
es de $30 y pagan con un billete de $50, el cajero puede calcular el cambio diciendo “30 y 20
son 50”. Esta idea puede generalizarse: para cualesquier números completos a y b tales que
a Ú b, a - b es el único número completo tal que b + 1a - b2 = a. Esto es, a - b es la
única solución de la ecuación b + n = a. La definición se puede escribir como sigue:

Definición de resta de números completos


Para cualesquier números completos a y b tales que a Ú b, a - b es el único número completo c
tal que b + c = a.

Observa la siguiente página de muestra para que veas cómo se enseña a los estudiantes de
grado 3 la relación existente entre suma y resta por medio de una familia de hechos. Res-
ponde las preguntas planteadas en Tema de plática al final de la página.

Modelo de comparación
Otra manera de considerar la resta es usando el modelo de comparación. Supongamos que
Juan tiene 8 cubos y Susana tiene 3 cubos, y queremos saber cuántos cubos más tiene Juan
respecto a Susana. Podemos parear los cubos de Susana con algunos de los cubos de Juan,
como se muestra en la figura 3-9, y determinar que Juan tiene 5 cubos más que Susana.
También escribimos esto como 8 - 3 = 5.
8 cubos de Juan

Diferencia de 5 cubos

Cubos de Susana
Figura 3-9

Modelo de la recta numérica (medición)


También podemos modelar la resta en una recta numérica, como se sugiere en la figu-
ra 3-10, donde se muestra que 5 - 3 = 2.
5–3 3
5

0 1 2 3 4 5 6 7

Figura 3-10
Los siguientes cuatro problemas ilustran por qué deben considerarse los cuatro modelos
para la resta o substracción. En los cuatro problemas la respuesta es 5, pero cada uno se
puede pensar usando un modelo diferente.
1. Modelo de quitar elementos. Alicia tenía $9 y gastó $4. ¿Cuánto le quedó?
2. Modelo del sumando faltante. Alicia ha leído 4 capítulos de un libro de 9 capítulos.
¿Cuántos capítulos le quedan por leer?
3. Modelo de comparación. Alicia tiene 9 libros y Beti tiene 4 libros. ¿Cuántos libros
más tiene Alicia respecto a Beti?
4. Modelo de la recta numérica. Alicia recorrió en bicicleta 9 km en dos días. El
segundo día recorrió 4 km. ¿Cuánto recorrió el primer día?

© Lopez Mateos Editores. ISBN 978-607-95583-2-1, obra completa, versión electrónica, ISBN 978-607-95583-3-8, volumen 1, versión electrónica. Ejemplar asignado a: Helecto Villarroel gutierrez -
helecto@gmail.com. Fecha: 27 de octubre de 2014. Prohibida su modificación, copia o distribución.
122 Números completos y sus operaciones

Página de un libro de texto R E L AC I Ó N E N T R E L A


S U M A Y L A R ESTA

Lección
Calentamiento
´ Relación entre la suma
Idea clave
Las cuentas relacionadas, o
y la resta
familias de hechos, muestran
cómo están relacionadas la Aprende Hay 9 carros rojos
suma y la resta. y 8 carros azules.
¿Cómo están relacionadas ¿Cuántos carros
hay en total?
Vocabulario la suma y la resta?
familia de hechos
diferencia Puedes pensar partes del todo para ilustrar
cómo están relacionadas la suma y la resta.
ex
ef l i ó
R

¡Piensa! Todo
Puedo trazar una
figura del tipo todo-en
partes para ilustrar la
suma y la resta.
Parte Parte
Puedes escribir una familia de hechos
cuando conoces las partes y el todo.
Familia de hechos:

diferencia

Ejemplo
Halla
Lo que piensas Lo que escribes

Tema de plática
¿Cuáles son las otras tres proposiciones numéricas en la
familia de hechos que contiene 6 + 3 = 9?
¿Qué suma básica te puede ayudar a obtener 1123?

Fuente: Scott Foresman-Addison Wesley Math, Grade 3, 2008 (p. 70).

© Lopez Mateos Editores. ISBN 978-607-95583-2-1, obra completa, versión electrónica, ISBN 978-607-95583-3-8, volumen 1, versión electrónica. Ejemplar asignado a: Helecto Villarroel gutierrez -
helecto@gmail.com. Fecha: 27 de octubre de 2014. Prohibida su modificación, copia o distribución.
Sección 3-1 Suma y resta de números completos 123

Propiedades de la resta
En un intento por hallar 3 - 5, usamos la definición de resta: 3 - 5 = c si, y sólo si,
c + 5 = 3. Como no existe algún número completo c que satisfaga la ecuación, entonces
3 - 5 no tiene sentido en el conjunto de números completos. En general, se puede demos-
trar que si a 6 b, entonces a - b no tiene sentido en el conjunto de los números comple-
tos. Por lo tanto, la resta no es cerrada en el conjunto de los números completos.

AHORA INTENTA ÉSTE 3-6 ¿Cuáles de las siguientes propiedades se cumplen para la resta de números
completos? Explica.
a. Propiedad de la cerradura b. Propiedad asociativa
c. Propiedad conmutativa d. Propiedad de la identidad

Álgebra elemental usando suma y resta de


números completos
Expresiones como 9 + 5 = x y 12 - y = 4 pueden ser verdaderas o falsas dependiendo de
los valores de x y y. Por ejemplo, si x = 10, entonces 9 + 5 = x es falsa. Si y = 8, entonces
12 - y = 4 es verdadera. Si el valor usado hace que la ecuación sea verdadera, es una
solución de la ecuación.

AHORA INTENTA ÉSTE 3-7 Halla la solución en cada caso, donde x es un número completo:
a. x + 8 = 13 b. 15 - x = 8 c. x 7 9 y x 6 11

Evaluación 3-1A

1. Da un ejemplo que muestre por qué, en la definición de 6. Cada uno de los casos siguientes ejemplifica una de las
suma, los conjuntos A y B deben ser ajenos. propiedades de la suma de números completos. Identifica
2. ¿Para qué caso es cierto que n 1A2 + n1B2 = n1A ´ B2? la propiedad ilustrada.
a. A = 5a, b, c6, B = 5d, e6 a. 6 + 3 = 3 + 6
b. A = 5a, b, c6, B = 5b, c6 b. 16 + 32 + 5 = 6 + 13 + 52
c. A = 5a, b, c6, B = ⭋ c. 16 + 32 + 5 = 13 + 62 + 5
3. Si n1A2 = 3, n1B2 = 5 y n1A ´ B2 = 6, ¿qué sabes d. 5 + 0 = 5 = 0 + 5
acerca de n1A ¨ B2? e. 5 + 0 = 0 + 5
4. Si n1A2 = 3 y n1A ´ B2 = 6, f. 1a + c2 + d = a + 1c + d2
a. ¿Cuáles son los posibles valores de n1B2? 7. En la definición de menor que, ¿puede reemplazarse el
b. Si A ¨ B = ⭋, ¿cuáles son los posibles valores de n1B2? número natural k por el número completo k? ¿Por qué sí
5. Explica si los conjuntos dados son cerrados bajo la suma: o por qué no?
a. B = 506 8. a. Recuerda cómo definimos las relaciones de menor que
b. T = 50, 3, 6, 9, 12, Á 6 y mayor que. Da una definición análoga usando el
c. N = 51, 2, 3, 4, 5, Á 6 concepto de resta:
d. V = 53, 5, 76 i. a 6 b
e. 5x ƒ x 僆 C y x 7 106 ii. a 7 b
b. Usa la resta para definir a Ú b.

© Lopez Mateos Editores. ISBN 978-607-95583-2-1, obra completa, versión electrónica, ISBN 978-607-95583-3-8, volumen 1, versión electrónica. Ejemplar asignado a: Helecto Villarroel gutierrez -
helecto@gmail.com. Fecha: 27 de octubre de 2014. Prohibida su modificación, copia o distribución.
124 Números completos y sus operaciones

9. Halla los tres términos siguientes en cada una de las 13. a. En un juego de voleibol las jugadoras se formaron en
sucesiones aritméticas: una fila ordenadas por estatura. Si Queta es más baja
a. 8, 13, 18, 23, 28, ____, ____, ____ que Miriam, Sandra es más alta que Miriam y Vera es
b. 98, 91, 84, 77, 70, 63, ____, ____, ____ más alta que Sandra, ¿quién es la más alta y quién la
10. Si A, B y C representan cada uno un solo dígito del 1 al más baja?
9, y si A + B = C , contesta: b. Escribe posibles estaturas para las jugadoras de la
a. ¿Cuál es el mayor dígito que puede ser C? ¿Por qué? parte (a).
b. ¿Cuál es el mayor dígito que puede ser A? ¿Por qué? 14. Reescribe cada uno de los problemas de resta como un
c. ¿Cuál es el menor dígito que puede ser C? ¿Por qué? problema equivalente de suma:
d. Si A, B y C son pares, ¿qué número(s) puede(n) ser C? a. 9 - 7 = x
¿Por qué? b. x - 6 = 3
e. Si C es 5 más que A, ¿qué número(s) puede(n) ser B? c. 9 - x = 2
¿Por qué? 15. Revisa la Página de un libro de texto anterior para recor-
f. Si A es 3 veces B, ¿qué número(s) puede(n) ser C? dar la descripción de familia de hechos.
¿Por qué? a. Escribe la familia de hechos para 8 + 3 = 11.
g. Si A es impar y A es 5 más que B, ¿qué número(s) b. Escribe la familia de hechos para 13 - 8 = 5.
puede(n) ser C? ¿Por qué? 16. ¿Qué condiciones, si existen, deben pedirse a a, b y c en
11. Suponiendo que el patrón en la siguiente figura continúa, los casos siguientes para asegurar que el resultado sea un
halla el total de los términos en el renglón número 50: número completo?
a. a - b b. a - 1b - c2
1 1er renglón
17. Ilustra 8 - 5 = 3 usando cada uno de los modelos si-
1 - 1 2o renglón guientes:
1 - 1 + 1 3er renglón a. Quitar elementos b. Sumando faltante
1 - 1 + 1 - 1 4o renglón c. Comparación d. Recta numérica
1 - 1 + 1 - 1 + 1 5o renglón 18. Halla la solución para cada caso:
a. 3 + 14 + 72 = 13 + x2 + 7
12. Completa los cuadrados mágicos (en el capítulo 1 se de- b. 8 + 0 = x
finió cuadrado mágico): c. 5 + 8 = 8 + x
a. b. d. x + 8 = 12 + 5
1 6 17 10

5 7 14

4 2 13 18

Evaluación 3-1B

1. ¿Para qué caso es cierto que n 1A2 + n1B2 = n1A ´ B2? 5. El conjunto A es cerrado bajo la suma y contiene a los núme-
a. A = 5a, b6, B = 5d, e6 ros 2, 5 y 8. Lista otros seis elementos que deban estar en A.
b. A = 5a, b, c6, B = 5b, c, d6 6. Cada uno de los casos siguientes ejemplifica una de las
c. A = 5a6, B = ⭋ propiedades de la suma de números completos. Llena el
2. Si n1A2 = 3, n1B2 = 5 y n1A ¨ B2 = 1, ¿qué sabes espacio en blanco para obtener una proposición verda-
acerca de n(A ´ B2? dera e identifica la propiedad.
3. Explica si los conjuntos dados son cerrados bajo la suma: a. 3 + 4 = ____ + 3
a. B = 50, 16 b. 5 + 14 + 32 = 14 + 32 + ____
b. T = 50, 4, 8, 12, 16, Á 6 c. 8 + ____ = 8
c. F = 55, 6, 7, 8, 9, 10, Á 6 d. 3 + 14 + 52 = 13 + ____2 + 5
d. 5x ƒ x 僆 C y x 7 1006 e. 3 + 4 es un número _____ único.
4. El conjunto A tiene como elemento al 1. ¿Qué otros 7. Cada uno de los casos siguientes ejemplifica una de las
números completos deben estar en el conjunto A para que propiedades de la suma de números completos. Identifica
sea cerrado bajo la suma? la propiedad ilustrada.
a. 6 + 8 = 8 + 6
b. 16 + 32 + 0 = 6 + 3

© Lopez Mateos Editores. ISBN 978-607-95583-2-1, obra completa, versión electrónica, ISBN 978-607-95583-3-8, volumen 1, versión electrónica. Ejemplar asignado a: Helecto Villarroel gutierrez -
helecto@gmail.com. Fecha: 27 de octubre de 2014. Prohibida su modificación, copia o distribución.
Sección 3-1 Suma y resta de números completos 125

c. 16 + 82 + 2 = 18 + 62 + 2 b. x - 5 = 8
d. 15 + 32 + 2 = 5 + 13 + 22 c. 11 - x = 2
8. Halla los tres términos siguientes en cada una de las su- 12. Revisa la Página de un libro de texto anterior para recor-
cesiones aritméticas: dar la descripción de familia de hechos.
a. 5, 12, 19, 26, 33, ____, ____, ____ a. Escribe la familia de hechos para 9 + 8 = 17.
b. 63, 59, 55, 51, 47, ____, ____, ____ b. Escribe la familia de hechos para 15 - 7 = 8.
9. Si A, B, C y D representan cada uno un solo dígito del 1 13. Muestra que cada uno de los casos siguientes es verdadero.
al 9, responde lo siguiente si Da una propiedad de la suma que justifique cada paso.
a. a + 1b + c2 = c + 1a + b2
A b. a + 1b + c2 = 1c + b2 + a
+ B 14. Ilustra 7 - 3 = 4 usando cada uno de los modelos si-
CD guientes:
a. ¿Cuál es el valor de C? ¿Por qué? a. Quitar elementos
b. ¿Puede ser D igual a 1? ¿Por qué? b. Sumando faltante
c. Si D es 7, ¿qué valores puede tomar A? c. Comparación
d. Si A es 6 veces más que B, ¿cuál es el valor de D? d. Recta numérica
10. a. Un juego de dominó contiene todos los pares de 15. Halla la solución para cada caso:
números desde el doble 0 hasta el doble 6, donde cada a. 12 - x = x + 6
par de números está presente una sola vez; por ejem- b. 19 - x2 - 6 = 1
plo, la siguiente ficha cuenta como 2-4 y 4-2. ¿Cuán- c. 3 + x = x + 3
tas fichas hay en ese juego? d. 15 - x = x - 7
e. 14 - x = 7 - x
16. Roberto tiene 11 lápices. Queta tiene 5 lápices. ¿Qué
número le corresponde a la expresión que ilustra cuán-
b. Explica la importancia de la propiedad conmutativa tos lápices más tiene Roberto?
cuando consideramos la suma de todos los puntos en (i) 11 + 5 = 16
una ficha de dominó.
(ii) 16 - 5 = 11
11. Reescribe cada uno de los problemas de resta como un
(iii) 11 - 5 = 6
problema equivalente de suma:
(iv) 11 - 6 = 5
a. 9 - 3 = x

Conexiones matemáticas 3-1


Comunicación b. Usa un modelo apropiado para la resta a fin de ex-
1. En una encuesta aplicada a 52 estudiantes, 22 dijeron que plicar por qué
cursaban álgebra y 30 dijeron que cursaban biología. ¿Es a - b - c = a - 1b + c2
necesariamente cierto que los 52 estudiantes cursaban ál-
gebra o biología? ¿Por qué? 5. Explica por qué crees que sea importante que los estu-
2. Para hallar 9 + 7 una estudiante dice que piensa 9 + 7 diantes de educación básica aprendan más de un modelo
como 9 + 11 + 62 = 19 + 12 + 6 = 10 + 6 = 16. para efectuar las operaciones de suma y resta.
¿Qué propiedad o propiedades está usando? 6. ¿Deben los estudiantes de nivel básico aprender las sumas
3. En la figura 3-2 se usaron flechas para representar nú- y restas básicas (es decir, las tablas), aun cuando aprender
meros y completar una suma. ¿Consideras que una fle- a usar la calculadora sea parte de su programa de estudios?
cha que comienza en 0 y termina en 3 representa el ¿Por qué sí o por qué no?
mismo número que una flecha que comienza en 4 y ter- 7. Explica cómo es posible usar el modelo siguiente para
mina en 7? ¿Cómo lo explicarías a un alumno? ilustrar cada una de las sumas y restas:
4. Cuando aparecen restas y sumas en una expresión sin pa- a. 9 + 4 = 13 b. 4 + 9 = 13
réntesis, hay el acuerdo de que las operaciones se efec- c. 4 = 13 - 9 d. 9 = 13 - 4
túen en orden de aparición, de izquierda a derecha.
Tomando esto en cuenta, responde lo siguiente:
a. Usa un modelo apropiado para la resta a fin de expli- 13
car por qué
a - b - c = a - c - b
9 4
suponiendo que las expresiones tengan sentido.

© Lopez Mateos Editores. ISBN 978-607-95583-2-1, obra completa, versión electrónica, ISBN 978-607-95583-3-8, volumen 1, versión electrónica. Ejemplar asignado a: Helecto Villarroel gutierrez -
helecto@gmail.com. Fecha: 27 de octubre de 2014. Prohibida su modificación, copia o distribución.
126 Números completos y sus operaciones

8. ¿Cómo están relacionadas la suma y la resta? Explica. 15. Cada una de las personas del grupo escoja un libro de
9. ¿Por qué el 0 no es una identidad para la resta? Explica. texto de diferente grado y describa cómo y cuándo intro-
duce la resta de números completos. Compárenlos con
Solución abierta las diferentes maneras en que se trató la resta en esta sec-
10. Describe algún modelo no incluido en este libro que ción.
usarías para enseñar a sumar a tus alumnos.
11. Supón que A 8 B. Si n1A2 = a y n1B2 = b, entonces Preguntas del salón de clase
b - a podría definirse como n1B - A2. Escoge dos con- 16. Un alumno dice que el 0 es la identidad para la resta.
juntos A y B e ilustra esta definición. ¿Cómo le respondes?
12. a. Redacta un problema para el cual el modelo de conjuntos 17. Un alumno afirma que en la siguiente recta numérica, la
sería el más apropiado para mostrar que 25 + 8 = 33. flecha realmente no representa al 3 pues el inicio de la fle-
b. Redacta un problema para el cual el modelo de la recta cha no comienza en 0. ¿Cómo le respondes?
numérica (medición) sería el más apropiado para
mostrar que 25 + 8 = 33. 0 1 2 3 4 5

Aprendizaje colectivo 18. Una alumna pregunta por qué usamos la resta para saber
cuántos lápices más tiene Roberto que Queta si no se ha
13. Analiza con tu grupo cada uno de los casos siguientes.
quitado nada. ¿Cómo le respondes?
Usa la tabla de sumas básica.
19. Una alumna afirma que la resta es cerrada respecto a los nú-
ⴙ 0 1 2 3 4 5 6 7 8 9 meros completos. Para mostrar que esto es cierto, ella
muestra que 8 - 5 = 3, 5 - 2 = 3, 6 - 1 = 5 y
0 0 1 2 3 4 5 6 7 8 9 12 - 7 = 5, y dice que ella puede seguir todo el día mos-
1 1 2 3 4 5 6 7 8 9 10 trando ejemplos como estos, en que se obtengan números
2 2 3 4 5 6 7 8 9 10 11 completos cuando se efectúe la resta. ¿Cómo le respondes?
3 3 4 5 6 7 8 9 10 11 12 20. Jonathan asegura que puede obtener la misma respuesta al
4 4 5 6 7 8 9 10 11 12 13 problema siguiente sumando hacia arriba (comenzando
5 5 6 7 8 9 10 11 12 13 14 con 4 + 7 ) o sumando hacia abajo (comenzando con
6 6 7 8 9 10 11 12 13 14 15 8 + 7). Él quiere saber por qué, y si esto funciona siem-
7 7 8 9 10 11 12 13 14 15 16 pre. ¿Cómo le contestas?
8 8 9 10 11 12 13 14 15 16 17
9 9 10 11 12 13 14 15 16 17 18 8
7
a.¿Cómo se muestra en la tabla la propiedad de la cerradura?
b.¿Cómo se muestra en la tabla la propiedad conmutativa? + 4
c.¿Cómo se muestra en la tabla la propiedad de la identidad?
Preguntas del Third International Mathematics and
d.¿Cómo pueden ayudar las propiedades de la suma para
Science Study (TIMSS) (Tercer Estudio Internacional
que los estudiantes aprendan la tabla de sumas básicas?
sobre las Matemáticas y la Ciencia)
14. Supón que un sistema numérico usa sólo cuatro símbolos,
a, b, c y d, y que la operación ¢ y el sistema funcionan se- Alberto tiene 50 manzanas. Vendió algunas y le queda-
gún se muestra en la tabla. Analiza con tu grupo cada caso. ron 20. ¿Cuál de las siguientes expresiones numéricas
muestra esto?
a. n - 20 = 50
¢ a b c d
b. 20 - n = 50
a a b c d c. n - 50 = 20
b b c d a d. 50 - n = 20
c c d a b TIMSS 2003, Grado 4
d d a b c
La regla de la tabla es que los números en cada renglón
y cada columna deben sumar lo mismo. ¿Qué número va
a. ¿El sistema es cerrado? ¿Por qué?
en el centro de la tabla?
b. ¿El sistema es conmutativo? ¿Por qué?
a. 1 4 11 6
c. ¿Hay un elemento identidad en el sistema? De ser
b. 2
así, ¿cuál es? 9 5
c. 7
d. Mediante varios ejemplos, investiga el comportamiento
d. 12
de la propiedad asociativa de esta operación. 8 3 10
TIMSS 2003, Grado 4

© Lopez Mateos Editores. ISBN 978-607-95583-2-1, obra completa, versión electrónica, ISBN 978-607-95583-3-8, volumen 1, versión electrónica. Ejemplar asignado a: Helecto Villarroel gutierrez -
helecto@gmail.com. Fecha: 27 de octubre de 2014. Prohibida su modificación, copia o distribución.
Sección 3-2 Algoritmos para la suma y la resta de números completos 127

ROMPECABEZAS  Usa la figura 3-11 para diseñar un cuadrado antimágico. Esto es, usa
cada uno de los dígitos 1, 2, 3, 4, 5, 6, 7, 8 y 9 exactamente una vez de manera que toda
columna, renglón y diagonal tenga una suma diferente.

Figura 3-11

3-2 Algoritmos para la suma y la resta de números completos


En los Puntos focales en el currículo correspondiente al grado 2 hallamos lo siguiente respecto
a la soltura para efectuar sumas y restas de números con varios dígitos:
Las niñas y los niños usan la comprensión que tienen de la suma para desarrollar una manera rápida
de recordar las tablas de la suma y las correspondientes tablas de la resta. Resuelven problemas arit-
méticos al aplicar la comprensión que tienen de los modelos de la suma y la resta (tales como com-
binar o separar conjuntos, o usar rectas numéricas), de las relaciones y propiedades del número
(como el valor posicional), y de las propiedades de la suma (conmutatividad y asociatividad). Desa-
rrollan, discuten y usan de manera eficiente, precisa y generalizable métodos para sumar y restar nú-
meros completos de varios dígitos. Seleccionan y aplican métodos apropiados para estimar sumas y
diferencias o para calcularlas mentalmente dependiendo del contexto y de los números implícitos.
Desarrollan soltura en el manejo de procedimientos eficientes, incluyendo los algoritmos conven-
cionales, para sumar y restar números completos; entienden por qué funcionan los procedimientos
(con base en el valor posicional y las propiedades de las operaciones) y los usan para resolver pro-
blemas. (p. 14)
En los Principios y objetivos también hallamos algo acerca de soltura computacional y los algo-
ritmos convencionales.
Para finales del grado 2 los estudiantes deberán saber las combinaciones básicas de suma y resta, te-
ner habilidad para sumar números de dos dígitos y conocer métodos para restar números de dos dí-
gitos. En el nivel de los grados 3 a 5, conforme los estudiantes desarrollen las combinaciones básicas
de números para multiplicar y dividir, también deberán desarrollar algoritmos confiables para resol-
ver, de manera eficiente y precisa, problemas aritméticos. Estos métodos deberán aplicarse a núme-
ros grandes y practicarlos para lograr un manejo hábil . . . los estudiantes deben lograr soltura para
efectuar cálculos aritméticos —deben tener métodos eficientes y precisos, basados en una com-
prensión de los números y las operaciones. Los algoritmos “convencionales” para cálculos aritméti-
cos son un medio de lograr esta soltura. (p. 35)
En la sección anterior introdujimos las operaciones de suma y resta de números comple-
tos y ahora, según se señala en los Puntos focales y en los Principios y objetivos, es el momento
de concentrarnos en lograr soltura computacional —disponer y usar métodos eficientes y pre-
cisos para calcular. Los Principios y objetivos sugieren que manejar los “algoritmos conven-
cionales” es un medio para lograr esta soltura. Un algoritmo (llamado así en honor del
matemático persa del siglo noveno Muhammad al-Jwârizmî) es un procedimiento sistemá-
tico utilizado para efectuar una operación. En el Anuario de 1998 del National Council of Te-
achers of Mathematics (NCTM) (Consejo Nacional de Maestros de Matemáticas de Estados
Unidos), Teaching and Learning Algorithms in School Mathematics, (Enseñanza y aprendizaje
de algoritmos en las matemáticas escolares), Usiskin afirmó que “los algoritmos son gene-
ralizaciones que dan cuerpo a una de las principales razones para estudiar matemáticas
—hallar la manera de resolver clases de problemas. Cuando conocemos un algoritmo
podemos completar no sólo una tarea, sino todas las tareas de un tipo determinado y tene-
mos la garantía de obtener una respuesta o respuestas. El poder de un algoritmo radica en
la amplitud de su aplicabilidad”. (p. 10)

© Lopez Mateos Editores. ISBN 978-607-95583-2-1, obra completa, versión electrónica, ISBN 978-607-95583-3-8, volumen 1, versión electrónica. Ejemplar asignado a: Helecto Villarroel gutierrez -
helecto@gmail.com. Fecha: 27 de octubre de 2014. Prohibida su modificación, copia o distribución.
128 Números completos y sus operaciones

En esta sección nos concentramos en desarrollar y comprender algoritmos para la suma y


la resta. Además de los algoritmos convencionales, desarrollaremos otros alternativos.

Algoritmos para la suma


Para enseñar matemáticas a niñas y niños pequeños, es importante apoyarlos en la transición
del pensamiento concreto al abstracto mediante el uso de técnicas similares a su proceso de
maduración. Para que niñas y niños comprendan cómo usar los algoritmos de papel y lápiz de-
berán, primero, explorar la suma usando recursos didácticos manipulables. Si pueden tocar y
mover objetos como fichas, cuentas, un ábaco o cubos de base diez, se les podrá conducir (y
con frecuencia lo harán, de manera natural, por sí mismos) a la creación de los algoritmos para
la suma. A continuación usaremos cubos de base diez para ilustrar el desarrollo de un algoritmo
para sumar números completos.
Supón que vamos a sumar 14 + 23. Comenzaremos con el modelo concreto de la figura
3-12(a), pasaremos al algoritmo expandido de la figura 3-12(b) y después abordaremos el algo-
ritmo convencional de la figura 3-12(c).
(a) 14 23 (b) 14
+ 23
7 (Suma las unidades)
+ 30 (Suma las decenas)
37
Algoritmo expandido

(c) 14
+ 23
37
Algortimo convencional
14 + 23 = 37
Modelo concreto
Figura 3-12

Una justificación más formal para esta suma, que usualmente no se presenta en el nivel
básico, es la siguiente:
14 + 23 = 11 # 10 + 42 + 12 # 10 + 32 Valor posicional
= 11 # 10 + 2 # 102 + 14 + 32 Propiedades conmutativa y asociativa de la
suma
= 11 + 2210 + 14 + 32 Propiedad distributiva de la multiplicación
sobre la suma
= 3 # 10 + 7 Suma básica de dígitos
= 37 Valor posicional
En la Página de un libro de texto (página 129) vemos un ejemplo de sumar números de dos
dígitos reagrupándolos mediante cubos de base diez. El método de Cati lleva al algoritmo
expandido y el método de Quique lleva al algoritmo convencional. Cada uno de estos algorit-
mos se analiza en detalle en la página 130. Nota que en la Página de un libro de texto se pide
a los estudiantes estimar sus respuestas antes de efectuar el algoritmo. Esto constituye una
buena práctica y permite desarrollar un sentido numérico, además de ayudar a los estudian-
tes a ver si sus respuestas son razonables. Estudia la página de un libro de texto y responde
las preguntas del Tema de plática.

© Lopez Mateos Editores. ISBN 978-607-95583-2-1, obra completa, versión electrónica, ISBN 978-607-95583-3-8, volumen 1, versión electrónica. Ejemplar asignado a: Helecto Villarroel gutierrez -
helecto@gmail.com. Fecha: 27 de octubre de 2014. Prohibida su modificación, copia o distribución.
Sección 3-2 Algoritmos para la suma y la resta de números completos 129

Página de un libro de texto S U M A D E N Ú M E ROS


D E D OS D Í G I TOS

Lección 3-1

Suma de números
Calentamiento
Idea clave
Para sumar, puedes Usa matemática mental.
separar números
usando el valor de dos dígitos
posicional.
Aprende
Vocabulario
reagrupar ¿Cómo sumas números de dos dígitos?
Ejemplo
ex
ef l i ó Carlos contó 46 catarinas en un tronco y 78 más en
R

unos arbustos. ¿Cuántas catarinas contó en total?

¡Piensa! Halla 46 + 78
Debo estimar Estima: redondeo 46 a 50, redondeo 78 a 80.
para saber si
mi respuesta 50 + 80=130; así, la respuesta debe estar alrededor de 130.
es razonable.
Puedo usar bloques Lo que piensas Lo que escribes
de valor posicional
para ilustrar la suma. Según Cati
Suma las unidades
unidades
Suma las decenas
4 decenas + 7 decenas=
11 decenas = 110
Obtén la suma 11 decenas 14 unidades

Según Quique
Suma las unidades
unidades
Reagrupa 14 unidades co-
mo 1 decena y 4 unidades.
Suma las decenas
1 decena + 4 decenas +
14 unidades = 1 decena 4 unidades
7 decenas = 12 decenas
Obtén la suma
Carlos contó 124 catarinas en total.

Tema de plática
¿Por qué Quique escribió un 1 pequeño
sobre el 4 en el lugar de las decenas?
¿Por qué debes estimar cuando
sumas números de dos dígitos?

Fuente: Scott Foresman-Addison Wesley Mathematics, Grade 3, 2008 (p. 126)

© Lopez Mateos Editores. ISBN 978-607-95583-2-1, obra completa, versión electrónica, ISBN 978-607-95583-3-8, volumen 1, versión electrónica. Ejemplar asignado a: Helecto Villarroel gutierrez -
helecto@gmail.com. Fecha: 27 de octubre de 2014. Prohibida su modificación, copia o distribución.
130 Números completos y sus operaciones

Después de dominar modelos concretos con reagrupamiento, niñas y niños deberán estar
en condiciones de usar los algoritmos expandido y convencional. En la figura 3-13 se mues-
tra el cálculo de 37 + 28 usando ambos algoritmos. Nota que en la figura 3-13(b), cuando
hay más de 10 unos, reagrupamos 10 unos como un diez y después sumamos las decenas.
Nota que ahora son de uso común en el salón de clase las palabras reagrupar o intercambiar
para describir lo que llamábamos llevar.
1
(a) 37 (b) 37
+ 28 + 28
15 (Sumamos las unidades) 65 (Sumamos las unidades, reagrupamos y
+ 50 (Sumamos las decenas) sumamos las decenas)
65
Algoritmo expandido Algoritmo convencional
Figura 3-13
A continuación sumamos dos números de tres dígitos mediante dos reagrupamientos. En la
figura 3-14 se muestra cómo sumar 186 + 127 usando cubos de base diez y cómo este mo-
delo concreto conlleva el algoritmo convencional.
Algoritmo
Modelo concreto convencional
1. Suma las unidades y reagrupa. 1
186
6 unidades + 7 unidades = 13 unidades
+ 127
13 unidades = 1 decena + 3 unidades
3

2. Suma las decenas y reagrupa. 11

1 decena + 8 decenas + 186


2 decenas = 11 decenas + 127
11 decenas = 1 ciento + 1 decena 13

3. Suma las centenas. 11

1 ciento + 1 ciento + 1 ciento = 3 cientos 186


186 + 127 = 313 + 127
313

Figura 3-14
© Lopez Mateos Editores. ISBN 978-607-95583-2-1, obra completa, versión electrónica, ISBN 978-607-95583-3-8, volumen 1, versión electrónica. Ejemplar asignado a: Helecto Villarroel gutierrez -
helecto@gmail.com. Fecha: 27 de octubre de 2014. Prohibida su modificación, copia o distribución.
Sección 3-2 Algoritmos para la suma y la resta de números completos 131

Con frecuencia los estudiantes desarrollan sus propios algoritmos. Al investigar cómo tra-
bajan y si en verdad funcionan varios algoritmos, se avanza en el aprendizaje. La suma por
medio de cubos conlleva, de manera natural, la forma expandida y el uso de intercambios
usados anteriormente. Por ejemplo, considera la suma siguiente:
3 # 102 + 7 # 10 + 6
376
ó 4 # 102 + 5 # 10 + 9
459
8 # 10 + 7 # 102
3
+ 1 # 10 + 6
+ 8716
8 # 103 + 14 # 102 + 13 # 10 + 21
Para completar la suma se usan intercambios. Ahora considera un problema de álgebra
análogo, de suma de polinomios:
13x2 + 7x + 62 + 14x2 + 5x + 92 + 18x3 + 7x2 + x + 62 ó 3x2 + 7x + 6
4x2 + 5x + 9
+ 8x + 7x2
3
+ x + 6
8x3 + 14x2 + 13x + 21
Nota que si x = 10, la suma es la misma que la anterior. Nota también que conocer el valor
posicional en problemas de suma ayuda a desarrollar el pensamiento algebraico. A conti-
nuación exploramos varios algoritmos que se han usado a lo largo de la historia.
Algoritmo de izquierda a derecha para la suma
Como niñas y niños aprenden a leer de izquierda a derecha, parece natural que traten de
sumar de izquierda a derecha. Al trabajar con cubos de base diez muchas niñas y niños com-
binan, en efecto, primero las piezas mayores y después las menores. Este método tiene la
ventaja de que hace énfasis en el valor posicional. Un algoritmo de izquierda a derecha es
como sigue:

568 568
+ 757 + 757
1500 + 7002 : 1200 ¡ 1215 : 1325
160 + 502 : 110 32

18 + 72 : 15
1325
Explica por qué funciona esta técnica y aplícala para sumar 9076 + 4689.
Algoritmo de retícula para la suma
Presentamos este algoritmo efectuando una suma de dos números de cuatro dígitos. Por
ejemplo,
3 5 6 7
+5 6 7 8
0 1 1 1
8 1 3 5
9 2 4 5

Para usar este algoritmo, suma los dígitos del valor posicional del número de arriba a los
dígitos del número de abajo, de derecha a izquierda, y registra el resultado en una retícula.
Después suma las diagonales. Nota que esto es muy parecido al algoritmo expandido que
introdujimos antes. Practica esta técnica con 4578 + 2691.

© Lopez Mateos Editores. ISBN 978-607-95583-2-1, obra completa, versión electrónica, ISBN 978-607-95583-3-8, volumen 1, versión electrónica. Ejemplar asignado a: Helecto Villarroel gutierrez -
helecto@gmail.com. Fecha: 27 de octubre de 2014. Prohibida su modificación, copia o distribución.
132 Números completos y sus operaciones

Algoritmo de la marca para la suma


Se considera que el algoritmo de la marca para la suma es un algoritmo relajado pues permite
a los estudiantes efectuar sumas complicadas mediante varias sumas de dos dígitos. Presen-
tamos un ejemplo:
1. 87 Suma los números en las unidades comenzando desde arriba. Cuando la
652 suma es 10 ó más, registra esta suma trazando una marca que cruce el úl-
+ 49 timo dígito sumado y escribiendo el número de unidades junto al dígito
marcado. Por ejemplo, como 7 + 5 = 12, la “marca” representa 10 y el 2
representa las unidades.
2. 87 Continúa sumando las unidades, incluidos los nuevos dígitos. Cuando, de
652 nuevo, el resultado de la suma sea 10 ó más, como en 2 + 9 = 11, repite
+ 491 el proceso descrito en (1).
2 Cuando se complete la primera columna de suma, escribe el número de
3. 87
652 las unidades, 1, abajo de la línea de suma en el valor posicional adecuado.
+ 491 Cuenta las marcas, 2, y suma este número en la segunda columna.
1
2
4. 807 Repite el procedimiento para cada columna sucesiva hasta la última
6 52 columna que tenga valores diferentes de cero. Ahora suma las marcas y
4091 coloca el número a la izquierda de la posición actual.
201
Prueba esta técnica con 56 + 23 + 34 + 67.

Algoritmos para resta


Como sucede con la suma, los cubos de base diez proporcionan un modelo concreto para la
resta. Observa cómo usamos cubos de base diez para efectuar la resta 243 - 61: primero re-
presentamos 243 con 2 losas, 4 barras y 3 unidades, como se muestra en la figura 3-15.

Figura 3-15

Para restar o substraer 61 de 243, tratamos de quitar 6 barras y 1 unidad de los cubos de
la figura 3-15. Podemos eliminar 1 unidad, como en la figura 3-16.

Figura 3-16

Para eliminar 6 barras de la figura 3-16, necesitamos intercambiar 1 losa por 10 barras,
como se muestra en la figura 3-17.

© Lopez Mateos Editores. ISBN 978-607-95583-2-1, obra completa, versión electrónica, ISBN 978-607-95583-3-8, volumen 1, versión electrónica. Ejemplar asignado a: Helecto Villarroel gutierrez -
helecto@gmail.com. Fecha: 27 de octubre de 2014. Prohibida su modificación, copia o distribución.
Sección 3-2 Algoritmos para la suma y la resta de números completos 133

Figura 3-17

Ahora podemos eliminar, quitar o “retirar” 6 barras, quedando 1 losa, 8 barras y 2 unidades,
ó 182, como se muestra en la figura 3-18.

Figura 3-18

El trabajo de los estudiantes por medio de discusiones y registro de resultados con cubos
de base diez los conduce al desarrollo del algoritmo convencional, como se ve en la página
134. Realiza el trabajo señalado en (a)–(f ) de la Página de un libro de texto.

AHORA INTENTA ÉSTE 3-8 Usa cubos de base diez y la suma para verificar que 243 - 61 = 182.

La resta de números completos usando cubos conlleva, de manera natural, la forma expan-
dida y el intercambio. Por ejemplo, considera el siguiente problema de resta, que ya hicimos
con cubos:
243 2 # 102 + 4 # 10 + 3 1 # 102 + 14 # 10 + 3
- 61 ó - 16 # 10 + 12 ó - 16 # 10 + 12
1 10 + 114 - 6210 + 13 - 12
# 2

cuyo resultado es 182. Nota que para completar la substracción usamos intercambios. Así
como con la suma, vemos que al calcular restas es útil comprender el valor posicional.

Algoritmo de los sumandos iguales


El algoritmo de los sumandos iguales para la resta está basado en el hecho de que la dife-
rencia entre dos números no cambia si se suma la misma cantidad a ambos números. Por
ejemplo, 93 - 27 = 193 + 32 - 127 + 32. Así, la diferencia se puede calcular como
96 - 30 = 66. Usando este enfoque, podríamos efectuar la resta en la Página de un libro de
texto como sigue:
255 255 + 7 262 262 + 30 292
- 163 : - 1163 + 72 : - 170 : - 1170 + 302 : - 200
92

© Lopez Mateos Editores. ISBN 978-607-95583-2-1, obra completa, versión electrónica, ISBN 978-607-95583-3-8, volumen 1, versión electrónica. Ejemplar asignado a: Helecto Villarroel gutierrez -
helecto@gmail.com. Fecha: 27 de octubre de 2014. Prohibida su modificación, copia o distribución.
134 Números completos y sus operaciones

Página de un libro de texto M O D E LOS PA R A R ESTA R


N Ú M E ROS D E T R ES D Í G I TOS

Lección 3-8

Modelos para la resta de


Calentamiento
Idea clave
Puedes usar cubos
para mostrar los
reagrupamientos números de tres dígitos
al restar.

Aprende
Materiales
Cubosde valor
posicional
Actividad
o ¿Cómo puedes restar con cubos de valor posicional?
Halla 255 - 163 Lo que
Lo que muestras escribes
Muestra 255 con cubos
de valor posicional.

ex
Resta las unidades.
ef l i ó Regrupa si es necesario
R

5 unidades - 3 unidades = 2 unidades


5 > 3. No es necesario reagrupar.
¡Piensa!
Puedo usar objetos
para ilustrar un problema
Resta las decenas.
de substracción con Reagrupa si es necesario.
reagrupamiento.
5 decenas < 6 decenas

Así, reagrupa 1 centena


como 10 decenas.
15 decenas - 6 decenas = 9 decenas

Resta las
centenas.
1 centena - 1 centena = 0 centenas

Halla el valor de los cubos restantes en el paso d:


9 decenas 2 unidades = 92, de modo que 255 - 163 = 92.
En el paso b, ¿tienes que reagrupar para restar las unidades?
Explica.
En el paso c, ¿tienes que reagrupar para restar las decenas?
Explica.
Usa cubos de valor posicional para restar:

Fuente: Scott Foresman-Addison Wesley Mathematics, Grade 3, 2008 (p. 150).

© Lopez Mateos Editores. ISBN 978-607-95583-2-1, obra completa, versión electrónica, ISBN 978-607-95583-3-8, volumen 1, versión electrónica. Ejemplar asignado a: Helecto Villarroel gutierrez -
helecto@gmail.com. Fecha: 27 de octubre de 2014. Prohibida su modificación, copia o distribución.
Sección 3-2 Algoritmos para la suma y la resta de números completos 135

AHORA INTENTA ÉSTE 3-9 Juanita asegura que un método similar al de los sumandos iguales para la
resta también funciona para la suma. Dice que en un problema de suma, “puedes sumar la misma cantidad
a un número que la que restas del otro”. Por ejemplo, 68 + 29 = 168 - 12 + 129 + 12. Así, la suma se
puede calcular como 67 + 30 = 97 o como 168 + 22 + 129 - 22 = 70 + 27 = 97. (i) Explica por
qué es válido este método y (ii) úsalo para calcular 97 + 69.

Comprensión de sumas y restas en


bases diferentes a diez
Conocer los cálculos en otras bases ayuda a comprender los cálculos en base diez. Usar cu-
bos multibase puede ayudar a construir la tabla de sumar para diferentes bases, y es alta-
mente recomendable. La tabla 3-1 es una tabla de sumar para la base cinco.

Tabla 3-1  Tabla de sumar para la base cinco

ⴙ 0 1 2 3 4
0 0 1 2 3 4
1 1 2 3 4 10
2 2 3 4 10 11
3 3 4 10 11 12
4 4 10 11 12 13

AHORA INTENTA ÉSTE 3-10 Escribe lo siguiente como numerales en base cinco:
a. 444cinco + 1cinco b. 13cinco + 13cinco

Usando los resultados de la tabla 3-1, desarrollamos algoritmos para la suma en base cinco
similares a los de la suma en base diez. En la figura 3-19(a) mostramos el cálculo usando un
modelo concreto; en la figura 3-19(b) usamos el algoritmo expandido; en la figura 3-19(c) usa-
mos el algoritmo convencional.
(a) 12cinco 31cinco (b) 12cinco (c) 12cinco
+ 31cinco + 31cinco
3 43cinco
+ 40 Algoritmo
43cinco convencional
Algoritmo expandido

12cinco + 31cinco 5 43cinco


Modelo concreto

Figura 3-19
© Lopez Mateos Editores. ISBN 978-607-95583-2-1, obra completa, versión electrónica, ISBN 978-607-95583-3-8, volumen 1, versión electrónica. Ejemplar asignado a: Helecto Villarroel gutierrez -
helecto@gmail.com. Fecha: 27 de octubre de 2014. Prohibida su modificación, copia o distribución.
136 Números completos y sus operaciones

La tabla de restar para la base cinco se puede obtener de la tabla de sumar, por medio de
la definición de resta. Por ejemplo, para hallar 12cinco - 4cinco, recordamos que 12cinco -
4cinco = ccinco si, y sólo si, ccinco + 4cinco = 12cinco. De la tabla 3-1, vemos que c = 3cinco. En
la figura 3-20 presentamos un ejemplo de resta, 32cinco - 14cinco, reagrupando.

(a) Quitar 14cinco

32cinco – 14cinco = 13cinco


32cinco

21
(b) (c) 32cinco
Cincos Unidades Cincos Unidades
– 14cinco
3 2 2 12 13cinco
– 1 4 – 1 4
1 3

Figura 3-20

AHORA INTENTA ÉSTE 3-11


a. Construye una tabla de sumar para la base dos.
b. Usa la tabla de sumar de la parte (a) para obtener (i) 1101dos - 111dos, (ii) 1111dos + 111dos

ROMPECABEZAS El número de una placa de automóvil consta de cinco dígitos. Al colocar la placa
de cabeza la podemos leer, pero el valor de la placa, de cabeza, es 78,633 mayor que el número real de la
placa. ¿Cuál es el número de la placa?

Evaluación 3-2A

1. Halla los dígitos faltantes: 4. La dieta de René le permite sólo 1500 calorías diarias.
a. - - 1 b.
- 0 2 5 En el desayuno René tomó leche descremada (90 calo-
+4 2 - 1 1 - 6 rías), un waffle sin miel (120 calorías) y un plátano (119
calorías). Para el almuerzo tomó 12 taza de ensalada (185
- 4 0 2 +3 1 4 8
calorías) con mayonesa (110 calorías) y té (0 calorías).
6 - 6 -
Después comió un pay de nuez (570 calorías). ¿Puede
2. Traza una figura semejante a la figura 3-14 para ilustrar
cenar pescado (250 calorías), una 12 taza de ensalada sin
el uso de cubos de base diez al calcular 29 + 37.
mayonesa y té?
3. Coloca los dígitos 7, 6, 8, 3, 5 y 2 en las cajas para obtener:
5. Guille registró sus gastos de la semana pasada. Su salario
a. la mayor suma. b. la menor suma.
fue de $1500 más $540 de tiempo extra y $2600 de pro-
nnn pinas. Sus gastos de transporte fueron de $220, los de
+ nnn comida fueron de $600, sus costos de lavandería fueron
de $150, en diversión gastó $580 y su renta fue de

© Lopez Mateos Editores. ISBN 978-607-95583-2-1, obra completa, versión electrónica, ISBN 978-607-95583-3-8, volumen 1, versión electrónica. Ejemplar asignado a: Helecto Villarroel gutierrez -
helecto@gmail.com. Fecha: 27 de octubre de 2014. Prohibida su modificación, copia o distribución.
Sección 3-2 Algoritmos para la suma y la resta de números completos 137

$1850. ¿Le quedó dinero? ¿Cuánto? piedades conmutativa y asociativa de la suma, y la propie-
6. En el siguiente problema la suma es correcta, pero los dad distributiva de la multiplicación sobre la suma:
dígitos de cada sumando están en desorden. Corrige los a. 68 + 23
sumandos para obtener la suma correcta. b. 174 + 285
c. 2458 + 793
2834 nnnn
12. Usa el algoritmo de la retícula para efectuar:
+ 6315 + nnnn a. 4358 + 3864
9059 9059 b. 4923 + 9897
7. Usa el enfoque de los sumandos iguales para calcular: 13. Efectúa cada una de las operaciones siguientes usando
a. 93 las bases mostradas:
a. 43cinco + 23cinco
- 37
b. 43cinco - 23cinco
b. 321 c. 432cinco + 23cinco
d. 42cinco - 23cinco
- 38
e. 110dos + 11dos
8. Juana resolvió sus problemas para sumar colocando la f.10001dos - 111dos
suma parcial como se muestra aquí: 14. Construye una tabla de sumar para la base ocho.
15. Efectúa cada una de las operaciones siguientes:
569 a. 3 h 36 min 58 s b. 5 h 36 min 38 s
+ 645 ⫹ 5 h 56 min 27 s ⫺ 3 h 56 min 58 s
14
10 16. La calculadora de Andrés no estaba funcionando correc-
tamente. Cuando tecleaba 8 + 6 = , aparecía el nu-
11
meral 20 en la pantalla. Cuando tecleaba 5 + 4 = ,
1214
se presentaba el 13. Cuando tecleaba 1 5 - 3 = ,
a. Usa este método para resolver: se mostraba el 9. ¿Qué piensas que estaba haciendo la
(i) 687 (ii) 359 calculadora de Andrés?
+ 549 + 673 17. Usa suma con marcas para efectuar:
a. 432
b. Explica por qué funciona el algoritmo.
976
9. Analiza los cálculos siguientes. Explica cuál es el error en
cada caso. + 1418
a. 28
b. 32cinco
+ 75
13cinco
9 13
22cinco
b. 28
43cinco
+ 75
23cinco
1 21
c. 305 + 12cinco
- 259 18. Realiza cada una de las operaciones siguientes:
a. 4 gruesas 4 docenas 6 unidades
154
- 5 docenas 9 unidades
d. 2 10
305 b. 2 gruesas 9 docenas 7 unidades
- 259 + 3 gruesas 5 docenas 9 unidades
56 19. Determina cuál es el error:
10. Da razones para cada uno de los pasos siguientes:
22cinco
16 + 31 = 11 # 10 + 62 + 13 # 10 + 12 + 33cinco
= 11 # 10 + 3 # 102 + 16 + 12 55cinco
= 11 + 3210 + 16 + 12
= 4 # 10 + 7 20. Coloca los números faltantes:
= 47 a. 2 - - cinco
- 2 2cinco
11. En cada caso, justifica el algoritmo convencional de la
suma usando el valor posicional de los números, las pro- - 0 3cinco

© Lopez Mateos Editores. ISBN 978-607-95583-2-1, obra completa, versión electrónica, ISBN 978-607-95583-3-8, volumen 1, versión electrónica. Ejemplar asignado a: Helecto Villarroel gutierrez -
helecto@gmail.com. Fecha: 27 de octubre de 2014. Prohibida su modificación, copia o distribución.
138 Números completos y sus operaciones

b. 2 0 0 1 0tres palíndromo? De no ser así, invierte el orden de los dígitos


- 2 - 2 - tres y suma este número al original. ¿El resultado es un palín-
1 - 2 - 1tres dromo? De no ser así, repite el procedimiento anterior
hasta obtener un palíndromo. Por ejemplo, comienza con
21. Halla el numeral que debe colocarse en el espacio en
78. Vemos que 78 no es un palíndromo, y sumamos:
blanco de manera que cada ecuación sea verdadera. No
78 + 87 = 165. Como 165 tampoco es un palíndromo,
conviertas a base diez.
sumamos: 165 + 561 = 726. Nuevamente, 726 no es un
a. 3423cinco - ____ = 2132cinco
palíndromo, luego entonces sumamos 726 + 627 para ob-
b. 11011dos + ____ = 100000dos
tener 1353. Finalmente, 1353 + 3531 produce 4884, que
c. DOOdoce - ____ = 1
es un palíndromo.
d. 1000cinco + ____ = 10000cinco
a. Aplica este método a los números siguientes:
22. Un palíndromo es cualquier número que se lee lo mismo
(i) 93 (ii) 588 (iii) 2003
hacia adelante que hacia atrás, por ejemplo 121 y 2332. In-
b. Halla un número para el cual el procedimiento descrito
tenta lo siguiente: comienza con cualquier número. ¿Es un
requiera más de cinco pasos para formar un palíndromo.

Evaluación 3-2B

1. Halla los dígitos faltantes: Usa este método para resolver:


a. 3- - a. 985
- 159 + 356
- 24 b. 413
b. 1 - - - 6 + 89
- 8 3 0 9 7. Analiza los cálculos siguientes. Explica cuál es el error en
4 9 8 7 cada caso.
2. Traza una figura semejante a la figura 3-12 para ilustrar el a. 135
uso de cubos de base diez al calcular 46 + 38. + 47
3. Coloca los dígitos 7, 6, 8, 3, 5 y 2 en los cuadros para ob- 172
tener: b. 87
a. la mayor diferencia. + 25
b. la menor diferencia. 1012
nnn c. 57
- nnn - 38
21
4. En el siguiente problema la suma es correcta, pero los dí-
d. 56
gitos de cada sumando están en desorden. Corrige los su-
mandos para obtener la suma correcta. - 18
48
8354 nnnn 8. Jorge está preparando una cena. Sólo puede cocinar un
+ 3456 + nnnn platillo a la vez en su horno de microondas. El pavo
11729 11729 tarda 75 min, el pastel tarda 18 min, los rollos tardan 45 s
y una taza de café tarda 30 s en calentar. ¿Cuánto tiempo
5. Usa el enfoque de los sumandos iguales para calcular: necesita para cocinar la cena?
a. 86 b. 582 9. Da razones para cada uno de los pasos siguientes:
- 38 - 44
6. Juana resolvió sus problemas para sumar colocando la 123 + 45 = 11 # 102 + 2 # 10 + 32 + 14 # 10 + 52
suma parcial como se muestra aquí: = 1 # 102 + 12 # 10 + 4 # 102 + 13 + 52
= 1 # 102 + 12 + 4210 + 13 + 52
569 = 1 # 102 + 6 # 10 + 8
+ 645 = 168
14 10. En cada caso, justifica el algoritmo convencional de la
10 suma usando el valor posicional de los números, las
propiedades conmutativa y asociativa de la suma, y la
11
1214

© Lopez Mateos Editores. ISBN 978-607-95583-2-1, obra completa, versión electrónica, ISBN 978-607-95583-3-8, volumen 1, versión electrónica. Ejemplar asignado a: Helecto Villarroel gutierrez -
helecto@gmail.com. Fecha: 27 de octubre de 2014. Prohibida su modificación, copia o distribución.
Sección 3-2 Algoritmos para la suma y la resta de números completos 139

propiedad distributiva de la multiplicación sobre la 17. Determina cuál es el error:


suma:
23seis
a. 46 + 32
b. 3214 + 783 + 43seis
11. Usa el algoritmo de la retícula para efectuar: 66seis
a. 2345 + 8888 18. Halla el numeral que debe colocarse en el espacio en
b. 8713 + 4214 blanco de manera que cada ecuación sea verdadera. No
12. Efectúa cada una de las operaciones siguientes usando conviertas a base diez.
las bases mostradas: a. 342cinco - ____ = 213cinco
a. 43cinco - 24cinco b. 1101dos - ____ = 1011dos
b. 143cinco + 23cinco c. O08doce - ____ = 9doce
c. 32cinco - 23cinco d. 100dos + ____ = 10000dos
d. 232cinco + 43cinco 19. Los Halcones jugaron un partido de baloncesto contra
e. 110dos + 111dos Los Ciervos. Con base en la información proporcionada
f. 10001dos - 101dos abajo, completa la tabla en donde se muestran los puntos
13. Construye una tabla de sumar para la base seis. anotados por cada equipo en cada cuarto, así como el
14. Efectúa las operaciones siguientes 12 c = 1 pt, marcador final.
2 pt = 1 qt, 4 qt = 1 gal2:
a. 1 qt 1 pt 1 c
Cuartos
+ 1 pt 1 c
Equipos 1 2 3 4 Marcador final
b. 1 qt 1c
- 1 pt 1c Halcones
Ciervos
c. 1 gal 3 qt 1 c
- 4 qt 2 c a. Los Halcones anotaron 15 puntos en el primer cuarto.
15. El siguiente es un cuadrado supermágico tomado de un b. Los Halcones iban abajo 5 puntos al final del primer
grabado de Durero llamado Melancolía. Nota el 1514 en cuarto.
el renglón inferior; es el año en que lo hizo. c. Los Ciervos anotaron 5 puntos más en el segundo
cuarto de los que anotaron en el primer cuarto.
d. Los Halcones anotaron 7 puntos más que Los Cier-
16 3 2 13 vos en el segundo cuarto.
e. Los Ciervos rebasaron por 6 puntos a Los Halcones
5 10 11 8 en el cuarto cuarto.
f. Los Halcones alcanzaron un marcador final de 120
9 6 7 12
puntos.
4 15 14 1 g. Los Halcones anotaron el doble de puntos en el ter-
cer cuarto de los que anotaron Los Ciervos en el pri-
mer cuarto.
a. Halla la suma de cada renglón, la suma de cada co- h. Los Ciervos anotaron la misma cantidad de puntos en
lumna y la suma de cada diagonal. el tercer cuarto que Los Halcones en la suma de los
b. Halla la suma de los cuatro números del centro. dos primeros cuartos.
c. Halla la suma de los cuatro números de las esquinas. 20. a. Coloca los números del 24 al 32 en los círculos si-
d. Suma 11 a cada número del cuadrado. ¿Sigue siendo guientes, de modo que las sumas sean las mismas en
un cuadrado mágico? Explica tu respuesta. cada dirección:
e. Resta 11 de cada número del cuadrado. ¿Sigue siendo
mágico?
16. Usa la suma con marcas para efectuar:
a. 537 b. 41seis
318 32seis
+ 2345 22seis
43seis
22seis
+54seis b. ¿Cuántos números diferentes pueden colocarse en el
centro para obtener una solución?

© Lopez Mateos Editores. ISBN 978-607-95583-2-1, obra completa, versión electrónica, ISBN 978-607-95583-3-8, volumen 1, versión electrónica. Ejemplar asignado a: Helecto Villarroel gutierrez -
helecto@gmail.com. Fecha: 27 de octubre de 2014. Prohibida su modificación, copia o distribución.
140 Números completos y sus operaciones

Conexiones matemáticas 3-2

Comunicación 6. Cati halló su propio algoritmo para la resta. Lo hace así:


1. Analiza el mérito del siguiente algoritmo para la suma en 97
donde primero sumamos las unidades, después las dece- - 28
nas, después las centenas y después el total:
- 1
479 + 70
+ 385 69
14
¿Cómo le responderías si fueras su maestra?
150 7. Analiza por qué, en los algoritmos para la suma y resta,
+ 700 se usan las palabras reagrupar y cambiar o intercambiar en
864 lugar de llevar y pedir.
8. Considera el siguiente algoritmo para la substracción.
2. En el ejemplo siguiente usamos un enfoque de reagrupa-
a. Explica cómo funciona.
miento para la resta. Analiza la conveniencia de este en-
b. Usa este algoritmo para hallar 787 - 398.
foque para la enseñanza.
585
843 800 + 40 + 3
- 1500 + 60 + 82
: : - 277
- 568
385
700 + 130 + 13 285
- 1500 + 60 + 82
800 + 30 + 13
- 1500 + 60 + 82
: + 23
200 + 70 + 5 = 275 308
3. Lara, una estudiante de cuarto grado, suma agregando y
restando el mismo número. Ella suma como sigue: Solución abierta

39 39 + 1 40 9. Busca o desarrolla un algoritmo para sumar o restar nú-


: : meros completos y escribe una descripción de tu algo-
+ 84 + 84 - 1 + 83
123 ritmo de modo que otros lo puedan comprender y usar.

¿Cómo responderías si fueras su maestra? Aprendizaje colectivo


4. Explica por qué funciona el algoritmo de las marcas.
5. En esta sección introdujimos el algoritmo de sumandos 10. En esta sección les hemos presentado varios algoritmos.
iguales. A continuación mostramos cómo funciona el al- Analiza con tu grupo si conviene impulsar a niñas y ni-
goritmo para 1464 - 687: ños para que desarrollen y usen sus propios algoritmos
para la suma y resta de números completos, o si se les
1 4 61 4 (Suma 10 a las 4 unidades para obtener 14 unidades.) debe enseñar un solo algoritmo por operación y todos
- 69 8 7 (Suma 1 decena a las 8 decenas para obtener 9 decenas.) los estudiantes deben usar sólo un algoritmo.
7 (Resta las unidades.) Preguntas del salón de clase
Pasemos ahora a la segunda columna.
11. Para hallar 68 - 19, Pepe comenzó restando 9 - 8.
1 41 61 4 (Suma 10 decenas a 6 decenas para obtener 16 decenas.) ¿Cómo le puedes ayudar?
- 769 8 7 (Suma 1 centena a las 6 centenas para obtener 7 centenas.) 12. Gilda restó 415 - 212 y obtuvo 303. Te pregunta si está
7 7 7 (Resta las 9 decenas de las 16 decenas y después las 7 bien. ¿Cómo le responderías?
centenas de las 14 centenas.) 13. Beti obtuvo que 518 - 49 = 469. Como ella no sabía si
a. Aplica esta técnica a otras tres restas. estaba bien, trató de verificar su respuesta sumando
b. Explica por qué funciona el algoritmo de sumandos 518 + 49. ¿Cómo le puedes ayudar?
iguales. 14. Se pide a una niña calcular 7 + 2 + 3 + 8 + 11 y ella

© Lopez Mateos Editores. ISBN 978-607-95583-2-1, obra completa, versión electrónica, ISBN 978-607-95583-3-8, volumen 1, versión electrónica. Ejemplar asignado a: Helecto Villarroel gutierrez -
helecto@gmail.com. Fecha: 27 de octubre de 2014. Prohibida su modificación, copia o distribución.
Sección 3-2 Algoritmos para la suma y la resta de números completos 141

escribe 7 + 2 = 9 + 3 = 12 + 8 = 20 + 11 = 31. Al Preguntas del National Assessment of Educational


notar que la respuesta es correcta, ¿cómo reaccionarías Progress (NAEP) (Evaluación Nacional del Progreso
si fueras su maestro? Educativo)
Problemas de repaso
15. ¿El conjunto 51, 2, 36 es cerrado bajo la suma? ¿Por qué?
16. Da un ejemplo de la propiedad asociativa de la suma de
los números completos.

La figura anterior representa 237. ¿Qué número es

mayor que 237?

a. 244 b. 249 c. 251 d. 377


NAEP 2007, Grado 4
El puente Ben Franklin tenía 75 años en 2001. ¿En qué
año tenía el puente 50 años?
a. 1951 b. 1976 c. 1984 d. 1986
NAEP 2007, Grado 4

ACTIVIDAD DE LABORATORIO
1. En la figura 3-21(a) se muestra un tipo de ábaco japonés, el soroban. En este ábaco una barra separa dos
conjuntos de cuentas. Cada cuenta arriba de la barra representa cinco veces las cuentas de abajo de la
barra. Los números se describen moviendo las cuentas hacia la barra. Se ilustra el número 7632. Practica
mostrando y sumando números en este ábaco.
de llar

M llar

nt s

U enas

s
M llar

nt s

U enas

D as
D as

Ce are

de
Ce are

de

en
en

i
i

da
i

da

en e m

m
ill
em

ill

ec
ec

ni
ni

ec d
sd

D nas
as
na

e
e

nt
ec

Ce
D

(a) (b)

Figura 3-21
2. El ábaco chino, suan pan (ver la Figura 3-21(b)), aún se usa hoy día. Este ábaco es similar al japonés, pero
tiene dos cuentas arriba de la barra y cinco cuentas debajo de ella. Se ilustra el número 7632. Practica
mostrando y sumando números con este ábaco. Compara la facilidad de uso de las dos versiones.

© Lopez Mateos Editores. ISBN 978-607-95583-2-1, obra completa, versión electrónica, ISBN 978-607-95583-3-8, volumen 1, versión electrónica. Ejemplar asignado a: Helecto Villarroel gutierrez -
helecto@gmail.com. Fecha: 27 de octubre de 2014. Prohibida su modificación, copia o distribución.
142 Números completos y sus operaciones

3-3 Multiplicación y división de números completos

En los Puntos focales del grado 3 hallamos lo siguiente respecto a la multiplicación y división
de números completos:

Las personas (estudiantes) comprenden los significados de multiplicación y división de números


completos por medio de representaciones (p.ej., grupos de igual tamaño, arreglos, modelos de
área y “saltos” iguales sobre la recta numérica para la multiplicación, así como substracciones suce-
sivas, particiones y repartos para la división). Usan las propiedades de la suma y la multiplicación
(p.ej., conmutatividad, asociatividad y distributividad) para multiplicar números completos y
aplicar estrategias cada vez más sofisticadas basadas en estas propiedades, para resolver proble-
mas de multiplicación y división que impliquen el manejo de las tablas. Al comparar varias estrate-
gias de solución, las personas relacionan a la multiplicación y la división como operaciones
inversas. (p. 15)

Más aún, en los Puntos focales del grado 3 vemos la relación entre el estudio de la multiplica-
ción y división de números completos y el estudio del álgebra.

Comprender las propiedades de la multiplicación y la relación entre multiplicación y división es


parte de la preparación en álgebra que se desarrolla en el grado 3. En este grado es cuando de-
biera ocurrir la creación y análisis de patrones y relaciones que incluyan la multiplicación y di-
visión. Los estudiantes construyen la base para una posterior comprensión de relaciones
funcionales al describir relaciones en un contexto, con proposiciones como “El número de patas
es 4 veces el número de sillas”. (p. 15)

Estas citas de los Puntos focales marcan el tono y propósito de esta sección. Analizamos repre-
sentaciones que pueden ayudar a los estudiantes a comprender los significados de multiplica-
ción y división. Desarrollamos la propiedad distributiva de la multiplicación sobre la suma
junto con la relación de la multiplicación y la división como operaciones inversas.

Multiplicación de números completos


En esta sección exploramos el tipo de problemas que tiene el Abuelo en la tira cómica Pea-
nuts. ¿Por qué piensas que tiene más problemas con “9 por 8” que con “3 por 4”? Si las ta-
blas, o multiplicaciones básicas, sólo se memorizan, se pueden olvidar. Pero si los estudiantes
tienen una comprensión conceptual de las multiplicaciones básicas cuando las necesiten, en-
tonces pueden obtener todas las tablas aunque no las recuerden automáticamente.

EL ABUELO DICE QUE LAS NUEVE POR OCHO SE HA OCHO POR SEIS SE DICE QUE AHORA
TABLAS DE MULTIPLICA- IDO POR COMPLETO...
ESTÁ DESVANE- VIVE ENTRE LOS
CIÓN ESTÁN DESAPARE- CIENDO... NÚMEROS BAJOS
CIENDO DE SU CABEZA...

© Lopez Mateos Editores. ISBN 978-607-95583-2-1, obra completa, versión electrónica, ISBN 978-607-95583-3-8, volumen 1, versión electrónica. Ejemplar asignado a: Helecto Villarroel gutierrez -
helecto@gmail.com. Fecha: 27 de octubre de 2014. Prohibida su modificación, copia o distribución.
Sección 3-3 Multiplicación y división de números completos 143

Modelo de la suma repetida


En la página 144 de muestra de un libro de texto, vemos que si tenemos 4 grupos de tres
brochas podemos usar la suma para colocar juntos a los grupos. Cuando juntamos grupos
del mismo tamaño podemos usar la multiplicación. Podemos pensar en combinar 4 conjun-
tos de 3 objetos en un solo conjunto. Los 4 conjuntos de 3 sugieren la suma siguiente:
3 + 3 + 3 + 3 = 12
cuatro 3
Escribimos 3 + 3 + 3 + 3 como 4 # 3 y decimos “cuatro veces tres” o “tres multiplicado
por cuatro”, o bien “tres por cuatro”. Cuando el número de sumandos es grande, es evi-
dente la ventaja de la notación de multiplicación sobre la suma repetida; por ejemplo, si te-
nemos 25 grupos de 3 brochas, podemos hallar el número total de brochas sumando 25 tres
ó 25 # 3.
El modelo de la suma repetida se puede ilustrar de diversas maneras, entre ellas el uso de
una recta numérica y de arreglos. Por ejemplo, si usamos barras coloreadas de longitud 4,
podemos ver que la longitud de cinco barras de 4 se puede hallar colocando una después de
otra, como en la figura 3-22(a). La figura 3-22(b) muestra el proceso por medio de flechas
en una recta numérica.
5.4 5.4
10 10 4 4 4 4 4
4 4 4 4 4
0 4 8 12 16 20
(a)
(b)

Figura 3-22
La característica de operación constante, en una calculadora, puede relacionar la multipli-
cación con la suma. Los estudiantes pueden obtener productos en la calculadora sin usar la
tecla * . Por ejemplo, si una calculadora tiene la característica de constante, entonces 5 * 3 se
puede hallar tecleando + 3 = = = = = . Cada presión del signo igual añadirá 3 al valor
en la pantalla. (Algunas calculadoras pueden funcionar de manera diferente.)
Como se señala en la Nota de investigación, si sólo se tiene acceso al modelo de la “suma
repetida” para la multiplicación, se puede provocar una confusión. En esta sección presen-
tamos otros tres modelos para estudiar la multiplicación: los modelos del arreglo y del área,
y el modelo del producto cartesiano.

◆ Nota de A los estudiantes que están aprendiendo la multiplicación como una operación concep-
investigación tual hay que presentarles varios modelos (por ejemplo, el arreglo y el área). Conocer “la
multiplicación sólo como suma repetida” y el término mismo veces, llevan a confundir
aspectos básicos de la multiplicación que complican las extensiones futuras a decimales y
fracciones (Bell et al. 1989; English y Halford 1995). ◆

◆ Nota William Oughtred (1574–1660), matemático inglés, hizo énfasis en el uso de símbolos
histórica matemáticos. Fue el primero en introducir el uso de la “cruz de San Andrés” 1*2 como
símbolo de la multiplicación. Este símbolo no se adoptó de inmediato pues, según lo objetó
Gottfried Wilhelm von Leibniz (1646–1716), se podía confundir fácilmente con la letra
x. Leibniz adoptó el uso del punto 1 # 2 para indicar la multiplicación, que se volvió de uso
común. ◆

© Lopez Mateos Editores. ISBN 978-607-95583-2-1, obra completa, versión electrónica, ISBN 978-607-95583-3-8, volumen 1, versión electrónica. Ejemplar asignado a: Helecto Villarroel gutierrez -
helecto@gmail.com. Fecha: 27 de octubre de 2014. Prohibida su modificación, copia o distribución.
144 Números completos y sus operaciones

Página de libro de texto L A M U LT I P L I CAC I Ó N


COM O S U M A R E P E T I DA

Lección
Calentamiento
´ La multiplicación como
Idea clave
Multiplicar es una
suma repetida
manera rápida de
sumar grupos Aprende
iguales.
Actividad
Vocabulario
multiplicación ¿Cómo puedes
factor
producto
hallar el total?
Hay 4
Material grupos de
fichas
3 brochas.
Puedes usar la suma para juntar los grupos.
Expresión de suma

ex Cuando juntas grupos iguales, también puedes


ef l i ó
usar la multiplicación.
R

Dices: 4 veces 3 es igual a 12


¡Piensa!
Puedo usar objetos para Escribes: Expresión de multiplicación
mostrar grupos iguales.
factor factor producto
Escribe una expresión de suma y una expresión de
multiplicación para mostrar el número total de fichas.

Usa fichas y traza una figura para ilustrar los grupos


descritos a continuación. Para cada figura, escribe
una expresión de suma y una expresión de
multiplicación que ilustre cuántas fichas hay en total.
5 grupos de 2
4 grupos de 5
3 grupos de 3

Consulta en la RED
More Examples

Fuente: Scott Foresman-Addison Wesley Math, Grade 3, 2008 (p. 260).

© Lopez Mateos Editores. ISBN 978-607-95583-2-1, obra completa, versión electrónica, ISBN 978-607-95583-3-8, volumen 1, versión electrónica. Ejemplar asignado a: Helecto Villarroel gutierrez -
helecto@gmail.com. Fecha: 27 de octubre de 2014. Prohibida su modificación, copia o distribución.
Sección 3-3 Multiplicación y división de números completos 145

Modelos de arreglos y de áreas


Otra representación que resulta útil al explorar la multiplicación de números completos es
el arreglo. Pensamos en un arreglo cuando tenemos objetos colocados en filas del mismo
tamaño, como en la figura 3-23.

(a) (b)

Figura 3-23
En la figura 3-24(a) cruzamos líneas para crear puntos de intersección, formando así un
arreglo de puntos. El número de puntos en una sola línea vertical es de 4, y hay 5 líneas,
formando un total de 5 # 4 puntos en el arreglo. En la figura 3-24(b), se muestra el modelo
del área como una malla de 4 por 5. El número de cuadrados unitarios requeridos para
llenar la malla es 20. Estos modelos motivan la siguiente definición de multiplicación de
números completos.

5
(a) (b)

Figura 3-24

Definición de multiplicación de números completos


Para cualesquier números completos a y n Z 0,
n # a = a + a + a + Á + a.
n términos
Si n = 0, entonces 0 # a = 0.

#
O B S E R VA C I Ó N Usualmente escribimos n a como na, donde a no es un número, sino
una variable.
Modelo del producto cartesiano
El modelo del producto cartesiano ofrece otra manera de estudiar la multiplicación. Supón
que puedes ordenar una hamburguesa de soya en pan blanco o negro, con un condimento:
mostaza, mayonesa o salsa. Para mostrar las diferentes órdenes que puede tomar un mesero,

© Lopez Mateos Editores. ISBN 978-607-95583-2-1, obra completa, versión electrónica, ISBN 978-607-95583-3-8, volumen 1, versión electrónica. Ejemplar asignado a: Helecto Villarroel gutierrez -
helecto@gmail.com. Fecha: 27 de octubre de 2014. Prohibida su modificación, copia o distribución.
146 Números completos y sus operaciones

usamos un diagrama de árbol. En la figura 3-25 listamos las maneras de ordenar, donde el
pan se escoge del conjunto P = 5blanco, negro6 y el condimento se escoge del conjunto
C = 5mostaza, mayonesa, salsa6.

Tipo
de Lo que ve
pan Condimento el cocinero

Mostaza Negro mostaza

Negro Mayonesa Negro mayo

Salsa Negro salsa


Orden
Mostaza Blanco mostaza

Blanco Mayonesa Blanco mayo

Salsa Blanco salsa

Figura 3-25

Cada orden se puede escribir como un par ordenado, por ejemplo (negro, mostaza). El con-
junto de pares ordenados forma el producto cartesiano P * C. El Principio Fundamental
del Conteo nos dice que el número de pares ordenados en P * C es 2 # 3.
En el análisis anterior ilustramos cómo se puede definir la multiplicación de números
completos por medio del producto cartesiano. Así, damos a continuación una definición al-
ternativa de multiplicación de números completos:

Definición alternativa de multiplicación de números completos


Para conjuntos finitos A y B, si n1A2 = a y n1B2 = b, entonces a # b = n1A * B2.

En esta definición alternativa no se requiere que los conjuntos A y B sean ajenos. La expre-
sión a # b, o simplemente ab, es el producto de a y b, y a y b son los factores. Nota que A * B
indica el producto cartesiano, no la multiplicación. Multiplicamos números, no conjuntos.

AHORA INTENTA ÉSTE 3-12 ¿Cómo usarías la definición de multiplicación como suma repetida
para explicar a un niño que no conoce el Principio Fundamental del Conteo que el número
posible de vestimentas consistente en una combinación de camisa y pantalón —dadas 6 camisas
y 5 pantalones— es 6 # 5?
Los siguientes problemas ilustran cada uno de los modelos mostrados para la multipli-
cación. En los cinco problemas la respuesta puede pensarse usando un modelo diferente.
Trabaja cada problema usando el modelo sugerido.
1. Modelo de la suma repetida. Un caramelo cuesta $5; ¿cuánto cuestan tres caramelos?
2. Modelo de la recta numérica. Si Alicia camina a 5 km por hora durante 3 h, ¿cuánto ha
caminado?
3. Modelo del arreglo. Una plana de estampillas tiene 4 filas de 5 estampillas. ¿Cuántas es-
tampillas hay en una plana?
4. Modelo del área. Si una alfombra mide 5 m por 3 m, ¿cuál es el área de la alfombra?
5. Modelo del producto cartesiano. Alberto tiene 5 camisas y 3 pantalones; ¿cuántas
combinaciones de ropa camisa-pantalón son posibles?

© Lopez Mateos Editores. ISBN 978-607-95583-2-1, obra completa, versión electrónica, ISBN 978-607-95583-3-8, volumen 1, versión electrónica. Ejemplar asignado a: Helecto Villarroel gutierrez -
helecto@gmail.com. Fecha: 27 de octubre de 2014. Prohibida su modificación, copia o distribución.
Sección 3-3 Multiplicación y división de números completos 147

Propiedades de la multiplicación de números completos


El conjunto de los números completos es cerrado bajo la multiplicación. Esto es, si multipli-
camos cualesquier dos números completos, el resultado es un número completo. Esta pro-
piedad se llama propiedad de la cerradura de la multiplicación de números completos. La
multiplicación en el conjunto de los números completos tiene, como en la suma, propieda-
des conmutativa, asociativa y de existencia de neutro, o identidad.

Teorema 3–5:    Propiedades de la multiplicación de números completos


Propiedad de cerradura de la multiplicación de números completos Para cualesquier
números completos a y b, a # b es un número completo único.
Propiedad conmutativa de la multiplicación de números completos Para cualesquier
números completos a y b, a # b = b # a.
Propiedad asociativa de la multiplicación de números completos Para cualesquier números
completos a, b y c, 1a # b2 # c = a # 1b # c2.
Propiedad de la identidad o neutro multiplicativo de los números completos Existe un
número completo único 1 tal que para cualquier número completo a, a # 1 = a = 1 # a.
Propiedad de la multiplicación por cero de los números completos Para cualquier número
completo a, a # 0 = 0 = 0 # a.

La propiedad conmutativa de la multiplicación de números completos se ilustra fácilmente cons-


3 truyendo una malla de 3 por 5 y haciéndola girar para colocarla sobre el otro lado, como se
muestra en la figura 3-26. Vemos que el número de cuadrados de 1 * 1 presentes en cada
caso es 15, esto es, 3 # 5 = 15 = 5 # 3. La propiedad conmutativa se puede verificar recor-
5
3 5 = 15
dando que n1A * B2 = n1B * A2.
La propiedad asociativa de la multiplicación de números completos se puede ilustrar como si-
gue. Supón que a = 3, b = 5 y c = 4. En la figura 3-27(a) vemos una representación gráfica
de 315 # 42 cubos. En la figura 3-27(b) vemos los mismos cubos, esta vez arreglados como
5 413 # 52. Por la propiedad conmutativa, esto se puede escribir como 13 # 524. Como ambos
conjuntos de cubos de la figura 3-27(a) y (b) se pueden juntar y obtener el conjunto mos-
trado en la figura 3-27(c), vemos que 315 # 42 = 13 # 524. La propiedad asociativa es útil
3 para efectuar cálculos como el siguiente:
3 # 40 = 314 # 102 = 13 # 4210 = 12 # 10 = 120
5 3 = 15

Figura 3-26
3

2
4
3 3 3
1 2
1
4 4
5 5 5
(a) 3(5 4) (b) (3 5)4 (c) 3(5 4) = (3 5)4
Figura 3-27
La identidad multiplicativa para los números completos es 1. Por ejemplo, 3 # 1 =
1 + 1 + 1 = 3. En general, para cualquier número completo a,
a#1 = 1 + 1 + 1 + Á + 1 = a
a términos
Así, a # 1 = a, lo cual, debido a la propiedad conmutativa de la multiplicación, implica que
a # 1 = a = 1 # a. También se puede mostrar que a # 1 = a = 1 # a usando productos cartesianos.
© Lopez Mateos Editores. ISBN 978-607-95583-2-1, obra completa, versión electrónica, ISBN 978-607-95583-3-8, volumen 1, versión electrónica. Ejemplar asignado a: Helecto Villarroel gutierrez -
helecto@gmail.com. Fecha: 27 de octubre de 2014. Prohibida su modificación, copia o distribución.
148 Números completos y sus operaciones

Ahora consideremos la multiplicación por 0. Por ejemplo, por definición 0 # 6 significa que
tenemos cero 6, ó 0. También 6 # 0 = 0 + 0 + 0 + 0 + 0 + 0 = 0. Así vemos que al mul-
tiplicar 0 por 6 ó 6 por 0 se obtiene el producto 0. Éste es un ejemplo de la propiedad de la
multiplicación por cero. Esta propiedad también se puede verificar usando la definición de
multiplicación en términos de productos cartesianos. En álgebra, 3x significa 3 veces x ó
x + x + x. Por lo tanto, 0 # x significa 0 veces x, ó 0.
Propiedad distributiva de la multiplicación sobre la suma y la resta
Ahora vamos a estudiar las bases para comprender los algoritmos de la multiplicación para
números completos. El área del rectángulo grande de la figura 3-28 es igual a la suma de las
áreas de los dos rectángulos pequeños y, por lo tanto, 513 + 42 = 5 # 3 + 5 # 4.
5(3 + 4) = 5 3 + 5 4

5 5 5

3 4 3 4

(3 + 4)

Figura 3-28
También se pueden usar las propiedades de suma y multiplicación para justificar este
resultado:
513 + 42 = 13 + 42 + 13 + 42 + 13 + 42 + 13 + 42 + 13 + 42

Cinco términos: por la definición de


multiplicación.
= 13 + 3 + 3 + 3 + 32 + 14 + 4 + 4 + 4 + 42 , por las propiedades
conmutativa y asociativa de la suma.
= 5 # 3 + 5 # 4, por la definición de multiplicación.
Nota que 513 + 42 se puede pensar como 5 veces 13 + 42.

Este ejemplo ilustra la propiedad distributiva de la multiplicación sobre la suma para números
completos. También es válida la propiedad similar sobre la resta. Como en álgebra se acos-
tumbra escribir a # b como ab, enunciamos la propiedad distributiva de la multiplicación so-
bre la suma y la propiedad distributiva de la multiplicación sobre la resta como sigue:

Teorema 3–6:    Propiedad distributiva de la multiplicación sobre la suma para


números completos
Para cualesquier números completos a, b y c,
a1b + c2 = ab + ac

Teorema 3–7:    Propiedad distributiva de la multiplicación sobre la resta o subs-


tracción para números completos
Para cualesquier números completos a, b y c con b 7 c,
a1b - c2 = ab - ac

© Lopez Mateos Editores. ISBN 978-607-95583-2-1, obra completa, versión electrónica, ISBN 978-607-95583-3-8, volumen 1, versión electrónica. Ejemplar asignado a: Helecto Villarroel gutierrez -
helecto@gmail.com. Fecha: 27 de octubre de 2014. Prohibida su modificación, copia o distribución.
Sección 3-3 Multiplicación y división de números completos 149

O B S E R VA C I Ó N Como en los números completos se cumple la propiedad conmutativa


de la multiplicación, podemos reescribir la propiedad distributiva de la multiplicación
sobre la suma como 1b + c2a = ba + ca.
La propiedad distributiva se puede generalizar para cualquier número finito de
téminos. Por ejemplo, a1b + c + d2 = ab + ac + ad.

La propiedad distributiva se puede escribir como


ab + ac = a1b + c2
Esto se conoce como factorización. Así, los factores de ab + ac son a y 1b + c2.
La propiedad distributiva de la multiplicación sobre la suma ayudará a los alumnos a efec-
tuar cálculos mentales. Por ejemplo, 13 # 7 = 110 + 327 = 10 # 7 + 3 # 7 = 70 + 21 = 91.
La propiedad distributiva de la multiplicación sobre la suma es importante en el estudio del
álgebra y en el desarrollo de algoritmos para operaciones aritméticas. Por ejemplo, se usa
para combinar términos semejantes cuando se trabaja con variables, como en
3x + 5x = 13 + 52x = 8x ó 3ab + 2b = 13a + 22b.

Ejemplo 3-2 a. Usa un modelo de área para mostrar que 1x + y21z + w2 = xz + xw + yz + yw.








b. Usa la propiedad distributiva de la multiplicación sobre la suma para justificar el resul-


tado de la parte (a).

Solución a. Considera el rectángulo de la figura 3-29, cuyo ancho es x + y y su longitud es


z + w. El área de todo el rectángulo es 1x + y21z + w2. Si dividimos el rec-
tángulo en rectángulos más pequeños, según se muestra, notamos que la suma
de las áreas de los cuatro rectángulos pequeños es xz + xw + yz + yw. Como
el área del rectángulo original es igual a la suma de las áreas de los rectángulos
más pequeños, obtenemos el resultado deseado.

z w

x xz xw

y yz yw

Figura 3-29

b. Para aplicar la propiedad distributiva de la multiplicación sobre la suma, pen-


samos x + y como un número y procedemos como sigue:
1x + y21z + w2 = 1x + y2z + 1x + y2w Propiedad distributiva de la
multiplicación sobre la suma
= xz + yz + xw + yw Propiedad distributiva de la
multiplicación sobre la suma
= xz + xw + yz + yw Propiedades conmutativa y
asociativa de la suma

© Lopez Mateos Editores. ISBN 978-607-95583-2-1, obra completa, versión electrónica, ISBN 978-607-95583-3-8, volumen 1, versión electrónica. Ejemplar asignado a: Helecto Villarroel gutierrez -
helecto@gmail.com. Fecha: 27 de octubre de 2014. Prohibida su modificación, copia o distribución.
150 Números completos y sus operaciones

Las propiedades de la multiplicación de números completos pueden reducir las 100 mul-
tiplicaciones básicas, es decir la tabla con los números del 0 al 9, que los estudiantes tienen
que aprender. Por ejemplo, hay 19 multiplicaciones por 0, y 17 más tienen factor 1. Por lo
tanto, conocer las propiedades de la multiplicación por cero y de la multiplicación por la
identidad permite al estudiante dominar 36 multiplicaciones básicas. A continuación, 8
multiplicaciones son cuadrados, como 5 # 5, que los estudiantes parecen conocer; esto deja
56 multiplicaciones. La propiedad conmutativa parte el número en dos pues si los estudian-
tes saben 7 # 9, entonces, por la propiedad conmutativa, saben 9 # 7. Esto deja 28 multiplica-
ciones que los estudiantes pueden aprender, o usar las propiedades asociativa y distributiva
para obtenerlas. Por ejemplo, 6 # 5 se puede pensar como 15 + 125 = 5 # 5 + 1 # 5, ó 30.

División de números completos


Estudiaremos la división mediante tres modelos: el modelo de conjuntos (reparto), el modelo
del factor faltante y el modelo de la resta repetida.

Modelo de conjuntos (reparto)


Supongamos que tenemos 18 caramelos y queremos dar un número igual de caramelos a
cada uno de tres amigos: Beto, David y Carlos. ¿Cuántos deberá recibir cada persona? Si
trazamos una figura, veremos que podemos dividir (o partir) los 18 caramelos en 3 con-
juntos, con igual número de caramelos en cada conjunto. La figura 3-30 muestra que
cada amigo recibió 6 caramelos.

Beto David Carlos

Figura 3-30

Podemos representar la respuesta como 18 , 3 = 6. Así, 18 , 3 es el número de cara-


melos en cada uno de los tres conjuntos ajenos cuya unión tiene 18 caramelos. Con este en-
foque de la división, partimos un conjunto en cierto número de subconjuntos equivalentes.

Modelo del factor faltante


Otra estrategia para dividir 18 caramelos entre tres amigos es usar el modelo del factor faltante.
Si cada amigo recibe c caramelos, entonces los tres amigos recibirán 3c, ó 18, caramelos. En-
tonces, 3c = 18. Como 3 # 6 = 18, tenemos que c = 6. Hemos resuelto el cálculo de la di-
visión por medio de la multiplicación. Esto nos conduce a la siguiente definición de división de
números completos:

Definición de división de números completos


Para cualesquier números a y b, con b Z 0, a , b = c si, y sólo si, c es el único número
completo tal que b # c = a.

El número a es el dividendo, b es el divisor y c es el cociente. Nota que a , b también se


a
puede escribir como ó b 冄 a.
b

© Lopez Mateos Editores. ISBN 978-607-95583-2-1, obra completa, versión electrónica, ISBN 978-607-95583-3-8, volumen 1, versión electrónica. Ejemplar asignado a: Helecto Villarroel gutierrez -
helecto@gmail.com. Fecha: 27 de octubre de 2014. Prohibida su modificación, copia o distribución.
Sección 3-3 Multiplicación y división de números completos 151

Modelo de la resta repetida


Supongamos que tenemos 18 caramelos y queremos empacarlos en bolsitas con 6 cara-
melos cada una. ¿Cuántas bolsitas necesitamos? Podríamos razonar diciendo que si una
bolsita está llena, entonces nos quedarían 18 - 6 (ó 12) caramelos. Si llenamos una bol-
sita más, entonces quedan 12 - 6 (ó 6) caramelos. Finalmente, podemos colocar los 6
caramelos restantes en una tercera bolsita. La exposición anterior puede resumirse escri-
biendo 18 - 6 - 6 - 6 = 0. Hemos hallado, mediante resta repetida, que 18 , 6 = 3.
Tratar la división como resta repetida funciona bien mientras no queden caramelos. Si
quedan caramelos entonces surgirá un residuo no nulo.
Puedes usar una calculadora para ilustrar que la división de números completos se puede
pensar como una resta repetida. Por ejemplo, considera 135 , 15. Si la calculadora tiene
una tecla constante, K , presiona 1 5 - K 1 3 5 = Á y cuenta cuántas veces debes
presionar = hasta que aparezca 0 en la pantalla. Hay calculadoras cuya característica
“constante” es diferente y requieren que se teclee en diferente orden. Por ejemplo, si la cal-
culadora tiene constante automática, podemos teclear 1 3 5 - 1 5 = y contar el nú-
mero de veces que tecleamos = hasta obtener la lectura 0 .

Algoritmo de la división
Así como la resta de números completos no es cerrada, la división de números completos
tampoco es cerrada. Por ejemplo, para efectuar 27 , 5 buscamos un número completo c tal
que 5c = 27.
La tabla 3-2 muestra varios productos de números completos por 5. Como 27 está entre 25
y 30, no existe número completo c tal que 5c = 27. Como ningún número completo c satis-
face esta ecuación, vemos que 27 , 5 no tiene sentido en el conjunto de números completos
y que el conjunto de números completos no es cerrado bajo la división.
Tabla 3-2
5#1 5#2 5#3 5#4 5#5 5#6

5 10 15 20 25 30

Aunque sucede que el conjunto de los números completos no es cerrado bajo la división, la
operación de división tiene sentido con los números completos. Por ejemplo, si repartimos
27 manzanas entre cinco estudiantes, cada estudiante recibirá 5 manzanas y sobrarán 2 man-
zanas. El número 2 es el residuo. Así, 27 contiene cinco 5 con un residuo de 2. Observa que
el residuo es un número completo menor que 5. Ilustramos esta operación en la figura 3-31.
El concepto ilustrado es el algoritmo de la división.

Residuo

27 = 5 . 5 + 2 con 0 ≤ 2 < 5

Figura 3-31

© Lopez Mateos Editores. ISBN 978-607-95583-2-1, obra completa, versión electrónica, ISBN 978-607-95583-3-8, volumen 1, versión electrónica. Ejemplar asignado a: Helecto Villarroel gutierrez -
helecto@gmail.com. Fecha: 27 de octubre de 2014. Prohibida su modificación, copia o distribución.
152 Números completos y sus operaciones

Algoritmo de la división
Dados cualesquier números completos a y b con b Z 0, existen números completos únicos c (co-
ciente) y r (residuo) tales que
a = bc + r con 0 … r 6 b

Cuando “dividimos” a entre b y el residuo es 0, decimos que a es divisible entre b o que b es


un divisor de a, o bien que b divide a a. Por el algoritmo de la división, a es divisible entre b
si a = bc para un único número completo c. Así, 63 es divisible entre 9 porque 63 = 9 # 7.
Nota que 63 también es divisible entre 7 y que el residuo es 0.

Ejemplo 3-3 Si 123 se divide entre un número y el residuo es 13, ¿cuáles son los posibles divisores?








Solución Si dividimos 123 entre b, entonces, por el algoritmo de la división, tenemos


123 = bc + 13 y b 7 13
Usando la definición de resta, tenemos que bc = 123 - 13, y por lo tanto 110 = bc. Ahora
buscamos dos números cuyo producto sea 110, donde un número sea mayor que 13. La ta-
bla 3-3 muestra los pares de números cuyo producto es 110.

Tabla 3-3
1 110
2 55
5 22
10 11

Vemos que 110, 55 y 22 son los únicos valores posibles para b pues cada uno es mayor
que 13.

AHORA INTENTA ÉSTE 3-13 Cuando se organizó a la banda en filas de 5, sobró un elemento. Cuando
los elementos se colocaron en filas de 6, seguía sobrando un elemento. Sin embargo, cuando se colocaron
en filas de 7, nadie sobró. ¿Cuál es el menor número de elementos que podría haber en la banda?

Relacionar la multiplicación y la división como operaciones


inversas
En la sección 3-1 vimos que la resta y la suma estaban relacionadas como operaciones inver-
sas y vimos las familias de hechos de ambas. La división con residuo cero y la multiplicación
están relacionadas de manera análoga. La división es la inversa de la multiplicación. Pode-
mos verlo al considerar las familias de hechos que aparecen en la siguiente Página de un libro
de texto del grado 3. Nota que la pregunta 1 en Tema de plática hace que los estudiantes pien-
sen la división como un modelo de resta repetida al contar hacia abajo desde un punto inicial.
La pregunta 2 hace que piensen la división usando el modelo del factor faltante.

© Lopez Mateos Editores. ISBN 978-607-95583-2-1, obra completa, versión electrónica, ISBN 978-607-95583-3-8, volumen 1, versión electrónica. Ejemplar asignado a: Helecto Villarroel gutierrez -
helecto@gmail.com. Fecha: 27 de octubre de 2014. Prohibida su modificación, copia o distribución.
Sección 3-3 Multiplicación y división de números completos 153

Página de libro de texto RELACIONAR LA MULTIPLICACIÓN


Y LA DIVISIÓN

Lección
Calentamiento
´ Relacionar la
Idea clave
Las familias de hechos multiplicación y la división
muestran cómo están
conectadas la
multiplicación y la división.
Aprende
¿Cómo puede un arreglo ilustrar la división?
Vocabulario
. arreglo (p. 262)
familia de hechos
En 1818 sólo había 20 estrellas
en la bandera de Estados Unidos.
. (p.70)
factor (p.260)
producto (p.260) Había 4 filas iguales de estrellas.
dividendo
divisor
¿Cuántas estrellas había en cada fila?
cociente
El arreglo muestra:

ex Multiplicación División
ef l i ó
4 filas de 5 estrellas = 20 20 estrellas en 4 filas iguales = 5
R

estrellas estrellas en cada fila

¡Piensa!
Puedo usar lo que
ya sé sobre la Así, hay 5 estrellas en
multiplicación para
entender la división.
cada fila.

¿Cómo te puede ayudar a dividir una familia


de hechos?
Una familia de hechos ilustra cómo están
relacionadas la multiplicación y la división.
Familia de hechos para 4, 5, 20:

producto dividendo cociente

Tema de plática

Esteban contó de 5 en 5 para hallar 4 × 5. Después


comenzó en 20 y contó hacia atrás de 5 en 5 hasta
llegar a 0. El número de veces que cuentas hacia
atrás es el cociente de 20 ÷ 5.
.
2 ¿Cómo puedes usar la multiplicación básica 3×6=
18 para hallar 18÷3?
.
3 Sentido numérico ¿Es 3×5=15 parte de la familia de
hechos para 3, 4 y 12? Explica.

Fuente: Scott Foresman-Addison Wesley Math, Grade 3, 2008 (p. 384).

© Lopez Mateos Editores. ISBN 978-607-95583-2-1, obra completa, versión electrónica, ISBN 978-607-95583-3-8, volumen 1, versión electrónica. Ejemplar asignado a: Helecto Villarroel gutierrez -
helecto@gmail.com. Fecha: 27 de octubre de 2014. Prohibida su modificación, copia o distribución.
154 Números completos y sus operaciones

A continuación vemos cómo están relacionadas las cuatro operaciones de suma, resta,
multiplicación y división en el conjunto de los números completos. Esto se muestra en la fi-
gura 3-32. Nota que la suma y la resta son inversas entre sí, como lo son la multiplicación y
la división con residuo 0. Nota también que la multiplicación es una suma repetida y que la
división es una resta repetida.

Operaciones
1 inversas
2

Suma Relaciones Resta


repetida entre las repetida
operaciones

Operaciones
3 inversas
4

Figura 3-32
En la sección 3-1 vimos cómo el conjunto de los números completos es cerrado bajo la
suma y que la suma es conmutativa y asociativa y tiene un elemento identidad. Sin embargo,
la resta no tenía estas propiedades. En esta sección hemos visto que la multiplicación tiene al-
gunas de las propiedades que valen para la suma. ¿Se sigue que la división tiene algunas de las
propiedades de la resta? Investiga esto en Ahora intenta éste 3-14.

AHORA INTENTA ÉSTE 3-14


a. Proporciona contraejemplos para mostrar que el conjunto de números completos no es cerrado bajo la
división y que la división no es conmutativa ni asociativa.
b. ¿Por qué el 1 no es la identidad para la división?

División entre 0 y entre 1


Con frecuencia los estudiantes se equivocan con la división entre 0 y entre 1. Antes de se-
guir leyendo, intenta hallar valores para las tres expresiones siguientes:
1. 3 , 0 2. 0 , 3 3. 0 , 0
Considera las siguientes explicaciones:
1. Por definición, 3 , 0 = c si existe un número único c tal que 0 # c = 3. Como la pro-
piedad de la multiplicación por cero afirma que 0 # c = 0 para cualquier número com-
pleto c, no existe número completo c tal que 0 # c = 3. Así, 3 , 0 está indefinida pues
no existe respuesta al problema equivalente de multiplicación.
2. Por definición, 0 , 3 = c si existe un número único tal que 3 # c = 0. Cualquier
número multiplicado por 0 da 0 y, en particular, 3 # 0 = 0. Entonces, c = 0 y
0 , 3 = 0. Nota que c = 0 es el único número que satisface 3 # c = 0.
3. Por definición, 0 , 0 = c si existe un número completo único c tal que 0 # c = 0. Nota
que para cualquier c, 0 # c = 0. Pero, de acuerdo con el algoritmo de la división, c debe
ser único. Como no existe un único número c tal que 0 # c = 0, se sigue que 0 , 0 está
indeterminado o indefinido.

© Lopez Mateos Editores. ISBN 978-607-95583-2-1, obra completa, versión electrónica, ISBN 978-607-95583-3-8, volumen 1, versión electrónica. Ejemplar asignado a: Helecto Villarroel gutierrez -
helecto@gmail.com. Fecha: 27 de octubre de 2014. Prohibida su modificación, copia o distribución.
Sección 3-3 Multiplicación y división de números completos 155

Página de libro de texto DIVISIÓN ENTRE 0 Y ENTRE 1

Lección
Calentamiento
Idea clave
Pensar en División entre 0 y 1
multiplicaciones
básicas te puede Aprende
ayudar a
comprender las
reglas de la división ¿Cuáles son las reglas de la división para el 0 y el 1?
entre 0 y 1.
Ejemplo A
Lo que piensas Lo que escribes
Divide un ¿1 por cuánto=4?
número entre 1.
Así, 4÷1=4 ´

Regla: Cuando cualquier número se divide entre 1, el cociente es ese número.

Ejemplo B
Divide un ¿7 por cuánto =7?
número entre
Así, 7÷7=1.
sí mismo. ´

Regla: Cuando cualquier número (excepto el 0) se divide


entre sí mismo, el cociente es 1.

Ejemplo C
Divide cero ¿2 por cuánto=0?
entre un número.
Así, 0÷2=0 ó

Regla: Cuando cero se divide entre un número


(excepto el 0), el cociente es 0.

Ejemplo D
Divide un ¿0 por cuánto = 3?
No puede
número Ningún número funciona, así
entre cero. efectuarse 3÷0.
que no puede efectuarse 3÷0.

Regla: No puedes dividir un número entre 0.

Tema de plática
¿Cómo puedes decir, sin dividir, que 427÷1=427?

Fuente: Scott Foresman-Addison Wesley Math, Grade 3, 2008 (p. 396).

© Lopez Mateos Editores. ISBN 978-607-95583-2-1, obra completa, versión electrónica, ISBN 978-607-95583-3-8, volumen 1, versión electrónica. Ejemplar asignado a: Helecto Villarroel gutierrez -
helecto@gmail.com. Fecha: 27 de octubre de 2014. Prohibida su modificación, copia o distribución.
156 Números completos y sus operaciones

Las divisiones que incluyen el 0 se pueden resumir como sigue. Sea n cualquier número
completo distinto de cero. Entonces,
1. n , 0 está indefinido; 2. 0 , n = 0; 3. 0 , 0 está indefinido.
Recuerda que n # 1 = n para cualquier número completo n. Así, por la definición de di-
visión, n , 1 = n. Por ejemplo, 3 , 1 = 3, 1 , 1 = 1 y 0 , 1 = 0. En la Página de un li-
bro de texto de la página 155 puedes hallar un estudio correspondiente al grado 3 de la
división entre 0 y entre 1.

Orden de las operaciones


A veces surgen dificultades respecto al orden de efectuar operaciones aritméticas. Por ejemplo,
habrá estudiantes que traten 2 + 3 # 6 como 12 + 326, mientras que otros lo tratarán como
2 + 13 # 62. En el primer caso el valor es 30 y en el segundo caso el valor es 20. Para evitar con-
fusiones, los matemáticos están de acuerdo en que cuando no hay paréntesis, las multiplicacio-
nes y divisiones se efectúan antes que las sumas y restas. Las multiplicaciones y divisiones se
efectúan en el orden en que se presentan, y después las sumas y las restas se efectúan en el orden
en que se presenten. Así, 2 + 3 # 6 = 2 + 18 = 20. Este orden de efectuar las operaciones no
lo tienen algunas calculadoras, que presentan una respuesta equivocada de 30. El cálculo
8 - 9 , 3 # 2 + 3 se efectúa:
8 - 9 , 3#2 + 3 = 8 - 3#2 + 3
= 8 - 6 + 3
= 2 + 3
= 5
Evaluación 3-3A

1. Halla, si es posible, los números completos que hacen b. n1¢ + ~2


verdaderas las ecuaciones siguientes: c. a1b + c2 - ac
a. 3 # n = 15 b. 18 = 6 + 3 # n 6. Coloca paréntesis, si es necesario, para hacer que las
c. n # 15 + 62 = n # 5 + n # 6 ecuaciones siguientes sean verdaderas:
2. En términos de la teoría de conjuntos, se podría pensar a. 5 + 6 # 3 = 33
en el producto na como el número de elementos en la b. 8 + 7 - 3 = 12
unión de n conjuntos con a elementos en cada uno. Si c. 6 + 8 - 2 , 2 = 13
ése fuera el caso, ¿qué deberían cumplir, necesariamente, d. 9 + 6 , 3 = 5
los conjuntos? 7. Usando la propiedad distributiva de la multiplicación so-
3. Determina si los conjuntos siguientes son cerrados bajo bre la suma podemos factorizar, como en x2 + xy =
la multiplicación: x1x + y2. Usa la propiedad distributiva y otras propieda-
a. 50, 16 b. 52, 4, 6, 8, 10, Á 6 des de la multiplicación para factorizar lo siguiente:
c. 51, 4, 7, 10, 13, Á 6 a. xy + y2 b. xy + x
4. a. Si quitamos el 5 del conjunto de los números comple- c. a2b + ab2
tos, ¿el conjunto es cerrado respecto a la suma? Explica. 8. Halla números completos que hagan verdadera, si es po-
b. Si quitamos el 5 del conjunto de los números comple- sible, cada una de las proposiciones siguientes:
tos, ¿el conjunto es cerrado respecto a la multiplica- a. 18 , 3 = n b. n , 76 = 0
ción? Explica. c. 28 , n = 7
c. Responde las mismas preguntas que en (a) y (b) si qui- 9. Una tienda de artículos deportivos tiene 6 modelos de ca-
tamos el 6 del conjunto de los números completos. misas, 4 de pantalones y 3 de chalecos. ¿Cuántos unifor-
5. Reescribe lo siguiente usando la propiedad distributiva mes diferentes de camisa-pantalón-chaleco son posibles?
de la multiplicación sobre la suma, de manera que no 10. ¿Qué propiedad se ilustra en cada caso?
haya paréntesis en la respuesta: a. 615 # 42 = 16 # 524
a. 1a + b21c + d2

© Lopez Mateos Editores. ISBN 978-607-95583-2-1, obra completa, versión electrónica, ISBN 978-607-95583-3-8, volumen 1, versión electrónica. Ejemplar asignado a: Helecto Villarroel gutierrez -
helecto@gmail.com. Fecha: 27 de octubre de 2014. Prohibida su modificación, copia o distribución.
Sección 3-3 Multiplicación y división de números completos 157

b. 615 # 42 = 614 # 52 a. 1a , b2 , c = a , 1b , c2
c. 615 # 42 = 15 # 426 b. a , 1b + c2 = 1a , b2 + 1a , c2
d. 115 # 42 = 5 # 4 20. Supón que c es un divisor de a y de b. Muestra que
e. 13 + 42 # 0 = 0 1a + b2 , c = 1a , c2 + 1b , c2 utilizando
f. 13 + 4215 + 62 = 13 + 425 + 13 + 426 a. un modelo.
11. Se escuchó a unas estudiantes afirmar lo siguiente. ¿Qué b. la definición de división en términos de multiplica-
propiedades justifican sus afirmaciones? ción y la propiedad distributiva de la multiplicación
a. Yo sé que 9 # 7 es 63 ó 69 y que no pueden ser correc- sobre la suma.
tos ambos. 21. Halla el conjunto solución para cada caso:
b. Yo sé que 9 # 0 es 0 porque sé que cualquier número a. 5x + 2 = 22 b. 3x + 7 = x + 13
por 0 es 0. c. 31x + 42 = 18
c. Cualquier número por 1 es el mismo número con el 22. Miriam y Sabina comenzaron a ahorrar al mismo
que comencé, así que 9 # 1 es 9. tiempo. Miriam planea ahorrar $3 diarios y Sabina $5
12. El producto 6 # 14 se puede obtener pensando el problema diarios. ¿Después de cuántos días tendrá Sabina exacta-
como 6110 + 42 = 16 # 102 + 16 # 42 = 60 + 24 = 84. mente $10 más que Miriam?
a. ¿Qué propiedad se usó? 23. Para un día de campo hay 17 emparedados para 7 personas.
b. Usa esta técnica para calcular mentalmente 32 # 12. ¿Cuántos emparedados completos le tocan a cada persona
13. La propiedad distributiva de la multiplicación sobre la si se distribuyen equitativamente? ¿Cuántos sobraron?
resta es 24. a. Halla todos los pares de números completos cuyo
a1b - c2 = ab - ac producto sea 36.
Usa esta propiedad para obtener: b. Localiza en una malla los puntos hallados en (a).
a. 9110 - 22 c. Compara el patrón en la figura formada por los pun-
b. 2018 - 32 tos de la gráfica con el patrón de la figura que se for-
14. Muestra que 1a + b22 = a2 + 2ab + b2 por medio de
maría usando todos los pares de números completos
cuya suma es 36.
a. la propiedad distributiva de la multiplicación sobre la
25. Hay disponible un nuevo modelo de automóvil en 4 colo-
suma y otras propiedades.
res exteriores y 3 colores interiores. Usa un diagrama de
b. un modelo de área.
árbol y colores específicos para mostrar cuántas combina-
15. Si a y b son números completos con a 7 b, usa los rec-
tángulos de la figura para explicar por qué 1a + b22 -
ciones de color son posibles para un automóvil.
1a - b22 = 4ab. 26. Para efectuar 7 , 5 en la calculadora, se teclea 7 , 5 = ,
con lo cual se obtiene 1.4. Para hallar el número completo que
es el residuo, ignora la parte decimal de 1.4, multiplica 5 # 1 y
b a
resta de 7 este producto. El resultado es el residuo. Usa una cal-
b culadora para hallar el número completo que es el residuo de
a
las divisiones siguientes:
a a. 28 , 5 b. 32 , 10
b c. 29 , 3 d. 41 , 7
a b
e. 49,382 , 14
27. ¿Es posible hallar un número completo menor que 100
16. Demuestra, en cada caso, que el lado izquierdo de la que al dividirlo entre 10 deje un residuo de 4 y que al di-
ecuación es igual al lado derecho y da una razón para vidirlo entre 47 deje un residuo de 17?
cada paso: 28. Di, en cada caso, qué operación debe realizarse al último:
a. 1ab2c = 1ca2b b. 1a + b2c = c1b + a2 a. 5116 - 72 - 18
17. Factoriza: b. 54>110 - 5 + 42
a. xy - y2 c. 114 - 32 + 124 # 22
b. 47 # 101 - 47 d. 21,045>345 + 8
c. ab2 - ba2 29. Escribe una expresión algebraica para:
18. Reescribe cada uno de los siguientes problemas de di- a. El ancho de un rectángulo con área A y longitud l
visión como un problema de multiplicación: b. Pies, p, en yardas
a. 40 , 8 = 5 b. 326 , 2 = x c. Horas, h, en minutos
19. Muestra que, en general, para números completos a, b y d. Días, d, en semanas
c es falso que:

© Lopez Mateos Editores. ISBN 978-607-95583-2-1, obra completa, versión electrónica, ISBN 978-607-95583-3-8, volumen 1, versión electrónica. Ejemplar asignado a: Helecto Villarroel gutierrez -
helecto@gmail.com. Fecha: 27 de octubre de 2014. Prohibida su modificación, copia o distribución.
158 Números completos y sus operaciones

Evaluación 3-3B

1. Halla, si es posible, los números completos que hacen


verdaderas las ecuaciones siguientes: 11. Se escuchó a unas estudiantes afirmar lo siguiente. ¿Qué
a. 8 # n = 24 b. 28 = 4 + 6 # n propiedades justifican sus afirmaciones?
c. n 18 + 62 = n 8 + n # 6
# # a. Yo recuerdo cuánto es 7 # 9, y, por lo tanto, sé cuánto
2. Determina si los conjuntos siguientes son cerrados bajo es 9 # 7.
la multiplicación: b. Para obtener 9 # 6, sólo necesito recordar que 9 # 5 es
a. 506 b. 51, 3, 5, 7, 9, Á 6 45 y así 9 # 6 es 9 más que 45, ó 54.
c. 50, 1, 26 12. El producto 5 # 24 se puede obtener pensando el problema
3. Reescribe usando la propiedad distributiva de la multipli- como 5120 + 42 = 5 # 20 + 5 # 4 = 100 + 20 = 120.
cación sobre la suma, de manera que no haya paréntesis a. ¿Qué propiedad se usó?
en la respuesta. Simplifica cuando sea posible. b. Usa esta técnica para calcular mentalmente 8 # 34.
a. 31x + y + 52 13. La propiedad distributiva de la multiplicación sobre la
b. 1x + y21x + y + z2 resta es
c. x1 y + 12 - x a1b - c2 = ab - ac
4. Coloca paréntesis, si es necesario, para hacer que las Usa esta propiedad para obtener:
ecuaciones siguientes sean verdaderas: a. 15110 - 22 b. 3019 - 22
a. 4 + 3 # 2 = 14 14. Muestra que si b 7 c, entonces a1b - c2 = ab - ac usando:
b. 9 , 3 + 1 = 4 a. el modelo de área sugerido por la figura dada (expresa
c. 5 + 4 + 9 , 3 = 6 el área sombreada de dos maneras diferentes).
d. 3 + 6 - 2 , 1 = 7
5. La propiedad distributiva generalizada a tres términos dice b – c c
que para cualesquier números completos a, b, c y d,
a1b + c + d2 = ab + ac + ad. Justifica esta propiedad
usando la propiedad distributiva para dos términos.
6. Usando la propiedad distributiva de la multiplicación sobre a
la suma podemos factorizar, como en x2 + xy = x1x + y2.
Usa la propiedad distributiva y otras propiedades para fac-
torizar lo siguiente:
a. 47 # 99 + 47
b. 1x + 12y + 1x + 12
c. x2y + zx3 b
7. Halla números completos que hagan verdadera, si es po-
sible, cada una de las proposiciones siguientes: b. la definición de resta en términos de suma y de la
a. 27 , 9 = n b. n , 52 = 1 propiedad distributiva de la multiplicación sobre la
c. 13 , n = 13 suma.
8. Un carro nuevo viene en 5 colores exteriores y 3 colores in- 15. Muestra que el lado izquierdo de la ecuación es igual al
teriores . ¿Cuántas presentaciones de carros son posibles? lado derecho y da una razón para cada paso.
9. ¿Qué multiplicación sugieren los siguientes modelos? a. 1ab2c = b1ac2 b. a1b + c2 = ac + ab
a. 16. Factoriza:
a. xy - y
XX XX XX XX b. 1x + 12y - 1x + 12
X X X X c. a2b3 - ab2
17. Reescribe cada uno de los siguientes problemas de divi-
b. sión como un problema de multiplicación:
a. 48 , x = 16 b. x , 5 = 17
10. ¿Qué propiedad de los números completos se ilustra en 18. Piensa un número. Multiplícalo por 2. Súmale 2. Diví-
cada caso? delo entre 2. Réstale 1. ¿Se parece el resultado al número
a. 15 # 420 = 0 que pensaste? ¿Esto funcionará en todos los casos? Ex-
b. 713 # 42 = 714 # 32 plica tu respuesta.
c. 713 # 42 = 13 # 427 19. Muestra que, en general, para números completos a, b y
d. 13 + 421 = 3 + 4 c es falso que:
e. 13 + 425 = 3 # 5 + 4 # 5 a. a , b = b , a
f. 11 + 2213 + 42 = 11 + 223 + 11 + 224 b. a - b = b - a
© Lopez Mateos Editores. ISBN 978-607-95583-2-1, obra completa, versión electrónica, ISBN 978-607-95583-3-8, volumen 1, versión electrónica. Ejemplar asignado a: Helecto Villarroel gutierrez -
helecto@gmail.com. Fecha: 27 de octubre de 2014. Prohibida su modificación, copia o distribución.
Sección 3-3 Multiplicación y división de números completos 159

20. Halla la solución para cada caso: a. Usa las teclas 1 , 9 y 7 sólo una vez, en cualquier
a. 5x + 8 = 28 b. 5x + 6 = x + 14 orden y usando cualquier operación, para escribir la
c. 51x + 32 = 35 mayor cantidad posible de números completos del 1 al
21. Es posible hacer figuras artísticas conectando por medio de 20. Por ejemplo, 9 - 7 - 1 = 1 y 1 # 9 - 7 = 2.
un segmento de recta puntos regularmente espaciados, mar- b. Usa la tecla 4 las veces que quieras, con cualquier
cados en los ejes vertical y horizontal. Conecta el punto más operación, hasta obtener 13.
lejano del origen sobre el eje vertical con el punto más cer- c. Usa tres veces la tecla 2 con cualquier operación,
cano al origen en el eje horizontal. Continúa de esa manera para obtener 24.
hasta que estén conectados todos los puntos, como se mues-
d. Usa cinco veces la tecla 1 con cualquier operación,
tra en la figura siguiente. ¿Cuántos puntos de intersección se
para obtener 100.
crearon al marcar 10 puntos en cada eje?
25. Di, en cada caso, qué operación debe realizarse al último:
a. 5 # 6 - 3 # 4 + 2
b. 19 - 3 # 4 + 9 , 3
c. 15 - 6 , 2 # 4
d. 5 + 18 - 223
26. Halla infinidad de números completos que dejen residuo
3 al dividirlos entre 5.
27. La operación } se define en el conjunto C = 5a, b, c6,
3 marcas por eje como se muestra en la tabla siguiente. Por ejemplo,
3 intersecciones a } b = b y b } a = b.

22. Un grupo de estudiantes se divide en ocho equipos de } a b c


nueve personas cada uno. Más tarde los mismos estu-
a a b c
diantes se dividen en equipos de seis cada uno; ¿cuántos
b b a c
equipos formaron?
23. Juanito tiene una gran colección de canicas. Ha notado c c c c
que si algún amigo le presta 5 canicas, podría ordenar las a. ¿Es C cerrado respecto a } ?
canicas en filas de 13 cada una. ¿Cuál es el residuo que ob- b. ¿Es } conmutativa en C?
tiene al dividir su número original de canicas entre 13? c. ¿Existe algún elemento identidad para } en C? De
24. En los problemas siguientes sólo puedes teclear los núme- ser así, di cuál.
ros mencionados, pero puedes presionar cualquier tecla d. Por medio de ejemplos, investiga la propiedad asocia-
de función. tiva para } en C.

Conexiones matemáticas 3-3

Comunicación Aprendizaje colectivo


1. Un número deja residuo 6 al dividirlo entre 10. ¿Cuál es 9. Hay varias multiplicaciones básicas que la mayoría de los
el residuo de ese número al dividirlo entre 5? Justifica tu niños ha memorizado, como las de la tabla siguiente,
razonamiento. parcialmente llena:
2. ¿Puede ser el 0 un elemento identidad para la multipli-
cación? Explica por qué sí o por qué no.
3 1 2 3 4 5 6 7 8 9
3. Supón que olvidaste el producto 9 # 7. Describe varias
maneras de obtener el producto usando diferentes mul- 1
tiplicaciones básicas y propiedades.
2
4. ¿Es siempre cierto que x , x es igual a 1? Explica tu res-
puesta. 3
5. ¿Hay algún caso en que x # x sea igual a x? Explica tu res-
4 16
puesta.
6. Describe todos los pares de números completos tales que 5 35
su suma y su producto sean iguales. 6
Solución abierta
7
7. Describe una situación real que pudiera ser representada
por la expresión 3 + 2 # 6. 8 72
8. ¿Cómo explicarías a un niño que un número par es de la 9 81
forma 2c y que uno impar es de la forma 2c + 1, donde c
es un número completo?
© Lopez Mateos Editores. ISBN 978-607-95583-2-1, obra completa, versión electrónica, ISBN 978-607-95583-3-8, volumen 1, versión electrónica. Ejemplar asignado a: Helecto Villarroel gutierrez -
helecto@gmail.com. Fecha: 27 de octubre de 2014. Prohibida su modificación, copia o distribución.
160 Números completos y sus operaciones

a. Llena la tabla de multiplicación. Halla la mayor can- Preguntas del Third International Mathematics and
tidad de patrones que puedas. Lista todos los patro- Science Study (TIMSS) (Tercer Estudio Internacional
nes descubiertos por tu grupo y explica por qué se sobre las Matemáticas y la Ciencia)
presentan dichos patrones en la tabla. En la clase de Tobías hay el doble de niñas que de niños.
b. ¿Cómo puede usarse la tabla de multiplicación para Hay 8 niños en la clase. ¿Cuál es el número total de ni-
resolver problemas de división? ñas y niños en la clase?
c. Considera el número impar 35, que aparece en la tabla a. 12
de multiplicación. Considera todos los números que lo b. 16
rodean. Nota que todos son pares. ¿Sucede lo mismo c. 20
para todos los números impares de la tabla? Explica d. 24
por qué sí o por qué no. TIMSS 2003, Grado 4
Preguntas del salón de clase Una pieza de cuerda de 204 cm se corta en 4 partes igua-
10. Supón que un alumno argumenta que 0 , 0 = 0 porque les. ¿Cuál de las siguientes operaciones da la longitud de
“nada dividido entre nada” es “nada”. ¿Cómo podrías cada parte, en centímetros?
ayudarlo? a. 204 + 4
11. Susi asegura que lo siguiente es cierto, por la ley distri- b. 204 * 4
butiva, donde a y b son números completos: c. 204 - 4
d. 204 , 4
31ab2 = 13a213b2
TIMSS 2003, Grado 4
¿Cómo la puedes ayudar? Pregunta del National Assessment of Educational Progress
12. a. Un estudiante asegura que para todo número com- (NAEP) (Evaluación Nacional del Progreso Educativo)
pleto 1ab2 , b = a. ¿Qué le respondes?
b. El estudiante de la parte (a) asegura que 0 , 0 = 0.
El razonamiento del estudiante es, “Si a = 0 y b = 0 N
se substituyen en la ecuación de la parte (a), el resul- 3 3 3 3 3 3
tado es 0 # 0 , 0 = 0. Pero como 0 # 0 = 0, se sigue
que 0 , 0 = 0”. ¿Cómo le respondes?
13. Una estudiante pregunta si la división en el conjunto de
números completos es distributiva sobre la resta. ¿Qué
le respondes? Los pesos en la balanza anterior están equilibrados. Cada
14. Un estudiante dice que el 1 es la identidad para la di- cubo pesa 3 kilos. El cilindro pesa N kilos. ¿Cuál de las
visión. ¿Qué le respondes? expresiones numéricas describe mejor la situación?
Problemas de repaso a. 6 + N = 12
b. 6 + N = 4
15. Da un conjunto que no sea cerrado bajo la suma.
c. 2 + N = 12
16. ¿Es conmutativa la operación de resta para los números
d. 2 + N = 4
completos? De no ser así, exhibe un contraejemplo.
17. ¿Dónde está el error? NAEP, 2007, Grado 4
a. 137 b. 35 c. 56 d. 46
+ 56 + 47 - 29 - 17
183 712 33 39

ACTIVIDAD DE LABORATORIO Teclea en la calculadora un número menor que 20. Si el número


es par, divídelo entre 2; si es impar, multiplícalo por 3 y súmale 1. A continuación, usa el número que apa-
rece en la pantalla. Sigue las instrucciones dadas. Repite el proceso.
1. ¿Llegarás a 1?
2. ¿Cuál es el número menor que 20 que emplea el mayor número de pasos para llegar a 1?
3. ¿Qué números llegan más rápido a 1, los pares o los impares?
4. Investiga lo que sucede con los números mayores que 20

© Lopez Mateos Editores. ISBN 978-607-95583-2-1, obra completa, versión electrónica, ISBN 978-607-95583-3-8, volumen 1, versión electrónica. Ejemplar asignado a: Helecto Villarroel gutierrez -
helecto@gmail.com. Fecha: 27 de octubre de 2014. Prohibida su modificación, copia o distribución.
Sección 3-4 Algoritmos para multiplicar y dividir números completos 161

3-4 Algoritmos para multiplicar y dividir números completos

En los Puntos focales para el grado 4, hallamos lo siguiente respecto a la multiplicación y di-
visión, y respecto al uso de algoritmos para efectuar cálculos o cuentas:

Las y los estudiantes emplean su comprensión de la multiplicación para desarrollar una manera rápida
de recordar las tablas de la multiplicación y las tablas de la división relacionadas. Aplican su compren-
sión de los modelos de la multiplicación (es decir, grupos del mismo tamaño, arreglos, modelos de
área, intervalos iguales en la recta numérica), del valor posicional y de las propiedades de las opera-
ciones (en particular, la propiedad distributiva) cuando desarrollan, analizan y usan de manera efi-
ciente, precisa y generalizable métodos para multiplicar números completos de varios dígitos.
Seleccionan métodos apropiados y los aplican para estimar productos o calcularlos mentalmente, de-
pendiendo del contexto y de los números involucrados. Desarrollan soltura para usar procedimientos
eficientes, incluyendo el algoritmo usual para multiplicar números completos, y entienden por qué
funcionan los procedimientos (con base en el valor posicional y las propiedades de las operaciones) y
los usan para resolver problemas. (p. 16)

En esta sección se desarrollarán algoritmos para la multiplicación y división usando varios


modelos.
Algoritmos de multiplicación
A fin de desarrollar algoritmos para multiplicar números completos de varios dígitos, usa-
mos la estrategia de examinar primero cálculos sencillos. Considera 4 # 12. Este cálculo se puede
ilustrar como en la figura 3-33(a) con 4 filas de 12 cubos, ó 48 cubos. Los cubos de la figura
3-33(a) también pueden partirse para ilustrar que 4 # 12 = 4110 + 22 = 4 # 10 + 4 # 2. Los
números 4 # 10 y 4 # 2 son productos parciales.

(a) (b) Decenas Unidades 10 + 2 12 12


1 2 : * 4 : * 4 : * 4
4
* 4 40 + 8 :8 : 48
: 40
10 2 48

12

Figura 3-33

La figura 3-33(a) ilustra la propiedad distributiva de la multiplicación sobre la suma en el


conjunto de números completos. En la figura 3-33(b) se ve el proceso que conduce a un al-
goritmo para multiplicar 4 # 12. Nota la analogía entre la multiplicación de la figura 3-33 y
la siguiente multiplicación algebraica:
41x + 22 = 4x + 4 # 2
= 4x + 8
Asimismo, nota la analogía entre el producto
23 # 14 = 12 # 10 + 3211 # 10 + 42 y 12x + 3211x + 42

© Lopez Mateos Editores. ISBN 978-607-95583-2-1, obra completa, versión electrónica, ISBN 978-607-95583-3-8, volumen 1, versión electrónica. Ejemplar asignado a: Helecto Villarroel gutierrez -
helecto@gmail.com. Fecha: 27 de octubre de 2014. Prohibida su modificación, copia o distribución.
162 Números completos y sus operaciones

La analogía continúa según se muestra a continuación:

2 # 10 + 3 2x + 3
* 11 # 10 + 42 * 11x + 42
12 8x + 12
#
8 10 2x2 + 3x
3 # 10 2x2 + 11x + 12
2 # 102
2 # 102 + 11 # 10 + 12

Exploraremos la multiplicación de numerales de tres o más dígitos por un factor de un


dígito después de estudiar la multiplicación por una potencia de 10.
Multiplicación por 10n
Ahora consideraremos multiplicaciones por potencias de 10. Primero veremos qué sucede
cuando multiplicamos por 10 un número dado, como 10 # 23. Si comenzamos con nuestra
representación del 23 con cubos de base diez, tenemos 2 barras y 3 unidades. Para multipli-
car por 10 debemos reemplazar cada pieza con una pieza de base diez que represente la si-
guiente potencia mayor de 10. Esto se muestra en la figura 3-34. Nota que al multiplicar
por 10 las 3 unidades de 23, éstas se transforman en 3 barras ó 3 decenas. Por lo tanto, des-
pués de multiplicar por 10 no hay unidades y tenemos, así, 0 en el lugar de las unidades. En
general, si multiplicamos cualquier número por 10, anexamos un 0 al final del número.

23 Centenas Decenas Unidades


2 3
× 1 0
2 3 0

× 10

230

Figura 3-34

El cálculo de 23 # 10 en la figura 3-34 se puede explicar como sigue:


23 # 10 = 12 # 10 + 3210
= 12 # 10210 + 3 # 10
= 2110 # 102 + 3 # 10
= 2 # 102 + 3 # 10
= 2 # 102 + 3 # 10 + 0 # 1
= 230

© Lopez Mateos Editores. ISBN 978-607-95583-2-1, obra completa, versión electrónica, ISBN 978-607-95583-3-8, volumen 1, versión electrónica. Ejemplar asignado a: Helecto Villarroel gutierrez -
helecto@gmail.com. Fecha: 27 de octubre de 2014. Prohibida su modificación, copia o distribución.
Sección 3-4 Algoritmos para multiplicar y dividir números completos 163

Para calcular productos como 3 # 200, procedemos así:


3 # 200 = 312 # 1022
= 13 # 22102
= 6 # 102
= 6 # 102 + 0 # 10 + 0 # 1
= 600
Vemos que multiplicar 6 por 102 da como resultado la anexión de dos ceros a 6. Esta idea
puede generalizarse mediante la afirmación de que al multiplicar cualquier número natural
por 10n, donde n es un número natural, el resultado es anexar n ceros al número.

O B S E R VA C I Ó N El hecho de añadir n ceros cuando multiplicamos por 10n también


puede explicarse como sigue. Primero multiplicamos por 10, lo cual da como resultado
añadir un cero 1como en 23 # 10 = 2302. Cuando multiplicamos por otro 10, se añade
otro cero 1como en 230 # 10 = 23002. Como multiplicamos n veces por 10, se añaden n
ceros al número natural factor.

Cuando multiplicamos potencias de 10, usamos la definición de exponente. Por ejemplo,


102 # 101 = 110 # 10210 = 103, ó 102 + 1. En general, am # an, donde a es un número natural y
m y n son números completos, está dado por:
am # an = 1a # a # a # Á # a2 # 1a # a # a # Á # a2
m factores n factores
# # #
= a a a Á a = a # m+n

m + n factores
En consecuencia, am # an = am + n.

AHORA INTENTA ÉSTE 3-15 Usa el hecho de que am # an = am + n entre otras propiedades de la
multiplicación, para explicar por qué ambos cálculos en la tira cómica son ciertos.

PENSÉ QUE SI ERAN 20 ¡NO VEAN, 10X600 ES LO


MINUTOS A 300º, LO FUNCIONA MISMO QUE ¡NO, NO
PODRÍA HORNEAR 10 ASÍ, DOT! 20X300 LO ES!
MINUTOS A 600º

Multiplicar por una potencia de 10 es útil al calcular el producto de un número de un dígito


y un número de tres dígitos. En el ejemplo siguiente usamos el algoritmo desarrollado

© Lopez Mateos Editores. ISBN 978-607-95583-2-1, obra completa, versión electrónica, ISBN 978-607-95583-3-8, volumen 1, versión electrónica. Ejemplar asignado a: Helecto Villarroel gutierrez -
helecto@gmail.com. Fecha: 27 de octubre de 2014. Prohibida su modificación, copia o distribución.
164 Números completos y sus operaciones

anteriormente para multiplicar un numeral de un dígito por un numeral de dos dígitos:

4 # 367 = 413 # 102 + 6 # 10 + 72 367


= 413 # 1022 + 416 # 102 + 4 # 7 : * 4
= 14 # 32102 + 14 # 6210 + 4 # 7 28
= 1200 + 240 + 28 240
1200
= 1468
1468

AHORA INTENTA ÉSTE 3-16 Usa la suma expandida y un enfoque similar al anterior para calcular
7 # 4589.

Multiplicación con factores de dos dígitos


Considera 14 # 23. Modelemos este cálculo comenzando con el uso de cubos de base diez,
como se muestra en la figura 3-35(a), y después mostrando todos los productos parciales y su-
mando, como se muestra en la figura 3-35(b).
(10 3 20 5 200) (10 3 3 5 30)
20 3

23
3 14
10 12 (4 3 3)
80 (4 3 20)
30 (10 3 3)
+ 200 (10 3 20)
322 (12 + 80 + 30 + 200)

(a) (4 3 20 = 80) (4 3 3 5 12) (b)


Figura 3-35

Este último enfoque conduce a un algoritmo para la multiplicación:


23 23
* 14 110 + 42 * 14
92 (4 # 232 ó 92
230 110 # 232 23
322 322
Estamos acostumbrados a ver el producto parcial 230 escrito sin el cero, como 23. Colocar
el 23 con el 3 en la columna de las decenas evita tener que escribir el 0 en la columna de las
unidades. Cuando los niños comiencen a aprender los algoritmos de la multiplicación, se les
debe pedir que incluyan el cero. Esto propicia una mejor comprensión del procedimiento y
ayuda a evitar errores.
Se puede usar la propiedad distributiva de la multiplicación sobre la suma para explicar
por qué funciona el algoritmo de la multiplicación. Considera de nuevo 14 # 23.
14 # 23 = 110 + 4223
= 10 # 23 + 4 # 23
= 230 + 92
= 322
© Lopez Mateos Editores. ISBN 978-607-95583-2-1, obra completa, versión electrónica, ISBN 978-607-95583-3-8, volumen 1, versión electrónica. Ejemplar asignado a: Helecto Villarroel gutierrez -
helecto@gmail.com. Fecha: 27 de octubre de 2014. Prohibida su modificación, copia o distribución.
Sección 3-4 Algoritmos para multiplicar y dividir números completos 165

Como los algoritmos son tan poderosos, a veces se tiende a aplicarlos en exceso o a usar pa-
pel y lápiz para efectuar tareas que deberían hacerse mentalmente. Por ejemplo, considera
213
* 1000
000
000
000
213
213000
Esta aplicación del algoritmo no está mal, pero es ineficiente. Efectuar cálculos y estima-
ciones mentales es una herramienta importante en el aprendizaje de las matemáticas y debe
practicarse además de los cálculos con papel y lápiz. Se debe impulsar a niñas y niños a esti-
mar para saber si sus respuestas son razonables. Al calcular 14 # 23 ya sabemos que la res-
puesta debe estar entre 10 # 20 = 200 y 20 # 30 = 600 porque 10 6 14 6 20 y
20 6 23 6 30.

Multiplicación reticular
La multiplicación reticular tiene la ventaja de posponer todas las sumas hasta haber reali-
zado todas las multiplicaciones de un dígito. Quizá por ello es que se le llama “algoritmo
relajado”. Este algoritmo parece gustarles a los estudiantes de bajo rendimiento, quizá por
la estructura reticular. En la figura 3-36 se ilustra el algoritmo de la multiplicación reticu-
lar para multiplicar 14 y 23. (La explicación de por qué funciona la multiplicación reticular
se deja como ejercicio.)

1 4
0
2
8

1
3 Éste es el resultadoisde44. ?3.3. The
Las decenas
tens go van sobre
above the
2 la diagonal
diagonal andy debajo vanbelow.
the units las unidades. Continúa
Continue this proce-
este
dureprocedimiento para todos los cuadros.
for all the blocks.

1 4
1
0 0
2
2 8
Una veztheque se termina laismultiplicación,
multiplication complete, addse sumathe
along a
0 1 lo largo de It
diagonals. lasisdiagonales.
necessary inEsthis
necesario,
exampleentoeste
“carry”
3 3
ejemplo, “reagrupar”
1 to the hundreds un 1 a la diagonal de las centenas.
diagonal.
3 2
2 2

Figura 3-36

◆ Nota La multiplicación reticular se remonta a la India del siglo diez. Europa importó este al-
histórica goritmo y fue popular en los siglos catorce y quince. Los rodillos (o huesos) de Napier,
desarrollados por John Napier a principios de los años 1600, se modelaron mediante la
multiplicación reticular. Los rodillos de Napier se pueden usar en procedimientos de
multiplicación. ◆

© Lopez Mateos Editores. ISBN 978-607-95583-2-1, obra completa, versión electrónica, ISBN 978-607-95583-3-8, volumen 1, versión electrónica. Ejemplar asignado a: Helecto Villarroel gutierrez -
helecto@gmail.com. Fecha: 27 de octubre de 2014. Prohibida su modificación, copia o distribución.
166 Números completos y sus operaciones

Algoritmos de división
Uso de la resta o substracción consecutiva para desarrollar el algoritmo
convencional de la división
En una sección anterior se desarrolló un algoritmo para dividir números completos me-
diante restas consecutivas. Sin embargo, podría hacerse más eficiente. Considera lo si-
guiente:

Una empleada está almacenando botellas de jugo en cajas con capacidad para 6 botellas. Tiene
726 botellas. ¿Cuántas cajas necesita?

Podríamos razonar que si 1 caja contiene 6 botellas, entonces 10 cajas contendrán 60 bote-
llas y 100 cajas contendrán 600 botellas. Si se llenan 100 cajas, sobrarán 726 - 100 # 6, ó
126, botellas. Si se llenan 10 cajas más, entonces sobrarán 126 - 10 # 6, ó 66, botellas. De
manera análoga, si se llenan 10 cajas más, sobrarán 66 - 10 # 6, ó 6, botellas. Finalmente,
las 6 botellas restantes cabrán en 1 caja. El número total de cajas necesarias es de
100 + 10 + 10 + 1, ó 121. Este procedimiento se resume en la figura 3-37(a). En la figura
3-37(b) se muestra un método más eficiente.

(a) 6 冄 726 (b) 6 冄 726


- 600 100 seis - 600 100 seis
126 126
- 60 10 seis - 120 20 seis
66 6
-60 10 seis -6 1 seis
6 0 121 seis
-6 1 seis
◆ Nota de 0 121 seis
investigación Figura 3-37
Muchos estudiantes
nunca llegan a domi- Las divisiones como en la figura 3-38 se presentan de manera más eficiente en los libros
nar el algoritmo de texto de educación básica, como en la figura 3-38(b), donde se omiten los números a co-
convencional de la
lor de la figura 3-38(a) La técnica usada en la figura 3-38(a) se conoce como “andamiaje” y
división larga. Muy
pocos logran tener
una comprensión ra-
(a) 121 (b) 121
zonable ya sea del
algoritmo o de las 1 6 冄 726
respuestas que pro- 20 -6
duce. Una de las ra- 100 12
zones principales
detrás de esta difi- 6 冄 726 - 12
cultad es que en el - 600 6
algoritmo conven- 126 - 6
cional (como se en-
- 120 0
seña usualmente) se
pide que los estu- 6
diantes soslayen la -6
comprensión del va- 0
lor posicional (Sil-
ver et al. 1993). ◆ Figura 3-38

© Lopez Mateos Editores. ISBN 978-607-95583-2-1, obra completa, versión electrónica, ISBN 978-607-95583-3-8, volumen 1, versión electrónica. Ejemplar asignado a: Helecto Villarroel gutierrez -
helecto@gmail.com. Fecha: 27 de octubre de 2014. Prohibida su modificación, copia o distribución.
Sección 3-4 Algoritmos para multiplicar y dividir números completos 167

se puede usar como paso preliminar para llegar al algoritmo convencional, como en la
figura 3-38(b). Nota que el andamiaje toma los números de la derecha de la
figura 3-37(b) y los coloca arriba, como en la figura 3-38(a). Nota asimismo que el an-
damiaje muestra el valor posicional y, como se indica en la Nota de investigación, el valor
posicional es importante para entender el algoritmo convencional.

Uso de cubos de base diez para desarrollar el algoritmo convencional de la división


Como se señala en la Nota de investigación, es necesario que los estudiantes vean por qué
cada paso del algoritmo es adecuado en lugar de simplemente saber cuál es la sucesión de
pasos a seguir. A continuación usamos cubos de base diez para justificar por qué es apro-
piado cada paso del algoritmo convencional. En la tabla 3-4, del lado izquierdo están los
pasos del modelo de base diez y del lado derecho los pasos correspondientes del algo-
ritmo convencional.

◆ Nota de Los estudiantes, al ir dando sentido al procedimiento efectuado en una operación, como la
investigación división larga, necesitan concentrarse en comprender por qué cada paso es apropiado en un
algoritmo, en lugar de qué paso realizar y en qué sucesión. Además, los maestros deben
alentar a los estudiantes a inventar sus propios procedimientos para efectuar operaciones,
pero debe esperarse que expliquen por qué son válidas sus invenciones (Lampert 1992). ◆

Tabla 3-4
Cubos de base diez Algoritmo
1. Primero representamos 726 con cubos de base diez.

6 冄 726

2. A continuación determinamos cuántos conjuntos de 6 losas (centenas) hay en la representación. 1 conjunto de 6 losas
Hay 1 conjunto de 6 losas, sobrando 1 losa, 2 barras (decenas) y 6 unidades. 1
6 冄 726
-6
1
quedan 1 losa
2 barras
6 unidades

3. Ahora convertimos la losa sobrante a 10 barras (decenas). Tenemos así 12 barras (decenas) 1 conjunto de 6 losas
y 6 unidades. 1
6 冄 726
1 losa= 10 barras
-6
12
quedan 12 barras
6 unidades

(continúa)

© Lopez Mateos Editores. ISBN 978-607-95583-2-1, obra completa, versión electrónica, ISBN 978-607-95583-3-8, volumen 1, versión electrónica. Ejemplar asignado a: Helecto Villarroel gutierrez -
helecto@gmail.com. Fecha: 27 de octubre de 2014. Prohibida su modificación, copia o distribución.
168 Números completos y sus operaciones

Cubos de base diez Algoritmo


4. A continuación determinamos cuántos conjuntos de 6 barras (decenas) hay en las 12 barras y 6 1 conjunto de 6 losas
unidades. Tenemos 2 conjuntos de 6 barras, sobrando 6 unidades. 2 conjuntos de 6 barras
12
6 冄 726
- 6
12
- 12
6

quedan 6 unidades

5. Después determinamos cuántos conjuntos de 6 unidades hay en las restantes 6 unidades 1 conjunto de 6 losas
Hay 1 conjunto de 6 unidades y no sobran unidades (el residuo es 0). 2 conjuntos de 6 barras
1 conjunto de 6 unidades
121
6 冄 726
-6
12
- 12
6
-6
0

0 residuo

Así, vemos que en la representación de cubos de base diez del 726 hay 1 grupo de 6 losas
(centenas), 2 grupos de 6 barras (decenas) y 1 grupo de 6 unidades, sin que sobre algo. De
aquí que el cociente sea 121 con residuo 0. Los pasos del algoritmo se muestran al lado de
la manipulación de los cubos de base diez.

División corta
El proceso ilustrado en la figura 3-38(b) se conoce como división “larga”. Cuando el divisor
es un número de un dígito se puede usar otra técnica, llamada división “corta”, donde casi
todo el trabajo se realiza mentalmente. En la figura 3-39 damos un ejemplo de división
corta.

Decidimos dónde Dividimos las Dividimos las de- Dividimos las


comenzar. centenas. Escribi- cenas. Escribimos unidades.
mos el residuo el residuo junto a
junto a las decenas. las unidades.

5 5 57 57 6
5冄 2 8 8 0 5 冄 2 838 0 5 冄 2 83830 5 冄 2 83830

28 , 5 28 , 5 = 5 R3 38 , 5 = 7 R3 30 , 5 = 6 R0

Figura 3-39
© Lopez Mateos Editores. ISBN 978-607-95583-2-1, obra completa, versión electrónica, ISBN 978-607-95583-3-8, volumen 1, versión electrónica. Ejemplar asignado a: Helecto Villarroel gutierrez -
helecto@gmail.com. Fecha: 27 de octubre de 2014. Prohibida su modificación, copia o distribución.
Sección 3-4 Algoritmos para multiplicar y dividir números completos 169

En varios libros de texto de educación básica la división se enseña usando un algoritmo


de cuatro pasos: estimar, multiplicar, restar y comparar. Lo presentamos en la página de
muestra. Nota que los estudiantes verifican la división usando la operación inversa, la muti-
plicación. Estudia la página y responde las preguntas ahí planteadas.

División entre un divisor de dos dígitos


A continuación presentamos una división entre un divisor de más de un dígito. Considera
32 冄 2618.
1. Estima el cociente de 32 冄 2618. Como 1 # 32 = 32, 10 # 32 = 320, 100 # 32 = 3200, ve-
mos que el cociente está entre 10 y 100.
2. Halla el número de decenas en el cociente. Como 26 , 3 es aproximadamente 8,
entonces 26 centenas divididas entre 3 decenas es, aproximadamente, 8 decenas. Es-
cribimos el 8 en el lugar de las decenas, como se muestra:
80
32 冄 2618
- 2560 132 # 802
58
3. Halla el número de unidades en el cociente. Como 5 , 3 es aproximadamente 1, en-
tonces 5 decenas dividido entre 3 decenas es aproximadamente 1. Esto se muestra del
lado izquierdo, con el algoritmo convencional del lado derecho.
81
1
80 81 R26
32 冄 2618 32 冄 2618
- 2560 : - 256
58 58
- 32 132 # 12 - 32
26 26
4. Verifica: 32 # 81 + 26 = 2618.
Normalmente en los libros de primaria vemos el formato mostrado del lado derecho,
que coloca el residuo al lado del cociente.

Multiplicación y división en otras bases


Como sucedió con la suma y la resta, primero necesitamos identificar las multiplicaciones
básicas, o tabla de multiplicar, antes de poder usar algoritmos. En la tabla 3-5 damos las
multiplicaciones básicas en base cinco. Estos resultados se pueden obtener usando la suma
consecutiva.

Tabla 3-5    Base cinco


Tabla de multiplicar
x 0 1 2 3 4
0 0 0 0 0 0
1 0 1 2 3 4
2 0 2 4 11 13
3 0 3 11 14 22
4 0 4 13 22 31

© Lopez Mateos Editores. ISBN 978-607-95583-2-1, obra completa, versión electrónica, ISBN 978-607-95583-3-8, volumen 1, versión electrónica. Ejemplar asignado a: Helecto Villarroel gutierrez -
helecto@gmail.com. Fecha: 27 de octubre de 2014. Prohibida su modificación, copia o distribución.
170 Números completos y sus operaciones

Página de un libro de texto DIVIDIR NÚMEROS DE TRES DÍGITOS

Lección 7-7

Idea clave
Puedes dividir Dividir números de Calentamiento
Estima cada cociente.
números grandes
de la misma tres dígitos
manera que divides
números pequeños.
Aprende
¿Cómo divides números grandes?
Una compañía de autobuses escolares tiene 273 autobuses
y cinco estacionamientos. Si la compañía quiere colocar
el mismo número de autobuses en cada
estacionamiento, ¿cuántos autobuses
deberá colocar en cada estacionamiento?
Como la compañía quiere colocar el mismo número
de autobuses en cada estacionamiento, puedes dividir.

Ejemplo A
ex
ef l i ó Halla 273 ÷ 5.
R

Estima: 273 está cerca de 250 y 250 ÷ 5 es 50, por lo que el


cociente es un poco más que 50.
¡Piensa!
Puedo dividir PASO PASO
Verifica
cuando necesito
hallar cuántos hay
en cada grupo.
Divide las decenas. Baja las unidades Multiplica el cociente
Puedo trazar una y divide. por el divisor y
figura para ilustrar suma el residuo.
la idea principal.

Multiplica.
Resta.
Compara. 2 < 5 Multiplica.
Resta. La respuesta coincide.
El dividendo es 273.
Compara.

Así

La compañía puede colocar 54 autobuses en cada


estacionamiento, y van a sobrar 3 autobuses.

Tema de plática
Colócalo en la RED
¿Por qué, en el ejemplo A, comenzaste Más ejemplos
dividiendo las decenas?

Fuente: Scott Foresman-Addison Wesley Math, Grade 4, 2008 ( p. 386).

© Lopez Mateos Editores. ISBN 978-607-95583-2-1, obra completa, versión electrónica, ISBN 978-607-95583-3-8, volumen 1, versión electrónica. Ejemplar asignado a: Helecto Villarroel gutierrez -
helecto@gmail.com. Fecha: 27 de octubre de 2014. Prohibida su modificación, copia o distribución.
Sección 3-4 Algoritmos para multiplicar y dividir números completos 171

Hay varias maneras de realizar la multiplicación 21cinco # 3cinco:


Cincos Unidades 120 + 12cinco 21cinco 21cinco
2 1 : * 3cinco : * 3cinco : * 3cinco
1110 + 32cinco
f
* 3 :3 : 113cinco
: 110
113cinco
A continuación multiplicamos un número de dos dígitos por un número de dos dígitos:
23cinco 23cinco
* 14cinco 110 + 42cinco *14cinco
22 14 # 32cinco : 202
130 14 # 202cinco : 230
30 110 # 32cinco 432cinco
200 110 # 202cinco
432cinco
También se puede usar la multiplicación reticular para números en otras bases numéricas.
Esto se explora en la Evaluación 3-4.
La división en otras bases se puede efectuar usando la tabla de multiplicar y la definición de
división. Por ejemplo, 22cinco , 3cinco = c si, y sólo si, c # 3cinco = 22cinco. De la tabla 3-5 ve-
mos que c = 4cinco. Como sucede en la base diez, se requiere práctica para efectuar divisiones
multidígitos en otras bases. Las ideas detrás de los algoritmos de la división se pueden desarro-
llar usando la substracción consecutiva. Por ejemplo, en la figura 3-40(a) calculamos
3241cinco , 43cinco por medio de la técnica de la substracción o resta consecutiva, y en la fi-
gura 3-40(b) empleamos el algoritmo convencional. Así, 3241cinco , 43cinco = 34cinco con re-
siduo 14cinco.

43cinco 冄 3241cinco 34cinco R14cinco


(a) - 430 110 # 432cinco (b) 43cinco 冄 3241cinco
2311 - 234
- 430 110 # 432cinco 401
1331 - 332
- 430 110 # 432cinco 14cinco
401
- 141 12 # 432cinco
210
- 141 12 # 432cinco
14 134 # 432cinco

Figura 3-40

En el ejemplo 3-4 presentamos cálculos con la base dos.

Ejemplo 3-4 a. Multiplica: b. Divide:










101dos 101dos 冄 110110dos


* 11dos

© Lopez Mateos Editores. ISBN 978-607-95583-2-1, obra completa, versión electrónica, ISBN 978-607-95583-3-8, volumen 1, versión electrónica. Ejemplar asignado a: Helecto Villarroel gutierrez -
helecto@gmail.com. Fecha: 27 de octubre de 2014. Prohibida su modificación, copia o distribución.
172 Números completos y sus operaciones

Solución a. 101dos b. 1010dos R100dos


* 11dos 101dos 冄 110110dos
101 - 101
101 111
1111dos - 101
100 dos

Evaluación 3-4A

1. Halla, en cada caso, los números faltantes: 7. Considera lo siguiente:


a. 4_6 b. 327
* 9_1 476
* 783
12 # 4762
327 * 293
1_78
19 # 4762
1_08 952
3408
13 # 4762
_9_3 4284
_982
30_ _07 1428
3335_8
139468
2. Efectúa las siguientes multiplicaciones usando el algo-
ritmo de la multiplicación reticular: a. Usa el algoritmo convencional para mostrar que la
a. 728 b. 306 respuesta es correcta.
* 94 * 24 b. Explica por qué funciona el algoritmo.
3. Explica por qué funciona el algoritmo de la multiplica- c. Usa el método para multiplicar 84 * 363.
ción reticular. 8. Presentamos el algoritmo del campesino ruso para multiplicar
4. Simplifica cada caso usando propiedades de los exponen- 27 * 68. (No consideres los residuos cuando saques la mitad.)
tes. Da la respuesta en potencias.
a. 57 # 512 b. 610 # 62 # 63 Mitades Dobles
296 #
c. 10 1017
d. 27 # 105 # 57
5. a. ¿Cuál es mayor, 280 + 280 ó 2100? ¿Por qué? : 27 * 68
b. ¿Cuál es mayor, 2101, 3 # 2100 ó 2102? ¿Por qué? Mitad de 27 : 13 136 es el doble de 68
6. El modelo siguiente ilustra 22 # 13: Mitad de 13 : 6 272 es el doble de 136
Mitad de 6 : 3 544 es el doble de 272
20 2
Mitad de 3 : 1 1088 es el doble de 544

En la columna de las “Mitades” escoge los números im-


10 pares. En la columna de los “Dobles” señala los núme-
ros correspondientes a los impares de la columna de las
“Mitades”. Suma los números señalados.

3 68
136
a. Explica dónde se muestran, en la figura, los produc- 544
tos parciales. 1088
b. Dibuja un modelo similar para 15 # 21. 1836 Éste es el producto de 27 # 68.
c. Dibuja un modelo similar en base cinco para el pro-
Usa este algoritmo para 17 # 63 y para otros números.
ducto 43cinco # 23cinco. Explica cómo puede usarse el
9. Responde las preguntas siguientes con base en la tabla de
modelo para encontrar la respuesta en base cinco.
actividades dada a continuación:

© Lopez Mateos Editores. ISBN 978-607-95583-2-1, obra completa, versión electrónica, ISBN 978-607-95583-3-8, volumen 1, versión electrónica. Ejemplar asignado a: Helecto Villarroel gutierrez -
helecto@gmail.com. Fecha: 27 de octubre de 2014. Prohibida su modificación, copia o distribución.
Sección 3-4 Algoritmos para multiplicar y dividir números completos 173

Actividad Calorias quemadas por hora núan su lectura a la misma velocidad, ¿cuánto más tar-
dará Carla que Mónica en terminar el libro?
Tenis 462 16. Daniel tiene 4520 monedas de un peso en tres cajas. Dice
Caminata sobre nieve 708 que hay 3 veces más monedas en la primera que en la ter-
Esquí a campo traviesa 444 cera, y el doble de monedas en la segunda caja que en la
Voleibol 198 primera. ¿Cuántas monedas tiene en cada caja?
17. Gilda compra manzanas en una huerta y las vende en
a. ¿Cuántas calorías se queman durante 3 h de esquiar a
una feria, en bolsas de 3, a $10 la bolsa. Ella compró 50
campo traviesa?
cajas de manzanas, con 36 manzanas por caja, y pagó
b. Juana jugó tenis durante 2 h mientras que Carolina
$4520. Si vendió todas las manzanas, excepto 18, ¿cuál
jugó voleibol durante 3 h. ¿Quién quemó más calo-
fue su ganancia?
rías, y cuántas más?
18. Analiza un posible patrón de error en cada caso:
c. Lalo fue a caminar sobre nieve durante 3 h y Mauri-
cio fue a esquiar a campo traviesa durante 5 h. ¿Quién a. b.
35 53
quemó más calorías, y cuántas más? * 26 5 冄 2515
10. Durante unas vacaciones de 14 días, Gerardo incrementó 90 - 25
su consumo de calorías en 1500 calorías diarias. Pero tam-
15
bién hizo más ejercicio que el habitual, nadando 2 h dia-
- 15
rias. Nadar quema 666 calorías por hora, y un aumento
0
neto de 3500 calorías añade 1 lb de peso. ¿Aumentó Ge-
rardo al menos 1 lb durante sus vacaciones? 19. a. Justifica los pasos siguientes:
11. Efectúa cada una de las divisiones siguientes usando el 56 # 10 = 15 # 10 + 62 # 10
algoritmo de la resta consecutiva y el convencional. = 15 # 102 # 10 + 6 # 10
a. 8 冄 623 b. 36 冄 298 = 5 # 110 # 102 + 6 # 10
c. 391 冄 4001 = 5 # 102 + 6 # 10
12. En una calculadora, Rafa multiplicó por 10 cuando de- = 5 # 102 + 6 # 10 + 0 # 1
bió haber dividido entre 10. En la pantalla se lee 300. = 560
¿Cuál es la respuesta correcta?
13. En la figura siguiente se muestran cuatro máquinas-fun- b. Justifica cada paso al multiplicar 34 # 102.
ción. La salida de una es la entrada de la que sigue. Com- 20. Para transportar a 1672 estudiantes a una reunión con el
pleta la tabla. gobernador, la escuela planea rentar autobuses con un
Entrada
cupo de 29 estudiantes cada uno. ¿Cuántos autobuses se
Entrada Salida
necesitan? ¿Todos irán llenos?
2 11 21. Coloca los dígitos 7, 6, 8 y 3 en las cajas siguientes para
+5
4 obtener
nnn
0
×4 * n
19
a. el mayor producto.
31 b. el menor producto.
–6
22. ¿Para qué bases posibles son correctos estos cálculos ?
a. 213 b. 213
÷2 + 308 * 32
Salida
522 430
14. Considera las multiplicaciones siguientes. Nota que 1043
cuando se invierten los dígitos en los factores, los produc- 11300
tos son iguales. 23. a. Usa la multiplicación reticular para calcular
36 63 323cinco # 42cinco.
* 42 * 24 b. Halla los valores más pequeños de a y b tales que
1512 1512 32a = 23b.
24. Efectúa cada una de estas operaciones usando las bases
a. Halla otras multiplicaciones donde funcione el mismo mostradas:
procedimiento. a. 32cinco # 4cinco
b. Halla un patrón para los números que se comportan b. 32cinco , 4cinco
de esa manera. c. 43seis # 23seis
15. Mónica leyó 160 páginas de su libro en 4 h. Su hermana d. 143cinco , 3cinco
Carla empleó 4 h en leer 100 páginas del mismo libro. Si e. 10010dos , 11dos
el libro tiene 200 páginas, y si las dos muchachas conti- f. 10110dos # 101dos

© Lopez Mateos Editores. ISBN 978-607-95583-2-1, obra completa, versión electrónica, ISBN 978-607-95583-3-8, volumen 1, versión electrónica. Ejemplar asignado a: Helecto Villarroel gutierrez -
helecto@gmail.com. Fecha: 27 de octubre de 2014. Prohibida su modificación, copia o distribución.
174 Números completos y sus operaciones

Evaluación 3-4B

1. Halla, en cada caso, los números faltantes: a. Explica dónde se muestran, en la figura, los produc-
tos parciales.
b. Dibuja un modelo similar para 12 # 22.
4_4
7. a. Usa cubos de base cinco para calcular 14cinco # 23cinco.
* 327
3_8 8 b. Usa la propiedad distributiva de la multiplicación so-
968 bre la suma para explicar por qué la multiplicación de
_4 5 2 un número natural en base cinco por 10cinco resulta
1 5 8 2_8 en añadir un 0 al número.
2. Efectúa las siguientes multiplicaciones usando el algo- c. Explica por qué la multiplicación de un número na-
ritmo de la multiplicación reticular: tural en base cinco por 100cinco resulta en añadir dos
a. 327 b. 2618 0 al número.
* 43 * 137 d. Usa la propiedad distributiva de la multiplicación sobre
3. La tabla siguiente nos muestra el consumo promedio de la suma, y la parte (b), para calcular 14cinco # 23cinco.
agua de 1 persona en un día: 8. Completa la tabla siguiente:

Uso Cantidad promedio


a b a#b aⴙb
Baño de tina 110 L (litros)
56 3752
Ducha 75 L
Inodoro 22 L
32 110
Lavar manos, cara 7L 270 33
Beber 1L
Lavar dientes 1L 9. Susana compró una póliza de seguro de vida por
Lavar trastes (una comida) 30 L $300,000 al precio de $240 por cada $10,000 de cober-
Cocinar (una comida) 18 L tura. Si paga la prima en 12 mensualidades, ¿de cuánto
es cada mensualidad?
a. Usa la tabla para calcular cuánta agua consumes dia- 10. Efectúa cada una de las divisiones siguientes usando el
riamente. algoritmo de la resta consecutiva y el convencional:
b. El estadounidense promedio gasta 200 L de agua al a. 7冄 392
día. ¿Cuál es tu promedio? b. 37 冄 925
c. Si hay 100,000,000 de personas en México, ¿cuánta c. 423 冄 5002
agua se usa en promedio, al día, en el país? 11. Coloca en las cajas n 冄 n n n los dígitos 7, 6, 8 y 3, de
4. Simplifica cada caso usando propiedades de los exponen- modo que se obtenga
tes. Da la respuesta en potencias.
a. el mayor cociente.
a. 38 # 34 b. 52 # 54 # 52
b. el menor cociente.
c. 62 # 22 # 32
12. Veinte miembros de una banda musical planean asistir a
5. a. ¿Cuál es mayor, 220 + 220 ó 221? ¿Por qué?
un festival. Ellos lavaron 245 carros a $2 por carro, como
b. ¿Cuál es mayor, 331, 9 # 330 ó 333? ¿Por qué?
ayuda para sus gastos. La escuela contribuirá a los gastos
6. El modelo siguiente ilustra 13 # 12:
aportando $1 por cada $1 que colecte la banda. El costo
de rentar un autobús para transportar a la banda es de
10 3 72¢ por milla y el viaje redondo es de 350 mi. Los miem-
bros de la banda permanecerán en un dormitorio du-
rante 2 noches a $5 por persona por noche. Las comidas
costarán $28 por persona. ¿Ha colectado la banda sufi-
10 ciente dinero? De no ser así, ¿cuántos carros más deben
lavar?
13. En la figura siguiente se muestran tres máquinas-fun-
ción. La salida de una es la entrada de la que sigue. Com-
2 pleta la tabla.

© Lopez Mateos Editores. ISBN 978-607-95583-2-1, obra completa, versión electrónica, ISBN 978-607-95583-3-8, volumen 1, versión electrónica. Ejemplar asignado a: Helecto Villarroel gutierrez -
helecto@gmail.com. Fecha: 27 de octubre de 2014. Prohibida su modificación, copia o distribución.
Sección 3-4 Algoritmos para multiplicar y dividir números completos 175

Entrada Entrada Salida 19. a. Halla todos los números completos que dejan residuo
1 al dividirlos entre 4. Escribe tu respuesta usando la
2 22
+7 notación constructora de conjuntos.
4 b. Escribe los números de la parte (a) en una sucesión
0
que comience por el más pequeño.
×3 c. ¿Qué tipo de sucesión es la de la parte (b)?
34 20. Efectúa cada una de estas operaciones usando las bases
31 mostradas:
–5
a. 42cinco # 3cinco
Salida b. 22cinco , 4cinco
14. Escoge tres dígitos diferentes. c. 32cinco # 42cinco
a. Crea todos los números menores que 100 que sean d. 1313cinco , 23cinco
posibles con los dígitos escogidos. En cada número e. 101dos # 101dos
sólo se puede usar una vez cada dígito. f. 1001dos , 11dos
21. ¿Para qué bases posibles sucede que estos cálculos son
b. Suma los seis números.
correctos?
c. Suma los tres dígitos que escogiste.
a. 322 b. 101
d. Divide la respuesta en (b) entre la respuesta en (c).
- 233 11 冄 1111
e. Repite de (a) a (d) con otros tres números.
23 - 11
f. ¿Es siempre igual el resultado? ¿Por qué?
11
15. Juan ahorró $5340 en 3 años. Si ahorró $95 al mes en el
- 11
primer año y una cantidad fija por mes durante los 2
0
años siguientes, ¿cuánto ahorró mensualmente durante 22. a. Usa la multiplicación reticular para calcular
los últimos 2 años? 423cinco # 23cinco.
16. Un grupo de niñas de cuarto grado tenía que cortar cua- b. Halla los valores más pequeños de a y b tales que
tro piezas de listón de 4 pies de longitud, de un rollo de 41a = 14b.
44 yardas. ¿Cuál es la longitud del listón restante? (Una 23. Coloca los dígitos 7, 6, 8, 3 y 2 en las cajas siguientes
yarda tiene tres pies.) para obtener
17. Analiza un posible patrón de error en cada caso: nnn
a. 34 b. 34 * nn
* 8 *6
2432 114 a. el mayor producto. b. el menor producto.
18. Justifica los pasos siguientes: 24. Halla los productos siguientes y describe el patrón que
surja:
35 # 100 = 13 # 10 + 52100 a. 1 * 1 b. 99 * 99
= 13 # 10 + 52102 11 * 11 999 * 999
= 13 # 102102 + 5 # 102 111 * 111 9999 * 9999
= 3110 # 1022 + 5 # 102 1111 * 1111
= 3 # 103 + 5 # 102 c. Pon a prueba los patrones descubiertos. Si los patro-
= 3 # 103 + 5 # 102 + 0 # 10 + 0 # 1 nes no continúan como se esperaba, determina
= 3500 cuándo se detienen.

Conexiones matemáticas 3-4

Comunicación 4. ¿Piensas que conviene a los estudiantes conocer más de un


1. ¿Cómo explicarías a niñas y niños la manera de multiplicar método para resolver problemas de cálculo? ¿por qué?
345 # 678, suponiendo que saben y entienden la multipli- 5. Escoge el que consideres el “mejor” algoritmo estudiado
cación por un sólo dígito y la multiplicación por una po- en esta sección. Explica el razonamiento que funda-
tencia de 10? menta tu selección.
2. ¿Qué sucede cuando multiplicas cualquier número de 6. Tomás asegura que no debe prestarse mucha atención a
dos dígitos por 101? Explica por qué. la división larga en grupos elementales. ¿Estás de
3. Escoge un número. Dóblalo. Multiplica el resultado por 3. acuerdo con él, o no? Defiende tu respuesta.
Súmale 24. Divídelo entre 6. Réstale el número original. 7. Prueba que todos los números de la forma abba (a y b son
¿Es el resultado siempre el mismo? Escribe un argumento dígitos en base diez) dejan residuo 0 al dividirlos entre
convincente para tu respuesta.

© Lopez Mateos Editores. ISBN 978-607-95583-2-1, obra completa, versión electrónica, ISBN 978-607-95583-3-8, volumen 1, versión electrónica. Ejemplar asignado a: Helecto Villarroel gutierrez -
helecto@gmail.com. Fecha: 27 de octubre de 2014. Prohibida su modificación, copia o distribución.
176 Números completos y sus operaciones

11. ¿Sucede lo mismo para todos los números de la b. Otro estudiante dice que él ya sabe que 39 # 41 Z
forma abccba? ¿por qué? 40 # 40 pero encontró que 39 # 41 = 40 # 40 - 1.
También halló que 49 # 51 = 50 # 50 - 1. Desea sa-
Solución abierta
ber si el patrón continúa. ¿Cómo le respondes?
8. Si una estudiante presenta un nuevo “algoritmo” para
calcular con números completos, describe el proceso que Problemas de repaso
le recomendarías para determinar si su algoritmo fun- 16. Ilustra la propiedad de identidad de la suma para núme-
ciona en todos los casos. ros completos.
17. Reescribe cada caso usando la propiedad distributiva de
Aprendizaje colectivo
la multiplicación sobre la suma:
9. La secuencia tradicional para la enseñanza de las opera- a. ax + bx + 2x
ciones en la escuela elemental es ver primero la suma, b. 31a + b2 + x1a + b2
después la resta o substracción, luego la multiplicación y, 18. Al comienzo de un viaje el odómetro marcó 52,281. Al
finalmente, la división. Algunos educadores abogan por final del viaje el odómetro marcó 59,260. ¿Cuántas mi-
enseñar la suma seguida de la multiplicación y después la llas se recorrieron?
resta seguida de la división. Con tu grupo, preparen ar- 19. Escribe cada uno de los problemas de división como un
gumentos a favor de enseñar las operaciones en cada uno problema de multiplicación:
de los órdenes mencionados. a. 36 , 4 = 9
Preguntas del salón de clase b. 112 , 2 = x
c. 48 , x = 6
10. Una estudiante divide como se muestra. ¿Cómo la ayu-
d. x , 7 = 17
darías?
Preguntas del Third International Mathematics and
4 5
Science Study (TIMSS) (Tercer Estudio Internacional
3 冄 1215 sobre Matemáticas y Ciencia)
- 12
Cada estudiante necesita 8 cuadernos para la escuela.
15
¿Cuántos cuadernos se necesitan para 115 estudiantes?
- 15
0 Usa los cuadros 1 , 4 y 5 . Escribe los números de los
cuadros en las cajas para obtener la mayor respuesta al
11. Un estudiante divide como se muestra. ¿Cómo lo ayuda-
multiplicar.
rías?
nn
15
6 冄 36 * n
-6
37 * . = 703.
30 ¿Cuál es el valor de 37 * . + 6?
30
TIMSS, 2003, Grado 4
12. Un estudiante pregunta cómo puede hallar el cociente y
el residuo en un problema de división como 592 , 36 Pregunta del National Assessment of Educational Progress
usando una calculadora sin tecla de división entera. (NAEP) (Evaluación Nacional del Progreso Educativo)
13. Una estudiante asegura que para dividir entre 10 un nú- Asistirán 58 personas a un desayuno y cada persona co-
mero con 0 en el dígito de las unidades basta quitar el 0 merá 2 huevos. Hay 12 huevos en cada cartón. ¿Cuántos
para obtener la respuesta. Ella quiere saber si esto siem- cartones de huevo se necesitarán para el desayuno?
pre es cierto y por qué, y si el 0 debe estar en el dígito de a. 9
las unidades. ¿Cómo respondes? b. 10
14. Una estudiante asegura que si el residuo cuando m se di- c. 72
vide entre n es 0, entonces el dividendo (m) y el divisor d. 116
(n) se pueden multiplicar, cada uno, por el mismo nú- NAEP 2007, Grado 4
mero completo distinto de cero c y la respuesta a la divi-
sión será la misma. Esto es, m , n = 1mc2 , 1nc2. Ella
quiere saber por qué. ¿Cómo responderías, suponiendo
que la estudiante no sabe nada acerca de fracciones?
15. a. Un estudiante pregunta que si 39 + 41 = 40 + 40,
entonces es cierto que 39 # 41 = 40 # 40. ¿Cómo res-
pondes?

© Lopez Mateos Editores. ISBN 978-607-95583-2-1, obra completa, versión electrónica, ISBN 978-607-95583-3-8, volumen 1, versión electrónica. Ejemplar asignado a: Helecto Villarroel gutierrez -
helecto@gmail.com. Fecha: 27 de octubre de 2014. Prohibida su modificación, copia o distribución.
Sección 3-4 Algoritmos para multiplicar y dividir números completos 177

ROMPECABEZAS  En cada caso, reemplaza las letras con dígitos de manera que el cálculo sea correcto.
Cada letra puede representar sólo un dígito.

a. DANZON b. MA
* S MA
TORNEON + MA
EE L

ACTIVIDAD DE LABORATORIO
1. Se han codificado mensajes en cinta de papel, en base dos. Un agujero en la cinta representa 1, mientras
que un espacio representa 0. El valor de cada agujero depende de su posición, de izquierda a derecha, 16,
8, 4, 2, 1 (todas potencias de 2). Las letras del alfabeto se pueden codificar en base dos de acuerdo con su
posición en el alfabeto. Por ejemplo, Ñ es la letra número quince. Como 15 = 1 # 8 + 1 # 4 + 1 # 2 + 1,
los agujeros aparecen como en la 3-41:

16 8 4 2 1

Figura 3-41
a. Descodifica el mensaje de la figura 3-42.

Figura 3-42

b. Escribe tu nombre en una cinta usando base dos.


2. El juego numérico de la figura 3-43 usa aritmética de base dos.
Carta E Carta D Carta C Carta B Carta A

16 24 8 24 4 20 2 18 1 17
17 25 9 25 5 21 3 19 3 19
18 26 10 26 6 22 6 22 5 21
19 27 11 27 7 23 7 23 7 23
20 28 12 28 12 28 10 26 9 25
21 29 13 29 13 29 11 27 11 27
22 30 14 30 14 30 14 30 13 29
23 31 15 31 15 31 15 31 15 31

Figura 3-43
a. Supón que la edad de una persona aparece en las cartas E, C y B, y que la persona tiene 22 años.
¿Puedes descubrir cómo y por qué funciona?
b. Diseña la carta F de modo que los números 1 a 63 puedan usarse en el juego. Nota que las cartas A a
E también deben modificarse.

© Lopez Mateos Editores. ISBN 978-607-95583-2-1, obra completa, versión electrónica, ISBN 978-607-95583-3-8, volumen 1, versión electrónica. Ejemplar asignado a: Helecto Villarroel gutierrez -
helecto@gmail.com. Fecha: 27 de octubre de 2014. Prohibida su modificación, copia o distribución.
178 Números completos y sus operaciones

3-5 Matemática mental y estimación para


operaciones entre números completos

En los Principios y objetivos hallamos lo siguiente:

Parte de la capacidad de calcular con fluidez consiste en realizar selecciones sensatas acerca de
qué herramientas usar, y cuándo. Los estudiantes deben tener la experiencia suficiente para saber
escoger entre cálculo mental, alguna estrategia con papel y lápiz, y la estimación o uso de calcula-
dora. El contexto particular, la pregunta y los números utilizados, todo ello juega su papel en dicha
selección. ¿Los números permiten una estrategia mental? ¿El contexto nos pide un estimado? ¿El
problema requiere cálculos tediosos y repetidos? Los estudiantes, aprovechando su sentido nu-
mérico, deberán evaluar el planteamiento del problema para determinar si se necesita un esti-
mado o una respuesta exacta, y ser capaces de justificar su decisión. (p. 36)

Además, en los Puntos focales hallamos las siguientes afirmaciones respecto a la estimación
en varios grados escolares. Nota que conforme se avanza de grado se incluyen operaciones
adicionales hasta cubrir las cuatro operaciones.
En los Puntos focales para el grado 2:
Seleccionan (los estudiantes) y aplican métodos apropiados para estimar sumas y restas o para cal-
cularlas mentalmente, dependiendo del contexto y de los números empleados. (p. 14)
En los Puntos focales para el grado 4:
◆ Nota de Seleccionan (los estudiantes) métodos apropiados y los aplican de manera precisa para estimar
investigación productos o calcularlos mentalmente, dependiendo del contexto y de los números empleados.
Las personas que son (p. 16)
hábiles para estimar
suelen tener fuerte En los Puntos focales para el grado 5:
autoestima en lo que Seleccionan (los estudiantes) métodos apropiados y los aplican de manera precisa para estimar co-
respecta a los concep-
cientes o calcularlos mentalmente, dependiendo del contexto y de los números empleados. (p. 17)
tos relacionados con
las matemáticas, atri- En las secciones anteriores del presente capítulo nos concentramos en estrategias de papel
buyen su éxito más y lápiz. A continuación estudiaremos otras dos herramientas, a saber, matemática mental y
bien a sus habilidades estimación de cálculo. Matemática mental es el procedimiento de producir una respuesta a
que a sus esfuerzos, y un cálculo sin usar instrumentos computacionales. Estimación de cálculo es el procedi-
piensan que la capaci- miento de conformar una respuesta aproximada a un problema numérico. Tener facilidad de
dad de estimar es una
herramienta impor-
tante. Por el contra- y por
rio, quienes estiman
mal suelen tener una TU MAESTRA NO PUEDES
BIEN CALVIN, VAMOS A ¿MÁS TIEM- ESCRIBISTE QUE ¡CLARO
baja autoestima res- REVISAR TU TAREA DE DICE QUE LE PO? ¡YA LE 8 + 4 = 7. BUENO, SUMAR COSAS QUE PUEDO!
MATE. DEBES DEDIQUÉ TÚ SABES QUE FALLÉ POR Y TERMINAR ¡ESTAMOS
pecto a los conceptos MEJOR
NO, Y DEDICAR DIEZ MINU- ESO NO ES CON MENOS DE EN UN PAÍS
relacionados con las MÁS TIEMPO. TOS! ¡DIEZ UN POCO.
DIGAMOS CIERTO. LO QUE TENÍAS. LIBRE !
QUE YA LO SIÉNTATE. MINUTOS DEMÁN- ¡TENGO MIS
matemáticas, atribu- HICIMOS. PERDIDOS! DAME. DERECHOS!
yen el éxito ajeno al ¡TIRADOS
AL CAÑO!
esfuerzo y creen que
la capacidad de esti-
mar no es importante
ni útil ( J. Sowder
1989). ◆

© Lopez Mateos Editores. ISBN 978-607-95583-2-1, obra completa, versión electrónica, ISBN 978-607-95583-3-8, volumen 1, versión electrónica. Ejemplar asignado a: Helecto Villarroel gutierrez -
helecto@gmail.com. Fecha: 27 de octubre de 2014. Prohibida su modificación, copia o distribución.
Sección 3-5 Matemática mental y estimación para operaciones entre números completos 179

◆ Nota de manejar las estrategias de estimación ayuda a determinar si una respuesta es o no razonable.
En la tira cómica “Calvin y Hobbes” vemos que Calvin es malo estimando y bien podría
investigación creer, como se menciona en la Nota de investigación de la página 178, que la estimación no es
El cálculo mental es importante ni útil.
eficiente cuando em- Tener capacidad para la matemática mental es útil en el desarrollo de las habilidades re-
plea algoritmos dife- queridas día con día para realizar estimaciones. Es indispensable contar con dichas habili-
rentes de los dades, incluso cuando dispongas de una calculadora. Debes poder juzgar si la respuesta
algoritmos conven- obtenida con la calculadora es razonable. La matemática mental recurre a diversas estrate-
cionales de papel y gias y propiedades. Como se menciona en la Nota de investigación de la izquierda, el cálculo
lápiz. Además, las mental se vuelve eficiente cuando emplea algoritmos diferentes de los algoritmos conven-
estrategias de cionales de papel y lápiz. A continuación consideramos varias de las estrategias más comu-
cálculo mental son nes para efectuar mentalmente operaciones con números completos. Nota que el algoritmo
personales, de intercambio no es más que el algoritmo de sumas iguales que ya estudiamos.
dependen de la crea-
tividad, flexibilidad y
comprensión de los
Matemática mental: Suma
conceptos y propie- 1. Sumar desde la izquierda
dades numéricos.
a. 67 60 + 30 = 90 (Suma las decenas.)
Por ejemplo,
considera las habili- + 36 7 + 6 = 13 (Suma las unidades.)
dades y razonamien- 90 + 13 = 103 (Suma las dos sumas.)
to implícitos en el b. 36 30 + 30 = 60 (Duplica 30.)
calculo de la suma
+ 36 6 + 6 = 12 (Duplica 6.)
74 + 29 represen-
tando mentalmente 60 + 12 = 72 (Suma los dobles.)
el problema como
70 + 129 + 12 + 3
2. Separar y juntar
= 103 67 67 + 30 = 97 (Suma el primer número a las decenas del segundo número.)
( J. Sowder + 36 97 + 6 = 103 (Suma este número a las unidades del segundo número.)
1989). ◆
3. Intercambiar
a. 67 67 + 3 = 70 (Suma 3 para completar un múltiplo de 10.)
+ 36 36 - 3 = 33 (Resta 3 para compensar el 3 que sumaste.)
70 + 33 = 103 (Suma los dos números.)

b. 67 67 + 30 = 97 (Suma 30 (el siguiente múltiplo de 10 mayor que 29).)


+ 29 97 - 1 = 96 (Resta 1 para compensar el 1 adicional que sumaste.)

4. Usar números compatibles


Los números compatibles son aquellos cuya suma es fácil de calcular mentalmente.
130 130 + 70 = 200
50 200 50 + 50 = 100
70 100 + 200 = 300
20 300 + 20 = 320
100
+ 50

5. Completar para tener números compatibles


25 25 + 75 = 100 (25 + 75 da 100.)
+ 79 100 + 4 = 104 (Suma otras 4 unidades.)

© Lopez Mateos Editores. ISBN 978-607-95583-2-1, obra completa, versión electrónica, ISBN 978-607-95583-3-8, volumen 1, versión electrónica. Ejemplar asignado a: Helecto Villarroel gutierrez -
helecto@gmail.com. Fecha: 27 de octubre de 2014. Prohibida su modificación, copia o distribución.
180 Números completos y sus operaciones

Matemática mental: Resta


1. Separar y juntar
67 67 - 30 = 37 (Resta del primer número las decenas del segundo.)
-36 37 - 6 = 31 (Resta de la diferencia las unidades del segundo.)

2. Intercambiar
71 71 + 1 = 72; 39 + 1 = 40 (Suma 1 a ambos números. Efectúa la
-39 72 - 40 = 32 substracción, la cual es más fácil que
el problema original.)

Nota que sumar 1 a ambos números no altera la respuesta. ¿Por qué?


3. Quitar los ceros
8700 87 - 5 = 82 (Nota que hay dos ceros en cada número. Quita los ceros y
- 500 82 : 8200 realiza el cálculo. Después coloca de nuevo los dos ceros para
obtener el valor posicional adecuado.)

Otra técnica de matemática mental para la resta se llama “sumando”. Este método se
basa en el enfoque del sumando faltante y se conoce como “algoritmo del cajero”. A conti-
nuación damos un ejemplo del sumando o algoritmo del cajero.

Ejemplo 3-5 Noé debía $11 en la tienda. Usó un billete de $50 para pagar. Mientras le daba el cambio a








Noé, el cajero iba diciendo, “$11, $12, $13, $14, $15, $20, $30, $50”. ¿Cuánto cambio reci-
bió Noé?

Solución La tabla 3-6 muestra lo que iba diciendo el cajero y el dinero que iba recibiendo
Noé. Como $11 más $1 es $12, Noé debió recibir $1 cuando el cajero dijo $12. Se sigue el
mismo razonamiento para $13, $14 y lo que sigue. Así, la cantidad total de cambio que re-
cibió Noé está dada por $1 + $1 + $1 + $1 + $5 + $10 + $20 = $39. En otras pala-
bras, $50 - $11 = $39 pues $39 + $11 = $50.

Tabla 3-6   
El cajero dijo $11 $12 $13 $14 $15 $20 $30 $50
Cantidad recibida por
Noé cada vez 0 $1 $1 $1 $1 $5 $10 $20

AHORA INTENTA ÉSTE 3-17 Efectúa mentalmente cada una de las operaciones siguientes y explica
qué técnica usaste para hallar la respuesta:
a. 40 + 160 + 29 + 31
b. 3679 - 474
c. 75 + 28
d. 2500 - 700

© Lopez Mateos Editores. ISBN 978-607-95583-2-1, obra completa, versión electrónica, ISBN 978-607-95583-3-8, volumen 1, versión electrónica. Ejemplar asignado a: Helecto Villarroel gutierrez -
helecto@gmail.com. Fecha: 27 de octubre de 2014. Prohibida su modificación, copia o distribución.
Sección 3-5 Matemática mental y estimación para operaciones entre números completos 181

Matemática mental: Multiplicación


Así como sucede con la suma y la resta, la matemática mental es útil para multiplicar. Por
ejemplo, considera 8 * 26. Los estudiantes pueden pensar este cálculo de diferentes mane-
ras, como vemos a continuación.
26 = 20 + 6 26 = 25 + 1 26 = 30 - 4
8 * 20 es 160 y 8 * 25 es 200, entonces 8 * 30 es 240, luego
8 * 6 es 48, por tanto 8 * 1 es 8 más, así que quitas 8 * 4 = 32,
8 * 26 es 160 + 48, 8 * 26 es 200 + 8, por lo que 8 * 26 es
ó 208. ó 208. 240 - 32 = 208.
Ahora consideramos varias de las estrategias más comunes para realizar matemática men-
tal usando la multiplicación.
1. Multiplicar desde la izquierda
64 60 * 5 = 300 (Multiplica por 5 el número de las decenas del primer número.)
*5 4 * 5 = 20 (Multiplica por 5 el número de las unidades del primer número.)
300 + 20 = 320 (Suma los dos productos.)

2. Usar números compatibles


2 * 9 * 5 * 20 * 5 Rearréglalo como 9 * 12 * 52 * 120 * 52 =
9 * 10 * 100 = 9000.
3. Pensar en monedas
a. 64 Piensa el producto como 64 monedas de 5, que se pueden
* 5 pensar como 32 monedas de 10, lo cual da 32 * 10 = 320.
b. 64 Piénsalo como el producto de 64 monedas de 50, que son
*50 32 de 100, ó 3200.
c. 64 Piénsalo como el producto de 64 monedas de 25, que son 32
*25 de 50, ó 16 de 100, es decir, 1600.

Matemática mental: División


1. Partir el dividendo
7 冄 4256 7 冄 42 ƒ 56 (Partir el dividendo.)
600 + 8
7 冄 4200 + 56 (Dividir ambas partes entre 7.)
600 + 8 = 608 (Sumar las respuestas.)
2. Usar números compatibles
a. 3 冄 105 105 = 90 + 15 (Busca números que reconozcas como divisibles
30 + 5 entre 3 y cuya suma sea 105.)
= 35
3 冄 90 + 15 (Divide ambas partes y suma las respuestas.)
b. 8 冄 232 232 = 240 - 8 (Busca números que sean fácilmente divisibles
30 - 1 entre 8 y cuya diferencia sea 232.)
= 29
8 冄 240 - 8
(Divide ambas partes y toma la diferencia.)

© Lopez Mateos Editores. ISBN 978-607-95583-2-1, obra completa, versión electrónica, ISBN 978-607-95583-3-8, volumen 1, versión electrónica. Ejemplar asignado a: Helecto Villarroel gutierrez -
helecto@gmail.com. Fecha: 27 de octubre de 2014. Prohibida su modificación, copia o distribución.
182 Números completos y sus operaciones

AHORA INTENTA ÉSTE 3-18 Efectúa mentalmente los cálculos siguientes y explica la técnica que
usaste para obtener la respuesta:
a. 25 # 32 # 4 b. 123 # 3
c. 25 # 35 d. 5075 , 25

Estimación de cálculo
La estimación de un cálculo puede ayudar a determinar si una respuesta es razonable o no.
Esto es especialmente útil cuando el cálculo se efectúa mediante una calculadora. Damos a
continuación algunas de las estrategias comunes para realizar estimaciones.
1. Desde la izquierda
La estimación desde la izquierda comienza considerando los primeros dígitos del frente
o lado izquierdo del número. Se suman estos dígitos de la izquierda y se les asigna un
valor posicional adecuado. Hasta aquí hemos estimado por defecto (desde abajo); se re-
quiere un ajuste. Ajustamos considerando el siguiente grupo de dígitos. El ejemplo
siguiente muestra cómo funciona la estimación desde la izquierda:
423 20 Pasos: (1.) Suma los dígitos de la
4 + 3 + 5
338 120 izquierda 4 + 3 + 5 = 12.
12 centenas 100
+ 561 (2.) Valor posicional = 1200.
(3.) Ajusta 61 + 38 L 100 y
20 + 100 es 120.
(4.) El estimado ajustado es
1200 + 120 = 1320.
2. Completar números “agradables”
La estrategia usada para obtener el ajuste en el ejemplo anterior es la estrategia de
completar números agradables, lo cual significa juntar números “fáciles”. Damos otro
ejemplo.
23
39
Alrededor de 100 32 Alrededor de 100 Por lo tanto, la suma es alrede-
64 dor de 100 + 100, ó 200.
+ 49

3. Cúmulos
Los cúmulos se usan cuando un grupo de números se acumulan alrededor de un valor
común. Esta estrategia se limita a ciertos tipos de cálculos. En el ejemplo siguiente los
números parecen acumularse alrededor de 6000.
6200 Estima el “promedio”—alrededor de 6000
5842
Multiplica el promedio por el número de valores
6512 #
para obtener 5 6000 = 30,000.
5521
+ 6319

© Lopez Mateos Editores. ISBN 978-607-95583-2-1, obra completa, versión electrónica, ISBN 978-607-95583-3-8, volumen 1, versión electrónica. Ejemplar asignado a: Helecto Villarroel gutierrez -
helecto@gmail.com. Fecha: 27 de octubre de 2014. Prohibida su modificación, copia o distribución.
Sección 3-5 Matemática mental y estimación para operaciones entre números completos 183

4. Redondeo
El redondeo es una manera de “limpiar” números de modo que sean más fáciles de ma-
nejar. Al redondear podemos obtener respuestas aproximadas de cálculos:
4724 5000 (Redondea 4724 a 5000.)
+ 3192 + 3000 (Redondea 3192 a 3000.)
8000 (Suma los números redondeados.)

1267 1300 (Redondea 1267 a 1300.)


- 510 - 500 (Redondea 510 a 500.)
800 (Suma los números redondeados.)

Para efectuar estimaciones se requiere saber el significado del valor posicional y


conocer técnicas de redondeo. Ilustramos un procedimiento de redondeo que
puede generalizarse a todas las situaciones de redondeo. Por ejemplo, supongamos
que deseamos redondear 4724 al millar más cercano. Podemos proceder en cuatro
pasos (ver también la Figura 3-44).
4000 4724 5000
a. Determina dos millares consecutivos entre los cuales se ubique el número.
b. Determina el punto medio entre los dos millares.
4000 4500 4724 5000 c. Determina cuál millar está más cerca observando si el número es mayor o menor que
el punto medio. (No todos los libros de texto usan el mismo método para redondear cuando
4000 4500 4724 5000 el número cae en el punto medio.)
d. Si el número por redondear es mayor o igual que el punto medio, redondea el
Figura 3-44 número dado al millar mayor; de otro modo, redondea al millar menor. En este caso
redondeamos 4724 a 5000.
5. Usar el rango
Con frecuencia es útil saber en qué rango cae la respuesta. El rango queda determinado
al hallar un estimado bajo y un estimado alto, y señalando que la respuesta cae en ese
intervalo. Por ejemplo:
Problema Estimado bajo Estimado alto
378 300 400
+ 524 + 500 + 600
800 1000
Así un rango para este problema es de 800 a 1000.
En la Página de un libro de texto en la página 184 se muestran las estrategias de estimación de
redondeo y desde la izquierda aplicadas a un problema.

Estimación: Multiplicación y División


Damos a continuación ejemplos de estrategias de estimación para multiplicar y dividir.
1. Desde la izquierda
524 500 * 8 = 4000 (Comienza multiplicando desde la izquierda para obte-
* 8 ner un primer estimado.)
20 * 8 = 160 (Multiplica el siguiente dígito en importancia por 8.)
4000 + 160 = 4160 (Ajusta el primer estimado sumando los dos números.)

© Lopez Mateos Editores. ISBN 978-607-95583-2-1, obra completa, versión electrónica, ISBN 978-607-95583-3-8, volumen 1, versión electrónica. Ejemplar asignado a: Helecto Villarroel gutierrez -
helecto@gmail.com. Fecha: 27 de octubre de 2014. Prohibida su modificación, copia o distribución.
184 Números completos y sus operaciones

Página de un libro de texto EST I M AC I Ó N D E S U M A S


Y R ESTA S

Lección
Calentamiento
Idea clave
Hay más de una Estimación de Redondea cada

sumas y restas
manera de número al lugar del
estimar sumas y dígito subrayado.
restas
Aprende
Vocabulario
estimación desde ¿Cómo puedes estimar sumas?
la izquierda
redondeo Los estudiantes de una escuela primaria recolectan
latas de aluminio para reciclarlas. ¿Alrededor de
cuántas libras de latas recolectaron en total?
Latas para reciclar
Grado 3º 4º 5º 6º
Libras
recolectadas

ex
ef l i ó
R

Puedes estimar 398 + 257 + 285 + 318 de dos maneras.


¡Piensa! Juan usó redondeo. Kristi usó estimación desde la izquierda
Sólo necesito un y después ajustó el estimado.
estimado pues nos
preguntan alrededor
de cuántas libras Redondeo cada número a Primero sumo los dígitos de
la centena más cercana. la izquierda.

Después ajusto para incluir los


números restantes.

Alrededor de
1,300 libras Menos de 1,300 libras.

Fuente: Scott Foresman-Addison Wesley Math, Grade 5, 2008 ( p. 28).

© Lopez Mateos Editores. ISBN 978-607-95583-2-1, obra completa, versión electrónica, ISBN 978-607-95583-3-8, volumen 1, versión electrónica. Ejemplar asignado a: Helecto Villarroel gutierrez -
helecto@gmail.com. Fecha: 27 de octubre de 2014. Prohibida su modificación, copia o distribución.
Sección 3-5 Matemática mental y estimación para operaciones entre números completos 185

2. Números compatibles
5 冄 4163 5 冄 4000 (Substituye 4163 por un número cercano a él pero divisible
entre 5.)
800 (Efectúa la división y obtén el primer estimado de 800. Se
5 冄 4000 pueden usar varias técnicas para ajustar el primer estimado.)

AHORA INTENTA ÉSTE 3-19 Estima mentalmente cada caso y explica qué técnica usaste para
obtener la respuesta:
a. Se realizó un concierto en un teatro con capacidad de 4525 personas, Los boletos se agotaron. Cada
uno costó $9. ¿Cuánto dinero se juntó?
b. Se van a distribuir folletos en 3625 casas y hay 42 personas para hacerlo. Si se distribuyen equitativa-
mente, ¿cuántas casas visitará cada persona?

Evaluación 3-5A

1. Calcula mentalmente: c. 32 # 28
a. 180 + 97 - 23 + 20 - 140 + 26 d. 2215 + 3023 + 5967 + 975
b. 87 - 42 + 70 - 38 + 43 10. Usa la estimación desde la izquierda, con ajuste, para es-
2. Usa números compatibles para calcular mentalmente timar cada caso:
cada caso: a. 2215 + 3023 + 5987 + 975
a. 2 # 9 # 5 # 6 b. 8 # 25 # 7 # 4 b. 234 + 478 + 987 + 319 + 469
3. Usa las estrategias de separar y juntar, o de multiplica- 11. a. ¿Será bueno usar la estrategia de cúmulos en cada
ción desde la izquierda, para obtener mentalmente: uno de los casos siguientes? ¿Por qué sí o por qué no?
a. 567 + 38 b. 321 # 3
4. Usa el intercambio para calcular mentalmente: (i) 474 (ii) 483
a. 85 - 49 b. 87 + 33 1467 475
c. 143 - 97 d. 58 + 39 64 530
5. Un viaje en carro duró 8 horas a un promedio de 96 + 2445 503
kmph. Calcula mentalmente el total de kilómetros viaja- + 528
dos. Describe el método empleado.
6. Calcula cada caso usando el algoritmo del sumando fal- b. Estima cada parte de (a) usando las estrategias si-
tante (del cajero). guientes:
a. 53 - 28 b. 63 - 47 (i) Desde la izquierda
7. Calcula mentalmente cada caso. Explica brevemente el (ii) Completar números agradables
método usado. (iii) Redondeo
a. 86 + 37 12. Usa la estrategia del rango para estimar cada caso. Ex-
b. 97 + 54 plica cómo obtuviste tus estimados.
c. 230 + 60 + 70 + 44 + 40 + 6 a. 22 # 38
8. Redondea cada número al valor posicional indicado por b. 145 + 678
el dígito en negritas. c. 278 + 36
a. 5280 b. 115,234 13. Supón que tienes un saldo de $3287 en tu cuenta de che-
c. 115,234 d. 2,325 ques y que expides cheques por $85, $297, $403 y $523.
9. Estima cada respuesta por redondeo. Estima tu saldo, di cómo lo hiciste y di si piensas que tu es-
a. 878 , 29 timado es muy alto o muy bajo.
b. 25,201 - 19,987

© Lopez Mateos Editores. ISBN 978-607-95583-2-1, obra completa, versión electrónica, ISBN 978-607-95583-3-8, volumen 1, versión electrónica. Ejemplar asignado a: Helecto Villarroel gutierrez -
helecto@gmail.com. Fecha: 27 de octubre de 2014. Prohibida su modificación, copia o distribución.
186 Números completos y sus operaciones

14. Un teatro tiene 38 filas con 23 lugares en cada fila. Es- 17. La asistencia a la feria mundial durante una semana fue de:
tima el número de lugares en el teatro y di cómo llegaste
al estimado. Lunes 72,250
15. Sin efectuar los cálculos, di cuáles dan el mismo resul- Martes 63,891
tado. Describe tu razonamiento. Miércoles 67,490
a. 44 # 22 y 22 # 11 Jueves 73,180
b. 22 # 32 y 11 # 64 Viernes 74,918
c. 13 # 33 y 39 # 11 Sábado 68,480
16. La siguiente es una lista del área en kilómetros cuadra- Estima la asistencia de la semana y di qué estrategia usas-
dos de los países más grandes de Europa. Usa esta te y por qué.
información para decidir mentalmente cuáles de las afir-
18. En cada caso, determina si el estimado dado en el parén-
maciones son verdaderas.
tesis es alto (mayor que la respuesta real) o bajo (menor
Francia 543,965 que la respuesta real). Justifica tus respuestas sin calcular
España 505,990 los valores exactos.
Suecia 449,964 a. 299 # 300 190,0002
Finlandia 338,145 b. 6001 , 299 1202
Noruega 323,758 c. 6000 , 299 1202
a. Suecia es menos de 100,000 km2 más grande que Fin- d. 999 , 99 1102
landia. 19. Usa tu calculadora para obtener 252, 352, 452 y 552, y des-
b. Francia es más del doble que Noruega. pués ve si puedes hallar un patrón que te permita calcular
c. Francia es más de 250,000 km2 más grande que Noruega. mentalmente 652 y 752.
d. España es alrededor de 55,000 km2 mayor que Suecia

Evaluación 3-5B

1. Calcula mentalmente: 8. Redondea cada número al valor posicional indicado por


a. 160 + 92 - 32 + 40 - 18 el dígito en negritas.
b. 36 + 97 - 80 + 44 a. 3587
2. Usa números compatibles para calcular mentalmente b. 148,213
cada caso: c. 23,785
a. 5 # 11 # 3 # 20 d. 2,357
b. 82 + 37 + 18 + 13 9. Estima cada respuesta por redondeo.
3. Da razones para los primeros cuatro pasos. a. 937 , 28 b. 32,285 - 18,988
c. 52 # 48
1525 + 372 + 75 = 525 + 137 + 752
d. 3215 + 3789 + 5987
= 525 + 175 + 372
10. Usa la estimación desde la izquierda, con ajuste, para es-
= 1525 + 752 + 37
timar cada caso:
a. 2345 + 5250 + 4210 + 910
= 600 + 37
b. 345 + 518 + 655 + 270
= 637
11. a. ¿Será bueno usar la estrategia de cúmulos en cada
4. Usa las estrategias de separar y juntar, o de multiplica- uno de los casos siguientes? ¿Por qué sí o por qué no?
ción desde la izquierda, para obtener mentalmente:
(i) 318 (ii) 2350
a. 997 - 32 b. 56 # 30
2314 1987
5. Usa el intercambio para calcular mentalmente:
57 2036
a. 75 - 38 b. 57 + 35
+ 3489 2103
c. 137 - 29 d. 78 + 49
+ 1890
6. Calcula cada caso usando el algoritmo del sumando fal-
tante (del cajero):
a. 74 - 63 b. 73 - 57 b. Estima cada parte de (a) usando las estrategias siguientes:
7. Calcula mentalmente cada caso. Explica brevemente el (i) Desde la izquierda, con ajuste
método usado. (ii) Completar números agradables
a. 81 - 46 b. 98 - 19 (iii) Redondeo
c. 9700 - 600

© Lopez Mateos Editores. ISBN 978-607-95583-2-1, obra completa, versión electrónica, ISBN 978-607-95583-3-8, volumen 1, versión electrónica. Ejemplar asignado a: Helecto Villarroel gutierrez -
helecto@gmail.com. Fecha: 27 de octubre de 2014. Prohibida su modificación, copia o distribución.
Sección 3-5 Matemática mental y estimación para operaciones entre números completos 187

12. Usa la estrategia del rango para estimar cada caso. Ex- c. Alberto y Juan se postularon para puestos de elec-
plica cómo obtuviste tus estimados. ción. Van a recibir votos de dos distritos. Alberto re-
a. 32 # 47 cibe 3473 votos de un distrito y 5615 del otro distrito.
b. 123 + 780 Juan recibe 3463 votos del primer distrito y 5616 del
c. 482 + 246 segundo. ¿Quién resultó electo?
13. Tomás estimó 31 # 179 de las tres maneras mostradas abajo. d. Dos parcelas rectangulares tienen dimensiones de
(i) 30 # 200 = 6000 101 m por 120 m, y 103 m por 129 m. ¿Qué parcela
(ii) 30 # 180 = 5400 tiene mayor área? (Recuerda que el área de un rectán-
(iii) 31 # 200 = 6200 gulo es el largo por el ancho.)
Sin hallar el producto, ¿cuál estimado consideras más 17. En cada caso, determina si el estimado dado en el parén-
cercano al resultado real? ¿Por qué? tesis es alto (mayor que la respuesta real) o bajo (menor
14. Se deben quemar alrededor de 3,540 calorías para bajar que la respuesta real). Justifica tu respuesta sin calcular
medio kilo de peso. Estima cuántas calorías debes que- los valores exactos.
mar para bajar 3 kilos. a. 398 # 500 1200,0002
15. Sin efectuar los cálculos, di cuáles dan el mismo resul- b. 8001 , 398 1202
tado. Describe tu razonamiento. c. 10,000 , 999 1102
a. 88 # 44 y 44 # 22 d. 1999 , 201 1102
b. 93 # 15 y 31 # 45 18. Usa tu calculadora para multiplicar varios números de dos
c. 12 # 18 y 20 # 17 dígitos por 99. Después, ve si puedes hallar un patrón que
16. En cada caso, responde usando métodos de estimación si te permita obtener mentalmente el producto de cualquier
es posible. Cuando la estimación no sea apropiada, ex- número de dos dígitos por 99.
plica por qué.
a. José tiene $3800 en su cuenta de cheques. Quiere
emitir cheques por $390, $280, $590 y $2500. ¿Tiene
dinero suficiente para cubrir estos cheques?
b. Gilda depositó dos cheques en su cuenta, uno por
$9810 y el otro por $11,400. ¿Tiene suficiente dinero
en su cuenta para cubrir un cheque de $20,000 si sa-
bemos que inició con un saldo positivo?

Conexiones matemáticas 3-5

Comunicación Aprendizaje colectivo


1. ¿Cuál es la diferencia entre matemática mental y estima- 7. En un grupo cada persona escoge un libro de texto de di-
ción de cálculo? ferente grado (3-6) y hace una lista de estrategias de ma-
2. ¿La estimación desde la izquierda siempre es menor que temática mental o de estimación presentadas para cada
la suma exacta antes del ajuste? Explica por qué sí o por grado. ¿Cómo se comparan las listas?
qué no. 8. Como grupo, y sin obtener realmente la respuesta, digan
3. En los nuevos libros de texto se hace énfasis en matemá- cuál es mayor: 19,876 # 43 ó 19,875 # 44. Preparen una
tica mental y estimación. ¿Piensas que dichos temas son respuesta de grupo para exponer al resto de la clase.
importantes para el estudiante actual? ¿Por qué? Preguntas del salón de clase
4. Supón que x y y son números completos positivos (ma- 9. Mane calculó 261 - 48 restando primero 50 de 261 para
yores que 0). Si x es mayor que y y estimas x - y redon- obtener 211; después, para equilibrar la suma adicional
deando x hacia arriba y y hacia abajo, ¿tu estimado será de 2 a 48, restó 2 de 211 para obtener una respuesta de
siempre muy alto o muy bajo, o puede variar? Explica. 209. ¿Es correcto su razonamiento? De no ser así,
Solución abierta ¿cómo podrías ayudarla?
10. Un estudiante pregunta por qué tiene que aprender otras
5. Da varios ejemplos de situaciones del mundo real donde estrategias de estimación además del redondeo. ¿Cuál es
sea suficiente un estimado en lugar de una respuesta tu respuesta?
exacta. 11. Para terminar rápidamente su tarea, una estudiante de
6. a. Da un ejemplo numérico donde la estimación desde la nivel elemental resuelve sus problemas de estimación
izquierda y el redondeo produzcan el mismo resultado. usando una calculadora para hallar las respuestas exactas
b. Da un ejemplo de cuándo pueden producir resultados y después redondearlas a fin de obtener un estimado.
diferentes. ¿Qué le dices?

© Lopez Mateos Editores. ISBN 978-607-95583-2-1, obra completa, versión electrónica, ISBN 978-607-95583-3-8, volumen 1, versión electrónica. Ejemplar asignado a: Helecto Villarroel gutierrez -
helecto@gmail.com. Fecha: 27 de octubre de 2014. Prohibida su modificación, copia o distribución.
188 Números completos y sus operaciones

Problemas de repaso
12. Explica por qué cuando un número se multiplica por 10, Rockville 128 millas
se añade un cero al número.
13. Efectúa cada una de las divisiones siguientes usando ambos
algoritmos, el de la resta consecutiva y el convencional.
a. 18 冄 623
b. 21 冄 493
c. 97 冄 1000
14. Escribe cada una de las respuestas del problema 13 en la Mika y su mamá vieron el letrero mostrado cuando iban
forma a = b # c + r, donde 0 … r 6 b. en su automóvil hacia Rockville. Si su velocidad es de
alrededor de 65 millas por hora aproximadamente, ¿en
Preguntas del National Assessment of Educational cuántas horas más terminarán su viaje?
Progress (NAEP) (Evaluación Nacional del Progreso a. 1 b. 2 c. 3 d. 4 e. 5
Educativo) NAEP 2007, Grado 8
¿Cuál operación sería más fácil de resolver usando
matemática mental?
a. $65.12 - $28.19
b. 358 * 2
c. 1,625 , 3
d. $100.00 + $10.00
NAEP 2007, Grado 4

ROMPECABEZAS  En la Asociación de Padres y Maestros de una escuela organizaron un árbol tele-


fónico para localizar a todos sus miembros. La responsabilidad de cada persona, después de recibir
una llamada, es llamar a otros dos miembros asignados, y así hasta que todos los miembros hayan sido
localizados. Supón que todos estaban en su casa y contestaron el teléfono, y que cada llamada duró
30 segundos. Si uno de los 85 miembros realiza la primera llamada telefónica e inicia un cronómetro,
¿cuál es la menor cantidad de tiempo necesaria para localizar a los 85 miembros del grupo?

Sugerencia para resolver el problema preliminar


No es común usar varios 5, como 55 y 555; el problema puede resolverse usando va-
rios 5 sólo dos veces. El factor principal para resolver este problema, según se mostró en
el ejemplo, es usar símbolos de agrupamiento.

© Lopez Mateos Editores. ISBN 978-607-95583-2-1, obra completa, versión electrónica, ISBN 978-607-95583-3-8, volumen 1, versión electrónica. Ejemplar asignado a: Helecto Villarroel gutierrez -
helecto@gmail.com. Fecha: 27 de octubre de 2014. Prohibida su modificación, copia o distribución.
Resumen del capítulo 189

Resumen del capítulo

I. Números completos D. Relaciones en los números completos


A. El conjunto de los números completos C es 1. a 6 b si, y sólo si, existe un número natural
50, 1, 2, 3, Á 6. c tal que a + c = b.
B. Las operaciones básicas de los números comple- 2. a 7 b si, y sólo si, b 6 a.
tos son: suma, resta, multiplicación y división. II. Algoritmos para las operaciones con números completos
1. Suma: Si n1A2 = a y n1B2 = b, donde A. Algoritmos para la suma y la resta
A ¨ B = ⭋, entonces a + b = n1A ´ B2. 1. Modelos concretos
Los números a y b son sumandos y a + b es 2. Algoritmos expandidos
la suma. 3. Algoritmos convencionales o conocidos
2. Resta: Si a y b son números completos cuales- 4. Suma y resta reagrupando
quiera, entonces a - b es el único número 5. Suma con marcas
completo c tal que a = b + c. 6. Sumandos iguales
3. Multiplicación: Si a y b son números com- 7. Suma y resta en varias bases numéricas
pletos cualesquiera y a Z 0, entonces B. Algoritmos de multiplicación y división
1. Modelos concretos
ab = b + b + b + Á + b
2. Algoritmos expandidos
a términos 3. Algoritmos convencionales
donde a y b son factores y ab es el producto. 4. Retícula para multiplicar
4. Multiplicación: Si A y B son conjuntos ta- 5. Andamiaje para la división
les que n1A2 = a y n1B2 = b, entonces 6. División corta
ab = n1A * B2. 7. Multiplicación y división en otras bases nu-
5. División: Si a y b son números completos méricas
cualesquiera, con b Z 0, a , b es el único III. Estrategias de matemática mental y de estimación
número completo c tal que bc = a. El nú- de cálculos
mero a es el dividendo, b es el divisor y c es A. Matemática mental
el cociente. 1. Sumar desde la izquierda
6. Algoritmo de la división: Dados cuales- 2. Separar y juntar
quier números completos a y b, con b Z 0, 3. Intercambio
existen números completos únicos c y r ta- 4. Uso de números compatibles
les que a = bc + r, con 0 … r 6 b. 5. Completar a números compatibles
C. Propiedades de la suma y la multiplicación de 6. Quitar ceros
números completos 7. Algoritmo del cajero (sumandos iguales)
1. Cerradura: Si a, b 僆 C, entonces a + b 僆 C 8. Multiplicar desde la izquierda
y ab 僆 C. 9. Pensar en monedas
2. Conmutativa: Si a, b 僆 C, entonces a + b = 10. Partir el dividendo
b + a y ab = ba. B. Estrategias para estimar el cálculo
3. Asociativa: Si a, b, c 僆 C, entonces 1a + b2 + 1. Desde la izquierda
c = a + 1b + c2 y 1ab2c = a1bc2. 2. Completar números agradables
4. Identidad: 0 es el único elemento identidad para 3. Cúmulos
la suma de números completos; 1 es el único 4. Redondeo
elemento identidad para la multiplicación. 5. Rango
5. Propiedad distributiva de la multiplicación so- 6. Números compatibles
bre la suma: Si a, b, c 僆 C, entonces
a1b + c2 = ab + ac.
6. Propiedad distributiva de la multiplicación so-
bre la resta: Si a, b, c 僆 C, con b Ú c,
a1b - c2 = ab - ac.
7. Propiedad de la multiplicación por cero: Para
cualquier número completo a, a # 0 =
0 = 0 # a.

© Lopez Mateos Editores. ISBN 978-607-95583-2-1, obra completa, versión electrónica, ISBN 978-607-95583-3-8, volumen 1, versión electrónica. Ejemplar asignado a: Helecto Villarroel gutierrez -
helecto@gmail.com. Fecha: 27 de octubre de 2014. Prohibida su modificación, copia o distribución.
190 Números completos y sus operaciones

Revisión del capítulo

1. Identifica las propiedades de las operaciones de los 11. a. Piensa un número.


números completos que ilustra cada caso: Súmale 17.
a. 31a + b2 = 3a + 3b Dobla el resultado.
b. 2 + a = a + 2 Réstale 4.
c. 16 # 1 = 1 # 16 = 16 Dobla el resultado.
d. 6112 + 32 = 6 # 12 + 6 # 3 Súmale 20.
e. 31a # 22 = 312a2 Divídelo entre 4.
f. 312a2 = 13 # 22a Réstale 20.
Tu respuesta será tu número original. Explica cómo
2. Usando las definiciones de menor que o mayor
funciona este truco.
que, prueba que cada una de las siguientes desi-
b. Plantea tres pasos más que te regresen a tu nú-
gualdades es verdadera:
mero original.
a. 3 6 13
Piensa un número.
b. 12 7 9
Súmale 18.
3. Halla posibles reemplazos que hagan verdaderas las Multiplícalo por 4.
proposiciones siguientes acerca de números com- Réstale 7.
pletos: #
a. 4 # n - 37 6 27 #
b. 398 = n # 37 + 28 #
c. n # 13 + 42 = n # 3 + n # 4 c. Elabora una serie de instrucciones que te regre-
d. 42 - n Ú 16 sen siempre a tu número original.
4. Usa la propiedad distributiva de la multiplicación so- 12. Usa el algoritmo de marcas y el tradicional para
bre la suma y resultados de sumas, si es posible, para efectuar:
reescribir:
a. 3a + 7a + 5a 316
b. 3x 2 + 7x 2 - 5x 2 712
c. x1a + b + y2 + 91
d. 1x + 523 + 1x + 52y 13. En el caso siguiente usa el algoritmo tradicional y la
5. ¿Cuántas latas de jugo de 12 oz se necesitan para retícula para multiplicar:
servir a 60 personas una porción de 8 oz a cada una?
613
6. Nina tiene un pantalón café y uno gris; una blusa café,
* 98
una amarilla y una blanca; y un suéter azul y uno
blanco. ¿De cuántas maneras puede vestirse si lleva un 14. Usa el algoritmo de la resta consecutiva y el con-
pantalón, una blusa y un suéter? vencional para efectuar:
7. Estoy pensando un número completo. Si lo divido a. 912 冄 4803 b. 11 冄 1011
entre 13, después multiplico la respuesta por 12,
c. 23cinco 冄 3312cinco d. 11dos 冄 1011dos
luego le resto 20 y después le sumo 89, el resultado
15. Usa el algoritmo de la división para verificar las res-
es 93. ¿Cuál fue el número que pensé?
puestas obtenidas en el problema 14.
8. Un centro vacacional ofrece un paquete de fin de se- 16. En algunos cálculos conviene usar una combinación
mana con todo incluido por $800 por persona, o de de matemática mental y calculadora. Por ejemplo,
$60,000 para un grupo de 80 personas. ¿Cuál será la como
opción más barata para un grupo de 80 personas?
9. Julia tiene un trabajo de 30 h> sem y le pagan $50>h. 200 # 97 # 146 # 5 = 97 # 1461200 # 52
Si trabaja más de 30 h en una semana recibe $80> h = 97 # 146 # 1000
por cada hora extra 30 h. Si trabajó 38 h esta se- podemos calcular 97 # 146 en una calculadora y mul-
mana, ¿cuánto le pagaron? tiplicar mentalmente por 1000. Muestra cómo cal-
10. En un concurso de televisión hay que responder cinco cular cada uno de los casos siguientes combinando
preguntas. Cada pregunta vale el doble de la anterior. matemática mental y calculadora:
Si la última pregunta vale $64,000, ¿cuánto valía la a. 19 # 5 # 194 # 2
primera pregunta? b. 379 # 4 # 193 # 25

© Lopez Mateos Editores. ISBN 978-607-95583-2-1, obra completa, versión electrónica, ISBN 978-607-95583-3-8, volumen 1, versión electrónica. Ejemplar asignado a: Helecto Villarroel gutierrez -
helecto@gmail.com. Fecha: 27 de octubre de 2014. Prohibida su modificación, copia o distribución.
Bibliografía seleccionada 191

c. 8 # 481 # 73 # 125 27. Usa cúmulos para estimar la suma 2345 + 2854 +
d. 374 # 200 # 893 # 50 2234 + 2203.
17. Tenías en tu chequera un saldo de $720 antes de ex- 28. Explica cómo funciona el algoritmo convencional
pedir cheques por $162, $158 y $33 y de hacer un de la división para:
depósito de $28. ¿Cuál es tu nuevo saldo?
23
18. A Jaime le pagaron $3200 mensuales durante 6 me-
ses y $4100 mensuales durante 6 meses. ¿Cuál fue el 14 冄 322
total de sus ingresos en el año? - 28
19. Un fabricante de refrescos produce 15,600 latas de 42
su producto cada hora. Se empacan 24 latas en una - 42
caja. ¿Cuántas cajas podrá llenar con las latas produ- 0
cidas durante 4 h?
20. Una sociedad de 120 inversionistas vendió un te- 29. En algunos casos se puede usar la propiedad distri-
rreno en $461,040. Si esta cantidad se repartió equi- butiva de la multiplicación sobre la suma, o la pro-
tativamente, ¿cuánto recibió cada inversionista? piedad distributiva de la multiplicación sobre la
21. Normalmente, las manzanas cuestan 32¢ cada una. resta, para obtener rápidamente una respuesta. Usa
Las venden de oferta en 3 por 69¢. ¿Cuánto ahorras una de las propiedades distributivas para calcular
si compras 2 docenas de manzanas cuando están en cada uno de los casos siguientes de la manera más
oferta? sencilla posible:
22. El propietario de una tienda de bicicletas reportó a. 999 # 47 + 47
su inventario de bicicletas y triciclos de manera b. 43 # 59 + 41 # 43
poco común. Dijo que contó 126 ruedas y 108 pe- c. 1003 # 79 - 3 # 79
dales. ¿Cuántas bicicletas y cuántos triciclos tiene? d. 1001 # 113 - 113
23. Efectúa los cálculos siguientes: e. 101 # 35
a. 123cinco b. 1010dos f. 98 # 35
+ 34cinco -101dos 30. Recuerda que los problemas de suma como
3478 + 521 se pueden escribir y calcular usando
c. 23cinco d. 1001dos notación expandida como se muestra aquí, y res-
* 34cinco * 101dos ponde las preguntas siguientes.
24. Di cómo usar mentalmente números compatibles
para efectuar las operaciones siguientes: 3 # 103 + 4 # 102 + 7 # 10 + 8
a. 26 + 37 + 24 - 7 b. 4 # 7 # 9 # 25 + 5 # 102 + 2 # 10 + 1
25. Calcula mentalmente. Menciona la estrategia usada 3 10 + 9 # 102 + 9 # 10 + 9
# 3
para tu matemática mental (las estrategias pueden
variar). a. Escribe un problema algebraico de suma corres-
a. 63 # 7 b. 85 - 49 pondiente (usa x en lugar de 10) y obtén la res-
c. 118 # 522 d. 2436 , 6 puesta.
26. Estima la suma siguiente usando (a) estimación b. Escribe un problema de resta y el problema alge-
desde la izquierda con ajuste, y (b) redondeo. braico correspondiente, y obtén la respuesta.
c. Escribe un problema de multiplicación y el
543 problema algebraico correspondiente, y calcula
398 la respuesta.
255
408
+ 998

Bibliografía seleccionada

Baek, J. “Children’s Mathematical Understanding and Baroody, A. “Why Children Have Difficulties Master-
Invented Strategies for Multidigit Multiplication.” ing the Basic Number Combinations and How to
Teaching Children Mathematics 12 (December 2005): Help Them.” Teaching Children Mathematics 13
242–247. (August 2006): 22–31.

© Lopez Mateos Editores. ISBN 978-607-95583-2-1, obra completa, versión electrónica, ISBN 978-607-95583-3-8, volumen 1, versión electrónica. Ejemplar asignado a: Helecto Villarroel gutierrez -
helecto@gmail.com. Fecha: 27 de octubre de 2014. Prohibida su modificación, copia o distribución.
192 Números completos y sus operaciones

Bass, H. “Computational Fluency, Algorithms, and Gregg, J. “Interpreting the Standard Division Algorithm
Mathematical Proficiency: One Mathematician’s in a ‘Candy Factory’ Context.” Teaching Children
Perspective.” Teaching Children Mathematics 9 (Feb- Mathematics 14 (August 2007): 25–31.
ruary 2003): 322–329. Hedges, M., D. Huinker, and M. Steinmeyer. “Unpack-
Bell, A., B. Greer, C. Mangan, and L. Grimison. “Chil- ing Division to Build Teachers’ Mathematical Knowl-
dren’s Performance on Multiplicative Word Prob- edge.” Teaching Children Mathematics 11 (May 2005):
lems: Elements of a Descriptive Theory.” Journal 478–483.
for Research in Mathematics Education 1989, 20(5): Heuser, D. “Teaching without Telling: Computational
434–449. Fluency and Understanding through Invention.”
Bobis, J. “The Empty Number Line: A Useful Tool or Teaching Children Mathematics 11 (April 2005): 404–
Just Another Procedure?” Teaching Children Mathe- 412.
matics 13 (April 2007): 410–413. Huinker, D., J. Freckman, and M. Steinmeyer. “Sub-
Broadent, F. “Lattice Multiplication and Division.” traction Strategies from Children’s Thinking: Mov-
Arithmetic Teacher 34 ( January 1987): 28–31. ing Toward Fluency with Greater Numbers.”
Brownell, W. “From NCTM’s Archives: Meaning and Teaching Children Mathematics 9 (February 2003):
Skill—Maintaining the Balance.” Teaching Children 347–353.
Mathematics 9 (February 2003): 310–316. Lampert, M. “Teaching and Learning Long Division
Carpenter, T., and J. Moser. “The Acquisition of Addi- for Understanding in School.” In Analysis of Arith-
tion and Subtraction Concepts in Grades One metic for Mathematics Teaching, edited by G. Lein-
Through Three.” Journal for Research in Mathemat- hardt, R. Putnam, and R. Hattrup. Hillsdale, NJ:
ics Education 15 (May 1984): 179–202. LEA, 1992.
Crespo, S., A. Kyriakides, and S. McGee. “Nothing Postlewait, K., M. Adams, and J. Shih. “Promoting
Basic about Basic Facts: Exploring Addition Facts Meaningful Mastery of Addition and Subtraction.”
with Fourth Graders.” Teaching Children Mathemat- Teaching Children Mathematics 9 (February 2003):
ics 12 (September 2005): 60–67. 354–357.
deGroot, C., and T. Whalen, “Longing for Division.” Randolph, T., and H. Sherman. “Alternative Algorithms:
Teaching Children Mathematics 12 (April 2006): 410– Increasing Options, Reducing Errors.” Teaching Chil-
418. dren Mathematics 7 (April 2001): 480–484.
Ebdon, S., M. Coakley, and D. Legnard. “Mathemati- Resnick, L. “From Protoquantities to Operators:
cal Mind Journeys: Awakening Minds to Computa- Building Mathematical Competence on a Foun-
tional Fluency.” Teaching Children Mathematics 9 dation of Everyday Knowledge.” In Analysis of
(April 2003): 486–493. Arithmetic for Mathematics Teaching, edited by
English, L., and G. Halford. Mathematics Education D. Leinhardt, R. Putnam, and R. Hattrup. Hills-
Models and Processes. Mahwah, New Jersey: Lau- dale, NJ: LEA, 1992.
rence Erlbaum, 1995. Reys, R. “Mental Computation and Estimation: Past,
Flowers, J., K. Kline, and R. Rubenstein. “Developing Present, and Future.” Elementary School Journal 84
Teachers’ Computational Fluency: Examples in Sub- (1984): 547–557.
traction.” Teaching Children Mathematics 9 (February ———. “Computation Versus Number Sense.” Mathe-
2003): 330–346. matics Teaching in the Middle School 4 (October 1998):
Fuson, K. “Research on Learning and Teaching Addi- 110–112.
tion and Subtraction of Whole Numbers.” In Reys, B., and R. Reys. “Computation in the Elementary
Handbook of Research on Mathematics Teaching and Curriculum: Shifting the Emphasis.” Teaching Chil-
Learning, edited by D. Grouws. New York: Mac- dren Mathematics 5 (December 1998): 236–241.
Millan, 1992. Russell, S. “Developing Computational Fluency with
———. “Toward Computational Fluency.” Teaching Whole Numbers.” Teaching Children Mathematics 7
Children Mathematics 9 (February 2003): 300–309. (November 2000): 154–158.
Ginsburg, H., A. Klein, and P. Starkey. “The Develop- Scharton, S. “I Did It My Way: Providing Opportuni-
ment of Children’s Mathematical Thinking: Con- ties for Students to Create, Explain, and Analyze
necting Research with Practice.” In Child Psychology Computation Procedures.” Teaching Children Math-
in Practice, edited by Irving E. Sigel and K. Ann ematics 10 ( January 2004): 278–283.
Renninger, pp. 401–476, vol. 4 of Handbook of Child Siegler, R. Emerging Minds: The Process of Change in
Psychology, edited by William Damon. New York: Children’s Thinking. New York: Oxford University
John Wiley & Sons, 1998. Press, 1996.
Grant, T., J. Lo, and J. Flowers. “Shaping Prospective Silver, E., L. Shapiro, and A. Deutsch. “Sense Making
Teachers’ Justifications for Computation: Chal- and the Solution of Division Problems Involving
lenges and Opportunities.” Teaching Children Math- Remainders: An Examination of Middle School Stu-
ematics 14 (September 2007): 112–116. dents’ Solution Processes and Their Interpretations

© Lopez Mateos Editores. ISBN 978-607-95583-2-1, obra completa, versión electrónica, ISBN 978-607-95583-3-8, volumen 1, versión electrónica. Ejemplar asignado a: Helecto Villarroel gutierrez -
helecto@gmail.com. Fecha: 27 de octubre de 2014. Prohibida su modificación, copia o distribución.
Bibliografía seleccionada 193

of Solutions.” Journal for Research in Mathematics Wallace, A., and S. Gurganus. “Teaching for Mastery of
Education 24 (March 1993): 117–135. Multiplication.” Teaching Children Mathematics 12
Sisul, J. “Fostering Flexibility with Numbers in the (August 2005): 26–33.
Primary Grades.” Teaching Children Mathematics 9 Whitenack, J., N. Knipping, S. Novinger, and G. Under-
(December 2002): 202–204. wood. “Second Graders Circumvent Addition and
Sowder, J. “Affective Factors and Computational Estima- Subtraction Difficulties.” Teaching Children Mathe-
tion Abilities.” In Affect and Problem Solving: A New matics 8 (December 2001): 228–233.
Perspective, edited by D. McLeod and V. Adams. Wickett, M. “Discussion as a Vehicle for Demonstrating
New York: Springer-Verlag, 1989. Computational Fluency in Multiplication.” Teaching
———. “Mental Computation and Number Sense.” Children Mathematics 9 (February 2003): 318–321.
Arithmetic Teacher 37 (March 1990): 18–20.

© Lopez Mateos Editores. ISBN 978-607-95583-2-1, obra completa, versión electrónica, ISBN 978-607-95583-3-8, volumen 1, versión electrónica. Ejemplar asignado a: Helecto Villarroel gutierrez -
helecto@gmail.com. Fecha: 27 de octubre de 2014. Prohibida su modificación, copia o distribución.
Razonamiento algebraico
CAPÍTULO

Problema preliminar
Anita, una maestra del quinto grado, pide a sus alumnos que piensen un número, lo multipliquen
por 6 le sumen 4 y luego, que dividan el resultado entre 2, le sumen 5, multipliquen el nuevo re-
sultado por 2 y luego le resten 18. Después, pide que cada alumno le diga el resultado obtenido y
conforme le responden, ella dice a cada uno cuál fue el número que pensó. ¿Cómo es posible
que Anita supiera de inmediato el número de cada alumno?

194
© Lopez Mateos Editores. ISBN 978-607-95583-2-1, obra completa, versión electrónica, ISBN 978-607-95583-3-8, volumen 1, versión electrónica. Ejemplar asignado a: Helecto Villarroel gutierrez -
helecto@gmail.com. Fecha: 27 de octubre de 2014. Prohibida su modificación, copia o distribución.
Razonamiento algebraico 195

C omo el razonamiento algebraico es tan importante en las matemáticas, en todos los


niveles y desde los grados más elementales, incluimos un capítulo exclusivo sobre el
tema. En este capítulo estudiaremos no sólo patrones (introducidos en el capítulo 1), sino
también otras características del razonamiento algebraico como resolución de ecuaciones,
problemas narrados (formulados con palabras), funciones y graficación.
En años anteriores no se enseñaba álgebra a los estudiantes hasta finales de la enseñanza
media o secundaria. Actualmente comprendemos la importancia de integrar el razona-
miento algebraico y la resolución de problemas en todos los niveles, comenzando en el jar-
dín de niños. De hecho, como lo señala la Nota de investigación, el razonamiento algebraico
se debe enseñar a todos los estudiantes.
En los Principios y objetivos se recomienda que desde preescolar hasta el grado 2, los alum-
nos sean capaces de:

◆ Nota de • usar representaciones concretas, gráficas y verbales para desarrollar la comprensión de notacio-
nes simbólicas, ya sean inventadas o convencionales;
investigación
• modelar situaciones que incluyan la suma y resta de números completos, usando objetos,
Hemos hallado que
figuras y símbolos. (p. 90)
desde pequeños, las-
niñas y los niños pue- Para los grados de 3 a 5 se pide:
den aprender a • representar y analizar patrones y funciones usando palabras, tablas y gráficas;
participar en el razo- • representar el concepto de variable como cantidad desconocida usando una letra o símbolo;
namiento algebraico.
• expresar relaciones matemáticas usando ecuaciones;
Más aún, el aprendi-
• investigar cómo el cambio en una variable se relaciona con el cambio en una segunda variable;
zaje de las grandes
ideas y prácticas de • identificar y describir situaciones con razones de cambio constante o variable, y
compararlas. (p. 158)
las matemáticas no
está reservado para Para los grados de 6 a 8, los estudiantes deben ser capaces de:
unos cuantos
alumnos matemática-
• identificar funciones como lineales o no lineales y contrastar sus propiedades por medio de ta-
mente dotados . De
blas, gráficas o ecuaciones;
hecho, hay argumen-
tos acerca de que es • desarrollar una comprensión conceptual inicial de distintos usos de las variables;
de vital importancia • explorar las relaciones entre expresiones simbólicas y gráficas de rectas, prestando particular
que los alumnos con atención al significado de cruce y pendiente;
riesgo de reprobar • usar álgebra simbólica para representar situaciones y resolver problemas, especialmente aque-
matemáticas, partici- llos que involucren relaciones lineales;
pen de estas ideas y
• reconocer y generar formas equivalentes para expresiones algebraicas sencillas y resolver ecua-
prácticas (Carpenter ciones lineales. (p. 222)
et al. 2003) ◆
En los Puntos focales se afirma que a los estudiantes del grado 6 se les debe enseñar a:

• escribir expresiones matemáticas y ecuaciones que correspondan a situaciones dadas


• evaluar expresiones
• usar expresiones y fórmulas para resolver problemas
• entender que las variables representan números cuyos valores todavía no se han especificado
• usar variables de manera apropiada

© Lopez Mateos Editores. ISBN 978-607-95583-2-1, obra completa, versión electrónica, ISBN 978-607-95583-3-8, volumen 1, versión electrónica. Ejemplar asignado a: Helecto Villarroel gutierrez -
helecto@gmail.com. Fecha: 27 de octubre de 2014. Prohibida su modificación, copia o distribución.
196 Razonamiento algebraico

• entender que algunas expresiones presentadas en forma diferente pueden ser equivalentes

• reescribir una expresión para representar una cantidad de manera diferente

• saber que las soluciones de una ecuación son los valores de las variables que hacen verdadera a
la ecuación

• resolver ecuaciones sencillas, de un paso, usando sentido numérico, propiedades de las opera-
ciones y la idea de mantener la igualdad en ambos lados de una ecuación

• construir y analizar tablas

• usar ecuaciones para describir relaciones sencillas (como 3x = y) mostradas en una tabla (p. 18)

En este capítulo usamos nuestro conocimiento básico de las operaciones para construir
los principios del razonamiento algebraico. En los capítulos subsecuentes veremos más de
cerca las matemáticas que damos aquí por sentadas así como nuevos temas, y profundizare-
mos en el razonamiento algebraico.
El álgebra es una rama de las matemáticas en la cual los números o elementos de un
conjunto dado se representancon símbolos, usualmente con letras. El álgebra elemental
se usa para generalizar la aritmética. Por ejemplo, las expresiones 7 + 13 + 52 =
17 + 32 + 5 ó 9 + 13 + 82 = 19 + 32 + 8, son casos particulares de a + 1b + c2 =
1a + b2 + c, donde a, b y c son números de un conjunto dado, por ejemplo los números
completos, los enteros, los números racionales o los números reales. De manera análoga,
2 + 3 = 3 + 2 y 2 # 3 = 3 # 2 son casos particulares de a + b = b + a y a # b = b # a para
todos los números completos a y b.

◆ Nota La palabra álgebra, versión latinizada de la palabra árabe al-jabr, viene del libro Hidab al-jabr
histórica wa’l muqabalah, escrito por Muhammad ibn Musa al-Jwârizmî (ca. 825 ce). al-Jwârizmî
(de cuyo nombre viene la palabra algoritmo) formaba parte de la Casa de la Sabiduría (Bayt
al-Hikma), una institución para la educación y la investigación fundada por el califa al-Ma-
mun. En su libro, sintetizó trabajos hindúes anteriores sobre los conceptos del álgebra y usó
las palabras jabr y muqubalah para designar las dos operaciones básicas para resolver ecua-
ciones: jabr significaba pasar o compensar los términos restados de un lado al otro lado de
la ecuación; muqubalah significaba cancelar o substraer términos similares en ambos lados
de la ecuación. El título de su libro se traduce como La ciencia de compensar lo que falta y subs-
traer los iguales.
Otra persona que contribuyó de manera principal al desarrollo del álgebra fue Dio-
fanto (ca. 200– 284 ce). Aritmética es el más importante trabajo de Diofanto y el tra-
al-Jwârizmî bajo más prominente de álgebra en las matemáticas griegas. De los trece libros
originales de los que constaba Aritmética, sólo sobreviven seis.
Alrededor de 900 años después, Leonardo di Pisa (ca. 1170–1250) introdujo el álgebra a
Europa. También se le conoce como Fibonacci, que significa el hijo de Bonacci. Fibonacci
fue el más grande matemático de su época; hizo que las matemáticas fueran más accesibles
pues llevó a Europa occidental el sistema de numeración indoarábigo, incluyendo el cero.
En aquella época se hacía referencia al álgebra como Ars Magna o “El gran arte”.
Una tercera persona que también contribuyó de manera importante al álgebra fue Fran-
cois Viete (1540–1603), conocido como “el padre del álgebra moderna”, quien introdujo la
Fibonacci
primera notación algebraica sistemática en su libro In Artem Analyticam. Era un prominente
abogado que también fungió como consejero privado de Enrique iv, para quien descifró men-
sajes en época de guerra. ◆

© Lopez Mateos Editores. ISBN 978-607-95583-2-1, obra completa, versión electrónica, ISBN 978-607-95583-3-8, volumen 1, versión electrónica. Ejemplar asignado a: Helecto Villarroel gutierrez -
helecto@gmail.com. Fecha: 27 de octubre de 2014. Prohibida su modificación, copia o distribución.
Sección 4-1 Variables 197

4-1 Variables

Un aspecto importante del razonamiento algebraico es el concepto de variable. Compren-


der este concepto es fundamental en álgebra. Mientras que en la aritmética básica sólo te-
nemos números fijos, o constantes, como en 4 + 3 = 7, en álgebra también tenemos
cantidades que varían, de ahí el nombre de variable. Sin embargo, variable puede significar
muchas cosas en matemáticas.
Una variable puede hacer las veces de un elemento faltante o desconocido, como en
x + 2 = 5. En este caso, aunque podemos reemplazar la variable en la expresión con cual-
quier número, sólo hay un número que hace que la expresión sea verdadera. Aquí, cuando
reemplazamos la incógnita x con 3 hacemos que la expresión sea verdadera.
En una situación diferente, una variable puede representar más de una cosa. Por ejemplo,
para cierto grupo de niñas podemos decir que su estatura varía con la edad. Si a representa
su estatura y e representa su edad, entonces tanto a como e pueden tener diferentes valores
para diferentes niñas del grupo. Aquí una variable representa una cantidad cambiante.
Según vimos en la sección 1-2, también se pueden usar variables en la generalización de
patrones. Si usáramos los valores específicos en lugar de las variables, las instrucciones se
aplicarían sólo en un número limitado de casos.
Una variable puede ser también un elemento de un conjunto o un conjunto; por ejemplo,
en la definición de la intersección de dos conjuntos A ¨ B = 5x ƒ x 僆 A y x 僆 B6, x es cual-
quier elemento que pertenece a ambos conjuntos.
Para aplicar álgebra en la resolución de problemas necesitamos, con frecuencia, traducir
la información dada a una expresión matemática que incluya variables designadas con letras
o palabras. En todos esos casos podemos denominar las variables como queramos.
Las variables son útiles pues nos permiten especificar instrucciones de manera general.
Por ejemplo, si pedimos a cada estudiante pensar un número, duplicarlo y sumar 1 al resul-
tado, las instrucciones se pueden escribir como 2x + 1. En matemáticas, las letras que más
se usan para variables son x, y y z, pero se puede usar cualquier otra letra del alfabeto latino
o incluso del griego. Si se denomina con x a una variable entonces cada vez que aparece x
en un problema, ecuación o demostración, se refiere a la misma cantidad.
En los ejemplos 4-1 y 4-2, así como en la Página de un libro de texto que les sigue, algunas
expresiones en palabras se traducen en expresiones algebraicas (ver la definición en esa pá-
gina). Nota, en la página de muestra, las expresiones algebraicas para la división. En este ca-
pítulo usaremos el hecho de que la división es la inversa de la multiplicación y viceversa, esto

◆ Nota Mary Everest (1832–1916), quien nació en Inglaterra y creció en Francia, fue una mate-
histórica mática autodidacta y es muy conocida por sus trabajos en matemáticas y en educación cien-
tífica. En 1855, Everest se casó con su amigo y colega matemático George Boole. (El
Monte Everest se nombró así en honor de su tío, Sir George Everest.)
En Philosophy and Fun of Algebra (Filosofía y alegría del álgebra) (London: C. W. Daniel,
LTD, 1909), un libro para niñas y niños, escribió:
Pero cuando terminamos de estudiar aritmética no nos conformamos con estimar o adivinar;
pasamos al álgebra —es decir, a enfrentar lógicamente el hecho de nuestra propia ignorancia. . .
En lugar de llamarle nueve o siete o ciento veinte o mil cincuenta, acordemos llamarle x y recorde-
mos siempre que x representa lo Desconocido. . . . Este método de resolver problemas mediante una
confesión honesta de la propia ignorancia se llama Álgebra. ◆
Mary Everest Boole
© Lopez Mateos Editores. ISBN 978-607-95583-2-1, obra completa, versión electrónica, ISBN 978-607-95583-3-8, volumen 1, versión electrónica. Ejemplar asignado a: Helecto Villarroel gutierrez -
helecto@gmail.com. Fecha: 27 de octubre de 2014. Prohibida su modificación, copia o distribución.
198 Razonamiento algebraico

es, que 1a , b2 # b = a y 1a # b2 , b = a (donde 1b Z 02. Sin embargo, seguiremos la práctica


a#b
= a 1b Z 02.
a a
común de escribir en lugar de a , b; así # b = a y
b b b

Ejemplo 4-1 Escribe cada una de las siguientes proposiciones en forma algebraica:









a. 2 más un número
b. 2 más grande que un número
c. 2 menos que un número
d. 2 veces un número
e. Un número multiplicado por él mismo
f. El costo de rentar un carro cualquier número de días si la renta diaria es de $40
g. La distancia que viajó un carro a una velocidad constante de 65 mph durante cualquier
número de horas
Solución a. n + 2
b. n + 2
c. n - 2
d. 2 # n ó 2n
e. n # n ó n2
f. Si n es el número de días, el costo de rentar el carro durante n días a $40 dia-
rios es de 40 # n ó 40n dólares.
g. Si h es el número de horas de viaje a 60 mph, la distancia total recorrida en
h horas es 60 # h ó 60h millas.

Para aplicar álgebra en la resolución de problemas necesitamos, con frecuencia, traducir
la información dada a una expresión matemática que incluya variables designadas con letras
o palabras. En todos esos casos, podemos denominar las variables como queramos.

Ejemplo 4-2 En las proposiciones siguientes traduce la información dada a expresiones simbólicas, desig-
nando por medio de letras las cantidades implícitas:








a. Un fin de semana una tienda vendió el doble de CD que de DVD y 25 cintas menos que
CD. Si la tienda vendió d DVD, ¿cuántas cintas y cuántos CD vendió?
b. Las papas fritas tienen alrededor de 12 calorías por pieza. Una hamburguesa tiene alre-
dedor de 600 calorías. Arturo sigue una dieta de 2000 calorías al día. Si ya comió p papas
fritas y una hamburguesa, ¿cuántas calorías más puede comer ese día?
Solución a. Como se vendieron d DVD, el doble de CD que de DVD son 2d CD. Así, 25
cintas menos que CD son 2d - 25 cintas.
b. Primero calcula cuántas calorías consumió Arturo al comer p papas fritas y
una hamburguesa. Después halla cuántas calorías puede consumir restando
esta expresión de 2000.
1 papa frita 12 calorías
p papas fritas 12p calorías
Por lo tanto, el número de calorías en p papas fritas y una hamburguesa es
600 + 12p
El número de calorías restantes para ese día es 2000 - 1600 + 12p 2, ó
2000 - 600 - 12p, ó 1400 - 12p.

© Lopez Mateos Editores. ISBN 978-607-95583-2-1, obra completa, versión electrónica, ISBN 978-607-95583-3-8, volumen 1, versión electrónica. Ejemplar asignado a: Helecto Villarroel gutierrez -
helecto@gmail.com. Fecha: 27 de octubre de 2014. Prohibida su modificación, copia o distribución.
Sección 4-1 Variables 199

Página de un libro de texto VA R I A B L ES Y E X P R ES I O N ES

Lección
Calentamiento
´ Variables y Usa símbolos para

expresiones
escribir la expresión.
Idea clave
Se pueden escribir la suma de 16 y 29
relaciones entre
cantidades por Aprende la diferencia entre
medio del álgebra. 216 y 89
¿Cómo puedes escribir una
Vocabulario expresión algebraica?
evaluar
Ejemplo
Rita compró unas velas a $4 cada una. Número de Costo
¿Cómo puedes representar su costo total? velas total

Haz una tabla para mostrar el costo de


diferentes cantidades de velas. Usa una letra,
por ejemplo n, para representar el número de velas.
Como n representa una cantidad cuyo valor
puede variar, se llama variable.

El costo total de las velas se representa con


ó 4n.

Una expresión algebraica es una expresión matemática


que contiene variables, números y símbolos de
operaciones. Antes de escribir una expresión algebraica,
identifica la operación. La tabla a continuación muestra
cómo dos o más frases se pueden referir a una operación.

Frase Operación Expresión algebraica


la suma de 9 y un número n
un número m incrementado en 8
seis más que un número t Suma
añade dieciocho a un número h
setenta y siete más un número r
la diferencia de 12 y un número n
Resta o
siete menos que un número y
substracción
diez disminuido en un número p
el producto de 4 y un número k
quince veces un número t Multiplicación
dos multiplicado por un número m

el cociente de un número dividido entre cinco


División
veinticinco dividido entre un número m

Fuente: Mathematics, Diamond Edition, Grade Six, Scott Foresman-Addison Wesley 2008 ( p. 40).

© Lopez Mateos Editores. ISBN 978-607-95583-2-1, obra completa, versión electrónica, ISBN 978-607-95583-3-8, volumen 1, versión electrónica. Ejemplar asignado a: Helecto Villarroel gutierrez -
helecto@gmail.com. Fecha: 27 de octubre de 2014. Prohibida su modificación, copia o distribución.
200 Razonamiento algebraico

Ejemplo 4-3 Una maestra dijo a sus alumnos:











Tomen cualquier número y súmenle 15. Ahora multipliquen esa suma por 4. A continuación resten
8 y dividan la diferencia entre 4. Ahora resten 12 del cociente y díganme la respuesta. Yo les diré el
número original.

Analiza las instrucciones para ver cómo pudo la maestra determinar el número original.

Solución Traduce la información a una forma algebraica.

Instrucciones Análisis Símbolos

Toma cualquier Como se usa cualquier número, necesitas una


número. variable para representarlo. Sea n dicha
variable. n
Súmale 15. Dice que “le” sumes 15. Por “le” se entiende
la variable n. n + 15
Multiplica esa Dice multiplica “esa suma” por 4. “Esa suma”
suma por 4. es n + 15. 41n + 152
Resta 8. Dice que restes 8 al producto. 41n + 152 - 8
Divide la La diferencia es 41n + 152 - 8. 41n + 152 - 8
diferencia entre 4. Divídela entre 4. 4
Resta 12 del Dice que le restes 12 al cociente. 41n + 152 - 8
cociente y - 12
4
di la respuesta.

Al traducir las instrucciones de la maestra obtenemos la expresión algebraica


41n + 152 - 8
- 12. La maestra también pidió que le dijeran el resultado y ella daría el
4
número original. Usemos la estrategia de trabajar regresivamente para ver si podemos detec-
tar lo sucedido. Supongamos que decimos a la maestra que nuestro resultado final es r.
Piensa ahora cómo se obtuvo r. Antes de decirle “r,” a la maestra, restamos 12. Para revertir
esa operación podemos sumar 12 para obtener r + 12. Antes de eso habíamos dividido en-
tre 4. Para revertir eso podemos multiplicar por 4 y obtener 4r + 48. Para obtener ese re-
sultado habíamos restado 8, así que ahora sumamos 8 para obtener 4r + 56. Pero antes
habíamos multiplicado por 4, de modo que ahora dividimos 4r + 56 entre 4 para obtener
r + 14. La primera operación fue sumar 15, de modo que ahora restamos 15 de r + 14 para
obtener r - 1. Así, la maestra sabe, cuando le decimos que nuestro resultado final es r, que
es 1 más que el número con el que comenzamos, o, dicho de otra manera, que el número n
con que comenzamos es el resultado menos 1.
Podemos mostrarlo de la siguiente manera:

41n + 152 - 8 41n + 15 - 22


- 12 = - 12
4 4
= 1n + 132 - 12
= n + 1

© Lopez Mateos Editores. ISBN 978-607-95583-2-1, obra completa, versión electrónica, ISBN 978-607-95583-3-8, volumen 1, versión electrónica. Ejemplar asignado a: Helecto Villarroel gutierrez -
helecto@gmail.com. Fecha: 27 de octubre de 2014. Prohibida su modificación, copia o distribución.
Sección 4-1 Variables 201

Ejemplo 4-4 La figura 4-1 muestra una sucesión de figuras que contienen pequeños mosaicos cuadrados.
Algunos están sombreados. Nota que la primera figura tiene un mosaico sombreado. La se-









gunda figura tiene 2 # 2 ó 22 mosaicos sombreados. La tercera figura tiene 3 # 3 ó 32 mosai-
cos sombreados. Responde las preguntas siguientes:
a. ¿Cuántos mosaicos sombreados hay en la figura n-ésima?
b. ¿Cuántos mosaicos blancos hay en la figura n-ésima?

Primera
Segunda
Tercera
Figura 4-1
Solución a. Los cuadros formados por mosaicos sombreados tienen lados de longitud cre-
ciente 1, 2, 3 y así sucesivamente. En la figura n-ésima, la longitud de un lado
de la región sombreada será n. Por lo tanto, la figura n-ésima tiene n2 mosai-
cos sombreados.
b. Una manera de considerar el número de mosaicos blancos es percatarse de
que el número de mosaicos blancos en un lado es 2 más que n, ó n + 2. El nú-
mero de mosaicos blancos sería 4 por 1n + 22, menos los que se traslapen. En
este caso cada mosaico de la esquina se contó dos veces, de modo que hay 4
mosaicos blancos que se repitieron; esto nos da 41n + 22 - 4, ó 4n + 4, mo-
saicos blancos.
Otra manera de contar los mosaicos blancos en la n-ésima figura es contar
el número total de mosaicos y restar de este total el número de mosaicos som-
breados. Ya vimos que el número de mosaicos blancos en el lado inferior del
n-ésimo cuadro es n + 2 y el número de mosaicos sombreados en un lado es
n. Así, el número de mosaicos blancos es 1n + 222 - n2. Se puede demostrar
que esta respuesta es igual a 4n + 4 obtenida antes.

AHORA INTENTA ÉSTE 4-1


a. Hay otra manera de contar los mosaicos del ejemplo 4-4. Primero elimina los cuatro mosaicos blancos
de las esquinas y después cuenta el número de mosaicos blancos restantes. Completa el enfoque.
b. Noe tiene algunos mosaicos blancos y algunos azules. Son del mismo tamaño. Primero forma una fila
de mosaicos blancos y después la rodea con una sola capa de mosaicos azules, como se muestra en la
figura 4-2.

Figura 4-2
¿Cuántos mosaicos azules necesita para:
i. rodear una fila de 100 mosaicos blancos?
ii. rodear una fila de n mosaicos blancos?

Es común usar variables en hojas de cálculo. Tomaría mucho tiempo calcular a mano, o
incluso con una calculadora, el término 50-ésimo de la sucesión de Fibonacci,

© Lopez Mateos Editores. ISBN 978-607-95583-2-1, obra completa, versión electrónica, ISBN 978-607-95583-3-8, volumen 1, versión electrónica. Ejemplar asignado a: Helecto Villarroel gutierrez -
helecto@gmail.com. Fecha: 27 de octubre de 2014. Prohibida su modificación, copia o distribución.
202 Razonamiento algebraico

1, 1, 2, 3, 5, 8, 13, Á , en donde los primeros dos términos son 1, 1 y cada término subse-
cuente es la suma de los dos términos precedentes. Sin embargo, usando una hoja de cál-
culo aparece de manera instantánea cualquier término deseado de la sucesión de Fibanocci
y los términos anteriores. En la Página de un libro de texto se muestra cómo crear la suce-
sión de Fibonacci en una hoja de cálculo usando dos variables, A1 y A2.

Página de un libro de texto APRENDIZAJE CON TECNOLO G Í A

Aprendizaje con tecnología


Herramienta Hoja de cálculo/Datos/Graficador: Generación de una sucesión
Hace casi 800 años, un matemático italiano llamado Leonardo Fibonacci descubrió esta
sucesión de números: 1, 1, 2, 3, 5, 8, 13, 21, 34, 55, 89,144, 233, 377, 610,...
Comenzando con el número 1, cada número de la
sucesión es la suma de los dos números anteriores.

y así sucesivamente.
Crea una hoja de cálculo que genere los primeros 32 términos
de la sucesión de Fibonacci. Copia la fórmula en la celda A5 a
las celdas A6-A35 y la B6 a las celdas B7-B35.
¿Crees que hay un número
infinito de términos en la
sucesión? Explica.
Cambia el número de la celda La sucesión de Fibonacci 1er número 2º número

C2 a 3. ¿Qué sucede con los Término Número de Fibonacci


números en la columna B?
¿Cómo depende la sucesión
de los números en las celdas
B2 y C2?

Fuente: Mathematics, Diamond Edition, Grade Six, Scott Foresman-Addison Wesley 2008 ( p. 163).

El razonamiento algebraico se presenta de muchas maneras. Vemos a continuación


un ejemplo que usa figuras.

Ejemplo 4-5 En el mercado campesino local los precios se ven como se indica en la figura 4-3. ¿Cuál es
el costo de cada objeto?








$8 $9 $7

(a) (b) (c)

Figura 4-3
© Lopez Mateos Editores. ISBN 978-607-95583-2-1, obra completa, versión electrónica, ISBN 978-607-95583-3-8, volumen 1, versión electrónica. Ejemplar asignado a: Helecto Villarroel gutierrez -
helecto@gmail.com. Fecha: 27 de octubre de 2014. Prohibida su modificación, copia o distribución.
Sección 4-1 Variables 203

Solución  Los enfoques sobre este problema pueden variar. Por ejemplo, si se juntan los ob-
jetos correspondientes a los dos primeros precios, el costo total sería de $8 + $9, ó $17.
Ese costo sería de dos floreros, un melón y una sandía, como se indica en la figura 4-4.

$17 $7

Figura 4-4

Ahora, si se apartan del total el melón y la sandía, de acuerdo con el costo de estos dos
objetos, que aparece del lado derecho, nos quedaría que los dos floreros cuestan $10. Esto
significa que cada uno de los floreros cuesta $5. Esto, a su vez, nos dice que el melón
cuesta $8 - $5, ó $3, y que la sandía cuesta $9 - $5, ó $4.

La solución al ejemplo 4-5 podría incluir la estrategia de plantear una ecuación. Pero primero
necesitamos tener un conocimiento básico de cómo resolver ecuaciones.

Evaluación 4-1A

1. Escribe cada una de las proposiciones siguientes en 4. En los casos siguientes, escribe una expresión en térmi-
forma algebraica: nos de la variable dada que represente la cantidad indi-
a. El tercer término de una sucesión aritmética cuyo cada. Por ejemplo, la distancia recorrida a una rapidez
primer término es 10 y cuya diferencia es d constante de 60 kph durante t horas puede escribirse
b. 10 menos que el doble de un número como 60t kilómetros.
c. 10 veces el cuadrado de un número a. El costo de tener un plomero trabajando en tu casa h
d. La diferencia entre el cuadrado de un número y el horas, si el plomero cobra $200 por venir a tu casa y
doble del número $250 por hora de trabajo.
2. a. Traduce la información siguiente a forma algebraica: b. La cantidad de dinero en pesos en una jarra que con-
Toma cualquier número, súmale 3, multiplica la suma tiene c monedas de cinco y algunas de diez y de vein-
por 7, resta 14 y divide la diferencia entre 7. Final- te, si hay tres veces más monedas de diez que de cinco
mente, resta el número original. y dos veces más monedas de veinte que de diez.
b. Simplifica tu respuesta de la parte (a). c. La suma de tres enteros consecutivos si el menor en-
3. En el patrón de mosaicos de la sucesión de figuras mos- tero es x.
trada, cada figura, a partir de la segunda, tiene dos cua- d. La cantidad de bacterias después de n minutos si la
drados azules más que la anterior. Responde lo siguiente: cantidad inicial de bacterias es q y se duplica cada mi-
nuto. (Sugerencia: La respuesta debe contener q y n.)
e. La temperatura después de t horas si la temperatura
inicial es de 40°F y disminuye cada hora en 3°F.
f. El salario total de Pablo después de 3 años si el pri-
mer año de salario fue de s dólares, el segundo año
, , , ... fue de $5000 más y el tercer año fue el doble del se-
Primero Segundo Tercero Cuarto gundo año.
a. ¿Cuántos mosaicos azules hay en la n-ésima figura?
b. ¿Cuántos mosaicos blancos hay en la n-ésima figura?
© Lopez Mateos Editores. ISBN 978-607-95583-2-1, obra completa, versión electrónica, ISBN 978-607-95583-3-8, volumen 1, versión electrónica. Ejemplar asignado a: Helecto Villarroel gutierrez -
helecto@gmail.com. Fecha: 27 de octubre de 2014. Prohibida su modificación, copia o distribución.
204 Razonamiento algebraico

g. La suma de tres números naturales impares consecu- 8. Escribe una ecuación algebraica que relacione las variables
tivos si el menor es x. descritas en cada una de las situaciones siguientes:
h. La suma de tres números naturales consecutivos si el a. La paga, P, por t horas si te pagan a $8 la hora.
de en medio es m. b. La paga, P, por t horas si te pagan $15 por la primera
5. Si el número de profesores en una escuela es P y el nú- hora y $10 por cada hora adicional.
mero de estudiantes es E, y hay 20 veces más estudiantes 9. Para cierto evento, los estudiantes pagan $5 por boleto
que profesores, escribe una ecuación algebraica que y los no estudiantes pagan $13 por boleto. Si x estu-
muestre esta relación. diantes y 100 no estudiantes compran boletos, halla el
6. Si m es el número de mujeres en una clase y v es el ingreso total por la venta de los boletos en términos de
número de varones y si hay cinco mujeres (m) más que x.
varones (v) en una clase, escribe una ecuación algebraica 10. Supón que un testamento decreta que tres hermanas
que muestre esta relación. reciban una herencia en efectivo de acuerdo con lo si-
7. Ricardo está construyendo sucesiones de cuadrados con guiente: la mayor recibe 3 veces lo que la menor y dos
cerillos. ¿Cuántos cerillos usará para la n-ésima figura? veces más que la hermana de en medio. Responde lo si-
guiente:
a. Si la hermana menor recibe $x, ¿cuánto reciben las
otras dos en términos de x?
b. Si la hermana de en medio recibe $y, ¿cuánto reciben
las otras dos en términos de y?
c. Si la hermana mayor recibe $z, ¿cuánto reciben las
otras dos en términos de z?
, , ,...

Evaluación 4-1B

1. Escribe cada una de las proposiciones siguientes en for- 5. Supón que en un salón de clase hay 15 sillas (s) más que
ma algebraica: mesas (m). Escribe una ecuación algebraica que relacione s
a. 10 más que un número y m.
b. 10 menos que un número 6. Para cada caso, escribe una expresión en términos de las
c. 10 veces un número variables dadas que represente la cantidad indicada:
d. La suma de un número y 10 a. El costo de tener un plomero trabajando en tu casa h
e. La diferencia entre el cuadrado de un número y el horas, si el plomero cobra $300 por venir a tu casa y
número $x por hora de trabajo.
2. Traduce lo siguiente a forma algebraica: b. La cantidad de dinero en pesos en una jarra que con-
a. Toma cualquier número, súmale 25, multiplica la tiene algunas monedas de cinco, d de diez y algunas de
suma por 3, resta 60 y divide la diferencia entre 3. Fi- veinte si hay 4 veces más de cinco que de diez y el do-
nalmente, suma 5. ble de veinte que de cinco.
b. Simplifica tu respuesta de la parte (a). c. La suma de tres enteros consecutivos si el mayor en-
3. Descubre un posible patrón de mosaicos en la sucesión tero es x.
siguiente y contesta las preguntas: d. La cantidad de bacterias después de n min si la canti-
dad inicial de bacterias es q y se triplica cada 30 s.
(Sugerencia: La respuesta debe contener a q y a n.)
e. La temperatura hace t horas si la temperatura actual
es de 40°F y cada hora ha bajado 3°F.
f. El salario total de Pablo después de 3 años si el pri-
, , ... mer año su salario fue de s dólares, el segundo año
Primero Segundo Tercero
fue de $5000 más y el tercer año fue el doble del pri-
a. ¿Cuántos mosaicos sombreados hay en la n-ésima mer año.
figura de tu patrón? g. La suma de tres números completos pares consecuti-
b. ¿Cuántos mosaicos blancos hay en la n-ésima figura vos si el mayor es x.
de tu patrón?
4. Si en una escuela hay m mujeres y v varones y sabes que
hay 100 varones más que mujeres, escribe una ecuación
algebraica que relacione m y v.
© Lopez Mateos Editores. ISBN 978-607-95583-2-1, obra completa, versión electrónica, ISBN 978-607-95583-3-8, volumen 1, versión electrónica. Ejemplar asignado a: Helecto Villarroel gutierrez -
helecto@gmail.com. Fecha: 27 de octubre de 2014. Prohibida su modificación, copia o distribución.
Sección 4-1 Variables 205

7. Ricardo está construyendo sucesiones de cuadrados con d. El costo total, C, de la membresía de un club que co-
cerillos de manera que añade un cuadrado a la derecha bra $300 de inscripción y $4 por cada uno de los n
cada vez, según se muestra. ¿Cuántos cerillos usará para días que asististe.
la n-ésima figura y para la figura anterior a la n-ésima? e. El costo, C, de rentar un carro mediano por 1 día reco-
rriendo m millas si la renta es de $30 por día más 35¢
por milla.
, , , 9. Una maestra dio las siguientes instrucciones a sus alumnos:
Tomen cualquier número impar, multiplíquenlo por 4, su-
8. Escribe una ecuación algebraica que relacione las variables men 16 y dividan el resultado entre 2. Resten 7 del cociente
descritas en cada una de las situaciones siguientes: y díganme el resultado. Les diré el número original.
a. La paga, P, por t horas si te pagan a $ d la hora. Explica cómo pudo la maestra decir a cada alumno su
b. La paga, P, por t horas si te pagan $15 por la primera número original.
hora y $k por cada hora adicional.
c. La paga total, P, por una visita de t horas de jardine- 10. Mati tiene el doble de palillos que David. Si David tiene
ría si te pagan $20 por la visita y $10 por cada hora de d palillos y Mati m palillos, y Mati da a David 10 palillos,
jardinería. ¿cuántos palillos tiene cada uno en términos de d ?

Conexiones matemáticas 4-1

Comunicación 6. Un estudiante escribe a # 1b # c2 = 1a # b2 # 1a # c2. ¿Qué le


1. Se pidió a los alumnos escribir una expresión algebraica dices?
para la suma de tres números naturales consecutivos. 7. Una alumna pregunta si es posible considerar conjuntos
Uno escribió x + 1x + 12 + 1x + 22 = 3x + 3. Otro como variables. ¿Qué le dices?
escribió 1x - 12 + x + 1x + 12 = 3x. Explica quién
8. Un estudiante piensa que si A y B son conjuntos, entonces
las proposiciones A ´ B = B ´ A y A ¨ B = B ¨ A son
tiene razón y por qué.
generalizaciones algebraicas de las propiedades de los
Solución abierta conjuntos, de manera análoga a que las proposiciones
2. Una maestra dio instrucciones a sus alumnos para tomar a + b = b + a y ab = ba son generalizaciones de las pro-
cualquier número y realizar una serie de cálculos usando piedades aritméticas de los números. ¿Qué le respondes?
ese número. La maestra pudo decir a cada estudiante cuál Pregunta del Third International Mathematics and
era su número original restando 1 de la respuesta que daba Science Study (TIMSS) (Tercer estudio internacional
el estudiante. Crea instrucciones similares para los estu- sobre las matemáticas y la ciencia)
diantes de manera que la maestra sólo haga lo siguiente
para obtener el número original del estudiante: representa el número de revistas que Lina lee cada se-
a. Sumar 1 a la respuesta. mana. ¿Cuál de los siguientes representa el número de re-
b. Multiplicar la respuesta por 2. vistas que Lina lee en 6 semanas?
c. Multiplicar la respuesta por 1. a. 6 + n
3. Crea instrucciones parecidas a las del problema 2 que in- b. 6 * n
cluyan suma, resta, multiplicación y división, de manera c. n + 6
que siguiendo las instrucciones cada estudiante obtenga d. 1n + n2 * 6
su número original. TIMSS 2003, Grado 4
Aprendizaje colectivo Pregunta del National Assessment of Educational Progress
4. Examina varios libros de texto para los grados 1 a 5 y haz (NAEP) (Evaluación Nacional del Progreso Educativo)
un reporte acerca de qué conceptos algebraicos que inclu- N representa el número de horas que Juan duerme cada
yan variables se introducen en cada uno.
noche. ¿Cuál de los siguientes representa el número de
Preguntas del salón de clase horas que duerme Juan en una semana?
5. Un estudiante afirma que la suma de cinco enteros conse- a. N + 7
cutivos es igual a 5 por el entero de en medio y quisiera b. N - 7
saber si esto siempre es cierto y por qué. Él quisiera saber c. N * 7
si la proposición se generaliza a la suma de cinco términos d. N , 7
consecutivos de cualquier sucesión aritmética. ¿Cómo le NAEP, Grado 4, 2005
respondes?

© Lopez Mateos Editores. ISBN 978-607-95583-2-1, obra completa, versión electrónica, ISBN 978-607-95583-3-8, volumen 1, versión electrónica. Ejemplar asignado a: Helecto Villarroel gutierrez -
helecto@gmail.com. Fecha: 27 de octubre de 2014. Prohibida su modificación, copia o distribución.
206 Razonamiento algebraico

4-2 Ecuaciones

Las variables con frecuencia se asocian con ecuaciones. Cuando las variables se piensan
como incógnitas, podemos considerar ecuaciones como w + c = 7. El signo igual indica
que los valores en ambos lados de la ecuación son los mismos aunque se vean de manera di-
ferente. Como se señala en la Nota de investigación, a veces los estudiantes piensan, de ma-
nera errónea, que el signo de igual es sólo un símbolo de separación.
Para resolver ecuaciones necesitamos varias propiedades de la igualdad. Las niñas y niños
descubren muchas de ellas por medio de una balanza. Por ejemplo, considera dos pesos de
magnitudes a y b colocados en la balanza, como en la figura 4-5(a). Si la balanza está nive-
lada, entonces a = b. Cuando añadimos la misma cantidad de peso, c, en ambos lados de la
balanza, la balanza sigue nivelada, como en la figura 4-5(b).

◆ Nota de a b a c b c
investigación
Van de Walle
escribe que los estu- (a) a=b (b) a+c=b+c
diantes tienden a ver Figura 4-5
una ecuación como
3x + 7 = 5 + 9
Esto ilustra que si a = b, entonces a + c = b + c, que es la propiedad de suma de la igualdad.
como algo que tiene
De manera análoga, si la balanza está nivelada con las cantidades a y b, como en la figura
dos lados separados
4-6(a), y colocamos pesas adicionales de magnitud a en un lado de la balanza e igual número
con cosas que hacer,
de pesas adicionales de magnitud b en el otro lado, la balanza permanece nivelada, como en
en lugar de dos
la figura 4-6(b).
nombres para la
misma cosa. Con
frecuencia el a bb
a b aaaa bbb
símbolo de igual se
ve como un símbolo
usado para separar
un problema de su (a) a=b (b) 5a = 5b

respuesta (Van de Figura 4-6


Walle 2007). ◆
La figura 4-6 sugiere que si c es cualquier número natural y a = b, entonces ac = bc, que es la
propiedad de la multiplicación de la igualdad. A continuación se resumen estas propiedades, que
son válidas para todos los números, aunque en este capítulo las usamos sólo para números
completos.

O B S E R VA C I Ó N
a. En álgebra es común omitir el signo de multiplicación en un producto que incluya
letras. Así, escribimos ac en lugar de a # c y 3x en lugar de 3 # x .
b. En ciertas condiciones, las propiedades listadas aquí se pueden demostrar y por lo
tanto se llaman teoremas.

© Lopez Mateos Editores. ISBN 978-607-95583-2-1, obra completa, versión electrónica, ISBN 978-607-95583-3-8, volumen 1, versión electrónica. Ejemplar asignado a: Helecto Villarroel gutierrez -
helecto@gmail.com. Fecha: 27 de octubre de 2014. Prohibida su modificación, copia o distribución.
Sección 4-2 Ecuaciones 207

Teorema 4–1: Propiedad de la igualdad para la suma


Para cualesquier números a, b y c, si a = b, entonces a + c = b + c.
Propiedad de la igualdad para la multiplicación  Para cualesquier números a, b y c, si
a = b, entonces ac = bc.

Las propiedades implican que podemos sumar el mismo número en ambos lados de una
ecuación, o multiplicar ambos lados de una ecuación por un mismo número sin afectar la
igualdad. Si invertimos el orden de las partes correspondientes al si y al entonces en las pro-
piedades de la igualdad, obtenemos nuevas afirmaciones. La nueva afirmación se llama recí-
proca de la afirmación original. En el caso de la propiedad aditiva, la recíproca es una
afirmación verdadera. La recíproca de la propiedad multiplicativa también es verdadera
cuando c Z 0. A continuación resumimos estas propiedades.

Teorema 4–2:    Propiedades de cancelación para la igualdad


1. Para cualesquier números a, b y c, si a + c = b + c, entonces a = b.
2. Para cualesquier números a, b y c, con c Z 0, si ac = bc, entonces a = b.

O B S E R VA C I Ó N Nota que si dividimos entre c ambos lados de la ecuación ac = bc, ob-


tenemos a = b.
La igualdad no se afecta si substituimos un número por su igual. Nos referiremos a esta
propiedad como la de substitución. A continuación vemos ejemplos de substitución:
1. Si a + b = c + d y d = 5, entonces a + b = c + 5.
2. Si a + b = c + d, b = e y d = f, entonces a + e = c + f.
Con la propiedad de substitución vemos que podemos sumar o restar ecuaciones “lado a
lado”; esto es, tenemos lo siguiente:

Teorema 4–3:    Propiedades de suma y resta para las ecuaciones


Si a = b y c = d, entonces a + c = b + d y a - c = b - d.

Esta propiedad se puede justificar así: usando la propiedad de la suma para la igualdad, si
a = b, entonces a + c = b + c. Substituyendo d en lugar de c en el lado derecho, obtenemos
a + c = b + d. De manera análoga, a - c = b - d.
En los primeros grados se usan ampliamente las propiedades conmutativa y asociativa de
la suma y la multiplicación, que se pueden realizar en cualquier orden, por ejemplo
2 + 8 = 8 + 2 y 2 # 8 = 8 # 2. También, 2 + 18 + 52 = 12 + 82 + 5 y, en general, tene-
mos lo siguiente:

Teorema 4–4:    Propiedades conmutativas para la suma y la multiplicación


a + b = b + a, ab = ba

Teorema 4–5:    Propiedades asociativas para la suma y la multiplicación


1a + b2 + c = a + 1b + c2, 1ab2c = a1bc2

En la Página de un libro de texto que sigue verás las dos propiedades recién presentadas.

© Lopez Mateos Editores. ISBN 978-607-95583-2-1, obra completa, versión electrónica, ISBN 978-607-95583-3-8, volumen 1, versión electrónica. Ejemplar asignado a: Helecto Villarroel gutierrez -
helecto@gmail.com. Fecha: 27 de octubre de 2014. Prohibida su modificación, copia o distribución.
208 Razonamiento algebraico

Página de un libro de texto PATRONES DE MULTIPLICACIÓN

Lección
Calentamiento
Idea clave
Puedes usar tablas, Patrones de multiplicación
patrones y propiedades
para multiplicar Aprende
mentalmente.

Actividad
Vocabulario
factor ¿Cuál es el patrón?
producto
propiedad Usa una calculadora para hallar cada producto. Los factores son
conmutativa de la números que se
multiplicación multiplican para obtener
propiedad un producto.
asociativa de la
multiplicación
Halla los productos siguientes sin calculadora.
Después verifica tus resultados con una calculadora.
Materiales
calculadora

Describe una regla que diga cómo hallar cada producto.

ex
ef l i ó ¿Cómo te pueden ayudar las propiedades para
multiplicar más fácilmente?
R

Propiedad conmutativa para Propiedad asociativa para la


¡Piensa! la multiplicación multiplicación
Puedo buscar un
patrón para hallar Puedes cambiar el orden Puedes cambiar el agrupamiento
una regla. de los factores. de los factores.

Ejemplo Ejemplo
Halla Halla
Usando la propiedad asociativa Usa las propiedades para cambiar
puedes pensar que: el orden y los agrupamientos.
ó

Tema de plática
¿Cómo se usa la propiedad asociativa en el ejemplo B?
¿Cómo se usa la propiedad conmutativa en el ejemplo B?
¿Puedes usar la propiedad asociativa para 2 3 (5 + 6)? Explica.

Fuente: Mathematics, Diamond Edition, Grade Five, Scott Foresman-Addison Wesley 2008 ( p. 66).

© Lopez Mateos Editores. ISBN 978-607-95583-2-1, obra completa, versión electrónica, ISBN 978-607-95583-3-8, volumen 1, versión electrónica. Ejemplar asignado a: Helecto Villarroel gutierrez -
helecto@gmail.com. Fecha: 27 de octubre de 2014. Prohibida su modificación, copia o distribución.
Sección 4-2 Ecuaciones 209

AHORA INTENTA ÉSTE 4-2 Responde las tres preguntas al final de la Página de un libro de texto.

Una propiedad vital para resolver ecuaciones —y, en general, para el razonamiento alge-
braico— es la propiedad distributiva de la multiplicación sobre la suma. Esta propiedad se usa en
los primeros grados en problemas de multiplicación tales como hallar el producto 12 # 65.
Si pensamos el producto como el número de plantas en 12 filas de 65 plantas cada una, ese
número es igual al número de plantas en 10 + 2 filas, o el número en 10 filas más el número
en 2 filas, ó 10 # 65 + 2 # 65. Así, 12 # 65 = 110 + 2265 = 10 # 65 + 2 # 65.

Teorema 4–6:    Propiedad distributiva de la multiplicación sobre la suma


Para cualesquier números a, b y c, a1b + c2 = ab + ac.

Nota que usamos esta propiedad en la solución del ejemplo 4-3, cuando escribimos
41n + 152 como 4n + 4 # 15. Nota también la propiedad análoga entre conjuntos:
A ¨ 1B ´ C2 = 1A ¨ B2 ´ 1A ¨ C2. De manera análoga, tenemos

Teorema 4–7:    Propiedad distributiva de la multiplicación sobre la resta


Para cualesquier números a, b y c, a1b - c2 = ab - ac.

Nota que por medio de la propiedad conmutativa de la multiplicación, cada una de las
propiedades distributivas anteriores se puede escribir en las formas equivalentes
1b + c2a = ba + ca y
1b - c2a = ba - ca.
Cuando las propiedades distributivas se escriben de derecha a izquierda, nos referimos a
ellas como factorización. Así, ab + ac = a1b + c2 y ab - ac = a1b - c2. Decimos entonces
que a se ha “factorizado”.

Resolución de ecuaciones
Parte del razonamiento algebraico incluye operaciones con números y otros elementos
representados por símbolos. Hallar soluciones de ecuaciones es una parte importante del
álgebra. Como se señala en la Nota de investigación, el uso de objetos tangibles puede incre-
mentar la atención y comprensión de los estudiantes cuando trabajan con ecuaciones. El
modelo de la balanza es una excelente ayuda para entender los conceptos básicos usados
para resolver ecuaciones y desigualdades, y las ecuaciones se pueden explorar con una ba-
lanza nivelada. Las desigualdades inclinan la balanza.

◆ Nota de El uso de objetos y de modelos a escala para resolver ecuaciones produce una ganancia im-
investigación portante y una mejor actitud al resolver ecuaciones. (Quinlan 1992). ◆

© Lopez Mateos Editores. ISBN 978-607-95583-2-1, obra completa, versión electrónica, ISBN 978-607-95583-3-8, volumen 1, versión electrónica. Ejemplar asignado a: Helecto Villarroel gutierrez -
helecto@gmail.com. Fecha: 27 de octubre de 2014. Prohibida su modificación, copia o distribución.
210 Razonamiento algebraico

Por ejemplo, consideremos la figura 4-7. ¿Qué sucederá si soltamos el lado izquierdo de la
balanza? Al soltarla la balanza se inclinará hacia la derecha y tendremos una desigualdad,
2 # 3 6 3 + 12 # 22.

2 3 3 1 (2 2)

Figura 4-7

A continuación, considera la figura 4-8. Si soltamos la balanza, entonces los lados se nivela-
rán y tendremos la igualdad 2 # 3 = 11 + 12 + 4.

2 3 (1 1 1) 1 4

Figura 4-8

También se puede usar una balanza para reforzar la idea de reemplazo usada para las varia-
bles. Menciona algunas soluciones en la figura 4-9 que mantengan la balanza nivelada. Por
ejemplo, 3 # 2 se nivela con 2 # 3, 3 # 6 se nivela con 2 # 9 y así sucesivamente. ¿Ves algún pa-
trón en los números que nivelan la balanza?

3 2

Figura 4-9

Otros tipos de problemas de balanza pueden ayudar a los estudiantes a prepararse para el
álgebra. Antes de continuar, trabaja con Ahora intenta éste 4-3.

© Lopez Mateos Editores. ISBN 978-607-95583-2-1, obra completa, versión electrónica, ISBN 978-607-95583-3-8, volumen 1, versión electrónica. Ejemplar asignado a: Helecto Villarroel gutierrez -
helecto@gmail.com. Fecha: 27 de octubre de 2014. Prohibida su modificación, copia o distribución.
Sección 4-2 Ecuaciones 211

AHORA INTENTA ÉSTE 4-3 ¿Cuáles son los valores de n y ^ en la figura 4-10?
a.

y 18

Figura 4-10(a)
b.

8 y 12

Figura 4-10(b)

Para resolver ecuaciones podemos usar las propiedades de la igualdad desarrolladas an-
teriormente. Considera 3x - 14 = 1. Coloca las expresiones iguales en los platillos
opuestos de la balanza. Como las expresiones son iguales la balanza debe estar nivelada,
como en la figura 4-11.

3 x – 14 1

Figura 4-11

Para despejar x usamos las propiedades de la igualdad para manipular las expresiones en la
balanza de manera que después de cada paso ésta permanezca nivelada y, en el paso final, per-
manezca sólo x en un lado de la balanza. El número en el otro platillo de la balanza representa
la solución a la ecuación original. Para hallar x en la ecuación de la figura 4-11, considera las
balanzas dibujadas en pasos sucesivos en la figura 4-12. En la figura 4-12 cada balanza sucesiva
representa una ecuación que es equivalente a la ecuación original, esto es, cada una tiene la
misma solución que la original. La última balanza muestra que x = 5. Para verificar que 5 sea
la solución correcta, substituimos x por 5 en la ecuación original. Como 3 # 5 - 14 = 1 es una
afirmación verdadera, 5 es la solución de la ecuación original.

Suma 14 a Divide cada


cada lado lado entre 3

3x – 14 1 3x – 14 + 14 1 + 14 3x 15 x 5
(a) (b) (c) (d)

Propiedad de la Propiedad de la
igualdad de la suma cancelación de la igualdad

Figura 4-12

© Lopez Mateos Editores. ISBN 978-607-95583-2-1, obra completa, versión electrónica, ISBN 978-607-95583-3-8, volumen 1, versión electrónica. Ejemplar asignado a: Helecto Villarroel gutierrez -
helecto@gmail.com. Fecha: 27 de octubre de 2014. Prohibida su modificación, copia o distribución.
212 Razonamiento algebraico

AHORA INTENTA ÉSTE 4-4 Nota el uso de objetos concretos para resolver ecuaciones en la Página de
un libro de texto y responde la pregunta del “Tema de plática” que sigue al modelo de la balanza.

Ejemplo 4-6 Despeja x en las siguientes ecuaciones:










a. x + 4 = 20
b. 3x = x + 10
c. 4x + 5x = 99
d. 41x + 32 + 51x + 32 = 99

Solución a. x + 4 = 20 implica que 1x + 42 - 4 = 20 - 4; x = 16.


b. 3x = x + 10 implica que 3x - x = 10 + x - x; 3 # x - 1 # x = 10;
13 - 12x = 10; 2x = 10; x = 5.
c. 4x + 5x = 99; 14 + 52x = 99; 9x = 99; x = 11
d. Podemos multiplicar y obtener
41x + 32 + 51x + 32 = 99; 4x + 12 + 5x + 15 = 99
9x + 27 = 99; 9x + 27 - 27 = 99 - 27; 9x = 72, x = 8
O pudimos pensar x + 3 como una nueva incógnita n. De modo que si
x + 3 = n , obtenemos 4 # n + 5 # n = 99, así que 9 # n = 99, n = 11, lo
cual implica que x + 3 = 11 y, por lo tanto, x = 8.

◆ Nota Nacida dentro de una familia acomodada de Escocia, Mary Fairfax (1780–1872) estudió
histórica primero aritmética elemental superficialmente a la edad de 13 años. Por esa época se topó
con unos símbolos misteriosos en una revista femenina de modas y, después de persuadir
al tutor de su hermano de que le comprara algún libro básico sobre el tema, comenzó a es-
tudiar álgebra. Más adelante, como madre y viuda joven, adquirió una pequeña biblioteca
con material que le proporcionaría una base sólida en matemáticas. Por el resto de su vida,
Somerville se distinguió como una hábil escritora científica respetada por sus colegas, y
publicó numerosos trabajos. Su último libro científico, Molecular and Microscopic Science
(Ciencia molecular y microscópica), fue publicado en 1869, cuando ella tenía 89 años. En
su autobiografía Mary escribió acerca de cómo “a veces me molestaba cuando en medio de
Mary Fairfax Somerville un problema difícil” llegaba un visitante. Poco antes de su muerte escribió:
Tengo ahora noventa y dos años… estoy extremadamente sorda y mi memoria me falla para even-
tos ordinarios y especialmente para nombres de personas, pero no para temas matemáticos y
científicos. Todavía puedo leer libros de álgebra superior durante cuatro o cinco horas por la ma-
ñana e incluso resolver los problemas. A veces los encuentro difíciles pero sigo igual de obstinada
que siempre, así que si no los resuelvo hoy, los atacaré de nuevo mañana. ◆

© Lopez Mateos Editores. ISBN 978-607-95583-2-1, obra completa, versión electrónica, ISBN 978-607-95583-3-8, volumen 1, versión electrónica. Ejemplar asignado a: Helecto Villarroel gutierrez -
helecto@gmail.com. Fecha: 27 de octubre de 2014. Prohibida su modificación, copia o distribución.
Sección 4-2 Ecuaciones 213

Página de un libro de texto RESOLUCIÓN DE ECUACIONES


CO N N Ú M E ROS COM P L E TOS

Lección
Calentamiento
´ Resolución de ecuaciones con Explica cómo despejar la
Idea clave números completos variable.
Puedes usar operaciones
inversas y las
propiedades de la Aprende
igualdad para resolver
ecuaciones. ¿Cómo puedes resolver una ecuación?
Cuando resuelves una ecuación, hallas el valor de la variable
Vocabulario
ecuación (p.44) que hace que la ecuación sea verdadera.
operaciones
inversas (p.45) Ejemplo
propiedades de
la igualdad (p.44) Güicho vendió 6 dibujos, cada uno por la misma cantidad, y obtuvo
un total de $180 por la venta. ¿Cuánto cobró por cada dibujo?
Sea s igual a la cantidad pagada por cada dibujo.

ex Entonces la ecuación es 6s=180.


ef l i ó
R

Lo que escribes Nivelando la balanza


n

¡Piensa!
Puedo pensar en La balanza está
una balanza como nivelada.
ayuda para resolver
el problema.

Se ha separado
180 en 6
partes iguales.

Cada
es igual a

Güicho cobró $30 por cada dibujo.

Tema de plática
¿Por qué se dividió entre 6 cada lado de la ecuación del ejemplo A?

¿Cómo puedes verificar tu respuesta?


Para verificar tu respuesta, substitúyela por la variable en la ecuación
original. En el ejemplo A substituye s por 30 en 6s=180.
Verifica:

Cuando al simplificar ambos lados de la ecuación obtienes


el mismo número, el valor de la variable es correcto.

Fuente: Mathematics, Diamond Edition, Grade 6, Scott Foresman-Addison Wesley 2008 ( p. 48).

© Lopez Mateos Editores. ISBN 978-607-95583-2-1, obra completa, versión electrónica, ISBN 978-607-95583-3-8, volumen 1, versión electrónica. Ejemplar asignado a: Helecto Villarroel gutierrez -
helecto@gmail.com. Fecha: 27 de octubre de 2014. Prohibida su modificación, copia o distribución.
214 Razonamiento algebraico

Problemas de aplicación
El sencillo modelo de la figura 4-13 muestra un método para resolver problemas de aplica-
ción. Formula el problema como un modelo matemático, resuelve el modelo matemático y
después interpreta la solución en términos del problema original.

Problema de aplicación → Modelo matemático



Interpretación del problema original ← Solución matemática

Figura 4-13

En la figura 4-14 aparece un ejemplo de este modelo para tercer grado.

Problema de aplicación Modelo matemático

Amelia ganó $120 de niñera y $50 lavando → 120 + 50 = ?


el carro. ¿Cuánto ganó en total?

Interpretación del problema original Solución matemática

Las ganancias de Amelia suman $170. 120 + 50 = 170

Figura 4-14

AHORA INTENTA ÉSTE 4-5 Lee la siguiente Página de un libro de texto, tomada de un libro de cuarto
grado, y responde las preguntas del “Tema de plática”.

Podemos aplicar el procedimiento de Polya de los cuatro pasos para resolver problemas
expresados en palabras y en los que sea útil usar el razonamiento algebraico. En Entender el
problema identificamos lo que se nos da y lo que hay que hallar. En Trazar un plan asignamos
letras a las cantidades desconocidas y traducimos la información dada en el problema a un
modelo que incluya ecuaciones. En Realizar el plan resolvemos las ecuaciones o desigualda-
des. En Revisar interpretamos y verificamos la solución en términos del problema original.
En los siguientes problemas mostramos el uso del proceso de Polya de cuatro pasos para
resolver problemas.

© Lopez Mateos Editores. ISBN 978-607-95583-2-1, obra completa, versión electrónica, ISBN 978-607-95583-3-8, volumen 1, versión electrónica. Ejemplar asignado a: Helecto Villarroel gutierrez -
helecto@gmail.com. Fecha: 27 de octubre de 2014. Prohibida su modificación, copia o distribución.
Sección 4-2 Ecuaciones 215

Página de un libro de texto TRADUCCIÓN DE PALABRAS


A E C U AC I O N ES

Lección

Traducción de palabras
Calentamiento
´ Evalúa cada expresión.
Idea clave
Las ecuaciones te
a ecuaciones para n = 14
pueden ayudar a
resolver problemas. Aprende para n = 28

¿Cómo planteas una ecuación?


para n = 7
Vocabulario
ecuación
(p. 100) Un cachorro de gran danés pesó 4 libras al nacer.
Después de 3 semanas pesaba 6 libras.
Ejemplo
Plantea una ecuación para ilustrar cuánto
aumentó de peso el cachorro en 3 semanas.

ex
ef l i ó
R

¡Piensa!
Puedo trazar una Peso al Libras que Peso a las
figura y escribir nacer aumentó 3 semanas
una ecuación
para representar
la situación.
Puedo intentar,
verificar y revisar al La ecuación 4 + p = 6 ilustra cuánto
resolver una ecuación. aumentó de peso el cachorro en 3 semanas.

Tema de plática
En el ejemplo, ¿qué representa p?
Después de 5 semanas el cachorro pesaba 8 libras.
Plantea una ecuación para ilustrar cuánto peso ganó
el cachorro en 5 semanas.

VERIFICA Para otro ejemplo ver el 12-2 en la p. 728.

Plantea una ecuación para cada frase.


p páginas más 7 páginas 8 menos que k es 15.
igual a 17 páginas.
9 veces n es 27. 36 dividido entre y es 12.
Razonamiento Cati quiso saber a cuántos centímetros
equivalen 80 milímetros. Ella usó la ecuación
10x = 80. ¿Está Cati en lo correcto? Explica.

Fuente: Mathematics, Diamond Edition, Grade Four, Scott Foresman-Addison Wesley 2008 ( p.690).

© Lopez Mateos Editores. ISBN 978-607-95583-2-1, obra completa, versión electrónica, ISBN 978-607-95583-3-8, volumen 1, versión electrónica. Ejemplar asignado a: Helecto Villarroel gutierrez -
helecto@gmail.com. Fecha: 27 de octubre de 2014. Prohibida su modificación, copia o distribución.
216 Razonamiento algebraico

Resolver problemas Libros vencidos


Bruno tiene cinco libros de la biblioteca vencidos. La multa para los libros vencidos es de
10¢ diarios por libro. Él recuerda que sacó un libro de astronomía una semana antes de sacar
cuatro novelas. Si la multa total fue de $8.70, ¿cuánto tiempo estuvo vencido cada libro?

Comprender el problema Bruno tiene cinco libros vencidos. Sacó uno de astronomía siete
días antes que las cuatro novelas; por lo tanto, el libro de astronomía está vencido siete días
más que las novelas. La multa diaria por cada libro es de 10¢ y la multa total fue de $8.70.
Necesitamos hallar cuántos días estuvo vencido cada libro.

Trazar un plan Sea d el número de días que estuvo vencida cada una de las novelas. El libro
de astronomía estuvo vencido siete días más, esto es, d + 7 días. Para plantear una ecuación
para d, podemos expresar la multa total de dos maneras. La multa total es de $8.70. Esta
multa expresada en centavos es igual a la multa del libro de astronomía más la multa de las
cuatro novelas.
Multa por cada novela = multa por día por el número de días vencidos
10 . d
Multa por las cuatro novelas = 1 día de multa de las 4 novelas por número de días vencido
4 # 10 . d
= 14 # 102d
= 40d
Multa por el libro de astronomía = multa por día por el número de días vencidos
10 . 1d + 72
= 101d + 72
Como cada una de las expresiones anteriores está en centavos, necesitamos escribir la multa
total de $8.70 como 870¢ para producir lo siguiente:
Multa por las cuatro novelas + multa por el libro de astronomía = multa total
40d + 101d + 72 = 870

Realizar el plan Despejar d en la ecuación:


40d + 101d + 72 = 870
40d + 10d + 70 = 870
50d + 70 = 870
50d = 870 - 70
50d = 800
d = 16
Así, cada una de las novelas estuvo vencida 16 días y el libro de astronomía estuvo ven-
cido por d + 7, ó 23, días.

Revisar Para verificar nuestra respuesta, sigamos la información original. Cada una de las
cuatro novelas estuvo vencida 16 días y el libro de astronomía estuvo vencido 23 días. Como
la multa fue de 10¢ diarios por cada libro, la multa por cada una de las novelas fue de 16 # 10¢,
ó 160¢. Por lo tanto, la multa por el total de novelas fue de 4 # 160¢, ó 640¢. La multa por el

© Lopez Mateos Editores. ISBN 978-607-95583-2-1, obra completa, versión electrónica, ISBN 978-607-95583-3-8, volumen 1, versión electrónica. Ejemplar asignado a: Helecto Villarroel gutierrez -
helecto@gmail.com. Fecha: 27 de octubre de 2014. Prohibida su modificación, copia o distribución.
Sección 4-2 Ecuaciones 217

libro de astronomía fue de 23 # 10¢, ó 230¢. En consecuencia, la multa total fue de


640¢ + 230¢, u 870¢, lo cual concuerda con la información dada de $8.70 como la multa total.
Pudimos haber resuelto el problema sin emplear álgebra. Una manera de hacerlo es notar
que el libro de astronomía estuvo vencido 7 días, generando una multa de 70¢, antes de que
se vencieran los otros libros. Así, 870¢ - 70¢, u 800¢, es la multa por los cinco libros. Por lo
tanto, la multa por un libro es de 800¢>5, ó 160¢. Como la multa es de 10¢ diarios, cada libro
estuvo vencido por 160/10, ó 16, días. El libro de astronomía fue prestado una semana antes
y, por lo tanto, llevaba vencido 23 días.

Resolver problemas Entrega de periódicos
En un pueblo, tres niños entregan todos los periódicos. Abel entrega tres veces la cantidad
que entrega Brenda y Carla entrega 13 más que Abel. Si entre los tres niños entregan un
total de 496 periódicos, ¿cuántos periódicos entregó cada uno?

Comprender el problema El problema pregunta por el número de periódicos que entregó


cada niño. Da información que compara el número de periódicos que entregó cada niño, así
como el número total de periódicos entregados en el pueblo.

Trazar un plan Sean a, b y c el número de periódicos entregados por Abel, Brenda y Carla,
respectivamente. Traducimos la información dada en ecuaciones de la siguiente manera:
Abel entrega 3 veces la cantidad que entrega Brenda: a = 3b
Carla entrega 13 periódicos más que Abel: c = a + 13
La entrega total es de 496: a + b + c = 496
Para reducir el número de variables, substituimos a por 3b en la segunda y tercera ecuaciones.
c = a + 13 se convierte en c = 3b + 13
a + b + c = 496 se convierte en 3b + b + c = 496
A continuación plantea una ecuación en una variable, b, substituyendo c por 3b + 13 en la
ecuación 3b + b + c = 496, despeja b y después halla a y c.

Realizar el plan
3b + b + 3b + 13 = 496
7b + 13 = 496
7b = 483
b = 69
Así, a = 3b = 3 # 69 = 207. También, c = a + 13 = 207 + 13 = 220. Por lo tanto, Abel
entregó 207 periódicos, Brenda entregó 69 periódicos y Carla entregó 220 periódicos.

Revisar Para verificar la respuesta sigue la información original usando a = 207, b = 69 y


c = 220. La información de la primera frase, “Abel entrega tres veces la cantidad que en-
trega Brenda”, se verifica pues 207 = 3 # 69. La segunda frase, “Carla entrega 13 más que
Abel”, es verdadera pues 220 = 207 + 13. La información en la entrega total se verifica
◆ pues 207 + 69 + 220 = 496.

© Lopez Mateos Editores. ISBN 978-607-95583-2-1, obra completa, versión electrónica, ISBN 978-607-95583-3-8, volumen 1, versión electrónica. Ejemplar asignado a: Helecto Villarroel gutierrez -
helecto@gmail.com. Fecha: 27 de octubre de 2014. Prohibida su modificación, copia o distribución.
218 Razonamiento algebraico

AHORA INTENTA ÉSTE 4-6


1. Resuelve el problema anterior de Entrega de periódicos introduciendo una sola incógnita para el
número de periódicos que entrega Brenda.
2. Recuerda que el ejemplo 4-5 lo resolvimos sin usar ecuaciones y que después de la solución menciona-
mos que el problema se podía resolver usando la estrategia de plantear una ecuación. Ahora ya estás
preparado para resolver el problema de esta manera. Supón que el precio del melón es de x, el del flo-
rero es de y y el de la sandía de z. Verifica que de la información de la figura 4-3(a) obtenga x + y = 8
y plantea las ecuaciones correspondientes a las partes (b) y (c) de la figura. Resuelve las ecuaciones re-
duciéndolas a dos ecuaciones con dos incógnitas y después a una ecuación con una incógnita.

Evaluación 4-2A

1. 4. Para cierto evento se vendieron 812 boletos para un total


de $19,120. Si los estudiantes pagaron $20 por boleto y
los no estudiantes pagaron $30, ¿cuántos boletos para es-
Si 12 tudiante se vendieron?
5. Hay un legado de $486,000 para tres hermanas. La mayor
recibe 3 veces más que la menor. La de en medio recibe
$14,000 más que la menor. ¿Cuánto recibió cada una?
6. Una tabla de 10 pies se va a cortar en tres piezas, dos pie-
y 18 zas de igual longitud y la tercera 3 pulgadas más corta
que cada una de las otras dos. Si no hay merma de longi-
tud al cortar, ¿cuál es la longitud de cada pieza?
7. Una caja contiene 67 monedas, sólo de diez y de cinco.
La cantidad de dinero en la caja es de $4.20. ¿Cuántas
y 10 monedas de diez y cuántas de cinco hay en la caja?
8. Miriam es 10 años mayor que Ricardo. Hace dos años
Miriam tenía 3 veces la edad que Ricardo tiene ahora.
¿Cuál es la edad de cada uno?
¿cuál es el valor de cada figura? Di por qué.
9. En una universidad hay inscritos 15 veces más estudian-
2. De ser posible, resuelve: tes de licenciatura que de posgrado. Si el número total
a. x - 3 = 21 de estudiantes inscritos es de 10,000, ¿cuántos estudian-
b. 2x + 5 = x + 25 tes de posgrado hay?
c. 2x + 5 = 3x - 4 10. Una granjera tiene 700 yd de cerca para cercar un pas-
d. 512x + 12 + 712x + 12 = 84 tizal rectangular para sus chivos. Como un lado del
e. 312x - 62 = 412x - 62 pastizal está contiguo a un río, no requiere cerca. El
Resuelve los problemas 3 al 10 planteando y resolviendo una lado paralelo al río debe ser del doble de longitud que
ecuación. el lado perpendicular al río. Halla las dimensiones del
3. Ricardo está construyendo sucesiones de cuadrados con pastizal rectangular.
cerillos de modo que cada vez se agrega un cuadrado a la
derecha según se muestra. Él usó 67 cerillos para formar
la última figura de su sucesión. ¿Cuántos cuadrados hay Río
en la última figura?
Pastizal

, , ,

© Lopez Mateos Editores. ISBN 978-607-95583-2-1, obra completa, versión electrónica, ISBN 978-607-95583-3-8, volumen 1, versión electrónica. Ejemplar asignado a: Helecto Villarroel gutierrez -
helecto@gmail.com. Fecha: 27 de octubre de 2014. Prohibida su modificación, copia o distribución.
Sección 4-2 Ecuaciones 219

Evaluación 4-2B

1. 4. La suma de dos términos consecutivos de la sucesión arit-


mética 1, 4, 7, 10, Á es 299; halla estos dos términos.
5. La suma de los dos primeros términos de una sucesión
Si geométrica es 100 veces mayor que el primer término.
¿Cuál es la razón común?
6. Se dejó un legado de $1,000,000 a cuatro hermanas. La
mayor debe recibir el doble de la menor. Las otras dos
hermanas deben recibir, cada una, $16,000 más que la
y , menor. ¿Cuánto recibirá cada una?
7. Miriam es cuatro años mayor que Ricardo. Hace diez años
Miriam tenía 3 veces la edad que Ricardo tenía entonces.
a. ¿qué figura pesa más? Di por qué. a. ¿Cuál es la edad de cada uno, ahora?
b. ¿qué figura pesa menos? Di por qué. b. Determina si tu respuesta es correcta verificando que
2. De ser posible, resuelve: satisfaga las condiciones del problema.
a. 3x + 13 = 2x + 100 8. En una universidad hay 13 veces más estudiantes que
b. 2x + 5 = 21x + 52 profesores. Si el total de estudiantes y profesores es de
c. 713x + 62 + 513x + 62 = 144 28,000, ¿cuántos estudiantes hay en la universidad?
d. 22 - x = 3x + 6 9. Un granjero tiene 800 yd de cerca para cercar un pasti-
e. 22 - 12x - 62 = 312x - 62 + 6 zal rectangular. Como un lado del pastizal está contiguo
f. 512x - 102 = 412x - 102 a un río, no requiere cerca. Si el lado perpendicular al río
Resuelve los problemas 3 al 11 planteando y resolviendo debe ser del doble de longitud que el lado paralelo al río,
una ecuación. ¿cuáles son las dimensiones del pastizal rectangular?
3. Ricardo está construyendo sucesiones de cuadrados con 10. Dentro de diez años la edad de Ale será 3 veces su edad
cerillos, según se muestra. Ha usado 599 cerillos para actual. Halla la edad actual de Ale.
formar las últimas dos figuras de su sucesión. ¿Cuántos 11. Mati tiene el doble de palillos que David. ¿Cuántos pali-
cerillos usó en cada una de las dos últimas figuras de su llos debe dar Mati a David de modo que cada uno tenga
sucesión? 120 palillos? Verifica que tu respuesta sea correcta.

, , ,

Conexiones matemáticas 4-2

Comunicación Aprendizaje colectivo


1. Se pidió a los alumnos que hallaran tres números com- 4. Examina varios libros de texto de educación básica, de
pletos consecutivos cuya suma sea 393. Una alumna es- los grados 1 a 5, y reseña cómo se introducen en cada
cribió la ecuación x + 1x + 12 + 1x + 22 = 393. Otra uno los conceptos algebraicos que incluyen ecuaciones.
escribió 1x - 12 + x + 1x + 12 = 393. ¿Es posible tra-
Preguntas del salón de clase
bajar con ambos enfoques para obtener la respuesta a la
pregunta? Explica por qué sí o por qué no. 5. Un alumno asegura que la ecuación 3x = 5x no tiene so-
2. Explica cómo resolver la ecuación 3x + 5 = 5x - 3 lución porque 3 Z 5. ¿Qué le respondes?
usando una balanza. 6. Una alumna asegura que como en el siguiente problema
necesitamos hallar tres cantidades desconocidas, ella
Solución abierta debe plantear ecuaciones con tres incógnitas. ¿Qué le
3. Inventa una ecuación con x en ambos lados para cada respondes?
caso: Abel entrega el doble de periódicos que Julián y
a. Todo número completo es solución. Brenda entrega 50 periódicos más que Abel. ¿Cuántos
b. Ningún número completo es solución. periódicos entrega cada uno si el total de periódicos en-
c. 0 es solución. tregados es de 550?

© Lopez Mateos Editores. ISBN 978-607-95583-2-1, obra completa, versión electrónica, ISBN 978-607-95583-3-8, volumen 1, versión electrónica. Ejemplar asignado a: Helecto Villarroel gutierrez -
helecto@gmail.com. Fecha: 27 de octubre de 2014. Prohibida su modificación, copia o distribución.
220 Razonamiento algebraico

7. Se dijo a una estudiante que para verificar la solución a b. La paga total de Jaime después de 4 años si el primer
un problema narrado, como el 6, no es suficiente verifi- año su salario fue de d dólares y de ahí en adelante,
car que la solución hallada satisface la ecuación que ella cada año su salario duplica el del año anterior.
planteó, sino que es necesario verificar la respuesta en el
Preguntas del Third International Mathematics and
problema original. Ella quiere saber por qué. ¿Cómo le
Science Study (TIMSS) (Tercer Estudio Internacional
respondes?
sobre las Matemáticas y la Ciencia)
8. En un examen se pidió a una estudiante resolver la ecua-
ción 4x + 5 = 31x + 152. Ella procedió así: Arturo tiene 50 manzanas. Vendió algunas y le quedaron
20. ¿Cuál es la proposición que lo muestra?
4x + 5 = 3x + 45 = x + 5 = 45 = x = 40 a. n - 20 = 50
En consecuencia, x = 40. Ella verificó que x = 40 satis- b. 20 - n = 50
face la ecuación original; sin embargo, no obtuvo crédito c. n - 50 = 20
total por el problema. Ella quiere saber por qué. ¿Cómo d. 50 - n = 20
le respondes? TIMSS 2003, Grado 4

Problemas de repaso Los objetos en la balanza hacen que ésta se nivele. En el


platillo izquierdo hay un peso de 1 kg (masa) y medio la-
9. Si el número de todos los estudiantes de segundo, tercero
drillo. En el otro platillo hay un ladrillo.
y cuarto años se denota con x y si es 3 veces el número de
los de primer año, denotado con y, escribe una ecuación
algebraica que ilustre la relación. 1 kg
10. Escribe la suma de cinco números pares consecutivos si el
de en medio es n. Simplifica tu respuesta.
11. Si Julia tiene el doble de CD que Juan y Tina tiene 3 ve-
ces más que Julia, escribe una expresión algebraica para ¿Cuánto pesa (masa) un ladrillo?
el número de CD que tiene cada una, en términos de a. 0.5 kg
una variable. b. 1 kg
12. Escribe una ecuación algebraica que relacione las varia- c. 2 kg
bles descritas en cada caso: d. 3 kg
a. La paga P por t horas si te pagan $30 por la primera
TIMSS 2003, Grado 8
hora y $5 más que la hora anterior por cada hora que
sigue.

4-3 Funciones
El concepto de función es fundamental en todas las matemáticas, en particular en álgebra,
según se afirma en los Principios y objetivos:

◆ Nota de Al considerar el álgebra como un hilo conductor en la currícula de preescolar en adelante, los
maestros pueden ayudar a los estudiantes a construir una base sólida de comprensión y experien-
investigación cia que sirva de preparación para realizar un trabajo más complejo en álgebra, en los niveles medio
Los estudiantes sue- y medio superior. Por ejemplo, tener experiencia sistemática con patrones servirá para compren-
len ver una función der el concepto de función (Erick Smith, por aparecer), y tener experiencia con números y sus pro-
como una colección piedades forma la base para trabajar, más adelante, con símbolos y expresiones algebraicas. Al
de puntos o de pares comprender que hay situaciones que se pueden describir usando matemáticas, los estudiantes
ordenados. Esta pueden formarse nociones elementales de modelación matemática. (p. 37)
visión limitada puede Las funciones pueden modelar multitud de fenómenos del mundo real, según veremos en
dificultar el desarro- esta sección y en capítulos posteriores. En esta sección exploraremos varias maneras de re-
llo del concepto de presentar funciones —como reglas, máquinas, ecuaciones, diagramas de flechas, tablas, pares or-
función en el denados y gráficas. Es importante que los estudiantes vean una gran variedad de maneras de
estudiante (Adams
representar funciones, como se indica en la Nota de investigación.
1997). ◆
© Lopez Mateos Editores. ISBN 978-607-95583-2-1, obra completa, versión electrónica, ISBN 978-607-95583-3-8, volumen 1, versión electrónica. Ejemplar asignado a: Helecto Villarroel gutierrez -
helecto@gmail.com. Fecha: 27 de octubre de 2014. Prohibida su modificación, copia o distribución.
Sección 4-3 Funciones 221

Funciones como reglas


A continuación presentamos un ejemplo de un juego llamado “adivine mi regla”, que se usa
a menudo para introducir el concepto de función.
Cuando Tomás dijo 2, Noé dijo 5. Cuando David dijo 4, Noé dijo 7. Cuando Mari dijo 10, Noé dijo
13. Cuando Isabel dijo 6, ¿qué dijo Noé? ¿Cuál es la regla de Noé?
La respuesta a la primera pregunta puede ser 9, y la regla podría ser “Tomar el número ori-
ginal y sumarle 3”; esto es, para cualquier número n, la respuesta de Noé es n + 3.

Ejemplo 4-7 Sugiere la regla del maestro para las respuestas siguientes:








a. b. c.
Tú Maestro Tú Maestro Tú Maestro
1 3 2 5 2 0
0 0 3 7 4 0
4 12 5 11 7 1
10 30 10 21 21 1
Solución a. La regla del maestro podría ser “Multiplicar el número dado n por 3”, esto es, 3n.
b. La regla del maestro podría ser “Doblar el número original n y sumarle 1”,
esto es, 2n + 1.
c. La regla del maestro podría ser “Si el número n es par, responde 0; si el
número es impar, responde 1”. Otra posible regla es “Si el número es menor
que 5, responde 0; si es mayor o igual que 5, responde 1”.

Funciones como máquinas
Otra manera de preparar a los estudiantes para el concepto de función es usando una “má-
quina-función”. La siguiente Página de un libro de texto muestra un ejemplo de máquina-
función. Lo que va hacia la máquina se llama entrada o input y lo que sale de la máquina se
llama salida o output. Así, en la página de muestra, si la entrada a la función es 2, la salida es
110. Nota que la salida se denota con d. En grados superiores se usa una notación especial
para la salida. Para cada elemento de entrada x la salida se denota como f 1x2,* que se lee “f
de x”. Para la función d = 55t en el ejemplo de la página de muestra, si f es la función en-
tonces cuando la entrada es 2, la salida se puede escribir como d 122. Como la salida es 110,
tenemos d 122 = 110. Como la función trabaja de acuerdo con la regla d 1t2 = 55t, tenemos
d 122 = 2 # 55 = 110.

◆ Nota Los babilonios de Mesopotamia (ca. 2000 a.c.) desarrollaron un concepto precursor de lo
histórica que hoy llamamos función. Para ellos una función era una tabla o una correspondencia. Dos
tablillas halladas en Senkerah, en el Éufrates, en 1854 dan los cuadrados de números hasta
el 59 y los cubos de números hasta el 32.
En el siglo diecisiete la idea de función tuvo mayor desarrollo. En su libro Geometry (Geo-
metría) (1637), René Descartes (1596–1650) usó el concepto para describir multitud de rela-
ciones matemáticas. Casi 50 años después de la publicación del libro de Descartes, Gottfried
Wilhelm Leibniz (1646–1716) introdujo el término de función. La idea de función fue for-
malizada por Leonhard Euler (se pronuncia “oiler”, 1707–1783), quien introdujo la nota-
ción de función, y = f 1x2. Otras contribuciones al concepto fueron realizadas por los
Leonhard Euler matemáticos Joseph-Louis Lagrange (1736–1813) y Jean Joseph Fourier (1768–1830). ◆

* Cuando una función designa una cantidad específica, como d (distancia) en la página de muestra, y
la entrada es t (tiempo) se puede usar d1t2 en lugar de f1x).
© Lopez Mateos Editores. ISBN 978-607-95583-2-1, obra completa, versión electrónica, ISBN 978-607-95583-3-8, volumen 1, versión electrónica. Ejemplar asignado a: Helecto Villarroel gutierrez -
helecto@gmail.com. Fecha: 27 de octubre de 2014. Prohibida su modificación, copia o distribución.
222 Razonamiento algebraico

Página de un libro de texto REGLAS DE FUNCIONES

Reglas de funciones

Verifica tus Vas a aprender a


habilidades Escribir y evaluar funciones
Revisión de vocabulario Vocabulario nuevo función
¿Por qué 5 + 2 no es
una expresión
algebraica?
Evalúa -4x + 1 para
¿Por qué aprenderlo?
cada valor de x. La distancia que recorres en un Entrada (tiempo)
automóvil depende del tiempo de
manejo. Cuando una cantidad depende
de otra, dices que una es función de
la otra. Así, la distancia es función del
Ve por ayuda a la tiempo. Puedes usar funciones
como ayuda para hacer predicciones.
Lección 4-1

En el diagrama de la derecha, se introduce Salida


una entrada a la “máquina-función” para (distancia)
producir una salida.
Una función es una relación que asigna exactamente una salida para
cada valor de entrada.

EJEMPLO Escribir una regla de una función


Carros Estás viajando en un carro a una rapidez promedio de
55 mi/h. Escribe una regla de una función que describa la relación
entre el tiempo y la distancia que viajas.
Tú puedes hacer una tabla para resolver este problema.

Entrada: tiempo (h)


Salida: distancia (mi)

distancia en millas =55 · tiempo en horas d Escribe la regla en palabras


Usa las variables d y t para la distancia y el tiempo.

Verificación rápida
Escribe una regla de una función para la relación entre el tiempo y
la distancia que recorres a una rapidez promedio de 62 mi/h.

Capítulo 9 Patrones y reglas

Fuente: Mathematics, Course 2, Pearson Prentice Hall 2008 ( p. 452).

© Lopez Mateos Editores. ISBN 978-607-95583-2-1, obra completa, versión electrónica, ISBN 978-607-95583-3-8, volumen 1, versión electrónica. Ejemplar asignado a: Helecto Villarroel gutierrez -
helecto@gmail.com. Fecha: 27 de octubre de 2014. Prohibida su modificación, copia o distribución.
Sección 4-3 Funciones 223

O B S E R VA C I Ó N En la mayoría de las calculadoras gráficas la notación usada para fun-


ción es Y1, Y2, Y3, Á , y así sucesivamente. Aquí Y1 actúa como f 1x2 si la regla de la
función se escribe en términos de x.

Ejemplo 4-8 Considera la máquina-función de la figura 4-15. ¿Qué sucederá con la función llamada f, si









introducimos los números 0, 1, 3, 4 y 6?

Solución Si los números de salida se denotan con f 1x2, los valores correspondientes se
pueden describir usando la tabla 4-1. Nota que f 142 de la figura 4-15 es la salida de la función
“f ” cuando la entrada es 4.
4 Tabla 4-1

x f 1x2

0 3
Suma 3
1 4
3 6
f (4) = 7 4 7
6 9
Figura 4-15

Funciones como ecuaciones


En el ejemplo 4-8 podemos escribir una ecuación que describa la regla. Si la entrada es x, la
salida es x + 3; esto es, f 1x2 = x + 3. Los valores de salida se pueden obtener substitu-
yendo con los valores 0, 1, 3, 4 y 6 la x de f 1x2 = x + 3, como se muestra a continuación:
f 102 = 0 + 3 = 3
f 112 = 1 + 3 = 4
f 132 = 3 + 3 = 6
f 142 = 4 + 3 = 7
f 162 = 6 + 3 = 9
Nota la terminología de entrada y salida en la representación de la máquina-función de la
página de muestra anterior.
En multitud de aplicaciones, tanto las entradas como las salidas de una máquina-función
son números; Sin embargo, las entradas y salidas también pueden ser cualquier objeto. Por
ejemplo, considera una máquina que sólo acepta monedas de 25¢, 50¢ y 75¢ y despacha
uno de tres tipos de dulces, cada uno de los cuales cuesta 25¢, 50¢ y 75¢, respectivamente.
Una máquina-función asocia exactamente una salida a cada entrada. Si para el elemento de
entrada x obtenemos f 1x2 como salida, entonces cada vez que uses la misma x como en-
trada obtendrás la misma f 1x2 como salida. La idea de una máquina-función que asocia
exactamente una salida con cada entrada, de acuerdo con cierta regla, conduce a la defini-
ción siguiente.

Definición de función
Una función de un conjunto A a un conjunto B es una correspondencia de A a B en la cual cada
elemento de A está pareado con uno, y sólo uno, de los elementos de B.

© Lopez Mateos Editores. ISBN 978-607-95583-2-1, obra completa, versión electrónica, ISBN 978-607-95583-3-8, volumen 1, versión electrónica. Ejemplar asignado a: Helecto Villarroel gutierrez -
helecto@gmail.com. Fecha: 27 de octubre de 2014. Prohibida su modificación, copia o distribución.
224 Razonamiento algebraico

◆ Nota de En la definición anterior, el conjunto A es el conjunto de todas las entradas permitidas y


es el dominio de la función. El conjunto B, el contradominio o codominio, es cualquier
investigación conjunto que contenga todas las posibles salidas. El conjunto de todas las salidas es el rango
Los estudiantes fre- o imagen de la función. El contradominio B en la definición puede ser cualquier conjunto
cuentemente tienen que contenga el rango y puede coincidir con éste. La diferencia entre contradominio y
dificultades con el rango se hace por conveniencia, pues a veces no es fácil ubicar el rango. Por ejemplo, haz
lenguaje de las fun- corresponder a cada estudiante de una universidad con su número registro. Ésta es una fun-
ciones (por ejemplo ción que va del conjunto de todos los estudiantes al conjunto C de los números completos.
imagen, dominio, rango Es decir, el contradominio contiene al rango. El rango, en este caso, es el conjunto de nú-
y uno a uno), lo cual meros de registro que corresponden a los estudiantes inscritos en esa universidad. El rango
tiene impactos subse- es un subconjunto propio del contradominio C. Por lo regular, en caso de no señalar un domi-
cuentes en sus habili- nio para describir la función, se supone que el dominio está formado por todos los elementos para los
dades para trabajar cuales la regla tenga sentido. Como se señala en la Nota de investigación, los estudiantes pueden
representaciones grá- tener dificultades para comprender estos conceptos.
ficas de funciones Una calculadora contiene muchas funciones. Supón que un estudiante teclea 9 * K
(Markovits et al.
en una calculadora que tiene tecla de constante, K . A continuación, el estudiante oprime
1988). ◆
0 y pasa la calculadora a otro estudiante. El otro estudiante debe determinar la regla te-
x
cleando otros números y oprimiendo, a continuación, la tecla = . También se pueden usar
máquinas con la característica de constante automática.
Otras teclas de la calculadora son teclas de función. Por ejemplo la tecla p presenta en
la salida es menor
pantalla, siempre, una aproximación de p, como 3.1415927; la tecla +>- coloca un signo
que la entrada
menos antes de un número o quita un signo menos existente; y las teclas x2 y 1 elevan
al cuadrado y extraen la raíz cuadrada, respectivamente.
f (x)
¿Son máquinas-función todas las máquinas de entrada-salida? Considera la máquina de la
Figura 4-16 figura 4-16. Para cualquier número natural de entrada x, la máquina saca un número menor
que x. Si, por ejemplo, introduces el número 10 la máquina puede sacar 9, pues 9 es menor
que 10. Si de nuevo introduces 10, la máquina puede sacar 3, pues 3 es menor que 10. Di-
cha máquina no es una máquina-función, ya que la misma entrada puede producir diferen-
tes salidas.

Ejemplo 4-9 Un fabricante de bicicletas tiene un gasto diario fijo de $1400 y le cuesta $500 fabricar una
bicicleta. Responde lo siguiente:








a. Halla el costo C1x2 de fabricar x bicicletas al día.


b. Si el fabricante vende cada bicicleta en $700 y la ganancia (o pérdida) al producir y vender
x bicicletas al día es P1x2, expresa P1x2 en términos de x.
c. Halla el punto de equilibrio, esto es, el número de bicicletas, x, producidas y vendidas en
donde no hay pérdida ni ganancia.

Solución a. Como el costo de producir una sola bicicleta es de $500, el costo de producir
x será de 500x dólares. Debido al gasto fijo diario de $1400, el costo total, C1x2
en dólares, de producir x bicicletas en un día dado es C1x2 = 500x + 1400.
b. P1x2 = 700x - 1500x + 14002
= 200x - 1400
c. Necesitamos hallar el número x de bicicletas producidas de modo que P1x2 = 0;
esto es, necesitamos resolver 200x - 1400 = 0. Esta ecuación es equivalente a
1400
200x = 1400 ó x = ó7
200
Así, el fabricante necesita producir 7 bicicletas para quedar a mano (sin pér-
dida ni ganancia).

© Lopez Mateos Editores. ISBN 978-607-95583-2-1, obra completa, versión electrónica, ISBN 978-607-95583-3-8, volumen 1, versión electrónica. Ejemplar asignado a: Helecto Villarroel gutierrez -
helecto@gmail.com. Fecha: 27 de octubre de 2014. Prohibida su modificación, copia o distribución.
Sección 4-3 Funciones 225

Funciones como diagramas de flechas


Se pueden usar diagramas de flechas para examinar y determinar si una correspondencia re-
presenta una función. Esta representación se usa normalmente cuando los conjuntos A y B
son conjuntos finitos con unos cuantos elementos. El ejemplo siguiente muestra cómo se
pueden usar los diagramas de flechas para examinar tanto funciones como no funciones.

Ejemplo 4-10 ¿Qué parte, si la hay, de la figura 4-17 exhibe una función de A a B? Si una correspondencia








es una función de A a B, halla el rango de la función.

A B A B A B

a
1 a x
2 1 b
2 b y
4 2 c
3 c z
d

(a) (b) (c)

A B A B

Juan 0 1
Pérez
Miguel 1 4
López
Juana 2 7
Ramírez
Silvia 3 10

(d) (e)

Figura 4-17

Solución a. La figura 4-17(a) no define una función de A a B pues el elemento 1 está rela-
cionado con 2 y con 4.
b. La figura 4-17(b) no define una función de A a B pues el elemento b no está
relacionado con algún elemento de B. (Es una función de un subconjunto
de A a B.)
c. La figura 4-17(c) sí define una función de A a B pues hay una, y sólo una,
flecha que sale de cada elemento de A. El hecho de que d, un elemento de B,
no esté relacionado con un elemento del dominio no viola la definición. El
rango es 5a, b, c6 y no incluye a d pues d no es una salida en esta función, ya
que no hay un elemento de A que esté relacionado con d.
d. La figura 4-17(d) ilustra una función, puesto que hay sólo una flecha que sale
de cada elemento de A. No importa que un elemento del conjunto B, Pérez,
tenga dos flechas que le apunten. El rango es {Pérez, López, Ramírez}.
e. La figura 4-17(e) ilustra una función cuyo rango es {1, 4, 7, 10}.

© Lopez Mateos Editores. ISBN 978-607-95583-2-1, obra completa, versión electrónica, ISBN 978-607-95583-3-8, volumen 1, versión electrónica. Ejemplar asignado a: Helecto Villarroel gutierrez -
helecto@gmail.com. Fecha: 27 de octubre de 2014. Prohibida su modificación, copia o distribución.
226 Razonamiento algebraico

La figura 4-17(e) también ilustra una correspondencia biunívoca, o uno a uno, entre A y B.
De hecho, cualquier correspondencia biunívoca entre A y B define una función de A a B, así
como una función de B a A.

AHORA INTENTA ÉSTE 4-7 ¿Cuáles de las siguientes son funciones del conjunto de los números natu-
rales a {0, 1}? Justifica tu razonamiento.
a. Para todo número natural de entrada, la salida es 0.
b. Para todo número natural de entrada, la salida es 0 si la entrada es un número par y la salida es 1 si la
entrada es un número impar.

Funciones como tablas y pares ordenados


Otra manera útil para describir una función es por medio de una tabla. Considera la infor-
mación de la tabla 4-2 que relaciona la cantidad gastada en publicidad y las ventas en un
mes dado, de una empresa pequeña. Nota que para la Cantidad en publicidad y la Cantidad
en ventas, la información está dada en miles de pesos. Podríamos hablar de una función en-
tre la cantidad gastada en Publicidad y la cantidad obtenida en Ventas, o podríamos simplifi-
car usando una función definida como sigue: Si P = 50, 1, 2, 3, 46 y V = 51, 3, 5, 7, 96, la
tabla describe una función de P a V donde P representa los miles de pesos gastados en pu-
blicidad y V representa los miles de pesos obtenidos en ventas. Por ejemplo, (2, 5) significa
que se gastó $2000 en publicidad resultando en $5000 de ventas.

Tabla 4-2
Cantidad de publicidad Cantidad de ventas
(en miles de pesos) (en miles de pesos)
0 1
1 3
2 5
3 7
4 9

La función también se puede dar usando pares ordenados. Cuando la entrada es 0 y la sa-
lida es 1, registramos la información como el par ordenado (0, 1). De manera análoga, la in-
formación en el segundo renglón se registra como (1, 3) y el resto de la información como
(2, 5), (3, 7) y (4, 9). La primera componente del par ordenado siempre es un elemento del
dominio y la segunda componente es la salida correspondiente.

Ejemplo 4-11 ¿Cuáles de los siguientes conjuntos de pares ordenados representan funciones? Si un con-








junto representa una función, da su dominio y rango. De no ser así, explica por qué.
a. 511, 22, 11, 32, 12, 32, 13, 426 b. 511, 22, 12, 32, 13, 42, 14, 526
c. 511, 02, 12, 02, 13, 02, 14, 426 d. 51a, b2 ƒ a 僆 N y b = 2a6

Solución a. No es función pues la entrada 1 tiene dos salidas diferentes.


b. Es una función con dominio 51, 2, 3, 46. Como el rango es el conjunto de las
salidas, el rango es 52, 3, 4, 56.

© Lopez Mateos Editores. ISBN 978-607-95583-2-1, obra completa, versión electrónica, ISBN 978-607-95583-3-8, volumen 1, versión electrónica. Ejemplar asignado a: Helecto Villarroel gutierrez -
helecto@gmail.com. Fecha: 27 de octubre de 2014. Prohibida su modificación, copia o distribución.
Sección 4-3 Funciones 227

c. Es una función con dominio 51, 2, 3, 46 y rango 50, 46. La salida 0 aparece
más de una vez, pero esto no contradice la definición de función de que a cada
entrada corresponde una sola salida.
d. Ésta es una función con dominio N y rango P, el conjunto de todos los números
naturales pares.

Funciones como gráficas


Quizás una de las representaciones más conocidas de una función sea la de una gráfica. Las
gráficas son representaciones visuales de funciones y aparecen en periódicos y libros, y hasta
en la televisión. Para graficar la función de la tabla 4-2, considera el conjunto de pares or-
denados 510, 12, 11, 32, 12, 52, 13, 72, 14, 926 y haz corresponder cada par ordenado con un
punto en la malla de la figura 4-18. Usamos la escala horizontal para las entradas y la escala
vertical para las salidas, y marcamos el punto correspondiente a 10, 12 comenzando en 0 en
la escala horizontal y subiendo una unidad en la escala vertical. Para marcar el punto co-
rrespondiente a 11, 32, comenzamos en 0 y nos movemos una unidad horizontalmente y 3
unidades verticalmente. Marcar el punto que corresponde a un par ordenado se conoce
como graficar el par ordenado. El conjunto de todos los puntos que corresponden a todos
los pares ordenados es la gráfica de la función. Nota que la gráfica consta de cinco puntos.
Los puntos están conectados por medio de una recta punteada para ilustrar que están sobre
una recta pero que no todo punto de la recta pertenece a la gráfica.

Ventas como función de la publicidad


10

8
Ventas (miles de pesos)

0
1 2 3 4 5 6 7 8 9 10
Publicidad (miles de pesos)

Figura 4-18

Fíjate en los ejemplos A y C de las páginas de muestra parciales que siguen. La gráfica del
ejemplo C consta de puntos que están sobre una recta. ¿Por qué conectamos los puntos por
medio de una recta punteada en lugar de una recta sólida?

© Lopez Mateos Editores. ISBN 978-607-95583-2-1, obra completa, versión electrónica, ISBN 978-607-95583-3-8, volumen 1, versión electrónica. Ejemplar asignado a: Helecto Villarroel gutierrez -
helecto@gmail.com. Fecha: 27 de octubre de 2014. Prohibida su modificación, copia o distribución.
228 Razonamiento algebraico

Página de un libro de texto REGLAS, TABLAS Y GRÁFICAS

Lección
Calentamiento
´ Reglas, tablas y Evalúa cada expresión

gráficas
para x = 3.
Idea clave
Se pueden usar reglas,
tablas y gráficas para
mostrar cómo una Aprende
cantidad está
relacionada con otra. ¿Cómo usas una regla para
hacer una tabla?
Vocabulario
Los boletos para la feria cuestan $3 por persona más $1 del
tabla de valores estacionamiento. Se puede escribir una regla para ilustrar
que el costo total es $3 por el número de personas más $1.
Materiales Regla en palabras: Multiplica por 3 y después suma 1.
papel cuadriculado o
Regla usando una variable: 3x + 1.
Ejemplo A
ex
ef l i ó Haz una tabla de valores para la regla.
R

Multiplica por 3 y después suma 1: 3x + 1


Evalúa la expresión 3x+1 usando
¡Piensa! 1, 2, 3, 4 y 5 en lugar de x.
Yo sé que una
variable es cualquier
símbolo o letra que se
Para x = 1, 3x + 1 = 3 3 1 + 1 = 4.
usa para representar
un número. Para x = 2, 3x + 1 = 3 3 2 + 1 = 7.
Para x = 3, 3x + 1 = 3 3 3 + 1 = 10.
Para x = 4, 3x + 1 = 3 3 4 + 1 = 13.
Para x = 5, 3x + 1 = 3 3 5 + 1 = 16.

¿Cómo haces una gráfica para una tabla de valores?


Ejemplo C
Haz una gráfica para la tabla de valores del ejemplo A.
ex
ef l i ó
Grafica cada uno de los pares ordenados de la tabla:
R

¡Piensa!
Puedo usar múltiplos
en el eje x o en el eje
y para que la gráfica
tenga un tamaño
razonable.

Tema de plática
¿Qué notas acerca de los puntos de la gráfica?
Razonamiento ¿Cómo podrías usar la gráfica para hallar
otro par ordenado que satisfaga la regla del ejemplo A?
¿Crees que (6, 10) satisfaga la regla? ¿Cómo puedes
averiguarlo sin hacer cuentas?

Fuente: Mathematics, Diamond Edition, Grade Five, Scott Foresman-Addison Wesley 2008 ( pp. 176, 177).

© Lopez Mateos Editores. ISBN 978-607-95583-2-1, obra completa, versión electrónica, ISBN 978-607-95583-3-8, volumen 1, versión electrónica. Ejemplar asignado a: Helecto Villarroel gutierrez -
helecto@gmail.com. Fecha: 27 de octubre de 2014. Prohibida su modificación, copia o distribución.
Sección 4-3 Funciones 229

AHORA INTENTA ÉSTE 4-8 Contesta las preguntas 4 a 6 de la página de muestra.

Ejemplo 4-12 Explica por qué una compañía telefónica no adoptaría las tarifas descritas en la gráfica de la









figura 4-19.

Tarifas telefónicas

190

Costo de las llamadas


155

(centavos)
120

85

50

1 2 3 4 5 6 7
Número de minutos

Figura 4-19

Solución La gráfica no describe una función. Por ejemplo, a un cliente se le podría cobrar,
por una llamada de 2 minutos, $0.50 u $0.85; luego entonces no toda entrada tiene una salida
única.

Supón que te inscribes a un videoclub donde el costo de renta por película es de $5. Ya vimos
que una manera de describir una función es mediante una ecuación. Con base en la informa-
ción de la tabla 4-3, la ecuación que relaciona el número de películas rentadas con el costo es
C = n # 5, ó C = 5n, donde n es el número de películas rentadas.

Tabla 4-3

Número de Costo en
rentas pesos
1 1#5 = 5
2 2#5 = 10
3 3#5 = 15
4 4#5 = 20
5 5#5 = 25
# #
# #
# #
n #
n 5 ó 5n

© Lopez Mateos Editores. ISBN 978-607-95583-2-1, obra completa, versión electrónica, ISBN 978-607-95583-3-8, volumen 1, versión electrónica. Ejemplar asignado a: Helecto Villarroel gutierrez -
helecto@gmail.com. Fecha: 27 de octubre de 2014. Prohibida su modificación, copia o distribución.
230 Razonamiento algebraico

Esto también podría escribirse como f 1n2 = 5n, donde f 1n2 es el costo de las rentas
en pesos. Si restringimos el número de rentas a los primeros cinco números naturales, la
función se puede describir como el conjunto de pares ordenados 511, 5212, 102, 13, 152,
14, 202, 15, 2526. La figura 4-20 muestra la gráfica de la función, que consta de cinco pun-
tos que no están conectados por una recta sólida. Al graficar la función en la figura 4-20,
suponemos que el dominio es el conjunto 51, 2, 3, 4, 56. No tiene sentido conectar los pun-
tos pues no podemos, por ejemplo, rentar 1.5 películas. Sin embargo, para mostrar que los
puntos están sobre una recta los conectamos mediante una recta punteada.
Costo de las rentas
25
20
15

Costo
10
5

1 2 3 4 5
Número de películas rentadas

Figura 4-20

AHORA INTENTA ÉSTE 4-9 Halla el rango de la función de la figura 4-20.

Sucesiones como funciones


Las sucesiones aritméticas, geométricas y otras, introducidas en el capítulo 1, se pueden
pensar como funciones cuyas entradas son los números naturales y sus salidas son los tér-
minos de cada sucesión. Por ejemplo, la sucesión aritmética 2, 4, 6, 8, Á , cuyo término
n-ésimo es 2n puede describirse como una función que va del conjunto N (números
naturales) al conjunto P (números naturales pares) mediante la regla f 1n2 = 2n, donde n
es un número natural y f 1n2 representa el valor del término n-ésimo.
Ejemplo 4-13 Si f 1n2 denota el término n-ésimo de una sucesión, halla f 1n2 en términos de n:








a. Una sucesión aritmética cuyo primer término es 3 y cuya diferencia es 3


b. Una sucesión geométrica cuyo primer término es 3 y cuya razón es 3
c. La sucesión 1, 1 + 2, 1 + 2 + 3, 1 + 2 + 3 + 4, Á

Solución a. El primer término es 3, el segundo término es 3 + 3, ó 2 # 3, el tercer término


es 2 # 3 + 3, ó 3 # 3, y el cuarto término es 3 # 3 + 3, ó 4 # 3, el n-ésimo tér-
mino es n # 3, y por lo tanto f 1n2 = n # 3 = 3n, donde n es un número natural.
b. El primer término es 3, el segundo 3 # 3, ó 32, el tercero 3 # 32, ó 33, y así sucesi-
vamente. Por lo tanto, el término n-ésimo es 3n y f 1n2 = 3n, donde n es un
número natural.

c. El término n-ésimo es 1 + 2 + 3 + 4 + Á + n. En el capítulo 1 vimos que


n1n + 12 n1n + 12
esta suma es igual a , de modo que la función es f 1n2 = ,
2 2
donde n es un número natural.

© Lopez Mateos Editores. ISBN 978-607-95583-2-1, obra completa, versión electrónica, ISBN 978-607-95583-3-8, volumen 1, versión electrónica. Ejemplar asignado a: Helecto Villarroel gutierrez -
helecto@gmail.com. Fecha: 27 de octubre de 2014. Prohibida su modificación, copia o distribución.
Sección 4-3 Funciones 231

Composición de funciones
Considera las máquinas-función de la figura 4-21. Si introducimos 2 en la primera máquina,
entonces f 122 = 2 + 4 = 6. Después introducimos 6 en la segunda máquina y obtenemos
g162 = 2 # 6 = 12. Las funciones de la figura 4-21 ilustran la composición de dos funcio-
nes. En la composición de dos funciones, el rango de la primera función es un subconjunto
del dominio de la segunda función.
2

f (x) = x + 4

g(x) = 2x

12

Figura 4-21
Si la primera función f es seguida de una segunda función g, como en la figura 4-21, simbo-
lizamos la composición de las funciones como g ⴰ f. Si la entrada es 3 en las máquinas-fun-
ción de la figura 4-21, la salida se representa con 1g ⴰ f 2132. Como f actúa primero en 3, para
calcular 1g ⴰ f 2132 hallamos primero f 132 = 3 + 4 = 7 y después g172 = 2 # 7 = 14. Por
lo tanto, 1g ⴰ f 2132 = 14 y 1g ⴰ f 2132 = g1f 1322. Nota también que 1g ⴰ f 21x2 =
g1 f 1x22 = 2 # f 1x2 = 21x + 42 y, por lo tanto, g1 f 1322 = 213 + 42 = 14.

Ejemplo 4-14 Si f 1x2 = 2x + 3 y g1x2 = x - 3, halla:


a. 1 f ⴰ g2132 b. 1g ⴰ f 2132 c. 1 f ⴰ g21x2 d. 1g ⴰ f 21x2








Solución a. 1f ⴰ g2132 = f1g1322 = f13 - 32 = f 102 = 2 # 0 + 3 = 3


b. 1g ⴰ f 2132 = g1 f1322 = g12 # 3 + 32 = g192 = 9 - 3 = 6
c. 1 f ⴰ g21x2 = f 1g1x22 = 2 # g1x2 + 3 = 21x - 32 + 3 = 2x - 6 + 3 =
2x - 3
d. 1g ⴰ f 21x2 = g1 f 1x22 = f 1x2 - 3 = 12x + 32 - 3 = 2x

O B S E R VA C I Ó N El ejemplo 4-14 muestra que la composición de funciones no es con-


mutativa pues 1f ⴰ g2132 Z 1g ⴰ f 2132.

Hemos visto que podemos representar de muchas maneras una función. Las figuras con
conjuntos y flechas y las máquinas-función se usan principalmente como instrumentos pe-
dagógicos en el aprendizaje del concepto de función. Las representaciones más usadas son
como tabla, ecuación o gráfica. Dependiendo de la situación, una representación puede ser
más útil que otra. Por ejemplo, si el dominio de una función es un conjunto grande, no con-
viene usar una tabla. En capítulos posteriores aprenderemos cómo graficar cierto tipo de
ecuaciones. Las calculadoras gráficas pueden graficar la mayoría de las funciones dadas por
ecuaciones en un dominio específico.
© Lopez Mateos Editores. ISBN 978-607-95583-2-1, obra completa, versión electrónica, ISBN 978-607-95583-3-8, volumen 1, versión electrónica. Ejemplar asignado a: Helecto Villarroel gutierrez -
helecto@gmail.com. Fecha: 27 de octubre de 2014. Prohibida su modificación, copia o distribución.
232 Razonamiento algebraico

RINCÓN DE LA TECNOLOGÍA En la figura 4-22 se muestra


el trazo de la función y = 2x + 1 para x entre 0 y 5. Usa una
11
calculadora gráfica para trazar las gráficas de y = 2x + b para
tres valores de b. ¿Qué tienen en común las gráficas? ¿Por qué? 10
9
8
7
6
5
4
3
2
1

0
1 2 3 4 5

Figura 4-22

Relaciones
Así como en la definición de función de un conjunto A a un conjunto B, una relación de un
conjunto A a un conjunto B es una correspondencia entre elementos de A y elementos de B,
aunque a diferencia de las funciones no requerimos que cada elemento de A esté pareado con
un, y sólo un, elemento de B. En consecuencia, cualquier conjunto de pares ordenados es
una relación. Nota que cada función es una relación pero no toda relación es una función.
Como ejemplos de relaciones tenemos:
“es hija de” “es del mismo color que”
“es menor que” “es mayor o igual que”
Considera la relación “es hermana de”. La figura 4-23 ilustra esta relación entre niñas
y niños en un jardín, representando sus nombres con letras de la A a la J. Una flecha de
I a J indica que I “es hermana de” J. Nota que las flechas de F a G y de G a F, indican
que F es hermana de G y que G es hermana de F. Esto implica que F y G son niñas. Por
otro lado, la ausencia de flecha de J a I implica que J no es hermana de I. Así, I es una
niña y J es un niño.

E
A G
B
F
H
D C
I

Figura 4-23

© Lopez Mateos Editores. ISBN 978-607-95583-2-1, obra completa, versión electrónica, ISBN 978-607-95583-3-8, volumen 1, versión electrónica. Ejemplar asignado a: Helecto Villarroel gutierrez -
helecto@gmail.com. Fecha: 27 de octubre de 2014. Prohibida su modificación, copia o distribución.
Sección 4-3 Funciones 233

AHORA INTENTA ÉSTE 4-10 Si en la figura 4-23 están indicadas todas las relaciones de “ser hermana
de”, determina
a. cuáles son niños y cuáles son niñas.
b. para qué personas no hay suficiente información para determinar su género.

Otra manera de ilustrar la relación “es hermana de” es escribir la relación “A es hermana de
B” como el par ordenado 1A, B2. Nota que (B, A) significa que B es hermana de A. Usando
esta notación, la relación “es hermana de” se puede describir para las niñas y niños en el
jardín como el conjunto
51A, B2, 1A, C2, 1A, D2, 1C, A2, 1C, B2, 1C, D2, 1D, A2, 1D, B2, 1D, C2, 1F, G2, 1G, F2, 1I, J26
Nota que es un subconjunto de 5A, B, C, D, E, F, G, H, I, J6 * 5A, B, C, D, E, F, G, H, I, J6.
Esta observación motiva la siguiente definición de relación.

Definición
Relación del conjunto A al conjunto B Dados dos conjuntos A y B, una relación de A a B es
un subconjunto de A * B; esto es, R es una relación del conjunto A al conjunto B si, y sólo si,
R 8 A * B.

En la definición, la frase “de A a B” significa que las primeras componentes de los pares or-
denados son elementos de A y las segundas componentes son elementos de B. Si A = B,
decimos que la relación es en A.

Propiedades de las relaciones


La figura 4-24 representa un conjunto de niñas y niños de un grupo pequeño. Ellos han tra-
zado todas las flechas posibles que representan la relación “su nombre comienza con la
misma letra que”. Nota que tuvieron el cuidado de verificar que cada persona del grupo tu-
viera la misma letra como inicio de su nombre. En la figura 4-24 se ilustran tres propiedades
de las relaciones.

Juan
José
Beto
Rolo
Carolina
Candy
Beti

Debora
Cati

Figura 4-24

Definición de la propiedad reflexiva


Una relación R de un conjunto X es reflexiva si, y sólo si, para cada elemento a 僆 X, a está rela-
cionado con a. Esto es, para cada a 僆 X, 1a, a2 僆 R.

© Lopez Mateos Editores. ISBN 978-607-95583-2-1, obra completa, versión electrónica, ISBN 978-607-95583-3-8, volumen 1, versión electrónica. Ejemplar asignado a: Helecto Villarroel gutierrez -
helecto@gmail.com. Fecha: 27 de octubre de 2014. Prohibida su modificación, copia o distribución.
234 Razonamiento algebraico

En el diagrama hay un bucle en cada punto. Por ejemplo, Rolo tiene la misma inicial que
él mismo, a saber R. Una relación como “es más alto que” no es reflexiva pues ninguna per-
sona puede ser más alta que ella misma.

Definición de la propiedad simétrica


Una relación R en un conjunto X es simétrica si, y sólo si, para todos los elementos a y b en X,
cada vez que a está relacionado con b, entonces b también está relacionado con a. Esto es, si
1a, b2 僆 R, entonces 1b, a2 僆 R.

En términos del diagrama, cada par de puntos que tiene una flecha dirigida en una dirección
también tiene una flecha de regreso. Por ejemplo, si Beto tiene la misma inicial que Beti, enton-
ces Beti tiene la misma inicial que Beto. Una relación como “es hermano de” no es simétrica
pues Daniel puede ser hermano de Juana, pero Juana no puede ser hermano de Daniel.

Definición de la propiedad transitiva


Una relación R en un conjunto X es transitiva si, y sólo si, para todos los elementos a, b y c de X,
cada vez que a está relacionado con b y b está relacionado con c, entonces a está relacionado con
c. Esto es, si 1a, b2 僆 R y 1b, c2 僆 R, entonces 1a, c2 僆 R.

O B S E R VA C I Ó N a, b y c no tienen que ser diferentes. Se usan tres símbolos para permi-


tir la diferencia.

Nota que la relación en la figura 4-24 es transitiva. Por ejemplo, si Carolina tiene la
misma primera inicial que Candy y Candy tiene la misma primer inicial que Cati, entonces
Carolina tiene la misma primera inicial que Cati. Una relación como “es el padre de” no es
transitiva pues si Juan Pérez es el padre de Carlos Pérez y Carlos Pérez es el padre de José
Pérez, entonces Juan Pérez no es el padre de José Pérez, sino su abuelo.
La relación “es del mismo color que” es reflexiva, simétrica y transitiva. La conocida rela-
ción “es igual que” también satisface las tres propiedades. En general, las relaciones que sa-
tisfacen las tres propiedades se llaman relaciones de equivalencia.

Definición
Una relación de equivalencia es cualquier relación R que satisface las propiedades reflexiva,
simétrica y transitiva.

La relación de equivalencia natural que nos encontramos en la educación básica es “es igual
a” en el conjunto de todos los números. En capítulos subsecuentes veremos más ejemplos
de relaciones de equivalencia.
La propiedad de simetría de una relación es particularmente útil para determinar la natu-
raleza simétrica de funciones. Considera la relación x + y = 10, donde x y y son números

© Lopez Mateos Editores. ISBN 978-607-95583-2-1, obra completa, versión electrónica, ISBN 978-607-95583-3-8, volumen 1, versión electrónica. Ejemplar asignado a: Helecto Villarroel gutierrez -
helecto@gmail.com. Fecha: 27 de octubre de 2014. Prohibida su modificación, copia o distribución.
Sección 4-3 Funciones 235

completos. La relación consta de los 11 pares ordenados graficados en la figura 4-25; los
puntos están sobre la recta punteada. Nota que si el par 1a, b2 está en la relación —esto es,
está sobre la gráfica— también lo está el par (b, a). Por ejemplo, 11, 92 está sobre la gráfica
porque 1 + 9 = 10, pero entonces (9, 1) también está sobre la gráfica porque 9 + 1 = 10.
Nota también que la relación es una función. Esto se puede ver en la gráfica donde para
cada entrada x, x = 0, 1, 2, 3, Á , 10, hay exactamente una salida y. También podemos ver
que la relación es una función pues x + y = 10 da y = 10 - x; esto es, para cada x en el
dominio hay una única y. El dominio y el rango de la función son el mismo conjunto
50, 1, 2, 3, Á , 106.

y
10
9
8
7
6
5
4
3
2
1
0 1 2 3 4 5 6 7 8 9 10 x

Figura 4-25

AHORA INTENTA ÉSTE 4-11


a. A partir de la figura 4-25, explica por qué el dominio y el rango de la función y = 10 - x, donde tanto
x como y son números completos, es el conjunto 50, 1, 2, 3, Á , 106.
b. Muestra por qué la función y = x + 10, donde x y y son números completos, no es una relación simétrica.

Evaluación 4-3A

1. Los siguientes conjuntos de pares ordenados son funcio- 3. a. Traza un diagrama de flechas de una función con do-
nes. Da una regla que describa cada función. minio 51, 2, 3, 4, 56 y rango 5a, b6.
a. 512, 42, 13, 62, 19, 182, 112, 2426 b. ¿Cuántas funciones posibles hay en la parte (a)?
b. 512, 82, 15, 112, 17, 132, 14, 1026 4. Supón que f 1x2 = 2x + 1 y su dominio es 50, 1, 2, 3, 46.
2. ¿Cuáles son funciones del conjunto51, 2, 36 al conjunto Describe la función de las maneras siguientes:
5a, b, c, d6? Si el conjunto de pares ordenados no es una a. Traza un diagrama de flechas entre los dos conjuntos.
función, explica por qué. b. Usa pares ordenados.
a. 511, a2, 12, b2, 13, c2, 11, d26 c. Haz una tabla.
b. 511, a2, 12, b2, 13, a26 d. Traza una gráfica que describa la función.

© Lopez Mateos Editores. ISBN 978-607-95583-2-1, obra completa, versión electrónica, ISBN 978-607-95583-3-8, volumen 1, versión electrónica. Ejemplar asignado a: Helecto Villarroel gutierrez -
helecto@gmail.com. Fecha: 27 de octubre de 2014. Prohibida su modificación, copia o distribución.
236 Razonamiento algebraico

5. Di cuáles de las siguientes son funciones de C = 50, 1, 2, a. Haz una gráfica que muestre el costo por minuto
3, Á 6 o de un subconjunto de C a C. Si alguna no es como función del número de minutos y la otra que
función, explica por qué no. muestre el costo total para las llamadas como función
a. f 1x2 = 2 para toda x 僆 C del número de minutos, hasta 100 minutos.
b. f 1x2 = x b. Si conectas los puntos de la segunda gráfica de la parte
c. f 1x2 es la suma de los dígitos en x para toda x 僆 C. (a), ¿qué hipótesis debe hacerse acerca de la manera
6. a. Haz un diagrama de flechas para cada caso: como cobra las llamadas la compañía telefónica?
(i) Regla: “duplicado da” c. ¿Por qué el costo total de las llamadas consta de dos
segmentos de recta? ¿Por qué una está más inclinada
A B que la otra?
1 d. La función que representa el costo total de las llama-
1 2 das como función del número de minutos hablados,
2 3 se puede representar mediante dos ecuaciones. Es-
4
3 5 cribe las ecuaciones.
4 6 11. Para cada una de las sucesiones siguientes, halla una po-
7 sible función f 1n2 cuyo dominio sea el conjunto de los
8
números naturales y cuyas salidas sean los términos de la
sucesión.
(ii) Regla: “es mayor que” a. 3, 8, 13, 18, 23, Á
b. 3, 9, 27, 81, 243, Á
A B
12. Considera dos máquinas-función colocadas como se
1 1 muestra. Halla la salida final para cada una de las entra-
2 2 das siguientes:
3 3 a. 5 b. 10
4 4
x
b. ¿Cuál de las partes de (a), si la hay, exhibe una fun-
ción de A a B? Si existe una función, di por qué y ha-
lla su rango.
7. La dosis de cierta medicina está relacionada con el peso f (x) = 7x
de un niño de la manera siguiente: 50 mg, y 15 mg adi-
cionales por cada kilo o fracción, para un peso que re- f (x)
base 13 kg. Traza la gráfica de la dosis como función del
peso para niños cuyo peso esté entre 8 y 18 kg.
8. Si la tarifa de un taxi es de $35 por los primeros 500 me-
tros y $7.50 por cada 250 metros adicionales, entonces: g(x) = x – 5
a. ¿Cuál es el costo de un viaje de 2 km?
b. Escribe una regla para calcular la tarifa de un viaje en
taxi de n km si n es un número natural. (g f )(x)
9. Descubre una posible regla usada por Laura para res-
ponder. En cada caso, si n es tu entrada y L1n2 es la 13. Sea t1n2 la representación del término n-ésimo de una
respuesta de Laura, expresa L1n2 en términos de n. sucesión para n 僆 N. Responde:
a. b. a. Si t1n2 = 4n - 3, determina cuáles son valores de la
Tú Laura Tú Laura función:
3 8 0 1 (i) 1 (ii) 385 (iii) 389 (iv) 392
4 11 3 10 b. Si t1n2 = n2, determina cuáles son valores de la fun-
5 14 5 26 ción:
8 65 (i) 0 (ii) 25 (iii) 625 (iv) 1000 (v) 90
10 29
c. Si t1n2 = n1n - 12, determina cuáles están en el
rango de la función:
10. En los Principios y objetivos para los grados 6 a 8, en la sec- (i) 0 (ii) 2 (iii) 20 (iv) 999
ción de “Álgebra” (p. 229) se plantea el siguiente pro- 14. Considera una máquina-función que acepte pares orde-
blema: TuCelu anuncia un servicio mensual de teléfono nados como entradas. Supón que las componentes de los
celular por $5 el minuto para los primeros 60 minutos, pares ordenados son números naturales y que la primera
pero sólo $1 el minuto por cada minuto siguiente. Tu- componente es la longitud de un rectángulo y la segunda
Celu cobra, además, sólo el tiempo exacto usado. Con- componente es su ancho. La máquina siguiente calcula
testa lo siguiente:

© Lopez Mateos Editores. ISBN 978-607-95583-2-1, obra completa, versión electrónica, ISBN 978-607-95583-3-8, volumen 1, versión electrónica. Ejemplar asignado a: Helecto Villarroel gutierrez -
helecto@gmail.com. Fecha: 27 de octubre de 2014. Prohibida su modificación, copia o distribución.
Sección 4-3 Funciones 237

el perímetro (la distancia alrededor de una figura) del a. Halla A122, A162, A132 y A152. ¿Por qué son iguales
rectángulo. Así, para un rectángulo cuya longitud l es 3 algunas salidas?
y cuyo ancho a es 2, la entrada es (3, 2) y la salida es b. Grafica la función y, a partir de la gráfica, halla en qué
2 # 3 + 2 # 2, ó 10. Responde: instante la bola está en su punto más alto. ¿Cuál es su
altura en ese instante?
(l, a) c. ¿Cuánto tardará la bola en llegar al suelo?
d. ¿Cuál es el dominio de A?
e. ¿Cuál es el rango de A?
17. Para cada una de las siguientes sucesiones de figuras for-
a madas con cerillos, sea S1n2 la función que da el número
total de cerillos en la n-ésima figura.
a. Para cada una, halla el número total de cerillos em-
pleados en la cuarta figura.
a. Para cada una de las entradas: (1, 7), (2, 6), (6, 2) y (5, b. Para cada una, halla una fórmula lo más sencilla posi-
5), halla la salida correspondiente. ble para S1n2 en términos de n.
b. Halla el conjunto de todas las entradas cuya salida (i)
sea 20.
c. ¿Cuál es el dominio y el rango de la función?
15. La gráfica siguiente muestra la relación entre el número , , , ...
de carros en cierta carretera y la hora, en tiempos dife-
rentes, entre las 5:00 a.m. y las 9:00 a.m.: (ii)

Patrones de tráfico
, , ... , ...

900 18. Supón que continúa el patrón mostrado en cada una de


las sucesiones siguientes de figuras formadas por ladri-
800 llos cuadrados. Sea S1n2 la función que da el número to-
tal de ladrillos en la n-ésima figura. Para cada una, halla
Número de carros

700 una fórmula para S1n2 en términos de n. En la parte (b),


cada cuadrado se divide en cuatro cuadrados para obte-
600 ner la figura subsecuente.
500

400
a. , , , ...
300

5 A.M. 6 A.M. 7 A.M. 8 A.M. 9 A.M.


Tiempo
b. , , , ...

a. ¿Cuál es el incremento en el número de carros en la 19. Una función se puede representar como un conjunto de
carretera entre las 6:30 a.m. y las 7:00 a.m.? pares ordenados donde el conjunto de todas las primeras
b. ¿Durante cuál media hora hubo el mayor incremento componentes es el dominio y el conjunto de las segundas
en el número de carros? componentes es el rango. ¿Es cierto lo recíproco? Esto
c. ¿Cuál fue el incremento en el número de carros entre es, ¿todo conjunto de pares ordenados es una función
las 8:00 a.m. y las 8:30 a.m.? cuyo dominio es el conjunto de las primeras componen-
d. ¿Durante cuál o cuáles medias horas decreció el nú- tes y su rango el de las segundas componentes? Justifica
mero de carros? ¿En cuánto? tu respuesta.
e. La gráfica para este problema está compuesta de seg- 20. ¿Cuáles de las ecuaciones o desigualdades siguientes re-
mentos en lugar de sólo puntos como en la figura 4-20. presentan funciones y cuáles no? En cada caso x y y son
¿Por qué crees que se usan segmentos en lugar de sólo números completos. Justifica tus respuestas.
puntos? a. x + y = 2
16. Se lanza una bola hacia arriba. Sabemos que su altura A, b. x - y 6 2
en metros, después de t segundos está dada por la función c. y = x3 + x
A1t2 = 128t - 16t 2. d. xy = 2

© Lopez Mateos Editores. ISBN 978-607-95583-2-1, obra completa, versión electrónica, ISBN 978-607-95583-3-8, volumen 1, versión electrónica. Ejemplar asignado a: Helecto Villarroel gutierrez -
helecto@gmail.com. Fecha: 27 de octubre de 2014. Prohibida su modificación, copia o distribución.
238 Razonamiento algebraico

21. ¿Cuáles de las siguientes son gráficas de funciones y cuá- 22. Supón que cada punto en la figura representa a una niña o
les no? Justifica tus respuestas. niño en un jardín, que las letras representan sus nombres y
a. b. que una flecha de I a J significa que I “es hermana de” J.
y y

E
A G
B
F
3 H
2 1 D C
1 I
0 1 2 3 x 0 1 x J
c.
y a. Con base en la información de la figura, ¿quiénes son
con certeza niñas y quiénes son con certeza niños?
4 b. Supongamos que escribimos “A es hermana de B”
como el par ordenado 1A, B2. Con base en la infor-
3
mación del diagrama, escribe el conjunto de dichos
2 pares ordenados.
c. ¿Es el conjunto de los pares ordenados en (b) una fun-
1
ción con dominio igual al conjunto de todas las prime-
x ras componentes de los pares ordenados y rango igual
0 1 2 3 4
al conjunto de todas las segundas componentes?

Evaluación 4-3B

1. Los siguientes conjuntos de pares ordenados son funcio- 6. Dados los siguientes diagramas de flechas para funciones
nes. Da una regla que describa cada función. de A a B, da una posible regla para la función:
a. 515, 32, 17, 52, 111, 92, 114, 1226 a. A B
b. 512, 52, 13, 102, 14, 172, 15, 2626
2. ¿Cuáles son funciones del conjunto 51, 2, 36 al conjunto 3 7
5a, b, c, d6? Si el conjunto de pares ordenados no es una 5 11
función, explica por qué no. 7 15
a. 511, c2, 13, d26 10 21
b. 511, a2, 11, b2, 11, c26
3. a. Traza un diagrama de flechas de una función con do-
minio 51, 2, 36 y rango 5a, b6. b. A B
b. ¿Cuántas funciones posibles hay en la parte (a)?
4. Supón que f 1x2 = 21x + 12 y el dominio es 50, 1, 2, 3, 46.
(0, 3) 3
(3, 5) 14
Describe la función de las maneras siguientes: (2, 9) 13
a. Traza un diagrama de flechas entre dos conjuntos.
(4, 5) 21
b. Usa pares ordenados. c. Haz una tabla.
d. Traza una gráfica que describa la función.
5. Di cuáles de las siguientes son funciones de C = 50, 1, 2,
3, Á 6 o de un subconjunto de C a C. Si alguna no es
7. De acuerdo con expertos en vida silvestre, la tasa de chirri-
dos de los grillos es función de la temperatura; específica-
función, explica por qué no.
a. f 1x2 = 0 si x 僆 50, 1, 2, 36 y f 1x2 = 3 si
mente, C = T - 40, donde C es el número de chirridos
x 僆 50, 1, 2, 36
cada 15s y T es la temperatura en grados Fahrenheit.
b. f 1x2 = 0 para toda x 僆 C y f 1x2 = 1 si
a. ¿Cuántos chirridos por segundo emite un grillo si la
x 僆 53, 4, 5, 6, Á 6
temperatura es de 70°F?
c. f 1x2 es el dígito de las unidades de x para todo x 僆 C.
b. ¿Cuál es la temperatura si el grillo emite 40 chirridos
en 1 min?

© Lopez Mateos Editores. ISBN 978-607-95583-2-1, obra completa, versión electrónica, ISBN 978-607-95583-3-8, volumen 1, versión electrónica. Ejemplar asignado a: Helecto Villarroel gutierrez -
helecto@gmail.com. Fecha: 27 de octubre de 2014. Prohibida su modificación, copia o distribución.
Sección 4-3 Funciones 239

8. Descubre una posible regla usada por Laura para respon- a. 2, 4, 6, 8, 10, Á
der. En cada caso, si n es tu entrada y L1n2 es la respuesta b. 1, 3, 9, 27, 81, Á
de Laura, expresa L1n2 en términos de n. c. 2, 2 + 4, 2 + 4 + 6, 2 + 4 + 6 + 8, Á
a. Tú Laura b. Tú Laura 11. Considera dos máquinas-función colocadas como se
muestra. Halla la salida final para cada una de las entra-
6 42 0 1 das siguientes:
0 0 1 2 a. 5 b. 3 c. 10 d. a
8 72 5 32
2 6 6 64 x
10 1024
9. Los Principios y objetivos para los grados 6 a 8 señalan que
“al estudiar álgebra, los estudiantes de grados medios de-
berán encontrar preguntas acerca de cantidades que cam- f (x) = x – 1
bian” (p. 229). Se plantea el siguiente problema.
ChismeCel cobra $4.5 por minuto en llamadas de f (x)
teléfono celular. No cambia el costo por minuto, pero el
costo total cambia conforme se usa el teléfono.

Costo de teléfono celular por minuto g(x) = 7x

$36.0
$31.5
Costo por minuto

(g f )(x)
$27.0
$22.5
12. Sea t1n2 la representación del término n-ésimo de una
$18.0
$13.5 sucesión para n 僆 N. Responde:
$9.0 a. Si t1n2 = n2, determina cuáles son valores de la función:
$4.5 (i) 1 (ii) 4 (iii) 9 (iv) 10 (v) 900
b. Si t1n2 = n1n + 12, determina cuáles están en el
0 1 2 3 4 5 6 7 8 9 10 rango de la función:
Número de minutos (i) 2 (ii) 12 (iii) 2550 (iv) 2600
13. Considera una máquina-función que acepte pares
Costo total de teléfono celular ordenados como entradas. Supón que las componentes
de los pares ordenados son números naturales y que la
$36.0 primera componente es la longitud de un rectángulo y
Costo total por llamadas

$31.5 la segunda componente es su ancho. La máquina si-


$27.0 guiente calcula el perímetro (la distancia alrededor de
(8, $36.0)
$22.5 una figura) del rectángulo. Así, para un rectángulo cuya
$18.0 longitud l es 3 y cuyo ancho a es 1, la entrada es (3, 1) y
$13.5 la salida es 2 # 3 + 2 # 1, u 8. Responde:
$9.0 a. Para cada una de las entradas (1, 4), (2, 1), (1, 2), (2,
$4.5 2) y (x, y), halla la salida correspondiente.
b. Halla el conjunto de todas las entradas cuya salida
0 1 2 3 4 5 6 7 8 9 10
sea 20.
Número de minutos c. ¿Es (2, 2) una salida posible? Explica.
a. Cuando el número de minutos es 6, ¿qué represen-
tan los puntos correspondientes en cada gráfica?
b. ¿Qué tipo de hipótesis acerca de los cargos se nece- (l, a)
sita hacer para permitir la conexión de los puntos en
cada gráfica? Explica.
c. Si el tiempo en minutos es t y el costo total de las lla-
madas es c, escribe c como función de t en cada gráfica. a
10. Para cada una de las sucesiones siguientes, halla un pa-
trón y una posible función cuyo dominio sea el conjunto
de los números naturales y cuyas salidas sean los térmi-
nos de la sucesión: 14. Un club cobra un pago único de $1,000 por membresía
más una cuota mensual de $400.

© Lopez Mateos Editores. ISBN 978-607-95583-2-1, obra completa, versión electrónica, ISBN 978-607-95583-3-8, volumen 1, versión electrónica. Ejemplar asignado a: Helecto Villarroel gutierrez -
helecto@gmail.com. Fecha: 27 de octubre de 2014. Prohibida su modificación, copia o distribución.
240 Razonamiento algebraico

a. Escribe una expresión para la función de costo C(x) 18. Estás a 20 km de tu casa y te alejas en tu coche a una ra-
que dé el costo total de pertenencia al club durante x pidez constante de 60 km/h. Describe la distancia S a la
meses. que estás de tu casa como función del tiempo t en horas
b. Traza la gráfica de la función en (a). que llevas manejando.
c. El club decide brindar a sus miembros una oferta: au- 19. Supón que continúa el patrón mostrado en cada una de las
menta la membresía pero disminuye la cuota men- sucesiones siguientes de figuras formadas por ladrillos
sual. Si la membresía cuesta $3,000 y la cuota cuadrados. Sea S1n2 la función que da el número total de
mensual es de $300, usa otro color y traza sobre el ladrillos en la n-ésima figura. Para cada una, halla una fór-
mismo conjunto de ejes la gráfica que representa este mula para S1n2 en términos de n.
nuevo plan. a.
d. Determina, a partir de las gráficas, después de cuán-
tos meses el segundo plan es más barato. , , , ...
15. Se lanza una bola hacia arriba. Sabemos que su altura A
en metros, después de t segundos, está dada por la fun- b.
ción A1t2 = 128t - 16t2.
a. Grafica la función y, a partir de la gráfica, halla en qué , , , ...
instante la bola está en su punto más alto. ¿Cuál es su
altura en ese instante? 20. Una función se puede representar como un conjunto de
b. De la gráfica, halla todos los t tales que A1t2 = A112. pares ordenados donde el conjunto de todas las primeras
c. ¿Cuánto tardará la bola en llegar al suelo? Verifica tu componentes es el dominio y el conjunto de las segundas
respuesta. componentes es el rango. Si cada par ordenado 1a, b2 se
d. ¿Cuál es el dominio de A? reemplaza por 1b, a2, ¿es una función el nuevo conjunto?
e. ¿Cuál es el rango de A? 21. ¿Cuáles de las ecuaciones o desigualdades siguientes re-
16. Se va a cercar una porción rectangular de terreno acotado presentan funciones y cuáles no? En cada caso, x y y son
en un lado por un río recto y en los otros lados por una números completos. Justifica tus respuestas.
cerca. Supón que disponemos de 800 m de cerca y que de- a. x - y = 2 b. x + y 6 20
notamos con x el lado del rectángulo paralelo al río. c. y = 2x2 d. y = x3 - 1
a. Halla una expresión para el área A1x2 en términos de x. 22. ¿Cuáles de las siguientes son gráficas de funciones y cuá-
b. Grafica A1x2. les no? Justifica tus respuestas.
c. Usa la gráfica en (b), o tu calculadora, para estimar la a. y
longitud y el ancho del rectángulo cuya área sea la más 2
grande.
1
Río

0 1 2 3 4 x
b. y
x 2
17. Para cada una de las siguientes sucesiones de figuras for-
1
madas con cerillos, sea S(n) la función que da el número
total de cerillos en la n-ésima figura, suponiendo que el
patrón continúa 0 1 2 3 x
a. Para cada una, halla el número total de cerillos emple-
ados en la cuarta figura. c. y
2
b. Para cada una, halla una fórmula lo más sencilla posi-
ble para S1n2 en términos de n.
1
(i)

0 1 2 3 x

, , ... , ... d. y
2
(ii)
1

, , , ... 0 1 2 3 x

© Lopez Mateos Editores. ISBN 978-607-95583-2-1, obra completa, versión electrónica, ISBN 978-607-95583-3-8, volumen 1, versión electrónica. Ejemplar asignado a: Helecto Villarroel gutierrez -
helecto@gmail.com. Fecha: 27 de octubre de 2014. Prohibida su modificación, copia o distribución.
Sección 4-3 Funciones 241

23. a. ¿Cuáles de las siguientes relaciones del conjunto C de componentes de los pares ordenados al conjunto de las
números completos a C tienen la propiedad simétrica? segundas componentes?
Justifica tus respuestas. a. 5(Sinaloa, Guasave), (Yucatán, Mérida), (Coahuila,
(i) x + y = 10 (ii) x - y = 100 Monclova), (Michoacán, Uruapan)6
(iii) xy = 100 (iv) y = x b. 5(Sonora, Hermosillo), (Oaxaca, Huajuapan), (Oa-
(v) y = x2 xaca, Juchitán), (Chiapas, Tapachula)6
b. ¿Cuáles de las relaciones en la parte (a) son funcio- c. 51x, y2 ƒ x reside en Cuernavaca, Morelos y x es madre
nes? Justifica tus respuestas. de y, donde y es residente en México6
24. Supón que cada punto en la figura representa a una niña d. 511, 12, 12, 42, 13, 92, 14, 1626
e. 51x, y2 ƒ x y y son números naturales y x + y es un nú-
o niño en un jardín, que las letras representan sus nom-

mero par6
bres y que una flecha de I a J significa que I “es hermana
de” J.
26. a. Considera la relación formada por los pares ordenados
1x, y2 tales que y es la madre biológica de x. ¿Es una fun-
ción cuyo dominio es el conjunto de todas las personas?
E b. Como en la parte (a) pero ahora y es hermano de x.
A G ¿Es la relación una función del conjunto de todos los
B niños al conjunto de todos los niños?
F 27. Considerando el conjunto de todas las personas, men-
H
D C ciona cuál es reflexiva, simétrica o transitiva. ¿Cuáles son
I relaciones de equivalencia?
a. “Es padre de”
J
b. “Tiene la misma edad que”
c. “Tiene el mismo apellido que”
a. Con base en la información de la figura, ¿quiénes son d. “Tiene la misma estatura que”
con certeza niñas y quiénes son con certeza niños? e. “Está casada con”
b. Supongamos que escribimos “A es hermana de B” f. “Vive a menos de 10 km de”
como el par ordenado (A, B). Con base en la informa- g. “Es mayor que”
ción del diagrama, escribe el conjunto de dichos pa- 28. Di cuál es reflexiva, simétrica o transitiva en el conjunto
res ordenados. de subconjuntos de un conjunto no vacío. ¿Cuáles son
c. ¿Es el conjunto de los pares ordenados en (b) una fun- relaciones de equivalencia?
ción con dominio igual al conjunto de todas las prime- a. “Es igual a”
ras componentes de los pares ordenados y rango igual al b. “Es un subconjunto propio de”
conjunto de todas las segundas componentes? c. “No es igual a”
25. ¿Cuáles de las siguientes son funciones y cuáles son rela- d. “Tiene el mismo número cardinal que”
ciones, pero no funciones, del conjunto de las primeras

Conexiones matemáticas 4-3

Comunicación 3. ¿Cuáles de las siguientes son funciones de A a B? Si la res-


1. ¿Define el diagrama una función de A a B? ¿Por qué sí o puesta es “no es función”, explica por qué no.
por qué no? a. A es el conjunto de maestras y maestros de matemáti-
cas en la universidad. B es el conjunto de grupos de
A B
matemáticas. A cada maestra o maestro le asociamos el
Raúl Ale grupo al que imparte clases en ese semestre.
Ruiz b. A es el conjunto de grupos de matemáticas en la uni-
Mari
versidad y B es el conjunto de maestras y maestros. A
Sonia cada grupo de matemáticas le asociamos el o la maes-
Beto Jorge tra que imparte la clase.
Ruiz c. A es el conjunto de senadores y B es el conjunto de co-
Lichi
misiones del Senado. Asociamos a cada senador con el
comité que preside.
4. Si C es el conjunto de alumnos en la clase de la profesora
2. ¿Es función una correspondencia biunívoca? Explica tu
Carmelita y A es cualquier subconjunto de S, definimos:
respuesta y da un ejemplo.
© Lopez Mateos Editores. ISBN 978-607-95583-2-1, obra completa, versión electrónica, ISBN 978-607-95583-3-8, volumen 1, versión electrónica. Ejemplar asignado a: Helecto Villarroel gutierrez -
helecto@gmail.com. Fecha: 27 de octubre de 2014. Prohibida su modificación, copia o distribución.
242 Razonamiento algebraico

f 1A2 = A (donde A es el complemento de A). Nota que piso a la parte superior del libro. Anoten esa distancia.
en esta función la entrada es un subconjunto de C y que la b. Coloquen un segundo libro de matemáticas encima
salida es un subconjunto de C. Responde a lo siguiente: del primero y midan la distancia (el centímetro más
a. Explica por qué f es una función y describe el dominio cercano) del piso a la parte superior del segundo li-
y el rango de f. bro. Anoten esa distancia.
b. Si hay 20 niñas y niños en la clase, ¿cuál es el número c. Continúen este procedimiento con sus cuatro libros de
de elementos en el dominio y el número en el rango?
Explica. Número de libros Distancia desde el piso
c. ¿La función en esta pregunta es una correspondencia
biunívoca? Justifica tu respuesta. 1
2
Solución abierta 3
5. Busca en periódicos y revistas al menos tres ejemplos de 4
funciones y descríbelos. ¿Cuáles son el dominio y el

Distancia desde el piso (cm)


rango de cada función?
6. Da al menos tres ejemplos de funciones de A a B donde 95
90
ni A ni B sean conjuntos de números. 85
7. Traza una sucesión de figuras con cerillos y describe con 80
palabras el patrón. Halla la expresión más sencilla posi- 75
70
ble para S1n2, el número total de cerillos en la n-ésima 65
figura. 60
8. Una función cuya salida sea la misma, independiente-
mente de la entrada, es una función constante. Da varios 1 2 3 4 5 6
ejemplos de funciones constantes, tomados de la vida real. Número de libros de matemáticas
9. Una función cuya salida es igual que su entrada se llama matemáticas y completen la tabla y la gráfica siguientes:
función identidad. Da varios ejemplos concretos de funcio- d. Sin medir, ¿cuál es la distancia desde el piso con 0 li-
nes identidad. bros? ¿y con 5 libros?
Aprendizaje colectivo e. Escribe una regla o función para d1x2, donde d1x2 sea
la distancia del piso a la parte superior de la pila de li-
10. Cada persona de un grupo escoge un número natural y lo
bros y x sea el número de libros.
f. Supón que la distancia del piso al techo es de 2.5 m. Si
Entrada los libros se apilan como se describió, ¿cuántos libros
se necesitarán para llegar al techo?
g. La función h1x2 = 34x + 70 representa la altura de
Multiplicar por 4, sumar
otra pila de libros de x libros de matemáticas (en cen-
8, dividir entre 4,
tímetros) sobre un mueble. ¿Qué les dice la función
restar la entrada.
acerca de la altura del mueble? ¿Qué les dice acerca
Salida
del grosor de cada libro?
usa como entrada en la siguiente máquina-función: h. Supón ahora que una mesa con una pila similar de li-
a. Compara tus respuestas. Con base en las respuestas, bros de matemáticas (más de 10) mide 200 cm de alto.
emite una conjetura acerca de cuál es el rango de la Si quitamos el libro de arriba, la altura es de 197 cm.
función. Si quitamos un segundo libro, la altura es de 194 cm.
b. Con base en tu respuesta en (a), grafica la función. ¿Cuál es la altura si quitamos 5 libros?
c. Escribe la función de la manera más sencilla posible i. Escribe una función h1x2 para la altura de la pila des-
usando la notación f 1x2. pués de quitar x libros.
d. Justifica la conjetura emitida en (a). Preguntas del salón de clase
e. Construye máquinas-función similares y pruébalas
12. Un alumno asegura que la siguiente máquina no repre-
con diferentes entradas en tu grupo.
senta una máquina-función pues acepta dos entradas si-
f. Diseña una máquina-función en la cual se ejecuten
varias operaciones pero que la salida siempre sea igual Entrada (x, y)
a la entrada. Intercambia tu respuesta con otras per-
sonas y verifica que las máquinas-función de los otros
se comporten según lo requerido.
11. Trabajen la siguiente actividad en grupos de cuatro. Ne-
x1y
cesitarán una cinta métrica o un metro.
a. Coloquen su libro de matemáticas sobre un escritorio
y midan la distancia (al centímetro más cercano) del
Salida

© Lopez Mateos Editores. ISBN 978-607-95583-2-1, obra completa, versión electrónica, ISBN 978-607-95583-3-8, volumen 1, versión electrónica. Ejemplar asignado a: Helecto Villarroel gutierrez -
helecto@gmail.com. Fecha: 27 de octubre de 2014. Prohibida su modificación, copia o distribución.
Sección 4-3 Funciones 243

multáneamente en lugar de una sola. ¿Cómo le respondes? Cada figura en el patrón a continuación está formada
13. Una alumna pregunta, “Si toda sucesión es una función, por hexágonos que miden 1 centímetro por lado.
¿también es cierto que toda función es una sucesión?”
¿Cómo le respondes?
14. Un alumno asegura que lo siguiente no representa una
función pues todos los valores de x corresponden al mis-
Figura 1 Figura 2
x 0 1 2 3 4 5 Perímetro = 6 cm Perímetro = 10 cm

y 1 1 1 1 1 1

mo número.
¿Cómo le respondes? Figura 3 Figura 4
15. Una alumna piensa que la función f 1x2 = 3x + 5 con Perímetro = 14 cm Perímetro = 18 cm
dominio en todos los números completos es una corres-
pondencia biunívoca y quisiera saber por qué. ¿Cómo le Si se continúa el patrón de añadir un hexágono a cada fi-
respondes? gura, ¿cuál será el perímetro de la figura número 25 del
16. Un alumno quiere saber por qué a veces es incorrecto patrón? Ilustra cómo obtuviste tu respuesta.
conectar los puntos en la gráfica de una función. ¿Cómo NAEP, Grado 8, 2007
le respondes? En la ecuación y = 4x, si el valor de x se incrementa
en 2, ¿cuál es el efecto en el valor de y?
Problemas de repaso
a. Es 8 más que la cantidad original.
17. De ser posible, resuelve las ecuaciones siguientes: b. Es 6 más que la cantidad original.
a. 3x - 1 = x + 99 c. Es 2 más que la cantidad original.
b. 215x + 12 - 11 = x + 9 d. Es 16 por la cantidad original.
c. 31x - 12 = 21x - 12 + 99 e. Es 8 por la cantidad original.
d. 512x - 62 = 312x - 62
NAEP, Grado 8, 2007
18. Resuelve el siguiente problema planteando una ecuación
adecuada: Preguntas del Third International Mathematics and
Dos carros, cada uno viajando a rapidez constante, uno a Science Study (TIMSS) (Tercer Estudio Internacional
60 km/h y el otro a 70 km/h, comienzan al mismo tiempo sobre Matemáticas y Ciencia)
desde el mismo punto y viajan en la misma dirección. Una máquina numérica toma un número y opera sobre
¿Después de cuántas horas la distancia entre ellos será de él. Cuando el número de entrada es 5 el número de sa-
40 km? lida es 9, como se muestra a continuación.
Preguntas del National Assessment of Educational Número de Número de
Progress (NAEP) (Evaluación Nacional del Progreso entrada salida
Educativo) 32 12 23
5 10 12 9
Entrada Salida

2 5 Cuando el número es 7, ¿cuál de éstos es el número de sa-


3 7 lida?
4 9 a. 11
5 11 b. 13
15 31 c. 14
38 d. 25
TIMSS 2003, Grado 4
La tabla muestra cómo se relacionan los números en
“Entrada” con los números en “Salida”. Cuando entra
38, ¿qué número sale?
a. 41 b. 51 c. 54 d. 77
NAEP, Grado 4, 2007

© Lopez Mateos Editores. ISBN 978-607-95583-2-1, obra completa, versión electrónica, ISBN 978-607-95583-3-8, volumen 1, versión electrónica. Ejemplar asignado a: Helecto Villarroel gutierrez -
helecto@gmail.com. Fecha: 27 de octubre de 2014. Prohibida su modificación, copia o distribución.
244 Razonamiento algebraico

Alberto tiene 50 manzanas. Vendió algunas y le quedaron


20. ¿Cuál de las expresiones numéricas muestra esto? 1 kg
a. n - 20 = 50
b. 20 - n = 50
c. n - 50 = 20
d. 50 - n = 20
TIMSS 2003, Grado 4 ¿Cuánto pesa (masa) un ladrillo?
Los objetos en la balanza hacen que ésta se nivele. En el a. 0.5 kg b. 1 kg
platillo izquierdo hay un peso de 1 kg (masa) y medio la- c. 2 kg d. 3 kg
drillo. En el otro platillo hay un ladrillo. TIMSS 2003, Grado 8

Sugerencia para resolver el problema preliminar


Llama x al número de cada estudiante y a a la respuesta final. Plantea una ecuación
que incluya x y a y despeja x en términos de a.

Resumen del capítulo

I. Variables III. Funciones y relaciones


A. Incógnita en una ecuación A. Una función de un conjunto A a B es una co-
B. Cambio de cantidad rrespondencia en la que cada elemento a H A
C. Aplicar el álgebra para resolver problemas está pareado con un, y sólo un, elemento b H B.
D. En una hoja de cálculo Si la función se denota con f, escribimos
II. Ecuaciones f1a2 = b. El elemento a H A es la entrada y f1a2
A. Propiedad de la suma: Para números a, b y c, es la salida. A es el dominio de la función. B es
si a = b, entonces a + c = b + c. cualquier conjunto que contenga todas las sali-
B. Propiedad de la multiplicación: Para núme- das. El conjunto de todas las salidas es el rango
ros a, b y c, si a = b, entonces ac = bc. o imagen de la función.
C. Propiedades de la cancelación: Para números B. Una función se puede representar ya sea me-
a, b y c, diante una tabla, una ecuación, un diagrama de
1. si a + c = b + c, entonces a = b. flechas, una máquina-función, un conjunto de
2. si c Z 0, y ac = bc, entonces a = b. pares ordenados o una gráfica.
D. La igualdad no se afecta si substituimos un nú- C. Una sucesión es una función cuyo dominio es
mero por su igual. N, el conjunto de números naturales.
E. Propiedades de la suma y la resta para las ecuaciones D. Cualquier conjunto de pares ordenados es una
1. If a = b y c = d, entonces a + c = b + d y relación.
a - c = b - d. 1. Una relación de A a B es un subconjunto de
F. Propiedades distributivas A * B.
1. a1b + c2 = ab + ac 2. Una relación puede tener una o más de las
2. a1b - c2 = ab - ac propiedades siguientes:
G. Resolución de ecuaciones a. Reflexiva
H. Problemas de aplicación b. Simétrica
c. Transitiva

© Lopez Mateos Editores. ISBN 978-607-95583-2-1, obra completa, versión electrónica, ISBN 978-607-95583-3-8, volumen 1, versión electrónica. Ejemplar asignado a: Helecto Villarroel gutierrez -
helecto@gmail.com. Fecha: 27 de octubre de 2014. Prohibida su modificación, copia o distribución.
Revisión del capítulo 245

Revisión del capítulo

1. Hay 13 veces más estudiantes que profesores en una 9. Juanita tiene 10 libros vencidos en la biblioteca. Ella
secundaria. Usa E para el número de estudiantes y recuerda que sacó 2 libros de ciencias dos semanas
P para el número de profesores a fin de representar antes de que sacara 8 libros infantiles. La multa dia-
la información dada. ria por libro es de $0.20. Si su multa total fue de
2. Escribe una frase que dé la misma información que $11.60, ¿cuánto tiempo tuvo vencido cada libro?
la ecuación siguiente: A = 103 # B, donde A es el 10. En un pueblo, tres niños entregan todos los perió-
número de niñas en una colonia y B es el número de dicos. Jacobo entrega el doble de periódicos que
niños. Dalia, quien entrega 100 más que Raquel. Si entre
3. Escribe una acuación para hallar el número de pies todos entregan un total de 500 periódicos, ¿cuántos
dado el número de yardas (sea p el número de pies y periódicos entregó cada uno?
y el número de yardas). 11. ¿Cuáles de los siguientes conjuntos de pares ordena-
4. La suma de un conjunto de n números completos es dos son funciones del conjunto de las primeras com-
S. Si cada número se multiplica por 10 y después se ponentes al conjunto de las segundas componentes?
disminuye en 10, ¿cuál es la suma del nuevo con- a. 51a, b2, 1c, d2, 1e, a2, 1f, g26
b. 51a, b2, 1a, c2, 1b, b2, 1b, c26
junto en términos de n y S ?
c. 51a, b2, 1b, a26
5. Estoy pensando un número completo. Si lo divido
entre 13, después multiplico el resultado por 12,
luego le resto 20 y le sumo 89, el resultado es 93. 12. Dadas las siguientes reglas de la función y los domi-
¿Cuál es mi número original? nios, halla los rangos asociados:
6. a. Piensa un número. a. f1x2 = x + 3; dominio = 50, 1, 2, 36
Súmale 17. b. f1x2 = 3x - 1; dominio = 55, 10, 15, 206
Dobla el resultado. c. f1x2 = x2; dominio = 50, 1, 2, 3, 46
Réstale 4. d. f1x2 = x2 + 3x + 5; dominio = 50, 1, 26
Dobla el resultado.
13. ¿Cuáles de las correspondencias siguientes de A a B
Súmale 20.
describen una función? Si una correspondencia es
Divídelo entre 4.
función, halla su rango. Justifica tu respuesta.
Réstale 20.
Tu respuesta será tu número original. Explica a. A es el conjunto de los estudiantes de bachille-
cómo funciona este truco. rato y B es el conjunto de especialidades. A cada
b. Plantea tres pasos más que te regresen a tu nú- estudiante de bachillerato le corresponde una es-
mero original. pecialidad.
Piensa un número. b. A es el conjunto de libros de la biblioteca y B es
Súmale 18. el conjunto N de números naturales. A cada libro
Multiplícalo por 4. le corresponde el número de páginas del libro.
Réstale 7. c. A = 51a, b2 ƒ a 僆 N y b 僆 N6, y B = N. A cada
# elemento de A le corresponde el número 4a + 2b.
# d. A = N y B = N. Si x es par, entonces f1x2 = 0,
# y si x es impar, entonces f1x2 = 1.
c. Elabora una serie de instrucciones que te regre- e. A = N y B = N. A cada número natural le co-
sen siempre a tu número original. rresponde la suma de sus dígitos.
7. Halla todos los valores de x que satisfagan las ecua- 14. Un club cobra una membresía de $2,000 que incluye
ciones siguientes: 1 mes gratis, y después cuesta $550 mensuales.
a. 4x - 2 = 3x + 10 a. Si C1x2 es el costo total de pertenecer al club du-
b. 41x - 122 = 2x + 10 rante x meses, expresa C1x2 en términos de x.
c. 417x - 212 = 1417x - 212 b. Grafica C1x2 para los primeros 12 meses.
d. 213x + 52 = 6x + 11
c. Usa la gráfica en (b) para averiguar cuándo el
e. 31x + 12 + 1 = 3x + 4
costo total de pertenencia excederá de $6,000.
8. Miguel tiene 3 veces más tarjetas de beisbol que Juan,
d. ¿Cuándo el costo total excederá de $60,000?
quien tiene el doble de tarjetas que Pati. Entre los
tres tienen 999 tarjetas. Plantea una ecuación en una 15. Si la regla para la función es f1x2 = 4x - 5 y la sa-
variable y halla cuántas tarjetas tiene cada quien. lida es f1x2 = 15, ¿cuál es la entrada?

© Lopez Mateos Editores. ISBN 978-607-95583-2-1, obra completa, versión electrónica, ISBN 978-607-95583-3-8, volumen 1, versión electrónica. Ejemplar asignado a: Helecto Villarroel gutierrez -
helecto@gmail.com. Fecha: 27 de octubre de 2014. Prohibida su modificación, copia o distribución.
246 Razonamiento algebraico

16. ¿Cuáles de las siguientes gráficas representan fun- 17. a. Julia construye torres con cubos, colocando un
ciones? Di por qué. cubo arriba de otro y pintando la torre (incluida
a. y la tapa y la base pero no las caras que se tocan en-
tre sí). Halla el número de caras que Julia nece-
5 sita pintar para torres formadas con 1, 2, 3, 4, 5 y
4 6 cubos, y llena la tabla siguiente:
3 # de cubos # de cuadrados por pintar
2
1 6
1
2 10
x 3
1 2 3 4 5
4
b. y
5
5 6
4
3 b. Grafica la información hallada en la parte (a)
2 donde el número de cubos de la torre está sobre
el eje x horizontal y el número de cuadrados por
1
pintar está sobre el eje vertical.
1 2 3 4 5 x c. Si x es el número de cubos en una torre y y es el
c. y número correspondiente de cuadrados por pin-
tar, escribe una ecuación que exprese y como
5 una función de x.
4 d. ¿La gráfica que describe el número de cuadrados
como función del número de cubos contiene un
3
segmento de recta?
2
1

1 2 3 4 5 x

Bibliografía seleccionada

Adams, T.L. “Addressing Students’ Difficulties with the Carpenter, T.P., M.L. Franke, and L. Levi. Thinking
Concept of Function: Applying Graphing Calculators Mathematically: Integrating Arithmetic and Algebra in
and a Model of Conceptual Change.” Focus on Learn- Elementary School. Portsmouth, NH: Heinemann
ing Problems in Mathematics (1997) 19(2): 43–57. (2003).
Billings, E., T.L. Tiedt, and L.H. Slater. “Algebraic Chappell, M., and M. Strutchens. “Creating Connec-
Thinking and Pictorial Growth Patterns.” Teaching tions: Promoting Algebraic Thinking with Con-
Children Mathematics 14 (December 2007–January crete Models.” Mathematics Teaching in the Middle
2008): 302–308. School 7 (September 2001): 20–25.
Bishop, J., A. Otto, and C. Lubinski. “Promoting Alge- Ferrucci, B., B. Yeap, and J. Carter. “A Modeling Ap-
braic Reasoning Using Student Thinking.” Math - proach for Enhancing Problem-Solving in the Mid-
ematics Teaching in the Middle School 6 (May 2001): dle Grades.” Mathematics Teaching in the Middle
508–514. School 8 (May 2003): 470–475.
Booth, L. “Children’s Difficulties in Beginning Fouche, K. “Algebra for Everyone: Start Early.” Math -
Algebra.” In The Ideas of Algebra, K-12, edited by ematics Teaching in the Middle School 2 (February 1997):
A. Coxford and A. Shulte. Reston, VA: NCTM, 1988. 226–229.
Bradley. E. “Is Algebra in the Cards?” Mathematics Joram, E., and V. Oleson. “How Fast Do Trees Grow?
Teaching in the Middle School 2 (May 1997): 398–403. Using Tables and Graphs to Explore Slope.”

© Lopez Mateos Editores. ISBN 978-607-95583-2-1, obra completa, versión electrónica, ISBN 978-607-95583-3-8, volumen 1, versión electrónica. Ejemplar asignado a: Helecto Villarroel gutierrez -
helecto@gmail.com. Fecha: 27 de octubre de 2014. Prohibida su modificación, copia o distribución.
Bibliografía seleccionada 247

Mathematics Teaching in the Middle School 13 Rubenstein, R. “Building Explicit and Recursive Forms
( January 2008): 260–265. of Patterns with the Function Game.” Mathematics
Joram, E., V. Oleson, and K. Sabey. “Is It a Good Deal? Teaching in the Middle School 7 (April 2002):
Developing Number Sense in Algebra by Compar- 426–431.
ing Housing Prices.” Journal of the Iowa Council of . “The Function Game.” Mathematics Teaching in
Teachers of Mathematics 32 (Winter 2005): 15–19. the Middle School 2 (November–December 1996):
Kalchman, M.S. “Walking Through Space: A New 74–78.
Approach for Teaching Functions.” Mathematics Sakshaug, L., and K. Wohlhuter. Responses to the
Teaching in the Middle School 11 (August 2005): Which Graph Is Which Problem.” Teaching Chil-
12–17. dren Mathematics 7 (February 2001): 352–353.
Koirala, H., and P. Goodwin. “Teaching Algebra in the Sand, M. “A Function Is a Mail Carrier.” Mathematics
Middle Grades Using Mathmagic.” Mathematics Teacher 89 (September 1996): 468 – 469.
Teaching in the Middle School 5 (May 2000): 562–566. Schneider, S., and C. Thompson. “Incredible Equa-
Krebs, A. “Studying Students’ Reasoning in Writing tions Develop Incredible Number Sense.” Teaching
Generalizations.” Mathematics Teaching in the Middle Children Mathematics 7 (November 2000): 146–148,
School 10 (February 2005): 284–287. 165–168.
Lamdin, D., R. Lynch, and H. McDaniel. “Algebra in Shealy, B. “Becoming Flexible with Functions: Investigat-
the Middle Grades.” Mathematics Teaching in the ing United States Population Growth.” Mathematics
Middle School 6 (November 2000): 195–198. Teacher 89 (May 1996): 414 – 418.
Lannin, J. “Developing Algebraic Reasoning Through Siegel, M. “The Sum of Cubes: An Activity Review and
Generalization.” Mathematics Teaching in the Middle Conjecture.” Mathematics Teaching in the Middle
School 8 (March 2003): 342–348. School 10 (March 2005): 356–359.
Lubinski, C., and A. Otto. “Meaningful Mathematical Smith, E. “Patterns, Functions, and Algebra.” In A Re-
Representation and Early Algebraic Reasoning.” search Companion to NCTM’s Standards, edited by J.
Teaching Children Mathematics 9 (October 2002): Kirkpatrick, W. G. Martin, and D. S. Schifter. Re-
76–80. ston, VA: NCTM, 2000.
MacGregor, M., and Quinlan, C. “Research in Teach- Smith, J., and E. Phillips. “Listening to Middle School
ing and Learning Algebra.” In Research in Mathe- Students’ Algebraic Thinking.” Mathematics Teaching
matics Education in Australasia: 1992–1995, edited in the Middle School 6 (November 2000): 156–161.
by W. Atweh, K. Owens, and P. Sullivan. 365–381. Steinberg, R., D. Sleeman, and D. Ktorza. “Algebra
(1996) Students Knowledge of Equivalence of Equations.”
Markovits, Z., B.-S. Eylon, and M. Bruckheimer. “Func- Journal for Research in Mathematics Education (1990),
tions—Linearity unconstrained.” In Proceedings of 22(2): 112–121.
the Seventh International Conference for the Psychology Suh, J. M. “Developing “Algebra-‘Rithmetic” in the El-
of Mathematics Education, edited by R. Hershkowitz. ementary Grades.” Teaching Children Mathematics 14
Rehovot, Israel: Weizmann Institute of Science, (November 2007): 246–252.
1983: 271–277. Thompson, F. M. “Algebraic Instruction for the
Markovits, Z., B.-S. Eylon, and M. Bruckheimer. Younger Child.” In The Ideas of Algebra K–12,
“Functions Today and Yesterday?” For the Learning NCTM Yearbook, 1988: 69–77.
of Mathematics (1986), 6(2): 18–24. Thornton, S. “New Approaches to Algebra: Have We
Markovits, Z., B.-S. Eylon, and M. Bruckheimer. “Dif- Missed the Point?” Mathematics Teaching in the Mid-
ficulties Students have with the Function Concept.” dle School 6 (March 2001): 388–392.
In The Ideas of Algebra, K-12, edited by A. Coxford Usiskin, Z. “Doing Algebra in Grades K–4.” Teaching
and A. Shulte, Reston, VA: NCTM, 1988. Children Mathematics 3 (February 1997): 346–356.
Martinez-Cruz, A., and E. Barger. “Adding a la Gauss.” Van de Walle, J. Elementary and Middle School Mathe-
Mathematics Teaching in the Middle School 10 (October matics: Teaching Developmentally. New York: Addison
2004): 152–155. Wesley Longman, 2007.
Ploger, D. “Spreadsheets, Patterns, and Algebraic Think- Van Dyke, F., and J. Tomback. “Collaborating to Intro-
ing.” Teaching Children Mathematics 3 (February duce Algebra,” Mathematics Teaching in the Middle
1997): 330–334. School 10 (December/January 2005): 236–242.
Pólya, G. How to Solve It. Princeton, NJ: Princeton Van Reeuwijk, M., and M. Wijers. “Students’ Con-
University Press, 1957. struction of Formulas in Context.” Mathematics
Mathematical Discovery, Combined Edition. New Teaching in the Middle School 2 (February 1997):
York: John Wiley & Sons, Inc., 1981. 230–236.
Quinlan, Cyril R. E. (1992) Developing an Understanding
of Algebraic Symbols. Ph.D. thesis, University of
Tasmania.

© Lopez Mateos Editores. ISBN 978-607-95583-2-1, obra completa, versión electrónica, ISBN 978-607-95583-3-8, volumen 1, versión electrónica. Ejemplar asignado a: Helecto Villarroel gutierrez -
helecto@gmail.com. Fecha: 27 de octubre de 2014. Prohibida su modificación, copia o distribución.
Enteros y
teoría de números
CAPÍTULO

Problema preliminar
Halla una manera rápida de obtener la suma sin usar una calculadora.
502 - 492 + 482 - 472 + Á + 22 - 12

248
© Lopez Mateos Editores. ISBN 978-607-95583-2-1, obra completa, versión electrónica, ISBN 978-607-95583-3-8, volumen 1, versión electrónica. Ejemplar asignado a: Helecto Villarroel gutierrez -
helecto@gmail.com. Fecha: 27 de octubre de 2014. Prohibida su modificación, copia o distribución.
Enteros y teoría de números 249

E ntre las expectativas mencionadas en los Principios y objetivos para los estudiantes de
grados 3–5 se incluyen:
• explorar números menores que 0 extendiendo la recta numérica y recurriendo a aplicaciones
conocidas;
• describir clases de números de acuerdo con determinadas características tales como la naturaleza
de sus factores. (p. 148)
Las expectativas para los estudiantes de los grados 6–8 son:
• usar factores, múltiplos, factorización en números primos y primos relativos para resolver problemas;
• desarrollar significado para los enteros y, con ellos, representar y comparar cantidades. (p. 214)

Además, los Principios y objetivos señalan que en los grados 6–8:

Los estudiantes también podrán trabajar con números completos al estudiar teoría de números.
Algunas actividades, como las siguientes, que incluyen factores, múltiplos, números primos y divi-
sibilidad, darán oportunidad de aprender a resolver problemas y de razonar.
1.  Explicar por qué la suma de los dígitos de cualquier múltiplo de 3 es divisible entre 3.
2.Un número de la forma abcabc siempre tiene varios factores primos. ¿Qué números primos
siempre son factores de un número de esta forma? ¿Por qué?
Los estudiantes de grados medios también deberán trabajar con enteros. En grados inferiores es po-
sible que los estudiantes ya hayan conectado los números negativos de manera apropiada, con el co-
nocimiento informal derivado de la vida diaria, como las invernales temperaturas bajo cero o la pérdida
de yardas en los juegos de futbol americano. En los grados medios, los estudiantes deben extender esta
comprensión inicial a los enteros. Se deberá comprender la utilidad de los enteros positivos y negativos
para percibir cambios o valores relativos. Los estudiantes también apreciarán la utilidad de los enteros
negativos cuando trabajen con ecuaciones cuya solución los requiera, como 2x + 7 = 1. (pp. 217–18)

En este capítulo, comenzaremos con el sistema de los enteros y estudiaremos teoría de


números.
Los números negativos son útiles en la vida diaria. Por ejemplo, el monte Everest está a
8,850 m sobre el nivel del mar y el mar Muerto está a 395 m bajo el nivel del mar. Podemos
simbolizar estas elevaciones como 8,850 y - 395.
En matemáticas surge la necesidad de emplear los enteros negativos debido a que no
siempre es posible efectuar substracciones o restas exclusivamente dentro del conjunto de
los números completos. Para calcular 4 - 6 usando la definición de resta para números
completos, debemos hallar un número completo n tal que 6 + n = 4. Como no existe di-
cho número completo n, para efectuar el cálculo debemos inventar un nuevo número, un
entero negativo. Si tratamos de calcular 4 - 6 en una recta numérica, debemos trazar inter-

◆ Nota El matemático hindú Bhramagupta (ca. 598–665) fue el primero en dar un tratamiento
histórica sistemático a los números negativos y el cero. Sólo después de casi 1000 años el matemá-
tico italiano Gerolamo Cardano (1501–1576) consideró soluciones negativas para ciertas
ecuaciones, pero sintiéndose incómodo con el concepto de números negativos, los llamó
números “ficticios”. ◆

© Lopez Mateos Editores. ISBN 978-607-95583-2-1, obra completa, versión electrónica, ISBN 978-607-95583-3-8, volumen 1, versión electrónica. Ejemplar asignado a: Helecto Villarroel gutierrez -
helecto@gmail.com. Fecha: 27 de octubre de 2014. Prohibida su modificación, copia o distribución.
250 Enteros y teoría de números

valos a la izquierda del 0. En la figura 5-1, se ilustra 4 - 6 como una flecha que comienza
en el 0 y termina 2 unidades a la izquierda del 0. El nuevo número, que corresponde a un
punto a 2 unidades a la izquierda del 0 es el dos negativo, que simbolizamos como - 2.

–6
4–6 4

–5 –4 –3 –2 –1 0 1 2 3 4 5 6

Figura 5-1

De manera análoga, creamos otros números a la izquierda del 0. El nuevo conjunto de nú-
meros 5 - 1, - 2, - 3, - 4, Á 6 es el conjunto de los enteros negativos. El conjunto
51, 2, 3, 4, Á 6 es el conjunto de los enteros positivos. El entero 0 no es positivo ni nega-
tivo. La unión del conjunto de los enteros negativos, el conjunto de los enteros positivos y
el 506 constituye el conjunto de los enteros. El conjunto de los enteros se denota con E.

E = 5 Á , - 4, - 3, - 2, - 1, 0, 1, 2, 3, 4, Á 6

Como campo de estudio, la teoría de los números comenzó a florecer en el siglo diecisiete
con el trabajo de Pierre de Fermat (1601–1665). Los temas de la teoría de los números que
se presentan en los programas de la escuela elemental incluyen factores, múltiplos, criterios
de divisibilidad, números primos, factorizaciones en primos, máximo común divisor y mí-
nimo común múltiplo. El tema de las congruencias, introducido por Carl Gauss
(1777–1855), también está incorporado en los programas elementales por medio de la arit-
mética de reloj y la aritmética modular. La aritmética modular y la del reloj permiten al
alumno echar un vistazo a un sistema matemático.

5-1 Los enteros y las operaciones de suma y resta

Representación de los enteros


Es una pena que usemos el símbolo “ - ” para indicar una resta y también como signo negativo.
Para reducir la confusión entre los usos del símbolo, en este libro usamos un signo elevado “ - ”
para los números negativos, como en - 2, y para el opuesto de un número, como en - x, a dife-
rencia del signo para la resta que se coloca más abajo. A veces, para enfatizar que un entero es
positivo se usa un signo “más” elevado, como en + 3. En este libro usamos el signo “más” sólo
para la suma y escribimos + 3 simplemente como 3.

◆ Nota No siempre se usó una raya para denotar la operación de resta y el signo negativo. Se desa-
histórica rrollaron otras notaciones pero nunca se adoptaron de manera universal. Una de dichas
notaciones fue la usada por Abu al-Khwârizmî (ca. 825), quien indicaba un número nega-
tivo colocando un pequeño círculo arriba del número. Por ejemplo, - 4 lo escribía como 4° .
Los hindúes denotaban un número negativo encerrándolo en un círculo; por ejemplo, escri-
bían - 4 como ➃ . Los símbolos + y - aparecieron impresos por primera vez en la matemá-
tica europea al final del siglo quince, época en que los símbolos no se referían a sumas o restas
ni a números positivos o negativos, sino a excedentes o déficits en problemas de negocios. ◆

© Lopez Mateos Editores. ISBN 978-607-95583-2-1, obra completa, versión electrónica, ISBN 978-607-95583-3-8, volumen 1, versión electrónica. Ejemplar asignado a: Helecto Villarroel gutierrez -
helecto@gmail.com. Fecha: 27 de octubre de 2014. Prohibida su modificación, copia o distribución.
Sección 5-1 Los enteros y las operaciones de suma y resta 251

Los enteros negativos son los opuestos de los enteros positivos. Por ejemplo, el opuesto
de 5 es - 5. Análogamente, los enteros positivos son los opuestos de los enteros negativos.
Como el opuesto de 4 se denota con - 4, el opuesto de - 4 puede denotarse con - 1 - 42, ó 4.
El opuesto de 0 es 0. En el conjunto E de enteros, el opuesto de todo elemento también está
en E.

O B S E R VA C I Ó N Pronto veremos que, usando la suma de enteros, cuando sumamos un


opuesto de un entero al entero la suma es 0. De hecho, - a se puede definir como la so-
lución de x + a = 0.

Ejemplo 5-1 Para cada caso, halla el opuesto de x:










a. x = 3
b. x = - 5
c. x = 0

Solución a. - x = - 3
b. - x = - ( - 5) = 5
c. - x = - 0 = 0

El valor de - x en el ejemplo 5-1(b) es 5. Nota que - x es el opuesto de x y no necesariamente re-


presenta un número negativo. Es una variable que puede reemplazarse por cualquier número, ya
sea positivo, cero o negativo. Nota: - x se lee “el opuesto de x”; no “menos x” ni “negativo de x”.
En los Puntos focales para el grado 7 hallamos lo siguiente:

Al aplicar las propiedades de la aritmética y considerando los números negativos en el contexto de


la vida diaria (p. ej. en situaciones de deber dinero o medir alturas sobre o bajo el nivel del mar), los
alumnos se explican por qué tienen sentido las reglas para sumar, restar, multiplicar y dividir números
negativos. (p. 19)

A continuación investigamos varias maneras informales de introducir las operaciones con


◆ Nota de enteros; comenzamos con la suma.
investigación
Es importante Suma de enteros
manipular objetos al
Como se menciona en la Nota de investigación, es importante usar materiales para manipu-
trabajar con números
lar cuando se trabaja con enteros. A continuación se presentan varios modelos para moti-
negativos.
var la suma de enteros. Los maestros pueden trazar en el piso una recta numérica para
(Thompson
que los estudiantes caminen sobre ella cuando usen ese modelo.
1988). ◆

Modelo de las fichas para la suma


En el modelo de las fichas, los enteros positivos se representan con fichas negras y los ente-
ros negativos con fichas rojas. Una ficha roja neutraliza una ficha negra. Por lo tanto, el en-
tero - 1 se puede representar con 1 ficha roja o con 2 rojas y 1 negra, o bien con 3 rojas y 2
negras, y así sucesivamente. De manera análoga, cada entero se puede representar de mu-
chas maneras usando fichas, como se muestra en la figura 5-2.

© Lopez Mateos Editores. ISBN 978-607-95583-2-1, obra completa, versión electrónica, ISBN 978-607-95583-3-8, volumen 1, versión electrónica. Ejemplar asignado a: Helecto Villarroel gutierrez -
helecto@gmail.com. Fecha: 27 de octubre de 2014. Prohibida su modificación, copia o distribución.
252 Enteros y teoría de números

+2 +2 +2 +2

–2 –2 –2 –2

Figura 5-2

– 4 + 3 = –1
En la figura 5-3 se muestra el modelo para la suma - 4 + 3. Colocamos cuatro fichas rojas
Figura 5-3 junto con 3 fichas negras. Como 3 rojas neutralizan 3 negras, la figura 5-3 representa el equi-
valente de 1 ficha roja, ó - 1.

Modelo del campo de cargas para la suma


+ + +
– – – – – Un modelo similar al de las fichas usa cargas positivas y negativas. Un campo tiene carga 0 si
posee el mismo número de cargas positivas 1+ 2 y negativas 1- 2. Como en el modelo de las
fichas, un entero dado se puede representar de muchas maneras usando el modelo del campo
de cargas. La figura 5-4 usa el modelo para 3 + - 5. Como 3 cargas positivas “neutralizan” 3
3 + –5 = –2 cargas negativas, el resultado neto es de 2 negativas. Por tanto, 3 + - 5 = - 2.
Figura 5-4

Modelo de la recta numérica


Otro modelo para la suma incluye una recta numérica. Se puede introducir con la idea de
un excursionista caminando por la recta numérica, como se ve en la Página de un libro de
texto en la página 253. Estudia esa página para ver cómo funciona el modelo del excursio-
nista y cómo se pueden registrar sobre la recta numérica los movimientos que realiza. Sin el
excursionista, - 3 + - 5 se puede representar como en la figura 5-5.

–3 + –5
–5 –3

–8 –7 –6 –5 –4 –3 –2 –1 0 1 2 3 4 5 6 7 8

Figura 5-5

© Lopez Mateos Editores. ISBN 978-607-95583-2-1, obra completa, versión electrónica, ISBN 978-607-95583-3-8, volumen 1, versión electrónica. Ejemplar asignado a: Helecto Villarroel gutierrez -
helecto@gmail.com. Fecha: 27 de octubre de 2014. Prohibida su modificación, copia o distribución.
Sección 5-1 Los enteros y las operaciones de suma y resta 253

Página de un libro de texto S U M A D E E N T E ROS

Lección
Calentamiento
´ Suma de enteros
Idea clave
Puedes usar la Aprende
recta numérica
para sumar enteros.
¿Cómo puedes sumar enteros usando una recta numérica?
Piensa que vas caminando a lo largo de un recta numérica. Camina hacia
Vocabulario
valor absoluto
adelante para los enteros positivos y camina hacia atrás para los enteros negativos.

Ejemplo A
ex
ef l i ó En el primer down, después de obtener la pelota, un equipo
R

avanzó 5 yardas. En el siguiente down perdió 7 yardas. ¿El


equipo ganó o perdió yardas después de los dos downs?
¡Piensa!
Cuando sumo Halla 5 + (−7).
enteros en la recta
numérica necesito
comenzar en cero.

Parte de cero viendo hacia los Después camina hacia atrás 7 pasos
números positivos. Camina hacia para representar −7. Te detienes en −2.
adelante 5 pasos para representar 5.
Así, 5 + (−7)= −2.
El equipo perdió 2 yardas.

Ejemplo B
Halla −4 + (−2).

Parte de cero viendo hacia los Después camina hacia atrás 2 pasos
números positivos. Camina hacia para representar −2. Te detienes en −6.
atrás 4 pasos para representar −4.
Así, −4 + (−2)= −6.

Ejemplo C
Halla −6 + 11.

Parte de cero viendo hacia los Después camina hacia adelante 11 pasos
números positivos. Camina hacia para representar 11. Te detienes en 5.
atrás 6 pasos para representar −6.
Así, −6 + 11= 5.

Fuente: Scott Foresman-Addison Wesley, Grade 6, 2008 (p. 418).

© Lopez Mateos Editores. ISBN 978-607-95583-2-1, obra completa, versión electrónica, ISBN 978-607-95583-3-8, volumen 1, versión electrónica. Ejemplar asignado a: Helecto Villarroel gutierrez -
helecto@gmail.com. Fecha: 27 de octubre de 2014. Prohibida su modificación, copia o distribución.
254 Enteros y teoría de números

Análogamente, la figura 5-6 ilustra la suma de 3 + - 5.

–5
3 + –5 3

–8 –7 –6 –5 –4 –3 –2 –1 0 1 2 3 4 5 6 7 8

Figura 5-6

AHORA INTENTA ÉSTE 5-1


a. En referencia al ejemplo B de la página de muestra, ¿la suma de dos enteros negativos siempre es negativa?
b. En referencia a los ejemplos A y C de la página de muestra, ¿la suma de un entero positivo y uno nega-
tivo, es positiva o negativa? Explica.
c. Usa una recta numérica para sumar 6 + ( - 8) + ( - 2).

En el ejemplo 5-2 presentamos un termómetro con una escala en la forma de una recta
numérica vertical.

Ejemplo 5-2 La temperatura era de - 4°C. En una hora subió 10°C. ¿Cuál es la nueva temperatura?








Solución En la figura 5-7 se muestra que la nueva temperatura es de 6°C y que - 4 + 10 = 6.






Modelo del patrón


En el capítulo 3 vimos la suma de números completos. La suma de enteros también puede
+10˚ 1˚ motivarse usando patrones de suma de números completos. Nota que en la columna de la
0˚ izquierda los primeros cuatro resultados se conocen a partir de la suma de números com-
– 1˚
– 2˚ pletos. Nota también que el 4 permanece fijo y que conforme los números sumados a 4 de-
– 3˚
– 4˚
crecen en 1, la suma se reduce en 1. Siguiendo este patrón, 4 + - 1 = 3 y podemos, así,
– 5˚ completar el resto de la primera columna. Razonando de manera similar podemos comple-
tar los cálculos en la columna derecha, donde - 2 permanece fijo y los otros números decre-
cen en 1 cada vez.
Figura 5-7 4 + 3 = 7 -2 + 4 = 2
4 + 2 = 6 -2 + 3 = 1
4 + 1 = 5 -2 + 2 = 0
4 + 0 = 4 -2 + 1 = -1
4 + -1 = 3 -2 + 0 = -2
-
4 + 2 = 2 - 2 + -1 = -3
4 + -3 = 1 -2 + -2 = -4
4 + -4 = 0 -2 + -3 = -5
4 + -5 = -1 -2 + -4 = -6
-
4 + 6 = 2 - -2 + -5 = -7

Nota que el razonamiento con patrones es un razonamiento inductivo y, por lo tanto, no


constituye una demostración.

© Lopez Mateos Editores. ISBN 978-607-95583-2-1, obra completa, versión electrónica, ISBN 978-607-95583-3-8, volumen 1, versión electrónica. Ejemplar asignado a: Helecto Villarroel gutierrez -
helecto@gmail.com. Fecha: 27 de octubre de 2014. Prohibida su modificación, copia o distribución.
Sección 5-1 Los enteros y las operaciones de suma y resta 255

RINCÓN DE LA TECNOLOGÍA En una hoja de cálculo, en la columna A coloca el 4 y llena 20 renglo-


nes. (Para ayuda con las hojas de cálculo, ver el Technology Manual.) En la columna B coloca 3 como pri-
mer registro y después escribe una fórmula para sumar - 1 a 3 para obtener el segundo registro, suma - 1
al segundo registro para obtener el tercer registro, y llena la columna continuando con el patrón. En la
columna C obtén la suma de los registros respectivos en las columnas A y B. ¿Qué patrones observas? Re-
pite el problema cambiando los registros de la columna A a - 4 y repitiendo el proceso.

Valor absoluto
Como 4 y - 4 son opuestos, están en lados opuestos del 0 en la recta numérica y se encuen-
tran a la misma distancia (4 unidades) del 0, como se muestra en la figura 5-8.

4 unidades 4 unidades

–5 –4 –3 –2 –1 0 1 2 3 4 5

Figura 5-8

La distancia siempre es positiva o cero. La distancia entre el punto correspondiente a un


entero y el 0 es el valor absoluto del entero. Así, el valor absoluto de 4 y de - 4 es 4, que se
escribe ƒ 4 ƒ = 4 y ƒ - 4 ƒ = 4, respectivamente. Nota que si x Ú 0, entonces ƒ x ƒ = x, y que si
x 6 0, entonces - x es positivo. Por lo tanto, tenemos lo siguiente:

Definición de valor absoluto


ƒ x ƒ = x si x Ú 0
ƒ x ƒ = - x si x 6 0

O B S E R VA C I Ó N Algunos estudiantes quieren abreviar la definición anterior


escribiendo ƒ x ƒ = ; x. Esto no es cierto pues ƒ x ƒ tiene un solo valor.

Ejemplo 5-3 Evalúa lo siguiente:










a. ƒ 20 ƒ
b. ƒ -5 ƒ
c. ƒ0ƒ
d. - ƒ -3 ƒ
e. ƒ 2 + -5 ƒ

Solución a. ƒ 20 ƒ = 20
b. ƒ -5 ƒ = 5
c. ƒ0ƒ = 0
d. - ƒ -3 ƒ = -3
e. ƒ 2 + -5 ƒ = ƒ -3 ƒ = 3

© Lopez Mateos Editores. ISBN 978-607-95583-2-1, obra completa, versión electrónica, ISBN 978-607-95583-3-8, volumen 1, versión electrónica. Ejemplar asignado a: Helecto Villarroel gutierrez -
helecto@gmail.com. Fecha: 27 de octubre de 2014. Prohibida su modificación, copia o distribución.
256 Enteros y teoría de números

AHORA INTENTA ÉSTE 5-2 Escribe cada caso de la forma más sencilla sin la notación de valor abso-
luto en la respuesta final. Muestra tu trabajo.
a. ƒ x ƒ + x si x … 0
b. - ƒ x ƒ + x si x … 0
c. - ƒ x ƒ + x si x Ú 0

O B S E R VA C I Ó N   Es posible describir la suma de enteros como el proceso de hallar


la diferencia o la suma de los valores absolutos de los enteros y asignarles el signo
apropiado.

Propiedades de la suma de enteros


La suma de enteros tiene todas las propiedades de la suma de los números completos. Estas
propiedades se pueden demostrar si se define la suma de enteros en términos de suma y
resta de números completos.

Teorema 5–1:    Propiedades


Dados enteros a, b y c:
Propiedad de la cerradura de la suma de enteros a + b es un entero único.
Propiedad conmutativa de la suma de enteros a + b = b + a.
Propiedad asociativa de la suma de enteros 1a + b2 + c = a + 1b + c2.
Elemento identidad de la suma de enteros 0 es el único entero tal que para todos
los enteros a, 0 + a = a = a + 0.

Pon atención al nombre elemento identidad en el teorema 5–1. El cero es el elemento identidad
de la suma pues cuando se suma a cualquier entero, no cambia el resultado; deja el entero sin
modificación.
Hemos visto que todo entero tiene su opuesto. Este opuesto es el inverso aditivo del en-
tero. El hecho de que cada entero tenga un único (uno y sólo uno) inverso aditivo se plan-
tea como sigue.

Teorema 5–2:    Unicidad del inverso aditivo


Para todo entero a existe un entero único - a, el inverso aditivo de a, tal que
a + - a = 0 = - a + a.

O B S E R VA C I Ó N Nota que por definición el inverso aditivo, - a, es la solución de la


ecuación x + a = 0. El hecho de que el inverso aditivo sea único equivale a decir
que la ecuación anterior tiene una sola solución. De hecho, para cualesquier enteros
a y b, la ecuación x + a = b tiene solución única, b + - a.

La unicidad del inverso aditivo se puede usar para justificar otros teoremas. Por ejemplo
el opuesto, o inverso aditivo, de - a se puede escribir como - 1 - a2. Sin embargo, como
© Lopez Mateos Editores. ISBN 978-607-95583-2-1, obra completa, versión electrónica, ISBN 978-607-95583-3-8, volumen 1, versión electrónica. Ejemplar asignado a: Helecto Villarroel gutierrez -
helecto@gmail.com. Fecha: 27 de octubre de 2014. Prohibida su modificación, copia o distribución.
Sección 5-1 Los enteros y las operaciones de suma y resta 257

a + - a = 0, el inverso aditivo de - a es también a. Como el inverso aditivo de - a debe ser


único, tenemos - 1 - a2 = a. Se pueden investigar otros teoremas de la suma de enteros al
considerar las nociones recién desarrolladas. Por ejemplo, vemos que - 2 + - 4 = - 6 y sa-
bemos que - 6 es el inverso aditivo de 6, ó 2 + 4. Esto nos conduce a lo siguiente:
- 2 + - 4 = - 12 + 42

Esta relación es verdadera en general y se enuncia junto con su demostración.

Teorema 5–3
Para cualesquier enteros a y b:
1. - 1 - a2 = a
2. - a + - b = - 1a + b2

Demostramos la segunda parte del teorema 5–3 de la manera siguiente: por definición
- 1a + b2 es el inverso aditivo de 1a + b2, esto es, 1a + b2 + - 1a + b2 = 0. Si pudiéramos
mostrar que también - a + - b es inverso aditivo de a + b, la unicidad del inverso aditivo
implicará que - 1a + b2 y - a + - b son iguales. Para mostrar que - a + - b también es in-
verso aditivo de a + b, sólo necesitamos mostrar que 1a + b2 + 1 - a + - b2 = 0. Para ello
se pueden usar las propiedades asociativa y conmutativa de la suma de enteros y la definición
del inverso aditivo, de la siguiente manera:
1a + b2 + 1 - a + - b2 = 1a + - a2 + 1b + - b2
= 0 + 0
= 0
Ahora tenemos que
1a + b2 + 1 - a + - b2 = 0
1a + b2 + - 1a + b2 = 0
Por lo tanto, - 1a + b2 = - a + - b.

O B S E R VA C I Ó N Nota que en la demostración se omitieron algunos pasos correspon-


dientes a las propiedades conmutativa y asociativa de la suma de enteros. A continuación
presentamos una demostración más detallada que muestra todos los pasos:
1a + b2 + 1 - a + - b2 = 1a + b2 + 1 - b + - a2
= 31a + b2 + - b4 + - a
= 3a + 1b + - b24 + - a
= 1a + 02 + - a
= a + -a
= 0

Ejemplo 5-4 Halla el inverso aditivo de cada caso:


a. - 13 + x2








b. a + - 4
c. - 3 + - x
© Lopez Mateos Editores. ISBN 978-607-95583-2-1, obra completa, versión electrónica, ISBN 978-607-95583-3-8, volumen 1, versión electrónica. Ejemplar asignado a: Helecto Villarroel gutierrez -
helecto@gmail.com. Fecha: 27 de octubre de 2014. Prohibida su modificación, copia o distribución.
258 Enteros y teoría de números

Solución a. 3 + x
b. - 1a + - 42, que se puede escribir como - a + - 1 - 42, ó - a + 4
c. - 1 - 3 + - x2, que puede escribirse como - 1 - 32 + - 1 - x2, ó 3 + x

Resta o substracción de enteros


Como hicimos con la suma de enteros, exploramos diversos modelos para la resta o substrac-
ción de enteros.

Modelo de las fichas para la resta


Para hallar 3 - - 2, queremos restar - 2 (o eliminar 2 fichas rojas) de 3 fichas negras.
Como se ve en la figura 5-9(a), si sólo tenemos 3 fichas negras, no podemos quitar 2 ro-
jas. Por lo tanto, necesitamos representar 3 de manera que tengamos al menos 2 fichas
rojas. Recordemos que 1 ficha roja neutraliza 1 ficha negra y así, añadir una ficha negra y
una ficha roja (o 2 fichas negras y 2 fichas rojas) es lo mismo que sumar 0 y el problema
es el mismo. Como necesitamos 2 fichas rojas, podemos añadir 2 fichas negras y 2 fichas
rojas sin que se modifique el problema. En la figura 5-9(b), vemos a 3 representado por
medio de 5 fichas negras y 2 fichas rojas. Ahora, al “retirar” 2 fichas rojas en la figura 5-
9(c), quedan 5 fichas negras y por lo tanto, 3 - - 2 = 5.

(a) 3 (b) 3 (c) 3 – –2 = 5

Figura 5-9

Modelo de campo de cargas para la resta


La resta de enteros se puede modelar con un campo de cargas. Por ejemplo, considera
- 3 - - 5. Para restar - 5 de - 3, primero representamos - 3 de modo que estén presentes
al menos 5 cargas negativas. Se muestra un ejemplo en la figura 5-10(a). Para restar - 5,
quita 5 cargas negativas, lo cual deja 2 cargas positivas, como en la figura 5-10(b). Por lo
tanto, - 3 - - 5 = 2.

+ + + +
– –
– – –

(a) –3 (b) –3 – –5 = 2

Figura 5-10

© Lopez Mateos Editores. ISBN 978-607-95583-2-1, obra completa, versión electrónica, ISBN 978-607-95583-3-8, volumen 1, versión electrónica. Ejemplar asignado a: Helecto Villarroel gutierrez -
helecto@gmail.com. Fecha: 27 de octubre de 2014. Prohibida su modificación, copia o distribución.
Sección 5-1 Los enteros y las operaciones de suma y resta 259

Nota cómo se combinan el modelo de las fichas y el de campo de cargas en la parte de la


página que mostramos a continuación.

Página de un libro de texto MODELO FICHA/CAMPO DE CARGA

¡A practicar!

Actividad
Halla 5 − 2. Muestra Retira Quedan 3 fichas “+”. De modo
fichas “+” fichas “+” que

Halla −5 − (−2). Muestra Retira Quedan 3 fichas “−”. De modo


fichas “−” fichas “−” que

A veces es necesario insertar cero en pares para poder restar.


Halla 5 − (−2). Muestra Inserta dos 0 en pares. Quedan 7 fichas “+”. De modo
fichas “+” Después retira 2 fichas “−”. que

Halla −5 − 2.
Muestra Inserta dos 0 en pares. Quedan 7 fichas “−”. De modo
fichas “−” Después retira 2 fichas “+”. que

Fuente: Prentice Hall Mathematics, Grade 7, 2008, Course 2 (p. 37).

Modelo de la recta numérica para la resta


El modelo de la recta numérica usado para la suma de enteros también puede usarse para
modelar la resta de enteros. Mientras que la suma se modela manteniendo la misma direc-
ción y el movimiento hacia adelante o hacia atrás depende de si se suma un entero positivo
o negativo, la resta se modela dando la vuelta. Para ver cómo funciona esto examina la pá-
gina de muestra que sigue. Estudia el modelo para asegurarte de que estás cómodo con él y
úsalo para hallar 5 - - 3 - - 2.

© Lopez Mateos Editores. ISBN 978-607-95583-2-1, obra completa, versión electrónica, ISBN 978-607-95583-3-8, volumen 1, versión electrónica. Ejemplar asignado a: Helecto Villarroel gutierrez -
helecto@gmail.com. Fecha: 27 de octubre de 2014. Prohibida su modificación, copia o distribución.
260 Enteros y teoría de números

Página de un libro de texto R ESTA D E E N T E ROS

Lección
Calentamiento
´ Resta de enteros
Idea clave
Puedes usar Aprende
una recta
numérica para ¿Cómo restas enteros?
ilustrar la resta
de enteros.
Ejemplo

Vocabulario Halla
enteros positivos Para restar enteros puedes pensar que estás caminando a lo largo de una
recta numérica. Comienza en 0 viendo hacia los enteros positivos.
enteros negativos Camina hacia adelante El signo de resta (−) Después camina hacia
4 pasos para significa date vuelta. atrás 3 pasos para 3.

Materiales
Rectas numéricas

Terminas en + 7. De modo que +4 − − 3= +7


ex
ef l i ó
R

El problema anterior ilustra cómo restar un entero negativo de un


¡Piensa! entero positivo. A continuación se muestran otros casos posibles.
Puedo trazar una
recta numérica Ejemplo
para ilustrar la
idea principal. Halla
Comienza en 0, viendo hacia los enteros positivos.
Camina 2 pasos hacia El signo de resta (−) Después camina hacia
+
adelante para +2 significa date vuelta. adelante 4 pasos para 4.

Terminas en −2. De modo que +2 − +4 = −2

Ejemplo
Halla
Comienza en 0, viendo hacia los enteros positivos.
Camina 6 pasos hacia El signo de resta (−) Después camina hacia
atrás para −6 significa date vuelta. atrás 3 pasos para −3.

Terminas en −3. De modo que −6 − −3 = − 3

Fuente: Scott Foresman-Addison Wesley, Grade 5, 2008 (p. 718).

© Lopez Mateos Editores. ISBN 978-607-95583-2-1, obra completa, versión electrónica, ISBN 978-607-95583-3-8, volumen 1, versión electrónica. Ejemplar asignado a: Helecto Villarroel gutierrez -
helecto@gmail.com. Fecha: 27 de octubre de 2014. Prohibida su modificación, copia o distribución.
Sección 5-1 Los enteros y las operaciones de suma y resta 261

AHORA INTENTA ÉSTE 5-3 Supón que un servicio de mensajería te entrega tres cartas, una con un
cheque de $25 y otras dos con cuentas de $15 y $20, respectivamente. Registras esto como
25 + - 15 + - 20, ó - 10; esto es, eres $10 más pobre. Supón que al día siguiente te encuentras con que
la cuenta de $20 era, en realidad, para otra persona y la devuelves al mensajero. Registras tu nuevo saldo
como
- 10 - - 20
o como
25 + - 15 + - 20 - - 20
que es igual a 25 + - 15, ó 10.
Para cada uno de los siguientes casos, construye un relato acerca de envíos y explica cómo puede
ayudar a encontrar la respuesta.
a. 23 + - 13 + - 12
b. 18 - - 37

Modelo del patrón para la resta


Usando razonamiento inductivo podemos hallar la diferencia de dos enteros al considerar
los patrones siguientes, donde comenzamos con restas que ya sabemos cómo hacer. Los dos
patrones mostrados, el de la izquierda y el de la derecha, comienzan con 3 - 2 = 1.
3 - 2 = 1 3 - 2 = 1
3 - 3 = 0 3 - 1 = 2
3 - 4 = ? 3 - 0 = 3
3 - 5 = ? 3 - -1 = ?
En el patrón de la izquierda la diferencia decrece en 1. Si continuamos el patrón tenemos
que 3 - 4 = - 1 y 3 - 5 = - 2. En el patrón de la derecha la diferencia se incrementa en
1. Si continuamos el patrón, tenemos que 3 - - 1 = 4 y 3 - - 2 = 5.

Resta usando el enfoque del sumando faltante


La resta de enteros, como la resta de números completos, se puede definir en términos de
la suma. Usando el enfoque del sumando faltante, 5 - 3 se puede calcular hallando un nú-
mero completo n como sigue:
5 - 3 = n si, y sólo si, 5 = 3 + n
Como 3 + 2 = 5, entonces n = 2.
De manera análoga, calculamos 3 - 5 como sigue:
3 - 5 = n si, y sólo si, 3 = 5 + n
Como 5 + - 2 = 3, entonces n = - 2. En general, para enteros a y b, tenemos la
siguiente definición de resta o substracción.

Definición de resta o substracción


Para enteros a y b, a - b es el único entero n tal que a = b + n.

© Lopez Mateos Editores. ISBN 978-607-95583-2-1, obra completa, versión electrónica, ISBN 978-607-95583-3-8, volumen 1, versión electrónica. Ejemplar asignado a: Helecto Villarroel gutierrez -
helecto@gmail.com. Fecha: 27 de octubre de 2014. Prohibida su modificación, copia o distribución.
262 Enteros y teoría de números

O B S E R VA C I Ó N La suma “deshace” la resta; esto es, 1a - b2 + b = a. También la


resta “deshace” la suma; esto es, 1a + b2 - b = a.

Ejemplo 5-5 Usa la definición de resta para calcular lo siguiente:


a. 3 - 10








b. - 2 - 10
Solución a. Sea 3 - 10 = n. Entonces 10 + n = 3, de modo que n = - 7. Por lo tanto,
3 - 10 = - 7.
b. Sea - 2 - 10 = n. Entonces 10 + n = - 2, de modo que n = - 12. Por lo
tanto, - 2 - 10 = - 12.

Resta usando el enfoque de sumar el opuesto


A continuación, estudia la parte de la página de muestra del libro Scott Foresman-Addison
Wesley Mathematics, Grade 6, 2008 (p. 423). Considera las restas y sumas en las partes A–D
y después la regla que descubrieron los estudiantes. Esta técnica es muy útil para efectuar
restas de enteros.

Página de un libro de texto R ESTA D E E N T E ROS

¿Cuál es una regla para restar enteros?


Un grupo hizo la tabla de la derecha para Resta Suma
mostrar algunas sumas y restas que hallaron
usando rectas numéricas.
Veo una regla
¡En cada renglón el para restar. Puedo
sumar el opuesto
resultado es el mismo!
del segundo número
al primero.

Regla para restar enteros


Restar un entero es lo mismo
que sumar su opuesto.

Tema de plática
¿Cómo puedes cambiar
8−(−3) a una suma?
Sección B Lección 8-6

Fuente: Scott Foresman-Addison Wesley, Mathematics 2008, Grade 6 (p. 423).

© Lopez Mateos Editores. ISBN 978-607-95583-2-1, obra completa, versión electrónica, ISBN 978-607-95583-3-8, volumen 1, versión electrónica. Ejemplar asignado a: Helecto Villarroel gutierrez -
helecto@gmail.com. Fecha: 27 de octubre de 2014. Prohibida su modificación, copia o distribución.
Sección 5-1 Los enteros y las operaciones de suma y resta 263

De nuestro trabajo anterior con la suma de enteros, sabemos que 3 - 5 = - 2 y


3 + - 5 = - 2. Por lo tanto, 3 - 5 = 3 + - 5. En general, lo siguiente es verdadero.

Teorema 5–4
Para todos los enteros a y b, a - b = a + - b.

El teorema anterior se puede justificar usando el hecho de que la ecuación b + x = a tiene


solución única para x. De la definición de resta, la solución de la ecuación es a - b. Para
mostrar que a - b = a + - b, sólo necesitamos mostrar que a + - b también es una solu-
ción. Para ello, substituimos a + - b en lugar de x y verificamos si b + 1a + - b2 = a:
b + 1a + - b2 = b + 1 - b + a2
= 1b + - b2 + a
= 0 + a
= a
En consecuencia, a - b = a + - b.

O B S E R VA C I Ó N A veces la propiedad anterior se usa como definición de resta.

AHORA INTENTA ÉSTE 5-4


a. ¿El conjunto de los enteros es cerrado bajo la resta? ¿Por qué?
b. Serán válidas para la resta de enteros las propiedades conmutativa, asociativa o de identidad? ¿Por qué
sí o por qué no?

Muchas calculadoras tienen una tecla de cambio de signo, ya sea CHS o +/- . Otras cal-
culadoras usan (-) , una tecla que permite realizar cálculos con enteros. Por ejemplo, para
calcular 8 - 1 - 32, debes teclear 8 - 3 +/- = . Investiga qué sucede si tecleas
8 - - 3 =.

Ejemplo 5-6 Usa el hecho de que a - b = a + - b para calcular lo siguiente:










a. 2 - 8 b. 2 - - 8 c. - 12 - - 5 d. - 12 - 5

Solución a. 2 - 8 = 2 + -8 = -6
b. 2 - - 8 = 2 + - 1 - 82 = 2 + 8 = 10
c. - 12 - - 5 = - 12 + - 1 - 52 = - 12 + 5 = - 7
d. - 12 - 5 = - 12 + - 5 = - 17

Ejemplo 5-7 Usa el hecho de que a - b = a + 1 - b2 y los teoremas sobre el inverso aditivo para escribir








expresiones iguales a cada una de las siguientes sin paréntesis en la respuesta final.
a. - 1b - c2 b. a - 1b + c2

Solución a. - 1b - c2 = - 1b + - c2 = - b + - 1 - c2 = - b + c
b. a - 1b + c2 = a + - 1b + c2 = a + 1 - b + - c2 = 1a + - b2 + - c =
a + -b + -c

© Lopez Mateos Editores. ISBN 978-607-95583-2-1, obra completa, versión electrónica, ISBN 978-607-95583-3-8, volumen 1, versión electrónica. Ejemplar asignado a: Helecto Villarroel gutierrez -
helecto@gmail.com. Fecha: 27 de octubre de 2014. Prohibida su modificación, copia o distribución.
264 Enteros y teoría de números

O B S E R VA C I Ó N Es posible simplificar todavía más las respuestas en el ejemplo 5-7 (a)


y (b), como sigue: - b + c = c + - b = c - b; y a + - b + - c = 1a - b) - c.

Ejemplo 5-8 Simplifica lo siguiente:


a. 2 - 15 - x2 b. 5 - 1x - 32 c. - 1x - y2 - y








Solución a. 2 - 15 - x2 = 2 + - 15 + - x2 b. 5 - 1x - 32 = 5 + - 1x + - 32
= 2 + - 5 + - 1 - x2 = 5 + - x + - 1 - 32
= 2 + -5 + x = 5 + -x + 3
= -3 + x ó x - 3 = 8 + -x
c. - 1x - y2 - y = - 1x + - y2 + - y = 8 - x
= 3 - x + - 1 - y24 + - y
= 1 - x + y2 + - y
= - x + 1 y + - y2
= -x + 0
= -x

Orden de las operaciones


La resta en el conjunto de los enteros no es conmutativa ni asociativa, como se ilustra en
estos contraejemplos:
5 - 3 Z 3 - 5 porque 2 Z - 2
13 - 152 - 8 Z 3 - 115 - 82 porque - 20 Z - 4
Una expresión como 3 - 15 - 8 es ambigua a menos que sepamos en qué orden realizar
las restas. Los matemáticos están de acuerdo en que 3 - 15 - 8 significa 13 - 152 - 8;
esto es, las restas en 3 - 15 - 8 se efectúan de izquierda a derecha. De manera análoga,
3 - 4 + 5 significa 13 - 42 + 5 y no 3 - 14 + 52. Así, 1a - b2 - c se puede escribir sin
paréntesis como a - b - c. Revisaremos el orden de las operaciones después de estudiar la
multiplicación y la división.

Ejemplo 5-9 Calcula lo siguiente:


c. 3 - 17 - 32








a. 2 - 5 - 5 b. 3 - 7 + 3

Solución a. 2 - 5 - 5 = - 3 - 5 = - 8
b. 3 - 7 + 3 = - 4 + 3 = - 1
c. 3 - 17 - 32 = 3 - 4 = - 1

RINCÓN DE LA TECNOLOGÍA
a. En una calculadora graficadora, grafica la función con ecuación y = x - - 4.
b. Usando la gráfica en (a), describe lo que sucede cuando x toma valores menores que - 4, iguales a - 4
y mayores que - 4.

© Lopez Mateos Editores. ISBN 978-607-95583-2-1, obra completa, versión electrónica, ISBN 978-607-95583-3-8, volumen 1, versión electrónica. Ejemplar asignado a: Helecto Villarroel gutierrez -
helecto@gmail.com. Fecha: 27 de octubre de 2014. Prohibida su modificación, copia o distribución.
Sección 5-1 Los enteros y las operaciones de suma y resta 265

Evaluación 5-1A

1. Halla el inverso aditivo de cada uno de los enteros siguien- a. La temperatura es de 55°F y se supone que baja 60°F
tes. Escribe tu respuesta de la forma más sencilla posible. por la noche. ¿Cuál es la temperatura esperada para
a. 2 b. - 5 la medianoche?
c. m d. 0 b. A Miguel, su banco le permite sobregirarse. Si él
e. - m f. a + b tiene $200 en su cuenta de cheques y expide un
2. Simplifica lo siguiente: cheque por $220, ¿cuál es el saldo?
a. - 1 - 2) b. - 1 - m) c. - 0 14. Los aceites para motor brindan protección en un rango de
3. Evalúa lo siguiente: temperaturas. Estos aceites tienen nombres como 10W–40
a. ƒ - 5 ƒ b. ƒ 10 ƒ ó 5W–30. La siguiente gráfica muestra las temperaturas, en
c. - ƒ - 5 ƒ d. - ƒ 5 ƒ grados Fahrenheit, a las cuales la máquina está protegida
4. Ilustra cada una de las sumas siguientes por medio del por un aceite particular. Usa la gráfica y halla qué aceites se
modelo de las fichas o del campo de carga: pueden usar para las temperaturas siguientes:
a. 5 + - 3 b. - 2 + 3 a. Entre - 5° y 90°
-
c. 3 + 2 d. - 3 + - 2 b. Debajo de - 20°
5. Ilustra cada una de las sumas del problema 4 usando un c. Entre - 10° y 50°
modelo de recta numérica. d. De - 20° a más de 100°
6. Calcula cada una de las siguientes usando a - b = a + - b: e. De - 8° a 90°
a. 3 - - 2
b. - 3 - 2 5W–30
c. - 3 - - 2 20W–30
7. Responde cada parte del problema 6 usando la defini-
ción de resta con el enfoque del sumando faltante. 10W–40

8. Escribe una suma básica que corresponda a cada una de las 10W–30
expresiones siguientes y después responde la pregunta: – 30° – 20° – 10° 0° 10° 20° 30° 40° 50° 60° 70° 80° 90° 100°
a. Una acción cae 17 puntos y al día siguiente gana 10 pun-
tos. ¿Cuál es el cambio neto en el valor de la acción? 15. Simplifica cada caso lo más posible. Muestra todos los
b. La temperatura era de - 10°C y después subió en 8°C. pasos.
¿Cuál es la nueva temperatura? a. 3 - 12 - 4x2
c. El avión estaba a 5000 pies y bajó 100 pies. ¿Cuál es b. x - 1 - x - y2
la nueva altitud del avión? 16. ¿Para qué enteros a, b y c sucede que a - b - c =
9. El 1º de enero el saldo en la cuenta de Juana era de $300. a - 1b - c2? Justifica tu respuesta.
Durante el mes ella expidió cheques por $45, $55, $165, 17. Denotemos con C el conjunto de los números comple-
$35 y $100 e hizo depósitos por $75, $25 y $400. tos, con E el conjunto de los enteros, E + como el con-
a. Si se representa un cheque con un número negativo y junto de los enteros positivos y E - como el conjunto de
un depósito con un número positivo, expresa las tran- los enteros negativos. Halla lo siguiente:
sacciones de Juana como una suma de enteros positi- a. C ´ E
vos y negativos. b. C ¨ E
b. ¿Cuál es el saldo de la cuenta de Juana al final del c. E + ´ E -
mes? d. E + ¨ E -
10. Usa un modelo de recta numérica para encontrar lo si- e. C - E
guiente: f. E - C
a. - 4 - - 1 b. - 4 - - 3 18. Sea f 1x2 = - x - 1 con dominio E. Halla lo siguiente:
11. Usa patrones para ilustrar lo siguiente: a. f 1 - 12 b. f 11002
a. - 4 - - 1 = - 3 b. - 2 - 1 = - 3 c. f 1 - 22 d. f 1 - a2 en términos de a
12. Efectúa cada una de las operaciones siguientes:
a. - 2 + 13 - 102
e. ¿Para qué valores de x la salida es 3?
b. 38 - 1 - 524 - 10
19. Halla todos los enteros x, si los hay, tales que lo siguiente
c. 1 - 2 - 72 + 10
sea verdadero:
a. - x es positivo.
13. En cada caso, escribe un problema de substracción y un b. - x es negativo.
problema de suma que correspondan a la pregunta, y c. - x - 1 es positivo.
después responde:
d. ƒ x ƒ = 2.

© Lopez Mateos Editores. ISBN 978-607-95583-2-1, obra completa, versión electrónica, ISBN 978-607-95583-3-8, volumen 1, versión electrónica. Ejemplar asignado a: Helecto Villarroel gutierrez -
helecto@gmail.com. Fecha: 27 de octubre de 2014. Prohibida su modificación, copia o distribución.
266 Enteros y teoría de números

20. Sea f (x2 = ƒ 1 - x ƒ con dominio E. Halla lo siguiente: 25. En una sucesión aritmética, el octavo término menos el pri-
a. f 1102 b. f 1 - 12 mer término es igual a 21. La suma del primer y el octavo
c. Todas las entradas cuya salida sea 1 términos es - 5. Halla el quinto término de la sucesión.
d. El rango 26. Clasifica cada una de las siguientes expresiones como
21. En cada caso, halla todos los enteros x que satisfagan la verdadera o falsa. Si es falsa, exhibe un contraejemplo.
ecuación dada: a. ƒ - x ƒ = ƒ x ƒ
a. ƒ x - 6 ƒ = 6 b. ƒ x - y ƒ = ƒ y - x ƒ
b. ƒ x ƒ + 2 = 10 c. ƒ - x + - y ƒ = ƒ x + y ƒ
c. ƒ - x ƒ = ƒ x ƒ 27. Resuelve las ecuaciones siguientes:
22. Determina cuántos enteros hay entre los siguientes ente- a. x + 7 = 3 b. - 10 + x = - 7
ros dados (sin incluir los enteros dados): -
c. x = 5
a. 10 y 100 b. - 30 y - 10 28. Efectúa con tu calculadora cada uno de los siguientes
23. Supón que a = 6, b = 5, c = 4 y d = - 3. Inserta parén- problemas de aritmética con enteros, usando la tecla de
tesis en la expresión a - b - c - d para obtener el mayor cambio de signo. Por ejemplo, en algunas calculadoras,
y el menor valor posibles. ¿Cuáles son estos valores? para hallar - 5 + - 4 se teclea 5 +/- + 4 +/- =
24. Una sucesión aritmética puede tener diferencia positiva o .
negativa. En cada una de las sucesiones aritméticas siguien-
tes, halla la diferencia y escribe los dos términos siguientes: a. - 12 + - 6 b. - 12 + 6
a. 0, - 3, - 6, - 9 c. 27 + - 5 d. - 12 - 6
b. x + y, x, x - y e. 16 - - 7

Evaluación 5-1B

1. Halla el inverso aditivo de cada uno de los enteros siguien- perdió 2 yd, ganó 7 yd, ganó 0 yd y perdió 8 yd. ¿Cuál
tes. Escribe tu respuesta de la forma más sencilla posible. es el total ganado o perdido?
a. 3 b. - 4 9. Usa un modelo de recta numérica para encontrar lo
c. q d. 6 siguiente:
e. - n f. 3 + x a. - 3 - - 2 b. - 4 - 3
2. Simplifica lo siguiente: 10. Usa patrones para ilustrar lo siguiente:
a. - 1 - 52 a. - 2 - - 3 = 1 b. - 3 - 2 = - 5
b. - 1 - x2 11. Efectúa cada una de las operaciones siguientes:
3. Evalúa lo siguiente: a. - 2 - 17 + 102
a. ƒ - 3 ƒ b. ƒ 15 ƒ c. - ƒ - 3 ƒ b. 8 - 11 - 10
4. Ilustra cada una de las sumas siguientes por medio del c. - 2 - 7 + 3
modelo de las fichas o del campo de carga: 12. Responde a lo siguiente:
a. - 2 + 5 b. - 5 + 2 c. - 3 + - 3 a. En un juego de triminos, los marcadores de Javier en
5. Ilustra cada una de las sumas del problema 4 usando un cinco turnos sucesivos son 17, - 8, - 9, 14 y 45. ¿Cuál
modelo de recta numérica. es el total al final de los cinco turnos?
6. Calcula cada cada caso usando a - b = a + - b. b. La cámara de burbujas más grande del mundo tiene
a. - 3 - 5 15 pies de diámetro y contiene 7259 gal de hidrógeno
b. 5 - 1 - 32 líquido a una temperatura de - 247°C. Si la tempera-
7. Responde cada parte del problema 6 usando la defini- tura desciende 11°C por hora durante 2 horas conse-
ción de resta con el enfoque del sumando faltante. cutivas, ¿cuál es la nueva temperatura?
8. Escribe una suma básica que corresponda a cada una de las c. Los mayores rangos de temperatura medidos en el
expresiones siguientes y después responde la pregunta: mundo están alrededor del “polo frío” en Siberia. Las
a. En un casino de Las Vegas, un visitante perdió $200, temperaturas en Verjoyansk han variado de - 94°F a
ganó $100 y después perdió $50. ¿Cuál es el cambio 98°F. ¿Cuál es la diferencia entre las temperaturas
que hubo en el valor neto del jugador? mayor y menor en Verjoyansk?
b. En cuatro “downs” el equipo de futbol americano 13. Simplifica cada caso lo más posible. Muestra todos los

© Lopez Mateos Editores. ISBN 978-607-95583-2-1, obra completa, versión electrónica, ISBN 978-607-95583-3-8, volumen 1, versión electrónica. Ejemplar asignado a: Helecto Villarroel gutierrez -
helecto@gmail.com. Fecha: 27 de octubre de 2014. Prohibida su modificación, copia o distribución.
Sección 5-1 Los enteros y las operaciones de suma y resta 267

pasos. teros dados (sin incluir los enteros dados):


a. 4x - 2 - 3x b. 4x - 12 - 3x2 a. - 10 y 10
14. Denotemos con C el conjunto de los números comple- b. x y y (si x 6 y)
tos, con E el conjunto de los enteros, E + como el con- 23. De la media noche a la 1:00 a.m. en enero, la temperatura
junto de los enteros positivos y E - como el conjunto de descendió 5°C. Después de descender, la temperatura en
los enteros negativos. Halla lo siguiente: el exterior era de - 2°C. ¿Cuál era la temperatura a la me-
a. C - E + b. C - E - c. E ¨ E dia noche?
15. a. Demuestra que x - y = - y - x para todos los enteros
- 24. Una sucesión aritmética puede tener diferencia positiva o
entx y y. negativa. En cada una de las sucesiones aritméticas siguien-
b. ¿Implica la parte (a) que la resta es conmutativa? Ex- tes, halla la diferencia y escribe los dos términos siguientes:
plica. a. 7, 3, - 1, - 5
16. Completa el cuadrado mágico usando los enteros si- b. 1 - 3x, 1 - x, 1 + x
25. Halla las sumas de las siguientes sucesiones aritméticas:
a. - 20 + - 19 + - 18 + Á + 18 + 19 + 20
b. 100 + 99 + 98 + Á + - 50
–1 c. 100 + 98 + 96 + Á + - 6
26. Clasifica cada una de las siguientes expresiones como
verdadera o falsa. Si es falsa, exhibe un contraejemplo.
a. ƒ x2 ƒ = x2 b. ƒ x3 ƒ = x3
3 2
guientes: - 13, - 10, - 7, - 4, 2, 5, 8, 11. c. ƒ x ƒ = x ƒ x ƒ
17. Sea f 1x2 = - 3x - 2 con dominio E. Halla lo siguiente: 27. Resuelve las ecuaciones siguientes:
a. f 1 - 12 b. f 11002 a. - x + 5 = 7 b. 1 - x = - 13
c. f 1 - 22 d. f 1 - a2 en términos de a c. - x - 8 = - 9
e. ¿Para qué valores de x la salida es - 11? 28. Supón que el engranaje A tiene 56 dientes y que el en-
18. Halla todos los enteros x, si los hay, tales que lo siguiente granaje B tiene 14 dientes. Supón que el número de ro-
sea verdadero: taciones en sentido contrario al que giran las manecillas
a. - ƒ x ƒ = 2. del reloj está representado por un número positivo y que
b. - ƒ x ƒ es negativo. el número de rotaciones en el sentido en que giran las
c. - ƒ x ƒ es positivo. manecillas del reloj está dado por un número negativo.
d. - x - 1 es positivo. Si el engranaje A gira 7 veces por minuto, ¿cuántas veces
e. - x - 1 es negativo. por minuto gira el engranaje B? Explica tu razona-
19. Sea f 1x2 = ƒ x - 5 ƒ con dominio E. Halla lo siguiente: miento.
a. f 1102 b. f 1 - 12
c. Todas las entradas cuya salida sea 7
d. El rango
20. a. Para cada una de las funciones siguientes, halla f 1 f 1x22:
i. f 1x2 = x
ii. f 1x2 = - x
iii. f 1x2 = - x + 2
b. Interpreta tus respuestas en la parte (a) usando el mo- A B
delo de la máquina-función.
c. Halla otras funciones para las cuales f 1 f 1x22 = x.
Justifica tu respuesta.
21. Por la definición de valor absoluto, la función f 1x2 = ƒ x ƒ 29. Estima cada caso y después usa una calculadora para ha-
se puede escribir como sigue: llar la respuesta real:
a. 343 + - 42 - 402
x, si x Ú 0
f 1x2 = e - b. - 1992 + 3005 - 497
x, si x 6 0
c. 992 - - 10,003 - 101
Escribe la función f 1x2 = ƒ x - 6 ƒ de manera análoga, d. - 301 - - 1303 + 4993
sin usar el símbolo de valor absoluto. 30. Halla una manera rápida de calcular lo siguiente:
1 - 2 + 3 - 4 + 5 - 6 + Á - 2004 + 2005 -
22. Determina cuántos enteros hay entre los siguientes en- 2006 + 2007

© Lopez Mateos Editores. ISBN 978-607-95583-2-1, obra completa, versión electrónica, ISBN 978-607-95583-3-8, volumen 1, versión electrónica. Ejemplar asignado a: Helecto Villarroel gutierrez -
helecto@gmail.com. Fecha: 27 de octubre de 2014. Prohibida su modificación, copia o distribución.
268 Enteros y teoría de números

Conexiones matemáticas 5-1

Comunicación Solución abierta


1. Un chofer en una autopista tuvo problemas con su carro. 8. Describe con palabras un problema que se modele con
Sabía que cuando se presentó la falla, hacía 12 minutos - 50 + 1 - 852 - 1 - 302.
que había pasado por la señal de la milla 68. Suponiendo 9. En una biblioteca algunos pisos están debajo del nivel de
que estaba confundido y desorientado cuando llamó por la superficie y otros están arriba. Si el piso de la superfi-
su teléfono celular solicitando ayuda, ¿cómo puede de- cie se designa como piso cero, diseña un sistema que nu-
terminar su posible posición? Explica. mere los pisos.
2. Dolores asegura que la mejor manera de entender que 10. a. Escojo un entero. Después le resto 10, tomo el
a - b = a + - b, para todos los enteros a y b, es mostrar opuesto del resultado, le sumo - 3 y hallo el opuesto
que cuando sumas b a cada expresión obtienes la misma del nuevo resultado. Mi resultado es - 3. ¿Cuál es el
respuesta. número original?
a. Explica por qué Dolores haría esta afirmación. b. Julia quiere realizar la actividad descrita en la parte
b. ¿Estás de acuerdo con Dolores en que su enfoque es (a) con sus compañeras de clase. Lo más probable es
la “mejor manera”? De no ser así, ¿cuál es un mejor que cada compañera escoja un número diferente, y
enfoque? Julia quiere decir rápidamente a cada compañera el
3. La suma de enteros con signos iguales se puede describir número que escogió. Julia se da cuenta de que basta
usando valores absolutos, como sigue: sumar 7 a cada respuesta. ¿Siempre funciona? Explica
por qué sí o por qué no.
Para sumar enteros con signos iguales, suma los valores abso-
c. Construye tu propio “truco” análogo al de la parte (b)
lutos de los enteros. La suma tiene el mismo signo que los enteros.
que funcione para cada respuesta que recibas de tus
Describe de un modo similar cómo sumar enteros con compañeros de clase.
distinto signo. 11. a. Escribe una función f 1x2 tal que para toda entrada
4. Explica por qué b - a y a - b son inversos aditivos en- entera, la salida sea negativa.
tre sí. b. Escribe una función f 1x2 tal que la sucesión f 1 - 12,
5. a. El valor absoluto de un entero nunca es negativo. ¿Con- f1 - 22, f1 - 32, Á sea una sucesión aritmética.
tradice esto el hecho de que el valor absoluto de x podría
Aprendizaje colectivo
ser igual a - x? Explica por qué sí o por qué no.
b. Explica cómo escribir el inverso aditivo de a - b - c 12. Examinen varios libros de texto de matemáticas elemen-
usando la menor cantidad de símbolos. tales. Expliquen cómo se tratan la suma y la resta de en-
6. Si un entero a se coloca en una recta numérica, entonces la teros y cómo se justifican varias propiedades. Analicen
distancia del punto sobre la recta numérica que representa en grupo cómo se compara el tratamiento de la suma y la
el entero al origen es ƒ a ƒ . Usa esta idea para responder a lo resta de enteros de esta sección con el tratamiento en los
siguiente: libros de texto elementales.
a. Explica por qué ƒ a - b ƒ es la distancia entre los pun- 13. Busca en varios libros de historia de las matemáticas y en
tos que representan a los enteros a y b. Internet cuándo y cómo se introdujeron por primera vez
b. Una manera de definir “menor que” para enteros es los números negativos, y explícalo a tu grupo.
como sigue: a 6 b si, y sólo si, a está a la izquierda de b 14. Toma 21 tarjetas y numéralas con enteros del - 10 al 10.
sobre la recta numérica. En consecuencia, b 7 a si, y Coloca las tarjetas en el piso para formar una recta
sólo si, b está a la derecha de a. Usa estas ideas para numérica. Escoge a una persona de tu grupo para que
marcar sobre una recta numérica todos los enteros x actúe como el caminante de las páginas de muestra para
tales que la suma y resta usando la recta numérica. Da instruc-
i. ƒ x ƒ 6 5. ii. ƒ x ƒ 6 1. ciones al caminante para que se desplace por la recta
iii. ƒ x ƒ Ú 5. iv. ƒ x ƒ 7 - 1. numérica para resolver los problemas 5 y 9 del conjunto
7. Recuerda la definición de “menor que” para números de problemas 5-lA. Resuelve otros problemas de suma y
completos usando la suma y define a 6 b cuando a y b son resta para asegurarte de que todo el grupo comprendió
cualesquier enteros. Usa tu definición para mostrar que el modelo de la recta numérica y que ya lo pueden usar
- 8 6 - 7. en un grupo de educación básica.

© Lopez Mateos Editores. ISBN 978-607-95583-2-1, obra completa, versión electrónica, ISBN 978-607-95583-3-8, volumen 1, versión electrónica. Ejemplar asignado a: Helecto Villarroel gutierrez -
helecto@gmail.com. Fecha: 27 de octubre de 2014. Prohibida su modificación, copia o distribución.
Sección 5-2 Multiplicación y división de enteros 269

Preguntas del salón de clase Pregunta del Third International Mathematics and
15. Una estudiante de cuarto grado diseñó el siguiente algo- Science Study (TIMSS) (Tercer Estudio Internacional
ritmo de la resta para restar 84 - 27: sobre las Matemáticas y la Ciencia)
4 menos 7 igual a 3 negativo. Cuando Patricia salió rumbo a la escuela, la temperatura
era de menos 3 grados.
84
- 27
-3 10 10
80 menos veinte igual a 60.
84 0 0
- 27
-3
60 210 210
60 más 3 negativo igual a 57.
84 En el recreo la temperatura era de 5 grados.
- 27
-3
+ 60 10 10
57
Así, la respuesta es 57. ¿Qué le respondes como maestro? 0 0
16. Una estudiante de octavo grado asegura que puede demos-
trar que la resta de enteros es conmutativa. Ella señala que
si a y b son enteros, entonces a - b = a + - b. Como la 210 210
suma es conmutativa, también lo es la resta. ¿Cuál es tu
respuesta?
17. Una estudiante trazó la siguiente figura de un entero y su ¿Cuántos grados se elevó la temperatura?
opuesto. Otros estudiantes del grupo no estuvieron de a. 2 grados b. 3 grados c. 5 grados d. 8 grados
acuerdo, argumentando que - a a debería estar a la iz- TIMSS, Grado 4, 2003
quierda del 0. ¿Cómo respondes?
Pregunta del National Assessment of Educational Progress
(NAEP) (Evaluación Nacional del Progreso Educativo)
a 0 –a
Paco tenía 32 estampas. Le dio N estampas a su amigo.
18. Una estudiante halló que la suma de enteros se puede ¿Cuál es la expresión que dice cuántas estampas tiene
efectuar obteniendo la suma o la diferencia de los valo- Paco ahora?
res absolutos de estos enteros y después agregando el a. 32 + N b. 32 - N c. N - 32 d. 32 , N
signo “ - ”, de ser necesario. Ella quisiera saber si esto NAEP, Grado 4, 2007
siempre es cierto. ¿Cómo le respondes?

ROMPECABEZAS  Si se escriben en orden los dígitos 1 a 9, es posible colocar signos más y menos entre
los números, o no usar un símbolo de operación, para obtener un total de 100. Por ejemplo,
1 + 2 + 3 - 4 + 5 + 6 + 78 + 9 = 100
¿Puedes obtener un total de 100 usando una menor cantidad de signos más o menos que en el ejemplo
dado? Nota que se pueden combinar dígitos, como 7 y 8 en el ejemplo.

5-2 Multiplicación y división de enteros

Enfocamos las multiplicación de enteros por medio de diversos modelos: patrones, campos de
carga, fichas y recta numérica. Nota que el razonamiento empleado en estos modelos es razo-
namiento inductivo y por lo tanto no constituye una demostración.
© Lopez Mateos Editores. ISBN 978-607-95583-2-1, obra completa, versión electrónica, ISBN 978-607-95583-3-8, volumen 1, versión electrónica. Ejemplar asignado a: Helecto Villarroel gutierrez -
helecto@gmail.com. Fecha: 27 de octubre de 2014. Prohibida su modificación, copia o distribución.
270 Enteros y teoría de números

Modelo del patrón para la multiplicación de enteros


Podemos enfocar la multiplicación de enteros usando sumas repetidas. Por ejemplo, si un
corredor pierde 2 yd en cada uno de sus tres intentos en un juego de futbol americano, en-
tonces tendrá una pérdida neta de - 2 + - 2 + - 2, ó - 6, yardas. Como - 2 + - 2 + - 2 se
puede escribir como 31 - 22, usando suma repetida tenemos 31 - 22 = - 6.
Considera 1 - 223. No tiene sentido decir que hay - 2 tres en una suma. Pero si queremos
que la propiedad conmutativa sea válida para todos los enteros, debemos tener 1 - 223 =
31 - 22 = - 6.
A continuación, considera 1 - 321 - 22. Podemos desarrollar el patrón siguiente:
31 - 22 = - 6
21 - 22 = - 4
11 - 22 = - 2
01 - 22 = 0
- 11 - 22 = ?
- 21 - 22 = ?
- 31 - 22 = ?

Los primeros cuatro productos, - 6, - 4, - 2 y 0, son términos de una sucesión aritmética


con diferencia fija de 2. Si el patrón continúa, los siguientes tres términos de la sucesión son
2, 4 y 6. Así, parece que 1 - 321 - 22 = 6. Asimismo, 1 - 221 - 32 = 6.

O B S E R VA C I Ó N Nota el uso de la frase “parece que 1 - 321 - 22 = 6”. Más adelante en


esta sección veremos por qué - 31 - 22 = 6.

RINCÓN DE LA TECNOLOGÍA En una hoja de cálculo, en la columna A coloca 5 como primer regis-
tro y después escribe una fórmula para sumar - 1 a 5 para obtener el segundo registro. Después suma - 1
al segundo registro y llena hacia abajo, continuando el patrón. Repite el proceso en la columna B. En la
columna C, halla el producto de los registros respectivos en las columnas A y B. ¿Qué patrones observas?

A continuación veremos la multiplicación de los enteros usando el modelo de las fichas,


el modelo de campo de carga y el modelo de la recta numérica. En todos estos modelos co-
menzamos con 0, representado de varias maneras.

Modelo de campo de carga y modelo de las fichas para la multiplicación


Se puede usar tanto el modelo de las fichas como el modelo de campo de carga para ilustrar la
multiplicación de los enteros. Considera la figura 5-11(a), donde se ilustra 31 - 22 usando
un modelo de fichas. El producto 31 - 22 se interpreta como 3 grupos de 2 fichas rojas cada
uno. En la figura 5-11(b), 31 - 22 se ilustra como 3 grupos de 2 cargas negativas.

3 grupos
3 grupos de – – de 2 cargas
2 fichas rojas
– – negativas

– – 3(–2) = –6
3(– 2) = –6
(a) (b)

Figura 5-11
© Lopez Mateos Editores. ISBN 978-607-95583-2-1, obra completa, versión electrónica, ISBN 978-607-95583-3-8, volumen 1, versión electrónica. Ejemplar asignado a: Helecto Villarroel gutierrez -
helecto@gmail.com. Fecha: 27 de octubre de 2014. Prohibida su modificación, copia o distribución.
Sección 5-2 Multiplicación y división de enteros 271

Para obtener 1 - 321 - 22 usando el modelo de fichas, interpretamos los signos como sigue: - 3 se
toma como “quitar 3 grupos de”; - 2 se toma como “2 fichas rojas”. Para hacer esto comenzamos con un
valor de 0 que incluya al menos 6 fichas rojas, como se muestra en la figura 5-12(a). Cuando quitamos 6
fichas rojas quedamos con 6 fichas negras. El resultado es un 6 positivo, de modo que 1 - 321 - 22 = 6.
Se puede usar un razonamiento análogo en la figura 5-12(b) con el modelo de campo de carga.

Quitamos 3 Quitamos 3
grupos de 2 grupos de 2
fichas rojas. cargas negativas

+ + + + + + + +
– –– –
+ + + +
– –

0 6 0 Cargas + 6 Cargas
(a) (b)

Figura 5-12
Modelo de la recta numérica
Como con la suma y la resta, ilustramos la multiplicación mediante un caminante que se
mueve a lo largo de una recta numérica, de acuerdo con las reglas siguientes:
1. Viajar hacia la izquierda (al oeste) significa moverse en dirección negativa, y viajar ha-
cia la derecha (el este) significa moverse en dirección positiva.
2. El tiempo futuro se denota con un valor positivo y el tiempo en el pasado se denota con
un valor negativo.
Considera la recta numérica mostrada en la figura 5-13. Damos a continuación varios casos
usando esta recta numérica.

W E
– 18 – 15 – 12 –9 –6 –3 0 3 6 9 12 15 18

1. Si estás ahora en el 0 5 × 4 = 20
te mueves hacia el
Hacia el este durante Estarás 20 km
este a 5 km/h, ¿dónde a 5 km/h 4 h. al este del 0.
estarás dentro de 4 h?
2. Si estás ahora en el 0 5 × –4 = – 20
moviéndote hacia el
Hacia el este hace Estabas 20 km
este a 5 km/h, ¿dónde
a 5 km/h 4 h. al oeste del 0.
estabas hace 4 horas?
3. Si estás ahora en el 0 –5 = – 20
× 4
y te mueves hacia el
oeste a 5 km/h, Hacia el durante Estarás 20 km
¿dónde estarás oeste a 5 km/h 4 h. al oeste del 0.
dentro de 4 h?
4. Si estás ahora en el 0 –5 × –4 = 20
moviéndote hacia el oeste Hacia el hace Estabas 20 km
a 5 km/h, ¿dónde estabas oeste a 5 km/h 4 h. al este del 0.
hace 4 horas?

Figura 5-13
© Lopez Mateos Editores. ISBN 978-607-95583-2-1, obra completa, versión electrónica, ISBN 978-607-95583-3-8, volumen 1, versión electrónica. Ejemplar asignado a: Helecto Villarroel gutierrez -
helecto@gmail.com. Fecha: 27 de octubre de 2014. Prohibida su modificación, copia o distribución.
272 Enteros y teoría de números

En la figura 5-14 se da un uso alternativo de la recta numérica para ilustrar la multipli-


cación de enteros.

a. 3 # 2 quiere decir tres grupos de 2 cada uno: 3 # 2 = 6.

+2 +2 +2

–6 –5 –4 –3 –2 –1 0 1 2 3 4 5 6

b. 31 - 22 quiere decir tres grupos de - 2 cada uno: 31 - 22 = - 6.

–2 –2 –2

–6 –5 –4 –3 –2 –1 0 1 2 3 4 5 6

c. Los enteros 3 y - 3 son opuestos. Puedes pensar a 1 - 322 como el


opuesto de tres grupos de 2 cada uno. Así, 1 - 322 = - 6.

+2 +2 +2

–6 –5 –4 –3 –2 –1 0 1 2 3 4 5 6

d. Puedes pensar a 1 - 321 - 22 como el opuesto de tres grupos de - 2


cada uno. Como 31 - 22 = - 6, - 31 - 22 = 6.

–2 –2 –2

–6 –5 –4 –3 –2 –1 0 1 2 3 4 5 6

Figura 5-14

Estos modelos ilustran el siguiente:

Teorema 5–5
Para cualesquier números completos a y b, es válido que:
1. 1 - a21 - b2 = ab
2. 1 - a2b = b1 - a2 = - 1ab2

O B S E R VA C I Ó N Más adelante en esta sección mostraremos que este teorema es cierto


para todos los enteros a y b.

Propiedades de la multiplicación de enteros


El conjunto de los enteros tiene propiedades bajo la multiplicación análogas a las del con-
junto de los números completos bajo la multiplicación. A continuación se resumen estas
propiedades.

© Lopez Mateos Editores. ISBN 978-607-95583-2-1, obra completa, versión electrónica, ISBN 978-607-95583-3-8, volumen 1, versión electrónica. Ejemplar asignado a: Helecto Villarroel gutierrez -
helecto@gmail.com. Fecha: 27 de octubre de 2014. Prohibida su modificación, copia o distribución.
Sección 5-2 Multiplicación y división de enteros 273

Teorema 5–6:    Propiedades de la multiplicación de enteros


El conjunto de los enteros E satisface las siguientes propiedades de la multiplicación para to-
dos los enteros a, b y c 僆 E:
Propiedad de la cerradura de la multiplicación de enteros ab es un entero único.
Propiedad conmutativa de la multiplicación de enteros ab = ba.
Propiedad asociativa de la multiplicación de enteros 1ab2c = a1bc2.
Propiedad de la identidad multiplicativa 1 es el único entero tal que para todos los enteros a,
1 # a = a = a # 1.
Propiedades distributivas de la multiplicación sobre la suma de enteros
a1b + c2 = ab + ac y 1b + c2a = ba + ca.
Propiedad de los enteros de la multiplicación por cero 0 es el único entero tal que para
todos los enteros a, a # 0 = 0 = 0 # a.

Un enfoque para mostrar que 1 - 223 = - 12 # 32 usa la propiedad de la unicidad de los in-
versos aditivos. Si podemos mostrar que 1 - 223 y - 12 # 32 son inversos aditivos del mismo
número, entonces deben ser iguales. Por definición, el inverso aditivo de 12 # 32 es - 12 # 32.
Que 1 - 223 también es inverso aditivo de 2 # 3 se puede demostrar al probar que
1 - 223 + 2 # 3 = 0. A continuación se presenta la demostración:
1 - 223 + 2 # 3 = 1 - 2 + 223 Propiedad distributiva de la multiplicación sobre la suma
= 0#3 Inverso aditivo
= 0 Multiplicación por cero
Tenemos ahora que
1 - 223 + 2 # 3 = 0
- 12 # 32 + 2 # 3 = 0
Como 1 - 223 y - 12 # 32 son inversos aditivos de 12 # 32 y como el inverso aditivo debe ser
único, 1 - 223 = - 12 # 32. Usando este enfoque podríamos demostrar el teorema siguiente
(la demostración se explora en la Evaluación 5-2A):

Teorema 5–7
Para todo entero a, 1 - 12a = - a.

Es importante tener presente que 1 - 12a = - a es verdadero para todos los enteros a. Así,
si substituimos a por - 1, obtenemos 1 - 121 - 12 = - 1 - 12. Como - 1 - 12 = 1, tenemos
otra justificación para el hecho de que 1 - 121 - 12 = 1. Usando este resultado, la propiedad
anterior y las propiedades de los enteros listados anteriormente, podemos mostrar que
1 - a2b = - 1ab2 y que 1 - a21 - b2 = ab para todos los enteros a y b, como sigue:

◆ Nota Emmy Noether (1882–1935) realizó importantes contribuciones al estudio de los anillos,
histórica sistemas algebraicos entre los cuales está el conjunto de los enteros. Cuando entró a la Uni-
versidad de Erlanger (Alemania) en 1900, Emmy Noether era una de las dos únicas muje-
res inscritas. Aún después de terminar el doctorado en 1907 no podía hallar un trabajo
apropiado pues era mujer. En 1919 obtuvo un nombramiento universitario sin recibir pago
y sólo después comenzó a recibir un muy modesto salario. En 1933, junto con muchos otros
profesores, fue despedida de la Universidad de Göttingen por ser judía. Emigró a Estados
Unidos e impartió clases en Bryn Mawr College hasta su muerte prematura sólo 18 meses
después de haber llegado a Estados Unidos. ◆

© Lopez Mateos Editores. ISBN 978-607-95583-2-1, obra completa, versión electrónica, ISBN 978-607-95583-3-8, volumen 1, versión electrónica. Ejemplar asignado a: Helecto Villarroel gutierrez -
helecto@gmail.com. Fecha: 27 de octubre de 2014. Prohibida su modificación, copia o distribución.
274 Enteros y teoría de números

1 - a2b = 31 - 12a4b
= 1 - 121ab2
= - 1ab2
Además: 1 - a21 - b2 = 31 - 12a431 - 12b4
= 31 - 121 - 1241ab2
= 11ab2
= ab
Hemos establecido los teoremas siguientes:

Teorema 5–8
Para todos los enteros a y b, 1 - a2b = - 1ab2
1 - a21 - b2 = ab

O B S E R VA C I Ó N Es importante notar que en el teorema 5–8, - a y - b no son necesa-


riamente negativos y que a y b no son necesariamente positivos.

La propiedad distributiva de la multiplicación sobre la resta se sigue de la propiedad dis-


tributiva de la multiplicación sobre la suma:
a1b - c2 = a1b + - c2
= ab + a1 - c2
= ab + - 1ac2
= ab - ac
En consecuencia, a1b - c2 = ab - ac. De aquí, y de la propiedad conmutativa de la
multiplicación, vemos que 1b - c2a = ba - ca.

Teorema 5–9:    Propiedad distributiva de la multiplicación sobre la resta de


enteros
Para cualesquier enteros a, b y c,
a1b - c2 = ab - ac y 1b - c2a = ba - ca

Ejemplo 5-10 Simplifica cada uno de los casos siguientes de modo que no haya paréntesis en la respuesta final:
a. 1 - 321x - 22 1a + b21a - b2








b.
Solución a. 1 - 321x - 22 = 1 - 32x - 1 - 32122 = - 3x - 1 - 62 = - 3x + - 1 - 62 = - 3x + 6
b. 1a + b21a - b2 = 1a + b2a - 1a + b2b
= 1a2 + ba2 - 1ab + b22
= a2 + ba + - 1ab + b22
= a2 + ab + - 1ab2 + - b2 (Nota: - b2 significa - 1b22.)
2
= a + 0 + b - 2

= a2 - b2
Así, 1a + b21a - b2 = a2 - b2.

© Lopez Mateos Editores. ISBN 978-607-95583-2-1, obra completa, versión electrónica, ISBN 978-607-95583-3-8, volumen 1, versión electrónica. Ejemplar asignado a: Helecto Villarroel gutierrez -
helecto@gmail.com. Fecha: 27 de octubre de 2014. Prohibida su modificación, copia o distribución.
Sección 5-2 Multiplicación y división de enteros 275

El resultado 1a + b21a - b2 = a2 - b2 en el ejemplo 5-10(b) es la fórmula de la diferencia


de cuadrados.

Ejemplo 5-11 Usa la fórmula de la diferencia de cuadrados para simplificar lo siguiente:


a. 14 + b214 - b2 b. 1 - 4 + b21 - 4 - b2 c. 1x + 322 - 1x - 322









Solución a. 14 + b214 - b2 = 42 - b2 = 16 - b2
b. 1 - 4 + b21 - 4 - b2 = 1 - 422 - b2 = 16 - b2
c. 1x + 322 - 1x - 322 = 31x + 32 + 1x - 324 31x + 32 - 1x - 324
= 2x1x + 3 - x + 32
= 2x # 6
= 12x

AHORA INTENTA ÉSTE 5-5 Determina cómo usar la fórmula de la diferencia de cuadrados para cal-
cular mentalmente lo siguiente:
a. 101 # 99 b. 22 # 18 c. 24 # 36 d. 998 # 1002

Cuando la propiedad distributiva de la multiplicación sobre la resta se escribe en orden


inverso como
ab - ac = a1b - c2 y ba - ca = 1b - c2a
y de manera similar para la suma, las expresiones del lado derecho de cada ecuación están
en forma factorizada. Decimos que hemos factorizado o sacado el factor común a. Tanto la fór-
mula de la diferencia de cuadrados como las propiedades distributivas de la multiplicación
sobre la suma y la resta se pueden usar para factorizar.

Ejemplo 5-12 Factoriza completamente lo siguiente:


b. 1x + y22 - z2








a. x2 - 9 c. - 3x + 5xy d. 3x - 6 e. 5x2 - 2x2

Solución a. x2 - 9 = x2 - 32 = 1x + 321x - 32
b. 1x + y22 - z2 = 1x + y + z21x + y - z2
c. - 3x + 5xy = x1 - 3 + 5y2
d. 3x - 6 = 31x - 22
e. 5x2 - 2x2 = 15 - 22x2 = 3x2

División de enteros
En el conjunto de los números completos, a , b, donde b Z 0, es el único número com-
pleto c tal que a = bc. Si no existe dicho número completo c, entonces a , b está indefinido.
La división en el conjunto de los enteros se define de manera análoga.

Definición de división de enteros


Si a y b son enteros cualesquiera, entonces a , b es el único entero c, si es que existe, tal que a = bc.

O B S E R VA C I Ó N Nota que a , b, si es que existe, es la solución de a = bx.

© Lopez Mateos Editores. ISBN 978-607-95583-2-1, obra completa, versión electrónica, ISBN 978-607-95583-3-8, volumen 1, versión electrónica. Ejemplar asignado a: Helecto Villarroel gutierrez -
helecto@gmail.com. Fecha: 27 de octubre de 2014. Prohibida su modificación, copia o distribución.
276 Enteros y teoría de números

Ejemplo 5-13 Usa la definición de división de enteros, de ser posible, para evaluar cada una de las siguien-









tes operaciones:
a. 12 , 1 - 42 b. - 12 , 4 c. - 12 , 1 - 42 d. - 12 , 5
e. 1ab2 , b, b Z 0 f. 1ab2 , a, a Z 0

Solución a. Sea 12 , 1 - 42 = c. Entonces 12 = - 4c y, en consecuencia, c = - 3. Así,


12 , 1 - 42 = - 3.
b. Sea - 12 , 4 = c. Entonces - 12 = 4c y, por lo tanto, c = - 3. Entonces,
- 12 , 4 = - 3.
c. Sea - 12 , 1 - 42 = c. Entonces - 12 = - 4c y, en consecuencia, c = 3.
Entonces, - 12 , 1 - 42 = 3.
d. Sea - 12 , 5 = c. Entonces - 12 = 5c. Como no existe entero c que satisfaga
esta ecuación (¿por qué?), decimos que - 12 , 5 está indefinido en los enteros.
e. Sea 1ab2 , b = x. Entonces ab = bx y, en consecuencia, x = a.
f. Sea 1ab2 , a = x. Entonces ab = ax y, por lo tanto, x = b.

El ejemplo 5-13 sugiere que el cociente de dos números negativos, de existir, es un entero posi-
tivo y que el cociente de un entero positivo y uno negativo, de existir, o de un entero negativo y uno
positivo, de existir, es negativo.

AHORA INTENTA ÉSTE 5-6 Usa la definición de división de enteros para mostrar que no es posible di-
vidir entre 0.

Orden de las operaciones en enteros


Cuando la suma, resta, multiplicación, división y exponenciación aparecen sin paréntesis, la
exponenciación se efectúa primero de derecha a izquierda, después las multiplicaciones y
divisiones en el orden en que aparecen de izquierda a derecha, y a continuación las sumas y
restas en el orden en que aparecen de izquierda a derecha. Las operaciones aritméticas que
aparecen dentro de paréntesis deben efectuarse primero.

Ejemplo 5-14 Evalúa cada caso:


b. 12 - 524 + 1








a. 2 - 5 # 4 + 1 c. 2 - 3 # 4 + 5 # 2 - 1 + 5
d. 2 + 16 , 4 # 2 + 8 e. 1 - 324 f. - 34

Solución a. 2 - 5 # 4 + 1 = 2 - 20 + 1 = - 18 + 1 = - 17
b. 12 - 524 + 1 = 1 - 324 + 1 = - 12 + 1 = - 11
c. 2 - 3 # 4 + 5 # 2 - 1 + 5 = 2 - 12 + 10 - 1 + 5 = 4
d. 2 + 16 , 4 # 2 + 8 = 2 + 4 # 2 + 8 = 2 + 8 + 8 = 10 + 8 = 18
e. 1 - 324 = 1 - 321 - 321 - 321 - 32 = 81
f. - 34 = - 1342 = - 1812 = - 81

O B S E R VA C I Ó N Nota que del ejemplo 5-14(e) y (f), tenemos que 1 - 324 Z - 34. Por
convención, 1 - x24 significa 1 - x21 - x21 - x21 - x2 y - x4 significa - 1x42 y por lo tanto es
igual a - 1x # x # x # x2.
© Lopez Mateos Editores. ISBN 978-607-95583-2-1, obra completa, versión electrónica, ISBN 978-607-95583-3-8, volumen 1, versión electrónica. Ejemplar asignado a: Helecto Villarroel gutierrez -
helecto@gmail.com. Fecha: 27 de octubre de 2014. Prohibida su modificación, copia o distribución.
Sección 5-2 Multiplicación y división de enteros 277

Orden en los enteros


Como en los números completos, se puede usar una recta numérica como la mostrada en la
figura 5-15 para describir las relaciones de mayor que y menor que para el conjunto de
los enteros. Como - 5 está a la izquierda de - 3 en la recta numérica, decimos que “ - 5 es
menor que - 3” y escribimos - 5 6 - 3. También podemos decir que “ - 3 es mayor que
- 5” y escribir - 3 7 - 5.

–5 –4 –3 –2 –1 0 1 2 3 4 5

Figura 5-15

Nota que debido a que - 5 está a la izquierda de - 3, existe un entero positivo que puede
sumarse a - 5 para obtener - 3, a saber, 2. Así, - 5 6 - 3 pues - 5 + 2 = - 3. La definición
de menor que para enteros es análoga a la usada para los números completos.

Definición de menor que para enteros


Para cualesquier enteros a y b, a es menor que b y se escribe a 6 b, si, y sólo si, existe un entero
positivo k tal que a + k = b.

La última ecuación implica que k = b - a. Así, tenemos el teorema siguiente:

Teorema 5–10
a 6 b (o, de manera equivalente, b 7 a) si, y sólo si, b - a es igual a un entero positivo; esto es,
b - a es mayor que 0.
◆ Nota de
investigación
Los estudiantes no Usando este teorema, - 5 6 - 3 pues - 3 - 1 - 52 = - 3 + - 1 - 52 = - 3 + 5 = 2 7 0.
comprenden muy (Además a … b significa que a 6 b ó a = b. Nota que b 7 a si, y sólo si, a 6 b. También b Ú a
bien el concepto de si, y sólo si, a … b.)
ecuaciones equivalen- Se puede usar la propiedad anterior para justificar cada uno de los casos siguientes para
tes. Por ejemplo, aun- enteros x, y y n:
que son capaces de
usar transformaciones Teorema 5–11
para resolver ecuacio- a. Si x 6 y y n es un entero cualquiera, entonces x + n 6 y + n.
nes sencillas b. Si x 6 y, entonces - x 7 - y.
1x + 2 = 5 se c. Si x 6 y y n 7 0, entonces nx 6 ny.
convierte en d. Si x 6 y y n 6 0, entonces nx 7 ny.
x + 2 - 2 = 5 - 22,
los estudiantes pare-
cen no estar conscien- A continuación se justifica lo anterior.
tes de que cada a. Como x 6 y, y - x 7 0. Necesitamos demostrar que 1y + n2 - 1x + n2 7 0. Tene-
transformación mos y + n - 1x + n2 = y + n - x - n = y - x. Como y - x 7 0, tenemos y + n -
produce una ecuación 1x + n2 7 0 y, por lo tanto, x + n 6 y + n.
equivalente b. Como x 6 y, y - x 7 0. Necesitamos demostrar que - x - 1 - y2 7 0. Tenemos
(Steinberg et al. - x - 1 - y2 = - x + - 1 - y2 = - x + y = y + - x = y - x. Como y - x 7 0, tene-
1990). ◆ mos - x - 1 - y2 7 0 y, por lo tanto, - x 7 - y.
© Lopez Mateos Editores. ISBN 978-607-95583-2-1, obra completa, versión electrónica, ISBN 978-607-95583-3-8, volumen 1, versión electrónica. Ejemplar asignado a: Helecto Villarroel gutierrez -
helecto@gmail.com. Fecha: 27 de octubre de 2014. Prohibida su modificación, copia o distribución.
278 Enteros y teoría de números

c. Como x 6 y, y - x 7 0. Necesitamos demostrar que ny - nx 7 0. Tenemos ny - nx =


n1 y - x2. Como n es un entero positivo y y - x es positivo, n1y - x2 también debe ser po-
sitivo. Como ny - nx 7 0, tenemos que nx 6 ny.
d. Para mostrar que nx 7 ny, sólo necesitamos demostrar que nx - ny 7 0. Tenemos
nx - ny = n1x - y2. Como y - x 7 0, x - y 6 0 (¿por qué?). Como n 6 0 y
x - y 6 0, n1x - y2 es positivo. Así, nx - ny 7 0 y, por lo tanto, nx 7 ny.
De la Nota de investigación en la página 277 vemos que los alumnos no comprenden bien el
concepto de ecuaciones equivalentes. En el ejemplo 5-15 se practica este concepto.

Ejemplo 5-15 Usa los teoremas desarrollados anteriormente para hallar todos los enteros x que satisfagan lo
siguiente:








a. x + 3 6 - 2
b. - x - 3 6 5
c. Si x … - 2, halla los valores de 5 - 3x.

Solución a. Si x + 3 6 - 2 entonces, por el teorema 5–11 (a),


x + 3 + -3 6 -2 + -3
x 6 - 5, x es un entero.
También podemos escribir el conjunto solución (el conjunto de todas las solu-
5 - 6, - 7, - 8, - 9, Á 6
ciones) como

Nota que, hablando estrictamente, sólo hemos demostrado que todo x que satis-
face la primera desigualdad también satisface x 6 - 5. Para estar seguros de que
x 6 - 5 representa todas las soluciones, necesitamos mostrar el recíproco; esto
es, si x 6 - 5 entonces x + 3 6 - 2. Esto se puede hacer fácilmente sumando 3
en ambos lados de x 6 - 5.
b. Si - x - 3 6 5, entonces
-x - 3 + 3 6 5 + 3
- x 6 8
- 1 - x2 7 - 8 por el teorema 5–11 1b2
x 7 - 8, x es un entero
c. Si x … - 2, entonces
- 3x Ú - 31 - 22
- 3x Ú 6
-
5 + 3x Ú 5 + 6
5 - 3x Ú 5 + 6
5 - 3x Ú 11; esto es, todos los enteros del conjunto 511, 14, 17, 20, Á 6.

Extensión del sistema coordenado
Extendimos una recta numérica para incluir todos los enteros. Esta nueva recta numérica ex-
tendida puede convertirse en los ejes x y y de un sistema coordenado. En el capítulo 12 in-
vestigaremos el sistema coordenado para todos los números reales. Mientras tanto, observa
la Página de un libro de texto donde el sistema coordenado se extiende para incluir los enteros
negativos. Responde las preguntas de esa página de muestra. Nota que cuando x = 0, las
descripciones de derecha o izquierda del eje y y la analogía para y = 0 no son ciertas.

© Lopez Mateos Editores. ISBN 978-607-95583-2-1, obra completa, versión electrónica, ISBN 978-607-95583-3-8, volumen 1, versión electrónica. Ejemplar asignado a: Helecto Villarroel gutierrez -
helecto@gmail.com. Fecha: 27 de octubre de 2014. Prohibida su modificación, copia o distribución.
Sección 5-2 Multiplicación y división de enteros 279

Página de un libro de texto EL SISTEMA COORDENADO

Lección
Calentamiento
´ Graficación de pares ordenados
Idea clave
La ubicación de un Aprende
punto en el plano
coordenado se puede ¿Qué es un plano coordenado?
describir por medio
de un par ordenado El término pixel es una contracción de picture elements
de números. y se usa para describir los puntos en un monitor de
computadora. Los programadores de computadoras usan
Vocabulario pares ordenados para colocar los pixeles en un monitor.
plano coordenado
cuadrante Un plano coordenado es una cuadrícula que contiene dos
par ordenado rectas numéricas que se intersecan formando un ángulo
origen
recto en el cero. Las rectas numéricas, que se llaman
coordenada x
coordenada y eje x y eje y, dividen el plano en cuatro cuadrantes.
eje x
Un par ordenado (x, y) de números da las
eje y
coordenadas y la ubicación de un punto.

Materiales
papel cuadriculado o La coordenada x La coordenada y
muestra la posición muestra la posición
a la izquierda o a la sobre o debajo del
derecha del eje y. eje x.

Si un punto está sobre


un eje, no pertenece a ningún eje y
cuadrante. El punto S está
sobre el eje y. Cuadrante Cuadrante

eje x

Para localizar
cualquier punto necesito
conocer su distancia y Cuadrante Cuadrante
dirección horizontales y
verticales a partir de (0, 0)
o el origen.

Tema de plática
¿Dónde estará el punto Q si ambas
coordenadas x y y son negativas?
Colócalo en la RED
¿Dónde estará el punto R si su Más ejemplos
coordenada y es 0?

Fuente: Scott Foresman-Addison Wesley, Mathematics 2008, Grade 6 ( p. 440).

© Lopez Mateos Editores. ISBN 978-607-95583-2-1, obra completa, versión electrónica, ISBN 978-607-95583-3-8, volumen 1, versión electrónica. Ejemplar asignado a: Helecto Villarroel gutierrez -
helecto@gmail.com. Fecha: 27 de octubre de 2014. Prohibida su modificación, copia o distribución.
280 Enteros y teoría de números

AHORA INTENTA ÉSTE 5-7 En los casos siguientes halla y grafica todos los puntos 1x, y2 que satis-
fagan la condición dada, donde x y y son enteros.
a. y = x b. y = - x c. y = ƒ x ƒ d. ƒ x ƒ + ƒ y ƒ = 5

ROMPECABEZAS Expresa cada uno de los números del 1 al 10 usando cuatro dígitos 4 y cualquier
operación. Por ejemplo,
1 = 44 , 44, ó
1 = 14 , 4244, ó
1 = - 4 + 4 + 14 , 42

Evaluación 5-2A

1. Usa patrones para mostrar que 1 - 121 - 12 = 1. c. - 8 , 1 - 8 + 82


2. Usa el modelo del campo de cargas para mostrar que d. 1 - 6 + 62 , 1 - 2 + 22
1 - 421 - 22 = 8. e. ƒ -24 ƒ , 3419 - 1524
3. Usa el modelo de la recta numérica para mostrar que 8. Evalúa cada uno de los siguientes productos y después, de ser
21 - 42 = - 8. posible, escribe dos afirmaciones de división que sean equi-
4. En cada uno de los siguientes modelos de campos de valentes a las afirmaciones sobre multiplicación. Si no es po-
carga se quitan las cargas encerradas con una línea. Es- sible construir dos afirmaciones de división, explica por qué.
cribe el problema correspondiente de multiplicación de a. 1 - 625
enteros cuya solución esté basada en el modelo. b. 1 - 521 - 42
a. b. c. 1 - 320
d. 0 # 0
111111 9. En los casos siguientes, x y y son enteros con y Z 0. Usa
11 11 11
222 222 la definición de división en términos de multiplicación
22 22 22
111 para efectuar las operaciones indicadas. Escribe tus res-
11 11 puestas en la forma más sencilla.
222
22 22 a. 14x2 , 4
b. 1 - xy2 , y
10. En un laboratorio, la temperatura de varias reacciones
5. El número de estudiantes que almuerza en la cafetería de
químicas cambiaba en un número fijo de grados por mi-
la escuela ha decrecido a una tasa de 20 por año. Supo-
nuto. Plantea un problema de multiplicación que des-
niendo que esta tendencia continúa, plantea un pro-
criba cada uno de los casos siguientes:
blema de multiplicación que describa el cambio en el
a. La temperatura a las 8:00 p.m. era de 32°C. Si disminuye
número de estudiantes que almuerzan en la cafetería de
3°C por minuto, ¿cuál es la temperatura a las 8:30 p.m.?
la escuela para los casos siguientes:
b. La temperatura a las 8:20 p.m. era de 0°C. Si disminuyó
a. El cambio en los siguientes 4 años
4°C por minuto, ¿cuál era la temperatura a las 7:55 p.m.?
b. La situación hace 4 años
c. La temperatura a las 8:00 p.m. era de - 20°C. Si disminuyó
c. El cambio en los siguientes n años
4°C por minuto, ¿cuál era la temperatura a las 7:30 p.m.?
d. La situación hace n años
d. La temperatura a las 8:00 p.m. era de 25°C. Si cada
6. Usa la definición de división para hallar cada cociente, de
minuto se incrementó en 3°C, ¿cuál era la tempera-
ser posible. Si un cociente no está definido, explica por qué.
tura a las 7:40 p.m.?
a. - 40 , - 8 b. - 143 , 13
11. Si se predijo que la tierra de cultivo perdida por uso ha-
c. - 5 , 0
bitacional en los próximos 9 años sería de 12,000 acres
7. Evalúa cada caso, de ser posible:
a. 1 - 10 , - 221 - 22
por año, ¿cuánta tierra de cultivo se perdería al dedicarla
b. 1 - 10 # 52 , 5
a casa habitación durante este periodo de tiempo?

© Lopez Mateos Editores. ISBN 978-607-95583-2-1, obra completa, versión electrónica, ISBN 978-607-95583-3-8, volumen 1, versión electrónica. Ejemplar asignado a: Helecto Villarroel gutierrez -
helecto@gmail.com. Fecha: 27 de octubre de 2014. Prohibida su modificación, copia o distribución.
Sección 5-2 Multiplicación y división de enteros 281

12. Muestra que la propiedad distributiva de la multiplica- 23. Factoriza completamente cada una de las expresiones
ción sobre la suma, a1b + c2 = ab + ac, es verdadera siguientes.
para cada uno de los valores de a, b y c: a. 3x + 5x
a. a = - 1, b = - 5, c = - 2 b. xy + x
b. a = - 3, b = - 3, c = 2 c. x2 + xy
13. Calcula los casos siguientes: d. 3xy + 2x - xz
a. 1 - 223 b. 1 - 224 e. abc + ab - a
c. 1 102 , 1 102
- 5 - 2
f. 16 - a2
d. 1 - 325 , 1 - 32 g. 4x2 - 25y2
e. 1 - 1250 f. 1 - 12151 24. a. Usa la propiedad distributiva de la multiplicación so-
g. 2 + 3 # 5 - 1
- h. 10 - 3 # 7 - 41 - 22 + 3 bre la suma o sobre la resta para mostrar que
14. Calcula lo siguiente sin usar una calculadora: 1a - b22 = a2 - 2ab + b2
a. 1 - 2264 - 264 b. - 28 + 28
c. 1 22 + 0 9 - ƒ 7 - 15 ƒ - 15
- - 5 # b. Usa los resultados que obtuviste en (a) para calcular
15. Si x es un entero y x Z 0, ¿cuáles de las siguientes expresio- mentalmente cada uno de los casos siguientes:
nes son siempre positivas y cuáles son siempre negativas? (i) 982 (Sugerencia: Escribe 98 = 100 - 2.)
a. - x2 b. x2 c. 1 - x22 (ii) 992
d. x- 3
e. 1 x2
- 3 (iii) 9972
16. ¿Cuáles de las expresiones del problema 15 son iguales 25. En cada uno de los casos a continuación halla los dos térmi-
entre sí para todos los valores de x? nos siguientes. Supón que la sucesión es aritmética o geo-
17. Identifica la propiedad de los enteros ilustrada en cada métrica y halla su diferencia o razón, y el término n-ésimo.
caso: a. - 10, - 7, - 4, - 1, 2, 5, _, _
a. 1 - 3214 + 52 = 14 + 521 - 32 b. - 2, - 4, - 8, - 16, - 32, - 64, _, _
b. 1 - 421 - 72 僆 E c. 2, - 22, 23, - 24, 25, - 26, _, _
c. 5341 - 324 = 15 # 421 - 32
26. Halla la suma de los primeros 100 términos en la suce-
sión aritmética - 10, - 7, - 4, - 1, 2, 5, Á .
d. 1 - 9235 + 1 - 824 = 1 - 92 # 5 + 1 - 921 - 82
27. Halla los primeros cinco términos de las sucesiones
18. Simplifica cada caso:
a. 1 - x21 - y2
cuyo término n-ésimo es
b. - 2x1 - y2
a. n2 - 10.
- -
c. 21 x + y2 + x + y d. - 1 # x
b. - 5n + 3.
19. Multiplica cada caso y combina términos donde sea posible: c. 1 - 22n - 1.
a. - 21x - y2 b. x1x - y2 28. Halla los dos primeros términos de una sucesión arit-
c. - x1x - y2 mética en donde el cuarto término es - 8 y el término
d. - 21x + y - z2 101 es - 493.
20. Halla todos los enteros x (de ser posible) que hagan ver- 29. Tita notó que cada 30 segundos la temperatura de una
dadero cada uno de los casos siguientes: reacción química en su laboratorio estaba decreciendo
a. - 3x = 6 b. - 3x = - 6 en el mismo número de grados. Inicialmente la tempe-
c. - 2x = 0 d. 5x = - 30 ratura era de 28°C y 5 minutos después de - 12°C. En
e. x , 3 = 12 - f. x , 1 - 32 = - 2 un segundo experimento, Tita notó que la temperatura
g. x , 1 x2 = 1
- -
de la reacción química era inicialmente de - 57°C e iba
h. 0 , x = 0 decreciendo 3°C cada minuto. Si ella comenzó los dos
21. Despeja x en cada uno de los casos siguientes: experimentos al mismo tiempo, ¿cuándo fueron iguales
a. - 3x - 8 = 7 las temperaturas de las reacciones? ¿Cuál fue esa tempe-
b. - 215x - 32 = 26 ratura?
c. 3x - x - 2x = 3 30. Halla todos los valores enteros (si los hay) de x y y para
d. - 215x - 62 - 30 = - x los cuales lo siguiente sea cierto.
e. x2 = 4 f. 1x - 122 = 9 a. xy = - ƒ x ƒ ƒ y ƒ
g. 1x - 12 = 1x + 322
2
b. - x2 = x2
h. 1x - 121x + 32 = 0 c. x2 7 y2.
22. Usa la fórmula de diferencia de cuadrados para simplifi-
car cada caso, de ser posible:
a. 52 # 48
b. 15 - 100215 + 1002
c. 1 - x - y21 - x + y2

© Lopez Mateos Editores. ISBN 978-607-95583-2-1, obra completa, versión electrónica, ISBN 978-607-95583-3-8, volumen 1, versión electrónica. Ejemplar asignado a: Helecto Villarroel gutierrez -
helecto@gmail.com. Fecha: 27 de octubre de 2014. Prohibida su modificación, copia o distribución.
282 Enteros y teoría de números

Evaluación 5-2B

1. Usa patrones para mostrar que 1 - 221 - 22 = 4. c. La temperatura a las 8:00 p.m. es de 20°C. Si cada mi-
2. Usa el modelo del campo de cargas para mostrar que nuto se incrementó en d grados, ¿cuál era la tempera-
1 - 221 - 22 = 4. tura m minutos antes?
3. Usa el modelo de la recta numérica para mostrar que 11. a. En cada una de cuatro jugadas consecutivas en un juego
21 - 22 = - 4. de futbol americano, un equipo perdió 11 yd. Si las yar-
4. En cada uno de los siguientes modelos de campos de das perdidas se interpretan como un entero negativo,
carga se quitan las cargas encerradas con una línea. Es- escribe la información como un producto de enteros y
cribe el problema correspondiente de multiplicación de determina el número total de yardas perdidas.
enteros cuya solución está basada en el modelo. b. Si Gerardo González perdió un total de 66 yardas
a. b. en 11 jugadas, ¿cuántas yardas, en promedio, perdió
en cada jugada?
11 11 12. Muestra que la propiedad distributiva de la multiplica-
111111 22 22 ción sobre la suma, a1b + c2 = ab + ac, es verdadera
222 222 11 11 para cada uno de los valores de a, b y c:
22 22 a. a = - 5, b = 2, c = - 6
b. a = - 2, b = - 3, c = 4
13. Calcula los casos siguientes:
5. Usa la definición de división para hallar cada cociente, de a. 10 - 3 - 12 b. 10 - 13 - 122
ser posible. Si un cociente no está definido, explica por qué. c. 1 - 322 d. - 32
a. 143 , 1 - 112 b. 0 , 1 - 52 - 2
e. 5 + 31 22 - 2
f. - 23
c. 0 , 0 g. 1 22
- 5
h. - 24
6. Evalúa cada caso, de ser posible: 14. Calcula lo siguiente sin usar una calculadora:
a. 1a , b2b b. 1ab2 , b a. - 263 + 264
c. 1 - 8 + 82 , 8 d. 1 - 23 - - 72 , 4 b. - ƒ - 6 ƒ - 82 + 1 - 1249 # 48 , 1 - 42 # 3 + 1 - 523
7. Evalúa cada uno de los siguientes productos y después, de ser 15. Si x es un entero y x Z 0, ¿cuáles de las siguientes expresio-
posible, escribe dos afirmaciones de división que sean equi- nes son siempre positivas y cuáles son siempre negativas?
valentes a las afirmaciones sobre multiplicación. Si no es po- a. - x4 b. 1 - x24 c. x4
sible construir dos afirmaciones de división, explica por qué. d. x e. - x
a. 1 - 524 16. ¿Cuáles de las expresiones del problema 15 son iguales
b. 1 - 421 - 32 entre sí para todos los valores de x?
8. Supón que a y b son enteros y que a , b es un entero. 17. Identifica la propiedad de los enteros ilustrada en cada
a. Usa la definición de división de enteros para demostrar
que si c Z 0, entonces 1ac2 , 1bc2 = a , b.
caso:
a. 1 - 22132 僆 E
b. ¿Por qué la afirmación en (a) no es verdadera si c = 0? ¿Es b. 1 - 420 = 0
c. - 213 + 42 = - 2132 + 1 - 224
verdadera si a , b = 0 y c Z 0? Justifica tus respuestas.
d. 1 - 223 = 31 - 22
9. En los casos siguientes, x y y son enteros. Usa la defini-
ción de división en términos de multiplicación para efec-
18. Simplifica cada caso:
tuar las operaciones indicadas. Escribe tus respuestas en
a. x - 21 - y2
b. a - 1a - b21 - 12
la forma más sencilla.
a. 1 - 4x2 , x
c. y - 1 y - x21 - 22
b. 1 - 10x + 52 , 5
d. - 11x - y2 + x
10. En un laboratorio, la temperatura de varias reacciones
19. Multiplica cada caso y combina términos donde sea posible:
químicas cambiaba en un número fijo de grados por mi-
a. - x1x - y - 32
b. 1 - 5 - x215 + x2
nuto. Plantea un problema de multiplicación que des-
c. 1x - y - 121x + y + 12
criba cada uno de los casos siguientes:
a. La temperatura a las 8:00 a.m. es de - 5°C. Si se in-
crementa en d grados por minuto, ¿cuál será la tem- d. 1 - x2 + 221x2 - 12
peratura m minutos después? 20. Halla todos los enteros x (de ser posible) que hagan ver-
b. La temperatura a las 8:00 p.m. es de 0°C. Si dismi- dadero cada uno de los casos siguientes:
nuye d grados por minuto, ¿cuál será la temperatura a. x , 0 = 1
m minutos antes?

© Lopez Mateos Editores. ISBN 978-607-95583-2-1, obra completa, versión electrónica, ISBN 978-607-95583-3-8, volumen 1, versión electrónica. Ejemplar asignado a: Helecto Villarroel gutierrez -
helecto@gmail.com. Fecha: 27 de octubre de 2014. Prohibida su modificación, copia o distribución.
Sección 5-2 Multiplicación y división de enteros 283

b. x2 = 9 JULIO
c. x2 = - 9 D L M M J V S
d. - x , -x = 1
1 2 3 4 5
e. - x2 es negativo.
6 7 8 9 10 11 12
f. - 11 - x2 = x - 1
13 14 15 16 17 18 19
g. x - 3x = - 2x
h. - 31x + 22 = - 3x + 6 20 21 22 23 24 25 26
21. Despeja x o halla los valores de la expresión indicada: 27 28 29 30 31
a. 12x - 122 = 11 - 2x22
夝 b. Demuestra que la suma de los nueve dígitos de cual-
b. x3 = - 29 quier conjunto de 3 * 3 números seleccionados de
c. - 6x 7 - x + 20 un calendario mensual siempre es igual a 9 por el nú-
d. - 51x - 32 7 - 5 mero del centro.
e. Si x 7 - 2, halla los valores de 3 - 5x. 26. En cada uno de los casos a continuación halla los dos térmi-
f. Si x 6 0, halla los valores de 2 - 7x. nos siguientes. Supón que la sucesión es aritmética o geo-
22. Usa la fórmula de diferencia de cuadrados para simplifi- métrica y halla su diferencia o razón, y el término n-ésimo.
car cada caso, de ser posible: a. 10, 7, 4, 1, - 2, - 5, _, _
a. 12 + 3x212 - 3x2 b. - 2, 4, - 8, 16, - 32, 64, _, _
b. 1x - 1211 + x2 27. Halla la suma de los primeros 100 términos de la sucesión
c. 2132 - 132 aritmética 10, 7, 4, 1, - 2, - 5, Á .
23. Factoriza completamente cada una de las expresiones si- 28. Halla los primeros cinco términos de las sucesiones cuyo
guientes y después simplifica, de ser posible: término n-ésimo es
a. ax + 2x b. ax - 2x a. 1 - 22n + 2n
c. 3x - 4x + 7x d. 3x2 + xy - x b. n21 - 12n
e. 1a + b21c + 12 - 1a + b2 c. ƒ 10 - n2 ƒ
f. x2 - 9y2 29. En la sucesión geométrica 1, - 2, 4, - 8, Á , determina si
g. 1x2 - y22 + x + y existe un término igual a los números siguientes:
24. Si x y y son enteros, clasifica cada uno de los casos si- a. 512 b. 1024
guientes como verdadero o falso. Si es verdadero, explica 30. Si x y y son enteros, clasifica cada caso como siempre
por qué. Si es falso, exhibe un contraejemplo. verdadero, verdadero a veces o nunca verdadero. Justi-
a. ƒ x + y ƒ = ƒ x ƒ + ƒ y ƒ fica tus respuestas.
b. ƒ xy ƒ = ƒ x ƒ ƒ y ƒ a. 1 - x23 = - x3 b. ƒ x ƒ 7 - 1
c. ƒ x2 ƒ = x2 c. Si x 6 y entonces a - x 6 a - y, para todos los en-
d. ƒ x ƒ 2 = x2 teros a.
25. a. Dado un calendario de cualquier mes del año, como el 31. José tiene dos cuentas de cheques. En la primera está sobre-
que sigue, escoge varios grupos de números de 3 * 3 y girado por $120, y en la segunda su saldo es de $300. Si de-
halla la suma de estos números. ¿Cómo se relacionan posita $40 cada día en la primera cuenta pero retira $20
las sumas obtenidas con el número del centro? diariamente de la segunda cuenta, ¿después de cuántos días
el saldo será igual en cada cuenta? Explica tu solución.

Conexiones matemáticas 5-2

Comunicación 3. a. Usa la propiedad distributiva de la multiplicación so-


1. ¿Se puede multiplicar 1 - x - y21x + y2 usando la fór- bre la suma para mostrar que 1 - 12a + a = 0. (Suge-
mula de diferencia de cuadrados? Explica por qué sí o rencia: Escribe a como 1 # a.)
por qué no. b. Usa la parte (a) para mostrar que 1 - 12a = - a.
2. Carla dice que usando la ecuación 1a + b22 = a2 + 4. Nina dio el siguiente argumento para mostrar que
2ab + b2, puede hallar una ecuación similar para 1 - a2b = - 1ab2, para todos los enteros a y b: Yo sé que
1a - b22. Examina su argumento. Si es correcto, propor- 1 - 12a = - a; por lo tanto:
ciona los pasos o razonamientos faltantes. Si es inco- 1 - a2b = 31 - 12a4b
rrecto, señala por qué. = 1 - 121ab2
- 1ab2
1a - b22 = 3a + 1 - b242 =
= a2 + 2a1 - b2 + 1 - b22 Si el argumento es válido, completa los detalles; si no es
= a2 - 2ab + b2 válido, explica por qué.

© Lopez Mateos Editores. ISBN 978-607-95583-2-1, obra completa, versión electrónica, ISBN 978-607-95583-3-8, volumen 1, versión electrónica. Ejemplar asignado a: Helecto Villarroel gutierrez -
helecto@gmail.com. Fecha: 27 de octubre de 2014. Prohibida su modificación, copia o distribución.
284 Enteros y teoría de números

5. Horacio dio el argumento de que - 1a + b2 = - a + - b, 13. a. ¿Cómo expondrían la multiplicación de enteros en


para todos los enteros a y b. Si el argumento es correcto, una escuela de educación media y cómo explicarían
proporciona el razonamiento faltante. Si es incorrecto, ex- que el producto de dos números negativos es posi-
plica por qué. tivo? Escriban una justificación de su enfoque.
- 1a + b2 = 1- 121a + b2 b. Presenten sus respuestas y compárenlas con las de
= 1 -12a + 1 - 12b otro grupo; juntos decidan acerca de la manera más
= -a + -b apropiada de exponer los conceptos. Escriban una
6. El matemático suizo Leonhard Euler (1707–1783) de- justificación de su enfoque.
mostró que 1 - 121 - 12 = 1 argumentando como sigue: 14. Discutan en su grupo acerca del enfoque favorito de cada
“El resultado debe ser - 1 ó 1. Si es - 1, entonces persona para justificar el hecho de que 1 - 121 - 12 = 1 y
1 - 121 - 12 = - 1. Como - 1 = 1 - 121, tenemos que por qué es el favorito.
1 - 121 - 12 = 1 - 121. Ahora, dividiendo ambos lados de Preguntas del salón de clase
la última ecuación entre - 1 tenemos que - 1 = 1, lo 15. Un estudiante de séptimo grado no cree que - 5 6 - 2.
cual, por supuesto, no puede ser cierto. Por lo tanto, El estudiante argumenta que una deuda de $5 es mayor
1 - 121 - 12 debe ser igual a 1”. que una deuda de $2. ¿Cómo le respondes?
a. ¿Cuál es tu reacción ante este argumento? ¿Es ló- 16. Un estudiante calcula - 8 - 21 - 32 escribiendo - 101 - 32
gico? ¿Por qué sí o por qué no? = 30. ¿Cómo puedes ayudar a este estudiante?
b. ¿Se puede usar el enfoque de Euler para justificar 17. Un estudiante dice que su papá le enseñó un método muy
otras propiedades de los enteros? Explica. sencillo para trabajar con expresiones como - 1a - b + 12
7. Si 5x + 3 6 - 20, responde a lo siguiente: y x - 12x - 32. La regla es que si hay un signo negativo
a. Halla el mayor entero x para el cual la desigualdad es antes del paréntesis, cambian los signos de la expresión den-
verdadera. Explica tu razonamiento. tro del paréntesis. Así, - 1a - b + 12 = - a + b - 1 y
b. ¿Existe un menor entero x para el cual la desigualdad x - 12x - 32 = x - 2x + 3. ¿Cuál es tu respuesta?
es verdadera? Explica por qué sí o por qué no. 18. Ale dice que 41 - 22 y - 41 - 22 no pueden ser iguales a 8.
8. Juanita pide a sus compañeras de clase que escojan un Dice que él sabe que 41 - 22 = - 8 pues 41 - 22 = - 2 +
número, después que multipliquen el número por - 3, le - 2 + - 2 + - 2 = - 8. Por lo tanto, - 41 - 22 debe ser 8.
sumen 2 al producto, multipliquen el resultado por - 2 y ¿Qué le dices a Ale?
después le resten 14. Finalmente, pide a cada estudiante 19. Beti usó el modelo de campos de carga para mostrar que
que divida el resultado entre 6 y registre la respuesta. - 21 - 32 = 6. Dice que esto demuestra que cualquier
Cuando Juanita obtiene una respuesta de una de sus número negativo multiplicado por cualquier número ne-
compañeras, ella le suma 3 mentalmente y anuncia el nú- gativo es un entero positivo. ¿Cómo le respondes?
mero que cada compañera eligió originalmente. ¿Cómo
supo Juanita que debía sumar 3 a cada respuesta? Problemas de repaso
Solución abierta 20. Encuentra - 8 + - 5 usando una recta numérica.
21. Halla el inverso aditivo de cada uno de los siguientes casos:
9. Construye un problema similar al problema 8 pero con
a. - 5 b. 7 c. 0
todas las cifras diferentes, y resuélvelo.
22. Calcula cada caso:
10. En una competencia nacional de matemáticas, el resul-
a. ƒ - 14 ƒ
tado se obtiene usando la fórmula 4 por el número de
respuestas correctas menos el número de respuestas in- b. ƒ - 14 ƒ + 7
correctas. En este esquema, los problemas dejados en c. 8 - ƒ - 12 ƒ
blanco no se consideran correctos ni incorrectos. Des- d. ƒ 11 ƒ + ƒ - 11 ƒ
cribe un escenario que permita que un estudiante ob- 23. En los años 1400, los mercaderes europeos usaban nú-
tenga un resultado negativo. meros positivos y negativos para marcar barriles de ha-
11. Selecciona un libro de texto de educación media que rina. Por ejemplo, un barril marcado +3 significaba que
presente la multiplicación y la división de enteros; ana- tenía 3 lb de sobrepeso, mientras que si estaba marcado
liza cualquiera de los modelos usados y expresa lo que con - 5 significaba que tenía 5 lb por debajo del peso. Si
piensas acerca de su efectividad para usarlo con un grupo se hallaron los números siguientes en barriles de 100 lb,
de estudiantes. ¿cuál era el peso total de los barriles?
Aprendizaje colectivo
12. Diseñen un esquema para determinar el promedio de un
estudiante al que se pueden asignar puntos negativos HARINA HARINA HARINA HARINA

como calificación reprobatoria. +4 –3 +5 –6


a. Usen su esquema para determinar posibles califica-
ciones para estudiantes con promedios positivo, cero
24. Escribe la función f 1x2 = 1x + ƒ x ƒ 2 , 2 sin los símbolos
y negativo.
b. Comparen su esquema con el de otro grupo y escriban
de valor absoluto. (Distingue los dos casos: x Ú 0 y x 6 0.)
una justificación del mejor esquema.
© Lopez Mateos Editores. ISBN 978-607-95583-2-1, obra completa, versión electrónica, ISBN 978-607-95583-3-8, volumen 1, versión electrónica. Ejemplar asignado a: Helecto Villarroel gutierrez -
helecto@gmail.com. Fecha: 27 de octubre de 2014. Prohibida su modificación, copia o distribución.
Sección 5-3 Divisibilidad 285

25. Despeja x de ser posible: Si x = - 3, ¿cuál es el valor de - 3x?


a. ƒ x ƒ = 3 b. ƒ x ƒ + 1 = 0 a. - 9
c. ƒ x ƒ = x d. ƒ x ƒ = - x b. - 6
c. - 1
Preguntas del Third International Mathematics and
d. 1
Science Study (TIMSS) (Tercera Reunión Internacional
e. 9
sobre el Estudio de las Matemáticas y la Ciencia)
TIMSS, Grado 8, 2003
Si n es un entero negativo, ¿cuál de estos números es el mayor?
a. 3 + n b. 3 * n c. 3 - n d. 3 , n

ROMPECABEZAS Si a, Á , z son letras consecutivas del alfabeto que representan enteros, halla el producto:
1x - a21x - b21x - c2 # Á # 1x - z2

5-3 Divisibilidad

Los conceptos de enteros par e impar son de uso común. Por ejemplo, durante los periodos
de escasez de agua en el verano, en muchos lugares las casas con número par pueden tener
agua los días con número par y las casas con número impar pueden tener agua los días con
número impar. Un entero par es aquel que es divisible entre 2; esto es, un entero que tiene
residuo 0 cuando se divide entre 2. Un entero impar es un entero que no es divisible entre 2.
El hecho de que 12 sea divisible entre 2 se puede enunciar mediante las siguientes afirma-
ciones equivalentes, en la columna de la izquierda:
Ejemplo Afirmación general
12 es divisible entre 2. a es divisible entre b.
2 es un divisor de 12. b es un divisor de a.
12 es un múltiplo de 2. a es un múltiplo de b.
2 es un factor de 12. b es un factor de a.
2 divide a 12. b divide a a.

La afirmación de que “2 divide a 12” se escribe con un segmento vertical, como en 2 ƒ 12,
donde el segmento vertical significa divide a. Asimismo, “b divide a a” se puede escribir
como b ƒ a. Cada afirmación en la columna de la derecha se puede escribir como b ƒ a. Escribi-
mos 5ⱈ12 para simbolizar que 5 no divide a 12 o que 12 no es divisible entre 5. La notación
5ⱈ12 también implica que 12 no es un múltiplo de 5 y que 5 no es un factor de 12.
En general, si a es un entero no negativo y b es un entero positivo, decimos que a es divi-
sible entre b o, de manera equivalente, que b divide a a si, y sólo si, el residuo cuando a se
divide entre b es 0. Usando el algoritmo de la división, esto significa que existe un único c
(cociente) tal que a = bc. En la siguiente definición extendemos este concepto de divisibili-
dad para todos los enteros.

Definición de “Divide”
Si a y b son enteros cualesquiera, entonces b divide a a, que se escribe b ƒ a, si, y sólo si, existe un
entero único c tal que a = bc.

Si b ƒ a, entonces b es un factor, o un divisor, de a, y a es un múltiplo de b.


© Lopez Mateos Editores. ISBN 978-607-95583-2-1, obra completa, versión electrónica, ISBN 978-607-95583-3-8, volumen 1, versión electrónica. Ejemplar asignado a: Helecto Villarroel gutierrez -
helecto@gmail.com. Fecha: 27 de octubre de 2014. Prohibida su modificación, copia o distribución.
286 Enteros y teoría de números

O B S E R VA C I Ó N En este capítulo, cuando se use el símbolo de “divide a”, como en


b ƒ a, suponemos que a y b son enteros y que b Z 0.

No confundas b ƒ a con b>a, que se interpreta como b , a. La primera expresión, que es una
relación, puede ser verdadera o falsa. La expresión posterior, una operación, tiene un valor
numérico si a Z 0. Nota que si b>a es un entero, entonces a ƒ b. Nota, además, que para ente-
ros positivos aⱈb es equivalente a decir que el residuo cuando b se divide entre a no es 0.

Ejemplo 5-16 Clasifica cada uno de los casos siguientes como verdadero o falso. Explica tus respuestas.








a. - 3 ƒ 12 b. 0 ƒ 2 c. 0 es par.
d. 8ⱈ2 e. Para todos los enteros a, 1 ƒ a . f. Para todos los enteros a, - 1 ƒ a.
g. 3 ƒ 6n para todos los enteros n. h. 1a - b2 ƒ 1a2 - b 22 Si a y b son enteros y a Z b.
i. 0 ƒ 0

Solución a. - 3 ƒ 12 es verdadero porque 12 = - 41 - 32.


b. 0 ƒ 2 es falso porque no existe entero c tal que 2 = c # 0.
c. 2 ƒ 0 es verdadero porque 0 = 0 # 2; por lo tanto, 0 es par.
d. 8ⱈ2 es verdadero porque no existe entero c tal que 2 = c # 8.
e. 1 ƒ a es verdadero para todos los enteros a pues a = a # 1.
f. - 1 ƒ a es verdadero para todos los enteros a pues a = 1 - a21 - 12.
g. 3 ƒ 6n es verdadero. Como 6n = 3 # 2n, 6n es un múltiplo de 3 y, por lo tanto, 3 ƒ 6n.
h. 1a - b2 ƒ 1a2 - b 22 es verdadero pues a2 - b 2 = 1a - b21a + b2 y a Z b.
i. 0 ƒ 0 es falso pues 0 = 0 # q para todos los enteros q, de modo que q no es único.

Supón que tenemos un paquete de goma de mascar y sabemos que el número de piezas,
a, es divisible entre 5. Entonces, si tenemos dos paquetes el número total de piezas de goma
de mascar sigue siendo divisible entre 5. Lo mismo es cierto si tenemos 7 paquetes o 20 pa-
quetes, o en general n paquetes donde n es cualquier entero positivo. Podemos registrar la
observación anterior como:
Si 5 ƒ a, entonces 5 ƒ na, donde a y n son enteros y n 7 0.
De manera más general, si d ƒ a entonces d divide a cualquier múltiplo de a. Enunciamos este
hecho en el siguiente:

Teorema 5–12
Para cualesquier enteros a y d, si d ƒ a y n es cualquier entero, entonces d ƒ na.

◆ Nota Pierre de Fermat (1601–1665) fue un abogado y magistrado que sirvió en el parlamento
histórica provincial de Toulouse, Francia. Dedicaba su tiempo libre a las matemáticas —tema en el
cual no tenía una educación formal. Después de su muerte, su hijo decidió publicar una
nueva edición de la Arithmetica de Diofanto con las notas de Fermat. Una de las notas en el
margen del ejemplar de Fermat aseguraba que la ecuación x n + yn = zn no tenía soluciones
enteras positivas si n era un entero mayor que 2 y comentaba, “He hallado una demostración
admirable de esto, pero el margen es muy estrecho para contenerla”. Muchos grandes
matemáticos dedicaron años a tratar de probar la afirmación de Fermat, ahora llamada “el
último teorema de Fermat”. En 1995 Andrew Wiles, un matemático de la Universidad de
Princeton, demostró el último teorema de Fermat. ◆
© Lopez Mateos Editores. ISBN 978-607-95583-2-1, obra completa, versión electrónica, ISBN 978-607-95583-3-8, volumen 1, versión electrónica. Ejemplar asignado a: Helecto Villarroel gutierrez -
helecto@gmail.com. Fecha: 27 de octubre de 2014. Prohibida su modificación, copia o distribución.
Sección 5-3 Divisibilidad 287

El teorema se puede enunciar en una forma equivalente:


Si d es un factor de a (esto es, si a es igual a algún entero multiplicado por d), entonces d es un fac-
Super Bomba tor de cualquier múltiplo de a.
A continuación considera dos paquetes de goma de mascar, cada uno con cinco piezas,
como en la figura 5-16. Podemos dividir equitativamente cada paquete de este producto
MascaMasca
entre cinco estudiantes. Además, si abrimos ambos paquetes y colocamos todas las pie-
zas en una bolsa, todavía podríamos dividir equitativamente las piezas de chicle entre los
cinco estudiantes. Para generalizar esta idea, si compramos goma de mascar en paquetes
Figura 5-16
más grandes con a piezas en un paquete y b piezas en un segundo paquete, y con a y b di-
visibles entre 5, podemos registrar el análisis anterior como:
Si 5 ƒ a y 5 ƒ b, entonces 5 ƒ 1a + b2.
Si el número, a, de piezas de goma de mascar en un paquete es divisible entre 5, pero el
número b de piezas en otro paquete no lo es, entonces el total, a + b, no puede dividirse de
manera equitativa entre cinco estudiantes. Esto se puede registrar como:
Si 5 ƒ a y 5ⱈb, entonces 5ⱈ1a + b2.

AHORA INTENTA ÉSTE 5-8 Si a, b 僆 E, ¿cuál es el error, si lo hay, en la afirmación: si 5ⱈa y 5ⱈb,
entonces 5ⱈ1a + b2?

Como la resta se puede definir en términos de la suma, resultados similares para la suma
son válidos para la resta. Estas ideas se generalizan en el teorema 5–13.

Teorema 5–13
Para cualesquier enteros a, b y d, d Z 0,

a. Si d ƒ a y d ƒ b, entonces d ƒ 1a + b2.
b. Si d ƒ a y dⱈb, entonces dⱈ1a + b2.
c. Si d ƒ a y d ƒ b, entonces d ƒ (a - b).
d. Si d ƒ a y dⱈb, entonces dⱈ1a - b2.

O B S E R VA C I Ó N El teorema 5–13 se puede extender. Por ejemplo, si a, b, c y d son en-


teros, con d Z 0, entonces,
Si d ƒ a, d ƒ b y d ƒ c, entonces d ƒ 1a + b + c2.

Las demostraciones de la mayoría de los teoremas en esta sección se dejan como ejerci-
cios, pero damos la demostración del teorema 5–13(a) como ilustración.

Demostración
El teorema 5–13(a) equivale a lo siguiente:
Si a es un múltiplo de d y b es un múltiplo de d, entonces a + b es un múltiplo de d.
Nota que “a es un múltiplo de d” significa que a = md, para algún entero m. Análogamente,
“b es un múltiplo de d” significa b = nd, para algún entero n. Para mostrar que a + b es un
múltiplo de d, sumamos estas ecuaciones como sigue:
a + b = md + nd
© Lopez Mateos Editores. ISBN 978-607-95583-2-1, obra completa, versión electrónica, ISBN 978-607-95583-3-8, volumen 1, versión electrónica. Ejemplar asignado a: Helecto Villarroel gutierrez -
helecto@gmail.com. Fecha: 27 de octubre de 2014. Prohibida su modificación, copia o distribución.
288 Enteros y teoría de números

¿Es md + nd un múltiplo de d ? Nota que md + nd = 1m + n2d, de modo que a + b =


1m + n2d. Por la propiedad de la cerradura de la suma de enteros, m + n es un entero.
Luego, a + b es un múltiplo de d y, por lo tanto, d ƒ 1a + b2.

Ejemplo 5-17 Clasifica cada uno de los casos siguientes como verdadero o falso, donde x, y y z son enteros. Si









una afirmación es verdadera, demuéstrala. Si una afirmación es falsa, exhibe un contraejemplo.
a. Si 3 ƒ x y 3 ƒ y, entonces 3 ƒ xy. b. Si 3 ƒ 1x + y2, entonces 3 ƒ x y 3 ƒ y.
c. Si 9ⱈa, entonces 3ⱈa.

Solución a. Verdadera. Por el teorema 5–12, si 3 ƒ x entonces, para cualquier entero y, 3 ƒ yx ó 3 ƒ xy.
b. Falsa; por ejemplo, 3 ƒ 17 + 22, pero 3ⱈ7 y 3ⱈ2.
c. Falsa; por ejemplo, 9ⱈ21, pero 3 ƒ 21.

AHORA INTENTA ÉSTE 5-9 Si x, y 僆 E, y si 3 ƒ x, ¿es cierto que 3 ƒ xy independientemente de que 3 ƒ y ó


3ⱈy? ¿Por qué?

Ejemplo 5-18 Cinco estudiantes hallaron una caja de dinero cerrada con una ficha de depósito pegada. La








ficha estaba mojada, de manera que la cantidad se veía como aparece en la figura 5-17. Un es-
tudiante observó que si el dinero listado en la ficha de depósito estaba en la caja, sería fácil di-
vidirlo equitativamente entre los cinco estudiantes sin usar monedas. ¿Cómo supo esto el
$ 0.00
estudiante?
Solución  Como el dígito de las unidades de la cantidad de dinero es 0, la solución al pro-
blema es determinar si todos los números naturales cuyo dígito de las unidades es 0 son divi-
Figura 5-17 sibles entre 5. Para resolver este problema, busca un patrón. Los números naturales cuyo
dígito de las unidades es 0 forman un patrón, esto es, 10, 20, 30, 40, 50, Á . Estos números
son múltiplos de 10. Ahora queremos determinar si 5 divide a todos los múltiplos de 10.
Como 5 ƒ 10, por el teorema 5–12, 5 divide a cualquier múltiplo de 10. Por lo tanto, 5 divide
a la cantidad de dinero en la caja, y el estudiante tiene razón.

Criterios de divisibilidad
Como se mostró en el ejemplo 5-18, a veces es útil saber si un número es divisible entre
otro sólo con verlo o mediante una prueba sencilla. Descubrimos que si un número termina
en 0, entonces el número es divisible entre 5. El mismo argumento se puede usar para mos-
trar que si un número termina en 5, es divisible entre 5. Éste es un ejemplo de una regla de
divisibilidad. Más aún, si el último dígito de un número no es 0 ó 5, entonces el número no
es divisible entre 5.
Los libros de texto elementales frecuentemente enuncian reglas de divisibilidad, mismas
que tienen uso limitado, excepto para aritmética mental. Es posible determinar si 1734 es
divisible entre 17 ya sea usando lápiz y papel o una calculadora. Para verificar la divisibili-
dad y evitar los decimales, podemos usar una calculadora con tecla de división entera,
INT , . En dicha calculadora se puede efectuar la división entera usando la siguiente suce-
sión de teclas:
1 7 3 4 INT , 1 7 =

para obtener en pantalla 102 0.


Q R

© Lopez Mateos Editores. ISBN 978-607-95583-2-1, obra completa, versión electrónica, ISBN 978-607-95583-3-8, volumen 1, versión electrónica. Ejemplar asignado a: Helecto Villarroel gutierrez -
helecto@gmail.com. Fecha: 27 de octubre de 2014. Prohibida su modificación, copia o distribución.
Sección 5-3 Divisibilidad 289

Esto implica que 1734>17 = 102 con residuo 0, lo cual, a su vez, implica que 17 ƒ 1734.
Podríamos haber determinado el mismo resultado mentalmente al considerar lo siguiente:
1734 = 1700 + 34
Como 17 ƒ 1700 y 17 ƒ 34, por el teorema 5–13(a), tenemos 17 ƒ 11700 + 342, ó 17 ƒ 1734. De
manera análoga, podríamos determinar mentalmente que 17ⱈ1735.

O B S E R VA C I Ó N Nota que 17 ƒ 1734 implica que 17 ƒ 1 - 121734, esto es, 17 ƒ - 1734. En


general, d ƒ - a si, y sólo si, d ƒ a.

Criterios de divisibilidad para 2, 5 y 10


Para determinar mentalmente si un entero dado n es divisible entre otro entero d, pensa-
mos n como suma o diferencia de enteros, donde d divide a al menos uno de estos números.
Usamos un ejemplo para tener idea de cómo funciona este concepto. Considera el número
1362. Este número se puede representar como en la figura 5-18. Nota que 2 divide a cada
parte de la figura (ve la línea punteada). Como 2 divide a cada parte de la figura, por la ex-
tensión del teorema 5–13, 2 divide a la suma de todas las partes, y por lo tanto 2 ƒ 1362.

1362

Figura 5-18
© Lopez Mateos Editores. ISBN 978-607-95583-2-1, obra completa, versión electrónica, ISBN 978-607-95583-3-8, volumen 1, versión electrónica. Ejemplar asignado a: Helecto Villarroel gutierrez -
helecto@gmail.com. Fecha: 27 de octubre de 2014. Prohibida su modificación, copia o distribución.
290 Enteros y teoría de números

Nota que si el número original era 1363, entonces 2 ƒ 1362 y 2ⱈ1; por lo tanto, 2ⱈ1363.
Vemos que todo lo que tenemos que hacer es determinar si el dígito de las unidades es di-
visible entre 2 para determinar si el número es divisible entre 2. Podemos desarrollar un
criterio similar para verificar la divisibilidad entre 5 y 10.

Teorema 5–14:    Criterio de divisibilidad para 2


Un entero es divisible entre 2 si, y sólo si, su dígito de las unidades es divisible entre 2.

Teorema 5–15:    Criterio de divisibilidad para 5


Un entero es divisible entre 5 si, y sólo si, su dígito de las unidades es divisible entre 5, esto es,
si, y sólo si, el dígito de las unidades es 0 ó 5.

Teorema 5–16:    Criterio de divisibilidad para 10


Un entero es divisible entre 10 si, y sólo si, su dígito de las unidades es divisible entre 10, esto
es, si, y sólo si, el dígito de las unidades es 0.

Criterios de divisibilidad para 4 y 8


Al considerar reglas de divisibilidad para 4 y 8 vemos que 4ⱈ10 y 8ⱈ10, de modo que no es
cuestión de verificar si el dígito de las unidades es divisible entre 4 y 8. Sin embargo, 4
1que es 222 divide a 102, y 8 1que es 232 divide a 103.
Primero desarrollamos una regla de divisibilidad para 4. Considera, por ejemplo, cual-
quier número n de cuatro dígitos, a, b, c y d, tal que n = a103 + b102 + c10 + d. Nuestro
objetivo parcial es escribir el número dado como suma de dos números, uno de los cuales debe ser
lo más grande posible y ser divisible entre 4. Sabemos que 4 ƒ 102 pues 102 = 4 # 25 y, en
consecuencia, 4 ƒ 103. Como 4 ƒ 102, entonces 4 ƒ b102; y como 4 ƒ 103, entonces 4 ƒ a103. Final-
mente, 4 ƒ a103 y 4 ƒ b102 implican que 4 ƒ 1a103 + b10 22. Ahora la divisibilidad de
a103 + b102 + c10 + d entre 4 depende de la divisibilidad de 1c10 + d2 entre 4. Nota que
c10 + d es el número representado por los últimos dos dígitos del número dado n. Resumi-
mos esto en el siguiente criterio.

Teorema 5–17:    Criterio de divisibilidad para 4


Un entero es divisible entre 4 si, y sólo si, los dos últimos dígitos del entero representan un
número divisible entre 4.

© Lopez Mateos Editores. ISBN 978-607-95583-2-1, obra completa, versión electrónica, ISBN 978-607-95583-3-8, volumen 1, versión electrónica. Ejemplar asignado a: Helecto Villarroel gutierrez -
helecto@gmail.com. Fecha: 27 de octubre de 2014. Prohibida su modificación, copia o distribución.
Sección 5-3 Divisibilidad 291

Para investigar la divisibilidad entre 8, notamos que la mínima potencia positiva de 10 di-
visible entre 8 es 103 como 103 = 8 # 125. En consecuencia, todas las potencias enteras de
10 mayores que 103 también son divisibles entre 8. Por lo tanto, lo siguiente es un criterio
de divisibilidad entre 8.

Teorema 5–18:    Criterio de divisibilidad para 8


Un entero es divisible entre 8 si, y sólo si, los últimos tres dígitos del entero representan un
número divisible entre 8.

Ejemplo 5-19 a. Determina si 97,128 es divisible entre 2, 4 y 8.










b. Determina si 83,026 es divisible entre 2, 4 y 8.

Solución a. 2 ƒ 97,128 porque 2 ƒ 8.


4 ƒ 97,128 porque 4 ƒ 28.
8 ƒ 97,128 porque 8 ƒ 128.
b. 2 ƒ 83,026 porque 2 ƒ 6.
4ⱈ83,026 porque 4ⱈ26.
8ⱈ83,026 porque 8ⱈ026.

Ejemplo 5-20 Usa los teoremas 5–12 y 5–13 para demostrar por qué funciona el criterio de divisibilidad para








8 en el ejemplo 5-19(a).
Solución Podemos escribir 97,128 como 97,000 + 128. Como 8 ƒ 1000, entonces 8 ƒ 97 # 1000
u 8 ƒ 97,000 (Teorema 5–12). A continuación necesitamos verificar que 8 ƒ 128. Es así, de modo
que 8 ƒ 197,000 + 1282 u 8 ƒ 97,128 (Teorema 5–13).

O B S E R VA C I Ó N En el ejemplo 5–19(a), hubiera bastado verificar que el número dado


es divisible entre 8 pues si 8 ƒ a, entonces 2 ƒ a y 4 ƒ a. ¿Por qué? Esta relación se muestra en
la figura 5-19.

E
Números divisibles
entre 2

Números divisibles
entre 4

Números divisibles
entre 8

Figura 5-19

Nota que si 8ⱈa, no podemos concluir que 4ⱈa ó 2ⱈa. ¿Por qué?

© Lopez Mateos Editores. ISBN 978-607-95583-2-1, obra completa, versión electrónica, ISBN 978-607-95583-3-8, volumen 1, versión electrónica. Ejemplar asignado a: Helecto Villarroel gutierrez -
helecto@gmail.com. Fecha: 27 de octubre de 2014. Prohibida su modificación, copia o distribución.
292 Enteros y teoría de números

Criterios de divisibilidad entre 3 y 9


A continuación consideramos un criterio de divisibilidad para 3. Ninguna potencia de 10 es
divisible entre 3, pero los números 9 y 99 y 999 y otros de este tipo son divisibles entre 3.
Por ejemplo, para determinar si 5721 es divisible entre 3, reescribimos el número usando
999, 99 y 9, como sigue:
5721 = 5 # 103 + 7 # 102 + 2 # 10 + 1
= 51999 + 12 + 7199 + 12 + 219 + 12 + 1
= 5 # 999 + 5 # 1 + 7 # 99 + 7 # 1 + 2 # 9 + 2 + 1
= 15 # 999 + 7 # 99 + 2 # 92 + 15 + 7 + 2 + 12

La suma en el primer conjunto entre paréntesis del último renglón es divisible entre 3, así que
la divisibilidad de 5721 entre 3 depende de la suma del segundo conjunto entre paréntesis. En
este caso, 5 + 7 + 2 + 1 = 15 y 3 ƒ 15, de modo que 3 ƒ 5721. Por lo tanto, para probar la di-
visibilidad de 5721 entre 3, probamos la divisibilidad de 5 + 7 + 2 + 1 entre 3. Nota que
5 + 7 + 2 + 1 es la suma de los dígitos de 5721. El ejemplo sugiere el siguiente criterio de
divisibilidad entre 3.

Teorema 5–19:    Criterio de divisibilidad entre 3


Un entero es divisible entre 3 si, y sólo si, la suma de sus dígitos es divisible entre 3.

Podemos usar un argumento similar al que usamos para demostrar que 3 ƒ 5721 para pro-
bar el criterio de divisibilidad entre 3 para cualquier entero, y en particular para cualquier
número de cuatro dígitos n = a103 + b102 + c10 + d. Aunque a103 + b102 + c10 + d
no sea necesariamente divisible entre 3, el número a999 + b99 + c9 es divisible entre 3.
Tenemos lo siguiente:

a103 + b102 + c10 + d = a1000 + b100 + c10 + d


= a1999 + 12 + b199 + 12 + c19 + 12 + d
= 1a999 + b99 + c92 + 1a1 + b1 + c1 + d2
= 1a999 + b99 + c92 + 1a + b + c + d2

Como 3 ƒ 9, 3 ƒ 99 y 3 ƒ 999, se sigue que 3 ƒ 1a999 + b99 + c 92. Si 3 ƒ 1a + b + c + d2, enton-


ces 3 ƒ 31a999 + b99 + c92 + 1a + b + c + d24; esto es, 3 ƒ n. Por otro lado, si 3ⱈ( a + b +
c + d2, del teorema 5–13(b) tenemos que 3ⱈn.
Como 9 ƒ 9, 9 ƒ 99, 9 ƒ 999 y así sucesivamente, un criterio similar al de la divisibilidad en-
tre 3 se aplica a la divisibilidad entre 9. ¿Por qué?

Teorema 5–20:    Criterio de divisibilidad entre 9


Un entero es divisible entre 9 si, y sólo si, la suma de los dígitos del entero es divisible entre 9.

Ejemplo 5-21 Usa criterios de divisibilidad para determinar si cada uno de los números siguientes es di-








visible entre 3 y entre 9:


a. 1002 b. 14,238

Solución a. Como 1 + 0 + 0 + 2 = 3 y 3 ƒ 3, se sigue que 3 ƒ 1002. Como 9ⱈ3, se sigue


que 9ⱈ1002.
© Lopez Mateos Editores. ISBN 978-607-95583-2-1, obra completa, versión electrónica, ISBN 978-607-95583-3-8, volumen 1, versión electrónica. Ejemplar asignado a: Helecto Villarroel gutierrez -
helecto@gmail.com. Fecha: 27 de octubre de 2014. Prohibida su modificación, copia o distribución.
Sección 5-3 Divisibilidad 293

b. Como 1 + 4 + 2 + 3 + 8 = 18 y 3 ƒ 18, se sigue que 3 ƒ 14,238. Como 9 ƒ 18,


se sigue que 9 ƒ 14,238.

Ejemplo 5-22 El gerente de una tienda tiene una factura de pago de 72 calculadoras de cuatro funciones.








▼ El primero y el último dígitos de la factura son ilegibles. El gerente puede leer
$ . 67.9.
¿Cuáles son los dígitos faltantes y cuál es el costo de cada calculadora?

Solución Sean x y y los dígitos faltantes, de modo que el número es x67.9y dólares, o x679y
centavos. Como se vendieron 72 calculadoras, el número en la factura debe ser divisible en-
tre 72. Como el número es divisible entre 72 y 72 = 8 # 9, debe ser divisible entre 8 y entre
9, que son factores de 72. Para que el número en la factura sea divisible entre 8, el número
de tres dígitos 79y debe ser divisible entre 8. Como 79y debe ser divisible entre 8, es un nú-
mero par. Por lo tanto, 79y debe ser 790, 792, 794, 796 ó 798. Sólo el número 792 es divisi-
ble entre 8, de modo que ya sabemos que el último dígito de la factura, y, debe ser 2.
Como el número en la factura debe ser divisible entre 9, sabemos que 9 debe dividir a
x + 6 + 7 + 9 + 2, ó 1x + 242. Como 3 es el único dígito que hace que 1x + 242 sea di-
visible entre 9, se sigue que x debe ser 3. Por lo tanto, el número en la factura debe ser
$367.92. Las calculadoras deben costar $367.92>72, ó $5.11, cada una.
. ◆

Criterios de divisibilidad para 11 y 6


El criterio de divisibilidad para 7 es más difícil de usar que realizar la división, de modo que
lo omitimos. Enunciamos el criterio de divisibilidad para 11 pero omitimos la demostra-
ción. Los lectores interesados pueden intentar hallarla.

Teorema 5–21:    Criterio de divisibilidad para 11


Un entero es divisible entre 11 si, y sólo si, la suma de los dígitos en los lugares que son poten-
cias pares de 10 menos la suma de los dígitos en los lugares que son potencias impares de 10 es
divisible entre 11.

Por ejemplo, para verificar si 8,471,986 es divisible entre 11, verificamos si 11 divide a la
diferencia 16 + 9 + 7 + 82 - 18 + 1 + 42, ó 17. Como 11ⱈ17, se sigue, del criterio de
divisibilidad para 11, que 11ⱈ8,471,986. Un número como 2772 es divisible entre 11 pues
(2 + 7) - (7 + 2) = 9 - 9 = 0 y 0 es divisible entre 11.
El criterio de divisibilidad para 6 está relacionado con los criterios de divisibilidad para 2
y 3. En la sección 5-4 te pediremos demostrar que si 2 ƒ n y 3 ƒ n, entonces 12 # 32 ƒ n, y en ge-
neral: si a y c no tienen factor común, entonces si a ƒ b y c ƒ b, podemos concluir que ac ƒ b. En
consecuencia, el siguiente criterio de divisibilidad es verdadero.

Teorema 5–22:    Criterio de divisibilidad para 6


Un entero es divisible entre 6 si y sólo si el entero es divisible entre 2 y 3.

Se exploran más criterios de divisibilidad para otros números en las Evaluaciones 5-3A y 5-3B.

© Lopez Mateos Editores. ISBN 978-607-95583-2-1, obra completa, versión electrónica, ISBN 978-607-95583-3-8, volumen 1, versión electrónica. Ejemplar asignado a: Helecto Villarroel gutierrez -
helecto@gmail.com. Fecha: 27 de octubre de 2014. Prohibida su modificación, copia o distribución.
294 Enteros y teoría de números

Ejemplo 5-23 El número 57,729,364,583 tiene demasiados dígitos para caber en la mayoría de las panta-









llas de calculadora. Determina si es divisible entre cada uno de los siguientes números:
a. 2 b. 3 c. 5 d. 6 e. 8 f. 9 g. 10 h. 11

Solución a. No, el último dígito, 3, no es divisible entre 2.


b. No, la suma de los dígitos es 59, que no es divisible entre 3.
c. No, el último dígito no es 0 ni 5.
d. No, el número no es divisible entre 2.
e. No, pues el número formado por los últimos tres dígitos, 583, no es divisible entre 8.
f. No, pues la suma de los dígitos es 59, que no es divisible entre 9.
g. No, pues el dígito de las unidades no es 0.
h. Si, pues 13 + 5 + 6 + 9 + 7 + 52 - 18 + 4 + 3 + 2 + 72 =
35 - 24 = 11, y 11 es divisible entre 11.

AHORA INTENTA ÉSTE 5-10 Llena los espacios en blanco de modo que el número sea divisible
entre 9. Lista todas las posibilidades.

12,506,5_ _.

Resolver Problemas Un error en el inventario


Un grupo escolar visitó una empacadora. El gerente les informó que había 11,368 latas
de jugo en el inventario y que las latas estaban empacadas en cajas de 6 o de 24, depen-
diendo del tamaño de la lata. Uno de los estudiantes, Pepito, lo pensó por un momento y
dijo que había un error en el inventario. ¿Es correcta la afirmación de Pepito? ¿Por qué sí
o por qué no?

Comprender el problema El problema es determinar si el inventario de 11,368 latas era co-


rrecto. Para resolver el problema, debemos suponer que no hay cajas parcialmente llenas;
esto es, una caja debe contener exactamente 6 o exactamente 24 latas de jugo.

Trazar un plan Sabemos que las cajas contienen 6 ó 24 latas, pero no sabemos cuántas cajas
hay de cada tipo. Una estrategia para resolver este problema es plantear una ecuación que in-
cluya el número total de latas en todas las cajas.

◆ Nota Una matemática del siglo xx que trabajó en el área de teoría de números fue la estadouni-
histórica dense Julia Robinson (1919–1985). Robinson fue la primera mujer matemática, en Esta-
dos Unidos, elegida como miembro de la Academia Nacional de Ciencias y la primera
mujer presidenta de la Sociedad Matemática de Estados Unidos. Murió de leucemia a la
edad de 65 años. ◆

© Lopez Mateos Editores. ISBN 978-607-95583-2-1, obra completa, versión electrónica, ISBN 978-607-95583-3-8, volumen 1, versión electrónica. Ejemplar asignado a: Helecto Villarroel gutierrez -
helecto@gmail.com. Fecha: 27 de octubre de 2014. Prohibida su modificación, copia o distribución.
Sección 5-3 Divisibilidad 295

El número total de latas, 11,368, es igual al número de latas en todas las cajas de 6 la-
tas más el número de latas en todas las cajas de 24 latas. Si hay n cajas de 6 latas, en esas
cajas hay 6n latas. De manera análoga, si hay m cajas con 24 latas, esas cajas contienen
un total de 24m latas. Como se reportó un total de 11,368 latas, tenemos la ecuación
6n + 24m = 11,368. Pepito afirmó que 6n + 24m Z 11,368.
Una manera de mostrar que 6n + 24m Z 11,368 es mostrar que 6n + 24m y 11,368 no
tienen los mismos divisores. Tanto 6n como 24m son divisibles entre 6. Esto implica que 6n +
24m debe ser divisible entre 6. Si 11,368 no es divisible entre 6, Pepito está en lo correcto.
Realizar el plan El criterio de divisibilidad para 6 dice que un número es divisible entre 6 si,
y sólo si, el número es divisible entre 2 y entre 3. Como 11,368 termina en 8, es divisible
entre 2. ¿Es divisible entre 3?
El criterio de divisibilidad para 3 dice que un número es divisible entre 3 si, y sólo si, la
suma de los dígitos en el número es divisible entre 3. Vemos que 1 + 1 + 3 + 6 + 8 = 19,
el cual no es divisible entre 3, de modo que 11,368 no es divisible entre 3. Por lo tanto, Pe-
pito está en lo correcto.
Revisar Supón que 11,368 hubiera sido divisible entre 6. ¿Ello habría implicado que el ge-
rente estaba en lo correcto? La respuesta es no; sólo hubiera implicado que deberíamos
cambiar el enfoque del problema.
Como parte de la actividad de revisar, supón que, dando datos diferentes, el gerente estu-
viera en lo correcto. ¿Podemos determinar valores para m y n? De hecho, es posible hacer -
lo. Si disponemos de una computadora, podemos usar una hoja de cáculo para determinar
◆ todos los valores numéricos naturales para m y n.

ROMPECABEZAS  El siguiente es un argumento para mostrar que una hormiga pesa lo mismo que un
elefante. ¿Dónde está el error?
Sea e el peso del elefante y h el peso de la hormiga. Sea e - h = d. En consecuencia, e = h + d. Mul-
tiplica cada lado de e = h + d por e - h. Después simplifica.

e(e - h) = (h + d)(e - h)
e2 - eh = he + de - h2 - dh
e - eh - de = he - h2 - dh
2

e(e - h - d) = h(e - h - d)
e = h
Así, el peso del elefante es igual al peso de la hormiga.

Evaluación 5-3A

1. Clasifica cada caso como verdadero o falso. Si es falso, di 2. Usa los criterios de divisibilidad para responder lo siguiente:
por qué. a. Hay 1379 niños inscritos para jugar en la liga de beis-
a. 6 es un factor de 30. b. 6 es un divisor de 30. bol. Si se van a asignar exactamente 9 jugadores a
c. 6 ƒ 30. d. 30 es divisible entre 6. cada equipo, ¿habrá algún equipo al que le falten ju-
e. 30 es un múltiplo de 6. f. 6 es un múltiplo de 30. gadores?
b. Un reforestador tiene 43,682 semillas para plantar.
¿Pueden plantarse en filas, con 11 semillas por fila?

© Lopez Mateos Editores. ISBN 978-607-95583-2-1, obra completa, versión electrónica, ISBN 978-607-95583-3-8, volumen 1, versión electrónica. Ejemplar asignado a: Helecto Villarroel gutierrez -
helecto@gmail.com. Fecha: 27 de octubre de 2014. Prohibida su modificación, copia o distribución.
296 Enteros y teoría de números

c. Hay 261 estudiantes que se asignarán a 9 maestras de 11. Una papelería bajó el precio de las libretas de $2.00,
modo que cada maestra tenga el mismo número de es- pero las mantuvo por arriba de $1.00. Vendió todas. El
tudiantes. ¿Es ello posible? monto total de la venta de las libretas fue de $31.45.
3. Sin usar calculadora, prueba cada uno de los números siguien- ¿Cuántas libretas vendió?
tes para ver si son divisibles entre 2, 3, 4, 5, 6, 8, 9, 10 y 11: 12. Un grupo de personas ordenó unos lápices. La cuenta fue
a. 746,988 de $2.09. Si el precio original de cada uno era de 12¢ pero
b. 81,342 el precio estaba inflado, ¿cuánto cuesta cada lápiz?
c. 15,810 13. Los años bisiestos ocurren en años divisibles entre 4. Sin
4. Determina cada caso sin realizar la división. Explica embargo, si el año termina en dos ceros, para que el año
cómo lo hiciste en cada caso. sea bisiesto debe ser divisible entre 400. Determina cuá-
a. ¿Es 34,015 divisible entre 17? les de los años siguientes son bisiestos:
b. ¿Es 34,051 divisible entre 17? a. 1776
c. ¿Es 19,031 divisible entre 19? b. 1986
d. ¿Es 2 # 3 # 5 # 7 divisible entre 5? c. 2000
e. ¿Es 12 # 3 # 5 # 72 + 1 divisible entre 5? d. 2100
5. Justifica cada una de las afirmaciones dadas suponiendo que 14. Hay un criterio para verificar cálculos que se llama
a, b y c son enteros. Si la afirmación no se puede justificar prueba del nueve. Considera la suma 193 + 24 + 786 =
por medio de los teoremas de esta sección, responde “no”. 1003. Los residuos cuando 193, 24 y 786 se dividen en-
a. 4 ƒ 20 implica 4 ƒ 113 # 20. tre 9 son 4, 6 y 3, respectivamente. La suma de los resi-
b. 4 ƒ 100 y 4ⱈ13 implica 4ⱈ1100 + 132. duos, 13, tiene residuo 4 al dividirlo entre 9, como
c. 4 ƒ 100 y 4ⱈ13 implica 4ⱈ1300. sucede con 1003. Verificar así los residuos proporciona
d. 3 ƒ (a + b) y 3ⱈc implica 3ⱈ1a + b + c2. una cuasiverificación del cálculo. Halla las sumas si-
e. 3 ƒ a implica 3 ƒ a2. guientes y usa la prueba del nueve para verificar tus re-
6. Clasifica cada uno de los casos siguientes como verda- sultados:
dero o falso. Justifica tus respuestas. a. 12,343 + 4546 + 56
a. Si b ƒ a, entonces 1b + c2 ƒ 1a + c2. b. 987 + 456 + 8765
b. Si b ƒ a, entonces b2 ƒ a3. c. 10,034 + 3004 + 400 + 20
c. Si b ƒ a, entonces b ƒ - a y - b ƒ - a. d. Prueba el criterio con la resta 1003 - 46.
7. Justifica cada caso: e. Prueba el criterio con la multiplicación 345 # 56.
a. 7 ƒ 210 f. ¿Tendría sentido probar el criterio en una división?
b. 19 ƒ 11900 + 382 ¿Por qué sí o por qué no?
c. 6 ƒ 23 # 32 # 174 15. a. Si 21 divide a n, ¿qué otros números naturales divi-
d. 7ⱈ14200 + 222 den a n? ¿Por qué?
8. Clasifica cada uno de los casos siguientes como verda- b. Si 16 divide a n, ¿qué otros números naturales divi-
dero o falso: den a n? ¿Por qué?
a. Si todo dígito de un número es divisible entre 3, el nú- 16. Los números x y y son divisibles entre 5.
mero mismo es divisible entre 3. a. ¿Es la suma de x y y divisible entre 5? ¿Por qué?
b. Si un número es divisible entre 3, entonces todo dí- b. ¿Es la diferencia de x y y divisible entre 5? ¿Por qué?
gito del número es divisible entre 3. c. ¿Es el producto de x y y divisible entre 5? ¿Por qué?
9. Llena cada uno de los siguientes espacios en blanco con 17. Usando sólo criterios de divisibilidad, explica si 6,868,395
el mayor dígito que haga verdadera la afirmación: es divisible entre 15.
a. 3 ƒ 74_ 18. Clasifica cada uno de los casos siguientes como verda-
b. 9 ƒ 83_45 dero o falso, suponiendo que a, b, c y d son enteros. Si
c. 11 ƒ 6_55 una afirmación es falsa, exhibe un contraejemplo.
10. Si es posible, coloca un dígito en el cuadro de modo que a. Si d ƒ (a + b), entonces d ƒ a y d ƒ b.
el número b. Si d ƒ (a + b), entonces d ƒ a o d ƒ b.
527,4 n 2 c. Si d ƒ ab, entonces d ƒ a o d ƒ b.
sea divisible entre d. Si ab ƒ c, entonces a ƒ c y b ƒ c.
a. 2 b. 3 e. Si a ƒ b y b ƒ a, entonces a = b.
c. 4 d. 9 19. Prueba el criterio para la divisibilidad entre 9 en cual-
e. 11 quier número de cinco dígitos.

© Lopez Mateos Editores. ISBN 978-607-95583-2-1, obra completa, versión electrónica, ISBN 978-607-95583-3-8, volumen 1, versión electrónica. Ejemplar asignado a: Helecto Villarroel gutierrez -
helecto@gmail.com. Fecha: 27 de octubre de 2014. Prohibida su modificación, copia o distribución.
Sección 5-3 Divisibilidad 297

Evaluación 5-3B

1. Clasifica cada caso como verdadero o falso. Si es falso, di 9. Cuando los dos dígitos faltantes en el número siguiente
por qué. se reemplazan, el número es divisible entre 99. ¿Cuál es
a. 5 es un múltiplo de 20. el número?
b. 10 es un divisor de 30. 85_ _1
c. 8 ƒ 32.
10. Sin usar calculadora, clasifica cada uno de los casos si-
d. 10 es divisible entre 1.
guientes como verdadero o falso. Justifica tus respuestas.
e. 30 es un factor de 6.
a. 7 ƒ 280021
f. 6 es un múltiplo de 20.
b. 19ⱈ3,800,018
2. Usa los criterios de divisibilidad para responder lo siguiente:
c. 23 ƒ 4610
a. Seis amigos ganaron la lotería con un boleto. El pre-
d. 23ⱈ460,046
mio es de $242,800. ¿Puede repartirse equitativa-
11. En futbol americano un touchdown con un punto extra vale
mente el dinero?
7 puntos y un gol de campo vale 3 puntos. Supón que en un
b. Juan pidió un préstamo de $7812 para un carro nue-
juego las únicas anotaciones realizadas por los equipos fue-
vo. ¿Es posible cubrir esta cantidad en 12 pagos men-
ron touchdowns con punto extra y goles de campo.
suales iguales?
a. ¿Cuáles de los números del 1 al 25 son imposibles como
3. Sin usar calculadora, prueba cada uno de los números si-
marcador final de las anotaciones de un equipo?
guientes para ver si son divisibles entre 2, 3, 4, 5, 6, 8, 9,
b. Lista todas las maneras posibles de que un equipo lo-
10 y 11:
gre 40 puntos.
a. 4,201,012 b. 1001
c. Un equipo logró 57 puntos con 6 touchdowns y 6 pun-
c. 10,001
tos extra. ¿Cuántos goles de campo anotó el equipo?
4. Determina cada caso sin realizar la división. Explica
12. Completa la siguiente tabla, donde n es el entero dado.
cómo lo hiciste en cada caso.
a. ¿Es 24,013 divisible entre 12?
Residuo
b. ¿Es 24,036 divisible entre 12?
cuando la
c. ¿Es 17,034 divisible entre 17?
d. ¿Es 2 # 3 # 5 # 7 divisible entre 3? Residuo suma de los
e. ¿Es 12 # 3 # 5 # 72 + 1 divisible entre 6? cuando n se Suma de los dígitos de n se
5. Justifica cada caso. n divide entre 9 dígitos de n divide entre 9
a. a3 ƒ a4, si a Z 0.
a. 31
b. a4 ƒ a10, si a Z 0.
c. an ƒ am, si 0 … n … m, si a Z 0. b. 143
d. Si b ƒ a y c Z 0, entonces bc ƒ ac. c. 345
6. Justifica cada caso: d. 2987
a. 26 ƒ 1134 # 1002 e. 7652
b. 13ⱈ124 # 53 # 26 + 12
f. Emite una conjetura acerca del residuo y la suma de los
c. 24ⱈ12 # 4 # 6 # 8 # 1710 + 12
dígitos en un entero cuando éste se divide entre 9.
d. 24 ƒ 1104 + 642 13. Un palíndromo es un número que se lee igual hacia ade-
7. Clasifica cada uno de los casos siguientes como verda- lante que hacia atrás.
dero o falso: a. Verifica la divisibilidad entre 11 de los siguientes pa-
a. Si un número es divisible entre 6, entonces es divisi- líndromos de cuatro dígitos:
ble entre 2 y entre 3. i. 4554 ii. 9339 iii. 2002 iv. 2222
b. Si un número es divisible entre 2 y entre 3, entonces b. ¿Son todos los palíndromos de cuatro dígitos divisi-
es divisible entre 6. bles entre 11? ¿Por qué sí o por qué no?
c. Si un número es divisible entre 2 y entre 4, entonces c. ¿Son todos los palíndromos de cinco dígitos divisibles
es divisible entre 8. entre 11? ¿Por qué sí o por qué no?
d. Si un número es divisible entre 8, entonces es divisi- d. ¿Son todos los palíndromos de seis dígitos divisibles
ble entre 2 y entre 4. entre 11? ¿Por qué sí o por qué no?
8. Diseña un criterio de divisibilidad para cada uno de los 14. Los números 5872 y 2785 son un par palindrómico de
números siguientes: números pues al invertir el orden de los dígitos de un
a. 16 número obtenemos el otro número. Explica por qué en
b. 25

© Lopez Mateos Editores. ISBN 978-607-95583-2-1, obra completa, versión electrónica, ISBN 978-607-95583-3-8, volumen 1, versión electrónica. Ejemplar asignado a: Helecto Villarroel gutierrez -
helecto@gmail.com. Fecha: 27 de octubre de 2014. Prohibida su modificación, copia o distribución.
298 Enteros y teoría de números

un par palindrómico, si un número es divisible entre 3 87 - 78 = 9. Emite una conjetura acerca de los resul-
también lo es el otro. tados al realizar esta operación.
15. Clasifica cada uno de los casos siguientes como verda- b. Escoge cualquier número de dos dígitos tal que el dí-
dero o falso, suponiendo que a, b, c, y d son enteros. Si gito en el lugar de las decenas sea 2 más que el dígito
una afirmación es falsa, exhibe un contraejemplo. en el lugar de las unidades. Invierte los dígitos en tu
a. Si d ƒ a y d ƒ b, entonces d ƒ 1ax + by2 para cualesquier número y resta este número del número original, por
enteros x y y. ejemplo 31 - 13 = 18. Emite una conjetura acerca de
b. Si dⱈa y dⱈb, entonces dⱈ1a + b2. los resultados al realizar esta operación.
c. Si d ƒ a2, entonces d ƒ a. c. Demuestra que para cualquier número de dos dígitos,
d. Si dⱈa, entonces dⱈa2. si los dígitos se invierten y los números se restan, la
16. Demuestra el teorema 5–13(b). diferencia es un múltiplo de 9.
17. a. Escoge un número de dos dígitos tal que el número en d. Investiga lo que sucede cuando se restan números de
el lugar de las decenas sea 1 más que el número en el dos dígitos con igual suma de dígitos, por ejemplo
lugar de las unidades. Invierte los dígitos de tu número 62 - 35 = 27.
y resta este número del número original, por ejemplo

Conexiones matemáticas 5-3

Comunicación 777,555,222 y 414,143,313 son divisibles entre 3. Ex-


1. Un cliente quiere enviar un paquete por correo. El em- plica por qué esta afirmación es verdadera.
pleado postal determina que el costo del envío es de 7. Introduce cualquier número de tres dígitos en la calcula-
$18.95, pero sólo tiene timbres de 6¢ y 9¢. ¿Se pueden dora, por ejemplo 243. Repítelo: 243,243. Divide entre 7.
usar las estampillas disponibles para juntar la cantidad Divide entre 11. Divide entre 13. ¿Cuál es la respuesta?
exacta del costo del envío? ¿Por qué sí o por qué no? Inténtalo de nuevo con cualquier otro número de tres dí-
2. a. Juan usa su calculadora para ver si un número n de gitos. ¿Siempre funciona? ¿Por qué?
ocho o menos dígitos es divisible entre un número d. 8. Ceci asegura que puede justificar el criterio de divisibili-
Encuentra que n , d presenta en pantalla 32. ¿Sucede dad entre 11. Ella dice: He notado que cada potencia par de
que d ƒ n? ¿Por qué? 10 se puede escribir como un múltiplo de 11 más 1 y cada po-
b. Si n , d da en pantalla 16.8. ¿Sucede que d ƒ n? ¿Por tencia impar de 10 se puede escribir como un múltiplo de 11
qué? menos 1. De hecho:
3. ¿Es el área de cada uno de los siguientes rectángulos di- 10 = 11 - 1
visible entre 4? Explica por qué sí o por qué no. 102 = 99 + 1 = 9 # 11 + 1
a. 103 = 10 # 102 = 1019 # 11 + 12 = 90 # 11 + 10
= 90 # 11 + 11 - 1 = 91 # 11 - 1
104 = 102 # 102 = 10019 # 11 + 12
= 900 # 11 + 9 # 11 + 1 = 909 # 11 + 1
52,832 cm

y así sucesivamente.
324,518 cm Ahora veo un número de cuatro dígitos abcd y procedo como en
b. la divisibilidad entre 3. Reúno las partes que son divisibles entre
11 sin importar qué dígitos son y agrupo el resto, que es
52,834 cm d - c + b - a
Completa los detalles del argumento de Ceci y justifica
324,514 cm el criterio para la divisibilidad entre 11.
4. ¿Puedes hallar tres números naturales consecutivos de ma- 9. Toma un número escrito en base diez con tres o más dí-
nera que ninguno sea divisible entre 3? Explica tu respuesta. gitos y resta el dígito de las unidades a la expresión indi-
5. Responde cada caso siguiente y justifica tus respuestas. cada. ¿Entre qué números puedes estar seguro que es
a. Si un número no es divisible entre 5, ¿puede ser divi- divisible la diferencia? Justifica tus respuestas.
sible entre 10? a. El dígito de las unidades
b. Si un número no es divisible entre 10, ¿puede ser di- b. El número formado por los últimos dos dígitos (esto
visible entre 5? es, el dígito de las decenas seguido del dígito de las
6. Un número en el que cada dígito excepto el 0 aparece unidades)
exactamente 3 veces es divisible entre 3. Por ejemplo,
© Lopez Mateos Editores. ISBN 978-607-95583-2-1, obra completa, versión electrónica, ISBN 978-607-95583-3-8, volumen 1, versión electrónica. Ejemplar asignado a: Helecto Villarroel gutierrez -
helecto@gmail.com. Fecha: 27 de octubre de 2014. Prohibida su modificación, copia o distribución.
Sección 5-3 Divisibilidad 299

c. La suma de los dígitos 20. Un estudiante dice que un número con una cantidad par
d. Responde las preguntas anteriores para un número de dígitos es divisible entre 7 si, y sólo si, cada uno de los
de tres o más dígitos escrito en base cinco. números formados juntando los dígitos en grupos de dos
10. a. ¿En qué bases la divisibilidad entre 2 dependerá sólo es divisible entre 7. Por ejemplo, 49,562,107 es divisible
del último dígito? Justifica tu respuesta. entre 7 porque cada uno de los números 49, 56, 21 y 07
b. ¿En qué bases la divisibilidad entre 2 dependerá sólo es divisible entre 7. ¿Es verdad esto?
de que la suma de los dígitos sea par o impar? Justi- 21. Una estudiante asegura que un número es divisible entre
fica tu respuesta. 24 si, y sólo si, es divisible entre 6 y entre 4, y que, en ge-
neral, un número es divisible entre a # b si, y sólo si, es di-
Solución abierta visible entre a y entre b. ¿Qué le respondes?
11. Una compañía que manufactura desayunos organizó un 22. Una estudiante halló que todos los números de tres dí-
concurso para el cual se colocaron números en cajas de gitos de la forma aba, donde a + b es un múltiplo de 7,
desayuno. Se asignó un premio de $1000 para la persona son divisibles entre 7. Ella quiere saber por qué es así.
que juntara números cuya suma fuera 100. La compañía ¿Cómo le respondes?
emitió miles de tarjetas con los siguientes números:
Problemas de repaso
3 12 15 18 27 33 45 51 66 75 84 90
23. Halla todos los enteros x (de ser posible) que hagan ver-
a. Si la compañía no hizo más tarjetas, ¿hay una combi- dadero cada uno de los casos siguientes:
nación ganadora? a. 31 - x2 = 6
b. Si la compañía va a añadir un número más a la lista y b. 1 - 22 ƒ x ƒ = 6
quiere asegurarse de que el concurso tenga a lo sumo c. 1 - x2 , 0 = - 1
d. - 1x - 12 = 1 - x
1000 ganadores, sugiere una estrategia de acción.
12. ¿Cómo usarías material físico para explicar a niños pe-
queñas lo siguiente? e. - ƒ - x ƒ = 5
a. Que un número sea par o impar f. - x 6 0
b. Que un número sea divisible entre 3 o que no sea di- 24. Simplifica cada caso:
visible entre 3 a. 3x - 11 - 2x2
c. Que si 4 ƒ a, entonces 2 ƒ a b. 1 - 2x22 - 3x2
c. y - x - 21 y - x2
Aprendizaje colectivo d. 1x - 122 - x2 + 2x
13. En tu grupo, analiza el valor de enseñar diversos crite- 25. Considera la función y = f 1x2 = - 2x - 3 cuyo domi-
rios de divisibilidad en la educación media. Si una maes- nio es el conjunto de los enteros y responde las pregun-
tra decide analizar varios criterios, ¿cómo los debería tas siguientes:
presentar? a. ¿Cuál es f 1 - 52?
b. ¿Para qué valores de x el valor de y es 17?
Preguntas del salón de clase c. ¿Es 2 una salida posible? Explica tu respuesta.
14. Juana asegura que un número es divisible entre 4 si cada d. ¿Cuál el rango de la función?
uno de los dos últimos dígitos es divisible entre 4. ¿Es co-
rrecta su afirmación? De no ser así, ¿qué sugerirías a Juana Pregunta del National Assessment of Educational Progress
que cambiara para que su afirmación fuera precisa? (NAEP) (Evaluación Nacional del Progreso Educativo)
15. Jaime dice que a ƒ b y a>b significan lo mismo. ¿Cómo le Escribe cada uno de los números en el círculo al cual
respondes? pertenecen.
16. Beti notó que 2 ƒ 36, 9 ƒ 36 y 18 ƒ 36. Notó que 18 = 2 # 9. NAEP, 2007, Grado 4
Entonces notó que 4 ƒ 36 y 6 ƒ 36 de modo que pensó que
4 # 6 ó 24 debe dividir a 36. ¿Qué le dices? 30, 47, 124
17. Un estudiante asegura que a ƒ a y a ƒ a implica que a ƒ 1a - a2,
y, por lo tanto, a ƒ 0. ¿El estudiante está en lo correcto? 27
38
18. Un estudiante escribe, “Si dⱈa y dⱈ b, entonces dⱈ1a + b2”.
¿Cómo le respondes? 53 6
1 42
19. Tu grupo de séptimo grado acaba de terminar una unidad
sobre reglas de divisibilidad. Una de las mejores estu-
diantes pregunta por qué la divisibilidad entre números di-
ferentes de 3 y 9 no puede verificarse dividiendo la suma Números impares Números pares
de los dígitos entre el número probado. ¿Cómo deberás
responderle?

© Lopez Mateos Editores. ISBN 978-607-95583-2-1, obra completa, versión electrónica, ISBN 978-607-95583-3-8, volumen 1, versión electrónica. Ejemplar asignado a: Helecto Villarroel gutierrez -
helecto@gmail.com. Fecha: 27 de octubre de 2014. Prohibida su modificación, copia o distribución.
300 Enteros y teoría de números

ROMPECABEZAS Diana vio un número extraordinario de nueve dígitos. Sus nueve dígitos
contienen todos los números del 1 al 9. Además, forma un número con las siguientes características:
cuando se lee de izquierda a derecha, sus primeros dos dígitos forman un número divisible entre 2,
sus primeros tres dígitos forman un número divisible entre 3, sus primeros cuatro dígitos forman un
número divisible entre 4, y así sucesivamente, hasta que el número completo es divisible entre 9.
¿Cuál es ese número extraordinario que vio Diana?

5-4 Números primos y compuestos

En los Principios y objetivos se afirma que:

Los estudiantes deberán reconocer que diferentes tipos de números tienen características parti-
culares; por ejemplo, los números cuadrados tienen un número impar de factores y los números
primos tienen sólo dos factores. 1p. 1512

Un método usado en escuelas de educación básica para determinar los divisores positivos
de un número natural es usar cuadrados de papel y representar el número como un rectán-
gulo. Dicho rectángulo recuerda una barra de dulce formada por pequeños cuadrados. Las
dimensiones del rectángulo son divisores o factores del número. Por ejemplo, la figura 5-20
muestra rectángulos que representan el 12.
1
12

2
3
6
4

Figura 5-20

Como muestra la figura, el número 12 tiene seis divisores positivos: 1, 2, 3, 4, 6 y 12. Si se


usan rectángulos para hallar los divisores de 7, entonces sólo hallaríamos un rectángulo de
1 * 7, como se muestra en la figura 5-21. Así, 7 tiene exactamente dos divisores: 1 y 7.

1
7

Figura 5-21

Para ampliar la ilustración de la cantidad de divisores positivos de un número natural, cons -


truimos la tabla 5-1. Debajo de cada número listado a lo largo de la primera fila, identifica-
mos los números menores o iguales a 37 que tienen ese número de divisores positivos. Por
ejemplo, 12 está en la columna 6 pues tiene seis divisores positivos, y 7 está en la columna 2
pues tiene sólo dos divisores positivos.

© Lopez Mateos Editores. ISBN 978-607-95583-2-1, obra completa, versión electrónica, ISBN 978-607-95583-3-8, volumen 1, versión electrónica. Ejemplar asignado a: Helecto Villarroel gutierrez -
helecto@gmail.com. Fecha: 27 de octubre de 2014. Prohibida su modificación, copia o distribución.
Sección 5-4 Números primos y compuestos 301

Tabla 5-1  Número de divisores positivos


1 2 3 4 5 6 7 8 9

1 2 4 6 16 12 24 36
3 9 8 18 30
5 25 10 20
7 14 28
11 15 32
13 21
17 22
19 26
23 27
29 33
31 34
37 35

AHORA INTENTA ÉSTE 5-11


a. ¿Qué patrones ves que se forman en la tabla 5-1?
b. ¿Habrá otros registros en la columna 1? ¿Por qué?
c. ¿Cuáles son los siguientes tres números en la columna 3?
d. Halla un registro para la columna 7.
e. ¿Qué tipos de números tienen un número impar de factores? ¿Por qué?

Los números en la columna 2 de la tabla 5-1 son de particular importancia. Nota que tie-
nen exactamente dos divisores positivos, a saber, 1 y ellos mismos. Cualquier entero positivo
que tenga exactamente dos distintos divisores positivos es un número primo, o un primo.
Cualquier entero mayor que 1 que tenga un factor positivo distinto de 1 y de sí mismo es un
número compuesto, o un compuesto. Por ejemplo, 4, 6 y 16 son compuestos pues tienen fac-
tores positivos distintos de 1 y de ellos mismos. El número 1 tiene sólo un factor positivo, de
modo que no es primo ni compuesto. De la columna 2 en la tabla 5-1, vemos que los prime-
ros 12 primos son 2, 3, 5, 7, 11, 13, 17, 19, 23, 29, 31 y 37. Nota que el número 2 es el único
primo par. En los problemas exploramos otros patrones de la tabla.

Ejemplo 5-24 Muestra que los números siguientes son compuestos:










a. 1564
b. 2781
c. 1001
d. 3 # 5 # 7 # 11 # 13 + 1

Solución a. Como 2 ƒ 4, 1564 es divisible entre 2 y es compuesto.


b. Como 3 ƒ 12 + 7 + 8 + 12, 2781 es divisible entre 3 y es compuesto.
c. Como 11 ƒ 311 + 02 - 10 + 124, 1001 es divisible entre 11 y es compuesto.
d. Como el producto de dos números impares es impar (¿por qué?), 3 # 5 # 7 # 11 # 13
es impar. Si sumamos 1 a un número impar, la suma es par. Un número par
(diferente de 2) tiene un factor de 2 y es, por lo tanto, compuesto.

© Lopez Mateos Editores. ISBN 978-607-95583-2-1, obra completa, versión electrónica, ISBN 978-607-95583-3-8, volumen 1, versión electrónica. Ejemplar asignado a: Helecto Villarroel gutierrez -
helecto@gmail.com. Fecha: 27 de octubre de 2014. Prohibida su modificación, copia o distribución.
302 Enteros y teoría de números

AHORA INTENTA ÉSTE 5-12 La tira cómica FoxTrot trata acerca de la divisibilidad y los números pri-
mos. Responde las siguientes preguntas basadas en la tira.
a. Escoge al azar un número de la tira y divídelo entre 13, después dividelo entre 17 y luego divídelo entre
19. Sigue haciendo esto hasta que encuentres un número que al dividirlo entre 13, 17 ó 19 te dé como
respuesta un número completo. ¿Qué significa obtener como respuesta un número completo?
b. El autor cometió un error. El número 2261 aparece a la izquierda del centro de la tira. Explica por qué
no se debería incluir este número.

¿Por las

según el número por
Colorea según ¡Asusta!
matemáticas?

¿verdad?

por
Clave de los colores

Divisible entre 13
= verde
Divisible entre 17
= naranja
Divisible entre 19
= rojo

Números primos
= amarillo

Factorización en primos
En los Puntos focales para el grado 7 hallamos la siguiente afirmación:

Los alumnos continúan desarrollando su comprensión de la multiplicación y la división así como la


estructura de los números, al determinar si un número natural mayor que 1 es primo, y si no lo es,
factorizándolo en producto de primos. (p. 19)

Los números compuestos se pueden expresar como productos de dos o más números com-
pletos mayores que 1. Por ejemplo, 18 = 2 # 9, 18 = 3 # 6 ó 18 = 2 # 3 # 3. Cada expresión de
18 como un producto de factores es una factorización.
Una factorización que contenga solamente números primos es una factorización en pri-
mos. Para hallar una factorización en primos de un número compuesto dado, primero rees-
cribimos el número como un producto de dos números más pequeños mayores que 1.
Continuamos el proceso, factorizando los números menores hasta que todos los factores
sean primos. Por ejemplo, considera 260:

260 = 26 # 10 = 12 # 13212 # 52 = 2 # 2 # 5 # 13 = 22 # 5 # 13

El procedimiento para hallar una factorización en primos de un número se puede organizar


usando un árbol de factores, como se ilustra en la figura 5-22(a). Las últimas ramas del ár-
bol presentan los factores primos de 260. Una segunda manera de factorizar 260 se muestra
en la figura 5-22(b). Los dos árboles producen la misma factorización en primos, excepto por
el orden en que aparecen los primos en los productos.

© Lopez Mateos Editores. ISBN 978-607-95583-2-1, obra completa, versión electrónica, ISBN 978-607-95583-3-8, volumen 1, versión electrónica. Ejemplar asignado a: Helecto Villarroel gutierrez -
helecto@gmail.com. Fecha: 27 de octubre de 2014. Prohibida su modificación, copia o distribución.
Sección 5-4 Números primos y compuestos 303

260 260

26 10 5 52

2 13 2 5 2 26

2 13
(a) (b)
Figura 5-22

El teorema fundamental de la aritmética, o teorema de la factorización única, afirma que, en


general, si se hace caso omiso del orden, la factorización en primos de un número es única.

Teorema 5–23
Teorema fundamental de la aritmética Cada número compuesto se puede escribir como un pro-
ducto de primos de una y sólo una manera, excepto por el orden de los factores primos en el producto.

El teorema fundamental de la aritmética nos asegura que una vez hallada una factoriza-
ción en primos de un número, no puede hallarse, de ese mismo número, una factorización
en primos diferente. Por ejemplo, considera 260. Comenzamos con el menor primo, 2, y
vemos si divide a 260. De no ser así, tratamos con el siguiente primo mayor y verificamos la
divisibilidad entre ese primo. Una vez que hallemos un primo que divida al número en
cuestión, debemos hallar el cociente del número dividido entre el primo. Este paso de la
factorización en primos de 260 se muestra en la figura 5-23(a). A continuación verificamos
si el primo divide al cociente. Si sucede así, repetimos el proceso; de no ser así, pasamos al
siguiente primo mayor, 3, y vemos si divide al cociente. Vemos que 130 dividido entre 2 da
65, como se muestra en la figura 5-23(b). Continuamos el procedimiento, usando primos
cada vez mayores, hasta alcanzar el cociente 1. El número original es el producto de todos
los divisores primos usados. El procedimiento completo para 260 se muestra en la figura
5-23(c). Una forma alternativa se muestra en la figura 5-23(d).

2 260 2 260 2 260 260


130 2 130 2 130 2 130
(a) 65 5 65 2 65
(b) 13 13 5 13
1 13 1
(c) (d) Forma alternativa
Figura 5-23
En la factorización en primos de un número, los primos suelen listarse en orden cre-
ciente, de izquierda a derecha, y si un primo aparece en un producto más de una vez, se usa
notación exponencial. Así, la factorización de 260 se escribe como 22 # 5 # 13. En la siguiente
página de muestra se ilustra la factorización en primos. Nota que el árbol de factores se de-
sarrolla de dos maneras diferentes que conducen al mismo resultado. Trabaja con la Práctica
guiada de la página de muestra.
© Lopez Mateos Editores. ISBN 978-607-95583-2-1, obra completa, versión electrónica, ISBN 978-607-95583-3-8, volumen 1, versión electrónica. Ejemplar asignado a: Helecto Villarroel gutierrez -
helecto@gmail.com. Fecha: 27 de octubre de 2014. Prohibida su modificación, copia o distribución.
304 Enteros y teoría de números

Página de un libro de texto ¿CÓMO PUEDES ESCRIBIR UN


NÚMERO COMO PRODUCTO
DE FACTORES PRIMOS?

Lección
Factorización en primos
¿Cómo puedes escribir la factorización en primos de un número?
¡Comprende!
Todo número completo Los números completos mayores que 1 son
Modelo Dimensión Factores
números primos o compuestos.
mayor que 1 es un
número primo o es un

Un número primo tiene precisamente dos


número compuesto.

factores, 1 y él mismo. Los números 2, 3


y 5 son números primos.

Otro ejemplo ¿Cómo puedes usar un árbol de factores para encontrar


la factorización en primos de un número?

Una manera Otra manera


Para hallar la factorización en primos de Para hallar la factorización en primos de
72, comenzamos con el menor factor 72, comenzamos con cualesquier dos factores
primo. Escribe factores hasta que todos de 72. Escribe factores hasta que todos ellos
ellos sean números primos. sean números primos.
Arregla los factores
primos en orden.

Hay sólo una factorización en primos para cada número.

Práctica guiada*
¿Sabes cómo? ¿Entiendes?
En los puntos del 1 al 8, escribe la factorización en ¿Cómo es que los dos anteriores árboles de
primos de cada número. Si es primo escribe: primo. factores ilustran que sólo hay una
factorización en primos para 72?

¿El 1 es primo o compuesto?

*Para otro ejemplo, ver el conjunto B de la página 140.

Fuente: Scott Foresman-Addison Wesley, enVisionMATH, 2008, Grade 6 (p. 124).

© Lopez Mateos Editores. ISBN 978-607-95583-2-1, obra completa, versión electrónica, ISBN 978-607-95583-3-8, volumen 1, versión electrónica. Ejemplar asignado a: Helecto Villarroel gutierrez -
helecto@gmail.com. Fecha: 27 de octubre de 2014. Prohibida su modificación, copia o distribución.
Sección 5-4 Números primos y compuestos 305

AHORA INTENTA ÉSTE 5-13 En las aulas de escuelas de educación básica se usan barras de colores
para enseñar muchos conceptos. La longitud de las barras varía de 1 cm a 10 cm. Las diversas longitudes
tienen colores asociados; por ejemplo, la barra 5 es amarilla. En la figura 5-24 se muestran las barras con
sus colores apropiados. Una fila con todas las barras del mismo color se llama tren de un color.
a. ¿Qué barras se pueden usar para formar un tren de un color para 18?
b. ¿Qué trenes de un color son posibles para 24?
c. ¿Cuántos trenes de un color de dos o más barras son posibles para cada número primo?
d. Si un número se puede representar con un tren rojo, un tren verde y un tren amarillo, ¿cuál es el me-
nor número de factores que debe tener? ¿Cuáles son éstos?

Naranja
Azul
Verde obscuro

Café
Negro
Amarillo
Blanco

Violeta
Verde
Rojo
1

10
Figura 5-24

Número de divisores
¿Cuántos divisores positivos tiene 24? Nota que la pregunta es acerca del número de divisores,
no sólo de los divisores primos. Como ayuda para listarlos, los agrupamos como sigue:

1, 2, 3, 4, 6, 8, 12, 24

Los divisores positivos de 24 se presentan en pares, donde el producto de cada par es 24. Si
3 es un divisor de 24, entonces 24>3, u 8, también es un divisor de 24. En general, si un nú-
mero natural k es un divisor de 24, entonces 24>k también es un divisor de 24.
Otra manera de pensar el número de divisores de 24 es considerar la factorización en pri-
mos 24 = 23 # 3. Los divisores positivos de 23 son 20, 21, 22 y 23. Los divisores positivos de 3
son 30 y 31. Sabemos que 23 tiene 13 + 12, ó 4, divisores y 31 tiene 11 + 12, ó 2, divisores.
Como cada divisor de 24 es el producto de un divisor de 23 y un divisor de 31, entonces usa-
mos el principio fundamental del conteo para concluir que 24 tiene 4 # 2, u 8, divisores posi-
tivos. Esto se resume en la tabla 5-2.

Tabla 5-2
Divisores positivos de 23 20 = 1 21 = 2 22 = 4 23 = 8

Divisores positivos de 31 30 = 1 31 = 3

Divisores positivos de 31 # 23 30 # 20 = 1 30 # 21 = 2 30 # 22 = 4 30 # 23 = 8
(Divisores positivos de 24) 31 # 20 = 3 31 # 21 = 6 31 # 22 = 12 31 # 23 = 24

© Lopez Mateos Editores. ISBN 978-607-95583-2-1, obra completa, versión electrónica, ISBN 978-607-95583-3-8, volumen 1, versión electrónica. Ejemplar asignado a: Helecto Villarroel gutierrez -
helecto@gmail.com. Fecha: 27 de octubre de 2014. Prohibida su modificación, copia o distribución.
306 Enteros y teoría de números

Este análisis puede generalizarse como sigue: si p es cualquier primo y n es cualquier nú-
mero natural, entonces los divisores positivos de pn son p0, p1, p2, p3, Á , pn. Por lo tanto,
hay 1n + 12 divisores de pn. Ahora, usando el principio fundamental del conteo, podemos
hallar el número de divisores positivos de cualquier número cuya factorización en primos
sea conocida.
Teorema 5–24
Si p y q son primos diferentes, entonces pnqm tiene 1n + 121m + 12 divisores positivos. En gene-
ral, si p1, p2 Á , pk son primos y n1, n2 Á , nk son números completos, entonces pn11 # pn22 # Á # pnkk
tiene 1n1 + 121n2 + 12 # Á # 1nk + 12 divisores positivos.

Ejemplo 5-25 Halla la cantidad de divisores positivos de cada número:










a. 1,000,000 b. 21010

Solución a. Primero hallamos la factorización en primos de 1,000,000.


1,000,000 = 106 = 12 # 526 = 12 # 5212 # 5212 # 5212 # 5212 # 5212 # 52
= 12 # 2 # 2 # 2 # 2 # 2215 # 5 # 5 # 5 # 5 # 52
= 26 # 56
Como 26 tiene 6 + 1 divisores positivos y 56 tiene 6 + 1 divisores positivos,
entonces, por el principio fundamental del conteo, 26 # 56 tiene
16 + 1216 + 12, ó 49, divisores positivos.
b. La factorización en primos de 210 es
210 = 21 # 10 = 3 # 7 # 2 # 5 = 2 # 3 # 5 # 7,
21010 = 12 # 3 # 5 # 7210 = 210 # 310 # 510 # 710
Por el principio fundamental del conteo, la cantidad de divisores positivos de
21010 es 110 + 12110 + 12110 + 12110 + 12 = 114 = 14,641.

AHORA INTENTA ÉSTE 5-14 Para ver si es necesario dividir 97 entre 2, 3, 4, 5, 6, Á , 96 para verificar
que es primo, responde lo siguiente (justifica tus respuestas):
a. Si 2 no es divisor de 97, ¿podría un múltiplo de 2 ser divisor de 97?
b. Si 3 no es divisor de 97, ¿qué otros números no podrían ser divisores de 97?
c. Si 5 no es divisor de 97, ¿qué otros números no podrían ser divisores de 97?
d. Si 7 no es divisor de 97, ¿qué otros números no podrían ser divisores de 97?
e. Conjetura qué números debemos verificar en cuanto a su divisibilidad para poder determinar si 97 es primo

Cómo determinar si un número es primo


Como se ilustra en la siguiente caricatura de Sidney Harris, los números primos han fasci-
nado a personas de los más diversos orígenes. En la sección Ahora intenta éste 5-14, quizá ha-
llaste que para determinar si un número es primo hay que verificar solamente la divisibilidad
entre los números primos menores que el número dado. ¿Por qué? Sin embargo, ¿necesita-
mos verificar todos los primos menores que el número? Supón que queremos verificar si 97
es primo y hallamos que 2, 3, 5 y 7 no dividen a 97. ¿Podría un primo mayor dividir a 97? Si
p es un primo mayor que 7, entonces p Ú 11. Si p ƒ 97, entonces 97>p también divide a 97. Sin
embargo, como p Ú 11 entonces 97>p debe ser menor que 10 y, por lo tanto, no puede divi-
dir a 97. ¿Por qué? Así vemos que no hay necesidad de verificar la divisibilidad entre otros
números además de 2, 3, 5 y 7. En los teoremas siguientes se generalizan estas ideas.

© Lopez Mateos Editores. ISBN 978-607-95583-2-1, obra completa, versión electrónica, ISBN 978-607-95583-3-8, volumen 1, versión electrónica. Ejemplar asignado a: Helecto Villarroel gutierrez -
helecto@gmail.com. Fecha: 27 de octubre de 2014. Prohibida su modificación, copia o distribución.
Sección 5-4 Números primos y compuestos 307

O,
M O MENT N. 1677
UN UÍ
JOAQ S
DON PRIMO, E E 43.
E S N R
T
NO I BLE E
I S
DIV

Teorema 5–25
n
Si d es un divisor de n, entonces también es un divisor de n.
d

Supón que p es el mínimo divisor de un número compuesto n (mayor que 1). Dicho divi-
sor debe ser primo (¿por qué?). Entonces n = pk, k Z 1. Como k ƒ n y p era el mínimo divi-
sor común de n, k Ú p. Por lo tanto, n = pk Ú pp = p2. Como n Ú p2, tenemos que
p2 … n. Esta idea se resume en el teorema siguiente.

Teorema 5–26
Si n es compuesto, entonces n tiene un factor primo p tal que p2 … n.

El teorema 5–26 puede usarse como ayuda para determinar si un número dado es primo o
compuesto. Por ejemplo, considera el número 109. Si 109 es compuesto, debe tener un divi-
sor primo p tal que p2 … 109. Los primos cuyos cuadrados no exceden 109 son 2, 3, 5 y 7.
Mentalmente, podemos ver que 2ⱈ109, 3ⱈ109, 5ⱈ109 y 7ⱈ109. Por lo tanto, 109 es primo.
El argumento usado conduce al teorema siguiente.

Teorema 5–27
Si n es un entero mayor que 1 y no es divisible entre ningún primo p tal que p2 … n, entonces n
es primo.

O B S E R VA C I Ó N Como p2 … n implica que p … 1n, el teorema 5–27 dice que para de-
terminar si un número n es primo, basta verificar que ningún primo menor o igual que
1n es un divisor de n.

© Lopez Mateos Editores. ISBN 978-607-95583-2-1, obra completa, versión electrónica, ISBN 978-607-95583-3-8, volumen 1, versión electrónica. Ejemplar asignado a: Helecto Villarroel gutierrez -
helecto@gmail.com. Fecha: 27 de octubre de 2014. Prohibida su modificación, copia o distribución.
308 Enteros y teoría de números

Ejemplo 5-26 a. ¿Es 397 compuesto o primo? b. ¿Es 91 compuesto o primo?











Solución a. Los primos posibles p tales que p2 … 397 son 2, 3, 5, 7, 11, 13, 17 y 19. Como
2ⱈ397, 3ⱈ397, 5ⱈ397, 7ⱈ397, 11ⱈ397, 13ⱈ397, 17ⱈ397 y 19ⱈ397, el número
397 es primo.
b. Los primos posibles p tales que p2 … 91 son 2, 3, 5 y 7. Como 91 is divisible
entre 7, es compuesto.

Una manera sencilla de hallar todos los primos menores que un número dado es usando
la criba de Eratóstenes, llamada así en honor del matemático griego Eratóstenes (ca.
276–194 ac). Si se consideran todos los números naturales mayores que 1 (o se colocan en
la criba), los números que no son primos se tachan metódicamente (o se dejan pasar por los
agujeros de la criba). Los números restantes son primos. La siguiente página de muestra
ilustra este proceso. Antes de continuar la lectura, responde las preguntas en la página de
muestra.
La criba de Eratóstenes es otra manera de motivar el teorema 5–27. Nota las observaciones
de la criba conforme tachamos números, en la tabla 5-3.
Tabla 5-3
Primo Observación
2 El primer número sin tachar divisible entre 2 es 4 = 22.
3 El primer número sin tachar divisible entre 3 es 9 = 32.
5 El primer número sin tachar divisible entre 5 es 25 = 52.
7 El primer número sin tachar divisible entre 7 es 49 = 72.

No necesitamos continuar el procedimiento para 11 pues el primer número sin tachar divi-
sible entre 11 es 112, ó 121, y la tabla sólo llega hasta 100. Por lo tanto, para saber si un nú-
mero como 137 es primo, primero averiguamos si es divisible entre los primos hasta, sin
incluirlo, el primer primo cuyo cuadrado sea mayor que 137. Como 132 = 169, cualquier
primo mayor o igual que 13 daría un cociente menor o igual a 13, y ya verificamos esos pri-
mos. Esto nos dice que al tratar de probar que un número es primo, necesitamos usar como
divisores sólo a primos cuyos cuadrados sean menores o iguales que el número que estamos
probando.

Más acerca de los primos


Hay infinidad de números completos, hay infinidad de números completos pares y hay infi-
nidad de números completos impares. ¿Hay infinidad de primos? Debido a que los números
primos no aparecen siguiendo un patrón conocido, la respuesta no es obvia. Euclides fue el
primero en demostrar que hay infinidad de números primos.

◆ Nota Eratóstenes (276–194 ac), un erudito griego, nació en Cirene pero pasó la mayor parte de
histórica su vida en Alejandría como director de la biblioteca del museo. En su obra Geográfica, dio
razones para afirmar que la Tierra tiene forma esférica. Actualmente, Eratóstenes es me-
jor conocido por su “criba” —un procedimiento sistemático para aislar números primos—
y por un método sencillo para calcular la circunferencia de la Tierra. ◆

© Lopez Mateos Editores. ISBN 978-607-95583-2-1, obra completa, versión electrónica, ISBN 978-607-95583-3-8, volumen 1, versión electrónica. Ejemplar asignado a: Helecto Villarroel gutierrez -
helecto@gmail.com. Fecha: 27 de octubre de 2014. Prohibida su modificación, copia o distribución.
Sección 5-4 Números primos y compuestos 309

Página de un libro de texto LA CRIBA DE ERATÓSTENES

Complemento
La criba de Eratóstenes
Alrededor de 230 a.c., el matemático griego Eratóstenes desarrolló un método
para identificar los números primos. El método se llama criba de Eratóstenes.

Sigue los pasos para identificar


los números primos del 1 al 100.

Paso 1 Copia la tabla de la derecha.


Paso 2 Tacha el 1 pues no es primo
ni compuesto.
Paso 3 Señala el 2. Después tacha
todos los múltiplos de 2.
Paso 4 Pasa al primer número que
no esté tachado. Señálalo y
tacha todos sus múltiplos.
Paso 5 Repite el paso 4 hasta que
todos los números de la tabla
estén señalados o tachados. Los
números señalados son primos.

Lista los números primos del 1 al 100.


Explica por qué algunos números pueden tacharse más de una vez.

Todas las páginas están disponibles en Internet y en CD. Sección A Lección

Fuente: Scott Foresman-Addison Wesley, Mathematics, 2008, Grade 6 ( p. 149).

Los matemáticos han buscado por largo tiempo una fórmula que produzca sólo primos,
pero no la han hallado. Un resultado fue la expresión n2 - n + 41, donde n es un número
completo. Al substituir n por 0, 1, 2, 3, Á , 40 en la expresión, siempre se produce un primo.
Sin embargo, al substituir n por 41 da 412 - 41 + 41, ó 412, un número compuesto. En 1998
Roland Clarkson, un estudiante de 19 años de California State University, mostró que
23021377 - 1 es primo. El número tiene 909,526 dígitos. La expansión decimal completa del
número llenaría varios cientos de páginas. Desde entonces se han descubierto más primos ma-
yores: 232,582,657 - 1 19,808,358 dígitos2 y 230,402,457 - 1 19,152,052 dígitos2. Éstos son
ejemplos de los primos de Mersenne. Un primo de Mersenne, llamado así en honor del monje

◆ Nota Sophie Germain (1776–1831) nació en París y creció durante la Revolución Francesa. Ella
histórica quería estudiar en la prestigiosa École Polytechnique, pero no se admitían estudiantes mu-
jeres. En consecuencia, estudió con notas de clase y con la monografía de Gauss sobre teo-
ría de números. Realizó importantes aportes a la teoría matemática de la elasticidad, por lo
cual se le otorgó el premio de la Academia Francesa de Ciencias. El trabajo de Germain fue
seguido de cerca por Gauss, quien la recomendó para un grado honorario en la Universi-
dad de Göttingen. Ella murió antes de que le pudieran otorgar el grado. ◆

© Lopez Mateos Editores. ISBN 978-607-95583-2-1, obra completa, versión electrónica, ISBN 978-607-95583-3-8, volumen 1, versión electrónica. Ejemplar asignado a: Helecto Villarroel gutierrez -
helecto@gmail.com. Fecha: 27 de octubre de 2014. Prohibida su modificación, copia o distribución.
310 Enteros y teoría de números

francés Marin Mersenne (1588–1648), es un primo de la forma 2n - 1, donde n es primo. El


23 de agosto de 2008, una computadora de UCLA (University of California Los Angeles) descu-
brió el 45º primo de Mersenne conocido, 243,112,609 - 1 112,978,189 dígitos2. El 6 de septiem-
bre de 2008 se descubrió, en Alemania, el 46º primo de Mersenne, 237,156,661 -
1 111,185,272 dígitos2.
La búsqueda de primos grandes ha propiciado avances en el cómputo distribuido, esto es, en el uso
de Internet para aprovechar la capacidad de cómputo ociosa de una gran cantidad de computado-
ras. La búsqueda de primos de Mersenne se ha utilizado como prueba para equipos de cómputo.
Otro tipo de primo interesante es un primo de Sophie Germain, el cual es un primo impar p
para el cual 2p + 1 también es primo. Nota que p = 3 es un primo de Sophie Germain pues
2 # 3 + 1, ó 7, también es primo. Verifica que 5, 11 y 23 también sean de ese tipo de primos.
Esos primos se llaman así en honor de la matemática francesa Sophie Germain. En 2007, el
mayor primo de Sophie Germain descubierto tenía 51,910 dígitos.

Resolver Problemas ¿Cuántos osos?


Una tienda de juguetes ofrece un tipo de osos de peluche. El lunes vendieron cierta canti-
dad de osos de peluche por un total de $1843 y el martes, sin cambiar el precio, la tienda
vendió cierta cantidad de osos de peluche por un total de $1957. ¿Cuántos osos se vendie-
ron diariamente si el precio de cada pieza es un número completo mayor que $1?

Comprender el problema Cierto día una tienda vendió una cantidad de osos de peluche por
$1843 y al día siguiente una cantidad de ellos por un total de $1957. Necesitamos hallar la
cantidad de osos vendidos cada día.

Trazar un plan Si x osos se vendieron el primer día y y osos el segundo día, y si el precio de
cada oso era de c dólares, tendríamos cx = 1843 y cy = 1957. Así, 1843 y 1957 deberán te-
ner un factor común: el precio c. Podríamos factorizar cada número y hallar los posibles fac-
tores. Si el problema ha de tener una solución única, los dos números deberán tener un solo
factor común además de 1. Cualquier factor común de 1957 y 1843 también será factor de
1957 - 1843 = 114 y los factores de 114 son más fáciles de hallar.

Tenemos 114 = 2 # 57 = 2 # 3 # 19. Así, si 1957 y 1843 tienen un factor co-


Realizar el plan
mún primo, debe ser 2, 3 ó 19. Pero ni 2 ni 3 dividen a los números, por lo que el único fac-
tor común posible es 19. Dividimos cada número entre 19 y hallamos
1843 = 19 # 97
1957 = 19 # 103

◆ Nota En la década de 1970 la determinación de primos grandes se volvió extremadamente útil para
histórica codificar y descodificar mensajes secretos. En todo tipo de codificación y descodificación, las
letras del alfabeto corresponden, de alguna manera, a enteros no negativos. Tres científicos
del Massachusetts Institute of Technology (Ronald Rivest, Adi Shamir y Leonard Adle-
man) diseñaron un sistema de codificación “seguro”, conocido como sistema RSA (sus ini-
ciales), en el cual los mensajes son ininteligibles para todos excepto para el destinatario. La
llave secreta para descifrar consta de dos primos grandes elegidos por el usuario. La llave
para cifrar es el producto de estos dos primos. Debido a la extrema dificultad y al tiempo re-
querido para factorizar números grandes, era prácticamente imposible obtener la llave para
descifrar a partir de la llave conocida para cifrar. En 1982 se inventaron nuevos métodos para
factorizar números grandes, lo cual provocó que se usaran primos aún mayores para prevenir
la ruptura de las llaves para descodificar. ◆
© Lopez Mateos Editores. ISBN 978-607-95583-2-1, obra completa, versión electrónica, ISBN 978-607-95583-3-8, volumen 1, versión electrónica. Ejemplar asignado a: Helecto Villarroel gutierrez -
helecto@gmail.com. Fecha: 27 de octubre de 2014. Prohibida su modificación, copia o distribución.
Sección 5-4 Números primos y compuestos 311

Nota que ni 97 ni 103 son divisibles entre 2, 3, 5 ó 7. Por lo tanto, 97 y 103 son primos
(¿por qué?) y, en consecuencia, el único factor común (mayor que 1) de 1843 y 1957 es 19.
Así, el precio de cada oso fue de $19. El primer día se vendieron 97 osos y el día siguiente
se vendieron 103.

Revisar Nota que el problema tuvo solución única pues el único factor común (mayor que
1) de los dos números fue 19. Podemos crear problemas similares haciendo que el precio del
objeto sea un número primo y que el número de objetos vendidos cada día también sea
primo. Por ejemplo, la venta total del primer día podría ser 23 # 101, ó $2323 y la del se-
gundo día 23 # 107, ó $2461 (nota que 23, 101 y 107 son números primos).
Para hallar un factor común de 1957 y 1843, hallamos todos los factores comunes de
1957 - 1843 = 114 = 2 # 3 # 19 y verificamos cuál de los factores de la diferencia era fac-
tor común de los números originales. Hemos usado la propiedad del teorema 5–13: Si
d ƒ a y d ƒ b, entonces d ƒ 1a - b2. Este teorema nos asegura que todo factor común de a y b
◆ también será factor de a - b.

Evaluación 5-4A

1. Halla el menor número positivo que sea divisible entre 7. a. Cuando la bandera de Estados Unidos tenía 48 estre-
tres primos diferentes. llas, éstas formaban un arreglo rectangular de 6 por 8.
2. Determina cuáles de los números siguientes son primos: ¿En qué otro arreglo rectangular podrían colocarse?
a. 109 b. 119 b. ¿Cuántos arreglos rectangulares de estrellas podrían
c. 33 d. 101 formarse si hubiera sólo 47 estados?
e. 463 f. 97 8. a. Usa el teorema fundamental de la aritmética para jus-
g. 2 # 3 # 5 # 7 + 1 h. 2 # 3 # 5 # 7 - 1 tificar que si 2 ƒ n y 3 ƒ n, entonces 6 ƒ n.
3. Usa un árbol de factores para hallar la factorización en b. ¿Es siempre cierto que si a ƒ n y b ƒ n, entonces ab ƒ n2?
primos para los números siguientes: Demuestra la afirmación o exhibe un contraejemplo.
a. 504 b. 2475 c. 11,250 9. Don Alfonso quiere plantar árboles frutales formando
4. a. Llena los números faltantes en el siguiente árbol de un arreglo rectangular. Para cada uno de los siguientes
factores: números de árboles, halla los posibles números de filas si
cada fila ha de tener el mismo número de árboles:
a. 36
b. 28
c. 17
2 d. 144
10. Algunos de los divisores de un número de casillero son
2, 5 y 9. Si hay exactamente nueve divisores adicionales,
5 ¿cuál es el número de casillero?
11. Extiende la criba de Eratóstenes para hallar todos los
primos entre 100 y 200.
3 7 12. Los números primos 11 y 13 se llaman primos gemelos
pues difieren en 2. (No se ha demostrado la existencia de
b. ¿Cómo podrías hallar el número de arriba sin hallar infinidad de primos gemelos). Halla todos los primos ge-
los otros dos números? melos menores que 200.
5. ¿Cuál es el mayor primo que debes tomar en cuenta para 13. Si 42 ƒ n, ¿qué otros enteros positivos dividen a n?
verificar si 5669 es primo? 14. Si 1000 es un factor de n, ¿qué otros enteros positivos di-
6. Halla las factorizaciones en primos de: viden a n? ¿Cuántos de esos enteros positivos hay?
a. 1 # 2 # 3 # 4 # 5 # 6 # 7 # 8 # 9 # 10 15. No se sabe si hay infinidad de primos en la sucesión infi-
b. 102 # 26 # 4910 nita formada sólo por unos; 1, 11, 111, 1111, . . . . Halla
c. 251 infinidad de números compuestos en la sucesión.
d. 1001

© Lopez Mateos Editores. ISBN 978-607-95583-2-1, obra completa, versión electrónica, ISBN 978-607-95583-3-8, volumen 1, versión electrónica. Ejemplar asignado a: Helecto Villarroel gutierrez -
helecto@gmail.com. Fecha: 27 de octubre de 2014. Prohibida su modificación, copia o distribución.
312 Enteros y teoría de números

16. ¿Es 32 # 24 un factor de 34 # 27? Explica por qué sí o por 19. Halla las factorizaciones en primos de los casos siguientes:
qué no. a. 3610 # 4920 # 615
17. Explica por qué cada uno de los números siguientes es b. 10060 # 30040
compuesto: c. 2 # 34 # 5110 # 7 + 4 # 34 # 5110
a. 3 # 5 # 7 # 11 # 13 d. 2 # 3 # 5 # 7 # 11 + 1
b. 13 # 4 # 5 # 6 # 7 # 82 + 2 20. Soy un número primo mayor que 40 y menor que 90.
c. 13 # 5 # 7 # 11 # 132 + 5 Mis dígitos de las unidades y de las decenas son primos.
d. 10! + 7 (Nota: 10! = 10 # 9 # 8 # 7 # 6 # 5 # 4 # 3 # 2 # 1.2 La diferencia entre mis dígitos de las unidades y de las
18. Explica por qué 23 # 32 # 253 no es una factorización en decenas no es 2. ¿Qué número soy?
primos y halla la factorización en primos de ese número.

Evaluación 5-4B

1. Determina cuáles de los números siguientes son primos: 7. Halla el menor número divisible entre cada número na-
a. 89 b. 147 tural menor o igual que 12.
c. 159 d. 187 8. Halla el mayor número de cuatro dígitos que tenga exac-
e. 2 # 3 # 5 # 7 + 5 f. 2 # 3 # 5 # 7 - 5 tamente tres factores positivos.
2. Usa un árbol de factores para hallar la factorización en 9. Muestra que si el 1 se considerara primo, todo número
primos para los números siguientes: tendría más de una factorización en números primos.
a. 304 10. ¿Es posible hallar enteros positivos x, y y z tales que
b. 1570 2x # 3y = 5z? ¿Por qué sí o por qué no?
c. 9550 11. a. Muestra que hay infinidad de números compuestos
3. a. Llena los números faltantes en el siguiente árbol de en la sucesión aritmética 1, 5, 9, 13, 17, Á .
factores: b. ¿Toda sucesión aritmética formada por enteros con
diferencia mayor que 0 tiene infinidad de números
compuestos? Justifica tu respuesta.
12. Si 2N = 26 # 35 # 54 # 73 # 117, explica por qué 2 # 3 # 5 # 7 # 11
es un factor de N.
2 13. ¿Es 32 # 24 un factor de 33 # 22? Explica por qué sí o por
qué no.
14. Explica por qué cada uno de los números siguientes es
3 compuesto:
a. 7 # 11 # 13 # 17 + 17
b. 10! + k, donde k = 2, 3, 4, 5, 6, 7, 8, 9 ó 10
2 3 15. Explica por qué 22 # 53 # 92 no es una factorización en pri-
b. ¿Cómo podrías hallar el número de arriba sin hallar mos y halla la factorización en primos de ese número.
los otros dos números? 16. Un primo como 7331 es un superprimo pues cualquier
4. ¿Cuál es el mayor primo que debes tomar en cuenta para entero obtenido suprimiendo dígitos de la derecha de
verificar si 503 es primo? 7331 es primo; por ejemplo, 733, 73 y 7.
5. Halla las factorizaciones en primos de: a. Para que un primo sea superprimo, ¿qué dígitos no
a. 1001 pueden aparecer en el número?
b. De los dígitos que pueden aparecer en un superprimo,
b. 10012
¿qué dígito no puede ser el dígito de la extrema iz-
c. 99910
quierda de un superprimo?
d. 11110 - 1119
c. Halla todos los superprimos de dos dígitos.
6. Supón que los 435 miembros de la Cámara de Represen-
d. Halla un superprimo de tres dígitos distinto de 733.
tantes se distribuyeran en comités formados por más de
17. ¿Es siempre verdadero lo siguiente? (Justifica tu res-
2 miembros, pero menos de 30. Cada comité debe tener
puesta). Si m ƒ ab, entonces m ƒ a o m ƒ b.
igual número de miembros y cada miembro ha de estar
18. Halla las factorizaciones en primos de los casos siguientes:
en sólo un comité.
a. 164 # 814 # 66
a. ¿Cuál es el tamaño de los posibles comités?
b. 84 # 325
b. ¿Cuántos comités hay de cada tamaño?
c. 22 # 35 # 755 + 24 # 34 # 755

© Lopez Mateos Editores. ISBN 978-607-95583-2-1, obra completa, versión electrónica, ISBN 978-607-95583-3-8, volumen 1, versión electrónica. Ejemplar asignado a: Helecto Villarroel gutierrez -
helecto@gmail.com. Fecha: 27 de octubre de 2014. Prohibida su modificación, copia o distribución.
Sección 5-4 Números primos y compuestos 313

19. Usa el teorema fundamental de la aritmética para justifi- 20. El producto de tres números primos menores que 30 es
car la siguiente afirmación acerca de los números com- 1955. ¿Cuáles son los tres primos?
pletos a y b mayores que 1. Si p es primo y p ƒ ab, entonces
p ƒ a o p ƒ b.

Conexiones matemáticas 5-4

Comunicación d. ¿El argumento muestra que hay infinidad de primos?


1. Explica por qué el producto de cualesquier tres enteros ¿Por qué sí o por qué no?
consecutivos es divisible entre 6. e. Sea M = 2 # 3 # 5 # 7 # 11 # 13 # 17 # 19 + 1. Sin multipli-
2. Explica por qué el producto de cualesquier cuatro enteros car, explica por qué algún primo mayor que 19 debe
consecutivos es divisible entre 24. dividir a M.
3. Para verificar la divisibilidad entre 12, una estudiante ve- Solución abierta
rificó la divisibilidad entre 3 y 4; otra verificó la divisibili-
dad entre 2 y 6. ¿Están ambas estudiantes usando un 9. a. ¿En cuál de los intervalos siguientes crees que hay más
enfoque correcto para la divisibilidad entre 12? ¿Por qué primos? ¿Por qué? Verifica si estás en lo correcto.
sí o por qué no? i. 0–99 ii. 100–199
4. En la criba de Eratóstenes para números menores que 100 b. ¿Cuál es la cadena más larga de números compuestos
explica por qué, después de que tachamos todos los múlti- consecutivos en los intervalos?
plos de 2, 3, 5 y 7, los números restantes son primos. c. ¿Cuántos primos gemelos (ver el problema 16) hay
5. Sea M = 2 # 3 # 5 # 7 + 11 # 13 # 17 # 19. Sin multiplicar, en cada intervalo?
muestra que ninguno de los primos menores o iguales que d. ¿Qué patrones, de haberlos, ves en las preguntas anterio-
19 divide a M. res? Predice lo que podría pasar en otros intervalos.
6. Una señora con una canasta de huevos halla que si retira 10. Un número es perfecto si la suma de sus factores (aparte de sí
los huevos de la canasta en grupos de 3 ó 5, siempre queda mismo) es igual a él. Por ejemplo, 6 es un número perfecto
1 huevo. Sin embargo, si retira los huevos en grupos de 7, pues sus factores suman 6, esto es, 1 + 2 + 3 = 6. Un
no queda ninguno. Si en la canasta caben hasta 100 hue- número abundante tiene factores cuya suma es mayor que el nú-
vos, ¿cuántos huevos tiene? Explica tu razonamiento. mero mismo. Un número deficiente es un número con factores
7. Explica por qué, cuando un número es compuesto, su me- cuya suma es menor que el número mismo.
nor divisor positivo, distinto de 1, debe ser primo. a. Clasifica cada uno de los siguientes números como
8. Euclides demostró que dada cualquier lista finita de pri- perfecto, abundante o deficiente:
mos, existe un primo que no está en la lista. Lee el si- i. 12 ii. 28 iii. 35
guiente argumento y responde las preguntas que siguen. b. Halla al menos un número que corresponda a cada
Sea 2, 3, 5, 7, Á , p una lista de todos los primos me- clase.
nores o iguales que cierto primo p. Mostraremos que
Aprendizaje colectivo
existe un primo que no está en la lista. Considera el pro-
ducto 11. Un grupo de 23 estudiantes usó losetas cuadradas para
2#3#5#7# Á #p construir formas rectangulares. Cada estudiante tenía
más de una loseta y cada uno tenía un número diferente
Nota que todo primo en nuestra lista divide ese producto. de losetas. Cada estudiante fue capaz de construir una
Sin embargo, si sumamos 1 al producto, esto es, si forma- sola forma rectangular. Tuvieron que usarse todas las lo-
mos el número N = 12 # 3 # 5 # 7 # Á # p2 + 1, entonces setas para construir un rectángulo, y el rectángulo no
ninguno de los primos de la lista divide a N. Nota que ya podía tener huecos. Por ejemplo, un rectángulo de 2 por
sea N primo o compuesto, algún primo q debe dividir a N. 6 usa 12 losetas y se considera igual al rectángulo de 6
Como ningún primo de nuestra lista divide a N, q no es de por 2, pero es diferente de un rectángulo de 3 por 4. El
los primos de nuestra lista. En consecuencia, q 7 p. He- grupo realizó la actividad usando el menor número de
mos mostrado que existe un primo mayor que p. losetas. ¿Cuántas losetas usó? Dividan el trabajo entre
a. Explica por qué ningún primo de la lista divide a N. los miembros del grupo para explorar los diversos rec-
b. Explica por qué algún primo debe dividir a N. tángulos que pudieron hacerse.
c. Alguien descubrió un primo que tiene 65,050 dígitos.
¿De qué manera el argumento anterior nos asegura
que existe un primo aún mayor?

© Lopez Mateos Editores. ISBN 978-607-95583-2-1, obra completa, versión electrónica, ISBN 978-607-95583-3-8, volumen 1, versión electrónica. Ejemplar asignado a: Helecto Villarroel gutierrez -
helecto@gmail.com. Fecha: 27 de octubre de 2014. Prohibida su modificación, copia o distribución.
314 Enteros y teoría de números

Preguntas del salón de clase 16. Un estudiante de sexto grado argumenta que hay infini-
12. María dice que su árbol de factores para 72 comienza dad de primos pues “no hay fin para los números”.
con 3 y 24, de modo que sus factores primos serán dife- ¿Cómo le respondes?
rentes de los de Lalo pues él comenzará con 8 y 9. ¿Qué 17. Un estudiante asegura que todo primo mayor que 3 es un
le dices a María? término en la sucesión aritmética cuyo término n-ésimo es
13. Beto dice que para verificar si un número es primo, él sólo 6n + 1 o en la sucesión aritmética cuyo término n-ésimo
usa las reglas de divisibilidad que conoce para 2, 3, 4, 5, 6, es 6n - 1. ¿Es cierto? De ser así, ¿por qué?
8 y 10. Él dice que si el número no es divisible entre esos Problemas de repaso
números, entonces es primo. ¿Cómo le respondes?
14. José dice que todo número impar mayor que 3 se puede 18. Clasifica lo siguiente como verdadero o falso:
escribir como la suma de dos primos. Para convencer al a. 11 es un factor de 189.
grupo, escribió: 7 = 2 + 5, 5 = 2 + 3, y 9 = 7 + 2. b. 1001 es un múltiplo de 13.
¿Cómo le respondes? c. 7 ƒ 1001 y 7ⱈ12 implica 7ⱈ11001 - 122.
15. Una estudiante del octavo grado en una escuela secunda- 19.Verifica la divisibilidad entre 2, 3, 4, 5, 6, 7, 8, 9, 10 y 11
ria asegura que debido a que hay tantos números pares de cada número:
como impares entre 1 y 1000, debe haber tantos números a. 438,162
que tengan un número par de divisores positivos como b. 2,345,678,910
números que tengan un número impar de divisores posi- 20. Demuestra que si un número es divisible entre 12, en-
tivos entre 1 y 1000. ¿Tiene razón la estudiante? ¿Por qué tonces es divisible entre 3.
sí o por qué no? 21. ¿Podrían dividirse $3376 exactamente entre siete u ocho
personas?

ACTIVIDAD DE LABORATORIO  En la figura 5-25, comienza una espiral con centro en 41 y continúa
en dirección contraria al giro de las manecillas del reloj. Los primos están sobre fondo blanco. Fíjate en los
primos a lo largo de la diagonal con fondo blanco. ¿Puedes obtener cada uno de los primos a partir de la
fórmula n2 + n + 41 substituyendo valores apropiados para n?

265 264 263 262 261 260 259 258 257 256 255 254 253 252 251
210 209 208 207 206 205 204 203 202 201 200 199 198 197 250
211 162 161 160 159 158 157 156 155 154 153 152 151 196 249
212 163 122 121 120 119 118 117 116 115 114 113 150 195 248
213 164 123 90 89 88 87 86 85 84 83 112 149 194 247
214 165 124 91 66 65 64 63 62 61 82 111 148 193 246
215 166 125 92 67 50 49 48 47 60 81 110 147 192 245
216 167 126 93 68 51 42 41 46 59 80 109 146 191 244
217 168 127 94 69 52 43 44 45 58 79 108 145 190 243
218 169 128 95 70 53 54 55 56 57 78 107 144 189 242
219 170 129 96 71 72 73 74 75 76 77 106 143 188 241
220 171 130 97 98 99 100 101 102 103 104 105 142 187 240
221 172 131 132 133 134 135 136 137 138 139 140 141 186 239
222 173 174 175 176 177 178 179 180 181 182 183 184 185 238
223 224 225 226 227 228 229 230 231 232 233 234 235 236 237
Figura 5-25

© Lopez Mateos Editores. ISBN 978-607-95583-2-1, obra completa, versión electrónica, ISBN 978-607-95583-3-8, volumen 1, versión electrónica. Ejemplar asignado a: Helecto Villarroel gutierrez -
helecto@gmail.com. Fecha: 27 de octubre de 2014. Prohibida su modificación, copia o distribución.
Sección 5-5 Máximo divisor común y mínimo múltiplo común 315

ROMPECABEZAS Un sábado Yoli interrumpió su visita a su amiga Natalia para llevar a otras tres ami-
gas al cine. “¿Qué edad tienen?”, preguntó Natalia. “El producto de sus edades es 2450 y la suma es exac-
tamente el doble de tu edad”, respondió Yoli. Natalia lo pensó por un momento y dijo: “Necesito más
información”. A lo cual Yoli respondió, “Debo decirte que yo soy por lo menos un año menor que la ma-
yor de mis tres amigas”. Con esta información, Natalia obtuvo de inmediato la edad de las amigas.
¿Cómo pudo saber Natalia la edad de las amigas y cuáles fueron sus edades?

5-5 Máximo divisor común y mínimo múltiplo común

Considera la situación siguiente:

Dos bandas se combinan para desfilar. Una banda de 24 miembros marchará detrás de una banda de
30 miembros. Las bandas combinadas deberán tener el mismo número de columnas y el mismo nú-
mero de miembros en cada columna. ¿Cuál es el mayor número de columnas en que pueden marchar?

Las bandas podrían marchar en 2 columnas y tendríamos el mismo número de columnas,


pero esto no satisface la condición de tener el mayor número de columnas posible. El nú-
mero de columnas debe dividir a ambos, 24 y 30. ¿Por qué? Los números que dividen a 24
y a 30 son 1, 2, 3 y 6. El mayor de estos números es 6, de modo que las bandas deberán mar-
char en 6 columnas. La primera banda tendrá 6 columnas con 4 miembros en cada co-
lumna, y la segunda banda tendrá 6 columnas con 5 miembros en cada columna. En este
problema hemos hallado el mayor número que divide tanto a 24 como a 30, esto es, el má-
ximo divisor común (MDC) de 24 y 30.

◆ Nota de Definición
investigación El máximo divisor común (MDC) de dos enteros a y b es el mayor entero que divide tanto a a
como a b.
Posiblemente debido
a que los estudiantes
confunden a menudo En la Nota de investigación vemos que a los estudiantes se les dificultan los máximos divi-
factores y múltiplos, sores comunes (MDC) y los mínimos múltiplos comunes (MMC). Proporcionamos varios
el máximo factor co- métodos para hallar MDC y MMC y así aclarar estos conceptos.
mún y el mínimo
múltiplo común son
Método de las barras de colores
temas difíciles de Podemos construir un modelo de dos o más enteros con barras de colores para determinar
comprender para el MDC de dos enteros positivos. Por ejemplo, considera hallar el MDC de 6 y 8 usando la
ellos (Graviss barra del 6 y la barra del 8, como en la figura 5-26.
y Greaver 1992). ◆
barras del 2 barras del 2

barra del 6 barra del 8

barra del 6 barra del 8


(a) (b)
Figura 5-26
© Lopez Mateos Editores. ISBN 978-607-95583-2-1, obra completa, versión electrónica, ISBN 978-607-95583-3-8, volumen 1, versión electrónica. Ejemplar asignado a: Helecto Villarroel gutierrez -
helecto@gmail.com. Fecha: 27 de octubre de 2014. Prohibida su modificación, copia o distribución.
316 Enteros y teoría de números

Para hallar el MDC de 6 y 8 debemos hallar la barra más larga tal que podamos usar múlti-
plos de esa barra para construir la barra del 6 y la barra del 8. Se pueden usar las barras del 2
para construir ambas barras, la del 6 y la del 8, como se muestra en la figura 5-26(b); se pue-
den usar barras del 3 para construir la barra del 6 pero no la barra del 8; se pueden usar las ba-
rras del 4 para construir la barra del 8, pero no la del 6; con las barras del 5 no podemos
construir ninguna, ni podemos usar barras del 6 para construir la barra del 8. Por lo tanto,
MDC16, 82 = 2.

AHORA INTENTA ÉSTE 5-15 Explica cómo puedes usar barras de colores para resolver el problema del
desfile de bandas enunciado al principio de esta sección.

Método de la intersección de conjuntos


En el método de intersección de conjuntos listamos todos los miembros del conjunto de divi-
sores positivos de los dos enteros, de entre éstos hallamos el conjunto de todos los divisores
comunes y, finalmente, escogemos el mayor elemento en ese conjunto. Por ejemplo, para ha-
llar el MDC de 20 y 32 denotamos los conjuntos de divisores positivos de 20 y 32 por D20 y
D32, respectivamente.
D20 = 51, 2, 4, 5, 10, 206

D32 = 51, 2, 4, 8, 16, 326


El conjunto de todos los divisores comunes positivos de 20 y 32 es
D20 ¨ D32 = 51, 2, 46
Como el mayor número en el conjunto de divisores comunes positivos es 4, el MDC de
20 y 32 es 4, que se escribe MDC120, 322 = 4.

AHORA INTENTA ÉSTE 5-16 El diagrama de Venn de la figura 5-27 ilustra los factores de 24 y 40. Res-
ponde a lo siguiente:
a. ¿Cuál es el significado de cada una de las regiones sombreadas?
b. ¿Qué factor es el MDC?
c. ¿Traza un diagrama de Venn similar para hallar el MDC de 36 y 44.

Factores de 24 Factores de 40

3 1 5
6 2 10
12 4 20
24 8 40

Figura 5-27

Método de factorización en primos


El método de la intersección de conjuntos es tardado y tedioso si los números tienen mu-
chos divisores. Otro método, más eficiente, es el método de la factorización en primos. Para
hallar MDC(180, 168), nota primero que
180 = 2 # 2 # 3 # 3 # 5 = 122 # 323 # 5
y
168 = 2 # 2 # 2 # 3 # 7 = 122 # 322 # 7
© Lopez Mateos Editores. ISBN 978-607-95583-2-1, obra completa, versión electrónica, ISBN 978-607-95583-3-8, volumen 1, versión electrónica. Ejemplar asignado a: Helecto Villarroel gutierrez -
helecto@gmail.com. Fecha: 27 de octubre de 2014. Prohibida su modificación, copia o distribución.
Sección 5-5 Máximo divisor común y mínimo múltiplo común 317

Vemos que 180 y 168 tienen en común dos factores de 2 y uno de 3. Estos primos comu-
nes dividen tanto a 180 como a 168. De hecho, los números distintos de 1 que dividen a
180 y 168 no deben tener más de dos 2 y un 3 y ningún otro factor primo en su factori-
za ción en primos. Los divisores comunes positivos posibles son 1, 2, 22, 3, 2 # 3 y 22 # 3.
Por lo tanto, el máximo divisor común de 180 y 168 es 22 # 3. El procedimiento para
hallar el MDC de dos o más números usando el método de la factorización en primos se
resume a continuación:
Para hallar el MDC de dos o más enteros positivos, halla primero las factorizaciones en primos de
los números dados y después identifica cada factor primo común de los números dados. El MDC
es el producto de los factores comunes, cada uno elevado a la menor potencia en la que ese
primo se presenta en cualquiera de las factorizaciones en primos.

Si aplicamos la técnica de factorización en primos para hallar MDC(4, 9), vemos que 4 y 9
no tienen factores primos comunes. Pero eso no significa que no exista el MDC. Todavía te-
nemos a 1 como divisor común, de modo que MDC14, 92 = 1. Números como 4 y 9, cuyo
MDC es 1, son primos relativos. En la siguiente página de muestra verás el método de la in-
tersección de conjuntos y el método de la factorización en primos. Estudia la página y trabaja
las preguntas del Tema de plática al final de la página de muestra. Nota que el MFC en la pá-
gina de muestra significa máximo factor común, que es lo mismo que MDC.

Ejemplo 5-27 Halla lo siguiente:










a. MDC1108, 722
b. MDC10, 132
c. MDC1x, y2 si x = 23 # 72 # 11 # 13 y y = 2 # 73 # 13 # 17
d. MDC1x, y, z2 si z = 22 # 7, usando x y y de 1c2
e. MDC1x, y2, donde x = 54 # 1310 y y = 310 # 1120

Solución a. Como 108 = 22 # 33 y 72 = 23 # 32, se sigue que MDC1108, 722 =


22 # 32 = 36.
b. Como 13 ƒ 0 y 13 ƒ 13, se sigue que MDC10, 132 = 13.
c. MDC1x, y2 = 2 # 72 # 13 = 1274.
d. Como x = 23 # 72 # 11 # 13, y = 2 # 73 # 13 # 17 y z = 22 # 7, entonces
MDC1x, y, z2 = 2 # 7 = 14. Nota que MDC1x, y, z2 también puede
obtenerse hallando el MDC de z y 1274, la respuesta de 1c2.
e. Como x y y no tienen factores comunes primos, MDC1x, y2 = 1.

Método de la calculadora

Las calculadoras que tienen la tecla Simp se pueden usar para hallar el MDC de dos números.
Por ejemplo, para hallar MDC1120, 1802 usamos la siguiente sucesión de teclas: primero teclea
1 2 0 > 1 8 0 Simp = para obtener en pantalla N/D : n/d 60/90 . Al presionar la
tecla x y vemos 2 en la pantalla como un divisor común de 120 y 180. Al presionar de nuevo
la tecla x y y oprimiendo, Simp = x y , vemos de nuevo 2 como factor. Se repite el pro-
ceso hasta obtener 3 y 5 como otros factores comunes. El MDC de 120 y 180 es el producto de
los factores comunes primos 2 # 2 # 3 # 5, ó 60.

© Lopez Mateos Editores. ISBN 978-607-95583-2-1, obra completa, versión electrónica, ISBN 978-607-95583-3-8, volumen 1, versión electrónica. Ejemplar asignado a: Helecto Villarroel gutierrez -
helecto@gmail.com. Fecha: 27 de octubre de 2014. Prohibida su modificación, copia o distribución.
318 Enteros y teoría de números

Página de un libro de texto MÁXIMO FACTOR COMÚN

Lección 3-3
Calentamiento
Idea clave
Hay diferentes Máximo factor común Lista los factores de
maneras de hallar cada número.
los factores que Aprende
son comunes a dos
o más números. ¿Cómo puedes usar factores?
Vocabulario Carmelita está haciendo paquetes de tentempiés para
• factor común un grupo de excursionistas. Cada paquete deberá
• máximo factor tener el mismo número de bolsas de granola y el
común (MFC) mismo número de botellas de agua. ¿Cuál es el
• factorización mayor número de paquetes que puede hacer sin que Excursión
en primos
(p. 147) sobren refrigerios? Refrigerios
ex 60 bolsas de granola
ef l i ó Para resolver este problema, necesitas hallar
90 botellas de agua
R

los números que son factores tanto de 60


como de 90. Estos son los factores comunes
¡Piensa!
• Puedo usar factores de 60 y 90. El máximo factor común (MFC) es
para identificar
el mayor número que es factor de 60 y de 90.
grupos iguales que
comparten.
Ejemplo
• Puedo hacer una
lista organizada para Halla el MFC de 60 y 90.
hallar los factores
Una manera Otra manera
comunes y el MFC.
Lista los factores de cada número. Usa la factorización en primos.

Engloba pares de factores comunes. Halla el producto de los factores


Escoge el mayor. comunes primos. De no haber
factores comunes primos, el
MFC es 1.

El MFC es 30.
El MFC es 30.

El mayor número de paquetes de tentempiés que ella puede hacer es 30.

Tema de plática
En el segundo método, ¿por qué se usa 2 sólo una
vez como factor del MFC?
En cada uno de los 30 paquetes de tentempiés, ¿cuántas
bolsas de granola y cuántas botellas de agua habrá?
Razonamiento Halla el MFC de Colócalo en la RED
48 y de 120. ¿Qué método usaste? Más ejemplos
¿Por qué?

Fuente: Scott Foresman-Addison Wesley Mathematics 2008, Grade 6 (p. 150).

© Lopez Mateos Editores. ISBN 978-607-95583-2-1, obra completa, versión electrónica, ISBN 978-607-95583-3-8, volumen 1, versión electrónica. Ejemplar asignado a: Helecto Villarroel gutierrez -
helecto@gmail.com. Fecha: 27 de octubre de 2014. Prohibida su modificación, copia o distribución.
Sección 5-5 Máximo divisor común y mínimo múltiplo común 319

Algunas calculadoras ya tienen incorporada la característica de MDC; seguramente tienes


que ir al menú MATH para encontrarla. Con esta característica, seleccionas MDC y tecleas
los números separados por una coma dentro de paréntesis; por ejemplo, MDC(120, 180).
Cuando se presiona = aparece en pantalla el MDC, que es 60.

Método del algoritmo de Euclides


Los números grandes pueden ser difíciles de factorizar. Para estos números hay otro mé-
todo más eficiente que la factorización para hallar el MDC. Por ejemplo, supón que quere-
mos hallar MDC(676, 221). Si pudiéramos hallar dos números más pequeños cuyo MDC
fuera el mismo que MDC(676, 221), nuestra tarea sería más fácil. Por el teorema 5–13(c),
todo divisor de 676 y 221es también un divisor de 676 - 221 y 221. Recíprocamente, todo
divisor de 676 - 221 y 221 también es un divisor de 676 y 221. Así, el conjunto de todos
los divisores comunes de 676 y 221 es igual que el conjunto de todos los divisores comunes
de 676 - 221 y 221. En consecuencia, MDC1676, 2212 = MDC1676 - 221, 2212. Este pro-
ceso se puede continuar restando tres 221 de 676 de modo que MDC1676, 2212 =
MDC1676 - 3 # 221, 2212 = MDC113, 2212. Para determinar cuántos 221 podemos restar de
676, pudimos haber dividido como sigue:
3 R 13 17 R 0
221 冄 676 13 冄 221

Cuando se obtiene 0 como residuo, esto significa que las divisiones se han completado.
Como MDC10, 132 = 13, MDC1676, 2212 = 13. Basados en esta ilustración, hacemos la
generalización esbozada en el siguiente teorema.

Teorema 5–28
Si a y b son cualesquier números completos mayores que 0 y a Ú b, entonces
MDC1a, b2 = MDC1r, b2, donde r es el residuo cuando a se divide entre b.

O B S E R VA C I Ó N Como MDC1x, y2 = MDC1 y, x2 para todos los números completos x


y y tales que no sean ambos 0, el teorema 5–28 puede reescribirse como
MDC1a, b2 = MDC1b, r2.

Hallar el MDC de dos números por medio del uso repetido del teorema 5-28 hasta al-
canzar el residuo 0 se conoce como algoritmo euclidiano. Este método se halla en el Li-
bro IV de Los Elementos de Euclides (300 a.c.). En la figura 5-28 se da un diagrama de flujo
para usar el algoritmo euclidiano.

Números Dividir el
¿El residuo Sí El último
positivos a y número mayor
es cero? divisor es el
b, a ≠ b. entre el menor. MDC de a y b.
No

Divide el
último divisor
entre el residuo.

Figura 5-28

© Lopez Mateos Editores. ISBN 978-607-95583-2-1, obra completa, versión electrónica, ISBN 978-607-95583-3-8, volumen 1, versión electrónica. Ejemplar asignado a: Helecto Villarroel gutierrez -
helecto@gmail.com. Fecha: 27 de octubre de 2014. Prohibida su modificación, copia o distribución.
320 Enteros y teoría de números

Ejemplo 5-28 Usa el algoritmo euclidiano para hallar MDC(10764, 2300).











Solución 4 Así, MDC110764, 23002 = MDC12300, 15642.
2300 冄 10764
9200
1564
1 Así, MDC12300, 15642 = MDC11564, 7362.
1564 冄 2300
1564
736
2 Así, MDC11564, 7362 = MDC1736, 922.
736 冄 1564
1472
92
8 Así, MDC1736, 922 = MDC192, 02.
92 冄 736
736
0
Como MDC192, 02 = 92, se sigue que MDC110764, 23002 = 92.

O B S E R VA C I Ó N El procedimiento para hallar el MDC usando el algoritmo euclidiano


se puede detener en cualquier paso en que el MDC sea obvio.

También se puede usar una calculadora con la característica de división entera para efec-
tuar el algoritmo euclidiano. Esta característica produce el cociente y el residuo cuando se
hace una división. Por ejemplo, si la tecla de división entera se ve como INT , , entonces
para hallar MDC110764, 23002 procedemos como sigue:

1 0 7 6 4 INT , 2 3 0 0 = que presenta en pantalla 4 1564


Q R
2 3 0 0 INT , 1 5 6 4 = 1 736
que presenta en pantalla Q R
1 5 6 4 INT , 7 3 6 = 2 92
que presenta en pantalla Q R
7 3 6 INT , 9 2 = 8 0
que presenta en pantalla Q R

El último número entre el que dividimos cuando obtuvimos residuo 0 es 92, así que
MDC110764, 23002 = 92
A veces podemos usar atajos para hallar el MDC de dos o más números, como en el
ejemplo siguiente.

Ejemplo 5-29 Halla lo siguiente:










a. MDC1134791, 6341, 63392


b. El MDC de dos enteros consecutivos cualesquiera.

Solución a. Cualquier divisor común de tres números también es un divisor común de cua-
lesquier dos de ellos (¿por qué?). En consecuencia, el MDC de tres números no

© Lopez Mateos Editores. ISBN 978-607-95583-2-1, obra completa, versión electrónica, ISBN 978-607-95583-3-8, volumen 1, versión electrónica. Ejemplar asignado a: Helecto Villarroel gutierrez -
helecto@gmail.com. Fecha: 27 de octubre de 2014. Prohibida su modificación, copia o distribución.
Sección 5-5 Máximo divisor común y mínimo múltiplo común 321

puede ser mayor que el MDC de cualesquier dos de los números. Los núme-
ros 6341 y 6339 son cercanos y, por lo tanto, es más fácil hallar su MDC:
MDC16341, 63392 = MDC16341 - 6339, 63392
= MDC12, 63392
= 1
Como MDC1134791, 6341, 63392 no puede ser mayor que 1, se sigue que
debe ser igual a 1.
b. Nota que MDC14, 52 = 1, MDC15, 62 = 1, MDC16, 72 = 1, y
MDC199, 1002 = 1. Parece que el MDC de dos enteros consecutivos es 1.
Para justificar esta conjetura necesitamos mostrar que para todos los enteros
n, MDC1n, n + 12 = 1. Tenemos
MDC1n, n + 12 = MDC1n + 1, n2 = MDC1n + 1 - n, n2
= MDC11, n2
= 1

Mínimo múltiplo común
Las salchichas se venden usualmente en paquetes de 10 y el pan para hacer hot-dogs suele
venderse en paquetes de 8 piezas. Esta disparidad causa problemas cuando uno trata de ha-
cer corresponder salchichas y panes. ¿Cuál es el menor número de paquetes de cada uno
que podrías ordenar de modo que haya igual número de salchichas que de pan? Las cantida-
des de salchichas que podemos tener son simplemente los múltiplos de 10, esto es, 10,
20, 30, 40, 50, Á . Asimismo, las cantidades posibles de pan son 8, 16, 24, 32, 40, 48, Á .
Podemos ver que el número de salchichas se corresponde con el número de panes cuando
10 y 8 tienen múltiplos en común. Esto sucede en 40, 80, 120, Á . En este problema esta-
mos interesados en el menor de estos múltiplos, 40. Por lo tanto, podemos obtener el mis-
mo número de salchichas y de pan en la menor cantidad si compramos cuatro paquetes de
salchichas y cinco paquetes de pan. La respuesta de 40 es el mínimo múltiplo común
(MMC) de 8 y 10.

Definición
Supón que a y b son números naturales. Entonces el mínimo múltiplo común (MMC) de a y b es
el menor número natural que es simultáneamente un múltiplo de a y un múltiplo de b.

Como en el MDC, hay diversos métodos para hallar mínimo múltiplo común.

Método de la recta numérica


Se puede usar una recta numérica para determinar el MMC de dos números. Por ejemplo, para
determinar MMC(3, 4) podemos mostrar los múltiplos de 3 y de 4 sobre la recta numérica por
medio de intervalos de longitud 3 y 4, como se ilustra en la figura 5-29.

4 4 4 4
3 3 3 3 3

0 1 2 3 4 5 6 7 8 9 10 11 12 13 14 15 16

Figura 5-29

© Lopez Mateos Editores. ISBN 978-607-95583-2-1, obra completa, versión electrónica, ISBN 978-607-95583-3-8, volumen 1, versión electrónica. Ejemplar asignado a: Helecto Villarroel gutierrez -
helecto@gmail.com. Fecha: 27 de octubre de 2014. Prohibida su modificación, copia o distribución.
322 Enteros y teoría de números

Comenzando en 0, vemos que las puntas de las flechas no coinciden sino hasta el punto
12 sobre la recta numérica. Si continuáramos la recta, las flechas coincidirían en 24, 36, 48
y así sucesivamente. Vemos que hay un número infinito de múltiplos comunes de 3 y 4, pero
el mínimo múltiplo común positivo es 12. Nota que este enfoque de la recta numérica es
ilustrativo y ayuda a comprender, pero no es práctico usarlo para números grandes.

Método de las barras de colores


Podemos usar barras de colores para determinar el MMC de dos números. Por ejemplo, con-
sidera la barra del 3 y la barra del 4 de la figura 5-30(a). Construimos trenes de barras del 3 y
de barras del 4 hasta que tengan la misma longitud, según se muestra en la figura 5-30(b). El
MMC es la longitud común del tren.
Tres barras del 4
Cuatro barras del 3

barra del 10 barra del 2

barra del 3 barra del 4 Longitud de 12


(a) (b)

Figura 5-30

Método de la intersección de conjuntos


En el método de intersección de conjuntos, primero hallamos el conjunto de todos los múlti-
plos positivos tanto del primero como del segundo números, después hallamos el conjunto
de todos los múltiplos comunes positivos de ambos números y finalmente escogemos el menor
elemento de ese conjunto. Por ejemplo, para hallar el MMC de 8 y 12 denotamos los con-
juntos de los múltiplos positivos de 8 y 12 con M8 y M12, respectivamente.
M8 = 58, 16, 24, 32, 40, 48, 56, 64, 72, Á 6

M12 = 512, 24, 36, 48, 60, 72, 84, 96, 108, Á 6
El conjunto de los múltiplos comunes positivos es
M8 ¨ M12 = 524, 48, 72, Á 6
Como el número menor en M8 ¨ M12 es 24, el MMC de 8 y 12 es 24, que se escribe
MMC18, 122 = 24.

AHORA INTENTA ÉSTE 5-17 Traza un diagrama de Venn para ilustrar M8 y M12 y muestra cómo hallar
MMC(8, 12) por medio del diagrama

Método de la factorización en primos


El método de la intersección de conjuntos para hallar el MMC con frecuencia es largo, es-
pecialmente cuando se usa para hallar el MMC de tres o más números naturales. Otro mé-
todo, más eficiente, para hallar el MMC de varios números es el método de la factorización en
primos. Por ejemplo, para hallar MMC140, 122, primero hallamos las factorizaciones en pri-
mos de 40 y 12, a saber 23 # 5 y 22 # 3, respectivamente.

© Lopez Mateos Editores. ISBN 978-607-95583-2-1, obra completa, versión electrónica, ISBN 978-607-95583-3-8, volumen 1, versión electrónica. Ejemplar asignado a: Helecto Villarroel gutierrez -
helecto@gmail.com. Fecha: 27 de octubre de 2014. Prohibida su modificación, copia o distribución.
Sección 5-5 Máximo divisor común y mínimo múltiplo común 323

Si m = MMC140, 122, entonces m es múltiplo de 40 y debe contener como factores a 23


y 5. Además, m es un múltiplo de 12 y debe contener 22 y 3 como factores. Como 23 es un
múltiplo de 22, entonces m = 23 # 5 # 3 = 120. En general, tenemos lo siguiente:

Para hallar el MMC de dos números naturales, primero halla la factorización de cada número. Des-
pués toma cada uno de los primos que son factores de alguno de los números dados. El MMC es
el producto de estos primos, cada uno elevado a la mayor potencia en la que ese primo se pre-
senta en cualquiera de las factorizaciones en primos.

Ejemplo 5-30 Halla el MMC de 2520 y 10,530.










Solución 2520 = 23 # 32 # 5 # 7.
10,530 = 2 # 34 # 5 # 13.
MMC12520, 105302 = 23 # 34 # 5 # 7 # 13 = 294,840

El método de factorización en primos también puede usarse para hallar el MMC de más de
dos números. Por ejemplo, para hallar MMC(12, 108, 120) podemos proceder como sigue:
12 = 22 # 3
108 = 22 # 33
120 = 23 # 3 # 5
Entonces, MMC112, 108, 1202 = 23 # 33 # 5 = 1080.

El método del producto MDC-MMC


Para ver la conexión entre el MDC y el MMC, considera el MDC y el MMC de 24 y 30.
Las factorizaciones en primos de estos números son
24 = 23 # 3
30 = 2 # 3 # 5
En la figura 5-31 se muestra un diagrama que ilustra la factorización en primos.

Factores primos de 24 Factores primos de 30

2 2 5
2 3

Figura 5-31

Nota que MDC124, 302 = 2 # 3 es el producto de los factores en la región sombreada y que
MMC124, 302 = 23 # 3 # 5 es el producto de los factores en las regiones combinadas. Nota
también que
MDC124, 302 # MMC124, 302 = 12 # 32123 # 3 # 52 = 123 # 3212 # 3 # 52 = 24 # 30
Esto muestra que el producto del MDC y el MMC de 24 y 30 es igual a 24 # 30. En general,
la conexión entre el MDC y el MMC de cualquier par de números naturales está dada por
el teorema 5–29.

© Lopez Mateos Editores. ISBN 978-607-95583-2-1, obra completa, versión electrónica, ISBN 978-607-95583-3-8, volumen 1, versión electrónica. Ejemplar asignado a: Helecto Villarroel gutierrez -
helecto@gmail.com. Fecha: 27 de octubre de 2014. Prohibida su modificación, copia o distribución.
324 Enteros y teoría de números

Teorema 5–29
Para cualesquier dos números naturales a y b,
MDC1a, b2 # MMC1a, b2 = ab

El teorema 5–29 puede justificarse de varias maneras. Aquí presentamos un ejemplo es-
pecífico que sugiere cómo puede demostrarse el teorema
a = 513 # 720 # 114 y b = 510 # 725 # 116 # 13
Entonces,
MMC1a, b2 = 513 # 725 # 116 # 13 y MDC1a, b2 = 510 # 720 # 114
Ahora tenemos
MMC1a, b2 # MDC1a, b2 = 513 + 10 # 725 + 20 # 116 + 4 # 13 y ab = 513 + 10 # 720 + 25 # 114 + 6 # 13
Para los valores anteriores de a y b, el teorema 5–29 es válido. Nota, sin embargo, que en
el producto MMC1a, b2 # MDC1a, b2 tenemos todas las potencias de los primos que apare-
cen en a o en b, pues para el MMC tomamos la mayor de las potencias de los primos comu-
nes y para el MDC la menor. También en ab tenemos todas las potencias. Por lo tanto, el
teorema 5–29 es válido en general.

Método del algoritmo euclidiano


El teorema 5–29 es útil para hallar el MMC de dos números a y b cuando sus factorizacio-
nes en primos no son fáciles de hallar. El MDC(a, b) se puede hallar por medio del algo-
ritmo euclidiano, el producto ab se puede hallar por simple multiplicación y MMC1a, b2 se
puede hallar dividiendo.
Ejemplo 5-31 Halla MMC1731, 9522.








Solución Por el algoritmo euclidiano, MDC1731, 9522 = 17. Por el teorema 5–29,
17 # MMC1731, 9522 = 731 # 952
En consecuencia,
731 # 952
MMC1731, 9522 = = 40,936
17

Método de la división entre primos


Otro procedimiento para hallar el MMC de varios números naturales incluye la división en-
tre primos. Por ejemplo, para hallar MMC112, 75, 1202 comenzamos con el menor primo
que divide por lo menos a uno de los números dados y dividimos como sigue:
2 :12, 75, 120
6, 75, 60
Como 2 no divide a 75, simplemente bajamos el 75. Para obtener el MMC usando este pro-
cedimiento, continuamos el proceso hasta que el renglón de respuestas conste de números
primos relativos como se muestra a continuación.

© Lopez Mateos Editores. ISBN 978-607-95583-2-1, obra completa, versión electrónica, ISBN 978-607-95583-3-8, volumen 1, versión electrónica. Ejemplar asignado a: Helecto Villarroel gutierrez -
helecto@gmail.com. Fecha: 27 de octubre de 2014. Prohibida su modificación, copia o distribución.
Sección 5-5 Máximo divisor común y mínimo múltiplo común 325

2 12, 75, 120


2 6, 75, 60
2 3, 75, 30
3 3, 75, 15
5 1, 25, 5
1, 5, 1
Así, MMC112, 75, 1202 = 2 # 2 # 2 # 3 # 5 # 1 # 5 # 1 = 23 # 3 # 52 = 600.

Evaluación 5-5A

1. Halla el MDC y el MMC para cada uno de los casos si- 9. Vendiendo galletas a 24¢ cada una, José juntó suficiente
guientes, usando el método de la intersección de conjuntos: dinero para comprar varias latas de refresco de 45¢ por
a. 18 y 10 b. 24 y 36 lata. Si no le quedó dinero después de comprar el re-
c. 8, 24 y 52 d. 7 y 9 fresco, ¿cuál es el menor número de galletas que pudo
2. Halla el MDC y el MMC para cada uno de los casos si- haber vendido?
guientes, usando el método de la factorización en primos: 10. Dos ciclistas recorren un trayecto circular. El primer
a. 132 y 504 b. 65 y 1690 ciclista completa una vuelta en 12 min y el segundo ci-
c. 900, 96 y 630 d. 108 y 360 clista la completa en 18 min. Si comienzan en el mismo
3. Halla el MDC para cada uno de los casos siguientes, lugar, al mismo tiempo y en la misma dirección, ¿des-
usando el algoritmo euclidiano: pués de cuántos minutos se encontrarán de nuevo en el
a. 220 y 2924 b. 14,595 y 10,856 lugar de partida?
4. Halla el MMC para cada uno de los casos siguientes, 11. Tres motociclistas recorren una pista circular de carreras
usando cualquier método: comenzando en el mismo lugar y al mismo tiempo. El
a. 24 y 36 primero pasa por el punto de partida cada 12 min, el se-
b. 72 y 90 y 96 gundo cada 18 min y el tercero cada 16 min. ¿Después
c. 90 y 105 y 315 de cuántos minutos pasarán de nuevo los tres juntos por
d. 9100 y 25100 el punto de partida? Explica tu razonamiento.
5. Halla el MMC para cada uno de los siguientes pares de 12. Supón que a y b son números naturales y responde lo
números usando el teorema 5–29 y las respuestas del siguiente:
problema 3: a. Si MDC1a, b2 = 1, halla MMC1a, b2.
a. 220 y 2924 b. Halla MDC1a, a2 y MMC1a, a2.
b. 14,595 y 10,856 c. Halla MDC1a2, a2 y MMC1a2, a2.
6. Usa barras de colores para hallar el MDC y el MMC de d. Si a ƒ b, halla MDC1a, b2 y MMC1a, b2.
6 y 10. 13. Clasifica cada caso como verdadero o falso:
7. En el dormitorio de Patricia hay tres relojes de alarma, a. Si MDC1a, b2 = 1, entonces a y b no pueden ser am-
cada uno programado para activarse en tiempos diferen- bos pares.
tes. El reloj A se activa cada 15 min, el reloj B se activa b. Si MDC1a, b2 = 2, entonces a y b son pares.
cada 40 min y el reloj C se activa cada 60 min. Si los tres c. Si a y b son pares, entonces MDC1a, b2 = 2.
relojes se activan a las 6:00 a.m., contesta lo siguiente: 14. Para hallar MDC124, 20, 122, es posible hallar
a. ¿Cuánto tiempo transcurrirá antes de que las alarmas MDC124, 202, que es 4, y después hallar MDC14, 122,
se activen juntas de nuevo, después de las 6:00 a.m.? que es 4. Usa este enfoque y el algoritmo euclidiano para
b. ¿La respuesta a (a) sería diferente si el reloj B se acti- hallar
vara cada 15 min y el reloj A se activara cada 40 min? a. MDC1120, 75, 1052.
8. Midas tenía 120 monedas de oro y 144 monedas de b. MDC134578, 4618, 46192.
plata. Quiere colocar sus monedas de oro y sus monedas 15. Muestra que 97,219,988,751 y 4 son primos relativos.
de plata en pilas de modo que haya el mismo número de 16. La estación de radio reparte cupones de descuento para
monedas en cada pila. ¿Cuál es el mayor número de mo- cada doce y trece llamadas. Cada vigésima llamada recibe
nedas que puede colocar en cada pila? boletos gratis para un concierto. ¿Qué llamada fue la pri-
mera en obtener un cupón y un boleto para el concierto?

© Lopez Mateos Editores. ISBN 978-607-95583-2-1, obra completa, versión electrónica, ISBN 978-607-95583-3-8, volumen 1, versión electrónica. Ejemplar asignado a: Helecto Villarroel gutierrez -
helecto@gmail.com. Fecha: 27 de octubre de 2014. Prohibida su modificación, copia o distribución.
326 Enteros y teoría de números

17. Susana gastó la misma cantidad de dinero en DVDs que a2b21a + b2, donde no es posible mayor factorización sin
en discos compactos. Si el DVD cuesta $12 y el CD $16, conocer los valores de a y b. Nota que a2b2 es el MDC de
¿cuál es la mínima cantidad que pudo haber gastado en
a3b2 y a2b3. Factoriza lo más posible:
cada uno?
18. En una tienda de artículos para fiestas, los platos de papel a. 12x4y3 + 18x3y4
vienen en paquetes de 30, las tazas de papel en paquetes de b. 12x3y2z2 + 18x2y4z3 + 24x4y3z4
15 y las servilletas en paquetes de 20. ¿Cuál es el menor nú- 22. Marca las siguientes afirmaciones como “siempre es ver-
mero de platos, tazas y servilletas que pueden comprarse de dadero”, “a veces es verdadero” o “nunca es verdadero”.
modo que se tenga el mismo número de cada uno? Justifica tus respuestas.
19. Se pueden usar diagramas para mostrar los factores de a. MDC1a, b2 = MDC1 ƒ a ƒ , b2 = MDC1 ƒ a ƒ , ƒ b ƒ 2
dos o más números. Traza diagramas para mostrar los b. MDC1-a, b2 = MDC1a, -b2 = MDC1-a, -b2
factores primos de cada uno de los siguientes conjuntos 23. Halla el MDC y el MMC de cada caso. (No calcules los
de tres números: productos.)
a. 10, 15, 60 b. 8, 16, 24 a. 10!, 11!
20. ¿Cuáles son los factores de 410? b. 10!, 10! + 1
21. En álgebra a menudo es necesario factorizar una ex- 24. Factoriza mil millones como el producto de dos números
que no contenga, ninguno de ellos, al cero.
presión lo más posible. Por ejemplo, a3b2 + a2b3 =

Evaluación 5-5B

1. Halla el MDC y el MMC para cada uno de los casos si- película en el Cinema II dura 90 min. Si la proyección es
guientes, usando el método de la intersección de conjuntos: continua, ¿cuándo comenzarán de nuevo a la misma hora?
a. 12 y 18 b. 18 y 36 10. Un terreno rectangular de 75 pies por 625 pies se va a divi-
c. 12, 18 y 24 d. 6 y 11 dir en porciones cuadradas del mismo tamaño. Si los lados
2. Halla el MDC y el MMC para cada uno de los casos si- de los cuadrados han de ser números completos de pies:
guientes, usando el método de la factorización en primos: a. ¿Cuáles son los mayores cuadrados posibles y cuántos
a. 11 y 19 b. 140 y 320 cuadrados cabrán en el terreno?
c. 800, 75 y 450 d. 104 y 320 b. ¿Cuáles son los cuadrados más pequeños posibles?
3. Halla el MDC para cada uno de los casos siguientes, c. ¿Qué otro tamaño de cuadrados es posible?
usando el algoritmo euclidiano:
11. La directora de una escuela primaria quiere dividir cada
a. 14,560 y 8250 b. 8424 y 2520
uno de los tres grupos de cuarto grado en grupos iguales
4. Halla el MMC para cada uno de los casos siguientes,
más pequeños con al menos 2 estudiantes en cada uno.
usando cualquier método:
Si los grupos tienen 18, 24 y 36 estudiantes, respectiva-
a. 25 y 36
mente, ¿qué tamaños de grupos son posibles?
b. 82 y 90 y 50
12. Supón que a y b son números naturales y responde lo si-
c. 80 y 105 y 315
guiente:
d. 8100 y 50100
5. Halla el MMC para cada uno de los siguientes pares de núme- a. Si a y b son dos números primos, halla MDC1a, b2 y
ros usando el teorema 5–29 y las respuestas del problema 3: MMC1a, b2.
a. 14,560 y 8250 b. ¿Cuál es la relación entre a y b si MDC1a, b2 = a?
b. 8424 y 2520 c. ¿Cuál es la relación entre a y b si MMC1a, b2 = a?
6. Una tienda que renta películas da unas palomitas gratis a 13. Clasifica cada caso como verdadero o falso: para todos los
cada cuatro clientes y una renta de película gratis a cada números naturales a y b.
seis clientes. Usa el método de la recta numérica para a. MMC1a, b2 ƒ MDC1a, b2.
saber cuáles clientes fueron los primeros en ganar ambos b. Para todos los números naturales a y b, MMC1a, b2 ƒ ab.
premios. c. MDC1a, b2 … a.
7. Usa barras de colores para hallar el MDC y el MMC de
d. MMC1a, b2 Ú a.
4 y 10.
14. Para hallar MDC(24, 20, 12) es posible hallar MDC(24, 20),
8. Beto y Sofía trabajan de noche. Beto tiene libre cada
que es 4, y después hallar MDC(4, 12), que es 4. Usa este en-
sexta noche y Sofía tiene libre cada octava noche. Si am-
foque y el algoritmo euclidiano para hallar
bos tienen noche libre hoy, ¿cuántas noches pasarán an-
tes de que vuelvan a tener la noche libre juntos? a. MDC1180, 240, 3062.
9. Los Cinemas I y II comienzan sus funciones a las 7:00 p.m. b. MDC15284, 1250, 12802.
La película en el Cinema I dura 75 min, mientras que la

© Lopez Mateos Editores. ISBN 978-607-95583-2-1, obra completa, versión electrónica, ISBN 978-607-95583-3-8, volumen 1, versión electrónica. Ejemplar asignado a: Helecto Villarroel gutierrez -
helecto@gmail.com. Fecha: 27 de octubre de 2014. Prohibida su modificación, copia o distribución.
Sección 5-5 Máximo divisor común y mínimo múltiplo común 327

15. Muestra que 181,345,913 y 11 son primos relativos. 19. Se pueden usar diagramas para mostrar los factores de
16. Laura y Miguel compraron una membresía especial de dos o más números. Traza diagramas para mostrar los
360 días en un club de tenis. Laura usará el club un día sí factores primos de cada uno de los siguientes conjuntos
y otro no, y Miguel usará el club cada tercer día. Ambos de tres números:
usaron el club el primer día. ¿Cuántos días ninguno de a. 12, 14, 70 b. 6, 8, 18
ellos lo usará, en los 360 días? 20. Halla todos los números naturales x tales que
17. Determina cuántas revoluciones completas debe dar el MDC125, x2 = 1 y 1 … x … 25.
engranaje 2 antes de que las flechas queden alineadas de 21. En álgebra a menudo es necesario factorizar una expre-
nuevo. sión lo más posible. Por ejemplo, a3b2 + a2b3 =
a2b21a + b2, donde no es posible mayor factorización sin
conocer los valores de a y b. Nota que a2b2 es el MDC de
a3b2 y a2b3. Factoriza en cada caso lo más posible.
a. 61x2 - y22 - 31x - y2 + 91 y - x2
b. 61x2 - y22 + 121x2 - y22 + 181 y2 - x22
Engranaje 1 Engranaje 2
22. Halla todos los valores de a y b para los cuales es verda-
dero que:
a. Si MDC1a, b2 = 1, entonces MDC1a2, b22 =
MDC1a, b32.
28 dientes
b. Si MMC1a, b, c2 = abc, entonces MDC1a, b, c2 = 1.
23. Halla el MDC y el MMC de cada caso. (No calcules los
48 dientes productos.)
a. pqr, qrs (donde p, q, r, s son números primos)
18. Determina cuántas revoluciones completas debe dar b. 210, 28
cada engranaje antes de que las flechas queden alineadas 24. Si hallas la suma de cualquier número de dos dígitos y del
de nuevo: número formado invirtiendo sus dígitos, ¿entre qué tres
números es siempre divisible el número resultante?
Engranaje 3
Engranaje 1
Engranaje 2

24 dientes
40 dientes
60 dientes

Conexiones matemáticas 5-5

Comunicación 6. Supón que MDC1a, b2 = MDC1b, c2 = 2. ¿Esto siem-


1. ¿Pueden tener dos números naturales un máximo múlti- pre implica que MDC1a, b, c2 = 2? Justifica tu respuesta.
plo común? Explica tu respuesta. 7. ¿Cómo puedes decir, a partir de la factorización en pri-
2. Explica a un estudiante de sexto grado la diferencia en- mos de dos números, si su MMC es igual al producto de
tre un divisor y un múltiplo. los números? Explica tu razonamiento.
3. ¿Es cierto que MDC1a, b, c2 # MMC1a, b, c2 = abc? Ex- 8. ¿Puede ser el MMC de dos números mayor que el pro-
plica tu respuesta. ducto de los números? Explica tu razonamiento.
4. Un terreno rectangular mide 558 m por 1212 m. Un pe- 9. Sean MDC1m, n2 = g y MMC1m, n2 = l. Beti conje-
rito necesita dividirlo en terrenos cuadrados del mismo ta- tura que MDC1m + n, l2 = g para todos los enteros m y
maño, lo más grandes posible, cuya longitud debe ser un n. Verifica la conjetura de Beti para tres diferentes pares
número entero no negativo de metros. ¿Cuál es el tamaño de enteros.
de cada cuadrado y cuántos terrenos se pueden crear? Ex-
Solución abierta
plica tu razonamiento.
5. Supón que MDC1a, b, c2 = 1. ¿Es necesariamente ver- 10. Inventa una situación que pueda resolverse hallando el
dadero que MDC1a, b2 = MDC1b, c2 = 1? Explica tu MDC y otra que pueda resolverse hallando el MMC.
razonamiento. Resuelve tus problemas y explica por qué estás seguro de
que tu enfoque es correcto.

© Lopez Mateos Editores. ISBN 978-607-95583-2-1, obra completa, versión electrónica, ISBN 978-607-95583-3-8, volumen 1, versión electrónica. Ejemplar asignado a: Helecto Villarroel gutierrez -
helecto@gmail.com. Fecha: 27 de octubre de 2014. Prohibida su modificación, copia o distribución.
328 Enteros y teoría de números

11. Halla tres pares de números para los cuales el MMC de 19. Diana asegura que ha hallado un atajo para obtener el
los números en cada par sea menor que el producto de MDC usando el algoritmo euclidiano. Ella dice que
los dos números. cuando el residuo es grande, usa un “residuo” negativo.
12. Describe infinidad de pares de números cuyo MDC sea Por ejemplo, para obtener MDC(2132, 534), divide
a. 2 2132 entre 534 y obtiene 2132 = 3 # 534 + 530, lo cual
b. 6 da un residuo de 530. En ese caso, ella escribe
c. 91 2132 = 4 # 534 - 4 y asegura que
Aprendizaje colectivo MDC12132, 5342 = MDC1 - 4, 5342
= MDC14, 5342
13. Cada miembro de tu grupo deberá examinar diferentes li- = 2 1porque 4ⱈ5342
bros de texto de nivel básico que cubran los temas de
MDC y MMC. Informa a la clase acerca de qué métodos ¿Es correcto este enfoque, y si es así, por qué?
usaron y cómo los usaron. Problemas de repaso
14. a. Analicen en el grupo si el algoritmo euclidiano para ha-
20. Halla dos números completos x y y tales que
llar el MDC de dos números debe enseñarse en la es-
xy = 1,000,000
cuela secundaria (¿a todos los estudiantes, a algunos?)
¿Por qué sí o por qué no? y que ni x ni y contengan dígitos cero.
b. Si deciden que deberá enseñarse en la escuela secun- 21. Llena cada espacio en blanco con un solo dígito que
daria, analicen cómo deberá enseñarse. Comuniquen haga que la afirmación correspondiente sea verdadera.
la decisión del grupo a todos los demás grupos. Halla todas las respuestas posibles.
a. 3 ƒ 83_51
Preguntas del salón de clase
b. 11 ƒ 8_691
15. Alba preguntó por qué no hablamos del mínimo divisor c. 23 ƒ 103_6
común y del máximo múltiplo común. ¿Cómo le res- 22. ¿Es 3111 un primo? Demuestra tu respuesta.
pondes? 23. Halla un número que tenga exactamente seis factores primos.
16. Una estudiante dice que para dos números naturales a y b, 24. Produce el menor número positivo que sea divisible en-
MDC(a, b) divide al MMC(a, b) y, por lo tanto, tre 2, 3, 4, 5, 6, 7, 8, 9, 10 y 11.
MDC1a, b2 6 MMC1a, b2. ¿Está la estudiante en lo co- 25. ¿Cuál es el mayor primo que debe usarse para determi-
rrecto? ¿Por qué sí o por qué no? nar si 2089 es primo?
17. Un estudiante pregunta acerca de la relación entre el mí-
Preguntas del National Assessment of Educational Progress
nimo múltiplo común y el mínimo denominador común.
(NAEP) (Evaluación Nacional del Progreso Educativo)
¿Qué le respondes?
18. Una estudiante quiere saber cuántos enteros entre 1 y El mínimo múltiplo común de 8, 12 y un tercer número es
10,000 inclusive son múltiplos de 3 o múltiplos de 5. Ella 120. ¿Cuál de los siguientes podría ser el tercer número?
se pregunta si es correcto hallar el número de esos ente- a. 15 b. 16 c. 24 d. 32 e. 48
ros que son múltiplos de 3 y sumarles el número de los
que son múltiplos de 5. ¿Cómo le respondes? NAEP, Grado 8, 1990

RINCÓN DE LA TECNOLOGÍA
1. Usa una hoja de cálculo para generar los primeros 50 múltiplos de 3 y los primeros 50 múltiplos de 4.
Describe la intersección de los dos conjuntos.
2. Usa una hoja de cálculo para hallar los factores de 2486. ¿Qué tan abajo necesitas copiar la fórmula
para tener la certeza de que hallaste todos los divisores?

A B
1 1 = 2486>A1
2 2
3 3
3. Forma una hoja de cálculo con cuatro columnas:
Columna A—Los múltiplos de 6
Columna B—Los múltiplos de 9
Columna C—Los múltiplos de 12
Columna D—Los múltiplos de 15
a. ¿Cuál es el menor número que aparece en las cuatro columnas?
b. Explica cómo hallar este número sin usar una hoja de cálculo.

© Lopez Mateos Editores. ISBN 978-607-95583-2-1, obra completa, versión electrónica, ISBN 978-607-95583-3-8, volumen 1, versión electrónica. Ejemplar asignado a: Helecto Villarroel gutierrez -
helecto@gmail.com. Fecha: 27 de octubre de 2014. Prohibida su modificación, copia o distribución.
Sección 5-6 Aritmética de reloj y modular 329

ROMPECABEZAS Para cualquier rectángulo de n * m tal que MDC1n, m2 = 1, halla una regla para
determinar el número de cuadrados unitarios 11 * 12 por los que pasa una diagonal. Por ejemplo, en la fi-
gura 5-32 la diagonal pasa por 8 y 6 cuadrados unitarios, respectivamente.

(a) (b)

Figura 5-32

*5-6 Aritmética de reloj y modular

En esta sección investigamos la aritmética del reloj. Considera lo siguiente:


a. Una receta de tu médico te dice que tomes una pastilla cada 8 h. Si tomas la primera pas-
tilla a las 7:00 a.m., ¿cuándo deberás tomar las dos pastillas siguientes?
b. Supón que sigues una receta para hacer sopa de frijoles que te pide que dejes en remojo
los frijoles por 12 h. Si los pones a remojar a las 8:00 p.m., ¿cuándo los debes sacar?
c. El odómetro de un carro marca hasta 99,999 millas y después comienza contando desde 0.
Si el odómetro muestra 99,124 millas, ¿qué mostrará después de un viaje de 2,116 millas?
Algunas de estas situaciones tienen que ver con la habilidad para resolver problemas de arit-
mética usando relojes. La mayoría de las personas pueden resolver estos problemas sin pensar
en lo que están haciendo. Es posible usar el reloj de la figura 5-33 para determinar que 8 h
después de las 7:00 a.m. son las 3:00 p.m., y que 8 h después son las 11:00 p.m. Asimismo, 12 h
después de las 8:00 p.m. son las 8:00 a.m. Podríamos escribir estas sumas en el reloj como
7 䊝 8 = 3, 3 䊝 8 = 11, 8 䊝 12 = 8
donde 䊝 indica suma en un reloj de 12 horas.

11 12 1
10 2
9 3
8 4
7 6 5

Figura 5-33

Probablemente notaste el papel especial del 12 cuando obtuviste 8 䊝 12 = 8. En la arit-


mética del reloj de 12 h, el 12 actúa como un 0 si estuvieras sumando en el conjunto de los
números completos. En la tabla 5-4 se muestra una tabla de sumar para el sistema finito ba-
sado en el reloj.

© Lopez Mateos Editores. ISBN 978-607-95583-2-1, obra completa, versión electrónica, ISBN 978-607-95583-3-8, volumen 1, versión electrónica. Ejemplar asignado a: Helecto Villarroel gutierrez -
helecto@gmail.com. Fecha: 27 de octubre de 2014. Prohibida su modificación, copia o distribución.
330 Enteros y teoría de números

Tabla 5-4
ⴙ 12 1 2 3 4 5 6 7 8 9 10 11

12 12 1 2 3 4 5 6 7 8 9 10 11
1 1 2 3 4 5 6 7 8 9 10 11 12
2 2 3 4 5 6 7 8 9 10 11 12 1
3 3 4 5 6 7 8 9 10 11 12 1 2
4 4 5 6 7 8 9 10 11 12 1 2 3
5 5 6 7 8 9 10 11 12 1 2 3 4
6 6 7 8 9 10 11 12 1 2 3 4 5
7 7 8 9 10 11 12 1 2 3 4 5 6
8 8 9 10 11 12 1 2 3 4 5 6 7
9 9 10 11 12 1 2 3 4 5 6 7 8
10 10 11 12 1 2 3 4 5 6 7 8 9
11 11 12 1 2 3 4 5 6 7 8 9 10

AHORA INTENTA ÉSTE 5-18 Examina la tabla 5-4 para determinar si las propiedades siguientes son
válidas para 䊝 en el conjunto de números de la tabla:
a. Propiedad conmutativa de la suma
b. Propiedad de la identidad de la suma
c. Propiedad del inverso de la suma

Cuando permitimos que se sumen otros números además de los que están en el reloj de
12 horas, como 8 䊝 24 = 8, hallamos que los números como 24, 36, 48, Á actúan como
el 12 (o el 0). Asimismo, los números 13, 25, 37, Á actúan como el número 1. De manera
análoga, podemos generar clases de números que actúan como cada uno de los números en
el reloj de 12 horas. Los miembros de cada una de las clases difieren en múltiplos de 12. En
consecuencia, para efectuar sumas en un reloj de 12 horas realizamos la suma común, divi-
dimos entre 12 y el residuo es la respuesta. Por ejemplo, podemos hallar 11 䊝 8 y 8 䊝 12
como sigue:
11 + 8 = 19. A continuación divide 19 , 12. El cociente es 1 con residuo de 7. Por lo
tanto, 11 䊝 8 = 7.
8 + 12 = 20. A continuación divide 20 , 12. El cociente es 1 con residuo de 8. Por lo
tanto, 8 䊝 12 = 8.
La multiplicación en el reloj se puede definir usando la suma repetida, como en los nú-
meros completos. Por ejemplo, 2 䊟 8 = 8 䊝 8 = 4, donde 䊟 denota la multiplicación en
el reloj. Análogamente, 3 䊟 5 = 15 䊝 52 䊝 5 = 10 䊝 5 = 3. También podemos encon-
trar 2 䊟 8 pues 12 # 82 , 12 tiene residuo 4, así que 2 䊟 8 = 4. Asimismo, podemos hallar
3 䊟 5 pues 13 # 52 , 12 tiene residuo 3, es decir, 3 䊟 5 = 3. Esto conduce a la siguiente
definición.

Definición
Sumas y productos en relojes de 12 horas Para calcular una suma o un producto en la arit-
mética del reloj de 12 horas, efectuamos la operación como en los números completos, dividimos
entre 12 y la respuesta será el residuo.

© Lopez Mateos Editores. ISBN 978-607-95583-2-1, obra completa, versión electrónica, ISBN 978-607-95583-3-8, volumen 1, versión electrónica. Ejemplar asignado a: Helecto Villarroel gutierrez -
helecto@gmail.com. Fecha: 27 de octubre de 2014. Prohibida su modificación, copia o distribución.
Sección 5-6 Aritmética de reloj y modular 331

Para efectuar otras operaciones en el reloj, como 2 䊞 9, donde 䊞 denota la resta en el reloj,
podemos interpretarlo como la hora que marca 9 horas antes de las 2. Al contar hacia atrás (en
el sentido contrario al que giran las manecillas del reloj) 9 unidades desde el 2, vemos que
2 䊞 9 = 5. Si la resta en el reloj se define en términos de la suma, tenemos que 2 䊞 9 = x si,
y sólo si, 2 = 9 䊝 x. En consecuencia, x = 5.
La división en el reloj se puede definir en términos de la multiplicación. Por ejemplo, 8 s4
5 = x, donde s 4 denota la división en el reloj, si, y sólo si, 8 = 5 䊟 x para un único x en el
conjunto 51, 2, 3, Á , 126. Como 5 䊟 4 = 8 y 8 es único, tenemos entonces que 8 s 4 5 = 4.

Ejemplo 5-32 Efectúa cada uno de los cálculos en un reloj de 12 horas:










a. 8 䊝 8 b. 4 䊞 12
c. 4 䊞 4 d. 4 䊞 8

Solución a. 18 + 82 , 12 tiene residuo 4. Por lo tanto, 8 䊝 8 = 4.


b. 4 䊞 12 = 4, pues al contar 12 h hacia adelante o hacia atrás se llega a la posi-
ción original.
c. 4 䊞 4 = 12. Esto deberá quedar claro viendo el reloj, pero también podría
obtenerse usando la definición de resta en términos de suma.
d. 4 䊞 8 = 8 pues 8 䊝 8 = 4.

Ejemplo 5-33 Efectúa las operaciones siguientes en un reloj de 12 horas, de ser posible:








a. 3 䊟 11 b. 2 s
47
c. 3 s
42 d. 5 s
4 12

Solución a. 3 䊟 11 = 111 䊝 112 䊝 11 = 10 䊝 11 = 9 ó 13 # 112 , 12 tiene residuo 9.


Por lo tanto, 3 䊟 11 = 9.
4 7 = x si, y sólo si, 2 = 7 䊟 x y x es único. En consecuencia, x = 2.
b. 2 s
4 2 = x si, y sólo si, 3 = 2 䊟 x y x es único. Multiplicando cada uno de los
c. 3 s
números 1, 2, 3, 4, Á , 12 por 2 se muestra que ninguna de las multiplica-
ciones produce 3. Así, la ecuación 3 = 2 䊟 x no tiene solución y, en conse-
cuencia, 3 s4 2 no está definido.
4 12 = x si, y sólo si, 5 = 12 䊟 x y x es único. Sin embargo, 12 䊟 x = 12
d. 5 s
para todo x en el conjunto 51, 2, 3, 4, Á , 126. Así, 5 = 12 䊟 x no tiene solu-
ción en el reloj y, por lo tanto, 5 s
4 12 no está definido.

Sumar o restar 12 en un reloj de 12 horas da el mismo resultado. Así, 12 se comporta


como lo hace 0 en la suma o resta en los enteros y es la identidad aditiva para la suma en el
reloj de 12 horas. Análogamente, en un reloj de 5 horas el 5 se comporta como lo hace el 0.
La suma, resta y multiplicación en un reloj de 12 horas se pueden efectuar para cuales-
quier dos números pero, como se mostró en el ejemplo 5-33(d), no se pueden efectuar to-
das las divisiones. La división entre 12, la identidad aditiva, nunca puede efectuarse o no
tiene sentido en un reloj de 12 horas pues no conduce a una respuesta única. Sin embargo,
hay relojes en los cuales se pueden efectuar todas las divisiones excepto entre la correspon-
diente identidad aditiva. Uno de dichos relojes es de 5 horas y se muestra en la figura 5-34.

© Lopez Mateos Editores. ISBN 978-607-95583-2-1, obra completa, versión electrónica, ISBN 978-607-95583-3-8, volumen 1, versión electrónica. Ejemplar asignado a: Helecto Villarroel gutierrez -
helecto@gmail.com. Fecha: 27 de octubre de 2014. Prohibida su modificación, copia o distribución.
332 Enteros y teoría de números

Tabla 5-5 5
(a) 4 1
䊝 1 2 3 4 5
3 2
1 2 3 4 5 1
2 3 4 5 1 2
3 4 5 1 2 3 Figura 5-34
4 5 1 2 3 4
5 1 2 3 4 5 En este reloj, 3 䊝 4 = 2, 2 䊞 3 = 4, 2 䊟 4 = 3 y 3 s 4 4 = 2. Como sumar 5 a
cada número produce el número original, 5 es la identidad aditiva para este reloj de
(b)
5 horas, como se ve en la tabla 5-5(a). En consecuencia, puedes sospechar que la di-
䊟 1 2 3 4 5 visión entre 5 no es posible en un reloj de 5 horas. Para determinar cuáles divisiones
1 1 2 3 4 5 son posibles, considera la tabla 5-5(b), una tabla de multiplicar para la aritmética en
2 2 4 1 3 5 el reloj de 5 horas. Para hallar 1 s
4 2, escribimos 1 s 4 2 = x, que es equivalente a
3 3 1 4 2 5 1 = 2 䊟 x. El segundo renglón de la tabla 5-5(b) muestra que 2 䊟 1 = 2,
4 4 3 2 1 5 2 䊟 2 = 4, 2 䊟 3 = 1, 2 䊟 4 = 3 y 2 䊟 5 = 5. La solución única de 1 = 2 䊟 x
5 5 5 5 5 5 es x = 3, de modo que 1 s 4 2 = 3. La información dada en el segundo renglón de
la tabla puede usarse para determinar las divisiones siguientes:
2s
4 2 = 1 porque 2 = 2䊟1
3s
4 2 = 4 porque 3 = 2䊟4
4s
4 2 = 2 porque 4 = 2䊟2
5s
4 2 = 5 porque 5 = 2䊟5
Como todo elemento está presente en el segundo renglón, siempre es posible la división
entre 2. De manera análoga la división entre todos los demás números, excepto el 5, siem-
pre es posible. En el conjunto de problemas se te pide efectuar aritmética en diferentes re-
lojes e investigar para qué relojes pueden efectuarse todos los cálculos, excepto la división
entre la identidad aditiva.

Aritmética modular
ABRIL
D L M Mi J V S
Muchos de los conceptos para la aritmética en un reloj se pueden usar para trabajar proble-
1 2 3 4 5 6 7 mas que traten con un calendario. En el calendario de la figura 5-35, los cinco domingos
8 9 10 11 12 13 14 tienen fechas 1, 8, 15, 22 y 29. Cualesquier dos de estas fechas para el domingo difieren en
15 16 17 18 19 20 21 un múltiplo de 7. La misma propiedad es verdadera para cualquier otro día de la semana.
22 23 24 25 26 27 28 Por ejemplo, el segundo y el trigésimo días caen en el mismo día, pues 30 - 2 = 28 y 28 es
29 30 un múltiplo de 7. Decimos que 30 es congruente con 2, módulo 7, y lo escribimos 30 K 2
(mod 7). De manera análoga, como 18 y 6 difieren en un múltiplo de 12, escribimos 18 K 6
Figura 5-35 (mod 12). Esto se generaliza en la siguiente definición.

Definición de congruencia modular


Para enteros a y b, a es congruente con b módulo m, que se escribe a K b (mod m), si, y sólo
si, a - b es un múltiplo de m, donde m es un entero positivo mayor que 1.

O B S E R VA C I Ó N Esta definición podría escribirse como a K b (mod m) si, y sólo si,


m ƒ 1a - b2, donde m es un número entero positivo mayor que 1.

© Lopez Mateos Editores. ISBN 978-607-95583-2-1, obra completa, versión electrónica, ISBN 978-607-95583-3-8, volumen 1, versión electrónica. Ejemplar asignado a: Helecto Villarroel gutierrez -
helecto@gmail.com. Fecha: 27 de octubre de 2014. Prohibida su modificación, copia o distribución.
Sección 5-6 Aritmética de reloj y modular 333

Nota que 18 y 25 son congruentes módulo 7 y que cada número deja el mismo residuo,
4, al dividirlo entre 7. En efecto, 18 = 2 # 7 + 4 y 25 = 3 # 7 + 4. En general, tenemos la
siguiente propiedad: Dos números completos son congruentes módulo m si, y sólo si, sus residuos al
dividirlos entre m son iguales.

Ejemplo 5-34 Di por qué cada uno de los casos siguientes es verdadero:









a. 23 K 3 (mod 10) b. 23 K 3 (mod 4) c. 23 [ 3 (mod 7)
d. 10 K - 1 (mod 11) e. m K 0 (mod m)

Solución a. 23 K 3 (mod 10) porque 23 - 3 es un múltiplo de 10 o porque 23 y 3 dejan


el mismo residuo, 3, al dividirlos entre 10.
b. 23 K 3 (mod 4) porque 23 - 3 es un múltiplo de 4.
c. 23 [ 3 (mod 7) porque 23 - 3 no es un múltiplo de 7.
d. 10 K - 1 (mod 11) porque 10 - 1 - 12 = 11 es un múltiplo de 11.
e. m K 0 (mod m) porque m - 0 es un múltiplo de m o porque m y 0 dejan el
mismo residuo, 0, al dividirlos entre m.

O B S E R VA C I Ó N El ejemplo 5-34(e) muestra que m se comporta como 0 módulo m.


Esto también es evidente para m = 12 en la tabla 5-4 y para m = 5 en la tabla 5-5.

Ejemplo 5-35 Halla todos los enteros x tales que x K 1 (mod 10).








Solución x K 1 (mod 10) si, y sólo si, x - 1 = 10k, donde k es cualquier entero. En consecuen-
cia, x = 10k + 1. Al hacer k = 0, 1, 2, 3, Á se produce la sucesión 1, 11, 21, 31, 41, Á . Asi-
mismo, al hacer k = - 1, - 2, - 3, - 4, Á se producen los números negativos - 9, - 19, - 29,
- 39, Á . Las dos sucesiones se pueden combinar para dar el conjunto solución

5 Á , - 39, - 29, - 19, - 9, 1, 11, 21, 31, 41, 51, Á 6


Se puede usar la tecla INT , en una calculadora para trabajar con aritmética modular.
Si presionamos la siguiente sucesión de teclas, vemos que 4325 K 5 (mod 9) pues cuando se
divide 4325 entre 9 el residuo es 5:
4 3 2 5 INT , 9 =
la pantalla muestra un residuo de 5.

Ejemplo 5-36 Hilda firmó un pagaré que vencerá en 90 días. Está preocupada de que venza un fin de se-








mana. Ella firmó el pagaré un lunes. ¿En qué día de la semana vence?

Solución Como 90 = 7 # 12 + 6, sabemos que 90 K 6 (mod 7). En una calculadora con


fracciones puedes teclear 9 0 INT , 7 = , y se despliega en pantalla un cociente 12
con residuo 6. Por lo tanto, el pagaré se vencerá 12 semanas y 6 días después del lunes, que
es domingo.

© Lopez Mateos Editores. ISBN 978-607-95583-2-1, obra completa, versión electrónica, ISBN 978-607-95583-3-8, volumen 1, versión electrónica. Ejemplar asignado a: Helecto Villarroel gutierrez -
helecto@gmail.com. Fecha: 27 de octubre de 2014. Prohibida su modificación, copia o distribución.
334 Enteros y teoría de números

Ejemplo 5-37 a. Si hoy es lunes 14 de octubre, ¿qué día de la semana será 14 de octubre el próximo año









si es que el próximo año no es bisiesto?
b. Si Navidad cae en jueves este año, ¿en qué día de la semana caerá Navidad el próximo
año si es un año bisiesto?

Solución a. Como el próximo año no es bisiesto, tenemos 365 días en el año. Como 365 =
52 # 7 + 1, tenemos 365 K 1 (mod 7). Así, 365 días después del 14 de octubre
serán 52 semanas más un día. Así, el 14 de octubre será martes.
b. Hay 366 días en un año bisiesto, y 366 K 2 (mod 7). Así, Navidad caerá 2 días
después del jueves, en sábado.

Evaluación 5-6A

1. El doctor Legorreta le recetó una medicina a Camila. Se 6. En un reloj de 5 horas, halla lo siguiente:
supone que ella debe tomar una dosis cada 6 h. Si toma a. El inverso aditivo de 2 b. El inverso aditivo de 3
su primera dosis a las 8:00 a.m., ¿a qué hora deberá tomar c. 1 - 22 䊝 1 - 22 d. - 12 䊝 22
su siguiente dosis? e. 1 - 22 䊞 1 - 32 f. 1 - 22 䊟 1 - 22
2. Efectúa cada una de las operaciones en un reloj de 5 horas, 7. a. Si el 23 de abril cae en martes, ¿cuáles son las fechas
de ser posible: de los otros martes de abril de ese año?
a. 7 䊝 8 b. 4 䊝 10 c. 3 䊞 9 b. Si el 2 de julio cae en martes, lista las fechas de todos
d. 4 䊞 8 e. 3 䊟 9 f. 2 䊟 2 los miércoles de julio.
g. 1 s43 h. 2 s45 c. Si el 3 de septiembre cae en lunes, ¿en qué día de la
3. Efectúa cada una de las operaciones en un reloj de 5 horas: semana caerá el próximo año si es un año bisiesto?
a. 3 䊝 4 b. 3 䊝 3 c. 3 䊟 4 8. Llena cada uno de los siguientes espacios en blanco de
modo que la respuesta sea no negativa y el número sea el
d. 1 䊟 4 e. 4 䊟 4 f. 2 䊟 3
menor posible:
g. 3 s
44 h. 1 s
44
a. 29 K _____ (mod 5)
4. a. Construye una tabla de sumar para un reloj de 9 horas.
b. 3498 K _____ (mod 3)
b. Usando la tabla de sumar en (a), halla 5 䊞 6 y 2 䊞 5.
c. 3498 K _____ (mod 11)
c. Usando la definición de resta en términos de suma,
d. - 23 K _____ (mod 10)
muestra que la suma siempre se puede efectuar en un
9. a. Halla todas las x tales que x K 0 (mod 2).
reloj de 9 horas.
b. Halla todas las x tales que x K 1 (mod 2).
5. a. Construye una tabla de multiplicación para un reloj
c. Halla todas las x tales que x K 3 (mod 5).
de 9 horas.
10. Un reloj nuevo comienza a caminar a las 10:00 p.m. del
b. Usa la tabla de multiplicación en (a) para hallar 3 s
45
domingo. Si el reloj continúa sin parar, ¿en qué día y a
y4s 4 6.
qué hora, redondeada a la hora más cercana, llegará el
c. Usa la tabla de multiplicación para hallar si es posible
reloj al segundo número 100,000?
dividir siempre entre números diferentes de 9.
11. Si continúa el siguiente patrón,
CLOCK CLOCK CLOCK CLOCK Á
¿cuál será la letra que ocupe el lugar 101?

Evaluación 5-6B

1. La familia González salió de viaje en carro a las 6:00 a.m. d. 5 䊞 8 e. 4 䊟 9 f. 3 䊟 3


Viajaron exactamente 15 h. ¿A qué hora llegaron? g. 2 s
43 h. 4 s46
2. Efectúa cada una de las operaciones en un reloj de 12 3. Efectúa cada una de las operaciones en un reloj de 5 horas:
horas, de ser posible: a. 4 䊝 5 b. 2 䊝 2 c. 4 䊟 4
a. 6 䊝 6 b. 5 䊝 11 c. 4 䊞 6 d. 1 䊟 3 e. 3 䊟 3 f. 5 䊟 3
g. 2 s
44 h. 4 s44

© Lopez Mateos Editores. ISBN 978-607-95583-2-1, obra completa, versión electrónica, ISBN 978-607-95583-3-8, volumen 1, versión electrónica. Ejemplar asignado a: Helecto Villarroel gutierrez -
helecto@gmail.com. Fecha: 27 de octubre de 2014. Prohibida su modificación, copia o distribución.
Sección 5-6 Aritmética de reloj y modular 335

4. a. Construye una tabla de sumar para un reloj de 7 horas. mana caerá el próximo año si no es un año bisiesto?
b. Usando la tabla de sumar en (a), halla 5 䊞 6 y 2 䊞 5. c. ¿Es cierto que los días número 125 y número 256 del
c. Usando la definición de resta en términos de suma, año caen en el mismo día de la semana? Explica por qué.
muestra que la resta siempre se puede efectuar en un 8. Llena cada uno de los siguientes espacios en blanco de
reloj de 7 horas. modo que la respuesta sea no negativa y el número sea el
5. a. Construye una tabla de multiplicación para un reloj menor posible:
de 7 horas. a. 29 K _____ (mod 3)
b. Usa la tabla de multiplicación en (a) para hallar 3 s
45 b. 3498 K _____ (mod 5)
y4s 4 6. c. 3498 K _____ (mod 10)
c. Usa la tabla de multiplicación para hallar si es posible d. - 23 K _____ (mod 11)
dividir siempre entre números diferentes de 7. 9. a. Halla todas las x tales que x K 0 (mod 3).
6. En un reloj de 12 horas, halla lo siguiente: b. Halla todas las x tales que x K 1 (mod 3).
a. El inverso aditivo de 2 b. El inverso aditivo de 3 c. Halla todas las x tales que x K 3 (mod 7).
c. 1 - 22 䊝 1 - 32 d. - 12 䊝 32 10. Continúa un posible patrón y lista los siguientes cuatro
e. 1 22 䊞 1 32
- - f. 1 - 22 䊟 1 - 32 términos de cada sucesión en aritmética del reloj.
7. a. Si el 8 de abril cae en viernes, ¿cuáles son las fechas a. 3, 8, 1, 6, 11, 4, 9, 2, 7, Á
de los otros viernes de abril? b. 3, 8, 13, 4, 9, 14, 5, 10, 1, Á
b. Si el 4 de julio cae en martes, ¿en qué día de la se-

Conexiones matemáticas 5-6

Comunicación b. Compara tus resultados. ¿En cuáles de los relojes de


1. Explica o halla lo siguiente: (a) siempre se pueden realizar divisiones entre núme-
a. Un número congruente módulo 10 con el número ros diferentes de la identidad aditiva?
formado por su último dígito c. ¿En qué forma las tablas de multiplicación de relojes
b. El último dígito de 2180 - 1 para los cuales siempre se puede efectuar la división
c. Un número congruente módulo 100 con el número (excepto entre una identidad aditiva) difieren de las ta-
formado por sus últimos dos dígitos blas de multiplicación de relojes para los cuales la divi-
2. a. Para todo a, b, c y d, explica por qué abcd K a + b + sión no siempre tiene sentido?
c + d (mod 9).
b. Si abcdcinco K a + b + c + d (mod m), ¿cuál es m? Preguntas del salón de clase
Explica tu razonamiento.
5. Daniel trata de ver cómo funcionan fracciones como 14 en
Solución abierta un sistema de reloj de 5 horas. Dice que mostró que 1 s
44
3. En un reloj definimos el inverso aditivo de a de la misma es mayor que 3. Él quiere saber si eso puede ser correcto.
manera en que definimos el inverso aditivo para los ente- ¿Cómo le respondes?
ros. Teniendo en cuenta esta definición, lista algunas ana- 6. Alina quiere saber qué elementos de un sistema de reloj
logías y algunas diferencias entre el sistema numérico en el de 5 horas tienen inverso multiplicativo. ¿Qué le dices?
reloj y el conjunto de los enteros. Justifica tus respuestas. 7. Zeneide asegura que en el reloj de 5 horas mostrado en la
figura 5-34 no hay 0 y por lo tanto no puede haber identi-
Aprendizaje colectivo dad aditiva. ¿Cómo le respondes?
4. a. Que miembros de tu grupo construyan las tablas de
multiplicación para relojes de 3 horas, 4 horas, 6 ho-
ras y 11 horas.

ROMPECABEZAS ¿Cuántos primos hay en la sucesión siguiente?


9, 98, 987, 9876, Á , 987654321, 9876543219, 98765432198, Á

© Lopez Mateos Editores. ISBN 978-607-95583-2-1, obra completa, versión electrónica, ISBN 978-607-95583-3-8, volumen 1, versión electrónica. Ejemplar asignado a: Helecto Villarroel gutierrez -
helecto@gmail.com. Fecha: 27 de octubre de 2014. Prohibida su modificación, copia o distribución.
336 Enteros y teoría de números

Sugerencia para resolver el problema preliminar


La fórmula de la diferencia de cuadrados, junto con el trabajo realizado con el pro-
blema de Gauss del capítulo 1, te ayudará a resolver este problema.

Resumen del capítulo

I. Conceptos básicos de los enteros B. Propiedad de multiplicación por cero de los en-
A. El conjunto de los enteros, E, es 5 Á , - 3, - 2, teros: Para cualquier entero a, a # 0 = 0 = 0 # a.
- 1, 0, 1, 2, 3, Á 6. C. Para todos los enteros a, b y c,
B. La distancia de cualquier entero a 0 es el valor 1. - 1 - a2 = a.
absoluto del entero. El valor absoluto de un 2. a - 1b - c2 = a - b + c.
entero x se denota con ƒ x ƒ . Si x Ú 0, entonces 3. 1a + b21a - b2 = a2 - b2 (fórmula de di-
ƒ x ƒ = x y si x 6 0, entonces ƒ x ƒ = - x. ferencia de cuadrados).
C. Operaciones con enteros III. Divisibilidad
1. Suma: Para cualesquier enteros a y b, A. Si a y b son cualesquier enteros, entonces b di-
- a + - b = - 1a + b2 vide a a, que se denota con b ƒ a, si, y sólo si,
existe un único entero c tal que a = cb.
2. Resta
B. Los siguientes son teoremas básicos de divisibi-
a. Para todos los enteros a y b, a - b = n
lidad para enteros a, b y d:
si, y sólo si, a = b + n.
1. Si d ƒ a y k es cualquier entero, entonces d ƒ ka.
2. Si d ƒ a y d ƒ b, entonces d ƒ 1a + b2 y d ƒ 1a - b2.
b. Para todos los enteros a y b, a - b =
a + - b.
3. Si d ƒ a y dⱈb, entonces dⱈ1a + b2 y
3. Multiplicación: Para todos los enteros a y b,
a. 1 - a21 - b2 = ab.
dⱈ1a - b2.
b. 1 - a2b = b1 - a2 = - 1ab2.
C. Criterios de divisibilidad
1. Un entero es divisible entre 2, 5 ó 10 si, y
4. División: Si a y b son enteros cualesquiera,
sólo si, su dígito de las unidades es divisible
con b Z 0, entonces a , b es el único en-
entre 2, 5 ó 10, respectivamente.
tero c, si existe, tal que a = bc.
2. Un entero es divisible entre 4 si, y sólo si,
5. Orden de las operaciones: Cuando la suma,
los dos últimos dígitos del entero represen-
resta, multiplicación y división aparecen sin
tan un número divisible entre 4.
paréntesis, primero se efectúan las multiplica-
3. Un entero es divisible entre 8 si, y sólo si,
ciones y divisiones en el orden en que aparecen
los últimos tres dígitos del entero represen-
de izquierda a derecha, y después se efectúan
tan un número divisible entre 8.
las sumas y restas en el orden en que aparecen
4. Un entero es divisible entre 3 ó entre 9 si, y
de izquierda a derecha. Cualquier operación
sólo si, la suma de sus dígitos es divisible
entre paréntesis se realiza primero.
entre 3 ó entre 9, respectivamente.
II. El sistema de los enteros
5. Un entero es divisible entre 11 si, y sólo si,
A. El conjunto de los enteros, junto con las opera-
la suma de los dígitos en los lugares de las
ciones de suma y multiplicación, satisfacen las
potencias pares de 10, menos la suma de los
propiedades siguientes:
dígitos en los lugares de las potencias impa-
res de 10, es divisible entre 11.
Propiedad + * 6. Un entero es divisible entre 6 si, y sólo si, el
entero es divisible entre 2 y 3.
Cerradura Sí Sí IV. Números primos y compuestos
Conmutatividad Sí Sí A. Los enteros positivos que tienen exactamente dos
Asociatividad Sí Sí divisores positivos son primos. Los enteros mayo-
Identidad Sí, 0 Sí, 1 res que 1 y que no son primos son compuestos.
Inverso Sí No B. Teorema fundamental de la aritmética: Todo
número compuesto tiene una, y sólo una, facto-
Propiedad distributiva de la rización en primos, aparte de la variación en el
multiplicación sobre la suma orden de los factores primos.

© Lopez Mateos Editores. ISBN 978-607-95583-2-1, obra completa, versión electrónica, ISBN 978-607-95583-3-8, volumen 1, versión electrónica. Ejemplar asignado a: Helecto Villarroel gutierrez -
helecto@gmail.com. Fecha: 27 de octubre de 2014. Prohibida su modificación, copia o distribución.
Revisión del capítulo 337

C. Criterio para determinar si un número dado n C. El mínimo múltiplo común (MMC) de dos o
es primo: Si n no es divisible entre ningún primo p más números naturales es el menor múltiplo
tal que p2 … n, entonces n es primo. positivo que tienen los números en común.
D. Si la factorización en primos de un número es D. MDC1a, b2 # MMC1a, b2 = ab.
pnqm, donde p y q son primos, entonces el nú- E. Si MDC1a, b2 = 1, entonces a y b son primos
mero de divisores de n es 1n + 121m + 12. relativos.
V. Máximo divisor común y mínimo múltiplo común * VI. Aritmética modular
A. El máximo divisor común (MDC) de dos o más A. Para cualesquier enteros a y b, a es congruente
números naturales es el máximo divisor, o factor, con b módulo m si, y sólo si, a - b es un múlti-
que tienen los números en común. plo de m, donde m es un entero positivo mayor
B. Algoritmo euclidiano: Si a y b son enteros positi- que 1.
vos y a Ú b, entonces MDC1a, b2 = MDC1b, r2, B. Dos enteros son congruentes módulo m si, y sólo
donde r es el residuo cuando se divide a entre b. El si, al dividirlos entre m sus residuos son iguales.
procedimiento de hallar el MDC de dos números a
y b usando de manera repetida este resultado es el
algoritmo euclidiano.

Revisión del capítulo

1. Halla el inverso aditivo de cada caso: c. 2x - 11 - x2


a. 3 b. - a c. - 2 + 3 d. 1 - x22 + x2
e. 1 - x23 + x3
d. x + y -
e. x + y f. - x - y
g. 1 22
- 5 -
h. 2 5
f. 1 - 3 - x213 + x2
2. Efectúa cada una de las operaciones siguientes: g. 1 - 2 - x21 - 2 + x2
a. 1 - 2 + - 82 + 3 b. - 2 - 1 - 52 + 5 7. Factoriza cada una de las expresiones siguientes y
c. - 31 - 22 + 2 d. - 31 - 5 + 52 después simplifica, si es posible:
e. 40 , 1 52
- - f. 1 - 25 , 521 - 32 a. x - 3x
3. Para cada uno de los casos siguientes, halla todos los b. x2 + x
valores enteros de x (si hay alguno) que hagan ver- c. x2 - 36
daderas las ecuaciones dadas: d. 81y4 - 16x4
a. - x + 3 = 0 e. 5 + 5x
b. - 2x = 10 f. 1x - y21x + 12 - 1x - y2
c. 0 , 1 - x2 = 0 8. Clasifica cada uno de los casos siguientes como ver-
d. - x , 0 = - 1 dadero o falso (todas las letras representan enteros).
e. 3x - 1 = - 124 Justifica tus respuestas.
f. - 2x + 3x = x a. ƒ x ƒ siempre es positivo.
4. Usa un enfoque de patrones para explicar por qué b. Para todo x y y, ƒ x + y ƒ = ƒ x ƒ + ƒ y ƒ .
1 - 221 - 32 = 6. c. Si a 6 - b, entonces a 6 0.
5. En cada uno de los siguientes modelos de fichas, se d. Para todo x y y, 1x - y22 = 1y - x22.
eliminan las fichas encerradas con una línea puntea - 9. Exhibe un contraejemplo para refutar cada una de las
da. Escribe el correspondiente problema con su so- siguientes propiedades en el conjunto de los enteros:
lución. a. Propiedad conmutativa de la división
b. Propiedad asociativa de la resta
c. Propiedad de la cerradura para la división
d. Propiedad distributiva de la división sobre la resta
10. Despeja x en cada uno de los casos siguientes,
donde x es un entero:
a. x + 3 = - x - 17
(a) (b) b. 2x = - 2100
c. 210x = 299
6. Simplifica cada una de las expresiones siguientes: d. - x = x
a. - 1x e. ƒ - x ƒ = 3
b. 1 - 121x - y2 f. ƒ x ƒ = - x

© Lopez Mateos Editores. ISBN 978-607-95583-2-1, obra completa, versión electrónica, ISBN 978-607-95583-3-8, volumen 1, versión electrónica. Ejemplar asignado a: Helecto Villarroel gutierrez -
helecto@gmail.com. Fecha: 27 de octubre de 2014. Prohibida su modificación, copia o distribución.
338 Enteros y teoría de números

g. ƒ x ƒ 7 3 yor verificación asegura que 353 es primo. Explica


h. 1x - 122 = 100 por qué la estudiante está en lo correcto.
11. Escribe los primeros seis términos de cada una de 24. Halla el MDC para cada uno de los casos siguientes:
las sucesiones cuyo término n-ésimo sea a. 24 y 52
a. 1 - 12n b. 5767 y 4453

b. 31 - 12n + 14
n 25. Halla el MMC para cada uno de los casos siguientes:
2 a. 23 # 52 # 73, 2 # 53 # 72 # 13 y 24 # 5 # 74 # 29
c. 1 - 22n b. 278 y 279
d. - 2 - 3n 26. Construye un número que tenga exactamente cinco
12. En cada parte del problema 11, si una sucesión es divisores positivos. Explica tu construcción.
aritmética, halla su diferencia; y si es geométrica, 27. Halla todos los divisores positivos de 144.
halla su razón. 28. Halla la factorización en primos de lo siguiente:
13. Clasifica cada caso como verdadero o falso: a. 172 b. 288
a. 8 ƒ 4 c. 260 d. 111
b. 0 ƒ 4 29. Halla el menor número positivo que sea divisible
c. 4 ƒ 0 entre todo entero positivo menor o igual que 10.
d. Si un número es divisible entre 4 y entre 6, en- 30. Unas barras de dulce no se vendieron a 50¢ así que
tonces es divisible entre 24. redujeron el precio. Después todas se vendieron en
e. Si un número no es divisible entre 12, entonces un día, para un total de $31.93. ¿Cuál fue el precio
no es divisible entre 3. reducido de cada barra de dulce?
14. Clasifica cada caso siguiente como verdadero o 31. Dos campanas suenan a las 8:00 a.m. Por el resto del
falso. Si es falso, exhibe un contraejemplo. día, una campana suena cada media hora y la otra
a. Si 7 ƒ x y 7ⱈy, entonces 7ⱈxy. cada 45 min. ¿A qué hora sonarán de nuevo las cam-
b. Si dⱈ1a + b2, entonces dⱈa y dⱈb. panas juntas?
c. Si d ƒ 1a + b2 y dⱈa, entonces dⱈb. 32. Si el MDC de dos números completos positivos es
d. Si d ƒ 1x + y2 y d ƒ x, entonces d ƒ y. 1, ¿qué puedes decir del MMC de los dos números?
Explica tu razonamiento.
e. Si 4ⱈx y 4ⱈy, entonces 4ⱈxy.
33. Si había 9 niños y 6 niñas en una fiesta, y el anfitrión
15. Verifica si cada uno de los números siguientes es di-
quería dar exactamente el mismo número de cara-
visible entre 2, 3, 4, 5, 6, 8, 9 y 11:
melos que pudieran comprarse en paquetes de 12 ca-
a. 83,160
ramelos, ¿cuál es el menor número de paquetes que
b. 83,193
podría comprar?
16. Supón que 10,007 es primo. Sin efectuar la división en-
34. Juliana y Ramón corren en una pista. Si comienzan
tre 17, demuestra que 10,024 no es divisible entre 17.
en el mismo tiempo y lugar y van en la misma direc-
17. Llena cada espacio en blanco con un dígito para ha-
ción, con Juliana corriendo una vuelta en 5 min y
cer que cada uno de los casos siguientes sea verda-
Ramón en 3 min, ¿cuánto tiempo les tomará coinci-
dero (halla todas las respuestas posibles):
dir en el punto de partida si continúan corriendo a
a. 6 ƒ 87_4
estas velocidades?
b. 24 ƒ 4_856
35. Flora, propietaria de un puesto de café, bajó el pre-
c. 29 ƒ 87_ _4
cio del capuchino de $2.00 la taza entre 7:00 a.m. y
18. Un estudiante asegura que la suma de cinco enteros
8:00 a.m. Si obtuvo $98.69 de la venta de capuchi-
positivos consecutivos siempre es divisible entre 5.
nos y sabemos que nunca vende un capuchino por
a. Verifica la afirmación del estudiante en algunos casos.
menos de un dólar, ¿cuántos capuchinos vendió en-
b. Demuestra o refuta la afirmación del estudiante.
tre 7:00 a.m. y 8:00 a.m.? Explica tu razonamiento.
19. Determina cuál de los números siguientes es primo
(Nota: 71 ƒ 9869.)
o compuesto:
36. Halla las factorizaciones en primos de cada caso.
a. 143 b. 223
a. 610
20. ¿Cómo puedes decir si un número es divisible entre
b. 34n
24? Verifica si 4152 es divisible entre 24.
c. 974
21. ¿El MMC de dos números es siempre mayor que el
MDC de los números? Justifica tu respuesta. d. 84 # 63 # 262
22. Explica cómo hallar el MMC de tres números con e. 23 # 32 + 24 # 33 # 7
la ayuda del algoritmo euclidiano. f. 24 # 3 # 57 + 24 # 56
23. Para saber si el número 2 # 3 # 5 # 7 + 11 # 13 es 37. ¿Cuáles son los posibles residuos cuando un nú-
primo, una estudiante halla que el número es igual a mero primo mayor que 3 se divide entre 12? Justi-
353. Ella verifica que 17ⱈ353 y 192 7 353 y sin ma- fica tu respuesta.

© Lopez Mateos Editores. ISBN 978-607-95583-2-1, obra completa, versión electrónica, ISBN 978-607-95583-3-8, volumen 1, versión electrónica. Ejemplar asignado a: Helecto Villarroel gutierrez -
helecto@gmail.com. Fecha: 27 de octubre de 2014. Prohibida su modificación, copia o distribución.
Bibliografía seleccionada 339

38. Demuestra el criterio de divisibilidad entre 9 usando fiestas religiosas. Los romanos, por ejemplo, en al-
un número n de tres dígitos a, b y c tal que gún momento usaron una semana de 8 días. Supo-
n = a # 102 + b # 10 + c. niendo que en ese entonces abril tenía 30 días pero
夝39. El trío 3, 5, 7 consta de enteros impares consecuti- estaba basado en una semana de 8 días, si el primer
vos que son todos primos. Da un argumento con- día del mes fue domingo y el día posterior al sábado
vincente de que éste es el único trío de enteros se llamaba vena, ¿en qué día caía el último día del
impares consecutivos donde todos son primos. mes?
(Sugerencia: usa el algoritmo de la división). * 41. ¿Qué sistema modular se usaría, y por qué, para me-
*40. La duración de una semana probablemente fue ins- dir los ángulos de rotación (en grados) que barre la
pirada por la necesidad de marcar días de mercado y luz de un faro en una pequeña isla?

Bibliografía seleccionada

Anthony, G., and M. Walshaw. “Zero A ‘None’ Num- Petrella, G. “Subtracting Integers: An Affective Les-
ber?” Teaching Children Mathematics 11 (August son.” Mathematics Teaching in the Middle School 7
2004): 38–42. (November 2001): 150–151.
Bay, J. “Developing Number Sense on the Number Ponce, G. “It’s All in the Cards: Adding and Subtract-
Line.” Mathematics Teaching in the Middle School 6 ing Integers.” Mathematics Teaching in the Middle
(April 2001): 448–451. School 13 (August 2007): 10–17.
Bennett, A., and L. Nelson. “Divisibility Tests: So Right Reeves, A., and M. Beasley. “Advanced Paint by Num-
for Discoveries.” Mathematics Teaching in the Middle bers.” Mathematics Teaching in the Middle School 12
School 7 (April 2002): 460–464. (April 2007): 447.
Bezuszka, S., and M. Kenney. “Even Perfect Numbers: Robbins, C., and T. Adams. “Get Primed to the Basic
(Update)2.” Mathematics Teacher 90 (November Building Blocks of Numbers.” Mathematics Teaching
1997): 628–633. in the Middle School 13 (September 2007): 122–127.
Brown, E., and E. Jones. “Using Clock Arithmetic to Schneider, S., and C. Thompson. “Incredible Equations
Teach Algebra Concepts.” Mathematics Teaching in Develop Incredible Number Sense.” Teaching Chil-
the Middle School 11 (September 2005): 104–109. dren Mathematics 7 (November 2000): 146–148,
Graviss, T., and J. Greaver. “Extending the Number 165–168.
Line to Make Connections with Number Theory.” Shultz, H. “The Postage-Stamp Problem, Number
Mathematics Teacher 85 (September 1992): 418–420. Theory, and the Programmable Calculator.”
Gregg, J., and D. Gregg. “A Context for Integer Com- Mathematics Teacher 92 (January 1999): 20–22.
putation.” Mathematics Teaching in the Middle School Steinberg, R., D. Sleeman, D. Ktorza. “Algebra Students
13 (August 2007): 46–50. Knowledge of Equivalent Equations.” Journal of
Nurnberger-Haag, J. “Integers Made Easy: Just Walk It Research in Mathematics Education 22 (February 1990):
Off,” Mathematics Teaching in the Middle School 13 112–121.
(September 2007): 118–121.
Peterson, J. “Fourteen Different Strategies for Multi-
plication of Integers, or Why 1 - 121 - 12 = 1 + 12.”
Arithmetic Teacher 19 (May 1972): 396–403.

© Lopez Mateos Editores. ISBN 978-607-95583-2-1, obra completa, versión electrónica, ISBN 978-607-95583-3-8, volumen 1, versión electrónica. Ejemplar asignado a: Helecto Villarroel gutierrez -
helecto@gmail.com. Fecha: 27 de octubre de 2014. Prohibida su modificación, copia o distribución.
Los números racionales
como fracciones
CAPÍTULO

Problema preliminar
Blanca compró un caballo por $270 y comenzó de inmediato a pagar su pensión. Vendió el ca-
ballo en $540. Considerando el costo de su pensión, ella se percató de que perdió una cantidad
igual a la mitad de lo que pagó por el caballo, más una cuarta parte del costo de su pensión.
¿Cuánto perdió Blanca al vender el caballo?

340
© Lopez Mateos Editores. ISBN 978-607-95583-2-1, obra completa, versión electrónica, ISBN 978-607-95583-3-8, volumen 1, versión electrónica. Ejemplar asignado a: Helecto Villarroel gutierrez -
helecto@gmail.com. Fecha: 27 de octubre de 2014. Prohibida su modificación, copia o distribución.
Sección 6-1 El conjunto de los números racionales 341

L os enteros como - 5 se inventaron para resolver ecuaciones como x + 5 = 0. De manera


análoga, se necesita un nuevo tipo de número para resolver una ecuación como 2x = 1.
Como no hay entero que la satisfaga, se inventó un número que sí lo haga. El nuevo número
debe estar entre 0 y 1. Una razón es que 0 6 1 6 2, que al substituir 2x por 1 se vuelve
2 # 0 6 2x 6 2 # 1, ó 0 6 x 6 1. No hay enteros entre 0 y 1 que cumplan con estas condicio-
nes. Por lo tanto, de manera análoga al desarrollo de una nueva notación para los enteros nega-
tivos, necesitamos una notación para este nuevo número. Si la multiplicación va a funcionar con
este nuevo número como con los números completos, entonces 2x = x + x, de modo que
x + x = 1. En otras palabras, el número creado sumado a sí mismo debe dar 1. El número in-
1
ventado para resolver la ecuación es un medio, denotado con . Es un elemento del conjunto de
a 2 a
números de la forma , donde b Z 0 y a y b son enteros. Más aún, los números de la forma son
b b
soluciones de las ecuaciones de la forma bx = a. Este conjunto, denotado con Q, es el conjunto
de los números racionales y se define de la manera siguiente:

Q = e
ƒ a y b son enteros y b Z 0 f
a
b
Q es un subconjunto de otro conjunto de números llamado fracciones. Las fracciones son de
a 1
la forma , donde b Z 0 pero a y b no son necesariamente enteros. Por ejemplo, es una
b 22
fracción pero no es un número racional. (En este libro nos restringimos a fracciones
donde a y b son números reales, pero esa restricción no es necesaria.) El hecho de que b Z 0
siempre es necesario, pues no está definida la división entre 0.
Como se indica en la cita de los Principios y objetivos que mostramos a continuación, los
niños deben introducirse al tema de las fracciones mediante actividades concretas.
◆ Nota de Además de comprender los números completos, se puede animar a los estudiantes a comprender
investigación y representar fracciones de uso común en contexto, como 1/2 galleta o 1/8 de pizza, y a ver las
fracciones como parte de una unidad o de una colección. Los maestros deberán ayudar a los estu-
Los niños consideran
diantes a desarrollar una comprensión de las fracciones como división de números. En los grados
con frecuencia una medios, como parte de la base para su trabajo con la proporcionalidad, los estudiantes necesitan
fracción como dos afirmar su comprensión de las fracciones como números. (p. 33)
números separados
por una línea, en lu- Según se desprende de la nota de investigación, es importante hacer énfasis en que una frac-
gar de verla como un 3
ción como representa un solo número.
solo número (Bana, 4
Farrell y
McIntosh 1997). ◆ Expectativas para los números racionales como fracciones
Las expectativas para los estudiantes mencionadas en los Principios y objetivos que se cubren
en este capítulo incluyen las siguientes:
En los grados K a 2, los niños deberán entender y representar fracciones de uso común como 1/4,
1/3 y 1/2. (p. 392)
En los grados 3 a 5, los niños deberán desarrollar la comprensión de las fracciones para incluirlas
como partes de un todo unitario, como partes de una colección, como ubicación en rectas numéricas
y como divisiones de números completos. Los niños deberán usar modelos, puntos de referencia y
formas equivalentes para juzgar el tamaño de las fracciones. Deberán tener capacidad para desarrollar
y usar estrategias para estimar cálculos que incluyan fracciones. (p. 392)

© Lopez Mateos Editores. ISBN 978-607-95583-2-1, obra completa, versión electrónica, ISBN 978-607-95583-3-8, volumen 1, versión electrónica. Ejemplar asignado a: Helecto Villarroel gutierrez -
helecto@gmail.com. Fecha: 27 de octubre de 2014. Prohibida su modificación, copia o distribución.
342 Los números racionales como fracciones

En los grados 6 a 8, los niños trabajan de manera flexible con fracciones para resolver proble-
mas, para comparar y ordenar fracciones y para localizarlas en una recta numérica. Comprenden y
usan razones y proporciones para representar relaciones cuantitativas. Los niños entienden el sig-
nificado y el efecto de las operaciones aritméticas con fracciones. Seleccionan métodos y herra-
mientas apropiados para calcular con fracciones, entre ellas cómputo mental, estimación,
calculadora o papel y lápiz, y aplican el método elegido. (p. 393)

6-1 El conjunto de los números racionales


a a
En el número racional , a es el numerador y b es el denominador. El número racional tam-
b b
bién se puede representar como a>b o como a , b. La palabra fracción se deriva de la palabra latina

fractus, que significa “romper”. La palabra numerador viene de una palabra del latín que sig-
nifica “numerar”, y denominador viene de una palabra del latín que significa “nombrar”. La
tabla 6-1 muestra diversas maneras en que usamos los números racionales.

Tabla 6-1 Usos de los números racionales

Uso Ejemplo

3
Problema de división o solución de La solución de 2x = 3 es .
2
un problema de multiplicación
1
Partición, o parte, de un todo José recibió del salario mensual de
2
María como pensión alimenticia.
Razón La razón de los republicanos a los
demócratas en el Senado de
Estados Unidos es de tres a cinco.

Probabilidad Cuando lanzas una moneda ideal, la


1
probabilidad de obtener cara es .
2

◆ Nota En sus inicios, el sistema egipcio de numeración tenía símbolos para las fracciones con nu-
histórica merador 1. La mayoría de las fracciones con numerador diferente de 1 se expresaban como
7 1 1
una suma de diferentes fracciones con numerador 1, por ejemplo = + .
12 3 4
Las fracciones con denominador de 60 o con potencias de 60 eran comunes en la antigua
35
Babilonia alrededor del año 2000 a.C., donde 12,35 significaba 12 + . El método fue
60
adoptado más tarde por el astrónomo griego Ptolomeo (aproximadamente en el año 125
1/3
d.C.). El mismo método fue usado en los países islámicos y europeos, y actualmente se usa
19 47
en la medida de los ángulos, donde 13° 19¿ 47– significa 13 + + grados.
60 602
La notación moderna para las fracciones —una barra entre el numerador y el deno-
minador— es de origen hindú. Se volvió de uso general en Europa en los libros del siglo
dieciséis. ◆

© Lopez Mateos Editores. ISBN 978-607-95583-2-1, obra completa, versión electrónica, ISBN 978-607-95583-3-8, volumen 1, versión electrónica. Ejemplar asignado a: Helecto Villarroel gutierrez -
helecto@gmail.com. Fecha: 27 de octubre de 2014. Prohibida su modificación, copia o distribución.
Sección 6-1 El conjunto de los números racionales 343

◆ Nota de Cuando los estudiantes desarrollan una comprensión del concepto de fracción, se prefiere
investigación el modelo del área al modelo de conjuntos porque el área total es un atributo visible, más
flexible. El modelo del área permite a los estudiantes visualizar casi cualquier fracción,
mientras que el modelo de conjuntos (por ejemplo, un grupo de fichas de colores) tiene va-
rias limitaciones, especialmente para la interpretación parte/todo. Por ejemplo, trata de re-
3
presentar usando cuatro galletas o una hoja de papel (English y Halford 1995; Hope y
5
Owens, 1987). ◆
La figura 6-1 ilustra el uso de los números racionales como parte de un todo y como parte
de un conjunto dado. Por ejemplo, en el modelo de área de la figura 6-1(a), una de tres partes
1
congruentes, ó del rectángulo grande, está sombreada. En la figura 6-1(b), dos de tres par-
3
(a) Modelo de área 2
tes congruentes, ó del segmento unitario, están sombreadas. En la figura 6-1(c), tres de
3
3
cinco círculos congruentes, ó de los círculos, están sombreados. Nota que, como se indica
0 1 5
(b) Modelo de la recta en la Nota de investigación, el modelo del área es el modelo preferido.
numérica
Nuestra primera relación con fracciones, o números racionales, usualmente es como una
descripción oral más que como una notación matemática. Escuchamos frases como “media
pizza”, “un tercio del pastel” o “tres cuartos de una tarta”. Nos encontramos preguntas
como “si tres barras idénticas de fruta se distribuyen equitativamente entre cuatro amigos,
3
¿cuánto obtiene cada uno?” La respuesta es que cada uno recibe de barra.
4
(c) Modelo de conjuntos Las palabras en español que se usan para los números racionales son las mismas que usamos para
3
Figura 6-1 “orden”, por ejemplo la cuarta persona en una fila y tres cuartos para . Esto causa confusión en los
4
3
estudiantes que aprenden fracciones. En contraste, en chino se lee “de cuatro partes, (toma)
4
tres”. El modelo chino refuerza la idea de partir cantidades en partes iguales y escoger algún
número de estas partes. El concepto de repartir cantidades y comparar el tamaño de las par-
tes puede proporcionar un punto de entrada para el estudio de los números racionales. Para
los números racionales, repartir puede jugar el papel de contar en los números enteros.
Cuando los números racionales se introducen como fracciones que representan una parte
del todo, debemos prestar atención al todo del cual se derivan. Por ejemplo,
3
si hablamos de de pizza, entonces la cantidad de pizza se determina tanto por la parte frac-
4
3
cional, que es , como por el tamaño de la pizza. Tres cuartos de una pizza grande es cierta-
4
mente más que los tres cuartos de una pizza pequeña, aunque el número de piezas sea el

mismo. Debe prestarse atención al contexto y al tamaño del todo considerado.


a
Para entender el significado de cualquier fracción, , usando el modelo parte-a-todo, de-
b
bemos tomar en cuenta lo siguiente:
1. El todo en consideración.
2. El número b de partes iguales en las que se divide el todo.
3. El número a de partes del todo que estamos seleccionando.

© Lopez Mateos Editores. ISBN 978-607-95583-2-1, obra completa, versión electrónica, ISBN 978-607-95583-3-8, volumen 1, versión electrónica. Ejemplar asignado a: Helecto Villarroel gutierrez -
helecto@gmail.com. Fecha: 27 de octubre de 2014. Prohibida su modificación, copia o distribución.
344 Los números racionales como fracciones

La caricatura muestra que el concepto de fracción no es fácil para algunos niños.

EL CIRCO FAMILIAR Por Bil Keane

“¿Cómo es que un cuarto de peso


son 25 centavos y un cuarto de
hora son sólo 15 minutos?”
AHORA INTENTA ÉSTE 6-1
a. ¿Cómo responderías a la pregunta de Billy en la caricatura?
1 1
b. Jaime asegura que 7 porque en la figura 6-2 la parte
3 2
1
sombreada correspondiente a es más grande que la
3
1
porción sombreada para . ¿Cómo le ayudarías?
2 1 1
3 2
(a) (b)

Figura 6-2

Los números racionales se pueden representar sobre una recta numérica. Una vez asigna-
dos los enteros 0 y 1 a puntos sobre una recta, se define el segmento unitario y se asigna cual-
3
quier otro número racional a un punto específico. Por ejemplo, para representar sobre la
4
recta numérica, dividimos el segmento de 0 a 1 en 4 segmentos de igual longitud y marca-
mos la línea según corresponda. Después, comenzando en el 0 contamos 3 de estos segmen-
tos y nos detenemos en la marca correspondiente al extremo derecho del tercer segmento
3
para obtener el punto asignado al número racional . La figura 6-3 muestra los puntos que
-5 -3 3 5 4
corresponden a - 2, , - 1, , 0, , 1, y 2.
4 4 4 4

© Lopez Mateos Editores. ISBN 978-607-95583-2-1, obra completa, versión electrónica, ISBN 978-607-95583-3-8, volumen 1, versión electrónica. Ejemplar asignado a: Helecto Villarroel gutierrez -
helecto@gmail.com. Fecha: 27 de octubre de 2014. Prohibida su modificación, copia o distribución.
Sección 6-1 El conjunto de los números racionales 345

–2 –5 – 1 –3 0 3 1 5 2
— — — —
4 4 4 4
Figura 6-3

En los Puntos focales para el grado 3 hallamos que los estudiantes desarrollan una com-
prensión del significado y los usos de las fracciones para representar partes de un todo, par-
tes de un conjunto o puntos o distancias en una recta numérica (p. 15).

AHORA INTENTA ÉSTE 6-2


3 1 1 3 2 4
a. Si representa , traza rectángulos para , , , y .
4 4 2 2 3 3
1 -1 3 -7
b. En la figura 6-3, localiza puntos que correspondan a , , y .
2 2 2 4

a 4
Una fracción , donde 0 … a 6 b, es una fracción propia. Por ejemplo, es una fracción
b 7
7 4 9 7 a
propia, pero , y no lo son; es una fracción impropia. En general, es una fracción impro-
4 4 7 4 b
pia si a Ú b 7 0.
Es típico que en los primeros grados sólo se presenten fracciones positivas a los estudian-
tes. Se muestra un ejemplo de esto en la siguiente Página de un libro de texto.

AHORA INTENTA ÉSTE 6-3 Responde las tres preguntas del “Tema de plática” en la página de muestra
(página 346).

O B S E R VA C I Ó N Nota que cada entero n se puede representar como un número racio-


nk 0#k 0
nal pues n = , donde k es cualquier entero distinto de cero. Entonces 0 = = .
k k k

AHORA INTENTA ÉSTE 6-4 Traza un diagrama de Venn para mostrar la relación entre números
naturales, números completos, enteros y números racionales.

Fracciones equivalentes o iguales


Las fracciones se pueden exponer en el salón de clase por medio de una actividad concreta, como
1
doblar hojas de papel. En la figura 6-4(a) una de las tres partes congruentes, ó , está sombreada.
3
En este caso el todo es el rectángulo. En la figura 6-4(b), cada uno de los tercios ha sido doblado a
2
la mitad, de modo que ahora tenemos seis secciones, y dos de seis partes congruentes, ó , están
6
1 2
sombreadas. Así, tanto como representan exactamente la misma porción sombreada. Aunque
3 6
1 2
los símbolos y no se parecen, representan el mismo número racional y son fracciones
3 6
equivalentes o fracciones iguales.
© Lopez Mateos Editores. ISBN 978-607-95583-2-1, obra completa, versión electrónica, ISBN 978-607-95583-3-8, volumen 1, versión electrónica. Ejemplar asignado a: Helecto Villarroel gutierrez -
helecto@gmail.com. Fecha: 27 de octubre de 2014. Prohibida su modificación, copia o distribución.
346 Los números racionales como fracciones

Página de un libro de texto NOMBRES DE PARTES FRACCIONARIAS

Lección 3-3
Leccíon 9-2

Idea clave Nombres de partes Calentamiento


Puedes escribir
una fracción fraccionarias Di si en cada caso se
muestran partes iguales.
para describir Aprende
partes iguales
de un todo.
¿Qué es una fracción?
Vocabulario Puedes usar fracciones para nombrar
• fracción partes iguales de un todo.
• numerador
• denominador

Ejemplo
La bandera de Nigeria tiene 3
partes iguales. ¿Qué fracción
de la bandera es verde?

Numerador 2 partes iguales son verdes


Denominador en total hay 3 partes iguales

Lo que escribes Lo que dices


Dos tercios de la bandera
de la bandera son verdes.
son verdes.

Tema de plática
¿Qué fracción de la bandera de
Nigeria no es verde?
¿Qué fracción de cada cuadrado es azul?

Regino cortó una manzana en 4


partes iguales. Él se comió una
pieza. ¿Qué fracción nos dice
cuánto se comió Llévalo a la RED
Más ejemplos
de la manzana?

Fuente: Mathematics, Diamond Edition, Grade 3, Scott Foresman-Addison Wesley, 2008 ( p. 502).

© Lopez Mateos Editores. ISBN 978-607-95583-2-1, obra completa, versión electrónica, ISBN 978-607-95583-3-8, volumen 1, versión electrónica. Ejemplar asignado a: Helecto Villarroel gutierrez -
helecto@gmail.com. Fecha: 27 de octubre de 2014. Prohibida su modificación, copia o distribución.
Sección 6-1 El conjunto de los números racionales 347

Las fracciones equivalentes son números que representan el mismo punto en una recta numé-
1 2 1 2
rica. Como representan cantidades iguales, escribimos = y decimos que es igual a .
3 6 3 6

1 2 4
3 6 12
(a) (b) (c)

Figura 6-4

La figura 6-4(c) muestra el rectángulo con cada uno de los tercios originales doblados en 4 par-
4 1
tes iguales, con 4 de las 12 partes sombreadas. Así, es igual apues la misma porción del mo-
12 3
1 2 3 4 5
delo está sombreada. De manera análoga, podríamos ilustrar que , , , , , Á son iguales.
3 6 9 12 15
También se pueden usar bandas de fracciones para generar fracciones equivalentes, como
se ve en la página de muestra (p.348). La parte (a) de la página de muestra ilustra que
1 3 6
= = . ¿Qué fracciones equivalentes están modeladas en la parte (b)? También en la
2 6 12
página de muestra, en el ejemplo A vemos otra manera de hallar fracciones equivalentes.
Esta técnica usa la ley fundamental de las fracciones, que puede enunciarse como sigue: El
valor de una fracción no cambia si el numerador y el denominador se multiplican por el mismo nú-
mero entero distinto de cero. Esta ley de las fracciones se puede demostrar bajo ciertas hipóte-
sis, de ahí que se enuncie como teorema.

Teorema 6–1: Ley fundamental de las fracciones


a a an
Sean cualquier fracción y n un número entero distinto de cero; entonces, = .
b b bn

O B S E R VA C I Ó N
1. En el teorema 6–1 se puede mostrar que n puede ser cualquier número distinto de cero.
Esta versión de la ley fundamental de las fracciones se usa más adelante en este libro.
a a , d
2. El teorema 6–1 implica que si d es un factor común de a y b, entonces = .
b b , d
3. A estas alturas, algunos estudiantes justifican la ley fundamental de las fracciones
como sigue:
an a#n a n a a
= # = # = #1 =
bn b n b n b b
Éste es un enfoque correcto; sin embargo, al desarrollar las propiedades de las frac-
ciones todavía no hemos analizado la multiplicación de fracciones.

AHORA INTENTA ÉSTE 6-5 Explica por qué en la ley fundamental de las fracciones, n debe ser
distinto de cero.

© Lopez Mateos Editores. ISBN 978-607-95583-2-1, obra completa, versión electrónica, ISBN 978-607-95583-3-8, volumen 1, versión electrónica. Ejemplar asignado a: Helecto Villarroel gutierrez -
helecto@gmail.com. Fecha: 27 de octubre de 2014. Prohibida su modificación, copia o distribución.
348 Los números racionales como fracciones

Página de un libro de texto FRACCIONES EQUIVALENTES

Lección 3-7
Calentamiento
Idea clave
Una parte de un todo Fracciones equivalentes Halla los factores
o de un conjunto se comunes de cada par
puede representar Aprende de números.
con fracciones
equivalentes.
Actividad
Vocabulario ¿Qué son las fracciones equivalentes?
• fracciones
equivalentes Las fracciones que representan la misma cantidad se llaman
• factor común fracciones equivalentes.
(p.150)
• mínimo
Usa bandas de fracciones
denominador para identificar una o más
común (MDC) fracciones equivalentes a
• máximo factor cada una de las
común (MFC) siguientes fracciones;
• mínima expresión

ex
Materiales ef l i ó

R
¿Cuáles dos fracciones equivalentes se

n
bandas de fracciones
o
muestran? Explica.
¡Piensa!
Puedo usar objetos como
ayuda para identificar
fracciones equivalentes.

¿Cómo puedes hallar fracciones equivalentes?


Ejemplo A
Halla dos fracciones que sean equivalentes a 18 .
24

Una manera Otra manera


Usa la multiplicación Usa la división
Multiplica el numerador y el Divide el numerador y el
denominador por el mismo denominador entre el mismo
número distinto de cero. número distinto de cero.
El número 2 es fácil de usar, así El número 3 es un factor común,
que multiplica por 2 el numerador y de modo que divide entre 3
el denominador. el numerador y denominador.

De modo que 18 , 36
y 6 son fracciones equivalentes.
24 48 8

Fuente: Scott Foresman-Addison Wesley Mathematics, Grade 6, 2008 ( p. 164).

© Lopez Mateos Editores. ISBN 978-607-95583-2-1, obra completa, versión electrónica, ISBN 978-607-95583-3-8, volumen 1, versión electrónica. Ejemplar asignado a: Helecto Villarroel gutierrez -
helecto@gmail.com. Fecha: 27 de octubre de 2014. Prohibida su modificación, copia o distribución.
Sección 6-1 El conjunto de los números racionales 349

7 -7 7 7 # 1 - 12 -7
15 1 12
De la ley fundamental de las fracciones, - = pues - = - # - = .
15 15 15 15
a -a -a
De manera análoga, - = . Usualmente se prefiere la forma , donde b es un número
b b b
positivo.
Ejemplo 6-1 12 x
Halla un valor para x de modo que = .









42 210

12 12 # 5
Solución Como 210 , 42 = 5, por la ley fundamental de las fracciones = =
42 42 # 5
60 x 60
. Por lo tanto, = y x = 60.
210 210 210
12 2#6 2 2 # 30 60
Enfoque alternativo: = # = = # = . Por lo tanto, x = 60.
42 7 6 7 7 30 210

Simplificación de fracciones
La ley fundamental de las fracciones justifica el proceso de simplificación de fracciones.
Considera lo siguiente:
60 6 # 10 6
= = .
210 21 # 10 21
También,
6 2#3 2
= # = .
21 7 3 7
60
Podemos simplificar porque el numerador y el denominador tienen un factor común de 10.
210
6
Podemos simplificar pues 6 y 21 tienen un factor común de 3. Sin embargo, no podemos
21
2
simplificar pues 2 y 7 no tienen otro factor común que 1. Nota que pudimos simplificar
7
60 60 2 # 30 2 2 60
en un solo paso: = # = . Tenemos que es la forma más simple de
120 120 7 30 7 7 210
a
pues 60 y 210 se han dividido entre su máximo divisor común, 30. Para escribir una fracción
b
en su forma más simple, esto es, en su mínima expresión, dividimos a y b entre el MDC(a, b).

Definición de la forma más simple


a
Un número racional está en su forma más simple si b 7 0 y MDC1a, b2 = 1; esto es, si a y b no
b
tienen un factor común mayor que 1, y b 7 0.

Podemos usar calculadoras científica/fraccional para simplificar fracciones. Por ejemplo,


6 3
para simplificar tecleamos 6 / 1 2 y presionamos SIMP = , y aparece en panta-
12 6
lla. En este momento, un indicador nos dice que ésta no es la forma más simple, de modo

© Lopez Mateos Editores. ISBN 978-607-95583-2-1, obra completa, versión electrónica, ISBN 978-607-95583-3-8, volumen 1, versión electrónica. Ejemplar asignado a: Helecto Villarroel gutierrez -
helecto@gmail.com. Fecha: 27 de octubre de 2014. Prohibida su modificación, copia o distribución.
350 Los números racionales como fracciones

1
que presionamos de nuevo SIMP = para obtener . Presionando la tecla x y
2
podemos ver en cualquier momento el factor cancelado.
La ley fundamental de las fracciones se puede usar para simplificar expresiones algebrai-
cas, como se ve en el ejemplo siguiente.

Ejemplo 6-2 Escribe cada caso siguiente en su forma más simple, si es que aún no está en esa forma:
1a + b22








28ab2 x2 + x 3 + x2
a. b. c. d.
42a2b2 3a + 3b x + 1 3x 2
3 + 3x2 a2 - b2 a2 + b2
e. f. g.
3x2 a - b a + b

28 ab2 2114ab22 2
3a114ab 2
Solución a. 2 2
= 2
= , donde a y b son distintos de 0.
42a b 3a
1a + b22 1a + b21a + b2 a + b
b. = = , donde a + b Z 0
3a + 3b 31a + b2 3
x2 + x x1x + 12 x1x + 12 x
c. = = = = x, donde x Z - 1
x + 1 x + 1 11x + 12 1
3 + x2
d. no se puede simplificar pues 3 + x2 y 3x2 no tienen factores en común
3x 2
excepto 1.
3 + 3x 2 311 + x 22 1 + x2
e. = =
3x 2 3x 2 x2
f. Recordemos que en el capítulo 5 usamos las propiedades distributivas para de-
mostrar que 1a - b21a + b2 = a2 - b2. Así,
a2 - b2 1a - b21a + b2 a + b
1a - b21
= = = a + b, donde a Z b
a - b 1
g. Ya está en su forma más simple, pues a2 + b2 no tiene a 1a + b2 como factor.
Nota que a2 + b2 Z 1a + b22.

O B S E R VA C I Ó N

1. Cuando escribimos una expresión algebraica que es una fracción, debemos supo-
ner que el denominador no es 0. Así, aunque se reduzca la fracción, esta restricción
x2 + x
debe mantenerse. Por ejemplo, en la parte (c) del ejemplo 6-2, = x si
x + 1
x Z - 1, y en la parte (f ) el resultado vale si a - b Z 0; esto es, si a Z b.
3 + x2
2. En la parte (d) señalamos que no se puede simplificar. Sin embargo, para al-
3x2
gunos valores particulares de x, la fracción correspondiente sí puede simplificarse.

Éste es el caso si x es un múltiplo de 3; por ejemplo, si x = 3 entonces el valor de


3 + x2 3 + 9 3#4 4
2
es # = # = .
3x 3 9 3 9 9

© Lopez Mateos Editores. ISBN 978-607-95583-2-1, obra completa, versión electrónica, ISBN 978-607-95583-3-8, volumen 1, versión electrónica. Ejemplar asignado a: Helecto Villarroel gutierrez -
helecto@gmail.com. Fecha: 27 de octubre de 2014. Prohibida su modificación, copia o distribución.
Sección 6-1 El conjunto de los números racionales 351

3. Algunos estudiantes, pensando la ley fundamental de las fracciones como una propiedad
6 + a2
de cancelación, a menudo “simplifican” una expresión como pensándola como
3a
2#3 + a#a
y “cancelando” números iguales en los productos para obtener 2 + a
3a
como respuesta. Hacer énfasis en el enfoque del factor en que ni 3 ni a es factor de

6 + a2 puede ayudar a evitar dichos errores.

Igualdad de fracciones
12 10
Podemos usar tres métodos para mostrar que dos fracciones como y , son iguales.
42 35
1. Simplifica las dos fracciones a su forma más simple:
12 22 # 3 2 10 5#2 2
= # # = y = # =
42 2 3 7 7 35 5 7 7
Así,
12 10
=
42 35
2. Reescribe ambas fracciones con el mismo mínimo denominador común. Como
MMC142, 352 = 210, entonces
12 60 10 60
= y =
42 210 35 210
Así,
12 10
=
42 35
3. Reescribe ambas fracciones con un denominador común (no necesariamente el
mínimo). Un múltiplo común de 42 y 35 puede obtenerse multiplicando 42 # 35, lo
cual da 1470. Ahora,
12 420 10 420
= y =
42 1470 35 1470
Por lo tanto,
12 10
=
42 35
a c
El tercer método sugiere un algoritmo general para determinar si dos fracciones y son
b d
iguales. Reescribe ambas fracciones con denominador común bd. Esto es,
a ad c bc
= y =
b bd d bd
ad bc
Como los denominadores son iguales, = si, y sólo si, ad = bc. Por ejemplo,
bd bd
24 6
= porque 24 # 9 = 216 = 36 # 6. En general resulta el siguiente teorema.
36 9

© Lopez Mateos Editores. ISBN 978-607-95583-2-1, obra completa, versión electrónica, ISBN 978-607-95583-3-8, volumen 1, versión electrónica. Ejemplar asignado a: Helecto Villarroel gutierrez -
helecto@gmail.com. Fecha: 27 de octubre de 2014. Prohibida su modificación, copia o distribución.
352 Los números racionales como fracciones

Teorema 6–2
a c
Dos fracciones y son iguales si, y sólo si, ad = bc.
b d

Con la ayuda de una calculadora podemos determinar si dos fracciones son iguales usan-
do la propiedad anterior. Como 2 * 2 1 9 6 = y 4 * 1 0 9 8 = presentan en
2 1098
pantalla 4392, vemos que = .
4 2196

Orden en los números racionales


Como se analizó en los Principios y objetivos (p. 392), los estudiantes de los grados menores debe-
rán experimentar comparando fracciones entre 0 y 1 en relación con puntos de referencia como
1 3
0, , y 1. En los grados medios, se vuelve más difícil la comparación de fracciones. Primero
2 4
consideramos la comparación de fracciones con denominadores iguales.
7 5
Los niños saben que 7 pues si una pizza se divide en 8 partes de igual tamaño, en-
8 8
3 4
tonces 7 partes son más que 5 partes. De manera análoga, 6 . Así, dadas dos fracciones
7 7
con denominador común positivo, la que tenga el mayor numerador es la fracción más

grande. Esto se puede enunciar así:

Teorema 6–3
a c
Si a, b y c son enteros y b 7 0, entonces 7 si, y sólo si, a 7 c.
b b

En la siguiente página de muestra se ilustra cómo comparar los tamaños de fracciones.

AHORA INTENTA ÉSTE 6-6 Determina si el teorema 6–3 es verdadero para b 6 0.

Comparar fracciones con denominadores distintos es más difícil. Por ejemplo, los estu -
1 1
diantes pueden pensar de manera incorrecta que 7 porque 8 es mayor que 7. En otras
8 7
6 7
palabras, pueden creer, de manera errónea, que es igual a pues en ambos casos la di-
7 8
ferencia entre el numerador y el denominador es 1. Es posible auxiliar la comparación

de fracciones con denominadores distintos mediante bandas fraccionarias para comparar


© Lopez Mateos Editores. ISBN 978-607-95583-2-1, obra completa, versión electrónica, ISBN 978-607-95583-3-8, volumen 1, versión electrónica. Ejemplar asignado a: Helecto Villarroel gutierrez -
helecto@gmail.com. Fecha: 27 de octubre de 2014. Prohibida su modificación, copia o distribución.
Sección 6-1 El conjunto de los números racionales 353

Página de un libro de texto


Tema de plática
Explica cómo sabes que 3 es mayor que 1 .
4 4
Sentido numérico. Una pizza de queso se corta en 12 rebanadas
iguales y una pizza de vegetales del mismo tamaño se corta en 8
rebanadas iguales. ¿Cuál pizza tiene las rebanadas más grandes? Explica.

¿Cómo ordenas fracciones?

Ejemplo C

Escribe 1 , 3 y 1 ordenados del mayor al menor.


4 8 2
y

Tema de plática
Explica cómo ordenar 1 , 1 y 1 de menor
5 2 10
a mayor.

VERIFICA Para otro ejemplo, ver Set 9-4 en p. 554

Compara. Escribe >, < o =.

Ordena 2 , 1 y 1 de menor a mayor.


4 3 6

Sentido numérico. ¿Pueden dos fracciones unitarias


diferentes ser iguales? ¿Por qué sí o por qué no?

Sección A Lección 9-4

Fuente: Mathematics, Diamond Edition, Grade Three, Scott Foresman-Addison Wesley, 2008 ( p. 507).

© Lopez Mateos Editores. ISBN 978-607-95583-2-1, obra completa, versión electrónica, ISBN 978-607-95583-3-8, volumen 1, versión electrónica. Ejemplar asignado a: Helecto Villarroel gutierrez -
helecto@gmail.com. Fecha: 27 de octubre de 2014. Prohibida su modificación, copia o distribución.
354 Los números racionales como fracciones

4 11
visualmente las fracciones. Por ejemplo, considera las fracciones y mostradas en la fi-
5 12
gura 6-5.

4
5

11
12

Figura 6-5
De la figura 6-5, los estudiantes ven que cada fracción es una pieza menos que el tamaño
11
de la unidades idénticas. Sin embargo, ven que la pieza faltante en es menor que la pieza
12
4 11 4
faltante en , de modo que debe ser mayor que .
5 12 5
Comparar fracciones con denominadores distintos puede lograrse reescribiendo las frac-
ciones con el mismo denominador. Así podemos compararlas con la técnica recién apren-
a c
dida. Usando el denominador común bd, podemos escribir las fracciones y como
b d
ad bc
y . Como b 7 0 y d 7 0, entonces bd 7 0; y podemos aplicar el teorema 6–3 como sigue:
bd bd
a c ad bc ad bc
7 si, y sólo si, 7 y 7 si, y sólo si, ad 7 bc
b d bd bd bd bd
Por lo tanto, tenemos el teorema siguiente.

Teorema 6–4
a c
Si a, b, c y d son enteros con b 7 0 y d 7 0, entonces 7 si, y sólo si, ad 7 bc.
b d

3 9 5 2
AHORA INTENTA ÉSTE 6-7 Ordena las fracciones , , y de menor a mayor.
4 16 8 3

◆ Nota de Los estudiantes a quienes se ha enseñado el método del denominador común para compa-
investigación rar dos fracciones tienden a ignorarlo y a utilizar reglas asociadas con el ordenamiento de
los números completos. Los estudiantes que de manera correcta comparan numeradores si
los denominadores son iguales, a veces comparan los denominadores si los numeradores
son iguales (Behr et al. 1984). ◆
© Lopez Mateos Editores. ISBN 978-607-95583-2-1, obra completa, versión electrónica, ISBN 978-607-95583-3-8, volumen 1, versión electrónica. Ejemplar asignado a: Helecto Villarroel gutierrez -
helecto@gmail.com. Fecha: 27 de octubre de 2014. Prohibida su modificación, copia o distribución.
Sección 6-1 El conjunto de los números racionales 355

Como se sugiere en la nota de investigación anterior, si los estudiantes comparan deno-


minadores de fracciones si los numeradores son iguales, entonces esto puede ser una buena
3 3
estrategia si se maneja de manera adecuada. Por ejemplo, considera y . Si el todo es el
4 10
1 1 1
mismo para ambas fracciones, esto significa que tenemos tres y tres . Como es ma-
4 10 4
1
yor que , entonces tres de las partes mayores será mayor que tres de las partes menores,
10
3 3
de modo que 7 .
4 10

AHORA INTENTA ÉSTE 6-8 Generaliza el enfoque anterior para comparar fracciones cuyos numera-
dores y denominadores son enteros positivos.

Densidad de los números racionales


El conjunto de los números racionales tiene una propiedad que no tienen el conjunto de núme-
1 2
ros completos ni el conjunto de números enteros. Considera y . Para hallar un número ra-
2 3
1 2 3
cional entre y , primero reescribimos las fracciones con un denominador común, como y
2 3 6
4
. Como no hay un número entero entre los numeradores 3 y 4, buscamos dos fracciones igua-
6
1 2 1 6 2 8 7
les, respectivamente, a y con denominador mayor. Por ejemplo, = y = ,y
2 3 2 12 3 12 12
6 8 7 1 2
está entre las dos fracciones, y . Así, está entre y . Esta propiedad se generaliza
12 12 12 2 3
como sigue y se enuncia como teorema.

Teorema 6–5: Propiedad de la densidad de los números racionales


a c
Dados dos números racionales distintos y , hay otro número racional entre ellos.
b d

AHORA INTENTA ÉSTE 6-9 Explica por qué hay una infinidad de números racionales entre dos
números racionales cualesquiera.

Ejemplo 6-3 7 1
a. Halla dos fracciones entrey .








18 2
1 2 3 4
b. Demuestra que la sucesión , , , , Á es una sucesión creciente, esto es, que cada tér-
2 3 4 5
mino comenzando por el segundo es mayor que el término precedente.

1 1#9 9 8 4 7 9
Solución a. Como = # = , vemos que , ó , está entre y . Para hallar otra
2 2 9 18 18 9 18 18
7 9
fracción entre las fracciones dadas, hallamos dos fracciones iguales a y ,
18 18
© Lopez Mateos Editores. ISBN 978-607-95583-2-1, obra completa, versión electrónica, ISBN 978-607-95583-3-8, volumen 1, versión electrónica. Ejemplar asignado a: Helecto Villarroel gutierrez -
helecto@gmail.com. Fecha: 27 de octubre de 2014. Prohibida su modificación, copia o distribución.
356 Los números racionales como fracciones

7 14
respectivamente, pero con denominador mayor; por ejemplo, =
18 36
9 18
y = .
18 36
15 16 17 14 18
Vemos ahora que , y están entre y y, por lo tanto, entre
36 36 36 36 36
7 1
y .
18 2
n
b. Como el n-ésimo término de la sucesión es , el término siguiente es
n + 1
n + 1 n + 1
1n + 12 + 1 n + 2
,o . Necesitamos demostrar que para todos los enteros

n + 1 n
positivos n, 7 . Por el teorema 6–2, la desigualdad será verda-
n + 2 n + 1
dera si, y sólo si, 1n + 121n + 12 7 n1n + 22. Esta desigualdad es equiva-

lente a n2 + 2n + 1 7 n2 + 2n, la cual es verdadera.

a c a + c
Algunos estudiantes suman incorrectamente + como . Aunque dicha técnica
b d b + d
no produce la suma correcta, como lo veremos en la sección 6-2, proporciona una manera
de hallar un número racional entre dos números racionales cualesquiera. En el ejemplo 6-3,
7 1
para hallar un número entre y podríamos sumar los numeradores y sumar los denomi-
18 2
7 + 1 8 7 8
nadores para producir = . Vemos que 6 pues 140 6 144. Además,
18 + 2 20 18 20
8 1
6 pues 16 6 20.
20 2
Enunciamos la propiedad general en el teorema siguiente, cuya demostración se explora en
Ahora intenta éste 6-10.

Teorema 6–6
a c a c
Sean y números racionales cualesquiera con denominadores positivos, donde 6 .
b d b d
a a + c c
Entonces, 6 6 .
b b + d d

a c
AHORA INTENTA ÉSTE 6-10 Demuestra el teorema 6–6; esto es, si y son números racionales
b d
a c a a + c c
cualesquiera con denominadores positivos, donde 6 , entonces 6 6 ..
b d b b + d d
a c a a + c a + c c
a Sugerencia: Si 6 , entonces ad 6 bc. Usa esto para probar que 6 y 6 .b
b d b b + d b + d d

O B S E R VA C I Ó N Nota que la demostración del teorema 6–6 sugerida en Ahora intenta


éste 6-10 también demuestra el teorema 6–5.

© Lopez Mateos Editores. ISBN 978-607-95583-2-1, obra completa, versión electrónica, ISBN 978-607-95583-3-8, volumen 1, versión electrónica. Ejemplar asignado a: Helecto Villarroel gutierrez -
helecto@gmail.com. Fecha: 27 de octubre de 2014. Prohibida su modificación, copia o distribución.
Sección 6-1 El conjunto de los números racionales 357

ACTIVIDAD DE LABORATORIO Consigue piezas de tangram o dibújalas y córtalas como se mues-


tra en la figura 6-6. Responde lo siguiente.
a. Si el área del cuadrado entero es 1 unidad cuadrada, halla el área de cada pieza del tangram.
b. Si el área de la pieza a es de 1 unidad cuadrada, halla el área de cada pieza del tangram.

b c

a d

f g

Figura 6-6

Evaluación 6-1A

7
1. Escribe una frase que ilustre el uso de de cada una de 3. Para cada uno de los cuatro cuadrados siguientes, escribe
8
las maneras siguientes: una fracción que represente la parte sombreada. ¿Qué
a. Como un problema de división propiedad de las fracciones ilustra el diagrama?
b. Como parte de un todo
c. Como una razón
2. Para cada uno de los casos siguientes, escribe una frac-
ción que represente la parte sombreada:
a. b. c. d.
4. Basado en tus observaciones, ¿podrías decir si las partes
sombreadas en las figuras siguientes representan las frac-
ciones indicadas? Di por qué.
a. b.

1 3 1
c. d. 4 4 2
a. b. c.

© Lopez Mateos Editores. ISBN 978-607-95583-2-1, obra completa, versión electrónica, ISBN 978-607-95583-3-8, volumen 1, versión electrónica. Ejemplar asignado a: Helecto Villarroel gutierrez -
helecto@gmail.com. Fecha: 27 de octubre de 2014. Prohibida su modificación, copia o distribución.
358 Los números racionales como fracciones

5. En cada caso, subdivide el todo que aparece a la derecha 12. Determina si los pares siguientes son iguales modificán-
para mostrar la fracción equivalente. dolos para que tengan el mismo denominador:
a. b. 10 12 - 21 - 51
a. y b. y
5 5 16 18 86 215
3 6
13. Traza un modelo de área para mostrar que = .
4 8
1 2 1 3
4 5 8 3 5 9 3
14. Si una fracción es igual a y la suma del numerador y el
c. 4
denominador es 84, ¿cuál es la fracción?
5
15. El señor Gómez llenó el tanque de gasolina de su carro,
con una capacidad de 16 galones. Realizó un pequeño viaje
1 3 y usó 6 galones. Traza una flecha en la siguiente figura para
2 5 6
mostrar dónde está el marcador después del viaje:
6. Usa la figura como referencia para representar cada
caso como una fracción:

a. Los puntos dentro del círculo como parte de todos


los puntos
b. Los puntos dentro del rectángulo como parte de to-
dos los puntos
c. Los puntos en la intersección del rectángulo y el cír- 16. Despeja x en cada caso:
2 x 3 - 27
culo como parte de todos los puntos a. = b. =
d. Los puntos fuera del círculo pero dentro del rectán- 3 16 4 x
gulo como parte de todos los puntos 17. Para cada uno de los siguientes pares de fracciones, re-
7. En cada caso, escribe tres fracciones iguales a la fracción emplaza la coma con el símbolo correcto 16, =, 72 para
dada: obtener una proposición verdadera:
-2 7 5 4 3 -7 -4
2 a. , b. 2 , 2 c. ,
a. b. 8 6 5 6 8 5
9 5
0 a 18. Arregla cada uno de los casos siguientes en orden decreciente:
c. d. 11 11 11
3 2 a. , ,
8. Halla la forma más simple para cada una de las fraccio- 22 16 13
- 1 - 19 - 17
nes siguientes: b. , ,
156 27 - 65 5 36 30
a. b. c. 1 2 3 4 5 6
93 45 91 19. Muestra que la sucesión , , , , , , Á (en que cada
9. En cada caso, escoge la expresión entre paréntesis que sea 3 4 5 6 7 8
igual a la fracción o la describa mejor: término sucesivo se obtiene del anterior sumando 1 al nu-

a. 11, indefinida, 02
0
merador y al denominador), es una sucesión creciente; esto
0
es, muestra que cada término de la sucesión es mayor que el
b. 1indefinida, 5, 02
5
0 precedente.
c. 1indefinida, 5, 02
0
5 20. En cada caso, encuentra dos números racionales entre
las fracciones dadas:
12, 3, no se puede simplificar2
2 + a
d. 3 4
a a. y
7 7
a
15 + x 5 + x
e. , 5, no se puede simplificarb -7 -8
3x x b. y
10. Halla la forma más simple en cada caso: 9 9
a2 - b2 14x2y 21. La escala de un mapa es de 12 millas por pulgada. ¿Cuál
a. b. es el millaje aéreo entre dos ciudades que distan 38 pulg
3a + 3b 63xy2
11. Determina si los pares siguientes son iguales: en el mapa?
3 375 18 23 22. a. ¿Qué parte son seis onzas de una libra y de una tonelada?
a. y b. y b. ¿Qué fracción de un peso son diez centavos?
8 1000 54 69
c. ¿Qué fracción de una hora son 15 min?
d. ¿Qué fracción de un día son 8 horas?
© Lopez Mateos Editores. ISBN 978-607-95583-2-1, obra completa, versión electrónica, ISBN 978-607-95583-3-8, volumen 1, versión electrónica. Ejemplar asignado a: Helecto Villarroel gutierrez -
helecto@gmail.com. Fecha: 27 de octubre de 2014. Prohibida su modificación, copia o distribución.
Sección 6-1 El conjunto de los números racionales 359

Evaluación 6-1B

7 4. ¿Podrían las partes sombreadas en las figuras siguientes


1. Escribe una frase que ilustre el uso de de cada una de
10 representar las fracciones indicadas? Di por qué.
las maneras siguientes:
a. Como un problema de división
b. Como parte de un todo
c. Como una razón
2. Para cada uno de los casos siguientes, escribe una frac-
ción que represente la parte sombreada: 1 1 2
2 8 8
a. b. c.

5. Si cada uno de los modelos siguientes representa la frac-


a. ción dada, traza un modelo que represente el todo. Som-
brea tu respuesta.
a. b.

3 4
4 3
c. d.
1 1
5 4
6. Basado en tu observación, escribe una fracción que re-
b. c. presente la parte sombreada.
3. Completa cada una de las figuras siguientes de modo que
3
ilustren :
5

0 1 7. Usa la figura como referencia para representar cada caso


como una fracción:
a. b.

a. Los puntos fuera del círculo como parte de todos los


c.
puntos
b. Los puntos fuera del rectángulo como parte de todos
los puntos
c. Los puntos en la unión del rectángulo y el círculo
como parte de todos los puntos
d.
d. Los puntos dentro del círculo pero fuera del rectán-
gulo como parte de todos los puntos
8. Halla la forma más simple para cada una de las fraccio-
nes siguientes:
0 842 662
a. b. c.
68 912 703
e. f. 9. El maestro González y la maestra Priego aplicaron el
mismo examen a sus grupos de quinto grado. En el grupo
del maestro González pasaron el examen 20 de 25 estu-

© Lopez Mateos Editores. ISBN 978-607-95583-2-1, obra completa, versión electrónica, ISBN 978-607-95583-3-8, volumen 1, versión electrónica. Ejemplar asignado a: Helecto Villarroel gutierrez -
helecto@gmail.com. Fecha: 27 de octubre de 2014. Prohibida su modificación, copia o distribución.
360 Los números racionales como fracciones

diantes y en el grupo de la maestra Priego pasaron el exa- 18. Despeja x en cada caso:
men 24 de 30 estudiantes. Una de las alumnas de la maes- 2 x
a. =
tra Priego oyó acerca de los resultados del examen y 3 18
aseguró que los dos grupos tuvieron el mismo desempeño. 3 3x
¿Tiene razón la alumna? Explica. b. = 2
x x
10. En cada caso, escoge la expresión entre paréntesis que sea a b
igual a la fracción o la describa mejor: 19. a. Si = , ¿qué debe ser cierto?
c c
a
6 + x 2 + x
a. , 3, no se puede simplificarb a a
3x x b. Si = , ¿qué debe ser cierto?
b c
26 + 25
a 1, , no se puede simplificarb
2 20. En el grupo de álgebra de Ana, 6 de 31 estudiantes obtu-
b. 4
2 + 27 3 vieron A en un examen. El mismo examen se aplicó al
2100 + 298
a 2196, , muy grande para simplificarseb
5 grupo de Beto y 5 de 23 estudiantes obtuvieron A. ¿Qué
c. 100 98
2 - 2 3 clase tiene mayor razón de A?
21. Para cada uno de los siguientes pares de fracciones, re-
11. Halla la forma más simple en cada caso: emplaza la coma con el símbolo correcto 1 6, =, 7 2 para
a2 + ab a obtener una proposición verdadera:
a. b.
a + b 3a + ab 1 1 2 4 0 0
a. - , - b. , c. ,
12. Determina si los pares siguientes son iguales: 7 8 5 10 7 17
3 3,750 17 25 c ac
a. y b. y 22. Si 0 6 a 6 b y c 7 0, d 7 0, compara el tamaño de con .
8 10,000 27 45 d bd
13. Determina si los pares siguientes son iguales modificán- 2 3 4 5 6 7
dolos para que tengan el mismo denominador: 23. Muestra que la sucesión , , , , , es una sucesión
1 2 3 4 5 6
3 - 36 - 21 - 51
a. - y b. y decreciente; esto es, muestra que cada término de la su-
12 144 430 215 cesión es menor que el precedente.
11 24. En cada caso, encuentra dos números racionales entre
14. Se necesita una tabla de exactamente pulg de ancho para
32 las fracciones dadas:
3 5 83 -1 3
tapar un hoyo. ¿Puede rebajarse una tabla de pulg para ta- a. y b. y
8 6 100 3 4
par el hoyo? Si es así, ¿cuánto debe rebajarse la tabla? 25. Considera la siguiente malla numérica. Los números en-
15. Los dos medidores siguientes tienen las lecturas que se globados forman un rombo (esto es, todos lo lados tienen
muestran. ¿A cuál medidor le queda más tiempo? la misma longitud).
¿Cuánto más? 1 2 3 4 5 6 7 8 9 10

11 12 13 14 15 16 17 18 19 20
1–
1h

21 22 23 24 25 26 27 28 29 30
h
0

31 32 33 34 35 36 37 38 39 40
Medidor A Medidor B
41 42 43 44 45 46 47 48 49 50
16. Lee cada medición mostrada en la regla siguiente:

a. A
a. Si A es la suma de los cuatro números englobados y B es
b.
A
B la suma de los cuatro números interiores, halla .
c. C
B
b. Forma un rombo englobando los números 6, 18, 25
d. D A
y 37. Calcula A y B como en (a), y después halla .
B
c. ¿Cómo se comparan las respuestas en (a) y (b)? ¿Por
0 1 2 3
qué sucede así?
Pulgadas
26. Una escala en un mapa es de 120 mi por pulgada. ¿Cuál es
17. Determina si cada uno de los casos siguientes siempre es co- 3
rrecto. De no ser así, halla cuándo es verdadero. Explica. el millaje de avión entre dos ciudades que distan pulg.
4
ab + c a + b b en el mapa?
a. = a + c b. =
b a + c c 27. a. ¿Qué parte de una libra son 12 oz?
ab + ac b + c a2 + a b. ¿Qué fracción de un peso son cinco centavos?
c. = d. = a + 1
ac c a c. ¿Qué fracción de una hora son 25 min?
d. ¿Qué fracción de un día de 24 horas son 16 horas?
© Lopez Mateos Editores. ISBN 978-607-95583-2-1, obra completa, versión electrónica, ISBN 978-607-95583-3-8, volumen 1, versión electrónica. Ejemplar asignado a: Helecto Villarroel gutierrez -
helecto@gmail.com. Fecha: 27 de octubre de 2014. Prohibida su modificación, copia o distribución.
Sección 6-1 El conjunto de los números racionales 361

Conexiones matemáticas 6-1

Comunicación Solución abierta


1. Juana tiene una receta que pide 4 tazas de harina. Ella quiere 10. Lista tres tipos de mediciones que requieran números ra-
3 cionales como la cantidad apropiada de unidades en las me-
hacer de la receta. En lugar de determinar directamente
4 diciones.
3
cuántas tazas se necesitan para la nueva receta, ella llena 11. Algunas personas argumentan que el sistema de enteros
4 se entiende mejor que el sistema de números racionales
de la taza 4 veces. Explica por qué funciona el método de Juana.
positivos. Si pudieras decidir cuál debería enseñarse pri-
3
2. Ana asegura que no puede mostrar de las caras siguientes mero en la escuela, ¿cuál escogerías y por qué?
4 12. Haz tres afirmaciones acerca de ti o de tu medio am-
pues unas son grandes y otras son pequeñas. ¿Qué le dices? biente y usa fracciones en cada una. Explica por qué tus
afirmaciones son verdaderas (por ejemplo, tus padres tie-
nen 3 hijos, dos de los cuales viven en casa; por lo tanto,
2
de sus hijos viven en casa).
3
Aprendizaje colectivo
13. Supón que la persona más alta de tu grupo tiene 1 uni-
dad de estatura y realiza lo siguiente:
2 6 a. Halla números racionales para representar de manera
3. Un estudiante asegura que = pues si sumas 4 arriba y aproximada las estaturas de otros miembros del grupo.
3 7
abajo, las fracciones deben ser iguales. ¿Cómo le respondes? b. Construye una recta numérica marcando un número
1 racional para cada persona ordenada según su estatura.
4. Si de cada uno de dos grupos son mujeres, explica si cada
3 Preguntas del salón de clase
grupo debe contener el mismo número de mujeres. 0
1 14. Un estudiante pregunta si está en su forma más simple.
5. Considera el conjunto de todas las fracciones iguales a . 6
2
¿Cómo le respondes?
Si quitas cualesquiera 10 de esas fracciones, sumas sus 15 1
numeradores para obtener el numerador de una nueva 15. Un estudiante escribe 6 porque 3 # 15 6 53 # 1. Otro
53 3
fracción y sumas sus denominadores para obtener el de-
15 1
nominador de una nueva fracción, ¿cómo se relaciona estudiante escribe = . ¿Dónde está la falacia?
1 53 3
la nueva fracción con ? Generaliza lo que hallaste y explica. 16. Los numeradores y los denominadores de las siguientes
2
6. ¿Deberían reducirse siempre las fracciones a su forma fracciones forman sucesiones aritméticas. Un estudiante
más simple? ¿Por qué sí o por qué no? asegura que las fracciones forman una sucesión aritmé-
7. ¿Cómo le responderías a cada uno de los estudiantes si- tica. ¿Cómo le responderías?
guientes?
1 2 3 4 5 6 7
a. Iris asegura que si tenemos dos números racionales , , , , , , ,Á
positivos, el que tiene el numerador más grande es el 2 3 4 5 6 7 8
999
mayor. 17. Un estudiante asegura que no hay números entre
1000
b. Sira asegura que si tenemos dos números racionales y 1 pues están muy cerca. ¿Cuál es tu respuesta?
positivos, el que tiene el mayor denominador es el m + n m
menor. 18. Un estudiante simplifica la fracción a . ¿Cómo
1 p + n p
8. Si tomamos el conjunto de fracciones equivalentes a y ayudarías a este estudiante?
3 19. Sin pensar, un estudiante asegura que una pizza cortada
las graficamos como puntos en un sistema coordenado, en 12 partes era más que una pizza cortada en 6 partes.
de modo que el numerador sea la coordenada x y el de- ¿Cómo le respondes?
nominador la coordenada y de ese punto, explica qué 20. Una estudiante pregunta si al sumar el mismo número
tipo de gráfica obtendrías. muy grande tanto al numerador como al denominador de
9. Escribe una explicación de cómo convertir pulgadas a una fracción produce el cociente 1. ¿Qué le respondes?
yardas y viceversa. a c d
21. José escribió en su tarea que si = Z 1 , entonces =
b d c - d
b
. Gabriela dijo que ella no creía eso. ¿Qué le respondes?
a - b
© Lopez Mateos Editores. ISBN 978-607-95583-2-1, obra completa, versión electrónica, ISBN 978-607-95583-3-8, volumen 1, versión electrónica. Ejemplar asignado a: Helecto Villarroel gutierrez -
helecto@gmail.com. Fecha: 27 de octubre de 2014. Prohibida su modificación, copia o distribución.
362 Los números racionales como fracciones

22. Esteban asegura que las partes sombreadas no pueden Preguntas del Third International Mathematics and
2 2 Science Study (TIMSS) (Tercera Reunión Internacional
representar pues hay 5 círculos sombreados y es
3 3 sobre el Estudio de las Matemáticas y la Ciencia)
menor que 1. ¿Cómo le respondes? 2
¿Cuál de los cuadrados sombreados muestra ?
3

A B C D E

1
23. Daria dice que cada parte de la tarta representa de la TIMSS 2003, Grado 4
3
misma. ¿Cómo le respondes? En la figura, ¿cuántos cuadrados pequeños MÁS es necesario
4
sombrear de manera que de los pequeños cuadrados es-
5
tén sombreados?
a. 5 b. 4 c. 3 d. 2 e. 1

1 1 TIMSS 2003, Grado 8


24. a. Caro notó que 5 7 4, pero que 6 . Ella piensa
5 4
que, en general, si a y b son enteros positivos y a 7 b, Preguntas del National Assessment of Educational
1 1 Progress (NAEP) (Evaluación Nacional del Progreso
entonces 6 . Ella quisiera saber si esto siempre es Educativo)
a b
cierto. ¿Cómo le respondes? ¿Qué fracción de la figura está sombreada?

b. A Caro le gustaría saber si su descubrimiento en la


parte (a) es válido cuando a y b son enteros negativos
y de ser así, por qué sí o por qué no. ¿Cómo le res-
NAEP, Grado 4, 2007
pondes?
25. A Sara le gustaría saber si la siguiente afirmación es ver- ¿En cuál de los siguientes casos están las tres fracciones
dadera y por qué sí o por qué no. ¿Cómo respondes? ordenadas de menor a mayor?
-1 1 -3 3 2 1 5 1 2 5 1 5 2
Si 6 x 6 entonces 6 x 6 . a. , , b. , , c. , ,
2 2 2 2 7 2 9 2 7 9 2 9 7
3 2 5 1 2 5 2 1
26. Carlos dice que 7 porque 3 7 2 y 8 7 3. ¿Cómo d. , , e. , ,
8 3 9 2 7 9 7 2
puedes ayudar a Carlos? NAEP, Grado 8, 2007

ROMPECABEZAS En un viejo acertijo de Sam Loyd, se describe un reloj que se detuvo cuando las
manecillas de los minutos y de las horas formaban una línea recta y la segunda manecilla no estaba en el
12. ¿A qué horas pudo pasar esto?

© Lopez Mateos Editores. ISBN 978-607-95583-2-1, obra completa, versión electrónica, ISBN 978-607-95583-3-8, volumen 1, versión electrónica. Ejemplar asignado a: Helecto Villarroel gutierrez -
helecto@gmail.com. Fecha: 27 de octubre de 2014. Prohibida su modificación, copia o distribución.
Sección 6-2 Suma, resta y estimación con números racionales 363

6-2 Suma, resta y estimación con


números racionales

La suma y la resta de números racionales son muy parecidas a la suma y la resta


de números enteros. Primero ilustraremos la suma de dos números racionales con denomi-
2 1
nadores iguales, + , usando el modelo de área de la figura 6-7(a) y el modelo de la recta
5 5
numérica de la figura 6-7(b).

(a)

◆ Nota de
investigación 2–
5
+
1–
5
=
3–
5
Los estudiantes que
2 1 3
aprenden algoritmos
__ 1 __ 5 __
5 5 5
de cálculo que inclu-
(b)
2
__ 1
__
yen fracciones tienen 5 5

dificultades para rela-


0 1
__ 2
__ 3
__ 4
__ 1
cionar modelos que 5 5 5 5
incluyen
manipulación de ob- Figura 6-7
jetos con procedi -
¿Por qué tiene sentido el modelo del área de la figura 6-7(a)? Supón que te dan, para comenzar,
mientos simbólicos. 2 1 2
La habilidad de los de un pastel y después te dan otro del pastel. En la figura 6-7(a), se representa con 2 piezas
5 5 5
estudiantes para ma- 1
cuando el pastel se parte en 5 piezas de igual tamaño, y se representa con 1 pieza de las 5 piezas
nejar un procedi - 5
miento puede de igual tamaño. De este modo, en conjunto tenemos 2 + 1 = 3 piezas de las 5 piezas de igual ta-
“rebasar” su 3
maño, ó del total (el pastel completo). El modelo de recta numérica de la figura 6-7(b) trabaja
comprensión concep- 5
igual que el modelo de recta numérica para los números completos.
tual de las fracciones,
Las ideas ilustradas en la figura 6-7 se pueden aplicar a la suma de dos números racionales
con el posible resul-
con denominadores iguales y se resumen en la definición siguiente.
tado de que los estu-
diantes sin capacidad
para juzgar si la
respuesta es razonable Definición de suma de números racionales con denominadores iguales
sólo pueden verificar a c a c a + c
Si y son números racionales, entonces + = .
su trabajo repitiendo b b b b b
el procedimiento ruti-
nario. (Wearne y
Hiebert 1988). ◆ A continuación consideramos la suma de dos números racionales con denominadores dis-
tintos usando el procedimiento de Polya de cuatro pasos para resolver problemas.

© Lopez Mateos Editores. ISBN 978-607-95583-2-1, obra completa, versión electrónica, ISBN 978-607-95583-3-8, volumen 1, versión electrónica. Ejemplar asignado a: Helecto Villarroel gutierrez -
helecto@gmail.com. Fecha: 27 de octubre de 2014. Prohibida su modificación, copia o distribución.
364 Los números racionales como fracciones

Resolver Problemas Problema de suma de números racionales


2 1
Determina cómo sumar los números racionales y .
3 4
2 1
Comprender el problema Podemos modelar
y como partes de un todo, como se ve en la
3 4
figura 6-8, pero necesitamos una manera de combinar las dos figuras para hallar la suma.

2– 1–
3 4

Figura 6-8

Trazar un plan Usamos la estrategia resolver un problema relacionado y consideramos la suma


de números racionales con denominadores iguales. Podemos hallar la suma escribiendo
cada fracción con un denominador común y después completando el cálculo.
2
Realizar el plan Del trabajo anterior en el presente capítulo, sabemos que tiene infinidad de
3
4 6 8 1
representaciones, que incluyen , , y así sucesivamente. También tiene infinidad de re-
6 9 12 4
2 3 4
presentaciones, que incluyen , , y así sucesivamente. Al comparar los dos conjuntos
8 12 16
8 3
de números racionales vemos que y tienen el mismo denominador. Uno es 8 partes de
12 12
2 1 8 3 11 12 partes iguales, mientras que el otro es 3 partes de 12 partes iguales. En consecuencia, la

3 1 — — — —
4 5 12 1 12 5 12
2 1 8 3 11
suma es + = + = . La figura 6-9 ilustra esta suma.
Figura 6-9 3 4 12 12 12

Revisar Para sumar dos números racionales con denominadores distintos, consideramos nú-
2 1
meros racionales equivalentes con denominadores iguales. El denominador común de y
3 4
es 12. También es el mínimo denominador común, o MMC de 3 y 4. Para sumar dos fraccio-
5 7
nes con denominadores distintos, tales como y , podemos hallar fracciones equivalentes
12 18
con denominador MMC(12, 18), ó 36. Sin embargo, se puede usar cualquier denominador
◆ común, por ejemplo 72 o incluso 12 # 18.

2 1 2#4 1#3 8 3 11
Al considerar la suma + = # + # = + = , podemos generalizarla
3 4 3 4 4 3 12 12 12
a la suma de dos números racionales con denominador distinto, como en el teorema
siguiente.

© Lopez Mateos Editores. ISBN 978-607-95583-2-1, obra completa, versión electrónica, ISBN 978-607-95583-3-8, volumen 1, versión electrónica. Ejemplar asignado a: Helecto Villarroel gutierrez -
helecto@gmail.com. Fecha: 27 de octubre de 2014. Prohibida su modificación, copia o distribución.
Sección 6-2 Suma, resta y estimación con números racionales 365

Teorema 6–7
a c a c ad + bc
Si y son dos números racionales cualesquiera, entonces + = .
b d b d bd

◆ Nota de Como se señala en la Nota de investigación, los estudiantes no siempre suman correc-
tamente las fracciones. En los Puntos focales para el grado 5 se espera que los estudiantes
investigación apliquen su comprensión de modelos de fracción para representar la suma y la resta de
El error más fracciones con denominadores diferentes (p. 17).
frecuente que come-
ten los estudiantes al
O B S E R VA C I Ó N De la definición de suma de números racionales con denominadores
sumar fracciones es
a c a + c
que suman los nume- iguales tenemos que + = . Se obtendría el mismo resultado si usáramos el
b b b
radores y suman los teorema 6–7:
1a + c2b
denominadores
(Bana, Farrell, and a c ab + cb a + c
+ = # = = .
McIntosh 1997). ◆ b b b b b#b b

Ejemplo 6-4 Halla las sumas siguientes:










c. a + b +
2 4 2 1 3 1 1
a. + b. - +
15 21 3 5 4 5 6
3 4 2 3
d. + e. 2 + 2
x y ab ab
2 4 2#7 4#5
Solución a. Como MMC115, 212 = 3 # 5 # 7, entonces + = + =
15 21 15 # 7 21 # 5
14 20 34
+ = .
105 105 105
2 1 122152 + 1 - 3)112 10 + - 3 7 71 - 12 -7
1 3)152
b. - + = - = - = - = - - = .
3 5 15 15 151 12 15
3#5 + 4#1
; por lo tanto, a + b +
3 1 19 3 1 1 19 1
c. + = # = = + =
4 5 4 5 20 4 5 6 20 6
19 # 6 + 20 # 1 134 67
# = ,ó .
20 6 120 60
3 4 3y 4x 3y + 4x
d. + = + = .
x y xy xy xy

e. MMC1a2b, ab22 = a2b2; 2 + 2 = 2


2 3 2b 3a 2b + 3a
+ = .
ab ab ab b# #
a ab 2
a2b2

Fracciones mixtas
En la vida diaria usamos con frecuencia fracciones mixtas, esto es, números formados por un
entero y una fracción propia. Por ejemplo, la figura 6-10 muestra que el clavo tiene
3 3 3 3
2 pulg de largo. El número mixto 2 significa 2 + . A veces se infiere que 2 significa
4 4 4 4

© Lopez Mateos Editores. ISBN 978-607-95583-2-1, obra completa, versión electrónica, ISBN 978-607-95583-3-8, volumen 1, versión electrónica. Ejemplar asignado a: Helecto Villarroel gutierrez -
helecto@gmail.com. Fecha: 27 de octubre de 2014. Prohibida su modificación, copia o distribución.
366 Los números racionales como fracciones

3 3
2 por , pues xy significa x # y, pero esto no es correcto. Además, el número - 4 significa
4 4
- a4 3 b , ó - 4 - 3 , no - 4 + 3 .
4 4 4

0 1 2 3
Pulgadas

Figura 6-10

O B S E R VA C I Ó N En una prueba del National Assessment of Educational Progress (NAEP)


(Evaluación Nacional del Progreso Educativo), se les dio a los estudiantes el problema
siguiente:
1
5 es igual a:
4
1 1 1 1
(a) 5 + (b) 5 - (c) 5 * (d) 5 ,
4 4 4 4
Sólo el 47% de los alumnos de séptimo grado, y sólo el 44% de los estudiantes de grado
once, escogieron la respuesta correcta, (a).

Una fracción mixta es un número racional y, por lo tanto, siempre se puede escribir en la
a
forma . Por ejemplo,
b
3 3 2 3 2#4 + 1#3 8 + 3 11
2 = 2 + = + = # = =
4 4 1 4 1 4 4 4

Ejemplo 6-5 a
Cambia cada una de las fracciones mixtas siguientes a la forma , donde a y b son enteros:
b








1 2
a. 4 b. - 3
3 5

1 1 4 1 4#3 + 1#1 12 + 1 13
Solución a. 4 = 4 + = + = # = =
3 3 1 3 1 3 3 3
# #
b. - 3 = - a 3 + b = - a + b = - a b = a b =
2 2 3 2 3 5 + 1 2 - 17 -17
5 5 1 5 1#5 5 5

3 3
AHORA INTENTA ÉSTE 6-11 Usa las ideas del ejemplo 6-5 para escribir 2 + 5 como una fracción mixta.
4 8

© Lopez Mateos Editores. ISBN 978-607-95583-2-1, obra completa, versión electrónica, ISBN 978-607-95583-3-8, volumen 1, versión electrónica. Ejemplar asignado a: Helecto Villarroel gutierrez -
helecto@gmail.com. Fecha: 27 de octubre de 2014. Prohibida su modificación, copia o distribución.
Sección 6-2 Suma, resta y estimación con números racionales 367

29
Ejemplo 6-6 Transforma en una fracción mixta.
5









Solución Dividimos 29 entre 5 y usamos el algoritmo de la división como sigue:
29 5#5 + 4 5#5 4 4 4
= = + = 5 + = 5
5 5 5 5 5 5

O B S E R VA C I Ó N En las escuelas elementales, problemas como el del ejemplo 6-6 usual-


mente se resuelven efectuando una división, como sigue:
5
5 冄 29
25
4
29 4 4
Por lo tanto, = 5 + = 5 .
5 5 5

Podemos usar una calculadora científica/fraccional para transformar fracciones impropias a


fracciones mixtas. Por ejemplo, si tecleamos 2 9 / 5 y presionamos Ab/c , entonces
4
aparece 5 4>5, lo cual significa 5 .
5
También podemos usar calculadoras científico/fraccionales para sumar fracciones mixtas. Por
4 5
ejemplo, para sumar 2 + 3 tecleamos 2 Unit 4 / 5 + 3 Unit 5 / 6 = , y
5 6
en pantalla aparece 5 49>30. Después presionamos Ab/c para obtener 6 19>30, lo cual
19
significa 6 .
30
Como las fracciones mixtas son números racionales, los métodos para sumar racionales se
pueden extender para incluir las fracciones mixtas. La siguiente página de muestra ilustra
un método para calcular sumas de fracciones mixtas. Resuelve los tres problemas al final de
la página de muestra. Nota que todas las fracciones en esa página son no negativas. Esto es
típico en los grados 3–6.

Propiedades de la suma de números racionales


Los números racionales tienen las siguientes propiedades para la suma: cerradura, conmutati-
vidad, asociatividad, identidad aditiva e inverso aditivo. Para acentuar la propiedad del inverso
aditivo de los números racionales, la enunciamos de manera explícita como sigue.

Teorema 6–8: Propiedad del inverso aditivo de los números racionales


a a a
Para cualquier número racional , existe un único número racional - , el inverso aditivo de ,
b b b
tal que

+ a- b = 0 = a- b +
a a a a
b b b b

© Lopez Mateos Editores. ISBN 978-607-95583-2-1, obra completa, versión electrónica, ISBN 978-607-95583-3-8, volumen 1, versión electrónica. Ejemplar asignado a: Helecto Villarroel gutierrez -
helecto@gmail.com. Fecha: 27 de octubre de 2014. Prohibida su modificación, copia o distribución.
368 Los números racionales como fracciones

Página de un libro de texto SUMA DE FRACCIONES MIXTAS

Lección 8-7

Suma de fracciones mixtas


Calentamiento
Idea clave
Puedes sumar fracciones
mixtas de la misma
manera que sumas Aprende
números completos y
fracciones.
¿Qué parecido tiene la suma de fracciones mixtas
con la suma de fracciones y números completos?
La granola
proporciona una comida de Receta de la granola
alta energía para actividades
como acampar y caminar. lb de plátano seco lb de piña seca
Luis usó la receta de la
derecha para hacer su mezcla. lb de pasas lb de cacahuate

lb de nueces diversas lb de semilla de girasol

ex
ef l i ó
R

Ejemplo

¡Piensa! ¿Cuántas libras de plátano seco y de pasas usó Luis?


• Puedo separar las
fracciones mixtas para
Halla
resolver dos problemas
PASO PASO PASO
más sencillos.
• Puedo usar
estimación para Escribe fracciones Suma las fracciones. Suma los números completos.
verificar si la equivalentes con Simplifica la suma si es
respuesta exacta es el MDC. necesario
razonable.

Así, Luis usó 3 1 libras de plátano y pasas.


8

Tema de plática
Estima la suma hallada en el ejemplo. ¿El estimado está cerca
de la respuesta real? ¿Por qué es útil comparar una respuesta
exacta con un estimado?
¿Cuántas libras de cacahuates y nueces diversas
usó Luis? Muestra esta suma en una recta numérica.
Razonamiento ¿Qué parecido tiene la suma
de fracciones mixtas con la suma de
fracciones y números completos?

Fuente: Mathematics, Diamond Edition, Grade Five, Scott Foresman-Addison Wesley, 2008 ( p. 476).

© Lopez Mateos Editores. ISBN 978-607-95583-2-1, obra completa, versión electrónica, ISBN 978-607-95583-3-8, volumen 1, versión electrónica. Ejemplar asignado a: Helecto Villarroel gutierrez -
helecto@gmail.com. Fecha: 27 de octubre de 2014. Prohibida su modificación, copia o distribución.
Sección 6-2 Suma, resta y estimación con números racionales 369

a a -a
Otra forma de - se puede hallar al considerar la suma + . Como
b b b
a -a a + -a 0
+ = = = 0
b b b b
a -a a a -a
se sigue que - y son inversos aditivos de , de modo que - = .
b b b b b

Ejemplo 6-7 Halla los inversos aditivos para cada caso:










3 -5 1
a. b. c. 4
5 11 2
3 -3 -5 - 1 - 52 -9
b. - a b =
5 1
Solución a. - ó = c. -4 , ó
5 5 11 11 11 2 2

Las propiedades del inverso aditivo para números racionales son análogas a las del inverso
aditivo para los enteros, como se muestra en la tabla 6-2. Como sucede con el conjunto de
los enteros, el conjunto de los números racionales también tiene la propiedad de la igualdad
para la suma, que dice que puedes sumar el mismo número en ambos lados de una ecuación.

Tabla 6-2
Enteros Números racionales

1. - 1 - a2 = a 1. - a - b =
a a
b b
-a -c
2. - 1a + b2 = - a + - b 2. - a + b =
a c
+
b d b d

Teorema 6–9: Propiedad de la igualdad para la suma


a c a c e
Si y son números racionales cualesquiera tales que = , y si es cualquier número
b d b d f
a e c e
racional, entonces + = + .
b f d f

O B S E R VA C I Ó N El teorema 6–9 se puede expresar como sigue: Si x = y entonces


x + z = y + z donde x, y, z son números racionales cualesquiera.

Resta de números racionales


En la escuela elemental la resta de números racionales usualmente se introduce por medio de
6 2 4
un modelo de quitar. Si tenemos de una pizza y a éste se le quitan , quedan de la pizza
7 7 7

© Lopez Mateos Editores. ISBN 978-607-95583-2-1, obra completa, versión electrónica, ISBN 978-607-95583-3-8, volumen 1, versión electrónica. Ejemplar asignado a: Helecto Villarroel gutierrez -
helecto@gmail.com. Fecha: 27 de octubre de 2014. Prohibida su modificación, copia o distribución.
370 Los números racionales como fracciones

6 2 6 - 2 4
original; esto es,- = = . En general, la resta de números racionales con de-
7 7 7 7
nominadores iguales se determina como sigue:
a c a - c
- =
b b b
Como con los enteros, se puede usar una recta numérica para modelar la resta. Si se marca

una recta en unidades de longitud , entonces - es igual a 1a - c2 unidades de longitud ,


1 a c 1
b b b b
a c a - c
lo cual implica que - = . Cuando los denominadores no son iguales, podemos
b b b
efectuar la resta hallando un denominador común. Por ejemplo,
3 2 3#3 2#4 9 8 9 - 8 1
- = # - # = - = =
4 3 4 3 3 4 12 12 12 12
La resta de números racionales, al igual que la resta de enteros, se puede definir en térmi-
nos de la suma, como sigue.

Definición de la resta de números racionales en términos de la suma


a c a c e
Si y son números racionales cualesquiera, entonces - es el único número racional
b d b d f
a c e
tal que = + .
b d f

Como sucede con los enteros, podemos ver que la resta de números racionales se puede
efectuar sumando los inversos aditivos. El siguiente teorema lo enuncia.

Teorema 6–10
a c a c a -c
Si y son números racionales cualesquiera, entonces - = + .
b d b d b d

Ahora, usando el teorema 6–10 obtenemos lo siguiente:


a c a -c
- = + (Teorema 6–10)
b d b d
ad + b1 - c2
= (Teorema 6–7)
bd
ad + - 1bc2
= (Multiplicación con enteros)
bd
ad - bc
=
bd

© Lopez Mateos Editores. ISBN 978-607-95583-2-1, obra completa, versión electrónica, ISBN 978-607-95583-3-8, volumen 1, versión electrónica. Ejemplar asignado a: Helecto Villarroel gutierrez -
helecto@gmail.com. Fecha: 27 de octubre de 2014. Prohibida su modificación, copia o distribución.
Sección 6-2 Suma, resta y estimación con números racionales 371

Demostramos el siguiente teorema:

Teorema 6–11
a c a c ad - bc
Si y son números racionales cualesquiera, entonces - = .
b d b d bd

Ejemplo 6-8 Halla cada una de las diferencias:










5 1 1 3
a. - b. 5 - 2
8 4 3 4

Solución a. Un enfoque es hallar el MMC para las fracciones. Como el MMC18, 42 = 8,


tenemos
5 1 5 2 3
- = - =
8 4 8 8 8
Un enfoque alternativo es como sigue:
5 1 5#4 - 8#1 20 - 8 12 3
- = = = , ó
8 4 8#4 32 32 8
b. Se dan dos métodos de solución:
1 4 4 16 1 3 16 11
5 = 5 = 4 + 1 = 4 5 - 2 = -
3 12 12 12 3 4 3 4
3 9 9 9 16 4 - 3 # 11
#
- 2 = -2 = -2 = -2 =
4 12 12 12 3#4
7 64 - 33
2 =
12 12
31 7
= , ó2
12 12

Los siguientes ejemplos ilustran el uso de fracciones en álgebra.

Ejemplo 6-9 Suma o resta cada caso. Escribe tu respuesta en la forma más sencilla posible.








x x 2 - x 4 - 2x
a. + b. +
2 3 6 - 3x 3x - 6
2 2 1 1
c. - d. - 2
a + b a - b x 2x

x x 3x 2x
Solución a. + = # + #
2 3 3 2 2 3
3x + 2x 5x
= =
6 6

© Lopez Mateos Editores. ISBN 978-607-95583-2-1, obra completa, versión electrónica, ISBN 978-607-95583-3-8, volumen 1, versión electrónica. Ejemplar asignado a: Helecto Villarroel gutierrez -
helecto@gmail.com. Fecha: 27 de octubre de 2014. Prohibida su modificación, copia o distribución.
372 Los números racionales como fracciones

b. Primero escribimos cada fracción en su forma más simple:


2 - x 2 - x 1 # 12 - x2 1
3 12 - x2
= = # = si x Z 2
6 - 3x 312 - x2 3

4 - 2x - 21x - 22 -2
= = si x Z 2
3x - 6 31x - 22 3
1 -2 -1
Así, si x Z 2, la suma es + = .
3 3 3
c. Usando el teorema 6–11:
2 2 21a - b2 - 21a + b2
1a + b21a - b2
- =
a + b a - b
2a - 2b - 2a - 2b
1a + b21a - b2
=
- 4b - 4b
1a + b21a - b2 a - b2
= ó 2

1 1 2x # 1 1
d. - 2 = - 2
x 2x 2x # x 2x
2x 1
= 2 - 2
2x 2x
2x - 1
=
2x2

Definición de mayor que y menor que en términos de la


resta o substracción
Para enteros a y b, a 6 b (o, de manera equivalente, b 7 a) si, y sólo si, existe un entero po-
sitivo k tal que a + k = b, de donde se sigue que b - a = k y por lo tanto b - a 7 0. A conti-
nuación damos una definición de mayor que y menor que para números racionales:

Definición de menor que y mayor que para números racionales


a c c a
6 si - 7 0,
b d d b
c a a c
7 si, y sólo si, 6 .
d b b d

Esta definición es útil para justificar ciertas desigualdades. En el ejemplo 6-10 se usa la
definición de mayor que y se practica con álgebra.

Ejemplo 6-10 (a) Verifica cada caso:










2 3 3 4 5 7
(i) + 7 2 (ii) + 7 2 (iii) + 7 2
3 2 4 3 7 5

© Lopez Mateos Editores. ISBN 978-607-95583-2-1, obra completa, versión electrónica, ISBN 978-607-95583-3-8, volumen 1, versión electrónica. Ejemplar asignado a: Helecto Villarroel gutierrez -
helecto@gmail.com. Fecha: 27 de octubre de 2014. Prohibida su modificación, copia o distribución.
Sección 6-2 Suma, resta y estimación con números racionales 373

(b) Basado en estos ejemplos, realiza una conjetura y justifícala.

2 3 4 + 9 - 12 1 2 3
Solución (a) (i) + - 2 = = 7 0, por tanto + 7 2
3 2 6 6 3 2
3 4 9 + 16 - 24 1 3 4
(ii) + - 2 = = 7 0, por tanto + 7 2
4 3 12 12 4 3
5 7 25 + 49 - 70 4 5 7
(iii) + - 2 = = 7 0, por tanto + 7 2
7 5 35 35 7 5
a b
(b) Conjetura: Si a y b son enteros positivos y a Z b, entonces + 7 2.
b a
a b a2 + b2 - 2ab
Justificación: + - 2 =
b a ab
1a - b22
= 7 0, pues a Z b, y ab 7 0.
ab

Estimación con números racionales
La estimación nos ayuda a tomar decisiones prácticas en la vida real. Por ejemplo, supón que
3
necesitamos duplicar una receta que nos pide de taza de harina. ¿Necesitaremos más o menos de
8
una taza de harina? La mayoría de las técnicas de estimación y de matemática mental que hemos
aprendido a usar con los números completos también funcionan con los números racionales.
En los Puntos focales para el grado 5 se pide que los estudiantes hagan “estimados razona-
bles de sumas y restas de fracciones”. En la página de muestra (p. 374) de un libro de texto
de grado 5 se pide a los estudiantes que estimen fracciones de tablas para construir repisas.
La estimación desempeña un papel importante para juzgar si un cálculo es razonable. Los
estudiantes no necesariamente cuentan con esta habilidad. Por ejemplo, cuando se les pide
12 7
estimar + , sólo 24% de los estudiantes de 13 años, en una evaluación nacional, dije-
13 8
ron que la respuesta estaba cerca de 2. La mayoría dijo que estaba cerca de 1, 19 ó 21. Estos
estimados incorrectos sugieren que hay errores de cálculo comunes al sumar fracciones y una
falta de comprensión de la operación que se está realizando. Estos estimados incorrectos tam-
bién sugieren una falta de sentido numérico.

3 1 4
AHORA INTENTA ÉSTE 6-12 Un estudiante sumó + y obtuvo . ¿Cómo puedes usar la estima-
4 2 6
ción para mostrar a este estudiante que su respuesta podría no ser correcta?

A veces, para obtener un estimado es deseable redondear fracciones a una fracción conveniente, co-
1 1 1 1 2 3
mo , , , , , , ó 1. Por ejemplo, si un estudiante tuvo 59 respuestas correctas de 80 pregun-
2 3 4 5 3 4
59 60 3
tas, entonces respondió correctamente de las preguntas, lo cual es aproximadamente , ó . Sa-
80 80 4
60 59
bemos que es mayor que . En una recta numérica, la fracción mayor está a la derecha de la
80 80

© Lopez Mateos Editores. ISBN 978-607-95583-2-1, obra completa, versión electrónica, ISBN 978-607-95583-3-8, volumen 1, versión electrónica. Ejemplar asignado a: Helecto Villarroel gutierrez -
helecto@gmail.com. Fecha: 27 de octubre de 2014. Prohibida su modificación, copia o distribución.
374 Los números racionales como fracciones

Página de un libro de texto ESTIMACIÓN DE CANTIDADES


FRACCIONARIAS

Lección 7-4
Calentamiento
Idea clave
Pensar en fracciones Estimación de Traza un modelo para

cantidades fraccionarias
de referencia te ilustrar cada fracción.
puede ayudar a
estimar cantidades
fraccionarias. Aprende
Vocabulario ¿Cómo puedes estimar
fracciones de
referencia cantidades fraccionarias?
Un grupo de quinto grado está fabricando unas repisas para libros.
La tabla mostrada abajo está completa. La otra tabla se
ha cortado. ¿Alrededor de qué fracción queda?

Piensa en fracciones de referencia. Las fracciones de


referencia son 1, 1 , 1, 2 y 3. Una longitud que represente
4 3 23 4
toda la tabla se puede cortar en 2, 3 ó 4 piezas iguales
para representar estas fracciones de referencia.

2
alrededor de 3

Queda alrededor de 2 de la tabla.


3

Ejemplo A Ejemplo B
¿Alrededor de qué fracción ¿Alrededor de qué fracción del
de la repisa está pintada? pay se ha comido?

Alrededor de 3 de la tabla Alrededor de 12 del pay se ha


4
está pintada. comido.

Tema de plática
¿Cuál es la fracción de referencia en el ejemplo
A y en el ejemplo B?
Traza un rectángulo y sombrea alrededor de 13 de él.

Fuente: Mathematics, Diamond Edition, Grade Five, Scott Foresman-Addison Wesley 2008 ( p. 402).

© Lopez Mateos Editores. ISBN 978-607-95583-2-1, obra completa, versión electrónica, ISBN 978-607-95583-3-8, volumen 1, versión electrónica. Ejemplar asignado a: Helecto Villarroel gutierrez -
helecto@gmail.com. Fecha: 27 de octubre de 2014. Prohibida su modificación, copia o distribución.
Sección 6-2 Suma, resta y estimación con números racionales 375

3 59
menor. El estimado depara es un estimado alto. De manera similar, podemos estimar
4 80
31 30 1 1
mediante , ó . En este caso el estimado de es bajo.
90 90 3 3

Ejemplo 6-11 Un grupo de sexto grado está recolectando latas para llevarlas al centro de reciclado. El








▼ grupo de Berta trajo las siguientes cantidades (en libras). ¿Alrededor de cuántas libras
juntó todo el grupo?
1 4 7 6
1 ,3 ,5 ,
8 10 8 10

Solución Podemos estimar la cantidad usando la estimación de izquierda a derecha y después


1
ajustando con 0, y 1 como puntos de referencia. La estimación de izquierda a derecha es
2

11 + 3 + 52, ó 9. El ajuste es a0 + + 1 + b, ó 2. Un estimado ajustado sería de 9 + 2 u


1 1
2 2
11 libras.

Ejemplo 6-12 Estima cada uno de los casos siguientes:










27 10 9 7 11
a. + b. 3 + 2 +
13 9 10 8 12

27 10
Solución a. Como es un poco más que 2 y es un poco más que 1, un estimado será
13 9
un número cercano pero mayor que 3.

b. Primero sumamos las partes de números completos para obtener 3 + 2, ó 5.


9 7 11
Como cada una de las fracciones, , y , está cerca pero es menor que 1, su
10 8 12
suma está cerca pero es menor que 3. La respuesta aproximada es 5 + 3 u 8.

Evaluación 6-2A

1. Efectúa las siguientes sumas o restas: 5 1


e. + 2
1 2 6 8
a. +
2 3 1 1
f. - 4 - 3
4 2 2 6
b. -
12 3 2. Transforma cada una de las fracciones siguientes a frac-
5 -3 ciones mixtas:
c. + 56 293
x y
-3 a. b. -
5 7 3 100
d. 2 + 2
+ 2
2x y 2xy x

© Lopez Mateos Editores. ISBN 978-607-95583-2-1, obra completa, versión electrónica, ISBN 978-607-95583-3-8, volumen 1, versión electrónica. Ejemplar asignado a: Helecto Villarroel gutierrez -
helecto@gmail.com. Fecha: 27 de octubre de 2014. Prohibida su modificación, copia o distribución.
376 Los números racionales como fracciones

3. Transforma cada una de las fracciones mixtas en fracciones 9. Calcula mentalmente cada caso:
a 3 3 1 5
de la forma , donde a y b son enteros y b Z 0: a. 1 - b. 3 + 2 - 5
b 4 8 4 8
3 5 10. La siguiente regla tiene regiones marcadas M, A, T, E:
a. 6 b. - 3
4 8
M A T E
4. Aproxima cada una de las situaciones siguientes por me-
dio de una fracción conveniente. Explica tu razonamiento.
Di si tu estimado es alto o bajo. 1 2 3 4 5
a. Jorge bateó 15 hits en 46 turnos al bat. Regla en pulgadas
b. Rosa metió 7 goles en 41 tiros.
c. Laura respondió correctamente 62 problemas de 80. Usa matemática mental y estimación para determinar en qué
d. Juan hizo 9 canastas en 19 tiros. región cae cada uno de los casos siguientes (por ejemplo,
5. Usa la información de la tabla para responder cada una 12
pulg cae en la región A):
de las preguntas siguientes: 5
20 36
a. pulg b. pulg
Equipo Partidos jugados Partidos ganados 8 8
60 18
Patos 22 10 c. pulg d. pulg
16 4
Castores 19 10
11. Usa cúmulos para estimar la suma siguiente:
Tigres 28 9
Osos 23 8 1 1 7 2
3
+ 3 + 2 + 2
Leones 27 7 3 5 8 9
Gatos 25 6 2 1 1
12. Un grupo consta de de primer año, de segundo año,
Tejones 21 5 5 4 10
de tercer año y el resto es de especialidad. ¿Qué fracción del
grupo son alumnos de especialidad?
1
a. ¿Qué equipo ganó más de de sus partidos, pero es- 13. Un dependiente vendió tres piezas de un tipo de cinta a tres
2 1
1 clientes distintos. Una pieza tenía yd de longitud, otra te-
tuvo más cerca de ganar de sus partidos? 3
2 3 1
1 nía 2 yd de longitud y la tercera tenía 3 yd de longitud.
b. ¿Qué equipos ganaron menos de de sus partidos? 4 2
2 ¿Cuál es la longitud total vendida de ese tipo de cinta?
1
c. ¿Qué equipos ganaron más de de sus partidos? 3
3 14. Martina compró 8 yd de tela. Piensa hacer una falda usan-
6. Ordena las siguientes cartas con fracciones en los óvalos 4
7 3
estimando a qué óvalo pertenece la fracción: do 1 yd, unos pantalones usando 2 yd y un chaleco
8 8
Ordena estas cartas Alrededor Alrededor 2
de fracciones de 0 Alrededor de 1– de 1 usando 1 yd. ¿Cuánta tela le va a sobrar?
2 3
—1 4– —8 15. Los estudiantes de una secundaria formaron cuatro equipos
10 7 12
con la finalidad de recolectar latas para reciclar, durante los
7– 2– 1 9 meses de abril y mayo. Los estudiantes reciben 10¢ por
—– —
8 5 100 18 cada 5 lb de latas. A continuación se presenta un registro de
su esfuerzo:
1 1 3
7. Aproxima cada una de las fracciones siguientes por 0, , , ,
4 2 4 Cantidad de libras recolectadas
ó 1. Dí si tu estimado es alto o bajo.
19 3 Equipo 1 Equipo 2 Equipo 3 Equipo 4
a. b.
39 197 3 7 1 3
150 8 Abril 28 32 28 35
c. d. 4 8 2 16
201 9
8. Sin hallar en realidad la respuesta exacta, di cuál de los 1 5 3 1
Mayo 33 28 25 41
números dados entre paréntesis en los casos siguientes es 3 12 4 2
la mejor aproximación para la suma o resta dada:
a 1, 2, 3, 3 b
6 7 11 17 1 a. ¿Qué equipo recolectó más durante los dos meses?
a. + + +
13 15 23 35 2 ¿Cuánto recolectaron?
a , , 1, 2 b
30 1 3 3 3 b. ¿Cuál fue la diferencia en las cantidades totales reco-
b. + +
41 1000 2000 8 4 lectadas por todos los equipos durante los dos meses?
© Lopez Mateos Editores. ISBN 978-607-95583-2-1, obra completa, versión electrónica, ISBN 978-607-95583-3-8, volumen 1, versión electrónica. Ejemplar asignado a: Helecto Villarroel gutierrez -
helecto@gmail.com. Fecha: 27 de octubre de 2014. Prohibida su modificación, copia o distribución.
Sección 6-2 Suma, resta y estimación con números racionales 377

16. Ilustra con ejemplos cada una de las siguientes propieda- 22. a. Verifica que cada uno de los casos siguientes sea verdadero:
des de los números racionales: 1 1 1
(i) = + #
a. Cerradura b. Conmutatividad c. Asociatividad 3 4 3 4
17. Dado que la sucesión de la parte (a) es aritmética y en la 1 1 1
parte (b) la sucesión de los numeradores es aritmética y (ii) = + #
4 5 4 5
también lo es la sucesión de los denominadores, responde 1 1 1
a lo siguiente: (iii) = + #
5 6 5 6
(i) Escribe tres términos más de cada sucesión.
1
(ii) ¿Es aritmética la sucesión de la parte (b)? Justifica tu b. Con base en los ejemplos en (a), escribe como suma
r respuesta. n
1 1 3 5 de dos fracciones unitarias, esto es, como suma de
a. , , , 1, , Á fracciones con numerador 1.
4 2 4 4
23. Despejar x en cada caso:
1 2 3 4 5
b. , , , , , Á 1 1
2 3 4 5 6 a. x + 2 = 3
2 3
18. Halla el término n-ésimo en cada una de las sucesiones del
2 5
problema 17. b. x - 2 =
19. Inserta cinco fracciones entre los números 1 y 2 de modo 3 6
que los siete números (incluyendo 1 y 2) formen una su- 24. Halla cada suma o diferencia; simplifica, de ser posible.
3x y
cesión aritmética. a. 2 + 2
xy x
20. Sea f 1x2 = x +
3
, donde el dominio es el conjunto de nú- a b
4 b. 2 -
meros racionales. xy xyz
a. Halla las salidas si las entradas son las siguientes: a2 a - b
4 -3 c. 2 2
-
(i) 0 (ii) (iii) a - b a + b
3 4
b. ¿Qué entradas producen las salidas siguientes?
1
(i) 1 (ii) - 1 (iii)
2
x + 2
21. Sea f1x2 = y sea el dominio de la función el con-
x - 1
junto de todos los enteros excepto 1. Halla lo siguiente:
a. f102 b. f1 - 22
c. f 1 52
- d. f 152

Evaluación 6-2B

1. Efectúa las siguientes sumas o restas: 3. Transforma cada una de las siguientes fracciones mixtas en
-1 2 5 2 a
a. + b. - la forma , donde a y b son enteros y b Z 0.
2 3 12 3 b
5 -3 1
c. + a. 7
4x 2y 2
-3 5 7 2
d. 2 2 + + 2 b. - 4
2x y 2xy 2
xy 3
5 1 1 1 4. Coloca los números 2, 5, 6 y 8 en los cuadros siguientes
e. - 2 f. - 4 + 3 para que la ecuación sea verdadera:
6 8 2 6
2. Transforma cada una de las fracciones siguientes a frac-
ciones mixtas: n n 23
+ =
14 47 n n 24
a. b. -
5 8

© Lopez Mateos Editores. ISBN 978-607-95583-2-1, obra completa, versión electrónica, ISBN 978-607-95583-3-8, volumen 1, versión electrónica. Ejemplar asignado a: Helecto Villarroel gutierrez -
helecto@gmail.com. Fecha: 27 de octubre de 2014. Prohibida su modificación, copia o distribución.
378 Los números racionales como fracciones

5. Usa la información de la tabla para responder cada una Usa matemática mental y estimación para determinar en qué
de las preguntas siguientes: región cae cada uno de los casos siguientes (por ejemplo,
12
pulg cae en la región A):
Equipo Partidos jugados Partidos ganados 5
9 26
Patos 22 10 a. pulg b. pulg
8 8
Castores 19 10
50 14
Tigres 28 9 c. pulg d. pulg
16 4
Osos 23 8
1 1 1
Leones 27 7 11. Un grupo consta de de primer año, de segundo año,
4 5 10
Gatos 25 6 de tercer año y el resto es de especialidad. ¿Qué fracción del
Tejones 21 5 grupo son alumnos de especialidad?
12. La compañía Naturales vende sus productos en muchos
1 países. Las dos gráficas circulares siguientes muestran las
a. ¿Qué equipos ganaron menos de de sus partidos?
3 fracciones de las ganancias de la compañía para los años
1 2004 y 2009. Con base en esta información, responde las
b. ¿Qué equipo ganó más de de sus partidos? siguientes preguntas:
4
1 a. En el año 2004, ¿en cuánto excedió la fracción de
c. ¿Qué equipos ganaron casi menos de de sus partidos? ventas en Japón a la de Canadá?
4
b. En 2009, ¿cuánto faltó para que la fracción de ventas
6. Ordena las siguientes cartas con fracciones en los óvalos
en Inglaterra fuera la de Estados Unidos?
estimando a qué óvalo pertenece la fracción:
c. ¿En cuánto excedió la fracción de ventas totales para
Estados Unidos en 2009 a la de 2004?
Ordena estas cartas Alrededor Alrededor de 1– Alrededor
de fracciones de 0 2 de 1
d. Es cierto que la cantidad total de ventas en dólares en
Australia fue menor en 2004 que en 2009? ¿Por qué?
1
— 36
— 19
— —1 7

100 70 36 30 800
Fracciones de ventas
totales, 2004
Estados Canadá
Unidos
1– 1–
1 1 3 3 6
7. Aproxima cada una de las fracciones siguientes por 0, , , , Australia
4 2 4
1 1– 1–
ó 1. Di si tu estimado es alto o bajo. —
20 5 4
113 3
a. b.
100 1978
150 8
c. d. Japón Inglaterra
198 9
8. Sin hallar en realidad la respuesta exacta, di cuál de los
números dados entre paréntesis en los casos siguientes es
Fracciones de ventas
la mejor aproximación para la suma o resta dada: totales, 2009
a 1, 2, 3, 3 b
6 7 12 18 1 Estados Canadá
a. + + + Unidos
13 15 23 35 2 7
— 1–
a , , 1, 2b
30 3 5 3 3 20 6
b. + +
41 1000 2000 8 4
1 1–
9. Calcula mentalmente cada caso: —
10 2
7 3 1 3 — 4
15
a. 6 - b. 2 + 4 + 3
8 5 10 10 Australia
10. La siguiente regla tiene regiones marcadas M, A, T, E:
Japón Inglaterra
M A T E
1 3
13. Una receta requiere 3 tazas de leche. Ramón puso 1
2 4
1 2 3 4 5 tazas y vació el recipiente. ¿Cuánta más leche necesita
Regla en pulgadas agregar?

© Lopez Mateos Editores. ISBN 978-607-95583-2-1, obra completa, versión electrónica, ISBN 978-607-95583-3-8, volumen 1, versión electrónica. Ejemplar asignado a: Helecto Villarroel gutierrez -
helecto@gmail.com. Fecha: 27 de octubre de 2014. Prohibida su modificación, copia o distribución.
Sección 6-2 Suma, resta y estimación con números racionales 379

3 1 . Verifica que cada uno de los casos siguientes sea ver-


14. Una tabla de 15 pulg se corta de una tabla de 38 pulg. La
4 4 dadero:
3 1 1 1 1
sierra se lleva pulg. ¿Cuánto queda de la tabla de 38 pulg (i) - = #
8 4 3 4 3 4
después del corte?
1 1 1
15. En cada una de las siguientes sucesiones, los numeradores (ii) - = #
forman una sucesión aritmética. Escribe tres términos más 4 5 4 5
de la sucesión y determina cuáles de las sucesiones son arit- 1 1 1
(iii) - = #
méticas y cuáles no. Justifica tus respuestas. 5 6 5 6
2 5 8 11 14 5 3 1 -1 -3 1
a. , , , , , Á b. , , , , ,Á b. Con base en la parte (a), escribe como una dife-
3 3 3 3 3 4 4 4 4 4 n1n + 12
16. Halla el término n-ésimo en cada una de las sucesiones del rencia de dos fracciones con numerador 1. Justifica tu
problema 15. respuesta.
17. Inserta cuatro fracciones entre los números 1 y 3 de 21. Despeja x en cada caso:
modo que los seis números (incluyendo 1 y 3) formen 5 2
a. x - =
una sucesión aritmética. 6 3
3 7 5
18. Sea f1x2 = x - , donde el dominio es el conjunto de b. x - 3 2 = 2 2
4 2 3 # 2 #3
números racionales. 22. Halla la suma o diferencia.
a. Halla las salidas si las entradas son las siguientes: 3x a
4 -3 a. 2 + 2
(i) 0 (ii) (iii) xy x
3 4 a b 1
b. ¿Qué entradas producen las salidas siguientes? b. 2 - + 2
xy xyz xy
1
(i) 1 (ii) - 1 (iii)
2
x + 2 1
19. Sea f1x2 = + y sea el dominio de la función el
x - 1 3
conjunto de todos los enteros excepto 1. Halla lo siguiente:
a. f102 b. f1 - 22 d. f152 20 .
-
c. f1 52 a

Conexiones matemáticas 6-2

Comunicación 4. Explica por qué podemos hacer lo siguiente para conver-


1. Supón que una pizza grande se divide en 3 partes iguales y 3
tir 5 en un número mixto:
una pizza pequeña se divide en 4 partes iguales y tú tienes 4
1 1 5#4 + 3 23
una parte de cada pizza. ¿El número + representa la =
3 4 4 4
cantidad que recibiste? Explica por qué sí o por qué no. 3 11
2. Explica por qué escogemos un denominador común para 5. Para mostrar 2 = , la maestra dibujó la figura si-
4 4
1 3 11 11
sumar + . guiente. Cristina dijo que la figura era de , no de .
3 4 12 4
3. a. Cuando sumamos dos fracciones con denominadores ¿Qué estaba pensando Cristina y cómo le deberá respon-
distintos y las convertimos a fracciones con el mismo der la maestra?
denominador, ¿debemos usar el mínimo denomina-
dor común? ¿Cuáles son las ventajas de usar el mí-
nimo denominador común?
b. Cuando usamos el mínimo denominador común al
sumar o restar fracciones, ¿el resultado es siempre
una fracción en su forma más simple?

© Lopez Mateos Editores. ISBN 978-607-95583-2-1, obra completa, versión electrónica, ISBN 978-607-95583-3-8, volumen 1, versión electrónica. Ejemplar asignado a: Helecto Villarroel gutierrez -
helecto@gmail.com. Fecha: 27 de octubre de 2014. Prohibida su modificación, copia o distribución.
380 Los números racionales como fracciones

1 1 2
6. Carla gastó de su ingreso el sábado y de lo que le 14. a. Con excepción de , los egipcios usaban sólo fracciones
2 3 3
quedó, el domingo. ¿Puede modelarse esta situación unitarias (fracciones que tienen numerador 1).
1 1 Toda fracción unitaria se puede expresar como la
como - ? Explica por qué sí o por qué no. suma de dos fracciones unitarias en más de una sola ma-
2 3 1 1 1 1 1 1
3 1 nera, por ejemplo, = + y = + . Halla al
7. Resuelve 3 + 5 de dos diferentes maneras y deja tu 2 4 4 2 3 6
4 3 menos dos representaciones en fracciones unitarias di-
respuesta como un número mixto. Di qué camino prefie- ferentes en cada caso:
res y por qué. 1
8. Sandra asegura que es más fácil sumar dos fracciones si (i)
3
suma los numeradores y después los denominadores. 1
¿Cómo la puedes ayudar? (ii)
7
9. ¿Es válida cada una de las propiedades siguientes para la
1 1
resta de números racionales? Justifica tu respuesta. b. Calcula - y simplifica tu respuesta.
a. Cerradura b. Commutativa n n + 1
c. Asociatividad d. Identidad c. Reescribe la parte (b) como una suma y después usa la
e. Inverso suma para responder la pregunta de la parte (a).
10. Explica un error de patrón en cada caso: 1
Escribe como suma de dos fracciones unitarias
13 1 27 2 16 1 17
a. = , = , = diferentes.
35 5 73 3 64 4
4 2 6 2 3 5 7 1 8 Aprendizaje colectivo
b. + = , + = , + =
5 3 8 5 4 9 8 3 11 15. Entrevista a 10 personas y pregúntales si han sumado y
3 1 2 3 2 1 2 1 1 restado fracciones en su vida, y cuándo. Combina sus
c. 8 - 6 = 2 , 5 - 2 = 3 , 2 - 1 = 1
8 4 4 8 3 5 7 3 4 respuestas con las del resto de tu grupo para tener una
2 6 1 6 4# 8 idea de cómo las “personas comunes” usan cálculos de
d. # 3 = , # 6 = , 2 = números racionales en su vida diaria.
3 9 4 24 5 10
1 1 1 1 Preguntas del salón de clase
11. Nota que la sucesión 1, , , , , Á , cuyo término n-ési-
2 3 4 5 a c d
1 16. José escribió en su tarea que si = , entonces =
mo es , es una sucesión decreciente. Explica cómo usar b d c + d
n b
este hecho para determinar si las siguientes sucesiones . Gabriela dijo que ella no creía eso. ¿Qué le respondes?
a + b
son crecientes o decrecientes: 1 3 4
17. Queta mostró que + = usando la figura siguiente.
3 4 7
1 1 1 1 1
a. 2, 1 , 1 , 1 , 1 , Á ,1 ¿Cómo puedes ayudarla?
2 3 4 5 n
a Sugerencia:
1 2 3 4 n 2 1
b. , , , , Á , =1 - , 1 5
2 3 4 5 n + 1 3 3
=1 - b
3 1
1 3 4
4 4 3 1 4 5 7
1 1 1 1 1
c. 1, 2 , 2 , 2 , 2 , Á , 2 a a
2 3 4 5 n 18. Joaquín asegura que para fracciones positivas, + =
b c
Solución abierta a
pues las fracciones tienen un común numerador.
2 1 b + c
12. Inventa un problema para - .
3 4 ¿Cómo le respondes?
13. a. Escribe dos fracciones cuya suma sea 1. ¿Si una de las
a Problemas de repaso
fracciones es , cuál es la otra?
b 19. Simplifica si es posible.
b. Escribe tres fracciones cuya suma sea 1. 14
a.
c. Escribe dos fracciones cuya diferencia esté muy cerca 21
de 1 pero que no sea exactamente 1. 117
b.
153

© Lopez Mateos Editores. ISBN 978-607-95583-2-1, obra completa, versión electrónica, ISBN 978-607-95583-3-8, volumen 1, versión electrónica. Ejemplar asignado a: Helecto Villarroel gutierrez -
helecto@gmail.com. Fecha: 27 de octubre de 2014. Prohibida su modificación, copia o distribución.
Sección 6-3 Multiplicación y división de números racionales 381

52 22. a. Si se suma el mismo número positivo al numerador y


c. 2 al denominador de una fracción positiva propia, ¿es la
7
nueva fracción mayor, menor o igual que la fracción
a2 + a
d. original? Justifica tu respuesta.
1 + a b. ¿Qué sucede si el mismo número positivo, menor que
a2 + 1 el numerador y menor que el denominador, se subs-
e.
a + 1 trae del numerador y del denominador de una frac-
a2 - b2 ción propia positiva y el nuevo denominador es
f.
a - b positivo?
20. Determina si las fracciones en cada uno de los pares si-
Pregunta del Third International Mathematics and Science
guientes son iguales:
Study (TIMSS) (Tercer Estudio Internacional sobre las
a2 a2b
a. y 2 Matemáticas y la Ciencia)
b b
Juan, María y su mamá estaban comiendo un pastel. Juan
377 378
b. y 1 1
400 401 comió del pastel. María comió del pastel. Su mamá
2 4
0 0
c. y 1
10 - 10 comió del pastel. ¿Cuánto quedó del pastel?
4
a a + 1
d. y , donde a Z b 3 1 1
b b + 1 a. b. c. d. Nada
4 2 4
21. Si suponemos que 1 año = 365 días, responde lo siguiente:
a. ¿Qué mes del año tiene la menor fracción de días del TIMSS 2007, Grado 4
año?
b. ¿Qué fracción de días del año transcurre antes del 15
de septiembre?

6-3 Multiplicación y división de números racionales

Multiplicación de números racionales


Para motivar la definición de multiplicación de números racionales, usamos la interpre-
tación de multiplicación como suma repetida. Usando la suma repetida, podemos inter-

pretar 3 # a b como sigue:


3
4
3# a b =
3 3 3 3 9 1
+ + = = 2
4 4 4 4 4 4

El modelo de área en la figura 6-11 es otra manera de calcular este producto.

3 3 =
. — 3 + 3 + 3 = 9 , ó 1
— — — — 2—
4 4 4 4 4 4

Figura 6-11

© Lopez Mateos Editores. ISBN 978-607-95583-2-1, obra completa, versión electrónica, ISBN 978-607-95583-3-8, volumen 1, versión electrónica. Ejemplar asignado a: Helecto Villarroel gutierrez -
helecto@gmail.com. Fecha: 27 de octubre de 2014. Prohibida su modificación, copia o distribución.
382 Los números racionales como fracciones

A continuación considera a b # 3. ¿Cómo debemos interpretar este producto? Si es válida


3
4
la propiedad conmutativa de la multiplicación de números racionales, entonces a b # 3 =
3
4
3 a b = .
# 3 9
4 4
3 3
Ahora presentamos otra interpretación de la multiplicación. ¿Qué es de 3? Recuerda que
4 4
de una cantidad es el resultado de dividir la cantidad en 4 partes iguales y tomar 3 de estas par-
1 1 1
tes. Para ver qué es de 3 toma de 3 barras de igual tamaño. Esto se puede hacer tomando
4 4 4
1 1 1 3 3 3 3 3 3
de cada una de las 3 barras, esto es, + + ó . Así de 3 es + + , ó 3 # , lo cual,
4 4 4 4 4 4 4 4 4
3# 3# 3
según vimos arriba, es igual a 3. Así, podemos interpretar 3 como de 3 y en general,
4 4 4
a#c a c
para números racionales no negativos se puede representar como de .
b d b d
3 1
Si la vegetación alguna vez cubrió alrededor de de la Tierra y ahora sólo queda de esta
5 2
vegetación, ¿qué fracción de la Tierra está cubierta de vegetación hoy día? Necesitamos ha-
1 3
llar de , y podemos usar un modelo de área para obtener la respuesta.
2 5
3– 3
5 La figura 6-12(a) muestra un rectángulo que representa el todo separado en quintos, con
5
(a) 1 3
sombreado. Para hallar de , dividimos la porción sombreada del rectángulo de la figura
2 5
6-12(a) en dos partes congruentes y tomamos una de esas partes. El resultado será la porción
1– 3
2 verde de la figura 6-12(b). Sin embargo, la porción verde representa 3 partes de 10, ó , del
10
todo. Así,
3– 1#3 3 1#3
5 = = #
(b) 2 5 10 2 5

Figura 6-12 Este análisis conduce a la siguiente definición de multiplicación para números racionales.

Definición de multiplicación de números racionales


a c a c a#c
Si y son números racionales cualesquiera, entonces # = # .
b d b d b d

O B S E R VA C I Ó N
a#c a c
1. Nota que la definición anterior se sigue de la interpretación de como de .
b d b d
2. También nota que al multiplicar fracciones es posible y más eficiente “cancelar” fac-
tores comunes antes de multiplicar los numeradores y los denominadores.

Un modelo de área como el de la figura 6-12 se usa en la página de muestra que sigue (p. 383).

AHORA INTENTA ÉSTE 6-13 Responde las preguntas en la página de muestra.

© Lopez Mateos Editores. ISBN 978-607-95583-2-1, obra completa, versión electrónica, ISBN 978-607-95583-3-8, volumen 1, versión electrónica. Ejemplar asignado a: Helecto Villarroel gutierrez -
helecto@gmail.com. Fecha: 27 de octubre de 2014. Prohibida su modificación, copia o distribución.
Sección 6-3 Multiplicación y división de números racionales 383

Página de un libro de texto MULTIPLICACIÓN DE FRACCIONES

Lección 8-12
Calentamiento
Idea clave
Puedes multiplicar Multiplicación de fracciones Simplifica.
dos fracciones para
hallar una fracción Aprende
de una fracción.

Materiales
Actividad
• Hojas cuadradas ¿Cómo puedes usar un modelo para
de papel
• Lápices para colorear, hallar el producto de dos fracciones?
rojo y amarillo, o
Puedes usar papel doblado para hallar 1 de 3 .
2 4
Dobla una hoja cuadrada de papel
ex
ef l i ó verticalmente a la mitad.
R

Dobla de nuevo el papel verticalmente


a la mitad. ¿Qué fracción de la hoja de
¡Piensa!
papel es cada sección?
• Puedo hacer un
modelo para Ahora dobla el papel horizontalmente
ilustrar el problema.
a la mitad. ¿Qué fracción de la hoja
• Puedo buscar patrones de papel es cada sección?
en los productos para
hallar una regla de
Sombrea 34 de las secciones
multiplicación.
verticales de rojo y 12 de las
secciones horizontales de amarillo.
La parte que está sombreada por
ambos colores, rojo y amarillo, ilustra
1 3
× . ¿Qué es 12×43 ?
2 4
Dobla papel para hallar cada producto.

Usa las figuras para hallar los productos.

Estudia los numeradores y denominadores en


cada problema de las partes e y f. ¿Qué
patrón ves entre las fracciones
multiplicadas y sus productos?

Fuente: Mathematics, Diamond Edition, Grade Five, Scott Foresman-Addison Wesley 2008 ( p. 496).

© Lopez Mateos Editores. ISBN 978-607-95583-2-1, obra completa, versión electrónica, ISBN 978-607-95583-3-8, volumen 1, versión electrónica. Ejemplar asignado a: Helecto Villarroel gutierrez -
helecto@gmail.com. Fecha: 27 de octubre de 2014. Prohibida su modificación, copia o distribución.
384 Los números racionales como fracciones

Ejemplo 6-13 Responde lo siguiente:


5 7









Si de la población de cierta ciudad completó el bachillerato y de quienes lo completa-
6 11
ron son mujeres, ¿qué fracción de la población de esa ciudad son mujeres y terminaron el
bachillerato?
7 5 7 #5 7#5 35
Solución    La fracción deberá ser de , ó = = .
11 6 11 6 11 # 6 66
35
La fracción de la población que son mujeres y que terminó el bachillerato es .
66

Propiedades de la multiplicación de números racionales


La multiplicación de números racionales tiene propiedades análogas a las propiedades de
suma de números racionales. Éstas incluyen las siguientes propiedades para la multiplica-
ción: cerradura, conmutatividad, asociatividad, identidad multiplicativa e inverso multipli-
cativo. Para resaltarlas, listamos las dos últimas propiedades.

Teorema 6–12:    Identidad multiplicativa e inverso multiplicativo de los números


racionales
a
1. El número 1 es el único número tal que para todo número racional ,
b
1 a b = = a b 1
# a a a #
b b b
a b
2. Para cualquier número racional distinto de cero , es el único número racional tal que
b a
a#b b a
= 1 = # .
b a a b

a a
O B S E R VA C I Ó N   El inverso multiplicativo de también se llama recíproco de .
b b

Ejemplo 6-14 Halla el inverso multiplicativo, si es posible, de cada uno de los siguientes números racionales:








2 -2 1
a. b. c. 4 d. 0 e. 6
3 5 2

3
Solución a.
2
5 -5
b. - , ó
2 2
4 1
c. Como 4 = , el inverso multiplicativo de 4 es .
1 4
0 1
d. Aunque 0 = , está indefinido; no hay inverso multiplicativo del 0.
1 0
1 13 1 2
e. Como 6 = , el inverso multiplicativo de 6 es .
2 2 2 13

La multiplicación y la suma están conectadas por medio de la propiedad distributiva de la
multiplicación sobre la suma. También hay una propiedad multiplicativa de la igualdad para
números racionales y una propiedad multiplicativa del cero similar a las de los números
completos y los enteros.
© Lopez Mateos Editores. ISBN 978-607-95583-2-1, obra completa, versión electrónica, ISBN 978-607-95583-3-8, volumen 1, versión electrónica. Ejemplar asignado a: Helecto Villarroel gutierrez -
helecto@gmail.com. Fecha: 27 de octubre de 2014. Prohibida su modificación, copia o distribución.
Sección 6-3 Multiplicación y división de números racionales 385

Teorema 6–13:
1.  Propiedad distributiva de la multiplicación sobre la suma de números racionales
a c e
Si , y son números racionales cualesquiera, entonces
b d f

a + b = a # b + a # b
a c e a c a e
b d f b d b f

2.  Propiedad multiplicativa de la igualdad para números racionales 


a c a c e
Si y son números racionales cualesquiera tales que = , y es cualquier número
b d a e c e b d f
racional, entonces # = # .
b f d f

3.  Propiedad multiplicativa de la desigualdad para números racionales


a c e a e c e
(i) Si 7 y 7 0, entonces # 7 # .
b d f b f d f
a c e a e c e
(ii) Si 7 y 6 0, entonces # 6 # .
b d f b f d f
4.  Propiedad multiplicativa del cero para números racionales
a a a
Si es cualquier número racional, entonces # 0 = 0 = 0 # .
b b b

O B S E R VA C I Ó N El teorema 6–13 se puede demostrar usando las propiedades


a# a 0 a#0 0
correspondientes de los enteros. Por ejemplo, 0 = # = # = = 0.
b b 1 b 1 b

Ejemplo 6-15 3
Una bicicleta está en venta a de su precio original. Si el precio de venta es de $330, ¿cuál era el








4
precio original?
3 3
Solución    Sea x el precio original. Entonces del precio original es x. Como el precio de
4 4
3
venta es $330, tenemos x = 330. Despejando x tenemos
4
4#3 4
x = # 330
3 4 3
1 # x = 440
x = 440
Así, el precio original fue de $440.
3 1
A continuación se presenta un enfoque alternativo, que no usa álgebra. Como es 3 de las
4 4
1 1 1
partes, entonces 3 de las partes es $330 y 1 de las partes es $110. Si 1 de las partes es $110,
4 4 4
entonces 4 de éstas es 4 # $110, o $440.

© Lopez Mateos Editores. ISBN 978-607-95583-2-1, obra completa, versión electrónica, ISBN 978-607-95583-3-8, volumen 1, versión electrónica. Ejemplar asignado a: Helecto Villarroel gutierrez -
helecto@gmail.com. Fecha: 27 de octubre de 2014. Prohibida su modificación, copia o distribución.
386 Los números racionales como fracciones

Multiplicación con fracciones mixtas


En la tira cómica “Peanuts”, Sally tiene dificultades para multiplicar dos fracciones mixtas.
Si usara un estimado para verificar si su respuesta es razonable, habría notado que si multi-
plica dos números positivos menores que 3, la respuesta debe ser menor que 9.

A VER, VEAMOS ¿CUÁNTO ¡DIEZ NO, YA ESTOY


OTRO.. ES 2½x2½? MILLONES! ES 6¼ CERCA..

1# 1
Una manera de multiplicar 2 2 es transformar los números mixtos en fracciones im-
2 2
propias y usar la definición de multiplicación, como se muestra aquí.
1# 1 5 5 25
2 2 = # =
2 2 2 2 4
25 1
Podemos entonces transformar la fracción mixta 6 .
4 4
Otra manera de multiplicar fracciones mixtas es usar la propiedad distributiva de la mul-
tiplicación sobre la suma. Por ejemplo,

2 = a2 + b # a2 + b
1# 1 1 1
2
2 2 2 2

= a2 + b 2 + a2 + b #
1 # 1 1
2 2 2
1 1 1 1
= 2#2 + #2 + 2# + #
2 2 2 2
1
= 4 + 1 + 1 +
4
1
= 6 +
4
1
= 6
4
La multiplicación de fracciones nos permite obtener fracciones equivalentes, efectuar su-
mas y restas de fracciones así como resolver ecuaciones de otra manera, como se ilustra en
el siguiente ejemplo.

Ejemplo 6-16 Usa la definición de multiplicación de fracciones y otras propiedades para justificar o resol-








ver lo siguiente:
a an
a. La ley fundamental de las fracciones, = si n Z 0.
b bn
b. Suma de fracciones usando un denominador común.
c. Agrandar el denominador de una fracción positiva en un factor de m es lo mismo que
1
multiplicar la fracción por .
mISBN 978-607-95583-3-8, volumen 1, versión electrónica. Ejemplar asignado a: Helecto Villarroel gutierrez -
© Lopez Mateos Editores. ISBN 978-607-95583-2-1, obra completa, versión electrónica,
helecto@gmail.com. Fecha: 27 de octubre de 2014. Prohibida su modificación, copia o distribución.
Sección 6-3 Multiplicación y división de números racionales 387

d. Resuelve las ecuaciones siguientes:


2 5
(i) + = 2
3x 4x
x x
(ii) + = 1 (halla x en términos de a y b)
a b
a a a n an a c a d c b
Solución a. = #1 = # = b. + = # + #
b b b n bn b d b d d b
ad bc
= +
bd bd
ad + bc
=
bd
a a#1 a 1
c. = # = #
bm b m b m
d. Hay muchas maneras de resolver estas ecuaciones; aquí mostramos una que
usa multiplicación de fracciones.
2 5
(i) + = 2 es equivalente a cada una de las siguientes expresiones:
3x 4x

+ # = 2, a + b = 2
2#1 5 1 2 5 1
3 x 4 x 3 4 x
23 # 1 23 # 1
= 2, x = 2x
12 x 12 x
23 # 23 # 1 1 23
1 = 2x, = # 2x, x = .
12 12 2 2 24
De manera alternativa, podríamos resolver la ecuación si sumamos las
fracciones usando el mínimo denominador común de 12x:
2 5 8 + 15
+ = 2, = 2
3x 4x 12x
23 #
12x = 2 # 12x
12x
23
23 = 24x, x =
24
x x
(ii) + = 1 es equivalente a cada una de las siguientes expresiones:
a b
1 1
x# + x# = 1
a b

xa + b = 1
1 1
a b

b = 1
b + a
xa
ab

b
b + a # ab ab
xa = 1# , y a Z -b
ab a + b a + b
ab
x#1 =
a + b
ab
x =
a + b

© Lopez Mateos Editores. ISBN 978-607-95583-2-1, obra completa, versión electrónica, ISBN 978-607-95583-3-8, volumen 1, versión electrónica. Ejemplar asignado a: Helecto Villarroel gutierrez -
helecto@gmail.com. Fecha: 27 de octubre de 2014. Prohibida su modificación, copia o distribución.
388 Los números racionales como fracciones

División de números racionales


En los Principios y objetivos, hallamos la siguiente declaración acerca de la división de núme-
ros racionales:

La división de fracciones siempre ha sido desconcertante para los estudiantes. Aunque “invierte y multi-
plica” ha sido un recurso básico en la enseñanza convencional de las matemáticas y aunque parece ser una
manera sencilla de recordar cómo dividir fracciones, desde hace mucho tiempo los estudiantes han te-
nido dificultades para hacerlo. Algunos olvidan cuál es el número a invertir y otros se confunden acerca
de cuándo es apropiado aplicar el procedimiento. Una manera común de justificar formalmente el proce-
dimiento de “invierte y multiplica” es usar argumentos sofisticados que incluyan la manipulación de ex-
presiones algebraicas racionales —argumentos que sobrepasan el alcance de muchos estudiantes de los
grados medios. Este proceso puede parecer lejano y misterioso para muchos estudiantes. Al no compren-
der cabalmente la justificación subyacente, muchos estudiantes son entonces incapaces de reparar sus
errores y aclarar por sí mismos sus dudas sobre la división de fracciones. Un enfoque alternativo incluye
ayudar a los estudiantes a comprender la división de fracciones a partir de lo que saben acerca de la divi-
sión de números completos. (p. 219)

Tratamos de seguir ese consejo al desarrollar el concepto de división de números racionales.


Recuerda que 6 , 3 significa “¿Cuántos 3 hay en 6?” Hallamos que 6 , 3 = 2 pues
a
3 # 2 = 6 y, en general, si a, b, c 僆 C, entonces = c, si, y sólo si, c es el único número com-
b

pleto tal que bc = a. Considera 3 , a b , que es equivalente a encontrar cuántas mitades


1
2
hay en 3. Vemos en el modelo de área en la figura 6-13 que hay 6 mitades en las 3 piezas

completas. Registramos esto como 3 , a b = 6, lo cual es cierto porque a b # 6 = 3.


1 1
2 2

1 + 1 + 1 = 3

1– 1– 1– 1– 1– 1–
2 2 2 2 2 2

Figura 6-13

Con los números completos, una manera de pensar sobre la división era en términos de
restas repetidas. Hallamos que 6 , 2 = 3 porque 2 podría restarse tres veces de 6, esto es,

6 - 3 # 2 = 0. Con 3 , a b , queremos saber cuántas mitades podemos restar de 3. Como


1
2

3 - 6 # a b = 0, sabemos que 3 , a b = 6.
1 1
2 2

A continuación, considera a b , a b . Esto significa “¿Cuántos hay en ?” En la


3 1 1 3
4 8 8 4
1 3
figura 6-14 se muestra que hay seis en la parte sombreada, que representa del todo. Por lo
8 4

tanto, a b , a b = 6. Esto es cierto pues a b 6 = . Usando restas repetidas vemos


3 1 1 # 3
4 8 8 4
, y que - 6 a b = 0, de modo que ,
3 1 6 1 6 1 3 1
Figura 6-14 que , = = 6.
4 8 8 8 8 8 4 8
© Lopez Mateos Editores. ISBN 978-607-95583-2-1, obra completa, versión electrónica, ISBN 978-607-95583-3-8, volumen 1, versión electrónica. Ejemplar asignado a: Helecto Villarroel gutierrez -
helecto@gmail.com. Fecha: 27 de octubre de 2014. Prohibida su modificación, copia o distribución.
Sección 6-3 Multiplicación y división de números racionales 389

En la página de muestra (p. 390) aparecen modelos adicionales para la división de fraccio-
nes. También es útil el modelo de la medición o de la recta numérica usado en el ejemplo A
de la página de muestra.
7 3
Por ejemplo, considera , . Primero trazamos una medición o recta numérica dividida en oc-
8 4
3 7
tavos, como se muestra en la figura 6-15. A continuación queremos saber cuántos hay en .
4 8
3 1
La barra de longitud está formada de 6 partes iguales de longitud . Vemos que hay al menos
4 8
3 7 3
una longitud de en . Si colocamos otra barra de longitud sobre la recta numérica, vemos que
4 8 4
7 1 7
es 1 más de los 6 segmentos necesarios para formar . Por lo tanto, la respuesta es 1 , ó .
8 6 6

0 1 1 3 1 5 3 7 9 5 11 6
1
8 4 8 2 8 4 8 8 4 8 4

Figura 6-15

En los ejemplos anteriores vemos una relación entre la división y la multiplicación de nú-
meros racionales. Podemos definir formalmente la división entre números racionales en
términos de la multiplicación de la misma manera en que lo hicimos para los números com-
pletos.

Definición de división de números racionales


a c a c e e
Si y son números racionales cualesquiera, entonces , = si, y sólo si, es el único
b d b d f f
c e a
número racional tal que # = .
d f b

c
O B S E R VA C I Ó N En la definición de división, Z 0 pues no está definida la división
d
entre 0.

AHORA INTENTA ÉSTE 6-14 Los estudiantes a menudo confunden la división entre 2 y la división en-
1 a 1 1 1 1
tre . Nota que a , 2 = = a, pero a , = x si, y sólo si, x = a, 2 # x = 2a, x = 2a.
2 2 2 2 2 2
Escribe una historia de la vida real que ayude a los estudiantes a ver la diferencia entre la división entre 2 y
1
la división entre .
2

© Lopez Mateos Editores. ISBN 978-607-95583-2-1, obra completa, versión electrónica, ISBN 978-607-95583-3-8, volumen 1, versión electrónica. Ejemplar asignado a: Helecto Villarroel gutierrez -
helecto@gmail.com. Fecha: 27 de octubre de 2014. Prohibida su modificación, copia o distribución.
390 Los números racionales como fracciones

Página de un libro de texto D I V I S I Ó N D E F R ACC I O N ES

Lección 5-6
Calentamiento
Idea clave
Puedes usar la División de fracciones
multiplicación para
dividir fracciones.
Aprende
Vocabulario ¿Cómo puedes modelar la división de fracciones?
• recíproco
• inverso
multiplicativo Ejemplo A
El señor Gutiérrez fabrica posavasos de madera. Él divide pequeños postes
ex
ef l i ó redondos en rodajas de 1 2 pulgada de grosor. ¿Cuántas rodajas puede obtener
R

de un poste de 5 pulgadas? 1
Divide 5 pulgadas en secciones de 2 pulgada.

¡Piensa! Halla 5 ÷ 12
• Puedo usar modelos Hay diez rodajas
para ilustrar la de 1
2 pulgada
división de fracciones. Piensa: “¿Cuántas
en 5 pulgadas.
mitades hay en 5?”
• Puedo buscar un PULGADAS

patrón para entender


cómo están De este modo, 5 ÷12=10.
relacionadas la El señor Gutiérrez puede obtener 10 rodajas de un poste de 5 pulgadas.
multiplicación y la
división.
Ejemplo B

Halla 34 ÷4. Muestra 3 .


4
Puedes pensar 3 4 ÷4 como 3
“¿Cuánto es 3 dividido Divide 4 en 4
4
en 4 partes iguales?” partes iguales.
Cada una de las cuatro partes
De este modo, 3 ÷4= 3 . iguales contiene 3 dieciseisavos.
4 16

¿Cómo puedes dividir fracciones?


Actividad
Estudia los patrones a continuación. Compara la primera y segunda columnas.
Patrón 1 Patrón 2

¿Qué notas en los divisores de la primera columna y los factores en


la segunda columna de cada patrón?
¿Cómo se compara el cociente con el dividendo cuando el divisor es
una fracción menor que 1?

Fuente: Mathematics, Diamond Edition, Grade Six, Scott Foresman-Addison Wesley 2008 ( p. 266).

© Lopez Mateos Editores. ISBN 978-607-95583-2-1, obra completa, versión electrónica, ISBN 978-607-95583-3-8, volumen 1, versión electrónica. Ejemplar asignado a: Helecto Villarroel gutierrez -
helecto@gmail.com. Fecha: 27 de octubre de 2014. Prohibida su modificación, copia o distribución.
Sección 6-3 Multiplicación y división de números racionales 391

Algoritmo para la división de números racionales


Como vemos en la tira cómica “Peanuts”, Sally no entiende por qué funciona el algoritmo
para la división de fracciones. Ella no es la única confundida. Explicaremos por qué “para
dividir fracciones usamos el recíproco y multiplicamos”.

“PARA DIVIDIR FRACCIO- ¿POR QUÉ? ¿POR QUÉ USAR NO, ¿POR QUÉ
NES, USA EL RECÍPROCO EL RECÍPROCO? NACÍ?
Y MULTIPLICA”

¿Tiene sentido el método de la tira cómica, a menudo llamado invierte y multiplica, con base en lo
a
que sabemos acerca de los números racionales? Sabemos que una manera de interpretar es
b
a 4 5
como a , b. También sabemos que = a. Por lo tanto, 4 , 5 se puede escribir como , .
1 1 1
4 5 4 1
Si funciona la técnica de invierte y multiplica, entonces 4 , 5 = , = * =
1 1 1 5
4#1 4 4
= . Como 4 , 5 = , esto es consistente con lo hecho anteriormente.
1#5 5 5
Para desarrollar el algoritmo general para la división de números racionales, considera-
mos lo que esa división pueda significar. Por ejemplo,
2 5 2 5
, = x implica = x
3 7 3 7
7 5
Multiplicamos ambos lados de la ecuación por , el recíproco de . Así,
5 7

= # a xb = a # bx = 1 # x = x
7#2 7 5 7 5
5 3 5 7 5 7
2 5 2 7
Por lo tanto, , = # .
3 7 3 5
Damos también una justificación tradicional del algorítmo de la división. El algoritmo
para la división de fracciones usualmente se justifica en los grados medios usando la ley fun-
a ac
damental de las fracciones, = , donde a, b y c son fracciones, o de manera equivalente,
b bc
la propiedad de la identidad de la multiplicación. Por ejemplo,

2 5
2 2 2 7 #
2 7
2#7
, =
3
5
=
3
5
#1 =
3
5
# 57 =
3 5
=
3 7 7 7 7 5
1 3 5
Así,
2 5 2 7
, = #
3 7 3 5

© Lopez Mateos Editores. ISBN 978-607-95583-2-1, obra completa, versión electrónica, ISBN 978-607-95583-3-8, volumen 1, versión electrónica. Ejemplar asignado a: Helecto Villarroel gutierrez -
helecto@gmail.com. Fecha: 27 de octubre de 2014. Prohibida su modificación, copia o distribución.
392 Los números racionales como fracciones

a
AHORA INTENTA ÉSTE 6-15 Usa un argumento similar al anterior para mostrar que, en general, si y
b
c c a c a d
son números racionales y Z 0, entonces , = # .
d d b d b c

Resumimos el algoritmo como sigue:

Teorema 6–14:    Algoritmo para la división de fracciones


a c c
Si y son números racionales cualesquiera y Z 0, entonces
b d d
a c a d
, = #
b d b c

Algoritmo alternativo para la división de números racionales


Se puede hallar un algoritmo alternativo para la división de fracciones dividiendo primero las frac-
9 3 15 5
ciones que tienen denominadores iguales. Por ejemplo, , = 9 , 3y , = 15 , 5.
10 10 23 23
Estos ejemplos sugieren que cuando se dividen dos fracciones con el mismo denominador,
el resultado puede obtenerse dividiendo el numerador de la primera fracción entre el nu-
merador de la segunda. Para dividir fracciones con denominadores diferentes, reescribimos
las fracciones de manera que sus denominadores sean iguales. Así,
a c ad bc ad
, = , = ad , bc, o
b d bd bd bc

a c a d a c a , c
AHORA INTENTA ÉSTE 6-16 Muestra que , = # y , = son equivalentes.
b d b c b d b , d

Los tres ejemplos siguientes ilustran el uso de la división de números racionales.

Ejemplo 6-17 Una estación de radio proporciona un servicio público de anuncios de 36 minutos cada 24








horas de transmisión.
a. ¿Qué fracción de la transmisión diaria está asignada al servicio público de anuncios?
3
b. ¿Cuántos anuncios del servicio público con duración de min se permiten en los 36 min?
4
1
Solución a. Hay 60 min en una hora y 60 # 24 min en un día. Así, 36>160 # 242, ó , del
40
día está asignado para los anuncios.

b. 36> a b = 36a b , ó 48, anuncios están permitidos.


3 4
4 3

© Lopez Mateos Editores. ISBN 978-607-95583-2-1, obra completa, versión electrónica, ISBN 978-607-95583-3-8, volumen 1, versión electrónica. Ejemplar asignado a: Helecto Villarroel gutierrez -
helecto@gmail.com. Fecha: 27 de octubre de 2014. Prohibida su modificación, copia o distribución.
Sección 6-3 Multiplicación y división de números racionales 393

Ejemplo 6-18 1 3
Tenemos 35 yardas de material disponible para fabricar toallas. Cada toalla requiere de









2 8
yarda de material.

a. ¿Cuántas toallas pueden hacerse?


b. ¿Cuánto material sobrará?
1 3
Solución a. Necesitamos hallar la parte entera de la respuesta a 35 , . La división es:
2 8
1 3 71 # 8 284 2
35 , = = = 94
2 8 2 3 3 3
Por lo tanto, pueden hacerse 94 toallas.
3 2 3
b. Como la división en (a) fue entre , la cantidad de material que sobra es de ,
8 3 8
2#3 1 2
ó , ó de yarda. Esto también se puede responder notando que los de la
3 8 4 3
2 3
parte (a) son dos tercios de una toalla, que requiere de de yarda de material.
3 8

Ejemplo 6-19 1 3
Si de una cuerda para saltar mide 3 yd, ¿cuál es la longitud de toda la cuerda?








2 4
1 1
Solución    Si la longitud de la cuerda es l yd, entonces de la longitud de la cuerda es l. Así,
2 2
1 3
l = 3
2 4
3 34
l = 1
2
15 # 2 15 1
l = = , ó 7 yd
4 1 2 2
Nota que este problema se puede resolver sin usar la división, simplemente multiplicando
por 2 ambos lados de la primera ecuación.

Estimación y matemática mental con números racionales


Las estrategias para la estimación y la matemática mental que se desarrollaron con los nú-
meros completos también pueden usarse con los números racionales.

Ejemplo 6-20 Usa matemática mental para hallar










a. 112 # 252 # b. a5 b # 12
1 1 4#
c. 20
4 6 5

Solución    Cada caso siguiente es un enfoque posible:

a. 112 # 252 # = 25 # a12 # b = 25 # 3 = 75


1 1
4 4

© Lopez Mateos Editores. ISBN 978-607-95583-2-1, obra completa, versión electrónica, ISBN 978-607-95583-3-8, volumen 1, versión electrónica. Ejemplar asignado a: Helecto Villarroel gutierrez -
helecto@gmail.com. Fecha: 27 de octubre de 2014. Prohibida su modificación, copia o distribución.
394 Los números racionales como fracciones

12 = a 5 + b12 = 5 # 12 + # 12 = 60 + 2 = 62
1# 1 1
b. 5
6 6 6
4 1
c. # 20 = 4a # 20b = 4 # 4 = 16
5 5

Ejemplo 6-21 Estima lo siguiente:










1# 8 5 1
a. 3 7 b. 24 , 4
4 9 7 8

Solución a. Usando redondeo, el producto estará cerca de 3 # 8 = 24. Si usamos la estrate-


gia del rango, podemos decir que el producto debe estar entre 3 # 7 = 21 y
4 # 8 = 32.
b. Podemos usar números compatibles y pensar el estimado como 24 , 4 = 6 ó
25 , 5 = 5.

Extensión del concepto de exponentes
Recuerda que am se definió para cualquier número entero a y para cualquier número natu-
ral m como el producto de m números a. Para cualquier número racional definimos am de
manera similar, como sigue.

Definición de a a la potencia m
am = a # a # a # Á # a, donde a es cualquier número racional y m es cualquier número natural.
m factores

De la definición, a3 # a2 = 1a # a # a2 # 1a # a2 = a3 + 2. De manera similar, se sigue que


1. am # an = am + n
donde a es cualquier número racional y m y n son números naturales. Si (1) es verdadero
para todos los números completos m y n, entonces como a1 # a0 = a1 + 0 = a1, debemos
tener a0 = 1. Por lo tanto, es útil dar un significado a a0 cuando a Z 0 como sigue.
2. Para cualquier número distinto de cero a, a0 = 1.
Las propiedades anteriores también son verdaderas para valores de números racionales
de a. Por ejemplo, considera lo siguiente:
2 2 2 3 2 2+3 2 5
a b #a b = a # b#a # # b = a b = a b
2 2 2 2 2
3 3 3 3 3 3 3 3 3
Si am # an = am + n se va a extender a todas las potencias enteras de a, entonces ¿cómo deberá defi-
nirse a 3 ? Si (1) va a ser verdadero para todos los enteros m y n, entonces a 3 # a3 = a 3 + 3 =
- - -
-3 3
a0 = 1. Por lo tanto, a = 1>a . Esto es cierto en general y tenemos lo siguiente:
-n 1
3. Para un número a distinto de cero, a = .
an
-
En los grados básicos es típico que se justifique la definición de a n buscando patrones.
Nota que conforme los exponentes siguientes decrecen en 1, los números de la derecha se
dividen entre 10. Así, el patrón puede continuar como se muestra.

© Lopez Mateos Editores. ISBN 978-607-95583-2-1, obra completa, versión electrónica, ISBN 978-607-95583-3-8, volumen 1, versión electrónica. Ejemplar asignado a: Helecto Villarroel gutierrez -
helecto@gmail.com. Fecha: 27 de octubre de 2014. Prohibida su modificación, copia o distribución.
Sección 6-3 Multiplicación y división de números racionales 395

103 = 10 # 10 # 10
102 = 10 # 10
101 = 10
100 = 1
-1 1 1
10 = =
10 101
-2 1 # 1 1
10 = =
10 10 102
-3 1 # 1 1
10 = =
102 10 103
1 -n
Si el patrón se extiende, entonces predeciríamos que 10 . Nota que esto es razona-
=
10n
miento inductivo y, por lo tanto, no es una justificación matemática completa .
Considera si la propiedad am # an = am + n se puede extender para incluir todas las potencias
de a, donde los exponentes son enteros. Por ejemplo, ¿es cierto que 24 # 2 3 = 24 + 3 = 21?
- -
-3
Las definiciones de 2 y las propiedades de los exponentes no negativos demuestran que esto
es cierto, como se observa a continuación.
1 24 21 # 23
24 # 2 3 = 24 # 3 = 3 =
-
= 21
2 2 23
Además, 2
-4 # 2-3 = 2
-4 + -3
= 2
-7
es cierto pues
-4 # 2-3 1 # 1 1#1 1 1 -
2 = = 4 3 = 4+3 = 7 = 2 7
4
2 2 3 #
2 2 2 2
En general, con exponentes enteros, la propiedad siguiente es válida.

Teorema 6–15
Para cualquier número racional a distinto de cero y enteros cualesquiera m y n, am # an = am + n.

Si a = 0, entonces el teorema 6–15 sigue siendo válido en tanto que


O B S E R VA C I Ó N
m y n sean mayores que 0.

Se pueden desarrollar otras propiedades de exponentes usando las propiedades de los nú-
meros racionales. Por ejemplo,
25 23 # 22 25 25 1 -
= = 22 = 25-3 = 5# 3
= 3 = 2 3 = 25-8
23 23 28
2 2 2
Con exponentes enteros es válido el teorema siguiente.

Teorema 6–16
am
Para cualquier número racional a distinto de cero y para enteros cualesquiera m y n, = am-n.
an

A veces, podemos encontrar una expresión como 12423. Esta expresión se puede escribir
como una sola potencia de 2 como sigue:
12423 = 24 # 24 # 24 = 24 + 4 + 4 = 23
#4
= 212

© Lopez Mateos Editores. ISBN 978-607-95583-2-1, obra completa, versión electrónica, ISBN 978-607-95583-3-8, volumen 1, versión electrónica. Ejemplar asignado a: Helecto Villarroel gutierrez -
helecto@gmail.com. Fecha: 27 de octubre de 2014. Prohibida su modificación, copia o distribución.
396 Los números racionales como fracciones

En general, si a es cualquier número racional y m y n son enteros positivos, entonces


n términos
1a 2 = a
m n m # am # am # Á # am = am+m+ Á +m
= anm = amn
n factores
¿Esta propiedad es válida para exponentes enteros negativos? Por ejemplo, ¿es cierto que 1232 4 =
-

21321 42 = 2 12? La respuesta es sí pues 1232 4 = 3 4 = 12 = 2 12. Además, 12 324 =


- - - 1 1 - -
12 2 2
1 4 14
a 3 b = 3 # 3 # 3 # 3 = 3 4 = 12 = 2 12.
1 1 1 1 1 -
2 2 2 2 2 12 2 2

Teorema 6–17
Para cualquier número racional a Z 0 y enteros cualesquiera m y n,
1am2n = amn

Usando las definiciones y los teoremas desarrollados, podemos deducir propiedades adi-
cionales. Nota, por ejemplo, que
2 4 2#2#2#2 24
a b = # # # = # # # = 4
2 2 2 2
3 3 3 3 3 3 3 3 3 3
Esta propiedad puede generalizarse como sigue.

Teorema 6–18
a
Para cualquier número racional distinto de cero y cualquier entero m,
b
a m am
a b = m
b b

De la definición de exponentes negativos, el teorema anterior y la división de fracciones


tenemos
-
a m bm b m
a b a b
1 1
= = = =
a m am am
a b
b a
b bm

Teorema 6–19 -m
b m
y cualquier entero m, a b = a b .
a a
Para cualquier número racional distinto de cero
b b a

Una propiedad similar a la del teorema 6–18 vale para la multiplicación. Por ejemplo,

12 # 32 = 3 3 = a 3b # a 3b = 2 3 # 3 3
-3 1 1 1 1 - -
12 # 323
=
2 #3 2 3
y, en general, es cierto que 1a # b2m = am # bm si a y b son números racionales distintos de
cero y m es un entero.

© Lopez Mateos Editores. ISBN 978-607-95583-2-1, obra completa, versión electrónica, ISBN 978-607-95583-3-8, volumen 1, versión electrónica. Ejemplar asignado a: Helecto Villarroel gutierrez -
helecto@gmail.com. Fecha: 27 de octubre de 2014. Prohibida su modificación, copia o distribución.
Sección 6-3 Multiplicación y división de números racionales 397

Las definiciones y propiedades de los exponentes se resumen en la definición y teorema


siguientes. Para cualesquier números racionales distintos de cero a y b y enteros m y n (ex-
cepto los mencionados), tenemos lo siguiente.

Definición de a elevado a una potencia entera


1. am = a # a # a # Á # a, donde m es un entero positivo y a es cualquier número racional
m factores
2. a0 = 1
-m 1
3. a =
am

Teorema 6–20:    Propiedades de los exponentes


1. am # an = am + n
am
2. n = am - n
a
3. 1am2n = anm
a m am
4. a b = m
b b
5. 1ab2m = ambm
-
a m b m
6. a b = a b
b a

Nota que la propiedad 5 es para la multiplicación. Propiedades análogas no son válidas para
la suma y la resta. Por ejemplo, en general, 1a + b2 1 Z a 1 + b 1. Para ver por qué, es sufi-
- - -

ciente un ejemplo numérico, pero es instructivo escribir cada lado con exponentes positivos:

1a + b2
-1 1
=
a + b
-1 -1 1 1
a + b = +
a b
1 1 1
y sabemos que, en general, Z + .
a + b a b

Ejemplo 6-22 En cada uno de los casos siguientes, usa las propiedades de los exponentes para justificar la
igualdad o la desigualdad:








a. 1 - x2 2 Z - x 2
- -

b. 1 - x2 3 = - x 3
- -

c. ab 1 Z 1ab2 1 si a Z 1, ó - 1
- -

d. 1a 2 b 22 1 = a2 b2
- - -

e. 1a 2 + b 22 1 Z a2 + b2
- - -

Solución a. 1 - x2
-2 1 1
1 x2
-= 2
= 2
x
- -
- x-2 = -1x-22 = a 1 b = 1
2
x x2
Por lo tanto, 1 x2 Z - x .
- -2 -2

© Lopez Mateos Editores. ISBN 978-607-95583-2-1, obra completa, versión electrónica, ISBN 978-607-95583-3-8, volumen 1, versión electrónica. Ejemplar asignado a: Helecto Villarroel gutierrez -
helecto@gmail.com. Fecha: 27 de octubre de 2014. Prohibida su modificación, copia o distribución.
398 Los números racionales como fracciones

- 1
b. 1 - x2 a 3b
-3 1 1
1 - x23
= = - x3 =
x
- 1
= - 1x 32 = a x3 b
-3 -
-x

Por lo tanto, 1 - x2
-3 -
= - x 3.

c. ab 1 = a1b 12 = a # = , pero 1ab2 1 = . Por lo tanto, ab 1 Z 1ab2 1.


- - 1 a - 1 - -
b b ab
d. 1a 2b 22 1 = 1a 22 11b 22 1
- - - - - - -

= a1 221 12b1 221 12


- - - -

= a2b2
- -
1 1 a2 + b2 1 a2 b2
e. 1a + b 22 1 = a 2 + 2 b = a 2 2 b = 2
-2 - - 1
Z a2 + b2
a b a b a + b2

Observa que todas las propiedades de los exponentes se refieren a potencias con la misma
base o el mismo exponente. Para evaluar expresiones con exponentes donde se usen dife-
rentes bases o potencias, efectúa todos los cálculos o reescribe las expresiones en la misma
274
base o en el mismo exponente, si es posible. Por ejemplo, puede reescribirse como
274 (33)4 312 813
= 4 3 = 12 = 1.
813 (3 ) 3

Ejemplo 6-23 Escribe cada uno de los casos siguientes en la forma más simple usando exponentes positi-








vos en la respuesta final:

a. 162 # 8 + 5 # 10 + 3 # 10 3) # 103
-3 -1 -2 - - -4
b. 202 , 24 c. (10 d. (x3y 2)
1
Solución a. 162 # 8 = (24)2 # (23) = 28 # 2
-3 -3 -9 - -
= 28 + 9 = 2 1 =
2
202 (22 # 5)2 24 # 52
b. = = = 52
24 24 24
+ 5 # 10 + 3 # 10 3) # 103 = 10 # 103 + 5 # 10-2 # 103 +
-1 -2 - -1
c. (10
3 # 10 3 # 103
-

10 1 + 3 + 5 # 10 2 + 3 + 3 # 10 3 + 3
- - -
=
= 102 + 5 # 101 + 3 # 100
= 153
- -4 -12 8 1 # 8 y8
d. (x3y 2) = x y = y =
x12 x12

ROMPECABEZAS En las lejanas tierras de Natichuj había un castillo rodeado por cuatro fosos. Un día
el castillo fue atacado y tomado por una feroz tribu del norte. Se colocaron guardias en cada puente. Al
príncipe Juan se le permitió llevar cierto número de bolsas de oro para dirigirse al exilio. Sin embargo, el
guardia del primer puente le exigió la mitad de las bolsas de oro más una bolsa. Juan cumplió la demanda
y se encaminó al siguiente puente. Los guardias del segundo, tercero y cuarto puentes hicieron deman-
das idénticas, las cuales fueron cumplidas por el príncipe. Cuando Juan finalmente cruzó todos los puen-
tes, le quedaba una sola bolsa de oro. ¿Con cuántas bolsas comenzó Juan?

© Lopez Mateos Editores. ISBN 978-607-95583-2-1, obra completa, versión electrónica, ISBN 978-607-95583-3-8, volumen 1, versión electrónica. Ejemplar asignado a: Helecto Villarroel gutierrez -
helecto@gmail.com. Fecha: 27 de octubre de 2014. Prohibida su modificación, copia o distribución.
Sección 6-3 Multiplicación y división de números racionales 399

Evaluación 6-3A

1. En las figuras siguientes se usó un rectángulo unitario 9. De los números entre paréntesis, escoge el que más se
para ilustrar el producto de dos fracciones. Menciona las aproxime a cada una de las siguientes expresiones:
18, 20, 15, 162
fracciones y su producto. 11 # 3
a. 3 5
12 100
7 116, 14, 4, 32
1 # 7
b. 2
10 8
a 0, 1, , b
1 1 1 1
c. ,
101 103 2 4
10. Estima lo siguiente:
4 1
a. 5 # 3
5 10
a. b. 10 # 1
b. 4 5
11 8
2. Usa una región rectangular para ilustrar cada uno de los
11. Sin realizar los cálculos, escoge la frase entre paréntesis
productos siguientes:
que describa correctamente cada una de las expresiones:
1 3
a. # 13 # 17
3 4 a. (mayor que 1, menor que 1)
14 19
2 1
b. # 2 1
3 5 b. 3 , 5 (mayor que 1, menor que 1)
7 9
3. Halla cada uno de los productos siguientes. Escribe tu
1 3
respuesta en la forma más simple. c. 4 , 2 (mayor que 2, menor que 2)
49 # 26 a b2 xy z2 a 3 100
a. b. # 2 c. # 1
65 98 b a z x3 y2 12. Un trabajo de costura requiere 6 yardas de material que
4. Usa la propiedad distributiva para hallar cada producto. 8
1
cSugerencia: a 4 + b a 2 + b . d
1 1 1 1
a. 4 # 2 cuesta 62¢ la yarda, y 3 yardas que cuestan 81¢ la yarda.
4
2 3 2 3
1# 1 Escoge entre los siguientes el mejor estimado del costo
b. 3 2 del trabajo:
3 2
a. Entre $2 y $4 b. Entre $4 y $6
5. Halla el inverso multiplicativo de cada caso:
-1 c. Entre $6 y $8 d. Entre $8 y $10
1
a. b. 3 13. Cinco octavos de los estudiantes de una secundaria vi-
3 3 ven en dormitorios. Si 6000 estudiantes de la secunda-
x ria viven en dormitorios, ¿cuántos estudiantes hay en la
c. , si x Z 0 y y Z 0 d. - 7
y secundaria?
6. Despeja x en cada caso: 5
2 7 3 1 14. Alberto posee de las acciones de una compañía. Su her-
a. x = b. , x = 9
3 6 4 2 mana Renata posee la mitad del número de acciones de
5 2 3 2x 1 x 1 Alberto. ¿Qué parte de las acciones no pertenecen ni a
c. + x = d. - = +
6 3 4 3 4 6 2 Alberto ni a Renata?
7. Muestra que las propiedades siguientes no son válidas 15. Se vende un traje por $180. ¿Cuál es el precio original del
para la división de números racionales: 1
a. Conmutatividad traje si el descuento fue del precio original?
4
b. Asociatividad 16. Juan sacó todo el dinero de su cuenta de ahorro. Gastó $50
8. Calcula mentalmente lo siguiente. Obtén las respuestas 3
exactas. en un radio y de lo que quedó en regalos. La mitad de lo
5
1 1
a. 3 # 8 b. 7 # 4 que le quedó, lo regresó a su cuenta y los restantes $35 los
4 4 donó a una institución de beneficencia. ¿Cuánto dinero te-
1 1
c. 9 # 10 d. 8 # 2 nía Juan originalmente en su cuenta de ahorro?
5 4

© Lopez Mateos Editores. ISBN 978-607-95583-2-1, obra completa, versión electrónica, ISBN 978-607-95583-3-8, volumen 1, versión electrónica. Ejemplar asignado a: Helecto Villarroel gutierrez -
helecto@gmail.com. Fecha: 27 de octubre de 2014. Prohibida su modificación, copia o distribución.
400 Los números racionales como fracciones

1 1 24. Supón que la cantidad de bacterias en cierto cultivo está


17. Alex da de sus canicas a Beto. Beto da de éstas a Cali.
6 t
dada como una función del tiempo por Q(t) = 1010 a b ,
2 2
1 5
Cali da de éstas a Dani. Si Dani tiene cuatro canicas,
2 donde t es el tiempo en segundos y Q(t) es la cantidad de
¿cuántas canicas tenía Alex originalmente? bacterias después de t segundos. Halla lo siguiente:
18. Escribe cada una de las expresiones siguientes en la
forma más simple, usando exponentes positivos en la res- a. El número inicial de bacterias (esto es, el número de
puesta final: bacterias en t = 0)
a. 3 7 # 3 6 b. 37 # 36
- - b. El número de bacterias después de 2 s
- 1 1 1 1
c. 5 , 54
15
d. 515 , 5 4 25. Sea S = + 2 + 3 + Á + 64 .
2 2
e. 1 - 52 2
- a 2 2 2
f. -3 , donde a Z 0
a a. Usa la propiedad distributiva de la multiplicación so-
19. Escribe cada una de las expresiones siguientes en la forma bre la suma a fin de hallar una expresión para 2S.
1 64
b. Muestra que 2S - S = S = 1 - a b .
más sencilla, usando exponentes positivos en la respuesta
final: 2
1 3 1 7 1 9 1 6
a. a b # a b b. a b , a b
c. Halla una expresión sencilla para la suma
2 2 2 2 1 1 1 1
2 5 4 2 3 7 3 7 + 2 + 3 + Á + n
c. a b # a b d. a b , a b 2 2 2 2
3 9 5 5
26. Si f 1n2 =
2 # -n
20. Si a y b son números racionales, con a Z 0 y b Z 0, y si 3 , halla lo siguiente:
m y n son enteros, ¿cuáles de los casos siguientes son 5
verdaderos y cuáles falsos? Justifica tus respuestas. a. f 102, f 1 - 12, f 112
a. am # bn = (ab)m + n b. am # bn = (ab)mn
m# m
c. a b = (ab) 2m
d. 1ab20 = 1 b. n si f 1n2 =
2 # -10
9
5
e. 1a + b2m = am + bm f. 1a + b2 m = m + m
- 1 1
a b 27. Si el n-ésimo término de una sucesión está dado por an =
3 # 2 n, responde lo siguiente:
-
21. Despeja el entero n en cada caso:
a. 2n = 32 b. n2 = 36 a. Halla los primeros cinco términos.
n# 7
c. 2 2 = 2 5
d. 2n # 27 = 8 b. Muestra que los primeros cinco términos están en su-
22. Despeja x en cada una de las desigualdades siguientes, cesión geométrica.
donde x es un entero: 3
c. ¿Cuáles términos son menores que ?
a. 3x … 81 b. 4x 6 8 1000
2x
c. 3 7 27 d. 2x 7 1 28. En los siguientes casos, determina cuál es el número mayor:
b. 1 - 272 15 ó 1 - 32 75
- -
23. En cada uno de los pares siguientes, determina qué frac- a. 3250 ó 4100
ción es mayor: 29. Muestra que la media aritmética de dos números
1 3 1 4 3 10 3 8
a. a b ó a b b. a b ó a b
racionales está entre los dos números; esto es, prueba
2 2 4 4
6 a + b 6 .
a 1 a c c
4 10 4 8 3 10 4 10 que 0 6
c. a b ó a b d. a b ó a b b 2 b d d
3 3 4 5

Evaluación 6-3B

1. Usa una región rectangular para ilustrar cada uno de los -5


c. # 21 3# 1
2 d. 2
productos siguientes: 2 2 4 3
2 1 2 2 2 2 3 2
xy
a. # b. #
a b # z2
e. 3 # 3 f.
5 3 3 3 b a z xy
2. Halla cada uno de los productos siguientes de números 3. Usa la propiedad distributiva para hallar cada producto
racionales. Escribe tu respuesta de la forma más simple. de números racionales.
1 3 22 # 2
b. a + 1 b a - 1b
y
a. 2 # 3
1 3 x
b. 4 a. 2 # 4
3 4 7 3 3 5 y x
2# 1
c. 248 100
5 8
© Lopez Mateos Editores. ISBN 978-607-95583-2-1, obra completa, versión electrónica, ISBN 978-607-95583-3-8, volumen 1, versión electrónica. Ejemplar asignado a: Helecto Villarroel gutierrez -
helecto@gmail.com. Fecha: 27 de octubre de 2014. Prohibida su modificación, copia o distribución.
Sección 6-3 Multiplicación y división de números racionales 401

4. Despeja x en cada caso: 13. Los salarios de una compañía de software se incremen-
2 11 3 1 1
a. x = b. , x = tan cada año en del salario de cada empleado del
3 6 4 3 10
del año anterior.
5 2 3 2x 1 x 1
c. - x = d. + = -
6 3 4 3 4 6 2 a. Si el salario anual actual de Marta es de $100,000,
5. Halla una fracción tal que si sumas el denominador al nu- ¿cuál será su salario anual en 2 años?
merador y lo colocas sobre el denominador original, la b. Si el salario actual de Arturo es de $99,000, ¿cuál era
nueva fracción tiene el triple del valor de la fracción original. su salario hace un año?
6. Calcula mentalmente lo siguiente. Obtén las respuestas c. Si el salario actual de Juanita es de $363,000, ¿cuál
exactas. era su salario hace 2 años?
3
1 3 14. Jazmín está leyendo un libro. Ha terminado del libro y
a. 3 # 8 b. 7 # 4 4
2 4 le faltan 82 páginas. ¿Cuántas páginas ha leído?
1 1
c. 9 # 6 d. 8 # 2 15. Pedro, Pablo y María comenzaron al mismo tiempo a cami-
5 3 nar alrededor de un sendero circular en la misma dirección.
1 1 1 1 5
e. 3 , f. 3 , Pedro tardó h en dar una vuelta. Pablo tardó hy
2 2 2 2 12
1 1 1
g. 3 , h. 4 , 2 María tardó h.
3 2 3
7. De los números entre paréntesis, escoge el que más se a. ¿Cuántos minutos tarda cada persona en caminar al-
aproxime a cada una de las siguientes expresiones: rededor del sendero?
a. 20 , 9 a 2, 180, , 10b
2 7 1 b. ¿Cuántas vueltas dará cada persona alrededor de la
3 8 2 senda antes de encontrarse los tres de nuevo en el
a 21, 24, , 32 b
1 # 77 1 punto de inicio?
b. 3 7 16. La fórmula para convertir grados Celsius (C ) a grados
20 100 20
Fahrenheit (F ) es F = a b # C + 32.
9
a 3 , 1, 1001, 0b
1 1 1
c. 3 , 5
10 1001 10
8. Sin realizar los cálculos, escoge la frase entre paréntesis a. Si Sandra lee que en Veracruz la temperatura es de
que describa correctamente cada una de las expresiones: 32°C, ¿cuál es la temperatura en grados Fahrenheit?
1 13 b. Si la temperatura desciende a - 40°F en Chihuahua,
a. 4 , 2 (mayor que 2, menor que 2) ¿cuál es la temperatura en grados Celsius?
3 100
3 1 1
b. 16 , 4 (mayor que 4, menor que 4) 17. Alex da de sus canicas a Beto. Beto da de éstas a Cali.
18 2 3
1
8 Cali da de éstas a Dani. Si Dani tiene cuatro canicas, ¿cuán-
c. 16 , 3 (mayor que 4, menor que 4) 4
9 tas canicas tenía Alex originalmente?
9. Cuando multiplicas cierto número por 3 y después le restas 18. El peso normal del cerebro de un elefante africano es de
7
, obtienes el mismo resultado que cuando multiplicas el 9 1> 4 lib. ¿Aproximadamente cuál será el peso del cere-
18 bro de 13 elefantes?
5 19. Escribe cada una de las expresiones siguientes en la
número por 2 y le sumas . ¿Cuál es el número?
12 forma más simple, usando exponentes positivos en la res-
10. Una universidad tuvo una reducción de personal académico puesta final:
1 1 1
- -
a 3
y perdió de sus profesores. Si quedaron 320 profesores
5 a. a b b.
después de la reducción, ¿cuántos profesores había origi- 3 a
1a 423
-
nalmente? a
c. -4 d. -1
1 a a
11. Una persona tiene 29 yardas de material disponible para -
2 a 3
e. -2
hacer uniformes para muñecas. Cada uniforme requiere a
3 20. Escribe cada una de las expresiones siguientes en la forma
yd de material.
4 más sencilla, usando exponentes positivos en la respuesta
a. ¿Cuántos uniformes puede hacer? final:
-
1 10 1 2 2 5 4 2
a. a b , a b b. a b # a b
b. ¿Cuánto material le va a sobrar?
12. Se vende un traje por $240. ¿Cuál es el precio original del 2 2 3 9
1 3 7 5 4 5 7 3
traje si el descuento fue del precio original?
4 c. a b , a b d. c a b d
5 3 6

© Lopez Mateos Editores. ISBN 978-607-95583-2-1, obra completa, versión electrónica, ISBN 978-607-95583-3-8, volumen 1, versión electrónica. Ejemplar asignado a: Helecto Villarroel gutierrez -
helecto@gmail.com. Fecha: 27 de octubre de 2014. Prohibida su modificación, copia o distribución.
402 Los números racionales como fracciones

27. Si f 1n2 =
21. Si a y b son números racionales, con a Z 0 y b Z 0, y si 3# n
2 , halla lo siguiente:
m y n son enteros, ¿cuáles de los casos siguientes son 4
verdaderos y cuáles falsos? Justifica tus respuestas. a. f 102 b. f 152 c. f 1 - 52
am a m-n
a. n = a b b. 1ab2 m = m # m
- 1 1 d. El mayor valor entero de n para el cual
f 1n2 6
b b a b 3
-1
1400
c. a -1 -1 b
2 1 1
= # 28. En los siguientes casos, determina cuál es el número mayor:
a + b 2 a + b
a. 32100 ó 4200
d. 21a + b 2 =
-1 -1 -1 2ab
b. 1 - 272 15 ó 1 - 32 50
- -
a + b -
a 1
f. a b =
mn m# n b 29. Hay un método sencillo para elevar al cuadrado cualquier
e. a = a a 1
b a número que esté formado por un número completo y . Por
22. Despeja el entero n en cada caso: 2
1 2 1 2 1 2
ejemplo a3 b = 3 4 + a b = 12 ; a 4 b =
-1 # 1
a. 2n = - 32 b. n3 = 2 2 4 2
27 2 2 2
4 # 5 + a b = 20 ; a 5 b = 5 # 6 + a b = 30 .
1 1 1 1 1
c. 2n # 27 = 1024 d. 2n # 27 = 64
e. 12 + n22 = 22 + n2 f. 3n = 275 2 4 2 2 4
1 2
23. Despeja x en cada una de las desigualdades siguientes, a. Escribe una propuesta para a n + b que generalice
donde x es un entero: 2
a. 3x Ú 81 b. 4x Ú 8 estos ejemplos, donde n es un número completo.
2x
c. 3 … 27 d. 2x 6 1 b. Justifica este procedimiento.
24. En cada uno de los pares siguientes, determina qué frac- 30. Considera estos productos:
Primer producto: a 1 + b a1 + b
ción es mayor: 1 1
4 10 4 8 3 10 4 10
a. a b ó a b b. a b ó a b
1 2
Segundo producto: a1 + b a1 + b a1 + b
3 3 4 5 1 1 1
10 10 100
3 9 100 1 2 3
c. a b ó a b d. a b óa # b
4 5 3
Tercer producto: a1 + b a1 + b a1 + b a1 + b
3 4 4 4 10 1 1 1 1
1 1 1 1 1 2 3 4
25. Sea S = + 2 + 3 + Á + 64 . a. Calcula el valor de cada producto. Con base en el pa-
3 3 3 3
trón de tus respuestas, propón el patrón para el
a. Usa la propiedad distributiva de la multiplicación so-
cuarto producto. Después verifica y determina si tu
bre la suma a fin de hallar una expresión para 3S.
propuesta fue correcta.
1 64
b. Muestra que 3S - S = 2S = 1 - a b . b. Propón un valor para el producto número 100.
3
c. Halla la expresión más sencilla posible para el pro-
c. Halla una expresión sencilla para la suma
ducto n-ésimo.

Muestra que la media aritmética a + b de dos núme-


1 1 1 1 1 a c
+ 2 + 3 + Á + n. 31.
3 3 3 3 2 b d
a c
ros racionales y corresponde al punto medio sobre la
26. En una sucesión aritmética, el primer término es 1 y el b d
centésimo término es 2. Halla lo siguiente: recta numérica.
a. El término número 50 (quincuagésimo)
b. La suma de los primeros cincuenta términos

Conexiones matemáticas 6-3

Comunicación 3. Supón que divides un número natural, n, entre un nú-


1 mero racional positivo menor que 1. ¿La respuesta siem-
1. Ana dice que dividir un número entre es lo mismo que to-
2 pre será menor que n, a veces menor que n o nunca
mar la mitad del número. ¿Cómo le respondes? menor que n? ¿Por qué?
2. Nora dice que dividir un número entre 2 es lo mismo
1
que multiplicarlo por . Quiere saber si está en lo co-
2
rrecto y de ser así, por qué. ¿Cómo le respondes?
.
© Lopez Mateos Editores. ISBN 978-607-95583-2-1, obra completa, versión electrónica, ISBN 978-607-95583-3-8, volumen 1, versión electrónica. Ejemplar asignado a: Helecto Villarroel gutierrez -
helecto@gmail.com. Fecha: 27 de octubre de 2014. Prohibida su modificación, copia o distribución.
Sección 6-3 Multiplicación y división de números racionales 403

4. Si las fracciones representadas por los puntos C y D sobre Imagina que estás enseñando división con fracciones. Para que
la siguiente recta numérica se multiplican, ¿qué punto re- la situación tenga sentido para los niños, algo que muchos
presenta mejor el producto? Explica por qué. maestros tratan de hacer es relacionar las matemáticas con
otras cosas. A veces lo intentan por medio de una situación de
A 0 B D C E 1 F
la vida real o de problemas relatados como una historia, para
mostrar la aplicación de cierto tema particular. ¿Cuál sería la
5. Si el producto de dos números es 1 y uno de los núme- 3 1
ros es mayor que 1, ¿qué sabes del otro número? Explica historia que contarías o tu modelo para 1 , ? (p. 55)
4 2
tu respuesta. a. ¿Cómo responderías a su pregunta?
1 3
6. Beto dice que para resolver el problema 12 , 3 basta b. Usa tu historia para obtener las respuestas.
4 4
1 3 1 1 c. De ser posible, consigue el libro de Ma y lee cómo respon-
hallar 12 , 3 = 4 y , = para obtener 4 . den la pregunta maestros estadounidenses comparados con
4 4 3 3
¿Cómo le respondes? los maestros chinos. Comunica tus resultados.
7. Cali dice que todo número racional tiene inverso multi- 14. ¿Usarías el problema de la tira cómica siguiente en tu salón
plicativo. ¿Cómo le respondes? de clase? ¿Por qué sí o por qué no? Resuelve el problema.
3#2
8. Dani dice que si tenemos , podríamos simplemente
4 5 “UNA CÁSCARA DE PLÁ- “SI UN PLÁTANO SIN
3#2 3#1 3
hacer = = . ¿Está en lo correcto? Explica TANO PESA 1/8 DEL PELAR PESA LO MISMO
5 4 5 2 10 PESO TOTAL DEL QUE UN PLÁTANO PELA-
por qué. PLÁTANO” DO, MÁS 7/8 DE ONZA...”
2 4 4 2
9. José dice que 2 # 3 = 2 # 3 pues la multiplicación es
5 5 5 5
conmutativa. ¿Tiene razón? Explica por qué sí o por qué no.
1
10. a. Ana dice que puede terminar una tarea en día traba-
2
1
jando sola. Su hijo Pedro puede terminar la tarea en
4
de día trabajando solo. ¿En cuánto tiempo terminarán “..¿CUÁNTO PESA EL ¡¡ABANDO-
PLÁTANO CON
la tarea si trabajan juntos?
CÁSCARA?” NEN EL
b. Si Ana puede terminar una tarea en a horas y Pedro
en b horas, ¿cuánto tardarán en terminar la tarea si BARCO!!
trabajan juntos?
11. Beto y Nelson comienzan a correr al mismo tiempo ha-
cia el pueblo donde vive el otro, cada uno a rapidez cons-
tante, Beto desde su pueblo (B) y Nelson desde el suyo
(N). Después de 3 h Beto llegó al pueblo N, mien-
3
tras que Nelson sólo corrió de la distancia entre los
4
Aprendizaje colectivo
pueblos. Responde lo siguiente y explica cuidadosa-
mente tu razonamiento de manera que lo pueda com- 15. Escoge un edificio de ladrillo en tu campus. Mide la al-
prender un alumno del quinto grado. tura de un ladrillo y el grueso de la capa de mezcla entre
a. Si la rapidez de Beto era de 6 mph, halla la rapidez de Nelson. los ladrillos. Estima la altura del edificio y después cal-
1 cula la altura del edificio. ¿Usaste números racionales en
b. Si la rapidez de Beto durante el último de la distancia tus cálculos?
4
entre los pueblos se redujo en 1 mph respecto a la ante-
Preguntas del salón de clase
1
rior, ¿cuánto tardó en recorrer el último de la distancia? 16. Jaime no está seguro de cuándo multiplicar por una frac-
4
12. Adela camina del pueblo A al pueblo B a una rapidez ción y cuándo dividir. Tiene la siguiente lista de proble-
constante de 3 mph y regresa del pueblo B al pueblo A a mas. ¿Cómo le puedes ayudar a resolver estos problemas
una rapidez constante de 5 mph. Responde lo siguiente: de manera que sea capaz de resolver por sí mismo pro-
a. ¿Cuál es la rapidez promedio de Adela en toda su ca- blemas similares?
minata? (No es 4 mph.) Justifica tu razonamiento. 3 1
a. de un paquete de azúcar llena taza. ¿Cuántas tazas
b. ¿Cuál es la rapidez promedio del viaje redondo si ella 4 2
caminó de ida a a mph y regresó a b mph? hay en un paquete de azúcar?
b. ¿Cuántos paquetes de azúcar llenarán 2 tazas?
Solución abierta 1
c. Si se requiere taza de azúcar para hacer dos piezas de
13. En el libro Knowing and Teaching Elementary Mathematics, 3
Liping Ma presenta el escenario siguiente: pan, ¿cuántas tazas de azúcar se necesitan para tres piezas?

© Lopez Mateos Editores. ISBN 978-607-95583-2-1, obra completa, versión electrónica, ISBN 978-607-95583-3-8, volumen 1, versión electrónica. Ejemplar asignado a: Helecto Villarroel gutierrez -
helecto@gmail.com. Fecha: 27 de octubre de 2014. Prohibida su modificación, copia o distribución.
404 Los números racionales como fracciones

3 25. Cada estudiante de una escuela primaria lleva una lengua


d. Si se requieren de taza de azúcar para hacer 1 galón
4 extranjera. Dos tercios de los estudiantes toman inglés,
de ponche, ¿cuántos galones se pueden hacer con 2 1 1
tazas de azúcar? toman francés, toman alemán y el resto toma otra len-
9 18
3 1 gua extranjera. Si hay 720 estudiantes en la escuela, ¿cuántos
e. Si tienes 22 pulg de listón y necesitas 1 pulg para
8 4 no toman inglés, francés o alemán?
decorar una muñeca, ¿cuántas muñecas se pueden de-
corar y cuánto listón sobrará? Preguntas del Third International Mathematics and
17. Al trabajar en el problema de simplificar Science Study (TIMSS) (Tercer Estudio Internacional
3#1#2 sobre las Matemáticas y la Ciencia)
1
4 2 3 Hay 600 pelotas en una caja, y de las pelotas son rojas.
3
un estudiante hizo lo siguiente: ¿Cuántas pelotas rojas hay en la caja?
Respuesta: ____________ pelotas rojas
3#1 3#2
= a # b a # b = #
3#1#2 3 6 18
= TIMSS 2003, Grado 4
4 2 3 4 2 4 3 8 12 96
1
En una pala de cocina cabe kg de harina. ¿Cuántas palas
¿Cuál es el error, si es que lo hay? 5
18. Una estudiante asegura que la división siempre hace las co- de harina se necesitan para llenar una bolsa de 6 kg?
Respuesta: ____________
sas más pequeñas, de modo que 5 , a b no puede ser 10
1
2 TIMSS 2003, Grado 8
pues es mayor que el número con que comenzó. ¿Cómo le
Pregunta del National Assessment of Educational Progress
respondes?
(NAEP) (Evaluación Nacional del Progreso Educativo)
m + n m
19. Un estudiante simplifica la fracción a . ¿Cómo Ambas figuras presentan la misma escala. Las marcas en la
p + n p
ayudarías a este estudiante? escala no tienen nombre, excepto el punto cero.
a 1
20. Margarita dice que a , b = para cualesquier dos núme- El peso del queso es de libra. ¿Cuánto pesan las dos
b 2
ros completos a y b Z 0. Ella quiere saber por qué. ¿Cómo manzanas?
le respondes?
21. Julieta dice que aprendió que 17 entre 5 se puede escribir
1
como 17 , 5 = 3 R2, pero ella piensa que es mu- 2 Libra

17 2
cho mejor escribir 17 , 5 = = 3 . ¿Cómo le res-
5 5
pondes?
22. Tito quisiera saber cómo un problema que describa una 0 0
situación le puede ayudar a encontrar la respuesta a
2 1
4 , sin usar el algoritmo de invertir y multiplicar.
3 3
¿Cómo le respondes?
1 3 3
23. Franca afirmó que 2 # = 2 . ¿Qué hizo Franca y
2 5 10
cómo le podrías ayudar?
NAEP, Grado 8, 2007
Problemas de repaso
24. Efectúa cada uno de los cálculos siguientes. Deja tus res-
puestas en la forma más simple.
-3 7 1 -4 5
a. + b. + +
16 4 6 9 3
-5 -5 4 5
c. 3 2 - d. 3 + 4
2 3# #
2 3 3 5 6
1 5 - 1 5
e. 5 - 3 f. 4 - 5
6 8 3 12

© Lopez Mateos Editores. ISBN 978-607-95583-2-1, obra completa, versión electrónica, ISBN 978-607-95583-3-8, volumen 1, versión electrónica. Ejemplar asignado a: Helecto Villarroel gutierrez -
helecto@gmail.com. Fecha: 27 de octubre de 2014. Prohibida su modificación, copia o distribución.
Resumen del capítulo 405

1
ROMPECABEZAS Una señora piensa repartir sus gatos entre sus tres hijas de la manera siguiente: de los
2
1 1
gatos a la hija mayor, de los gatos a la hija de en medio y de los gatos a la hija menor. Como la señora tiene
3 9
17 gatos, las hijas decidieron que no podrían satisfacer los deseos de su mamá. El juez encargado del tes-
tamento estuvo de acuerdo en prestar a las hijas un gato de modo que la repartición pudiera hacerse se-
1 1 1
gún las indicaciones dadas en el testamento. Ahora, de 18 es 9; de 18 es 6; y de 18 es 2. Como
2 3 9
9 + 6 + 2 = 17, las hijas pudieron dividir los 17 gatos y regresar el gato prestado. Obviamente no nece-
sitaban el gato extra para cumplir la petición de su madre, pero no pudieron dividir 17 en mitades, tercios
y novenos. ¿Se cumplió realmente el deseo de la señora?

Sugerencia para resolver el problema preliminar


Plantea la ecuación en la que la incógnita sea el costo de mantenimiento del caballo.
Expresa la cantidad de la pérdida de dos maneras diferentes en términos de la incógnita.

Resumen del capítulo

I. Fracciones y números racionales a c


G. Si b 7 0 y d 7 0, entonces 7 si y sólo si
a b d
A. Los números de la forma , donde a y b son ad 7 bc.
b
enteros y b Z 0, son números racionales; a es a c a a + c c
H. Si 6 , entonces 6 6 .
el numerador y b es el denominador. b d b b + d d
B. Un número racional se puede usar como sigue: I. Definición de menor que y mayor que
1. Un problema de división o la solución a un
a c c a
problema de multiplicación 6 si y sólo si - 7 0
2. Una partición, o parte, de un todo b d d b
3. Una razón c a a c
7 si y sólo si 6
4. Una probabilidad d b b d
C. Ley fundamental de las fracciones: Para cualquier II. Operaciones en números racionales
a a ac
fracción y cualquier número c Z 0, = . a c a + c
b b bc A. + =
a c b b b
D. Dos fracciones y son iguales (equivalentes)
b d a c ad + bc
B. + =
si y sólo si ad = bc. b d bd
a c ad - bc
E. Si MDC1a, b2 = 1 y b 7 0, entonces se dice que C. - =
a b d bd
está en su forma más simple. a#c ac
b D. =
a b d bd
F. Si 0 … a 6 b, entonces es una fracción pro-
b a c a d ad
a
pia. Si a Ú b 7 0, es una fracción impropia. E. , = # = , donde c Z 0
b b d b c bc

© Lopez Mateos Editores. ISBN 978-607-95583-2-1, obra completa, versión electrónica, ISBN 978-607-95583-3-8, volumen 1, versión electrónica. Ejemplar asignado a: Helecto Villarroel gutierrez -
helecto@gmail.com. Fecha: 27 de octubre de 2014. Prohibida su modificación, copia o distribución.
406 Los números racionales como fracciones

III. Propiedades de los números racionales E. Propiedad de la multiplicación por cero para
a
A. números racionales: Si es cualquier número
b
a# a
Suma Resta Multiplicación División
racional, entonces 0 = 0 = 0 # .
b b
IV. Exponentes
Cerradura Sí Sí Sí Sí, excepto
A. am = a # a # a # Á # a, donde m es un entero positivo
para la
m factores
división
y a es un número racional (definición)
entre 0
B. Definiciones y teoremas de los exponentes que
Conmutatividad Sí No Sí No incluyen números racionales
Asociatividad Sí No Sí No 1. a 0 = 1, donde a Z 0 (definición)
Identidad Sí No Sí No - 1
Inverso Sí No Sí, excepto 0 No
2. a m = m , donde a Z 0 (definición)
a
3. am # an = am + n
B. Propiedad distributiva de la multiplicación am
4. n = am - n, donde a Z 0
sobre la suma para números racionales x, y y z: a
5. 1am2n = amn
x1 y + z2 = xy + xz
a m am
6. a b = m , donde b Z 0
C. Propiedad de densidad: Entre dos números ra- b b
cionales cualesquiera, hay otro número racional. 7. 1ab2m = am # b m
-
a m b m
D. Propiedad de la multiplicación para la igualdad: 8. a b = a b
b a
a c e
Si , y son números racionales cualesquiera
b d f
a c a e c e
tales que = , entonces # = # .
b d b f d f

Revisión del capítulo

1. Para cada uno de los casos siguientes, traza un dia- 5. Halla los inversos aditivo y multiplicativo de cada
grama que ilustre la fracción: caso:
3 2 1
a. b. a. 3 b. 3
4 3 7
3#2 5 3
c. c. d. -
4 3 6 4
5 6. Ordena los números siguientes de menor a mayor:
2. Escribe tres números racionales iguales a . 300 300
6 - 1 7 , 0, - 2 1 , 69 , 71 , a 71 b , 1 , a 74 b
3. Escribe cada uno de los siguientes números raciona- 8 3 140 140 140 2 73
les en su forma más simple:
4#7# 5 4 7 #5
24 ax 2 0 7. ¿Se puede escribir como # y obte-
a. b. c. 5 8 14 8 14 5
28 bx 17
45 b 2 + bx 16 ner la misma respuesta? ¿Por qué sí o por qué no ?
d. e. f.
81 b + x 216 8. Usa matemática mental para calcular lo siguiente.
x+a xa Explica tu método.
g. * h.
a. # 18 # 92
1 5
x-a x + a b. 36 # 1
4. Reemplaza la coma con 7 , 6 o = en cada uno de 3 6
los pares siguientes para obtener una proposición 1
9. Juan tiene 54 yd de material. Si necesita cortar la
verdadera: 4
6 120 -3 -5 1
a. , b. , tela en piezas de 3 yd de largo, ¿cuántas piezas
10 200 4 6 12
10 20
1 2 1 3 puede obtener? ¿Cuánto material le va a sobrar?
c. a b , a b d. a 1 + b , a 1 + b
4 4
5 5 3 3

© Lopez Mateos Editores. ISBN 978-607-95583-2-1, obra completa, versión electrónica, ISBN 978-607-95583-3-8, volumen 1, versión electrónica. Ejemplar asignado a: Helecto Villarroel gutierrez -
helecto@gmail.com. Fecha: 27 de octubre de 2014. Prohibida su modificación, copia o distribución.
Resumen del capítulo 407

10. Sin realizar las operaciones dadas, escoge la estima- 2


22. Si de todos los estudiantes de una academia son
ción más apropiada (de los números entre paréntesis) 3
para las expresiones siguientes: 2
mujeres y de ellas son rubias, ¿qué fracción des-
3 1 5
30 8 cribe el número de mujeres rubias en esa academia?
115, 20, 82
8# 3 - 11 - 12
a.
1 8 23. Explica cuál es mayor: ó .
4 3 9 10
9 9 24. Despeja x en cada uno de los siguientes casos:
b. a b 6 115, 0, 1322
3 4 15 # a. 7x = 343
+ +
800 5000 6 - 1
b. 2 3x =
a , 4, 0b
1 1 1 512
c. ,
407 1609 4 25. Despeja x en cada uno de los siguientes casos:
11. Justifica el algoritmo de invierte y multiplica para la 5 5 1 2
división de números racionales de dos maneras dife- a. 2x - = b. x + 2 = 5
3 6 2 3
rentes.
5 1 20 + x 4 1
12. Narra una situación que se modele con 4 , . Re- c. = d. 2x + 4 = 3x -
8 2 x 5 3
suelve el problema trazando diagramas apropiados. 26. Escribe cada uno de los casos siguientes en su forma
3 4 más simple. Deja todas las respuestas con exponen-
13. Halla dos números racionales entre y . tes positivos.
4 5
1x3a 12 2
- -
14. Supón que está rota la tecla , de tu calculadora,
a. -
pero que la tecla 1/x funciona. Explica cómo pue- xa 1
des calcular 504792> 23. -
x2y 2 2
-
15. Jaime está comenzando una dieta. Cuando llegó a casa b. a -3 2 b
1 x y
comió de la mitad de pizza que quedó de la noche an- 27. Halla cada suma o diferencia.
3
terior. La pizza completa contiene aproximadamente 3a b 5 2
a. 2 + 2 2 b. 2 -
2000 calorías. ¿Cuántas calorías consumió Jaime? xy xy xy 3x
16. Si una persona obtuvo cara en un volado con una a b 7 5
c. 3 2 - d. 3 2 + 2 3
moneda legal la mitad de las veces y obtuvo 376 ca- xyz xyz 23 23
ras, ¿cuántos volados realizó? 28. Miguel dibujó la figura siguiente para hallar cuántas
17. Si una persona obtuvo 240 caras al echar 1000 vola- 1
dos, ¿qué fracción de las veces la persona obtuvo piezas de cinta de yarda de largo podía cortar de
2
cara? Da la respuesta en su forma más simple. 3
3 una cinta de 1 yardas de largo.
18. Si una universidad ganó de sus partidos de baloncesto 4
4
5
femenino y de sus partidos de baloncesto masculino, (1) (2) (3) sobra
8
explica si es razonable decir que la universidad ganó 1 1 3
3 5 0 4 2 4 1 11
4
11
2 13
4 2
+ de sus partidos de baloncesto.
4 8
19. Explica por qué un número racional negativo por un nú- 3 1
De la figura concluyó que 1 , son 3 piezas con
mero racional negativo es un número racional positivo. 4 2
20. Un estudiante alega que la siguiente fracción no es 1 1
un número racional pues no es el cociente de dos yd sobrantes, de modo que la respuesta es 3 piezas.
4 4
enteros: 7 2 14 1
Él verificó usando el algoritmo # = = 3 y
2 4 1 4 2
está confundido pues tiene dos respuestas distintas.
3
¿Cómo puedes ayudarlo?
3 29. Si a, b y c son enteros diferentes de cero, expresa lo
4 x
siguiente como un número racional , donde x y y son
y
¿Qué le respondes? enteros:
21. Mónica quiere fertilizar 12 acres de tierra de un par- -1 - -1 -1
+ b 1 + c
a b
1 a
que. Si cada acre requiere 9 bolsas, ¿cuántas bol- 2
3
sas necesita ella?

© Lopez Mateos Editores. ISBN 978-607-95583-2-1, obra completa, versión electrónica, ISBN 978-607-95583-3-8, volumen 1, versión electrónica. Ejemplar asignado a: Helecto Villarroel gutierrez -
helecto@gmail.com. Fecha: 27 de octubre de 2014. Prohibida su modificación, copia o distribución.
408 Los números racionales como fracciones

Bibliografía seleccionada

Alcoro, P., A. Alston, and N. Katjims. “Fractions Kalder, R. “Teaching Preservice Secondary Teachers
Attack! Children Thinking and Talking Mathemat- How to Teach Elementary Mathematics Concepts.”
ically.” Teaching Children Mathematics 6 (May 2000): Mathematics Teacher 2007, 101(2): 146–149.
562–567. Lamon, S. Teaching Fractions and Ratios for Understanding.
Anderson, C., E. Anderson, and E. Wenzel. “Oil and Mahwah, NJ: Lawrence Erlbaum Associates, 1999.
Water Don’t Mix, But They Teach Fractions.” Ma, L. Knowing and Teaching Elementary Mathematics.
Teaching Children Mathematics 7 (November 2000): Mahwah, N.J.: Lawrence Erlbaum Associates, 1999.
174–178. Mack, N. “Connecting to Develop Computational
Bana, J., B. Farrell, and A. McIntosh. “Student Error Fluency with Fractions.” Teaching Children Mathe-
Patterns in Fraction and Decimal Concepts.” In matics 11 (November 2004): 226–231.
Proceedings of the 20th Annual Conference of the Middleton, J., M. van den Heuvel-Panhuizen, and
Mathematics Education Research Group of Australasia J. Shew. “Using Bar Representations as a Model
(pp. 81–87), edited by F. Biddulph and K. Carr. for Connecting Concepts of Rational Numbers.”
Aotearoa, New Zealand: MERGA (1997). Mathematics Teaching in the Middle School 3 (Janu-
Bay-Williams, J., and S. Martine. “Thinking Rationally ary 1998): 302–312.
about Number and Operations in the Middle Moss, J., and R. Case. “Developing Children’s Under-
School.” Mathematics Teaching in the Middle School 8 standing of the Rational Numbers: A New Model and
(February 2003): 282–287. an Experimental Curriculum.” Journal For Research in
Behr, M., I. Wachsmuth, T. Post, and R. Lesh. “Order Mathematics Education 1999, 30(2): 122–147.
and Equivalence of Rational Numbers; A Clinical Moyer, P., and E. Mailley. “Inchworm and a Half: Devel-
Teaching Experiment.” Journal for Research in Mathe- oping Fraction and Measurement Concepts Using
matics Education 1984, 15 (5): 323–341. Mathematical Representations.” Teaching Children
Boston, M., M. Smith, and A. Hillen. “Building on Mathematics 10 ( January 2004): 244–252.
Students’ Intuitive Strategies to Make Sense of Perlwitz, M. “Dividing Fractions: Reconciling Self-
Cross Multiplication.” Mathematics Teaching in the Generated Solutions with Algorithmic Answers.”
Middle School 9 (November 2003): 150–155. Mathematics Teaching in the Middle School 10
Burrill, G., et al. Figure This! Washington, DC: Wid- (October 2005): 278–283.
meyer Corporation, 2002. (www.figurethis.org) Perlwitz, M. “Two Students’ Constructed Strategies to
Conference Board of the Mathematical Sciences Divide Fractions.” Mathematics Teaching in the Middle
(CBMS). The Mathematical Education of Teachers, School 10 (February 2004): 121–126.
vol. 11 of CBMS Issues in Mathematics Education Roddick, C., and C. Silvas-Centeno. “Developing
Washington, DC: The American Mathematical So- Understanding of Fractions through Pattern
ciety and the Mathematical Association of America, Blocks and Fair Trade.” Teaching Children Mathe-
2001. See also http://www.cbmsweb.org/MET_ matics 14 (October 2007): 140–145.
Document/index.htm Tirosh, D. “Enhancing Prospective Teacher’s Knowledge
Empson, S. “Equal Sharing and the Roots of Fraction of Children’s Conceptions: The Case of Division of
Equivalence.” Teaching Children Mathematics 7 (March Fractions.” Journal For Research in Mathematics Educa-
2001): 421–425. tion 2000 31(5): 5–25.
English, L., and G. Halford. Mathematics Education: Tzur, R. “An Integrated Study of Children’s Construction
Models and Processes. Mahwah, N.J.: LEA, 1995. of Improper Fractions and the Teacher’s Role in Pro-
Fennel, F. “Focal Points—What’s Your Focus and Why?” moting Learning.” Journal For Research in Mathematics
Teaching Children Mathematics 14 (January 2008): Education 1999, 30(4): 390–416.
315–316. Wearne, D., and J. Hiebert, “Constructing and Using
Glidden, P. “Build Your Own Fraction Computer.” Meaning for Mathematical Symbols: The Case of
Mathematics Teaching in the Middle School 8 Decimal Fractions.” In Number Concepts and Opera-
(December 2002): 204–208. tions in the Middle Grades, edited by J. Hiebert and
Gregg, J., and D. Gregg. “Measurement and Fair-Sharing M. Behr. Hillsdale, NJ: LEA, 1988.
Models for Dividing Fractions.” Mathematics Teaching Wu, Z. “Multiplying Fractions.” Teaching Children Mathe-
in the Middle School 12 (May 2007): 390–396. matics 8 (November 2001): 174–177.
Hope, J., and D. Owens. “An Analysis of the Difficulty Yang, D., and R. Reys, “One Fraction Problem, Many
of Learning Fractions.” Focus on Learning Problems Solution Paths.” Mathematics Teaching in the Middle
in Mathematics 1987, 9(4): 25–40. School 7 (September 2001): 10–14.
© Lopez Mateos Editores. ISBN 978-607-95583-2-1, obra completa, versión electrónica, ISBN 978-607-95583-3-8, volumen 1, versión electrónica. Ejemplar asignado a: Helecto Villarroel gutierrez -
helecto@gmail.com. Fecha: 27 de octubre de 2014. Prohibida su modificación, copia o distribución.
Decimales y
números reales
CAPÍTULO

Problema preliminar
Juanita retó a Ricardo a una competencia en donde se usarían todos los dígitos 0, 1, 2, 3, Á , 9
para formar dos decimales cuya suma fuera 1. No habría otra restricción. ¿Cómo crees que
respondería Ricardo a ese reto?

409
© Lopez Mateos Editores. ISBN 978-607-95583-2-1, obra completa, versión electrónica, ISBN 978-607-95583-3-8, volumen 1, versión electrónica. Ejemplar asignado a: Helecto Villarroel gutierrez -
helecto@gmail.com. Fecha: 27 de octubre de 2014. Prohibida su modificación, copia o distribución.
410 Decimales y números reales

E n los Principios y objetivos (NCTM 2000), hallamos que los estudiantes de los grados 6
a 8 deberán:

• trabajar de manera flexible con fracciones, decimales y porcentajes para resolver problemas;
• comparar y ordenar, de manera eficiente, fracciones, decimales y porcentajes, y hallar su ubi-
cación aproximada en una recta numérica; . . .
• desarrollar una comprensión de números grandes y reconocer y usar, de manera apropiada, no-
tación exponencial, científica y de calculadora; . . .
• comprender el significado y efecto de las operaciones aritméticas con fracciones, decimales y
enteros. (p. 214)
En Puntos focales, se espera que desde el grado 4 los estudiantes comiencen a relacionar “su
comprensión de las fracciones con la lectura y escritura de decimales, que comparen y or-
denen decimales y que estimen cantidades decimales o fraccionarias al resolver problemas.
Ellos relacionan fracciones y decimales equivalentes al comparar modelos con símbolos y
localizar símbolos equivalentes en la recta numérica” (p. 31).
En este capítulo desarrollamos la comprensión necesaria que deben tener los maestros
para ayudar a los estudiantes a cumplir estas expectativas. Para comenzar, veamos una breve
historia de los decimales.
Aunque el sistema de numeración indoarábigo analizado en el capítulo 2 se usó en mu-
chos lugares alrededor del siglo sexto, la extensión del sistema a los decimales realizada
por el científico holandés Simon Stevin no tuvo lugar en las matemáticas europeas hasta
casi mil años después. La única mejora importante en el sistema desde la época de Stevin
había sido en la notación. Incluso ahora no hay una forma aceptada universalmente para
escribir un decimal. Por ejemplo, en Estados Unidos y México escribimos 6.75; en Ingla-
terra este número se escribe 6 # 75; y en Alemania, Francia y España se escribe 6,75.
Los decimales juegan un papel importante en la educación matemática de los estudiantes
de los grados 4 a 8. En la sección 7-1, exploramos las relaciones entre fracciones y decima-
les y vemos cómo los decimales son una extensión del sistema de base diez. Más adelante en
el capítulo consideramos operaciones con decimales, propiedades de los decimales, núme-
ros reales y elementos de álgebra con los números reales.

◆ Nota En 1584 Simon Stevin (1548–1620), un superintendente general del ejército holandés, es-
histórica cribió La Thiende (La tienda), un trabajo que proporcionaba reglas para calcular con deci-
males. Él no sólo estableció reglas para los cálculos con decimales, sino que también sugirió
aplicaciones prácticas de los decimales y recomendó que su gobierno las adoptara. Para
mostrar su valor posicional, Stevin usaba numerales dentro de círculos entre los dígitos.
Por ejemplo escribía 0.4789 como 4 s 1 7s 2 8s 3 9s 4. ◆

© Lopez Mateos Editores. ISBN 978-607-95583-2-1, obra completa, versión electrónica, ISBN 978-607-95583-3-8, volumen 1, versión electrónica. Ejemplar asignado a: Helecto Villarroel gutierrez -
helecto@gmail.com. Fecha: 27 de octubre de 2014. Prohibida su modificación, copia o distribución.
Sección 7-1 Introducción a los decimales 411

7-1 Introducción a los decimales

La palabra decimal viene del latín decem, que significa “diez”. La mayoría de las personas co-
nocen los decimales por el dinero. Por ejemplo, en un anuncio donde se dice que una bici-
95
cleta cuesta $128.95, el punto en $128.95 es el punto decimal. Como $0.95 es de un
100
95
peso, tenemos $128.95 = 128 + pesos. Como $0.95 es 9 monedas de 10 y 5 centavos,
100
1 1
una moneda de diez centavos es de un peso, y 1 centavo es de un peso, entonces,
10 100
1 1 95
$0.95 es 9 # + 5# ,ó de un peso.
10 100 100
En consecuencia,
1 1
128.95 = 1 # 102 + 2 # 10 + 8 # 1 + 9 # + 5# 2
10 10
Los dígitos en 128.95 corresponden respectivamente a los siguientes valores posicionales:
1 1
102, 10, 1, y 2 . (Cada término de la sucesión de valores posicionales después del pri-
10 10
1
mero es del término anterior.) Así, 12.61843 representa
10
6 1 8 4 3 61,843
1 # 10 + 2 # 1 + 1
+ 2
+ 3
+ 4
+ 5
, ó 12
10 10 10 10 10 100,000
El decimal 12.61843 se lee “doce y sesenta y un mil ochocientos cuarenta y tres cienmilési-
mos”. Cada lugar de un decimal se puede nombrar por medio de una potencia de 10. Por
ejemplo, en 12.61843 los lugares se pueden nombrar como se muestra en la tabla 7-1.

Tabla 7-1

1 2 # 6 1 8 4 3
Decenas
DS

Unidades

Décimas

Centésimas

Milésimas

Diezmilésimas

Cienmilésimas

61,843
O B S E R VA C I Ó N Nota que representa una división de dos números completos cuan-
100,000
61,843
do el divisor es una potencia de 10; en este caso, 105. El decimal para es 0.61843.
100,000

Los decimales se pueden introducir con materiales concretos. Por ejemplo, supón que
una barra en los cubos de base diez representa 1 unidad (en lugar de que un cubo repre-

© Lopez Mateos Editores. ISBN 978-607-95583-2-1, obra completa, versión electrónica, ISBN 978-607-95583-3-8, volumen 1, versión electrónica. Ejemplar asignado a: Helecto Villarroel gutierrez -
helecto@gmail.com. Fecha: 27 de octubre de 2014. Prohibida su modificación, copia o distribución.
412 Decimales y números reales

1
sente 1 unidad). Si ése es el caso, entonces el cubo representa . Entonces 5.4 se podría
10
representar como se ve en la figura 7-1(a).

1 1 1 1 1

1 1 1 1
— — — —
10 10 10 10

54 cubos o 54
5.4 10
(a) (b)

Figura 7-1

Si quisiéramos una interpretación diferente de 5.4, podríamos usar la figura 7-1(b), la cual
54
muestra que 5.4 también es equivalente a 54 décimas, o . Esta equivalencia se puede ex-
10
presar simbólicamente como
4 50 4 54
5.4 = 5 + 0.4 = 5 + = + =
10 10 10 10
Este enfoque da a los estudiantes una relación concreta entre las fracciones y los decimales.

AHORA INTENTA ÉSTE 7-1 En un conjunto de cubos de base diez, sea 1 losa quien represente 1 unidad.
a. ¿Qué representa 1 barra?
b. ¿Qué representa 1 bloque?
c. Representa 1.23 usando los cubos con estas equivalencias.

Para representar un decimal como 2.235, podemos pensar en el bloque de la figura 7-2(a)
1 1
como una unidad. Entonces una losa representa , una barra representa y un cubo
10 100
1
representa . Usando estos objetos podemos mostrar una representación de 2.235 en la
1000
figura 7-2(b).

Bloque 2 • 2 3 5
(a) (b)

Figura 7-2

La tabla 7-2 muestra otros ejemplos de decimales, su significado fraccional y su notación


fraccionaria común.

© Lopez Mateos Editores. ISBN 978-607-95583-2-1, obra completa, versión electrónica, ISBN 978-607-95583-3-8, volumen 1, versión electrónica. Ejemplar asignado a: Helecto Villarroel gutierrez -
helecto@gmail.com. Fecha: 27 de octubre de 2014. Prohibida su modificación, copia o distribución.
Sección 7-1 Introducción a los decimales 413

Tabla 7-2

Decimal Significado fraccional expandido Notación fraccionaria común


3 3 53
5.3 5 + 5 ,ó
10 10 10
0 2 2
0.02 0 + +
10 100 100
0 1 0 3 103 20,103
2.0103 2 + + + + 2 ,ó
10 100 1000 10,000 10,000 10,000
- 3.6 - a3 + 6 b - 3 6 , ó - 36
10 10 10

Los decimales también pueden escribirse en forma expandida usando valor posicional y ex-
ponentes negativos. Así,
12.61843 = 1 # 101 + 2 # 100 + 6 # 10 + 1 # 10 + 8 # 10 + 4 # 10 + 3 # 10
-1 -2 -3 -4 -5

AHORA INTENTA ÉSTE 7-2 A veces los decimales pueden causar confusión. En la tira cómica “Lo-
renzo y Pepita” los autores quieren decir que se elevó el precio de los tomates. ¿Es eso lo que dice la
tira? Da otros ejemplos de este tipo de error sobre pesos y centavos.
¿Son realmente buenos ¡Están buenísimos! Yo ¿Los acabas de Sí, costaban
estos tomates? mismo los acabo de subir. subir . . .? 90¢

El ejemplo 7-1 muestra cómo convertir números racionales cuyos denominadores son
potencias de 10 a decimales.

Ejemplo 7-1 Convierte cada uno de los casos siguientes a decimales:










25 56 205
a. b. c.
10 100 10,000

25 2 # 10 + 5 2 # 10 5 5
Solución a. = = + = 2 + = 2.5
10 10 10 10 10
56 5 # 10 + 6 5 # 10 6 5 6
b. = 2
= + = 0 + + = 0.56
100 10 102 102 10 102
205 2 # 102 + 0 # 10 + 5 2 # 102 0 # 10 5
c. = 4
= 4
+ 4
+
10,000 10 10 10 104
2 0 5 0 2 0 5
= 2
+ 3
+ 4
= 0 + 1
+ 2
+ 3
+ = 0.0205
10 10 10 10 10 10 104

© Lopez Mateos Editores. ISBN 978-607-95583-2-1, obra completa, versión electrónica, ISBN 978-607-95583-3-8, volumen 1, versión electrónica. Ejemplar asignado a: Helecto Villarroel gutierrez -
helecto@gmail.com. Fecha: 27 de octubre de 2014. Prohibida su modificación, copia o distribución.
414 Decimales y números reales

Reforzamos las ideas presentadas en el ejemplo 7-1 mediante el uso de una calculadora.
En el ejemplo 7-1(b), teclea 5 6 , 1 0 0 = y observa la pantalla. Divide entre 10 y
observa la nueva colocación del punto decimal. Una vez más, divide entre 10 (lo cual equi-
vale a dividir el número original, 56, entre 10,000) y nota la colocación del punto decimal.
Esto conduce a la regla general siguiente para dividir un decimal entre una potencia entera
positiva de 10:
Para dividir un decimal entre 10n, comienza en el punto decimal, cuenta n valores posicionales ha-
cia la izquierda, añadiendo ceros si es necesario e insertando el punto decimal a la izquierda del n-
ésimo valor posicional contado.

Las fracciones del ejemplo 7-1 son fáciles de convertir a decimales pues los denominadores
3
son potencias de 10. Si el denominador de una fracción no es una potencia de 10, como en ,
5
usamos la estrategia para resolver problemas convertir el problema en uno más sencillo donde ya
3
sepamos qué hacer. Primero reescribimos como una fracción en la cual el denominador es
5
una potencia de 10 y luego convertimos la fracción a decimal.
3 2#3 6
= # = = 0.6
5 2 5 10
La razón para multiplicar el numerador y el denominador por 2 es obvia cuando observa-
mos que 10 = 5 # 2 ó 2 # 5.

AHORA INTENTA ÉSTE 7-3 Considera las fracciones siguientes. ¿Cómo podrías convertirlas a decimales?
5
a.
2
3 3 3
b. (Sugerencia: = 3 . ¿Por cuál potencia de 5 debemos multiplicar para que el denominador sea
8 8 2
una potencia de 10?)
3 3 3
c. (Sugerencia: = 2 . ¿Por cuál potencia de 5 debemos multiplicar para que el denominador
20 20 2 #5
sea una potencia de 10?)

En Ahora intenta éste 7-3, se nota que si tenemos en el denominador de una fracción una po-
tencia de 10, entonces convertir la fracción a un decimal es simplemente dividir un número
entero entre una potencia de 10.
Nota que cualquier potencia de 10 es de la forma 10n y en general, como 10n =
12 # 52n = 2n # 5n, la factorización en primos del denominador que es una potencia de 10n debe
ser 2n # 5n. Usamos estas ideas para escribir como decimal cada fracción del ejemplo 7-2.

Ejemplo 7-2 Expresa cada uno de los casos siguientes como decimal:








7 1 1 7
a. b. c. d.
26 23 # 54 125 250

7 7 # 56 7 # 15,625 109,375
12 52
Solución a. = 6# 6
= = = 0.109375
26
2 5 # 6
106
1 1 # 21 2 2 2
12 # 524
b. 3 4 = 3 4 1 = 4 4 = = = 0.0002
2 #5 2 #5 #2 2 #5 104

© Lopez Mateos Editores. ISBN 978-607-95583-2-1, obra completa, versión electrónica, ISBN 978-607-95583-3-8, volumen 1, versión electrónica. Ejemplar asignado a: Helecto Villarroel gutierrez -
helecto@gmail.com. Fecha: 27 de octubre de 2014. Prohibida su modificación, copia o distribución.
Sección 7-1 Introducción a los decimales 415

1 1 1 # 23 8 8
15 22
c. = 3 = 3 3 = = = 0.008
125 5 5 2# # 3
103
7 7 7 # 22 28 28
12 52
d. = = = = = 0.028
250 #
2 5 3 #
2 5 23# 2 # 3
103

Una calculadora puede convertir rápidamente a decimales algunas fracciones cuyos denomi-
7
nadores se pueden escribir como potencias de 10. Por ejemplo, para hallar 6 , teclea 7 , 2
2
1
y x 6 = ; para convertir a decimal, teclea 1 , 1 2 5 = o teclea 1 2 5 1/x = .
125
1
La presentación en pantalla para en algunas calculadoras puede ser 8 - 03 , que es la no-
125
8
tación de calculadora para 3 , u 8 # 10-3. (Esta notación científica se analiza con más detalle
10
en el siguiente capítulo.)
En el ejemplo 7-2 las respuestas ilustran decimales exactos —decimales que se pueden es-
cribir con sólo un número finito de lugares a la derecha del punto decimal. No todo número racio-
nal se puede escribir como un decimal exacto. Por ejemplo, si tratamos de escri-
2
bir como un decimal usando el método recién desarrollado, primero tratamos de hallar
11
un número natural b tal que valga lo siguiente:

2 2b
= , donde 11b es una potencia de 10
11 11b

Por el teorema fundamental de la aritmética, los únicos factores primos de una potencia de 10
son 2 y 5. Como 11b tiene factor 11, no podemos escribir 11b como una potencia de 10. Por lo
2
tanto, parece que no se puede escribir como un decimal exacto. Un argumento similar
11
usando el teorema fundamental de la aritmética es válido en general, de manera que tene-
mos el resultado siguiente.

Teorema 7–1
a
Un número racional en su forma más simple puede reescribirse como decimal exacto si, y sólo
b
si, la factorización en primos del denominador no contiene otros primos aparte de 2 ó 5.

Ejemplo 7-3 ¿Cuáles de las siguientes fracciones se pueden escribir como decimales exactos?








7 11 21 37 3.85
a. b. c. d. e.
8 250 28 768 1000

Solución a. Como el denominador, 8, es 23 y la fracción está en su forma más simple, en-


7
tonces se puede escribir como un decimal exacto.
8
b. Como el denominador, 250, es 2 # 53 y la fracción está en su forma más simple,
11
entonces puede escribirse como un decimal exacto.
250

© Lopez Mateos Editores. ISBN 978-607-95583-2-1, obra completa, versión electrónica, ISBN 978-607-95583-3-8, volumen 1, versión electrónica. Ejemplar asignado a: Helecto Villarroel gutierrez -
helecto@gmail.com. Fecha: 27 de octubre de 2014. Prohibida su modificación, copia o distribución.
416 Decimales y números reales

21 21 3#7 3
c. se puede escribir en su forma más simple como = 2 = 2 . El deno-
28 28 2 #7 2
21
minador es ahora 22, de modo que se puede escribir como un decimal exacto.
28
37 37
d. En su forma más simple, la fracción = 8 tiene denominador con un fac-
768 2 #3
37
tor de 3, de modo que no se puede escribir como un decimal exacto.
768
3.85 # 100 385
e. = = 0.00385, un decimal exacto.
1000 100 105

O B S E R VA C I Ó N Como muestra el ejemplo 7-3(c) para determinar si un número racio-


a
nal se puede representar como un decimal exacto, consideramos la factorización en
b
primos del denominador sólo si la fracción está en su forma más simple.

Orden en los decimales exactos


Un decimal exacto se localiza fácilmente sobre una recta numérica pues puede repre-
a
sentarse como un número racional , donde b Z 0, y b es una potencia de 10. Por ejemplo,
b
56 56
considera 0.56, ó . Una manera de pensar acerca de es como el extremo derecho de
100 100
la 56ava (o quincuagésima sexta) parte de un segmento unitario dividido entre 100 partes
iguales, como en la figura 7-3.

0.56

0 5 0 1 2 3 55 56 57 1 5 100
100 100 100 100 100 100 100 100

Figura 7-3

Como apoyo para ordenar los decimales, sabemos que


5 5 # 10 50 56 60 6 # 10 6
= = 6 6 = =
10 10 # 10 100 100 100 10 # 10 10
56 5 6
de modo que , ó 0.56, está entre , ó 0.5; y , ó 0.6. Así, 0.5 6 0.56 6 0.6. De ma-
100 10 10
nera similar, dos decimales exactos se pueden ordenar convirtiendo cada uno de ellos
a
en números racionales de la forma , donde a y b son enteros con b Z 0, y determinar cuál
b
36 90 90 36
es mayor. Por ejemplo, como 0.36 = , 0.9 = 0.90 = ,y 7 , se sigue que
100 100 100 100
0.9 7 0.36. Se podría decir también que 0.9 7 0.36 porque $0.90 es 90¢ y $0.36 es 36¢ y
90¢ 7 36¢.

© Lopez Mateos Editores. ISBN 978-607-95583-2-1, obra completa, versión electrónica, ISBN 978-607-95583-3-8, volumen 1, versión electrónica. Ejemplar asignado a: Helecto Villarroel gutierrez -
helecto@gmail.com. Fecha: 27 de octubre de 2014. Prohibida su modificación, copia o distribución.
Sección 7-1 Introducción a los decimales 417

Página de un libro de texto VALOR POSICIONAL DECIMAL

Lección 11-2

Valor posicional decimal


Idea clave Calentamiento
Hay muchas Di el valor del dígito
maneras de rojo para cada
representar
números
Aprende número.

Algunas maneras de
decimales.

representar decimales
ex Aquí hay varias maneras de representar 1.48.
ef l i ó
R

Recta numérica:

¡Piensa!
Puedo usar objetos,
trazar figuras o
hacer una tabla para
representar 1.48.

Mallas:

1 unidad 4 décimas 8 centésimas

Tabla de valor decenas unidades décimas centésimas


posicional:

Forma expandida:
Forma común:
Forma oral: Uno y cuarenta y ocho centésimos

Ejemplo
Escribe la forma oral y la forma expandida
de 5.02. Después di el valor del dígito rojo. Escribo la palabra
Forma oral: cinco y dos centésimas “y” en vez del punto
decimal.
Forma expandida: 5 + 0.02
El dígito rojo está en el lugar de las centésimas, de
modo que su valor es 2 centésimas, 0.02.

Tema de plática
¿Qué dígito está en el lugar de las décimas en 1.48?
Explica cómo localizar 1.48 en Colócalo en la RED
Más ejemplos
una recta numérica.

Fuente: Scott Foresman-Addison Wesley Mathematics, Grade 4, 2008 (p. 628).

© Lopez Mateos Editores. ISBN 978-607-95583-2-1, obra completa, versión electrónica, ISBN 978-607-95583-3-8, volumen 1, versión electrónica. Ejemplar asignado a: Helecto Villarroel gutierrez -
helecto@gmail.com. Fecha: 27 de octubre de 2014. Prohibida su modificación, copia o distribución.
418 Decimales y números reales

Comparar 0.36 y 0.9 convirtiéndolos a “dinero” sugiere un método para comparar dos de-
cimales cualesquiera. Por ejemplo, considera los decimales 0.345 y 0.1474. Nota que 0.345
es un decimal con los milésimos como menor lugar posicional y que 0.1474 tiene los diezmi-
lésimos como su menor valor posicional. Para comparar los decimales, podemos escribir am-
bos con los diezmilésimos como menor valor posicional. Nota que 0.345 se puede escribir
como 0.3450. Ahora tenemos tanto 0.3450 como 0.1474 con los diezmilésimos como menor
valor posicional. Estos decimales se pueden escribir como sigue:
◆ Nota de 3450
10,000
y
1474
10,000
investigación Como en el capítulo 6, estas dos fracciones se pueden comparar simplemente viendo los numera-
La relación del valor 3450 1474
posicional entre nú- dores. Como 3450 es mayor que 1474, 7 , y por lo tanto 0.345 7 0.1474.
10,000 10,000
meros completos y Con frecuencia, este proceso se abrevia viendo los decimales alineados como sigue:
números decimales es
útil para el aprendi- 0.3450 = 0.345
zaje, pero los niños 0.1474 = 0.1474
suelen equivocarse al Y con los decimales alineados de esta manera, 3 décimos es mayor que 1 décimo. Así,
ajustarla para los as- 0.345 7 0.1474. Esto sugiere una manera de ordenar decimales sin convertirlos a fraccio-
pectos decimales. Un nes. Los pasos que usamos para comparar decimales exactos se parecen a los que usamos para
error común es que comparar números completos:
el estudiante asegure 1. Alinea los números según el valor posicional.
que 0.56 es mayor 2. Comienza por la izquierda y halla el primer lugar donde los valores nominales sean diferentes.
que 0.6 porque 56 es 3. Compara estos dígitos. El número que contenga el mayor valor nominal en este lugar
mayor que 6 es el mayor de los dos números.
(Hiebert 1992). ◆ Como dice Hiebert enla Nota de investigación, los estudiantes no siempre consideran los
decimales cuando tratan de ordenar números.
En la página de muestra (página 417) sobre el valor posicional decimal, vemos que incluso
en el grado 4 los estudiantes aprenden diferentes representaciones de los decimales. Re-
suelve el número 2 de la sección Tema de plática.

Evaluación 7-1 A

1. Escribe cada caso como una suma en forma expandida de 4. Escribe con palabras cada uno de los casos siguientes:
valor posicional (ver página 417): a. 0.34
a. 0.023 b. 20.34
b. 206.06 c. 2.034
c. 312.0103 d. 0.000034
d. 0.000132 5. Escribe cada uno de los siguientes decimales exactos en la
2. Reescribe lo siguiente como decimales: a
forma más simple donde a, b 僆 E y b Z 0.
a. 4 # 103 + 3 # 102 + 5 # 10 + 6 + 7 # 10 1 + 8 # 10 2
- -
b
b. 4 # 103 + 6 # 10 1 + 8 # 10 3
- -
a. 0.436 b. 25.16
c. 4 # 104 + 3 # 10 2 c. - 316.027
-
d. 28.1902
d. 2 # 10 + 4 # 10 4 + 7 # 10 7 e. - 4.3 f. - 62.01
-1 - -

3. Escribe cada caso como numeral: 6. Determina mentalmente cuáles de las siguientes expre-
a. Quinientos treinta y seis y setenta y seis diezmilésimos siones representan decimales exactos:
b. Tres y ocho milésimos 4 61
a. b. 2
c. Cuatrocientos treinta y seis millonésimos 5 2 #5
d. Cinco millones y dos décimos 3 1
c. d. 5
6 2

© Lopez Mateos Editores. ISBN 978-607-95583-2-1, obra completa, versión electrónica, ISBN 978-607-95583-3-8, volumen 1, versión electrónica. Ejemplar asignado a: Helecto Villarroel gutierrez -
helecto@gmail.com. Fecha: 27 de octubre de 2014. Prohibida su modificación, copia o distribución.
Sección 7-1 Introducción a los decimales 419

36 133 1
e. f. g.
55 625 3 13. Usa una malla con 100 cuadrados y representa 0.32. Ex-
2 1 26 plica tu representación.
h. i. j. 14. Si los decimales 0.804, 0.84 y 0.8399 se disponen en una
25 13 65
7. Donde sea posible, escribe cada uno de los números del recta numérica típica, ¿cuál queda más lejos a la derecha?
problema 6 como un decimal exacto. 15. Escribe un número decimal que tenga diezmilésimos y
8. Siete minutos es parte de una hora. Si 7 min se expresara esté entre 8.34 y 8.341.
como una parte decimal de una hora, explica si sería o no 16. a. Muestra que entre cualesquier dos decimales exactos,
un decimal exacto. hay otro decimal exacto.
9. Un segmento unitario tiene longitud 1. Si un segmento b. Argumenta que la parte (a) se puede usar para mos-
unitario tiene un extremo en 3, el otro en 4 sobre una rec- trar que hay infinidad de decimales exactos entre cua-
56 lesquier dos decimales exactos.
ta numérica, y consideramos la parte de ese segmento 17. a. Describe el decimal 0.613 usando cubos de base diez.
100
unitario, como en la figura 7-3, ¿qué decimal representaría b. Explica si puedes describir un decimal como 0.61345
56 usando cubos de base diez.
el extremo derecho del segmento de ? 18. El promedio de bateo de un jugador de beisbol se reportó
100
como “tres veintidós”. Un promedio de bateo se deter-
10. Dado un sistema monetario decimal, ¿qué razones crees
mina esencialmente cuando divides el número de veces
que hay para tener sólo monedas de uno, de cinco, de
que el jugador batea imparable entre el número de turnos
diez, de veinticinco y de cincuenta como partes de la
al bat. Explica por qué el promedio de bateo reportado no
unidad monetaria?
es matemáticamente correcto.
11. En cada uno de los casos siguientes, ordena los decima-
19. Si los “decimales” en otras bases numéricas funcionan de la
les de mayor a menor:
misma manera que en la base diez, explica el significado de
a. 13.4919, 13.492, 13.49183, 13.49199
la siguiente expresión: 3.145seis.
b. - 1.453, - 1.45, - 1.4053, - 1.493
20. Las cinco mejores nadadoras en un evento tuvieron los
12. Escribe los números en cada una de las siguientes ora-
tiempos siguientes:
ciones como decimales en símbolos:
Emily 64.54 s Kathy 64.02 s
a. El cuerpo de un ácaro mide alrededor de catorce mi-
Molly 64.46 s Rhonda 63.54 s
lésimos de pulgada.
Martha 63.59 s
b. La Tierra gira alrededor del Sol una vez cada trescien-
Lístalas en orden, comenzando por el “mejor” tiempo.
tos sesenta y cinco días y veinticuatro centésimas.

Evaluación 7-1 B

1. Escribe cada caso como una suma en forma expandida de 5. Escribe cada uno de los siguientes decimales exactos en la
valor posicional (ver página 417): a
forma más simple donde a, b 僆 E y b Z 0.
a. 0.045 b. 103.03 b
c. 245.6701 d. 0.00034 a. 28.32 b. 34.1736 c. - 27.32
2. Reescribe lo siguiente como decimales: 6. Determina mentalmente cuáles de las siguientes expre-
a. 5 # 103 + 2 # 102 + 4 # 10 1
-
siones representan decimales exactos:
# -3
b. 4 10 + 2 10 # 4
a.
4
b. 6
1
c.
137
c. 2 # 102 + 3 # 104 8 2 625
3. Escribe cada caso como numeral: 1 3
d. e.
a. Dos mil veintisiete milésimos 17 25
b. Dos mil y veintisiete milésimos 7. Donde sea posible, escribe cada uno de los números del
c. Dos mil veinte y siete milésimos problema 6 como un decimal exacto.
d. Cuatrocientos milésimos 8. ¿Qué número completo de minutos (menor que 60) podría
4. Escribe con palabras cada uno de los casos siguientes: expresarse como parte decimal exacta de una hora?
a. 0.45 9. Si de un conjunto de cubos de base diez un bloque re-
b. 2.035 1
c. 45.0006 presenta , ¿cuál es el valor de lo siguiente?
10
d. 0.0000445 a. 1 cubo
b. 1 losa

© Lopez Mateos Editores. ISBN 978-607-95583-2-1, obra completa, versión electrónica, ISBN 978-607-95583-3-8, volumen 1, versión electrónica. Ejemplar asignado a: Helecto Villarroel gutierrez -
helecto@gmail.com. Fecha: 27 de octubre de 2014. Prohibida su modificación, copia o distribución.
420 Decimales y números reales

c. 1 barra 15. Usa los resultados de los ejercicios 13 y 14 para describir


d. 3 bloques, 1 barra y 4 cubos cómo argumentarías que hay infinidad de decimales
10. En cada uno de los casos siguientes, ordena los decima- exactos entre cualesquier dos decimales exactos específi-
les de menor a mayor: cos, como 0.0625 y 0.125.
a. 24.9419, 24.942, 24.94189, 24.94199 16. Explica el significado matemático de un letrero en una
b. - 34.25, - 34.251, - 34.205, - 34.2519 máquina fotocopiadora que dice “.05¢ la copia”.
11. Usa una malla con 100 cuadrados y representa 0.23. 17. Si los “decimales” en otras bases numéricas funcionan
12. Escribe un decimal que esté entre 8.345 y 8.3456. igual que en la base diez, explica el significado de
13. La regla de los carpinteros está marcada en dieciseisavos. 0.00334siete.
¿Cuál sería el valor de cada marca si estuviera en forma 18. Si un nadador reduce su tiempo en 0.2 s en 50 m de nado
decimal? de dorso cada día durante los 10 primeros días de práctica,
14. Supón que la regla del ejercicio 13 se marcó en treintay- ¿cuánto tiempo ha reducido?
dosavos; ¿cuál sería el decimal que marque la regla entre
1 2
y ?
16 16

Conexiones matemáticas 7-1

Comunicación Aprendizaje colectivo


1. Usando la notación de Simon Stevin, di cómo escribirías 11. Con base en un sistema similar al de base diez, determi-
lo siguiente: nen cómo pueden introducir una notación “decimal” en
a. 0.3256 b. 0.0032 base cinco.
2. Si 1 mL es 0.0001 L, ¿en qué forma se expresan 18 mL 12. A Simon Stevin se le reconoce la difusión del sistema deci-
como un número decimal exacto de litros? mal. Divididos en pequeños grupos, investiguen la historia
3. Explica si un día se puede expresar como una parte deci- de los decimales en busca de contribuciones de los matemá-
mal exacta de un año. ticos árabes, chinos y del Renacimiento. Expliquen si creen
4. Usa el modelo de la recta numérica para describir decima- o no que Stevin “inventó” el sistema decimal.
les exactos y explica si crees que exista un mayor decimal
exacto menor que 1. Preguntas del salón de clase
5. Busca el significado de diezmo. ¿Cómo está relacionado 13. Un estudiante asegura que 0.36 es mayor que 0.9 porque
con los decimales? 36 es mayor que 9. ¿Cómo le respondes?.
6. Explica cómo usarías cubos de base diez para representar 14. Una estudiante asegura que las fracciones ya no deberían
dos y trescientos cuarenta y cinco milésimos. enseñarse una vez que los estudiantes aprenden a trabajar
7. Explica por qué en el teorema 7–1 el número racional con números decimales. ¿Qué le respondes?
debe estar en su forma más simple antes de examinar el
denominador. Preguntas del National Assessment of Educational
8. Explica con tus propias palabras qué es y qué hace el Progress (NAEP) (Evaluación Nacional del Progreso
punto decimal. Educativo)
¿Cuál es 4 centésimos escrito en notación decimal?
Solución abierta a. 0.004
9. Determina cómo se simboliza la notación decimal en di- b. 0.04
versos países. c. 0.400
10. Examina tres libros de texto de la escuela elemental e in- d. 4.00
forma cómo difieren, si lo hacen, las introducciones a los NAEP, 2007, Grado 8
temas de exponentes y decimales. Informa sobre el nivel
en que se introducen los decimales.

ROMPECABEZAS Halla dos “decimales” en base dos que se puedan representar como decimales
exactos en base diez. Argumenta que cualquier decimal exacto en base dos se puede representar como
un decimal exacto en base diez.

© Lopez Mateos Editores. ISBN 978-607-95583-2-1, obra completa, versión electrónica, ISBN 978-607-95583-3-8, volumen 1, versión electrónica. Ejemplar asignado a: Helecto Villarroel gutierrez -
helecto@gmail.com. Fecha: 27 de octubre de 2014. Prohibida su modificación, copia o distribución.
Sección 7-2 Operaciones con decimales 421

7-2 Operaciones con decimales

A fin de desarrollar un algoritmo para la suma de decimales exactos, considera la suma


2.16 + 1.73. En la escuela elemental se recomiendan cubos de base diez para ilustrar dicho
problema de suma. En la figura 7-4 se muestra cómo se puede efectuar la suma.

1
— —6
2 10 100 5 2.16

7
— —3
1 1 10 100 5 11.73

—8 —9
3 10 100 5 3.89

Figura 7-4
El cálculo en la figura 7-4 se puede explicar mediante el procedimiento cambiarlo por un
problema que ya sabemos resolver, esto es, por una suma de fracciones. Entonces usamos las
propiedades conmutativa y asociativa de la suma como ayuda en el cálculo, como sigue:

2.16 + 1.73 = a 2 + b + a1 + b
1 6 7 3
+ +
10 100 10 100

= 12 + 12 + a b + a b
1 7 6 3
+ +
10 10 100 100
8 9
= 3 + +
10 100
= 3.89
Una manera diferente de ver la suma 2.16 + 1.73 nos relaciona con una representación
diferente de los decimales como fracciones con denominadores que son potencias de 10,
como sigue:

216 173 216 173 216 + 173 389


2.16 = y 1.73 = así que 2.16 + 1.73 = + = = = 3.89.
100 100 100 100 100 100
21 173 210 173 383
De manera análoga, 2.1 + 1.73 = + = + = = 3.83.
10 100 100 100 100
© Lopez Mateos Editores. ISBN 978-607-95583-2-1, obra completa, versión electrónica, ISBN 978-607-95583-3-8, volumen 1, versión electrónica. Ejemplar asignado a: Helecto Villarroel gutierrez -
helecto@gmail.com. Fecha: 27 de octubre de 2014. Prohibida su modificación, copia o distribución.
422 Decimales y números reales

En cada caso, los decimales se convierten a fracciones con el mismo denominador, que es una
potencia de 10. Una vez hecho eso, los numeradores se pueden sumar como números comple-
tos y obtener el numerador de la fracción solución, que puede convertirse de nuevo en un deci-
mal. Si estos decimales se escriben como se muestra abajo, lo que hacemos, esencialmente, es
“alinear los puntos decimales” y sumar como se hace con los números completos, colocando el
punto decimal en el lugar apropiado de la respuesta.
2.16 2.1
+ 1.73 y + 1.73
3.89 3.83
El segundo cálculo se puede escribir como se muestra aquí:
2.10
+ 1.73
3.83
En los cálculos anteriores, sumamos unidades con unidades, décimas con décimas y centési-
mas con centésimas. Esto puede lograrse de manera más eficiente manteniendo los números en
su forma decimal, alineando el punto decimal y sumando como si los números fueran núme-
ros completos. Esta técnica funciona para la suma y la resta, como se ilustra en la página de
muestra (página 423). Nota, además, el uso de calculadora en la suma decimal, ejemplo C.

Multiplicación de decimales
Así como en los algoritmos expuestos para sumar decimales exactos representándolos como
fracciones, desarrollamos y explicamos un algoritmo para la multiplicación de decimales.
Considera el producto 4.62 # 2.4:
462 # 24
14.62212.42 =
462 # 24 462 # 24 11,088
= = = = 11.088
100 10 2
10 10 1 2 #
10 10 1
103
La respuesta a este cálculo se obtuvo multiplicando los números completos 462 y 24 y des-
pués dividiendo el resultado entre 103.
En la página de muestra (página 424), observa cómo se puede lograr la multiplicación de un nú-
mero completo por un decimal, primero sombreando 0.6 tres veces, en salmón, verde y morado.
Después 0.04 se sombrea tres veces en colores similares, como se ve en la parte derecha de la Ac-
tividad. El producto es la suma de todas las partes sombreadas: 0.6 + 0.6 + 0.6 + 0.04 +
0.04 + 0.04 ó 1.92. De esta manera la multiplicación de un número completo por un decimal
se interpreta casi como la multiplicación de un número completo por un número completo.
Considera el método del sombreado en la Actividad al final de la página para ilustrar por qué
0.5 * 0.7 es 0.35. Intenta resolver por tí mismo la parte (d). ¿Crees que este método servirá
para multiplicar números con más lugares decimales?
El algoritmo para multiplicar decimales se puede enunciar como sigue:

Si hay n dígitos a la derecha del punto decimal en un número y m dígitos a la derecha del punto de-
cimal en un segundo número, multiplica los dos números ignorando los decimales y luego coloca el
punto decimal de modo que haya n + m dígitos a la derecha del punto decimal en el producto.

O B S E R VA C I Ó N Hay n + m dígitos a la derecha del punto decimal en el producto


porque 10n # 10m = 10n + m.

© Lopez Mateos Editores. ISBN 978-607-95583-2-1, obra completa, versión electrónica, ISBN 978-607-95583-3-8, volumen 1, versión electrónica. Ejemplar asignado a: Helecto Villarroel gutierrez -
helecto@gmail.com. Fecha: 27 de octubre de 2014. Prohibida su modificación, copia o distribución.
Sección 7-2 Operaciones con decimales 423

Página de un libro de texto S U M A Y R ESTA D E N Ú M E ROS


COM P L E TOS Y D E C I M A L ES

Lección 2-5

Suma y resta de
Idea clave Calentamiento
Sumar y restar Estima:

números completos y
decimales es
similar a sumar

decimales
y restar
números completos.

Aprende
¿Cómo puedes sumar decimales?
Cuando se da el disparo de salida, los corredores de relevos
pueden perder 0.2 segundos en reaccionar y otros 1.8 segundos
en pasar del reposo inicial a la velocidad de carrera. ¿Alrededor de
cuántos segundos pasan antes de alcanzar la velocidad de carrera?

Ejemplo A
ex
ef l i ó Halla 0.2+1.8.
Lo que piensas Lo que escribes
Estimado: 0+2=2
R

PASO Escribe los


¡Piensa! números, alineando
Recuerdo que los puntos decimales.
añadir ceros no
cambia el valor
del decimal.
PASO Suma las décimas.
Reagrupa si es necesario.
Escribe el punto decimal
en tu respuesta.

La respuesta y el estimado coinciden, de modo que la respuesta


es razonable.

Ejemplo B Ejemplo C
Halla 5.6+2.973. Usa una calculadora para hallar
Estimado: 6+3=9
Escribe los números Estimado:
alineando los puntos
decimales. Suma las Teclea:
milésimas, centésimas y
décimas. Reagrupa si es Pantalla:
necesario. Escribe el punto
decimal en tu respuesta.
Como 855.39 está cerca de
Como 8.573 está cerca de 9, 800, la respuesta en pantalla
la respuesta es razonable. es razonable.

Fuente: Scott Foresman-Addison Wesley Mathematics, Grade 6, 2008 ( p. 86).

© Lopez Mateos Editores. ISBN 978-607-95583-2-1, obra completa, versión electrónica, ISBN 978-607-95583-3-8, volumen 1, versión electrónica. Ejemplar asignado a: Helecto Villarroel gutierrez -
helecto@gmail.com. Fecha: 27 de octubre de 2014. Prohibida su modificación, copia o distribución.
424 Decimales y números reales

Página de un libro de texto MULTIPLICACIÓN DE NÚMEROS


COMPLETOS Y DECIMALES

Lección 2-6
Calentamiento
Idea clave
Multiplicar Multiplicación de números Estima.
decimales es similar
a multiplicar enteros y decimales
números
completos. Sólo
necesitas saber
Aprende
dónde colocar el
punto decimal en el ¿Cómo puedes multiplicar un número
producto.
completo por un decimal?
Materiales Para multiplicar decimales puedes usar los mismos
cuadrícula decimal métodos que para multiplicar números completos.
de 10 por 10 ó

lápices o
Actividad
marcadores
de colores Para hallar 3 × 0.64,
sombrea las décimas del
ex
ef l i ó número decimal en una
R

cuadrícula. Hazlo tres


veces, usando cada vez
¡Piensa! un color diferente.
Puedo usar modelos para
ilustrar la multiplicación de Usando los mismos tres
decimales. colores, sombrea tres
grupos de las centésimas centésimas
del número decimal.
Cuenta todas la centésimas
sombreadas.
Usa cuadrículas para hallar 1.2 × 2 y 2 × 0.18.

¿Cómo puedes multiplicar un decimal por un decimal?

Actividad
Para hallar 0.5 × 0.7, sombrea
5 columnas usando el mismo
color para ilustrar 5 décimas.
Sombrea 7 renglones con otro
color para ilustrar 7 décimas.
Cuenta las centésimas en el área
centésimas
sombreada traslapada.
Usa cuadrículas para hallar 0.4 ×0.7 y 1.6 × 0.3.

Fuente: Scott Foresman-Addison Wesley Mathematics, Grade 6, 2008 ( p. 90).

© Lopez Mateos Editores. ISBN 978-607-95583-2-1, obra completa, versión electrónica, ISBN 978-607-95583-3-8, volumen 1, versión electrónica. Ejemplar asignado a: Helecto Villarroel gutierrez -
helecto@gmail.com. Fecha: 27 de octubre de 2014. Prohibida su modificación, copia o distribución.
Sección 7-2 Operaciones con decimales 425

Ejemplo 7-4 Calcula lo siguiente:


a. 16.2211.432 b. 10.02210.0132 c. 11000213.62









Solución a. 1.4 3 (2 dígitos después del punto decimal)
* 6.2 (1 dígito después del punto decimal)
286
858
8.8 6 6 (2 + 1, ó 3 dígitos después del punto decimal)
b. 0.0 1 3 c. 3.6
* 0.0 2 * 1000
0.0 0 0 2 6 3 6 0 0.0

AHORA INTENTA ÉSTE 7-4 En el ejemplo 7-4(c) sugiere que la multiplicación por 1000, ó 103, se rea-
liza moviendo el punto decimal tres lugares a la derecha. (a) Explica por qué esto es correcto usando la
notación expandida y la propiedad distributiva de la multiplicación sobre la suma. (b) En general, ¿cómo
afecta al producto la multiplicación por 10n cuando n es un entero positivo? ¿Por qué?

Notación científica
3 5
Muchas calculadoras presentan en pantalla las fracciones y como 6.5661319 -05
45,689 76,146
y 6.566333 -05 , respectivamente. La pantalla está en notación científica. La primera es una
notación para 6.5661319 # 10 5 y la segunda para 6.566333 # 10 5.
- -

Los científicos usan la notación científica para manejar números muy pequeños o muy
grandes. Por ejemplo, “el Sol está a 150,000,000 km de la Tierra” se expresa como “el Sol
está a 1.5 # 108 km de la Tierra”. Un micrón, unidad métrica de medida equivalente a
0.000001 m, se escribe 1 # 10 6 m.
-

Definición de notación científica


En notación científica, un número positivo se escribe como el producto de un número mayor o
igual que 1 y menor que 10 y una potencia entera de 10. Para escribir un número negativo en
notación científica, trata el número como positivo y añade el signo negativo al resultado.

Los números siguientes están en notación científica:


8.3 # 108, 1.2 # 1010, - 73.2 = - 17.32 # 1012 y 7.84 # 10 6
-

Los números 0.43 # 109 y 12.3 # 10 6 no están en notación científica pues 0.43 y 12.3 no son
-

mayores o iguales que 1 y menores que 10. Para escribir en notación científica un número
como 934.5, dividimos entre 102 para obtener 9.345 y luego multiplicamos por 102 para
retener el valor del número original:
934.5 = a b102 = 9.345 # 102
934.5
102

© Lopez Mateos Editores. ISBN 978-607-95583-2-1, obra completa, versión electrónica, ISBN 978-607-95583-3-8, volumen 1, versión electrónica. Ejemplar asignado a: Helecto Villarroel gutierrez -
helecto@gmail.com. Fecha: 27 de octubre de 2014. Prohibida su modificación, copia o distribución.
426 Decimales y números reales

Esto equivale a mover el punto decimal dos lugares a la izquierda (dividiendo entre 102) y
después multiplicar por 102. De manera análoga, para escribir 0.000078 en notación cien-
tífica primero multiplicamos por 105 para obtener 7.8 y después dividimos entre 105 (o
-
multiplicamos por 10 5) para mantener el valor original:
0.000078 = 10.000078 # 105210 5 = 7.8 # 10 5
- -

-
Esto equivale a mover el punto decimal cinco lugares a la derecha y multiplicar por 10 5.

Ejemplo 7-5 Escribe cada uno de los casos siguientes en notación científica:








a. 413,682,000 b. 0.0000231 c. 83.7 d. - 10,000,000

Solución a. 413,682,000 = a b108 = 4.13682 # 108


413,682,000
108
b. 0.0000231 = 10.0000231 # 105210 5 = 2.31 # 10 5
- -

c. 83.7 = a 1 b 101 = 8.37 # 101


83.7
10
d. - 10,000,000 = a
- 10,000,000
b107 = - 11 # 1072
107

Ejemplo 7-6 Convierte cada caso a numerales comunes:


c. - 14.08 # 1042








a. 6.84 # 10 b. 3.12 # 107


-5

b = 0.0000684
1
Solución a. 6.84 # 10
-5
= 6.84a
105
b. 3.12 # 107 = 31,200,000
c. - 14.08 # 1042 = - 40,800

Los números en notación científica son fáciles de manipular usando las leyes de los expo-
nentes. Por ejemplo, 15.6 # 105216 # 1042 puede reescribirse como 15.6 # 621105 # 1042 =
33.6 # 109, que es 3.36 # 1010 en notación científica. También,

12.35 # 10 212 # 1082 = 12.35 # 22110 # 1082 = 4.7 # 10


-15 -15 -7

Las calculadoras con tecla EE se pueden usar para representar números en notación
científica. Por ejemplo, para hallar 15.2 # 10162 19.37 # 1042 tecleamos
5 . 2 EE 1 6 * 9 . 3 7 EE 4 =

División de decimales
Bajo la división, algunos conjuntos son cerrados y otros no. Por ejemplo, el conjunto de los
números completos no era cerrado bajo la división. Considera que 1 , 3 no produce un
número completo pues no existe un número completo que al multiplicarlo por 3 dé 1.
Usando el mismo contraejemplo, vemos que el conjunto de los números enteros tampoco
es cerrado bajo la división. Sin embargo, el conjunto de los números racionales, sin el 0, es
a c ad
cerrado bajo la división. Recuerda que , , donde b, c y d Z 0 da , un número racional.
b d bc

© Lopez Mateos Editores. ISBN 978-607-95583-2-1, obra completa, versión electrónica, ISBN 978-607-95583-3-8, volumen 1, versión electrónica. Ejemplar asignado a: Helecto Villarroel gutierrez -
helecto@gmail.com. Fecha: 27 de octubre de 2014. Prohibida su modificación, copia o distribución.
Sección 7-2 Operaciones con decimales 427

¿Debemos esperar que el conjunto de los decimales, sin el 0, sea cerrado bajo la división?
La respuesta para el no iniciado no es automática. Hemos mostrado que algunas fracciones
2
se pueden representar por medio de decimales exactos. Vimos anteriormente que
11
no se puede representar como un decimal exacto. Que el conjunto de los números raciona-
les, sin el 0, sea cerrado bajo la división no implica que el conjunto de los decimales, sin el 0,
sea cerrado bajo la división.
Sin embargo, como en el conjunto de los números completos sin el 0, que no es cerrado
bajo la división, hay algoritmos para dividir decimales que permiten residuo. Algunas divi-
siones dejan residuo 0, como se ve a continuación. Considera 75.45 , 3:
7545 3 7545 # 1 7545 # 1 1
75.45 , 3 = , = = = 2515 # = 25.15
100 1 100 3 3 100 100
Al escribir el dividendo como fracción cuyo denominador es una potencia de 10 y usando la
propiedad conmutativa de la multiplicación, la división se cambia a una división de números
completos 17545 , 32 por una potencia de 10; esto es,
1 -
, ó 10 2. Un método diferente
100
de lograr la transformación de la división en una división entre números completos es mul-
tiplicar el dividendo y el divisor por la misma potencia de 10, como sigue:
75.45 75.45 # 102 7545 7545 # 1
75.45 , 3 = = = =
3 3 # 102 3 # 102 3 102
¿Por qué funciona esto?
El proceso mostrado aquí se ve en la actividad ubicada al final de la página de muestra (pá-
gina 428). Completa la actividad.
Cuando el divisor es un número completo, la división se puede manejar como con los nú-
meros completos y el punto decimal se coloca directamente sobre el punto decimal del di-
videndo. Cuando el divisor no es un número completo, como en 1.2032 , 0.32, podemos
transformarlo para obtener un número completo expresando el cociente como una fracción
y después multiplicando el numerador y el denominador de la fracción por una potencia de
10. Esto corresponde a reescribir el problema de la división en la forma (a) como un pro-
blema equivalente en la forma (b), como sigue:
1a2 0.32 冄 1.2032 1b2 32 冄 120.32
En los libros de texto de la escuela básica, este proceso suele describirse como que el punto
decimal “se mueve” dos lugares a la derecha en el dividendo y el divisor. Este proceso se in-
dica usualmente con flechas, como se muestra a continuación.
3.7 6
0.3 2 冄 1.20 3 2 Multiplica el divisor y el dividendo por 100.
96
24 3
2 24
192
19 2
0

© Lopez Mateos Editores. ISBN 978-607-95583-2-1, obra completa, versión electrónica, ISBN 978-607-95583-3-8, volumen 1, versión electrónica. Ejemplar asignado a: Helecto Villarroel gutierrez -
helecto@gmail.com. Fecha: 27 de octubre de 2014. Prohibida su modificación, copia o distribución.
428 Decimales y números reales

Página de un libro de texto DIVISIÓN ENTRE UN DECIMAL

Lección 2-9

División entre un decimal


Calentamiento
Idea clave
Para dividir entre un Halla cada cociente.
decimal, necesitas
transformar el Aprende
divisor en un
número completo.
¿Cómo puedes dividir entre un decimal?
Materiales Nilda y sus amigas dividieron el costo de una
• Modelos decimales bolsa de nueces de $2.70. Cada persona contribuyó con
o $0.90. ¿Cuántas personas compartieron las nueces?

ex Actividad
ef l i ó Puedes usar los mismos métodos para dividir decimales que los
R

usados para dividir números


completos.
¡Piensa! Halla 2.7 ÷ 0.9.
Puedo usar modelos
para ilustrar la división Engloba grupos de 0.9.
entre decimales.
Cuenta los grupos englobados.
Usa modelos decimales para
hallar 3.0 ÷ 0.3 y 2.5 ÷ 0.5.

dividendo divisor cociente

¿Cómo puedes transformar un divisor decimal en un divisor


que sea un número completo sin modificar el cociente?

Actividad
Estudia los patrones. Compara la primera y la segunda columnas.
Patrón 1 Patrón 2

Para el patrón 1, ¿qué sucede al cociente cuando multiplicas el


dividendo y el divisor por 10?
¿Por qué números se multiplicaron el dividendo y el divisor en
el patrón 2?
Completa la tabla de Multiplico el dividendo
Dividendo y el divisor por: Cociente
la derecha.

Fuente: Scott Foresman-Addison Wesley Mathematics, Grade 6, 2008 ( p. 100).

© Lopez Mateos Editores. ISBN 978-607-95583-2-1, obra completa, versión electrónica, ISBN 978-607-95583-3-8, volumen 1, versión electrónica. Ejemplar asignado a: Helecto Villarroel gutierrez -
helecto@gmail.com. Fecha: 27 de octubre de 2014. Prohibida su modificación, copia o distribución.
Sección 7-2 Operaciones con decimales 429

Ejemplo 7-7 Calcula lo siguiente:











a. 13.169 , 0.13 b. 9 , 0.75

1 01.3 12
Solución a. 0.1 3 冄 1 3.1 6 9 b. 0.7 5 冄 9.0 0
13 75
16 1 50
13 15 0
39 0
39
0

9 9 # 100 900
En el ejemplo 7-7(b), anexamos dos ceros al dividendo pues = = .
0.75 0.75 # 100 75

Ejemplo 7-8 El propietario de una estación de gasolina debe retener un impuesto de $0.11 por cada galón
de gasolina vendido. Una semana el propietario pagó $1595 en impuestos de gasolina. El








precio de un galón de gasolina despachado durante esa semana fue de $3.35 sin impuestos.

a. ¿Cuántos galones de gasolina vendió esa semana?


b. ¿Cuáles fueron los ingresos de esa semana después de pagar los impuestos?

Solución a. Para hallar el número de galones de gasolina vendidos durante esa semana,
debemos dividir el total de impuestos de gasolina entre la cantidad del im-
puesto por galón:
1595
= 14,500
0.11
Así, se vendieron 14,500 galones.
b. Para obtener el ingreso después de pagado el impuesto, primero determi-
namos el ingreso antes del pago de impuestos. Es decir multiplicamos el
número de galones vendidos por el costo por galón:
114,50021$3.352 = $48,575
A continuación, restamos los impuestos de gasolina del ingreso total:
$48,575 - $1,595 = $46,980
Así, el ingreso después de impuestos es de $46,980.

AHORA INTENTA ÉSTE 7-5 Usa la división decimal para ayudar a Hi, de la tira cómica “Hi y Lois”, a determi-
nar cuál es la mejor compra.

¿CUÁL ES MEJOR COMPRA: MI ESPOSA TAMBIÉN


32 ONZAS POR $1.19, Ó NO SÉ
EMPEZÓ A TRABAJAR
48 ONZAS POR HOY
$1.43?

© Lopez Mateos Editores. ISBN 978-607-95583-2-1, obra completa, versión electrónica, ISBN 978-607-95583-3-8, volumen 1, versión electrónica. Ejemplar asignado a: Helecto Villarroel gutierrez -
helecto@gmail.com. Fecha: 27 de octubre de 2014. Prohibida su modificación, copia o distribución.
430 Decimales y números reales

Ahora considera una división como la mostrada a continuación:


a. 3.6 4 Cociente b. 3 6 4 Cociente
0.3 3 冄 1.20 3 2 3 3冄 1 2 0 3 2
99 99
21 3 213
19 8 198
152 152
132 132
20 Residuo 2 0 Residuo

Nota que en la parte (a) se usan todos los dígitos en el dividendo y que 20 aparece al final
de la división. Esta división se podría tratar como la de la parte (b), donde el cociente es 364
y el residuo es 20. En la parte (a), el cociente es 3.64 y el residuo mostrado como 20 en rea-
lidad representa 0.0020, como se puede ver por el alineamiento decimal original. Esto se
puede verificar haciendo las cuentas: 3.64 * 0.33 + 0.0020 = 1.2032.

Cálculo mental
Algunas de las herramientas usadas para los cálculos mentales con números completos se
pueden usar para efectuar cálculos mentales con decimales, como se ve a continuación:
1. Separar y juntar
1.5 + 3.7 + 4.48 1.5 + 3

= 4.5 + 0.7 + 4.48 4.5 + 0.7

= 5.2 + 4.48 5.2 + 4

= 9.2 + 0.48 = 9.68 9.2 + 0.48


2. Usar números compatibles
(Los números decimales son compatibles cuando suman un entero.)
7.91 12 7.91 + 4.09
3.85
4.09 + 4 3.85 + 0.15
+ 0.15
16 12 + 4
3. Construir números compatibles
9.27 = 9.25 + 0.02
+ 3.79 = 3.75 + 0.04
13.00 + 0.06 = 13.06
4. Balancear con decimales en la resta
4.63 = 4.63 + 0.03 = 4.66
- 1.97 = - 11.97 + 0.032 = - 2.00
2.66

© Lopez Mateos Editores. ISBN 978-607-95583-2-1, obra completa, versión electrónica, ISBN 978-607-95583-3-8, volumen 1, versión electrónica. Ejemplar asignado a: Helecto Villarroel gutierrez -
helecto@gmail.com. Fecha: 27 de octubre de 2014. Prohibida su modificación, copia o distribución.
Sección 7-2 Operaciones con decimales 431

5. Balancear con decimales en la división


0.25 冄 8

* 4 * 4
32
1 冄 32

O B S E R VA C I Ó N Para balancear con decimales en la división, se usa la propiedad


a a#c
= # si c Z 0.
b b c

Redondeo de decimales
Con frecuencia no es indispensable conocer la respuesta numérica exacta a una pregunta.
Por ejemplo, si queremos saber la distancia a la luna o la población de la zona conurbada de
la ciudad de México, las respuestas aproximadas de 385,000 km y 20,000,000 de habitantes,
respectivamente, pueden ser adecuadas.
A menudo la misma situación indica la manera en que debemos redondear. Por ejemplo,
supón que se realiza una compra por $38.65 y el cajero usa una calculadora para obtener el
impuesto de venta del 6% multiplicando 0.06 # 38.65. La pantalla muestra 2.319. Como di-
cho número está entre 2.31 y 2.32, y está más cerca de 2.32, el cajero redondea el impuesto
hacia arriba, a $2.32.
Supón que en un informe se necesita incluir un resultado de 8.7345649 hasta las centési-
mas. El resultado se ubica entre 8.73 y 8.74 pero está más cerca de 8.73, de modo que lo re-
dondeamos hacia abajo, a 8.73. A continuación, supón que el número 6.8675 necesita
redondearse a la milésima más cercana. Nota que 6.8675 está exactamente a la mitad entre
6.867 y 6.868. En tales casos la práctica común es redondear hacia arriba; la respuesta a la milé-
sima más cercana es 6.868. Escribimos esto como 6.8675 L 6.868 y decimos que 6.8675 es
aproximadamente igual a 6.868.
Ejemplo 7-9 Redondea cada uno de los números siguientes:








a. 7.456 a la centésima más cercana


b. 7.456 a la décima más cercana
c. 7.456 a la unidad más cercana
d. 7456 al millar más cercano
e. 745 a la decena más cercana
f. 74.56 a la decena más cercana

Solución a. 7.456 L 7.46


b. 7.456 L 7.5
c. 7.456 L 7
d. 7456 L 7000
e. 745 L 750
f. 74.56 L 70

También se puede redondear en algunas calculadoras usando la tecla FIX . Si quieres re-
dondear el número 2.3669 a milésimas, teclea FIX 3 . La pantalla mostrará 0.000. Si luego
colocas 2.3669 y presionas la tecla = , la pantalla mostrará 2.367.

© Lopez Mateos Editores. ISBN 978-607-95583-2-1, obra completa, versión electrónica, ISBN 978-607-95583-3-8, volumen 1, versión electrónica. Ejemplar asignado a: Helecto Villarroel gutierrez -
helecto@gmail.com. Fecha: 27 de octubre de 2014. Prohibida su modificación, copia o distribución.
432 Decimales y números reales

Estimación de cálculos con decimales mediante redondeo


Los números redondeados pueden ser útiles para estimar respuestas de cálculos. Por ejem-
plo, considera cada uno de los casos siguientes:
1. Biani va a la tienda de abarrotes a comprar artículos que cuestan las cantidades si-
guientes. Ella estima el costo total redondeando cada cantidad al peso más cercano y
sumando los números redondeados.
$2.39 : $2
0.89 : 1
6.13 : 6
4.75 : 5
+ 5.05 : 5
$19
Así, Biani estima que su cuenta es de $19.
2. La cuenta de la reparación del auto de Biani fue de $72.80 y ella tenía un cupón de
descuento de $17.50. Biani pudo estimar el costo total redondeando cada cantidad a
la decena de pesos más cercana y restando.
$72.80 : $70
- 17.50 : - 20
$50
Así, un estimado para la cuenta de la reparación es $50.
3. Biani ve la luz de un rayo y escucha el trueno 3.2 segundos más tarde. Ella sabe que
el sonido viaja a 0.33 km>s Ella puede estimar la distancia a la que está del rayo re-
dondeando el tiempo a la unidad más cercana y la velocidad a la décima más cercana,
y multiplicando.
0.33 : 0.3
* 3.2 : * 3
0.9
Así, Biani estima que está aproximadamente a 0.9 km del rayo.
1
Un enfoque alternativo es reconocer que 0.33 L y 3.2 está cerca de 3.3, de
3
modo que una aproximación usando números compatibles es a b3.3, ó 1.1 km.
1
3
4. Biani quiere estimar el número de millas que le rinde un galón de gas. Si ha recorrido
298 mi y ha gastado 12.4 gal para llenar el tanque, ella redondea y divide como sigue:
25
12.4 冄 298 : 12 冄 300
Así, obtiene alrededor de 25 mi>gal.
5. Juanita quiere colocar el punto decimal en el producto resultante cuando compró
21.45 lb de semillas de ajonjolí para molerlas y hacer pasta a $3.40 la libra. El resul-
tado de la multiplicación (sin decimal) es de 7293. Ella sabe que se puede obtener
un estimado redondeando 21.45 a 20 y $3.40 a $3. El producto debe estar cerca de
20 * 3, ó 60. Así que su elección para colocar el punto decimal fue $72.93.
Cuando los cálculos se efectúan con números redondeados, los resultados pueden ser
muy diferentes de la respuesta real.

© Lopez Mateos Editores. ISBN 978-607-95583-2-1, obra completa, versión electrónica, ISBN 978-607-95583-3-8, volumen 1, versión electrónica. Ejemplar asignado a: Helecto Villarroel gutierrez -
helecto@gmail.com. Fecha: 27 de octubre de 2014. Prohibida su modificación, copia o distribución.
Sección 7-2 Operaciones con decimales 433

AHORA INTENTA ÉSTE 7-6 Otras estrategias de estimación, como: desde la izquierda, cúmulos y com-
pletar números “agradables”, que estudiaste con los números completos, también funcionan con decimales.
Toma el ejemplo de la tienda de abarrotes de la parte 1 de la página anterior y usa la estrategia “desde la
izquierda con ajuste” para estimar la cuenta.

Errores de redondeo
Los errores de redondeo se acumulan cuando hay cálculos incluidos. Por ejemplo, si dos dis-
tancias, redondeadas a la décima más cercana, son 42.6 km y 22.4 km, entonces la suma de las
distancias aparecerá como 42.6 + 22.4, ó 65 km. Las distancias se habrían comunicado de
manera más precisa si se hubieran redondeado a la centésima más cercana, como 42.55 km y
22.35 km, respectivamente. La suma de estas distancias sería 64.9. De manera alternativa, las
distancias presentadas originalmente podrían haberse redondeado de 42.64 km y 22.44 km. La
suma ahora es 65.08, ó 65.1 redondeado a la décima más cercana. La suma original de 65 km
está entre 64.9 km y 65.1 km, pero el simple proceso de redondeo utilizado no es preciso.
Errores similares surgen en otras operaciones aritméticas. Cuando se realizan cálculos con
números aproximados, el resultado final no se debe comunicar empleando más dígitos significativos
que el número usado con menos dígitos significativos. Los dígitos distintos de cero siempre son
significativos. Los ceros antes de otros dígitos no son significativos. Los ceros entre dígitos
diferentes de cero son significativos. Los ceros a la derecha del punto decimal son significa-
tivos. Para evitar incertidumbre, ceros antes de un número son significativos sólo si están a
la derecha de un punto decimal.

AHORA INTENTA ÉSTE 7-7 En la siguiente tira cómica “Hi y Lois”, responde lo siguiente:
a. Estima si el cálculo de Trixie es preciso.
b. A la centésima de millón más cercana, ¿cuántas veces es Trixie mayor que su hermano gemelo?

VEAMOS...AMBOS ¡CINCO ¡SOY


TIENEN 7 MINUTOS VECES MAYOR QUE TÚ!
AÑOS, ¡QUE ES
SON 3,679,200 NADA!
MINUTOS!

Evaluación 7-2A

1. Si Maura fue a la tienda y compró una silla por $17.95, un b. Si a cada celda del cuadrado mágico se le suma 0.85,
rastrillo por $13.59, una pala por $14.86, una podadora (i) ¿Sigue siendo un cuadrado mágico?
por $179.98 y dos paquetes de agua mineral por $2.43 cada (ii) Si la respuesta a la parte (i) es “sí”, ¿cuál es la suma
uno, ¿cuál fue la cuenta? de cada renglón?
2. a. Completa el siguiente cuadrado mágico; esto es, haz que c. Si cada celda del cuadrado mágico se multiplica por
la suma de cada renglón, columna y diagonal sea la misma: 0.5, ¿sigue siendo un cuadrado mágico? En caso afir-
mativo, ¿cuál es la suma de cada renglón?
8.2 3. Una acción subió 0.24 en el mercado el jueves. Si el
precio resultante fue de 73.245, ¿cuál era el precio de la
3.7 5.5 acción antes de subir?

9.1 2.8
© Lopez Mateos Editores. ISBN 978-607-95583-2-1, obra completa, versión electrónica, ISBN 978-607-95583-3-8, volumen 1, versión electrónica. Ejemplar asignado a: Helecto Villarroel gutierrez -
helecto@gmail.com. Fecha: 27 de octubre de 2014. Prohibida su modificación, copia o distribución.
434 Decimales y números reales

4. Un billete de $1 dólar estadounidense valía 1.0156 dólares cana- 16. Escribe en forma común los numerales de las frases
dienses el 22 de febrero de 2008. ¿Cuál era el valor de $27.32 siguientes:
-
dólares estadounidenses en dólares canadienses ese día? a. La masa de una partícula de polvo es 7.53 * 10 10 g.
5. Un kilowatt-hora significa 1000 watts de electricidad usados b. La rapidez de la luz es de aproximadamente 2.98 *
de manera continua durante 1 h. En el pueblo donde vive 105 km por segundo.
Laura, la compañía eléctrica cobra $0.0715 por cada kilo- c. Júpiter dista aproximadamente 7.78570000 * 108 km
watt-hora usado. Laura calienta su casa con tres calentadores del Sol.
eléctricos de pared que usan 1200 watts por hora cada uno. 17. Escribe en notación científica los resultados de cada caso:
a. ¿Cuánto cuesta calentar la casa por un día? a. 18 # 1012216 # 10152
b. ¿Cuántas horas debe estar prendido un foco de 75 b. 116 # 10122 , 14 # 1052
watts para que aporte $1 a la cuenta de luz? c. 15 # 108216 # 1092 , 115 # 10152
6. Un litro es igual a 4.224 c. ¿Cuántos litros hay en 36.5 c? 18. Redondea cada caso según se especifica:
7. Florence Griffith-Joyner estableció un récord mundial a. 203.651 a la centena más próxima
para la carrera femenina de 100 m en los Juegos Olímpi- b. 203.651 a la decena más próxima
cos de Verano de 1988 en Seúl, Corea del Sur. Ella cubrió c. 203.651 a la unidad más próxima
la distancia en 10.49 s. Si 1 m es equivalente a 39.37 pulg, d. 203.651 a la décima más próxima
expresa la velocidad de Griffith-Joyner en términos de e. 203.651 a la centésima más próxima
millas por hora. 19. El carro de Juana viaja 224 mi con 12 gal de gas. Redon-
8. Si cada una de las sucesiones siguientes es aritmética o deando a la milla más cercana, ¿cuántas millas por galón
geométrica, continúa los siguientes patrones decimales: rinde su carro?
a. 0.9, 1.8, 2.7, 3.6, 4.5, , , 20. Adri quiere comprar una cámara fotográfica para utili-
b. 0.3, 0.5, 0.7, 0.9, 1.1, , , zarla en el cumpleaños de su hija. Para estimar el costo
9. Si el primer término de una sucesión geométrica finita es total, redondea cada precio al peso más cercano y suma
0.9 y si su razón es 0.2, ¿cuál es la suma de los primeros los precios redondeados. ¿Cuál es su estimado para los
cinco términos? artículos listados?
10. Interpreta el decimal 0.2222 como la suma de una suce-
sión geométrica finita cuyo primer término es 0.2. Cámara $54.56
(Sugerencia: Escribe 0.2222 como la suma de fracciones Película $4.50
cuyos denominadores son potencias de 10.) Estuche $17.85
11. En una sucesión geométrica finita, el primer término es
0.2 y el sexto término es 0.000486. ¿Cuáles son los tér- 21. Estima la suma o diferencia de cada caso usando (i) re-
minos segundo a quinto? dondeo y (ii) estimación desde la izquierda. Luego
12. Estima la ubicación de cada uno de los casos siguientes efectúa los cálculos para ver cuán cerca están tus esti-
en la recta numérica dada, colocando la letra de cada cál- mados de las respuestas reales.
culo en el cuadro apropiado. a. 65.84 b. 89.47
a. 0.3 , 0.31 b. 0.3 # 0.31 24.29 - 32.16
12.18
0 0.5 1
+ 19.75

13. Un estado de cuenta de cierto banco local muestra que una c. 5.85 d. 223.75
cuenta de cheques tiene un saldo de $83.62. El saldo calcu- 6.13 - 87.60
lado en la chequera muestra sólo $21.69. Después de verificar 9.10
con el banco, el cliente halla que éste todavía no ha registrado + 4.32
seis cheques por las cantidades de $3.21, $14.56, $12.44, 22. Escribe dos números en notación científica tales que un
$6.98, $9.51 y $7.49. ¿Es correcto el registro del banco? (Su- número sea mayor que 1, el otro sea menor que 1 y el
pón que el registro de la chequera del cliente sí lo es.) producto de los dos números sea 1.
14. Convierte -cada caso siguiente a numerales comunes: 23. Halla el menor y el mayor productos posibles para la ex-
a. 3.2 # 10 9 presión usando los dígitos 1 a 9. Cada dígito puede
b. 3.2 # 109- usarse sólo una vez en cada parte.
c. 4.2 # 10 1
d. 6.2 # 105 . *
15. Escribe en notación científica los numerales de cada una 24. En el número siguiente, algunos dígitos están cubiertos
de las frases siguientes: por cuadros:
a. El diámetro de la Tierra es de alrededor de 12,700,000 m.
b. La distancia de Plutón al Sol es de alrededor de 4 3 . 8
4,486,000,000 km. Si cada uno de los dígitos 1 a 9 se usó exactamente una
c. Cada año se desechan alrededor de 50,000,000 de
vez en el número, determina el mayor número posible.
latas en Estados Unidos.
© Lopez Mateos Editores. ISBN 978-607-95583-2-1, obra completa, versión electrónica, ISBN 978-607-95583-3-8, volumen 1, versión electrónica. Ejemplar asignado a: Helecto Villarroel gutierrez -
helecto@gmail.com. Fecha: 27 de octubre de 2014. Prohibida su modificación, copia o distribución.
Sección 7-2 Operaciones con decimales 435

25. Iris trabajó una semana de 40 horas a $8.25>h. Calcula 27. ¿Cuáles casos producen iguales cocientes?
mentalmente su salario de la semana y explica cómo lo a. 7 , 0.25 b. 70 , 2.5
hiciste. c. 0.7 , 0.25 d. 700 , 25
26. En cada una de las expresiones siguientes, calcula men- 28. En cada uno de los productos siguientes, usa la estimación
talmente el número por colocar en el espacio en blanco: para colocar el punto decimal en su posición correcta:
a. 8.4 # 6 = 4.2 # a. 534 # 0.34 = 18156
b. 10.2 , 0.3 = 20.4 , b. 5.07 # 29.3 = 148551
c. ab = 1a>22 #
d. a , b = 2a ,

Evaluación 7-2 B

1. El precio de unas acciones cayó de $63.28 a $27.45 por ac- 8. Si cada una de las sucesiones siguientes es aritmética o
ción. ¿Cuál fue la pérdida en una acción? geométrica, continúa los siguientes patrones decimales:
2. Quique trajo 30 lb de nueces a $3.00> lb y 20 lb de nue- a. 1, 0.5, 0.25, 0.125, , ,
ces a $5.00> lb. Si quisiera comprar 10 libras más de una b. 0.2, 1.5, 2.8, 4.1, 5.4, , ,
clase diferente de nuez para hacer que el promedio del 9. Si el primer término de una sucesión geométrica finita es
precio por libra sea igual a $4.50, ¿qué precio debería 0.4 y si su razón es 0.3, ¿cuál es la suma de los primeros
pagar por las 10 lb adicionales? cinco términos?
3. Los motores de automóviles se medían en pulgadas cúbicas 10. Interpreta el decimal 0.3333333 como la suma de una
1pulg32, pero ahora se miden en centímetros cúbicos 1cm32. sucesión geométrica finita cuyo primer término es 0.3.
Si 2.54 cm es equivalente a 1 pulg, responde lo siguiente: (Sugerencia: Escribe 0.3333333 como la suma de fraccio-
a. El Thunderbird 1963 de Susana tiene un motor de nes cuyos denominadores son potencias de 10.)
390 pulg3. ¿Aproximadamente de cuántos centíme- 11. Estima la ubicación de cada uno de los casos siguientes
tros cúbicos es? en la recta numérica dada, colocando la letra de cada
b. El Taurus 1991 de Daniel tiene un motor de 3000 cm3 cálculo en el cuadro apropiado:
¿Aproximadamente de cuántas pulgadas cúbicas es? a. 0.3 + 0.31 b. 0.3 - 0.31
4. En las Olimpiadas de 2004, en lanzamiento de jabalina el
grupo 1 obtuvo los resultados siguientes (con distancias en
metros): Carolina Klüft, SUE, 48.89; Svetlana Sokolova, 0 0.5 1
RUS, 47.86; Shelia Burrell, USA, 47.69. Por qué longi-
tudes venció Klüft a Sokolova y a Burrel? 12. Convierte cada caso siguiente a numerales comunes:
a. 3.5 # 107 b. 3.5 # 10 7
-

c. 12.4 10 2
5. Completa el siguiente cuadrado mágico; esto es, haz que la
suma de cada renglón, columna y diagonal sea la misma:
- # -3

13. Escribe en notación científica los numerales de cada una


de las frases siguientes:
a. La población de un estado es de 999,347 habitantes.
7.5 b. La superficie de México es de 1,964,375 km2.
c. Se estima que el diámetro del núcleo de un átomo es
-
3.0 4.8 de 10 15 m.
14. Escribe en notación científica los resultados de cada caso:
8.4 2.1 a. 15 # 107217 # 10122
b. 1 - 13 # 1042 , 65
c. 13 # 107214 # 1052 , 16 # 10 72
-

6. En una tienda Samuel compró un ramo de flores por 15. Redondea cada caso según se especifica:
$4.99, una barra de dulce por $0.79, una memoria usb a. 715.04 a la centena más próxima
por $49.99 y una botella de agua por $1.49. ¿Cuál fue su b. 715.04 a la decima más próxima
cuenta? c. 715.04 a la unidad más próxima
7. La compañía de electricidad cobra $0.0485 por cada d. 715.04 a la decena más próxima
kilowatt-hora usado. Tere usó su computadora durante e. 715.04 a los miles más próximos
3.4 h en que la computadora misma gastó 45 watts por 16. Juana viaja con una rapidez constante de 55.5 mph.¿Qué
hora y el monitor gastó 35 watts por hora. ¿Cuál fue el 3
cargo por el uso de la computadora? . distancia espera cubrir en h?
4

© Lopez Mateos Editores. ISBN 978-607-95583-2-1, obra completa, versión electrónica, ISBN 978-607-95583-3-8, volumen 1, versión electrónica. Ejemplar asignado a: Helecto Villarroel gutierrez -
helecto@gmail.com. Fecha: 27 de octubre de 2014. Prohibida su modificación, copia o distribución.
436 Decimales y números reales

17. Despeja x, donde x es un decimal: a. 47.62 b. 79.86


a. 2x + 1.3 = 4.1 b. 4.2 - 3x = 10.2 27.99 - 27.37
c. 8.56 = 3 - 2x 13.14
18. Escribe en forma común los numerales de las frases si- + 7.61
guientes:
a. Una computadora requiere 4.4 # 10 6 s para hacer un
-
c. 5.85 d. 232.65
problema de suma. 6.17 - 78.92
b. Hay alrededor de 1.99 # 104 km de costa en Estados 9.1
Unidos. + 4.23
c. La Tierra ha existido por aproximadamente 3 # 109 años. 24. ¿Cuáles casos producen iguales cocientes?
19. Queta Flores invirtió $964 en 18 acciones. Un mes des- a. 9 , 0.35 b. 90 , 3.5
pués vendió las acciones a $61.48 cada una. También in- c. 900 , 35
virtió en 350 acciones por un total de $27,422.50. Las 25. ¿Es lo mismo restar 0.3 que
vendió en $85.35 cada una y pagó $495 de comisión. a. restar 0.30?
¿Cuál fue la ganancia o pérdida, al peso más cercano, de b. restar 0.03? Explica.
Queta Flores en las transacciones? 26. Usa la estimación para escoger un decimal para multiplicar
20. Halla el menor y el mayor productos posibles para la ex- por 9 y obtener un número que esté a menos de 1 de 93.
presión usando los dígitos 1 a 9. Cada dígito puede Explica cómo hiciste tu selección y verifica tu estimado.
usarse sólo una vez en cada parte.
27. a. Llena los paréntesis siguientes para obtener una ecua-
. * . ción verdadera:

21. En el número siguiente, algunos dígitos están cubiertos 1 # 2 + 0.25 = 1 22


por cuadros: 2 # 3 + 0.25 = 1 22

4 3 . 8 Emite una conjetura acerca de cuáles serán las dos ecua-


ciones siguientes en este patrón.
Si cada uno de los dígitos 1 a 9 se usó exactamente una b. Realiza los cálculos para determinar si tus dos ecua-
vez en el número, determina el menor número posible. ciones siguientes son correctas.
22. Iris trabajó una semana de 40 horas a $6.25/h. Calcula c. Generaliza tu respuesta en la parte (a) llenando la
mentalmente su salario de la semana y explica cómo lo ecuación con una expresión apropiada,
hiciste.
23. Estima la suma o diferencia de cada caso usando (i) re- n1n + 12 + 0.25 = 1 22
dondeo y (ii) estimación desde la izquierda. Luego efec- donde n es el número de términos en esta sucesión de
túa los cálculos para ver cuán cerca están tus estimados de ecuaciones.
las respuestas reales.

Conexiones matemáticas 7-2

Comunicación Solución abierta


1. Da un ejemplo de una chequera con su saldo calculado, 6. Halla varios ejemplos del uso de decimales en los periódi-
donde los registros pudieran estar incorrectos. cos. Di en qué caso crees que los números sean exactos o
2. ¿En qué se parecen la multiplicación de decimales y la mul- estimados. Di, además, por qué crees que se usaron deci-
tiplicación de números completos? ¿En qué se diferencian? males en lugar de fracciones
3. ¿Por qué son importantes las habilidades de estimación al 7. ¿Cómo podría usarse una calculadora para desarrollar o
dividir decimales? reforzar la comprensión de la multiplicación de números
4. En el texto realizamos multiplicaciones y divisiones decimales?
usando las formas fraccional y decimal. Analiza las venta- 8. ¿Cómo podrían ciertas calculadoras impedir el aprendi-
jas y desventajas de cada una. zaje de la multiplicación de decimales si se presenta en
5. Explica por qué la resta de decimales exactos se puede pantalla la forma más simple de un decimal exacto, por
efectuar alineando los puntos decimales, restando como si ejemplo 0.3 en lugar de 0.30?
los números fueran números completos y después colo- 9. Wearne y Hiebert (1988) reportaron que los estudiantes
cando el punto decimal en la diferencia. que conectan representaciones físicas de decimales con

© Lopez Mateos Editores. ISBN 978-607-95583-2-1, obra completa, versión electrónica, ISBN 978-607-95583-3-8, volumen 1, versión electrónica. Ejemplar asignado a: Helecto Villarroel gutierrez -
helecto@gmail.com. Fecha: 27 de octubre de 2014. Prohibida su modificación, copia o distribución.
Sección 7-2 Operaciones con decimales 437

notación decimal tienen mayor posibilidad de crear sus Problemas de repaso


propios procedimientos para convertir una fracción a su 15. Escribe 14.0479 en forma expandida.
notación decimal. ¿Piensas que esta investigación apoya, 16. Sin dividir, determina cuál de las siguientes expresiones
o no, la conversión de decimales a fracciones como mé- representa decimales exactos:
todo para explicar procedimientos de cálculos? 24
a.
Aprendizaje colectivo 36
10. En tu grupo, digan cuáles son todas las habilidades reque- 49
b.
ridas por los estudiantes para aprender a efectuar opera- 56
ciones aritméticas con decimales. 17. Si el denominador de una fracción es 26, ¿es posible escri-
11. Vas a necesitar una calculadora y una pareja para jugar el juego bir la fracción como un decimal exacto? ¿Por qué sí o por
descrito en la página de muestra 438 tomada de McDougal qué no?
Littell MATH Thematics, New Edition Book 1, 2008. 35
18. se puede escribir como un decimal exacto. Explica por
56
Preguntas del salón de clase
qué.
12. Una estudiante multiplica 16.5218.52 para obtener lo
siguiente: Preguntas del Third International Mathematics and
8.5 Science Study (TIMSS) (Tercera Reunión Internacional
* 6.5 sobre el Estudio de las Matemáticas y la Ciencia)
425 Una pelota de hule rebota a la mitad de la altura a la que se
510 soltó. Si la pelota se soltó de un techo ubicado a 18 m sobre
55.25 la superficie, ¿cuál es el total de la distancia recorrida hasta
1# 1 el momento en que golpea el suelo por tercera vez?
Sin embargo, cuando la estudiante multiplica 8 6 , ob- a. 31.5 m
2 2
tiene lo siguiente: b. 40.5 m
c. 45 m
1 d. 63 m
8
2 En una competencia de lanzamiento de disco, el lanza-
1 miento ganador fue de 61.60 m. El segundo lugar fue de
*6
2 59.72 m. ¿Qué tanto más largo fue el lanzamiento ganador
a 8 b
1 1# 1 que el del segundo lugar?
4
4 2 2 a. 1.18 m
48 16 # 82 b. 1.88 m
1 c. 1.98 m
52
4 d. 2.18 m
¿Cómo es posible esto? TIMSS, Grado 8, 1994
13. Una estudiante trata de calcular 0.99910,000 en una calcula- Preguntas del National Assessment of Educational
dora y obtiene que la respuesta es 4.5173346 # 10 5. La es-
-
Progress (NAEP) (Evaluación Nacional del Progreso
tudiante se pregunta cómo es posible que un número como Educativo)
0.999, tan cercano a 1, cuando se eleva a alguna potencia Cuesta $0.25 operar una secadora durante 10 minutos en
resulte en un número cercano a 0. ¿Qué le respondes? una lavandería. ¿Cuál es el costo de operar una secadora
14. Explica cómo puedes responder a lo siguiente: durante 30 minutos, una segunda por 40 minutos y una
9443 9444 tercera por 50 minutos?
a. Un estudiante asegura que y son iguales pues
9444 9445 a. $3.25
ambos se presentan como 0.9998941 en su calculadora b. $3.00
científica cuando se realizan las divisiones. c. $2.75
d. $2.00
b. Otro estudiante asegura que las fracciones no son iguales
e. $1.20
y quiere saber si la misma calculadora puede determinar
7 7 7
cuál es mayor. Suma los números , y . Escribe la suma como
10 100 1,000
decimal.
NAEP, Grado 8, 2007

© Lopez Mateos Editores. ISBN 978-607-95583-2-1, obra completa, versión electrónica, ISBN 978-607-95583-3-8, volumen 1, versión electrónica. Ejemplar asignado a: Helecto Villarroel gutierrez -
helecto@gmail.com. Fecha: 27 de octubre de 2014. Prohibida su modificación, copia o distribución.
438 Decimales y números reales

Página de un libro de texto ESTIMACIÓN DE PRODUCTOS


DECIMALES
Objetivo
Aprende cómo. . .
Ex loración
mejorar tus
habilidades
de estimación
Mientras. . . Estimación
juegas Acierta
al número en más
o menos 1
de productos
.decimales
Requerimientos Trabaja con una pareja. Necesitarán una calculadora.

En un nuevo juego, Acierta al número en más o menos 1, el objetivo es hallar


un producto que diste menos de 1 de un número dado. Para jugar,
sigan el diagrama de flujo. Deben usar estimación y matemática mental
para decidir qué números colocar en la calculadora.

Comienzo El jugador que


El jugador 1 escoge un hizo la última

número y lo escribe. propuesta
gana.

El jugador 2 escoge
un número y lo coloca
en la calculadora. Sí

El jugador 1 toma la ¿El El jugador 2 toma la ¿El


calculadora y resultado calculadora y resultado
presiona , dista menos de 1 presiona , dista menos de 1
coloca una propuesta del número coloca una propuesta del número
y luego presiona .. escrito? y luego presiona .. escrito?

Partida de muestra

El jugador 1 escoge 196, mismo que escribe.


El jugador 2 escoge el número 7.

Jugador Teclas presionadas Pantalla

Como 196.35 dista


menos de 1 de 196,
¡gana el jugador 1!

Módulo 4 Juegos mentales

Fuente: McDougal Littell Math Thematics, New Edition, 2008 ( p. 188).

© Lopez Mateos Editores. ISBN 978-607-95583-2-1, obra completa, versión electrónica, ISBN 978-607-95583-3-8, volumen 1, versión electrónica. Ejemplar asignado a: Helecto Villarroel gutierrez -
helecto@gmail.com. Fecha: 27 de octubre de 2014. Prohibida su modificación, copia o distribución.
Sección 7-3 Decimales que no terminan 439

7-3 Decimales que no terminan

En los Puntos focales para el grado 7 hallamos que:

Ahora los estudiantes usan la división para expresar cualquier fracción como un decimal, inclu-
yendo fracciones que deben representar con decimales infinitos. (p. 38)

En este capítulo ya desarrollamos procedimientos para convertir algunos números racionales


7
a decimales. Por ejemplo, se puede escribir como un decimal exacto:
8
7 7 7 # 53 875
= 3 = 3 3 = = 0.875
8 2 2 5 # 1000
7
El decimal para también se puede obtener mediante una división:
8
0.875
8 冄 7.000
64
60
56
40
40
0
2
Sin embargo, mostramos que era un decimal que no termina. Ahora vamos a investigar los
11
decimales que no terminan.

Decimales periódicos
2
Si usamos una calculadora a fin de encontrar una representación decimal para , la calcula-
11
dora puede presentar 0.1818181. Parece que 18 se repite. Para examinar qué dígitos, si los
hay, no presentó la calculadora, considera la división siguiente:

0.18
11 冄 2.00
11
90
88
2
En este punto, si se continúa la división se repetirá el patrón de la división, pues el residuo
2 dividido entre 11 repite la división. Así, el cociente es 0.181818 Á . Un decimal de este
tipo es un decimal periódico, y los dígitos que se repiten forman el periodo. El decimal
periódico se escribe 0.18, donde la barra indica que los dígitos debajo de ella se repiten con-
tinuamente.

© Lopez Mateos Editores. ISBN 978-607-95583-2-1, obra completa, versión electrónica, ISBN 978-607-95583-3-8, volumen 1, versión electrónica. Ejemplar asignado a: Helecto Villarroel gutierrez -
helecto@gmail.com. Fecha: 27 de octubre de 2014. Prohibida su modificación, copia o distribución.
440 Decimales y números reales

Ejemplo 7-10 Convierte a decimales:


1 2









a. b.
7 13
Solución Usamos una calculadora para dividir, y parece que los patrones de división se
repiten.
1 2
a. = 0.142857 b. = 0.153846
7 13

Para ver por qué en el ejemplo 7-10 los patrones de división se repiten como se predijo,
considera las divisiones siguientes:
a. 0.142857 b. 0.153846
7 冄 1.000000 13 冄 2.000000
7 13
30 70
28 65
20 50
14 39
60 110
56 104
40 60
35 52
50 80
49 78
1 2
1
En , los residuos obtenidos en la división son 3, 2, 6, 4, 5 y 1. Éstos son todos los residuos
7
distintos de cero que pueden obtenerse al dividir entre 7. Si hubiéramos obtenido un resi-
duo de 0, el decimal terminaría. En consecuencia, la séptima división no puede producir un
nuevo residuo. Cuando un residuo se repite, el proceso mismo se repite. Usando un razo-
2
namiento similar, podemos predecir que el periodo para no puede tener más de 12 dígi-
13
tos, pues hay sólo 12 posibles residuos distintos de cero. Sin embargo, uno de los residuos
a
podría repetirse más rápido, lo que en realidad sucedió en la parte (b). En general, si es cual-
b
quier número racional en su forma más simple, con b Z 0 y b 7 a, y si no representa un deci-
mal exacto, el periodo tiene a lo sumo b - 1 dígitos. Por lo tanto, un número racional siempre
puede representarse ya sea como un decimal exacto o como un decimal periódico.

AHORA INTENTA ÉSTE 7-8


1
a. Escribe como decimal.
9
b. Con base en tu respuesta a la parte (a), calcula mentalmente la representación decimal de cada caso:

2 3 5 8
(i) (ii) (iii) (iv)
9 9 9 9

© Lopez Mateos Editores. ISBN 978-607-95583-2-1, obra completa, versión electrónica, ISBN 978-607-95583-3-8, volumen 1, versión electrónica. Ejemplar asignado a: Helecto Villarroel gutierrez -
helecto@gmail.com. Fecha: 27 de octubre de 2014. Prohibida su modificación, copia o distribución.
Sección 7-3 Decimales que no terminan 441

Ejemplo 7-11 1
Usa una calculadora para convertir a un decimal periódico.









17

Solución Si tecleamos 1 4 1 7 = en una calculadora, obtenemos lo siguiente como


parte de un problema de división:
0.0588235
17 冄 1.
Sin saber si la calculadora tiene una característica interna de redondeo y con una pantalla de
ocho dígitos, hallamos que el mayor número de dígitos en los que podemos confiar en el co-
ciente es de seis después del punto decimal. (¿Por qué?) Si usamos esos seis lugares y multipli-
camos 0.058823 por 17, podemos continuar la operación como sigue:
. 0 5 8 8 2 3 3 1 7 =
Obtenemos entonces 0.999991, que podemos colocar en la división anterior:
0.0 5 8 8 2 3
1 7 冄 1.00 0 0 0 0
999991
9
A continuación dividimos 9 entre 17 para obtener 0.5294118. De nuevo, ignorando el dí-
gito más a la derecha, continuamos como antes, completando la división como sigue, donde
se percibe el patrón de repetición:
0.05882352941176470588235
17 冄 1.00000000000000000000000
999991
9000000
8999987
13000000
12999985
15
1
Así, = 0.0588235294117647, y el periodo es de 16 dígitos de largo.
17 ◆
El ejemplo 7-11 sugiere un método para mostrar que todo número racional se puede es-
cribir como un decimal. Además, el ejemplo 7-11 ilustra cómo se puede usar una calcula-
dora con sólo un número finito de dígitos en pantalla para hacer divisiones más largas de las
que se supone estaba diseñada para realizar. Una sugerencia común para estudiantes de en-
señanza básica es que usen el algoritmo de la división de manera selectiva para aprender el
proceso. El ejemplo de la calculadora y otros similares deberán usarse para ver si en verdad
entiendes el proceso de división y los valores posicionales que se usan.
a
Escribir un decimal periódico en la forma ,
donde a, b “ E, b ⴝ 0
b
a
Ya vimos cómo escribir decimales exactos en la forma , donde a y b son enteros y
b
b Z 0. Por ejemplo,
55 55
0.55 = 2
=
10 100

© Lopez Mateos Editores. ISBN 978-607-95583-2-1, obra completa, versión electrónica, ISBN 978-607-95583-3-8, volumen 1, versión electrónica. Ejemplar asignado a: Helecto Villarroel gutierrez -
helecto@gmail.com. Fecha: 27 de octubre de 2014. Prohibida su modificación, copia o distribución.
442 Decimales y números reales

Para escribir 0.5 de manera similar, vemos que el decimal periódico tiene infinidad de dígi-
tos; no hay una sola potencia de 10 que pueda colocarse en el denominador. Para superar esta
dificultad, debemos eliminar de alguna manera la parte que se repite infinitamente en el deci-
mal. Si tenemos n = 0.5, entonces nuestro objetivo parcial es escribir una ecuación para n sin un
decimal periódico. Se puede mostrar que 1010.555 Á 2 = 5.555 Á = 5.5. Por lo tanto,
10n = 5.5. Usando esta información, restamos los lados correspondientes de la ecuación para
obtener una ecuación cuya solución se pueda escribir sin un decimal periódico.

10n = 5.5
- n = 0.5
9n = 5
5
n =
9

5
Así, 0.5 = . Este resultado puede verificarse efectuando la división 5 , 9. (Nota: La resta
9
da una ecuación que contiene solamente enteros. Los dígitos que se repiten se “cancelan”
entre sí.)
Las sucesiones geométricas infinitas proporcionan un método general para convertir decimales
a
que se repiten a números racionales de la forma , donde a y b son enteros y b Z 0. Consi-
b
dera 0.5 como suma de la sucesión geométrica 0.5, 0.05, 0.005, 0.0005 Á . En esta sucesión geo-
métrica, el primer término es 0.5 y la razón es 0.1. Así, tenemos la siguiente igualdad:

0.5 = 0.5 + 0.05 + 0.005 + 0.0005+ Á

Con el fin de determinar un método para hallar la suma de una sucesión geométrica
infinita, usa lo siguiente:
Sea S = 0.5 + 0.05 + 0.005 + 0.0005 + Á

Si multiplicamos ambos lados de la ecuación por 0.1, tenemos el siguiente par de ecuaciones:

S = 0.5 + 0.05 + 0.005 + 0.0005 + Á


0.1S = 0.05 + 0.005 + 0.0005 + Á

Restando la ecuación de abajo de la de arriba, tenemos

0.9S = 0.5

0.5 5
de modo que S = , ó , el mismo resultado que antes.
0.9 9
A continuación presentamos el procedimiento para hallar la suma de una sucesión geométrica
infinita cuya razón es r, donde 0 6 r 6 1.

S = a + ar + ar2 + ar3 + Á
- rS = - 1ar + ar2 + ar3 + Á 2
S - rS = a
S11 - r2 = a
a
S =
1 - r

© Lopez Mateos Editores. ISBN 978-607-95583-2-1, obra completa, versión electrónica, ISBN 978-607-95583-3-8, volumen 1, versión electrónica. Ejemplar asignado a: Helecto Villarroel gutierrez -
helecto@gmail.com. Fecha: 27 de octubre de 2014. Prohibida su modificación, copia o distribución.
Sección 7-3 Decimales que no terminan 443

AHORA INTENTA ÉSTE 7-9


a. Si r = 1, ¿cómo sabes que S no existe?
b. Si r 7 1, ¿qué sabemos acerca de r n + 1 conforme n se hace más y más grande?
c. Halla una fórmula para la suma de una sucesión geométrica finita.

En textos escolares los decimales se escriben como “fracciones”, según vemos en la por-
ción de la siguiente página de muestra. Responde el número 4.

Página de un libro de texto M Á S P R Á CT I CA

Halla 4 + 2 × 3 – 4 4 2.

Más práctica
Escribir como fracciones decimales periódicos
Una manera breve de escribir el decimal periódico
0.323232... es 0.32. Puedes escribir este
decimal periódico, como fracción.
Escribe 1n=0.323232... Como se repiten dos
dígitos, multiplica ambos lados por 100 para
obtener 100n=32.323232...
Resta y despeja n.

Para escribir 0.1643 como fracción, escribe


1n=0.1643643...
Se repiten tres dígitos, de modo que multiplicas ambos
lados por 1,000 para obtener 1,000n=164.3643643...
Resta y despeja n. Simplifica.
Para 1–4, escribe cada decimal como una fracción con
números completos en el numerador y denominador.

Todas las páginas de texto están disponibles en línea o en CD-ROM. Sección A Lección

Fuente: Scott Foresman-Addison Wesley Mathematics, Grade 6, 2008 ( p. 251).

© Lopez Mateos Editores. ISBN 978-607-95583-2-1, obra completa, versión electrónica, ISBN 978-607-95583-3-8, volumen 1, versión electrónica. Ejemplar asignado a: Helecto Villarroel gutierrez -
helecto@gmail.com. Fecha: 27 de octubre de 2014. Prohibida su modificación, copia o distribución.
444 Decimales y números reales

Supón que un decimal tiene periodo de más de un dígito, como en 0.235. Podemos usar
la fórmula recién desarrollada, donde el primer término de una sucesión geométrica es
1
0.235, r = , y hallar la suma como sigue:
1000
0.235 0.235 235
S = = =
1 999 999
1 -
1000 1000
235
Así, 0.235 = .
999
Un enfoque equivalente es multiplicar el decimal 0.235 por 103, pues hay un periodo de tres
dígitos. Sea n = 0.235. Nuestro objetivo parcial es escribir una ecuación para n que no tenga el
decimal periódico:
1000n = 235.235
- n = 0.235
999n = 235
235
n =
999

235
De nuevo, 0.235 = .
999
Generalizamos el método anterior notando primero que 0.5 se repite con periodo de un
a
dígito. Por lo tanto, para escribirlo en la forma , multiplicamos primero por 101. Como
b
0.235 se repite con periodo de tres dígitos, multiplicamos primero por 103. En general, si el
periodo está inmediatamente a la derecha del punto decimal, primero multiplicamos por 10m, donde
m es el número de dígitos en el periodo, y después continuamos como en los casos anteriores.
Ahora supongamos que el periodo no se presenta inmediatamente después del punto de-
cimal. Por ejemplo, sea n = 2.345. Una estrategia para resolver este problema es cambiarlo
a un problema relacionado que ya sepamos cómo solucionar; esto es, cambiarlo a un problema
en donde el periodo siga de manera inmediata al punto decimal. Éste es nuestro nuevo
objetivo parcial. Para lograrlo multiplicamos ambos lados por 10:
n = 2.345
10n = 23.45

Procedemos ahora como en los problemas anteriores. Como 10n = 23.45 y el número de
dígitos en el periodo es 2, multiplicamos por 102 como sigue:

100110n2 = 2345.45

Así,
1000n = 2345.45
- 10n = 23.45
990n = 2322
2322 129
n = ,ó
990 55

2322 129
Por lo tanto, 2.345 = ,ó .
990 55

© Lopez Mateos Editores. ISBN 978-607-95583-2-1, obra completa, versión electrónica, ISBN 978-607-95583-3-8, volumen 1, versión electrónica. Ejemplar asignado a: Helecto Villarroel gutierrez -
helecto@gmail.com. Fecha: 27 de octubre de 2014. Prohibida su modificación, copia o distribución.
Sección 7-3 Decimales que no terminan 445

AHORA INTENTA ÉSTE 7-10 Usa la suma de una sucesión geométrica infinita para hallar una fracción
equivalente para 0.235 y 2.345.

Un resultado sorprendente
a
Para hallar la forma de 0.9, procedemos como sigue. Sea n = 0.9, y entonces 10n = 9.9.
b
A continuación restamos y continuamos despejando n:

10n = 9.9
- n = 0.9
9n = 9
n = 1
De aquí que 0.9 = 1. Puede ser que este enfoque del problema no sea convincente. A conti-
nuación se muestra otro enfoque para hacer ver que 0.9 es en realidad otro nombre para 1.
1 2
(a) = 0.33333333 Á (b) = 0.66666666 Á
3 3
Sumando las ecuaciones (a) y (b), tenemos que 1 = 0.99999999 Á . ó 0.9. Este último deci-
9 9 9
mal representa la suma infinita + 2
+ + Á.
10 10 103
Quizá algunos prefieran un enfoque visual para demostrar que 0.9 = 1. Considera la recta
numérica de la figura 7-5(a). La mayoría estará de acuerdo en que 0.9 está entre 0.9 y 1.0, de
modo que ahí comenzamos. Entonces 0.9 está entre 0.99 y 1.0, como en la figura 7-5(b).
0.9 ?

0 0.3 0.6 0.9 1


(a)

0.9 ?

0 0.3 0.6 0.9 1


(b)
0.99

Figura 7-5
Podemos proceder de manera análoga argumentando que 0.9 debe estar entre 0.999 y 1, y
así sucesivamente. Ahora, es razonable preguntar si existe alguna pequeña cantidad positiva
a tal que 0.9 + a = 1.La respuesta tiene que ser no. (¿Por qué?) Si no existe dicho número
a, entonces 0.9 no puede ser menor que 1. Con un argumento análogo vemos que 0.9 no
puede ser mayor que 1; así, tenemos que 0.9 = 1. En cursos más avanzados de matemáti-
cas, sumas como 0.9, ó 0.9 + 0.09 + 0.009 + Á , se definen como límites de sumas finitas.

Orden en los decimales periódicos


a
Sabemos que cualquier decimal periódico se puede escribir como número racional en la forma ,
b
donde b Z 0. Así, cualquier decimal periódico puede representarse sobre una recta numérica de ma-
nera similar a como los decimales exactos se pueden colocar sobre una recta numérica.
© Lopez Mateos Editores. ISBN 978-607-95583-2-1, obra completa, versión electrónica, ISBN 978-607-95583-3-8, volumen 1, versión electrónica. Ejemplar asignado a: Helecto Villarroel gutierrez -
helecto@gmail.com. Fecha: 27 de octubre de 2014. Prohibida su modificación, copia o distribución.
446 Decimales y números reales

a
Además, como entre dos números racionales de la forma hay infinidad de racionales de
b
esa forma, es razonable pensar que debe haber infinidad de decimales periódicos entre cua-
lesquier otros dos decimales.
Para ordenar los decimales periódicos, podemos considerar dónde puede estar un deci-
mal periódico en esa recta, o podríamos compararlos usando el valor posicional de ma-
nera análoga a como ordenamos decimales exactos. Por ejemplo, para ordenar decimales
periódicos tales como 1.3478 y 1.347821, escribimos los decimales uno debajo del otro,
en su forma equivalente sin las barras, y alineamos los puntos decimales (o valores posi-
cionales) como sigue:
1.34783478 Á
1.34782178 Á

Los dígitos a la izquierda de los puntos decimales y los primeros cuatro dígitos después del
punto decimal son los mismos en cada uno de los números. Sin embargo, como el dígito de
las cienmilésimas del número de arriba, que es 3, es mayor que el dígito 2 en el lugar de las
cienmilésimas del número de abajo, 1.3478 es mayor que 1.347821.
21 37
Es fácil comparar dos fracciones como y , usando una calculadora. Convertimos
43 75
cada uno a decimal y luego comparamos los decimales.

2 1 , 4 3 = : 0.4883721
3 7 , 7 5 = : 0.4933333

Examinando los dígitos en el lugar de las centésimas, vemos que

37 21
7
75 43

Ejemplo 7-12 Halla un número racional en forma decimal entre 0.35 y 0.351.








Solución Primero alineamos los decimales.


0.353535 Á
0.351351 Á
Después, para hallar un decimal entre estos dos, observamos que, comenzando desde la
izquierda, el primer lugar en el que difieren los dos números es el lugar de las milésimas.
Claramente, un decimal entre estos dos es 0.352. Otros serían 0.3514, 0.3515 y 0.35136.
De hecho, hay infinidad de ellos.

AHORA INTENTA ÉSTE 7-11 Dado el decimal exacto 0.36, halla dos decimales que se repitan, uno me-
nor que 0.36 pero no más de 0.01 menor, y uno mayor que 0.36 pero no más de 0.01 mayor.

© Lopez Mateos Editores. ISBN 978-607-95583-2-1, obra completa, versión electrónica, ISBN 978-607-95583-3-8, volumen 1, versión electrónica. Ejemplar asignado a: Helecto Villarroel gutierrez -
helecto@gmail.com. Fecha: 27 de octubre de 2014. Prohibida su modificación, copia o distribución.
Sección 7-3 Decimales que no terminan 447

Evaluación 7-3A

1. Halla la representación decimal para cada caso: 3


13. a. ¿Qué dígito hay en el lugar 21 de la expansión decimal de ?
4 2 7
a. b.
9 7 b. ¿Qué dígito hay en el lugar 5280 de la expansión de-
3 1 1
c. d. cimal de ?
11 15 17
2 1 14. a. Escribe cada uno de los casos siguientes como una frac-
e. f. a
75 99 ción de la forma , donde a y b son enteros y b Z 0.
5 1 b
g. h. (i) 0.1 (ii) 0.01 (iii) 0.001
6 13
1 3 b. ¿Qué fracción esperarías para 0.0001 ?
i. j. 1
21 19 c. Calcula mentalmente el decimal equivalente a .
2. Convierte cada uno de los siguientes decimales periódicos 90
a 1
a la forma , donde a, b son enteros y b Z 0. 15. Usa el hecho de que 0.1 = para convertir mental-
b 9
a. 0.4 b. 0.61 mente cada una de las fracciones siguientes:
c. 1.396 d. 0.55 a. 0.2 b. 0.3
e. - 2.34 f. - 0.02 c. 0.5 d. 2.7
3. Expresa 1 min como una parte decimal periódica de una e. 9.9
hora. 1 1
16. Usa el hecho de que 0.01 = y 0.001 = para
4. Ordena de mayor a menor cada uno de los decimales si- 99 999
guientes: convertir mentalmente cada caso en fracciones:
- 1.454, - 1.454, - 1.45, - 1.454, - 1.454 a. 0.05 b. 0.003
c. 3.25 d. 3.125
5. Describe posibles términos para el siguiente patrón de 17. Halla la suma de la sucesión geométrica finita cuyo primer
sucesión: término es 0.4, la razón es 0.5 y tiene cinco términos.
0, 0.5, 0.6, 0.75, 0.8, 0.83, , , 18. Usa la fórmula para la suma de una serie geométrica infi-
a
nita para hallar un número racional de la forma , donde
6. En el decimal periódico 0.45 interpretándolo como una b
sucesión geométrica, ¿cuál es la razón? a y b son enteros y b Z 0, para los siguientes décimales
1 periódicos:
7. Da un argumento de por qué 3 debe ser un decimal
7 a. 0.29 b. 2.029
periódico. 19. En esta sección viste que 0.9 es igual a 1. En otras pala-
8. Supón que a = 0.32 y b = 0.123. 1
a. Halla a + b sumando de izquierda a derecha. ¿Cuántos bras, 0.9 es otra representación de 1, como lo es .
1
dígitos hay en el periodo de la suma? ¿Cómo escribirías 1 como un decimal periódico, en
b. Halla a + b si a = 1.234 y b = 0.1234. ¿Es la res- forma diferente a 0.9? Explica tu representación.
puesta un número racional? ¿Cuántos dígitos hay en el 20. a. Determina si la suma de cualesquier dos decimales
periodo? periódicos debe ser un decimal periódico. Supón que
9. Explica si piensas que un decimal exacto se puede escri- cada periodo no es de nueves o de ceros.
bir como un decimal periódico. b. Explica por qué siempre puedes escribir la suma de
10. Halla tres decimales entre cada uno de los dos pares de un decimal exacto y un decimal periódico como un
decimales: decimal periódico.
a. 3.2 y 3.22 b. 462.24 y 462.243 c. Explica por qué la suma de cualesquier dos decimales
11. Halla el decimal que está a la mitad de los decimales si- exactos se puede escribir como un decimal periódico.
guientes: 0.4 y 0.5. d. Explica por qué no siempre es posible escribir la suma
3
12. a. Halla tres números racionales entre y 0.75. de cualesquier dos decimales periódicos como un deci-
4
1 mal exacto.
b. Halla tres números racionales entre y 0.34.
3

© Lopez Mateos Editores. ISBN 978-607-95583-2-1, obra completa, versión electrónica, ISBN 978-607-95583-3-8, volumen 1, versión electrónica. Ejemplar asignado a: Helecto Villarroel gutierrez -
helecto@gmail.com. Fecha: 27 de octubre de 2014. Prohibida su modificación, copia o distribución.
448 Decimales y números reales

Evaluación 7-3B

1. Halla la representación decimal para cada caso: 1


12. ¿Qué dígito hay en el lugar 23 de la expansión decimal de ?
2 7 1 17
a. b. c. 13. a. Escribe cada uno de los casos siguientes como una frac-
3 9 24
3 2 7 a
d. e. f. ción de la forma , donde a y b son enteros y b Z 0:
60 99 6 b
2 4 (i) 0.2 (ii) 0.02 (iii) 0.002
g. h. b. ¿Qué fracción esperarías para 0.0002 ?
21 19
4
2. Convierte cada uno de los siguientes decimales periódicos c. Calcula mentalmente el decimal equivalente a .
a 90
a la forma , donde a y b son enteros y b Z 0:
b 14. Con base en el hecho de que 0.9 = 1, ¿cuáles son los
a. 0.7 b. 0.46 c. 2.37 equivalentes de cada caso siguiente:
d. 2.34 e. - 4.34 f. - 0.03 a. 1.9 b. 2.9 c. 3.9
3. Expresa 1 s como una parte decimal periódico de una 1 1 1
15. Usa el hecho de que 0.1 = , 0.01 = , y 0.001 =
hora. 9 99 999
4. Ordena de menor a mayor cada uno de los decimales si- para convertir mentalmente cada caso en fracciones de la
guientes: a
forma , donde a y b son enteros y b Z 0:
b
- 4.34, - 4.34, - 4.34, - 4.343, - 4.434
a. 0.4 b. 0.12 c. 0.111
5. Describe posibles términos para la siguiente sucesión 16. Halla la suma de la sucesión geométrica finita cuyo primer
aritmética: término es 0.1, la razón es 0.3, y tiene cuatro términos.
a
17. Halla la suma en la forma , donde a y b son enteros y
0, 0.3, 0.6, 1, 1.3, ____ , ____ , ____ b
b Z 0 de la sucesión geométrica representada por:
夝 6. Halla una sucesión geométrica infinita cuya suma sea 2.34. a. 0.29 b. 0.00029
7. Explica en qué caso es posible que un decimal periódico
se puede escribir como un decimal exacto. 18. Considera los decimales periódicos, 0.23 y 0.235, ¿de
1 cuántos dígitos esperas que sea el periodo de la suma de
8. Da un argumento de por qué debe ser un decimal que se los dos decimales? ¿Por qué?
17
repite. 19. Halla valores de x de manera que cada caso siguiente sea
a
9. Halla tres decimales entre cada uno de los dos pares de verdadero. Escribe las respuestas tanto en la forma ,
b
decimales. donde a y b son enteros y b Z 0, y como decimales.
a. 4.3 y 4.3
b. 203.76 y 203.7 a. 3x = 8 b. 3x + 1 = 8
10. Halla el decimal que está a la mitad de los dos decimales c. 3x - 1 = 8 d. 1 - 3x = 8
siguientes: 0.9 y 1.1. e. 1 = 3x + 8 f. 1 = 8 - 3x
11. Halla tres números racionales entre los siguientes pares
de números:
2 2
a. y 0.67 b. y 0.67
3 3

Conexiones matemáticas 7-3

Comunicación d. Explica si la tienda debería o no usar un decimal pe-


1. a. Si una tienda de abarrotes anuncia tres limones por riódico como el costo de un artículo.
$2.00, ¿cuál es el costo de un limón? e. Explica tu idea sobre cómo trabajan las cajas registra-
b. Si escoges comprar exactamente un limón al costo doras con decimales periódicos.
dado en la parte (a), ¿cuánto te van a cobrar? 2. Un amigo asegura que todo decimal exacto es igual a al-
c. ¿Cómo trata la tienda el decimal periódico del costo de gún decimal infinito. ¿Es cierta esta afirmación? Explica
un limón? por qué sí o por qué no.

© Lopez Mateos Editores. ISBN 978-607-95583-2-1, obra completa, versión electrónica, ISBN 978-607-95583-3-8, volumen 1, versión electrónica. Ejemplar asignado a: Helecto Villarroel gutierrez -
helecto@gmail.com. Fecha: 27 de octubre de 2014. Prohibida su modificación, copia o distribución.
Sección 7-3 Decimales que no terminan 449

3. Algunos problemas de suma son más fáciles de calcular con nos una vez. Después de jugar, analicen en grupo una es-
fracciones y otros son más fáciles de hacer con decimales. trategia para formular a tu pareja el menor número de
1 5 preguntas que te permitan hallar el periodo buscado.
Por ejemplo, + es más fácil de calcular que 0.142857 +
7 7 Preguntas del salón de clase
2 1
0.714285 y 0.4 + 0.25 es más fácil de calcular que + . 9. Un estudiante argumenta que los decimales periódicos
5 4
son de poca utilidad pues ninguna calculadora realiza
Describe situaciones en las que pienses que sería más fácil cal-
cálculos con decimales periódicos, y que en la vida real
cular sumas con fracciones que con decimales, y viceversa.
los decimales no tienen un número infinito de dígitos.
Solución abierta ¿Cómo le respondes?
10. Susana dice que la base dos no puede tener decimales, ya
1 2 3
4. Nota que = 0.142857, = 0.285714, = 0.428571, sea periódicos o exactos. ¿Cómo la puedes convencer de
7 7 7 que eso no es cierto?
4 5 6
= 0.571428, = 0.714285 y = 0.857142. Problemas de repaso
7 7 7
a. Describe una propiedad común que comparten todos 11. Juan es pagador en una pequeña compañía. El mes pasado
estos decimales periódicos. las ganancias brutas (ganancias antes de deducciones) de
1 los empleados sumaron $27,849.50. Juan dedujo $1520.63
b. Supón que memorizas la forma decimal de . ¿Cómo
7 para el seguro social, $723.30 para el seguro de desempleo
podrías hallar rápidamente las respuestas para la expan- y $2843.62 para el impuesto federal sobre el ingreso. ¿Cuál
sión decimal del resto de las fracciones anteriores? Des- fue la paga neta (ganancias después de pagar las deduccio-
cribe todas las maneras que puedas. nes) de los empleados?
1
c. Halla otras fracciones que se comportan como . 12. La velocidad de la luz es de aproximadamente 300,000 km/s.
7
¿Por qué tienen un comportamiento similar? La luz emitida por la estrella más cercana, Alpha Centauri,
d. Con base en tu respuesta en (a), describe atajos para tarda aproximadamente 4 años en llegar a la Tierra. ¿A cuán-
k tos kilómetros está Alpha Centauri de la Tierra? Expresa la
escribir como un decimal periódico para k = 1,
14 respuesta en notación científica.
2, 3, Á , 13. 13. Halla el producto de 0.22 y 0.35 en una calculadora. ¿Cómo
se compara la colocación del punto decimal en la respuesta
5. a. Demuestra que todo entero se puede escribir como
de la calculadora con la colocación del punto decimal usando
un decimal.
la regla planteada en este capítulo? Explica.
b. Multiplica cada uno de los casos siguientes y da tu
14. Responde lo siguiente: .
respuesta como un decimal en su forma más simple.
a. Halla un número que sumado a - 0.023 sea mayor que
(Recuerda tu respuesta a la parte (a)).
3 pero menor que 4.
(i) 2 # 0.3 (ii) 3 # 0.3 (iii) 3 # 0.35
b. Halla un número que restado a - 0.023 genere una di-
c. Explica si el algoritmo tradicional para multiplicar
ferencia mayor que 3 pero menor que 4.
decimales se puede usar para multiplicar decimales
c. Halla un número que multiplicado por 0.023 genere
periódicos.
un producto mayor que 3 pero menor que 4.
d. Explica si se pueden multiplicar decimales periódicos.
d. Halla un número que dividido entre 0.023 produzca un
6. a. ¿Tu calculadora permite colocar decimales periódi-
cociente mayor que 3 pero menor que 4.
cos? Si es así, ¿en qué forma?
b. Explica si en tu calculadora es posible realizar aritmé- Preguntas del Third International Mathematics and
tica con decimales periódicos. Science Study (TIMSS) (Tercer Estudio Internacional
7. Explica cómo preferirías que se expresara la solución de sobre las Matemáticas y la Ciencia)
3x = 7: ¿como fracción o como un decimal periódico? ¿En qué lista están ordenados los números de mayor a
menor?
Aprendizaje colectivo a. 0.233, 0.3, 0.32, 0.332
8. Escoge una pareja para realizar el siguiente juego. Escribe b. 0.3, 0.32, 0.332, 0.233
un decimal periódico de la forma 0.abcdef . Di a tu pareja c. 0.32, 0.233, 0.332, 0.3
que el decimal está en esa forma pero no reveles los valo- d. 0.332, 0.32, 0.3, 0.233
res específicos de los dígitos. El objetivo de tu pareja es TIMSS, Grado 8, 2003
hallar tu decimal periódico. A tu pareja se le permite pre- ¿En cuál de los siguientes pares de números es 2.25 ma-
guntarte los valores de seis dígitos que estén en el lugar yor que el primer número pero menor que el segundo?
número 100, o mayores, a la derecha del punto decimal, 5
pero que no sean consecutivos. Por ejemplo, tu oponente a. 1 y 2 b. 2 y
2
puede preguntar por los dígitos ubicados en los lugares
5 11 11
100, 200, 300, … , pero no puede preguntar por los dígi- c. y d. y3
tos en los lugares 100 y 101. Intercambien papeles al me- 2 4 4
TIMSS, Grado 8, 2003
© Lopez Mateos Editores. ISBN 978-607-95583-2-1, obra completa, versión electrónica, ISBN 978-607-95583-3-8, volumen 1, versión electrónica. Ejemplar asignado a: Helecto Villarroel gutierrez -
helecto@gmail.com. Fecha: 27 de octubre de 2014. Prohibida su modificación, copia o distribución.
450 Decimales y números reales

Pregunta del National Assessment of Educational Progress


(NAEP) (Evaluación Nacional del Progreso Educativo)
La señora Pérez compró 6 medidas de fresas. Cada medida le
costó 87¢. La señora Pérez usó su calculadora para hallar el +/– % 4

costo de las fresas y la pantalla mostró 522. ¿Cuál fue el costo MRC M– M+ 3

de las fresas? 7 8 9 2

a. $522 4 5 6 1

b. $52.20 1 2 3 5

c. $5.22 ON/C 0 .

d. $0.52
NAEP, Grado 4, 2003

7-4 Números reales

Todo número racional se puede expresar ya sea como un número decimal exacto o como
uno periódico. Los antiguos griegos descubrieron números que no son racionales. Dichos
números deben tener una representación decimal que ni termina ni se repite. Para hallar
dichos decimales nos concentramos en las características que deben tener:
1. Deben tener un número infinito de dígitos distintos de cero a la derecha del punto decimal.
2. No se pueden repetir, no tienen periodo.
Una manera de construir un decimal que no termina ni se repite, es decir, ni exacto ni pe-
riódico, es diseñar un patrón infinito de dígitos en forma tal que, en definitiva, no haya pe-
riodo. Considera el número 0.1010010001 Á . Si continúa el patrón, los siguientes grupos
de dígitos tienen cuatro ceros seguidos de 1, cinco ceros seguidos de 1, y así sucesivamente.
Es posible describir un patrón para este decimal, pero no hay un grupo de dígitos que se re-
pita. Como este decimal no termina ni se repite, no puede representar un número racional.
Esos números que no son racionales son los números irracionales.
A mediados del siglo dieciocho se demostró que la razón de la circunferencia de un círculo
22
a su diámetro, simbolizada por P (pi), es un número irracional. Los números , 3.14 ó
7
3.14159 son números racionales aproximaciones de p. El valor de p se ha calculado hasta un
billón de lugares decimales sin hallar un patrón aparente.

Raíces cuadradas
Los números irracionales se presentan en el estudio de áreas. Por ejemplo, para hallar el área de
un cuadrado usamos la fórmula A = l 2, donde A es el área y l es la longitud del lado al cuadrado.
Si un lado de un cuadrado mide 3 cm de longitud, entonces el área del cuadrado es de 9 cm2 .

◆ Nota Diez investigadores, incluyendo al profesor Yasumasa Kanada, del Information Technology
histórica Center de la Universidad de Tokyo calcularon el valor de pi hasta 1.2411 billones de lugares
decimales en una supercomputadora Hitachi, en septiembre de 2002. La computadora
tardó 600 horas en efectuar el cálculo.
Pi es el tema de varios libros para jóvenes estudiantes, incluyendo Sir Cumference and the
Dragon of Pi: A Math Adventure (Neuschwander 1999). ◆
© Lopez Mateos Editores. ISBN 978-607-95583-2-1, obra completa, versión electrónica, ISBN 978-607-95583-3-8, volumen 1, versión electrónica. Ejemplar asignado a: Helecto Villarroel gutierrez -
helecto@gmail.com. Fecha: 27 de octubre de 2014. Prohibida su modificación, copia o distribución.
Sección 7-4 Números reales 451

(centímetros cuadrados). Recíprocamente, podemos usar la fórmula para hallar la longitud


de un lado de un cuadrado dada su área. Si el área de un cuadrado es de 25 cm2, entonces
l 2 = 25, de modo que l = 5 ó - 5. Cada una de estas soluciones es una raíz cuadrada de
25. Sin embargo, como las longitudes siempre son no negativas, 5 es la única solución posi-
ble. La solución positiva de l 2 = 25 (a saber, 5) es la raíz cuadrada principal de 25 y se de-
nota con 125. De manera análoga, la raíz cuadrada principal de 2 se denota con 12. Nota
que 116 Z - 4 porque - 4 no es la raíz cuadrada principal de 16. ¿Puedes hallar 1 0 ?

Definición de la raíz cuadrada principal


Si a es cualquier número no negativo, la raíz cuadrada principal de a (denotada con 1a ) es el
número no negativo b tal que b 2 = a.

Ejemplo 7-13 Halla lo siguiente:










a. Las raíces cuadradas de 144


b. La raíz cuadrada principal de 144
4
c.
A9

Solución a. Las raíces cuadradas de 144 son 12 y - 12.


b. La raíz cuadrada principal de 144 es 12.
4 2
c. =
A9 3

Otras raíces
Hemos visto que la solución positiva de l 2 = 25 se denota con 125. De manera análoga, la
solución positiva de l4 = 25 se denota 2 4 25. En general, si n es par, la solución no negativa
n
de xn = 25 es 225 y es la n-ésima raíz principal de 25. El número n es el índice. Nota
que en la expresión 125, el índice 2 se sobreentiende y no se expresa. En general, la solu-
n
ción no negativa de xn = b, donde b es no negativa, es 2b.
n n
Substituyendo 2b por x en la ecuación x = b se tiene lo siguiente:
12b2n = b
n

n
Si b es negativo, 2b puede no ser un número real. Por ejemplo, considera 2 4 - 16. Si
24 - 16 = x, entonces x = - 16. Como cualquier número real distinto de cero elevado a la
4

cuarta potencia es positivo, no hay solución en números reales a x4 = - 16 y, por lo tanto,

◆ Nota Es posible que los sabios hindú-vedas supieran evaluar raíces cuadradas antes de 600 a.C.
histórica Los textos sánscritos Sulbasutra (reglas de las cuerdas) contienen aproximaciones de algunas
raíces cuadradas que son asombrosamente precisas. El descubrimiento de los números irra-
cionales por miembros de la Sociedad Pitagórica fue muy perturbador, de modo que deci-
dieron mantener el asunto en secreto. Cierta leyenda dice que un miembro de la sociedad
fue ahogado por haber transmitido el secreto a personas ajenas a la sociedad. En 1525,
Christoff Rudolff, un matemático alemán, fue el primero en usar el símbolo 1 como
radical o raíz. ◆

© Lopez Mateos Editores. ISBN 978-607-95583-2-1, obra completa, versión electrónica, ISBN 978-607-95583-3-8, volumen 1, versión electrónica. Ejemplar asignado a: Helecto Villarroel gutierrez -
helecto@gmail.com. Fecha: 27 de octubre de 2014. Prohibida su modificación, copia o distribución.
452 Decimales y números reales

24 - 16 no es un número real. De manera análoga, si nos restringimos a los números reales,


no es posible hallar cualquier raíz par (de índice par) de un número negativo. Sin embargo,
el valor - 2 satisface la ecuación x3 = - 8. Por lo tanto, 2 3 - 8 = - 2. En general, la raíz im-
par (de índice impar) de un número negativo es un número negativo.
Como 1a, si existe, es no negativa por definición, 21- 322 = 19 = 3 y no - 3. Mu-
chos estudiantes piensan que 2a2 siempre es igual a a. Esto sucede si a Ú 0, pero es falso
si a 6 0. En general, 2a2 = ƒ a ƒ .
De manera análoga, 2 4 a4 = ƒ a ƒ y 2
6 a6 = ƒ a ƒ , pero 2
3 a3 = a para todo a (¿por qué?).
n
Nota que cuando n es par y b 7 0, la ecuación x = b tiene dos soluciones que son núme-
n n
ros reales, 2b y - 2b. Si n es impar o cero, la ecuación tiene sólo una solución que es un
n
número real, 2b, para cualquier número real b.

Irracionalidad de las raíces cuadradas y de otras raíces


Algunas raíces cuadradas son números racionales. Otras, como 1 2, son números irracio-
nales. Para ver esto, nota que 12 = 1 y 22 = 4 y que no hay un número completo s tal que
a 2
s 2 = 2. ¿Hay un número racional tal que a b = 2? Usamos la estrategia del razonamiento
a
b b
a
indirecto. Si suponemos que existe dicho número racional , entonces lo siguiente debe ser cierto:
b
a 2
a b = 2
b
a2
= 2
b2
a2 = 2b2
2 2
Si a = 2b , entonces, por el teorema fundamental de la aritmética, las factorizaciones en pri-
mos de a2 y 2b2 deben ser iguales. En particular, el primo 2 aparece el mismo número de veces
en la factorización en primos de a2 que en la de 2b2. Como b2 = bb, no importa cuántas veces
aparezca el 2 en la factorización en primos de b, aparecerá el doble de veces en bb.
Además, a2 tiene una cantidad par de 2 por la misma razón que los tiene b2. En 2b2, se in-
troduce otro factor de 2, resultando en un número impar de 2 en la factorización en primos
de 2b2 y, por lo tanto, de a2. Pero el 2 no puede aparecer una cantidad impar de veces y una
cantidad par de veces en la misma factorización en primos de a2. Tenemos una contradicción.
Esta contradicción sólo podría haber surgido por asumir la hipótesis de que 12 es un nú-
mero racional. En consecuencia, 12 debe ser un número irracional. Podemos usar un ar-
gumento similar para mostrar que 13 es irracional o 1n es irracional, donde n es un
número completo que no es el cuadrado de otro número completo.
Muchos números irracionales se pueden interpretar geométricamente. Por ejemplo, po-
demos hallar un punto sobre una recta numérica que represente 12 usando el teorema de
Pitágoras. Esto es, si a y b son las longitudes de los lados más cortos (los catetos) de un
triángulo rectángulo y c es la longitud del lado más largo (la hipotenusa), entonces
a2 + b2 = c 2, como se muestra en la figura 7-6.

c
a a2 + b2 = c2

Figura 7-6
© Lopez Mateos Editores. ISBN 978-607-95583-2-1, obra completa, versión electrónica, ISBN 978-607-95583-3-8, volumen 1, versión electrónica. Ejemplar asignado a: Helecto Villarroel gutierrez -
helecto@gmail.com. Fecha: 27 de octubre de 2014. Prohibida su modificación, copia o distribución.
Sección 7-4 Números reales 453

La figura 7-7 muestra un segmento de 1 unidad de longitud construida perpendicular-


mente a una recta numérica en un punto P. Así, los dos lados del triángulo mostrados mi-
den cada uno 1 unidad de longitud. Si a = b = 1, entonces c 2 = 2 y c = 12. Para hallar un
punto sobre la recta numérica que corresponda a 12, necesitamos hallar un punto Q sobre
la recta numérica tal que la distancia de 0 a Q sea 12. Como 12 es la longitud de la hipo-
tenusa, el punto Q se puede hallar marcando un arco con centro en 0 y radio c. La intersec-
ción de la parte positiva de la recta real con el arco es Q.

c
1

21 0 P1 Q 2

Figura 7-7

De manera análoga se pueden construir otras raíces cuadradas, como se muestra en la


figura 7-8.

1 1

1
1
4 3
2
5
1
6

Figura 7-8

Estimación de una raíz cuadrada


De la figura 7-7 vemos que 12 debe tener un valor entre 1 y 2; esto es, 1 6 12 6 2. Para
obtener una aproximación más cercana de 12, intentamos “atrapar” 12 entre dos núme-
ros que estén entre 1 y 2. Como 11.522 = 2.25 y 11.422 = 1.96, se sigue que
1.4 6 12 6 1.5. Como a2 se puede interpretar como el área de un cuadrado con longitud
de lado a, este análisis se puede ilustrar geométricamente como en la figura 7-9.

1.5
2 1.4

1.4
2
1.5

Figura 7-9

© Lopez Mateos Editores. ISBN 978-607-95583-2-1, obra completa, versión electrónica, ISBN 978-607-95583-3-8, volumen 1, versión electrónica. Ejemplar asignado a: Helecto Villarroel gutierrez -
helecto@gmail.com. Fecha: 27 de octubre de 2014. Prohibida su modificación, copia o distribución.
454 Decimales y números reales

Si deseamos una aproximación más precisa para 12, continuamos el proceso de atrapar.
Vemos que 11.422, ó 1.96, está más cerca de 2 que 11.522, ó 2.25, de modo que escogemos
números más cercanos a 1.4 para hallar la siguiente aproximación. Hallamos lo siguiente:
11.4222 = 2.0164
11.4122 = 1.9981
Así, 1.41 6 12 6 1.42. Podemos continuar este proceso hasta obtener la aproximación
deseada. Nota que si tu calculadora tiene una tecla de raíz cuadrada, puedes obtener direc-
tamente la aproximación.

AHORA INTENTA ÉSTE 7-12 Un algoritmo para calcular raíces cuadradas se atribuye con frecuencia a
Arquímedes. El algoritmo consiste en obtener estimados más y más cercanos a la raíz cuadrada. Para ha-
llar la raíz cuadrada de un número positivo, n, primero se hace una propuesta. Se le llama Propuesta 1.
Ahora calculamos como sigue:
Paso 1: Divide n entre la Propuesta 1.
Paso 2: Ahora suma la Propuesta 1 al cociente obtenido en el paso 1.
Paso 3: Divide la suma del paso 2 entre 2. El cociente se convierte en la Propuesta 2.
Repite los pasos usando la Propuesta 2 para obtener propuestas sucesivas o hasta lograr la precisión deseada.
a. Usa el método descrito para hallar la raíz cuadrada de 13 hasta cuatro lugares decimales.
b. Escribe los pasos para el algoritmo en una fórmula recursiva.

El sistema de los números reales


El conjunto de números reales R es la unión del conjunto de los números racionales y el
conjunto de los números irracionales. Los números reales representados como decimales
pueden terminar, repetirse o no terminar ni repetirse.
Todo entero es un número racional así como un número real. Todo número racional es
un número real, pero no todo número real es racional, como se mostró con 12. La rela-
ción entre conjuntos de números se resume en el árbol de la figura 7-10.

Reales = 5x ƒ x es un decimal6

Irracionales = 5x ƒ x es un decimal que Racionales = e x ƒ x = donde a, b 僆 E y b Z 0 f


a
b
no se repite = 5x ƒ x es un decimal periódico o exacto6
ni se termina6
Enteros = 5 Á , - 3, - 2, - 1, 0, 1, 2, 3, Á 6

Números completos = 50, 1, 2, 3, 4, Á 6

Números naturales = 51, 2, 3, 4, 5, Á 6

Figura 7-10
a
El concepto de fracciones puede extenderse ahora para incluir todos los números de la forma ,
b
13
donde a y b son números reales con b Z 0, tales como . La suma, resta, multiplicación y
5
división están definidas en el conjunto de los números reales de manera que todas las propiedades
de estas operaciones sobre los racionales mantienen su validez. Las propiedades se resumen a
continuación.
© Lopez Mateos Editores. ISBN 978-607-95583-2-1, obra completa, versión electrónica, ISBN 978-607-95583-3-8, volumen 1, versión electrónica. Ejemplar asignado a: Helecto Villarroel gutierrez -
helecto@gmail.com. Fecha: 27 de octubre de 2014. Prohibida su modificación, copia o distribución.
Sección 7-4 Números reales 455

Teorema 7–2:    Propiedades de los números reales


Propiedades de cerradura Para números reales a y b, a + b y ab son números reales únicos.
Propiedades conmutativas Para números reales a y b, a + b = b + a y ab = ba.
Propiedades asociativas Para números reales a, b y c, a + 1b + c2 = 1a + b2 + c y
a1bc2 = 1ab2c.
Propiedades de identidad El número 0 es la única identidad aditiva y 1 es la única identidad
multiplicativa tales que, para cualquier número real a, 0 + a = a = a + 0 y 1 # a = a = a # 1.
Propiedades de inverso (1) Para todo número real a, - a es su único inverso aditivo; esto es,
1
a + - a = 0 = - a + a. (2) Para todo número real a distinto de cero, es su único inverso mul-
a
tiplicativo; esto es, a a b = 1 = a ba.
1 1
a a
Propiedad distributiva de la multiplicación sobre la suma Para números reales a, b y c,
a1b + c2 = ab + ac.
Propiedad de la densidad Para números reales a y b, existe un número real c tal que a 6 c 6 b.

Radicales y exponentes racionales


Las calculadoras científicas tienen una tecla y x con la cual podemos hallar los valores de expre-
siones como 3.412>3 y 41>2. ¿Qué significa 41>2? Al extender las propiedades de los exponentes
previamente desarrolladas para exponentes enteros,

41>2 # 41>2 = 411>2 + 1>22

Así,
141>222 = 41 = 4

y
41>2 = 14

Se supone que el número 41>2 es la raíz cuadrada principal de 4, esto es, 41>2 = 14.
En general, si x es un número real no negativo, entonces x1>2 = 1x. Análogamente, 1x1>323 =
x11>323 = x1 y x1>3 = 2 3 x. Este análisis conduce a lo siguiente:
n n
1. x1>n = 2x, donde 2x tenga sentido.
2. 1xm21>n = 2xm si mdc1m, n2 = 1.
n

3. xm>n = 2xm si mdc1m, n2 = 1


n

◆ Nota En 1874 Georg Cantor demostró que el conjunto de todos los números reales no puede
histórica ponerse en correspondencia biunívoca con el conjunto de los números naturales, y en el pro-
ceso continuó comparando infinitos. ◆

© Lopez Mateos Editores. ISBN 978-607-95583-2-1, obra completa, versión electrónica, ISBN 978-607-95583-3-8, volumen 1, versión electrónica. Ejemplar asignado a: Helecto Villarroel gutierrez -
helecto@gmail.com. Fecha: 27 de octubre de 2014. Prohibida su modificación, copia o distribución.
456 Decimales y números reales

Más propiedades de los exponentes


Es posible demostrar que las propiedades de los exponentes enteros también valen para ex-
ponentes racionales. Estas propiedades son equivalentes a las propiedades correspondientes
de los radicales si las expresiones con radicales tienen sentido.
Sean r y s números racionales cualesquiera, x y y números reales cualesquiera, y n cual-
quier entero distinto de cero. ¿Cuáles son las restricciones en las variables en cada uno de
los casos siguientes?
- 1
a. x r = r
x
b. 1xy2r = xryr implica 1xy21>n = x1>ny1>n y 2xy = 2x 2y
n n n

n
x r xr x 1>n x1>n n x 2x
c. a b = r implica a b = 1>n y = n
y y y y Ay 2y
d. 1xr2s = xrs implica 1x1>n2s = x s>n y, por lo tanto, 12x2s = 2xs
n n

Las propiedades anteriores se pueden usar para escribir expresiones equivalentes de las raíces
de muchos números. Por ejemplo, 196 = 116 # 6 = 116 # 16 = 4 # 16. Análogamente,
23
54 = 2 3 27 # 2 = 23 27 # 23 2 = 3# 23 2.

Ejemplo 7-14 Simplifica cada caso si es posible:


c. 1 - 821>3 e. 1 - 1621>4








-4>3
a. 161>4 b. 165>4 d. 125

Solución a. 161>4 = 12421>4 = 21 = 2, ó 161>4 = 2 4 16 = 2


11>425
b. 16 5>4
= 16 = 116 2 = 2 = 32
1>4 5 5

c. 1 - 821>3 = 11 - 22321>3 = 1 - 221 = - 2 ó 1 - 821/3 = 2


3 -8 = -2

. 125 4>3 = 1532 4>3 = 5 4 = 4 =


- - - 1 1
5 625
e. Como todo número real elevado a la cuarta potencia es positivo, 24 - 16 no es
un número real. En consecuencia, 1 - 162 no es un número real.
1>4

AHORA INTENTA ÉSTE 7-13 Calcula 2


8 10 en una calculadora usando la siguiente sucesión de teclas:
10 1 1 1
a. Explica por qué funciona este enfoque.
n
b. ¿Para qué valores de n puede calcularse 210 usando sólo la tecla 1 ? ¿Por qué?

Evaluación 7-4A

1. Escribe un número irracional cuyos dígitos sean 2 y 3.


2. Usa el teorema de Pitágoras para hallar x. 3. Arregla los números reales siguientes en orden decreciente:
a. b. c.
2
0.9, 0.9, 0.98, 0.988, 0.998, 0.898, 20.98
x
x
4. Determina cuáles de los casos siguientes representan nú-
2 4 5
meros irracionales:
a. 151 b. 164 c. 1324
2
d. 1325 e. 2 + 3 12 f. 12 , 5
2 x
© Lopez Mateos Editores. ISBN 978-607-95583-2-1, obra completa, versión electrónica, ISBN 978-607-95583-3-8, volumen 1, versión electrónica. Ejemplar asignado a: Helecto Villarroel gutierrez -
helecto@gmail.com. Fecha: 27 de octubre de 2014. Prohibida su modificación, copia o distribución.
Sección 7-4 Números reales 457

5. De ser posible, halla las raíces cuadradas de cada caso sin 16. Determina para qué valores de x, si los hay, es verdadera
usar calculadora: cada una de las afirmaciones siguientes:
a. 225 b. 169 a. 1x = 8 b. 1x = - 8
c. - 81 d. 625 -
c. 2 x = 8 d. 2 - x = - 8
6. Halla las raíces cuadradas aproximadas para cada uno de los e. 1x 7 0
casos siguientes, redondeados a las centésimas, usando el f. 1x 6 0
método del “refinamiento”: 17. Se coloca un soporte diagonal en una puerta rectangular
a. 7 b. 0.0120 de 4 pies * 5 pies. ¿Cuál es la longitud del soporte a la
7. Clasifica cada caso como verdadero o falso. Si es falso, décima de pie más cercana? (Sugerencia: Usa el teorema
exhibe un contraejemplo. de Pitágoras.)
a. La suma de cualquier número racional y cualquier 18. Escribe cada una de las raíces cuadradas siguientes en la
número irracional es un número racional. forma a 1b, donde a y b son enteros y b tiene el mínimo
b. La suma de dos números irracionales cualesquiera es valor posible:
un número irracional. a. 1180 b. 1363
c. El producto de dos números irracionales cualesquiera c. 1252
es un número irracional. 19. Escribe cada caso en la forma más simple o como a2b,
n

d. La diferencia de dos números irracionales cuales- donde a y b son enteros, b 7 0 y b tiene el mínimo valor
quiera podría ser un número racional. posible:
8. Halla tres números irracionales entre 1 y 3.
9. Halla un número irracional entre 0.53 y 0.54. 3 - 54
a. 2 b. 2
5 96
10. Entre los decimales exactos 0.5 y 0.6, halla un número c. 2 3 250 5 - 243
d. 2
irracional. 20. En cada una de las sucesiones geométricas siguientes,
11. ¿Qué tipo de número resulta si sumamos un número ra- halla los términos faltantes:
cional a un número irracional? Demuestra tu respuesta.
a. 5, , , 10
12. Con base en tu respuesta al problema 11, prueba que hay
infinidad de números irracionales. b. 2, , , ,1
13. Si R es el conjunto de números reales, Q es el conjunto 21. La siguiente función exponencial aproxima el número de
de números racionales, E es el conjunto de los enteros, C bacterias después de t horas: E1t2 = 210 # 16t.
es el conjunto de los números completos y S es el con- a. ¿Cuál es el número inicial de bacterias, esto es, el nú-
junto de los números irracionales, halla lo siguiente: mero cuando t = 0 ?
a. Q ´ S b. Q ¨ S c. Q ¨ R 1
b. ¿Cuántas bacterias hay después de de hora?
d. S ¨ C e. C ´ R f. Q ´ R 4
14. Si las letras siguientes corresponden a los conjuntos listados 1
en el problema 13, completa la tabla marcando las columnas c. ¿Cuántas bacterias hay después de de hora?
2
adecuadas. (N es el conjunto de números naturales.) 22. Despeja x en los casos siguientes, donde x es un número
racional:
N E Q R S
a. 3x = 81 b. 4x = 8
a. 6.7 ✔ ✔
4 3x
d. a b =
- 32
b. 5 c. 128 x = 16
9 243
c. 12 23. Clasifica cada uno de los números siguientes como ra-
d. - 5 cional o irracional:
b. 1122 4
1 2 -
e. 3 a. 12 -
7 12
24. a. ¿Alguna vez es cierto que 2a2 = - a ? Explica tu res-
15. Si las letras siguientes corresponden a los conjuntos listados puesta.
en el problema 13, marca la casilla adecuada si existe solución b. ¿Para qué valores de x, si los hay, es cierto que
al problema. (N es el conjunto de números naturales.) (i) x 2 Ú 2 ?
(ii) x 2 Ú - 2?
N E Q R S 25. En la figura 7-8 se mostró una manera de construir la raíz
a. x 2 + 1 = 5 cuadrada de cualquier entero positivo. En la figura 7-7 se
b. 2x - 1 = 32 mostró cómo colocar la raíz cuadrada de un entero positivo
c. x 2 = 3 sobre una recta numérica. Usando estos métodos, describe
dónde estaría 1342 sobre una recta numérica.
d. x 2 = 4 夝26. Usa los pasos para la fórmula recursiva hallada en Ahora
e. 1x = - 1 intenta éste 7-12 para mostrar que el algoritmo podría
3 producir la raíz cuadrada de un número real positivo.
f. x = 4
4
© Lopez Mateos Editores. ISBN 978-607-95583-2-1, obra completa, versión electrónica, ISBN 978-607-95583-3-8, volumen 1, versión electrónica. Ejemplar asignado a: Helecto Villarroel gutierrez -
helecto@gmail.com. Fecha: 27 de octubre de 2014. Prohibida su modificación, copia o distribución.
458 Decimales y números reales

Evaluación 7-4B

1. Escribe un número irracional cuyos dígitos sean 4 y 5. 10. Si R es el conjunto de los números reales, Q es el con-
2. Usa el teorema de Pitágoras para hallar x. junto de los números racionales, E es el conjunto de los
a. b. enteros, C es el conjunto de los números completos, N es
el conjunto de los números naturales y S es el conjunto
x
de los números irracionales, halla lo siguiente:
3 a. Q ¨ E b. S - Q c. R ´ S
3 x d. ¿Cuál de los conjuntos podría ser un conjunto uni-
versal para el resto de los conjuntos?
11. Si R es el conjunto de los números reales, ¿cómo describi-
3
rías S, donde S es el conjunto de los números irracionales?
12. Si las letras siguientes corresponden a los conjuntos lis-
3
tados en el ejercicio 10, completa la tabla siguiente mar-
c. 7 cando las columnas adecuadas:

N E Q R S
24 25 a. 13
1
b. 4 2
c. - 3
1
7
x
13. Si las letras siguientes corresponden a los conjuntos lis-
3. Arregla los números reales siguientes en orden decreciente: tados en el problema 10, marca la casilla adecuada si
existe solución al problema:
0.8, 0.8, 0.89, 0.889, 10.7744
4. Determina cuáles de los casos siguientes representan nú- N E Q R S
2
meros irracionales: a. x + 2 = 4
a. 178 b. 181 c. 23 343 b. 1 - 2x = 32
d. 3 + 181 e. 2 , 12 3
c. x = 4
5. De ser posible, halla los números completos que son raí-
d. 1x = - 2
ces cuadradas de cada caso, sin usar calculadora:
a. 256 b. 324 e. 0.7x = 5
c. - 25 d. 1024
14. Determina para qué valores reales de x, si los hay, es ver-
6. Halla las raíces cuadradas aproximadas para cada uno de los
dadera cada una de las afirmaciones siguientes:
casos siguientes, redondeados a las centésimas, usando el
a. 1x = 7 b. 1x = - 7
método del “refinamiento”:
c. 2 - x = 7 d. - 1x = 7
a. 20.3 b. 1.64
e. - 1x = - 7
7. Clasifica cada caso como verdadero o falso. Si es falso,
15. Escribe cada una de las raíces cuadradas siguientes en la
exhibe un contraejemplo.
forma a 1b, donde a y b son enteros y b tiene el mínimo
a. La suma de dos números racionales cualesquiera es
valor posible:
un número racional.
a. 1360 b. 140
b. La diferencia de dos números irracionales cuales-
c. 1240
quiera es un número irracional. n
16. Escribe cada caso en la forma más simple o como a 2b,
c. El producto de cualquier número racional por cual-
donde a y b son enteros, b 7 0 y b tiene el mínimo valor
quier número irracional es un número irracional.
posible:
8. Halla tres números irracionales entre cada uno de los pa-
res siguientes: a. 23 - 102 b. 2 6 64
a. 3 y 4 b. 7.05 y 7.06 c. 2 3 64
c. 0.7 y 0.8 17. En cada una de las sucesiones geométricas siguientes,
9. Sabiendo que 12 es un número irracional, deduce que halla los términos faltantes:
12 a. 4, , ,8
también es un número irracional.
2 b. 1, , ,2
18. Si los dos lados más cortos de un triángulo rectángulo mi-
den 5 y 12, ¿cuánto mide la hipotenusa (el lado mayor)?
© Lopez Mateos Editores. ISBN 978-607-95583-2-1, obra completa, versión electrónica, ISBN 978-607-95583-3-8, volumen 1, versión electrónica. Ejemplar asignado a: Helecto Villarroel gutierrez -
helecto@gmail.com. Fecha: 27 de octubre de 2014. Prohibida su modificación, copia o distribución.
Sección 7-4 Números reales 459

19. En una función exponencial, E1t2 = 8t, donde t repre- 21. Clasifica cada uno de los números siguientes como
senta el tiempo en horas. racional o irracional:
a. ¿Es lineal la gráfica de la función exponencial? Traza 1 4
a. b. - 12
una figura para ilustrar tu respuesta. 1 + 12 12
b. ¿Cuál es el valor de E102?
22. Explica en qué caso 2a2 = - ƒ a ƒ .
c. Después de de hora, ¿cuál es el valor de E a b ?
1 1
1
3 3 23. Si a 7 2, describe la posición aproximada de sobre
20. Despeja x en los casos siguientes, donde x es un número 1a
real: una recta numérica.
a. 2x = 64 b. 4x = 64 2 3
24. Describe cómo verificarías si es cierto que 6 .
- 3x 13 15
c. 2 x = 64 d. = 36
A2

Conexiones matemáticas 7-4

Comunicación Aprendizaje colectivo


1. Un matemático describió en alguna ocasión al conjunto 10. Que cada miembro del grupo escoja un número entre 0 y
de los números racionales como las estrellas y al de los nú- 1 en una calculadora y verifique lo que sucede cuando se
meros irracionales como el negro del cielo nocturno. presiona la tecla x 2 sucesivamente hasta que se ve claro
¿Qué piensas que quiso decir el matemático? que no tiene sentido continuar.
2. Halla el valor de 13 en una calculadora. Explica por qué a. Comparen sus respuestas y emitan una conjetura con
no puede ser el valor exacto de 13. base en lo observado.
3. ¿Es cierto que 1a + b = 1a + 1b para toda a y b? Ex- b. Usen de manera similar otras teclas de la calculadora.
plica. Describan el proceso y emitan una conjetura corres-
22 pondiente.
4. Pi (p) es un número irracional. ¿Podría p = ? ¿Por c. ¿Por qué se obtienen esos resultados en la parte (a) y (b)?
7
qué sí o por qué no? 11. Una calculadora presenta lo siguiente: 13.722.4 = 23.103838.
5. Sin usar una calculadora y sin hacer cálculos, determina si Analicen en grupo el significado de la expresión 13.722.4
113 = 3.605. Explica por qué sí o por qué no. en vista de lo que saben acerca de exponentes. Comparen
6. ¿Es 2x 2 + y 2 = x + y para todos los valores de x y y? sus hallazgos con los de otros grupos.
Explica tu razonamiento. Preguntas del salón de clase
7. Sin usar una calculadora, ordena los números siguientes
de manera creciente. Explica tu razonamiento. 12. Una estudiante argumenta que la única solución a x2 = 4
es 2 porque 14 es igual a 2. ¿Cómo le respondes?
14>252 , 125>421>3, 14>252
-1>3 -1>4
12
13. Jaime preguntó que si 12 se puede escribir como ,
1
Solución abierta ¿por qué no es racional? ¿Cómo le responderías?
8. La sucesión 1, 1.01, 1.001, 1.0001, Á es una sucesión in- 14. María dice que 1 + 12 no es un número pues no se
finita de números racionales. puede completar la suma. ¿Cómo le respondes?
a. Escribe otras sucesiones infinitas de números racio- 15. Una estudiante asegura que 5 = - 5 pues ambos son raíz
nales. cuadrada de 25. ¿Cómo la ayudarías?
b. Escribe una sucesión infinita de números irracionales. 16. José dice que la ecuación 2 - x = 3 no tiene solución
1 3 pues no existe la raíz cuadrada de un número negativo.
9. a. Coloca cinco números irracionales entre y . ¿Por qué está mal el argumento?
2 4
b. Escribe una sucesión infinita de números irracionales 17. Un estudiante asegura que en el mundo real nunca se usan
1 3 los números irracionales. ¿Qué le respondes?
cuyos términos estén entre y .
2 4

© Lopez Mateos Editores. ISBN 978-607-95583-2-1, obra completa, versión electrónica, ISBN 978-607-95583-3-8, volumen 1, versión electrónica. Ejemplar asignado a: Helecto Villarroel gutierrez -
helecto@gmail.com. Fecha: 27 de octubre de 2014. Prohibida su modificación, copia o distribución.
460 Decimales y números reales

18. Alfonso dice que hay infinidad de números irracionales 23. Escribe 0.00024 como una fracción en su forma más simple.
p 24. Escribe 0.24 como una fracción en su forma más simple.
entre 0 y 1 porque , donde n Ú 4 y n es un número na-
n 25. Escribe cada caso como un numeral común:
a. 2.08 # 105 b. 3.8 # 10 4
-
tural, está en el intervalo. ¿Está equivocado Alfonso?
19. La investigación de M. Brown de 1981 afirmaba que los
estudiantes tienen muchas más dificultades con los cálcu- Pregunta del National Assessment of Educational Progress
los involucrados en problemas que narran situaciones si se (NAEP) (Evaluación Nacional del Progreso Educativo)
incluyen números decimales. ¿Cuál es tu reacción a esa
investigación?
Problemas de repaso Término 1 2 3 4
20. a. Los huesos humanos son el 0.18 del peso de una per- Fracción 1>2 2>3 3>4 4>5
sona. ¿Cuánto pesan los huesos de una persona de
120 lb?
Si la anterior lista de fracciones continúa siguiendo el
b. Los músculos son el 0.4 del peso de una persona.
mismo patrón, ¿qué término será igual a 0.95?
¿Cuánto pesan los músculos de una persona de 120 lb?
a a. El 100-ésimo
21. Escribe cada uno de los decimales siguientes en la forma , b. El 95-ésimo
b
donde a, b 僆 E, b Z 0: c. El 20-ésimo
a. 16.72 b. 0.003 -
c. 5.07 d. 0.123 d. El 19-ésimo
22. Escribe un decimal periódico igual a cada uno de los si- e. El 15-ésimo
guientes números sin usar más de un cero:
1 NAEP, Grado 8, 2003
a. 5 b. 5.1 c.
2

ROMPECABEZAS Varias escuelas celebran el día de Pi el 14 de marzo. Usa la caricatura para sugerir por qué se usa
ese día y a qué hora comienza la celebración. (En inglés el nombre de la letra Pi suena pai.)

LE LLAMAMOS
“PI DE CALABAZA”.
PONGO ESTO AL FINAL HASTA
QUE COMPREMOS MÁS.

© Lopez Mateos Editores. ISBN 978-607-95583-2-1, obra completa, versión electrónica, ISBN 978-607-95583-3-8, volumen 1, versión electrónica. Ejemplar asignado a: Helecto Villarroel gutierrez -
helecto@gmail.com. Fecha: 27 de octubre de 2014. Prohibida su modificación, copia o distribución.
Sección 7-5 Uso de los números reales en ecuaciones 461

*7-5 Uso de los números reales en ecuaciones

En los Principios y objetivos hallamos lo siguiente:

Las tecnologías electrónicas —calculadoras y computadoras— son herramientas esenciales para la


enseñanza, el aprendizaje y la producción en matemáticas. Proporcionan imágenes visuales de ideas
matemáticas, facilitan la organización y el análisis de datos y realizan cálculos de manera eficiente y
precisa. Pueden apoyar la investigación de los estudiantes en cada área de las matemáticas, inclu-
yendo geometría, estadística, álgebra, medición y números. (p. 24)

Además, en los Principios y objetivos hallamos:

Las computadoras y calculadoras modifican las actividades que los estudiantes pueden realizar con
representaciones convencionales y extienden el conjunto de representaciones con las cuales pue-
◆ Nota den trabajar. . . . Pueden usar sistemas de álgebra computacional para manipular expresiones y pue-
den investigar conjuntos complejos de datos por medio de hojas de cálculo. En tanto los
de investigación estudiantes aprendan a usar estas nuevas y versátiles herramientas, también podrán comprender
Los estudiantes las maneras en que algunas representaciones usadas en la tecnología electrónica difieren de las re-
pueden aprender más presentaciones convencionales. (pp. 68–69)
matemáticas y de man-
En esta sección extendemos lo que los estudiantes han estudiado en las secciones previas y
era más profunda con comenzamos a explorar algebraicamente algunos temas usando la tecnología de las hojas de
el uso apropiado de cálculo. Quienes estudien esta sección lo pueden hacer sin la tecnología, pero como lo in-
tecnología dica la Nota de investigación, hay conocimientos que se pueden adquirir con mayor profun-
(Dunham and Dick, didad con el uso de la tecnología.
1994; Sheets, 1993; En los capítulos anteriores hemos visto relaciones lineales cuyo dominio era el conjunto
Boers-van Oosterum, de los números completos o el conjunto de los enteros, pero no cuando el dominio era todo
1990; Rojano, 1996; el conjunto de los números reales. Por ejemplo, si en la figura 7-11 usamos sólo enteros,
Groves, 1994). ◆ parte de la gráfica de la recta que representa la relación y = 2x + 3 se verá como sigue.

12

10

0
–6 –4 –2 0 2 4 6
–2

–4

Figura 7-11

Con todos los números reales, la recta completa de la figura 7-11 se puede ver como en la
figura 7-12.

© Lopez Mateos Editores. ISBN 978-607-95583-2-1, obra completa, versión electrónica, ISBN 978-607-95583-3-8, volumen 1, versión electrónica. Ejemplar asignado a: Helecto Villarroel gutierrez -
helecto@gmail.com. Fecha: 27 de octubre de 2014. Prohibida su modificación, copia o distribución.
462 Decimales y números reales

12

10

0
–6 –4 –2 0 2 4 6
–2

–4

Figura 7-12

RINCÓN DE LA TECNOLOGÍA Abre una nueva hoja de cálculo y en la columna A coloca 1 en la celda
A1. En la celda A2, teclea =A1 +1 y usa el comando Llenar abajo para ver los primeros 20 elementos de esa co-
lumna. En la celda B1, teclea =2*A1 +3 y oprime Intro. El uso de la característica función permite leer el valor
de la variable A1 en la celda A1, insertarlo en la fórmula dada y obtener el resultado 5 como 2(12 + 3. Pica en la
celda B1. Usa el comando Llenar abajo para tantos valores de esta columna como se necesiten para ha-
cerlos corresponder con los de la columna A.
Para producir una gráfica de los datos en las columnas, selecciona los elementos en ambas columnas y
usa la barra de herramientas para localizar la sección de Gráficas de la barra. Escoge el tipo de gráfica en
que quieras presentar los datos. En la figura 7-12 anterior se escogió la opción de recta. Una vez seleccio-
nada la opción, se traza la gráfica. Usa este procedimiento para graficar y = - 2x + 5.

Trazo de otras funciones en una hoja de cálculo


En la figura 7-13 vemos la gráfica de y = x2 y de y = x producida en una hoja de cálculo. Ob-
serva que la gráfica de y = x2 no es lineal y “crece” mucho más rápido que la gráfica de y = x
para x 7 1.
y
2.5

y = x2

1.5
y=x

0.5

0
x
0 0.5 1 1.5 2

Figura 7-13

© Lopez Mateos Editores. ISBN 978-607-95583-2-1, obra completa, versión electrónica, ISBN 978-607-95583-3-8, volumen 1, versión electrónica. Ejemplar asignado a: Helecto Villarroel gutierrez -
helecto@gmail.com. Fecha: 27 de octubre de 2014. Prohibida su modificación, copia o distribución.
Sección 7-5 Uso de los números reales en ecuaciones 463

Las gráficas de funciones que tienen como dominio los números reales nos permiten com-
parar diferentes funciones. Por ejemplo, en la figura 7-14 considera la gráfica de y = x2, y = x
y y = 1x cuando x Ú 0.
y
2.5

y = x2

y=x
1.5

y= &x

0.5

0
x
0 0.5 1 1.5 2

Figura 7-14
En la figura 7-14, parece que las dos gráficas son simétricas respecto a la “recta diagonal”
que pasa por el origen. De hecho, las dos funciones son imágenes de espejo una de otra res-
pecto a la recta y = x. ¿Es válido esto para todos los valores de x para los cuales se definen
las funciones?
3
Las funciones y = x3 y y = 2x tienen una relación similar a la ilustrada en la figura 7-14.
En Ahora intenta éste 7-14 se explora otro par de funciones con este tipo de relación.

AHORA INTENTA ÉSTE 7-14 Grafica las funciones y = 2x + 1 y y = 1x - 12>2 sobre una gráfica
con la función y = x. ¿Qué observas?

AHORA INTENTA ÉSTE 7-15 La tecnología de graficación se puede usar para investigar varios tipos de
relaciones. Por ejemplo, traza las siguientes en la misma gráfica.
a. y = 2x b. y = 2x + 1 c. y = 2x + 2 d. y = 2x - 1 e. y = 2x - 2
¿Qué observas en este conjunto de rectas? ¿Qué observas en este conjunto de ecuaciones?

Estimación de soluciones de ecuaciones lineales


Antes de que los estudiantes aprendan a resolver ecuaciones algebraicamente, se puede usar
una hoja de cálculo para estimar soluciones del sistema. Por ejemplo, considera la gráfica
de la figura 7-15 donde están trazadas las rectas y = a bx + 5 y y = - x + 10.
3
4
Parece que las dos rectas se cruzan (o intersecan) cuando x tiene un valor entre 2 y 3. Re-
crea la gráfica usando valores de x en intervalos de longitud 0.1 y ve si puedes predecir cuá-
les son las coordenadas del punto de intersección, como se ve en la figura 7-16.

© Lopez Mateos Editores. ISBN 978-607-95583-2-1, obra completa, versión electrónica, ISBN 978-607-95583-3-8, volumen 1, versión electrónica. Ejemplar asignado a: Helecto Villarroel gutierrez -
helecto@gmail.com. Fecha: 27 de octubre de 2014. Prohibida su modificación, copia o distribución.
464 Decimales y números reales

y y
12 10
11
y = –x + 10 9
10 y = –x + 10
9
8
8
7 7
6 3
6 y= 4
x +5
5 y= 3
x +5
4
4 5
3
2 4
1
3
0
x
0 1 2 3 4 2

Figura 7-15 1

0 x
0 1 2 3 4 5

Figura 7-16

Ahora la solución parece ser un punto con coordenadas cerca de (2.8, 7.2). Al substituir x por
2.8 en cada ecuación tenemos y = a b2.8 + 5 = 7.1 en la primera y y = - 2.8 + 10, ó 7.2,
3
4
en la segunda. Así, un mejor estimado de la solución será un valor de x un poco mayor que
2.8 que producirá un valor entre 7.1 y 7.2 para y. Si continúas con este procedimiento, po-
drás acercarte más y más al estimar una solución. En un capítulo posterior de este libro con-
tinuaremos examinando la solución por medio de sistemas de ecuaciones.

Tabla 7-3
Solución de una ecuación con una incógnita
A B
Así como es posible estimar la solución de un sistema de ecuaciones, según se ilus-
1 1 –1
tró anteriormente, podemos estimar (o hallar) la solución de una ecuación con
2 2 1
3 3 3 una incógnita usando una hoja de cálculo. Se puede usar esta técnica cuando la
4 4 5 ecuación es difícil. Vamos a ilustrarlo usando una ecuación sencilla, 2x - 3 = 17.
5 5 7 Una solución exploratoria a la ecuación podría considerar valores de x para los
6 6 9 cuales 2x - 3 diera un resultado cercano a 17. Ve la tabla 7-3 producida por me-
7 7 11 dio de una hoja de cálculo, donde los valores de x se presentan en la columna A y
8 8 13 los valores de 2x - 3 se presentan en la columna B. En la tabla 7-3 vemos que
9 9 15 cuando x tiene un valor de 10, entonces el valor de 2x - 3 es igual a 17. Así, la so-
10 10 17 lución a la ecuación es x = 10. Podríamos haber deducido esto algebraicamente
11 11 19 como sigue:
12 12 21 2x - 3 = 17
13 13 23
2x = 17 + 3
2x = 20
x = 10
Considera una ecuación no lineal como la que vimos antes en esta sección, y = x2. Hay in-
finidad de soluciones, incluyendo (0, 0). Sin embargo, no es fácil para algunos deducir para
qué valores de x, si existen, se tiene y = 0 en la ecuación y = x2 - 5x + 6.
Considera la hoja de cálculo de la tabla 7-4, donde la columna A representa valores de x y
la columna B se creó usando la fórmula =A1^2- 5*A1+6 para representar x2 - 5x + 6.
© Lopez Mateos Editores. ISBN 978-607-95583-2-1, obra completa, versión electrónica, ISBN 978-607-95583-3-8, volumen 1, versión electrónica. Ejemplar asignado a: Helecto Villarroel gutierrez -
helecto@gmail.com. Fecha: 27 de octubre de 2014. Prohibida su modificación, copia o distribución.
Sección 7-5 Uso de los números reales en ecuaciones 465

Tabla 7-4
A B
1 1 2
2 2 0
3 3 0
4 4 2
5 5 6
6 6 12
7 7 20
8 8 30
9 9 42
10 10 56
11 11 72
12 12 90
13 13 110

En la tabla 7-4, vemos que cuando el valor de x en la columna A es 2 y cuando es 3, enton-


ces los valores correspondientes de y son 0 en cada caso. Así, predecimos que y = 0 en la
ecuación y = x2 - 5x + 6 cuando x = 2 ó 3. Esto también se puede apreciar en la gráfica
de la figura 7-17 trazada con la hoja de cálculo en donde la gráfica está restringida a la zona
de interés y los valores de x están espaciados en 0.1.

y
2.5

1.5

0.5

Table 7-5 0
x
0 1 2 3 4
x x - 2
2
–0.5

-2 2
- 1.9 Figura 7-17
1.61
- 1.8 1.24
- 1.7 Aunque el uso de hojas de cálculo no siempre determina una respuesta exacta al resolver
0.89
- 1.6 ecuaciones, usar una gráfica para presentar los datos y una hoja de cálculo para examinar va-
0.56
- 1.5 lores más finos puede ser útil para ayudarte a predecir soluciones.
0.25
- 1.4 - 0.04 Una hoja de cálculo usa sólo decimales finitos para sus cálculos. Así, al resolver una ecua-
- 1.3 - 0.31 ción como x2 - 2 = 0, que según hemos visto tiene soluciones x = 12 y x = - 12, las so-
- 1.2 - 0.56 luciones sólo pueden ser estimadas. En la tabla 7-5 vemos que cuando x se incrementa en
- 1.1 - 0.79 valores de 0.1, el valor de y cambia de 0.25 positivo cuando x = - 1.5 a - 0.04 cuando
x = - 1.4. Entre los valores de x = - 1.5 y x = - 1.4, debe haber un valor de x que pro-
duzca el valor de 0 para y. Refinamientos sucesivos de los valores de x pueden producir una
aproximación de 12.
La siguiente página de muestra parcial presenta la solución de una ecuación que tiene como
solución un número irracional. Haz el número 3 en la sección de Verificación rápida. En la pá-
gina de muestra nota que la solución sugiere que para resolver 51 = t2 , se deberá sacar raíz
cuadrada en ambos lados de la ecuación. Para el ejemplo de la página de muestra, esto es apro-
© Lopez Mateos Editores. ISBN 978-607-95583-2-1, obra completa, versión electrónica, ISBN 978-607-95583-3-8, volumen 1, versión electrónica. Ejemplar asignado a: Helecto Villarroel gutierrez -
helecto@gmail.com. Fecha: 27 de octubre de 2014. Prohibida su modificación, copia o distribución.
466 Decimales y números reales

Página de un libro de texto APLICACIÓN: PARACAIDISMO

Ejemplo Aplicación: Paracaidismo


La fórmula d = 16t2 representa la distancia aproximada d en pies que
un paracaidista recorre en t segundos antes de abrir el paracaídas. La
fórmula supone que no hay resistencia del aire. Halla el tiempo que
tarda un paracaidista en caer 816 pies antes de abrir el paracaídas.

Ve por ayuda Usa la fórmula para la distancia y el tiempo.

Para ayuda en el uso de Substituye d por 816.


fórmulas ve a la lección 2-6,
ejemplo 1. Divide ambos lados entre 16 para aislar a t.

Simplifica.

Halla la raíz cuadrada positiva en cada lado.

Usa una calculadora.

Redondea a la décima más cercana.

El paracaidista tarda alrededor de 7.1 segundos en caer 816 pies.

Verificación rápida
Halla el tiempo que tarda un paracaidista en caer cada distancia.
Redondea a la décima de segundo más cercana.
pies pies

Los números irracionales son números que no se pueden escribir en la forma ab ,


donde a es cualquier entero y b es cualquier entero distinto de cero. Los
números racionales y los irracionales forman el conjunto de los números reales.

Exploración de raíces cuadradas y números irracionales

Fuente: Prentice Hall Mathematics, Course 3, 2008 ( p. 107).

piado pues la solución es una unidad de tiempo y debe ser un número positivo. No hay solucio-
nes que sean números negativos. A continuación desarrollaremos un enfoque alternativo para
resolver una ecuación similar y después regresaremos de nuevo a este problema.
Considera la ecuación x2 = 9. Podemos ver que x = 3 ó - 3 son soluciones. Algunos po-
drían sacar raíz cuadrada en ambos lados de la ecuación. Un método diferente utiliza la dife-
rencia de cuadrados y resuelve como sigue:
x2 = 9
x2 - 9 = 0
x2 - 32 = 0
1x - 321x + 32 = 0 (factorizando)
Ahora, uno de los factores 1x - 32 o 1x + 32 debe ser 0, de modo que x - 3 = 0 o
x + 3 = 0. Así, x = 3 o x = - 3.

© Lopez Mateos Editores. ISBN 978-607-95583-2-1, obra completa, versión electrónica, ISBN 978-607-95583-3-8, volumen 1, versión electrónica. Ejemplar asignado a: Helecto Villarroel gutierrez -
helecto@gmail.com. Fecha: 27 de octubre de 2014. Prohibida su modificación, copia o distribución.
Sección 7-5 Uso de los números reales en ecuaciones 467

Este procedimiento se puede aplicar a la ecuación 51 = t2. De la página de muestra anterior:

51 = t2
0 = t2 - 51
t2 - 51 = 0
t - 115122 = 0
2

1t - 1512(t + 1512 = 0 (factorizando)

Por lo tanto, algún factor 1t - 1512 o 1t + 1512 debe ser 0. Así, t - 151 = 0 o
t + 151 = 0. Entonces, t = 151 o t = - 151. Pero como en el ejemplo el tiempo no
puede ser negativo, debemos usar la solución positiva.
Es natural preguntarse si este procedimiento se puede adaptar para resolver ecuaciones un
poco más complicadas, como x2 + 4x + 3 = 0. ¿Es posible escribir esta ecuación como una
diferencia de cuadrados igual a 0, como en el ejemplo anterior? Una posible respuesta viene
de un enfoque histórico al considerar en realidad dos cuadrados. Sabemos que x2 es un cua-
drado perfecto cuya longitud de lado es x, pero tenemos un 4x adicional. La figura 7-18(a)
muestra una colocación de x2 + 4x y la figura 7-18(b) muestra una diferente configuración
de la parte 4x de la figura en un intento de crear un cuadrado usando la información dispo-
nible. La parte 4x se dividió en dos porciones, cada una con área 2x, y se colocó como se ilus-
tra. Si se pudiera hallar un porción adicional para “llenar el hueco” o para “completar el
cuadrado”, como en la figura 7-18(c), entonces habríamos completado una parte de la tarea
requerida.
2

2 2x 2 2x 4 2

x x2 2x x x2 2x
x x2 x 4x

x x
x 4 2 2
(a) (b) (c)

Figura 7-18

Al añadir un pequeño cuadrado de 2 * 2 en la esquina superior derecha de la figura 7-


18(c), tendríamos un cuadrado con longitud de lado x + 2. Para traducir esto en términos
de nuestra ecuación original, habríamos cambiado el lado izquierdo a x2 + 4x + 4 + 3.
Para mantener la ecuación “balanceada” deberíamos también restar 4, pues ya sumamos 4.
Lo hacemos como sigue: 1x2 + 4x + 42 + 3 - 4 = 0, o 1x2 + 4x + 42 - 1 = 0. En este
caso la expresión entre paréntesis es 1x + 222 y 1 se puede escribir como 12, lo cual nos da:
1x + 222 - 12 = 0
31x + 22 - 1431x + 22 + 14 = 0
Ahora, 1x + 22 - 1 = 0 o 1x + 22 + 1 = 0, de modo que x = - 1 o x = - 3 son solucio-
nes de la ecuación original. Nota que usamos x como una longitud en el trazo de la visuali-
zación del procedimiento. Pero la solución x es un número negativo.
En Ahora intenta éste 7-16, investigamos cómo resolver una ecuación cuadrática de la
forma ax2 + bx + c = 0, donde a Z 0.
© Lopez Mateos Editores. ISBN 978-607-95583-2-1, obra completa, versión electrónica, ISBN 978-607-95583-3-8, volumen 1, versión electrónica. Ejemplar asignado a: Helecto Villarroel gutierrez -
helecto@gmail.com. Fecha: 27 de octubre de 2014. Prohibida su modificación, copia o distribución.
468 Decimales y números reales

AHORA INTENTA ÉSTE 7-16 Para hallar soluciones de ecuaciones de la forma ax2 + bx + c = 0, si es
que existen, donde a Z 0, podemos usar el método completando cuadrados para hacer que la ecuación
adquiera la forma anterior de diferencia de cuadrados. Para resolver esta ecuación la relacionamos con un
problema que ya sabemos cómo resolver, esto es, una ecuación de diferencia de cuadrados.
Entonces la pregunta ahora es ¿podemos reescribir ax2 + bx + c = 0 en la forma m2 - n2 = 0, que
podemos reescribir como 1m - n21m + n2 = 0 y que podemos resolver? Para ver cómo reescribir
ax2 + bx + c en la forma deseada, primero simplificamos la ecuación dividiendo ambos lados entre a para
b c
obtener x2 + x + = 0 y después usar conceptos de geometría. Considera x2 como el área de
a a
un cuadrado con longitud de lado x, como en la figura 7-19(a).

b b x b b x b2
2a 2a 2a 2a 4a2

x2
b x b x
x x2 bx x x2
2a
x
2a
a

x x x
b b b
a 2a 2a
(a) (b) (c)

Figura 7-19
b b
De manera análoga, x se puede considerar el área de un rectángulo cuyo largo es x y cuyo ancho es ,
a a
b b
como en la figura 7-19(a). Para construir un cuadrado de área x + x, podemos dividir x en dos partes
2
a a
iguales y colocarlas como se ve en la figura 7-19(b). Después, para completar un cuadrado con área cercana
b
a x2 + x necesitamos otro cuadrado pequeño, como se ilustra en la figura 7-19(c). Necesitamos que el
a
b b 2 b2
pequeño cuadrado tenga lados de longitud y por lo tanto tenga área a b , o 2 . Añadido este pe-
2a 2a 4a
b
queño cuadrado, la longitud del lado del cuadrado completado es x + .
2a
b c b2
Ahora considera nuestra expresión simplificada x2 + x + = 0. Si sumamos 2 a los dos primeros térmi-
a a 4a
b b2
nos, obtendríamos x + x + 2 y tendríamos que ajustar la ecuación para mantenerla equivalente. Lo
2
a 4a
b2
hacemos al restar 2 como sigue:
4a
b b2 b2 c
x2 + x + 2 - 2 + = 0
a 4a 4a a
b 2 b2 c
ax + b + a- 2 + b = 0
2a 4a a
o, de manera equivalente,
b 2 b2 c
ax + b - a 2 - b = 0
2a 4a a
b 2 b2 c
La primera expresión es un cuadrado perfecto a x + b y el resto del lado izquierdo de la ecuación es 2 - ,
2a 4a a
b2 - 4ac
o . Si queremos escribir esta última expresión como un cuadrado perfecto y si b2 - 4ac Ú 0, lo
4a2

© Lopez Mateos Editores. ISBN 978-607-95583-2-1, obra completa, versión electrónica, ISBN 978-607-95583-3-8, volumen 1, versión electrónica. Ejemplar asignado a: Helecto Villarroel gutierrez -
helecto@gmail.com. Fecha: 27 de octubre de 2014. Prohibida su modificación, copia o distribución.
Sección 7-5 Uso de los números reales en ecuaciones 469

2
podemos hacer al sacar la raíz cuadrada de la expresión y elevarla al cuadrado como a b2 - 4ac b .
A 4a2

b 2 2
Combinando las partes, tenemos ax + b - a b - 4ac b = 0. El lado izquierdo de esta última ecua-
2

2a A 4a2
ción es de la forma m2 - n2 y se puede reescribir usando la diferencia de cuadrados como

b b
ax + - b - 4ac b ax + + b - 4ac b = 0
2 2

2a A 4a2 2a A 4a2
b
- b - 4ac = 0
2
Como el producto es 0, entonces uno de los factores debe ser 0; así, tenemos x +
2a A 4a2
b
+ b - 4ac = 0, lo cual conduce a lo siguiente:
2
ox +
2a A 4a2
b b
+ b - 4ac o x = - - b - 4ac .
2 2
x = -
2a A 4a2 2a A 4a2

Así,
-b + 2b2 - 4ac -b - 2b2 - 4ac
x = o x =
2a 2a

Con frecuencia este resultado final se escribe en forma combinada, como se ve abajo, y se llama fórmula
cuadrática:
-b ; 2b2 - 4ac
x =
2a
a. Usa el procedimiento de “completar el cuadrado” para resolver x2 + 5x + 6 = 0.
b. Usa la fórmula cuadrática para hallar soluciones de 2x2 - 14x + 24 = 0.
c. Considera la expresión b2 - 4ac para averiguar cuándo una ecuación cuadrática no tiene una solución
que sea número real.

Al resolver x2 + 4x + 3 = 0, vimos que el lado izquierdo de la ecuación finalmente se


factorizó en 1x + 121x + 32. ¿Podrían hallarse esos factores sin usar la diferencia de cua-
drados o la fórmula cuadrática que vimos en Ahora intenta éste 7-16?
Para factorizar una expresión como x2 + 4x + 3, queremos escribirla con dos factores de
la forma 1x + a21x + b2. Si tenemos los factores de esa forma y los multiplicamos usando la
propiedad distributiva, podemos tener lo siguiente:

1x + a21x + b2 = 1x + a)x + 1x + a2b


= x2 + ax + xb + ab
= x2 + 1a + b2x + ab

Así, para factorizar x2 + 4x + 3 necesitamos buscar dos números a y b cuya suma sea 4 y
cuyo producto sea 3. Ese método nos conduciría a 1x + 121x + 32.

Ejemplo 7-15 Resuelve x2 + 12x + 20 = 0










a. factorizando.
b. usando la fórmula cuadrática en Ahora intenta éste 7-16.

© Lopez Mateos Editores. ISBN 978-607-95583-2-1, obra completa, versión electrónica, ISBN 978-607-95583-3-8, volumen 1, versión electrónica. Ejemplar asignado a: Helecto Villarroel gutierrez -
helecto@gmail.com. Fecha: 27 de octubre de 2014. Prohibida su modificación, copia o distribución.
470 Decimales y números reales

Solución a. Dos números cuya suma es 12 y cuyo producto es 20 son 10 y 2. Así, los fac-
tores son 1x + 1021x + 22. Luego,

x2 + 12x + 20 = 0
1x + 1021x + 22 = 0

x = - 10 o x = - 2
-12 ; 2122 - 4 # 1 # 20
b. x = ; x = - 10 o x = - 2.
2#1

Evaluación 7-5A

1. Grafica cada una de las rectas siguientes: 7. Una versión de la fórmula para calcular la distancia que viaja
una bala cuando se dispara con rapidez inicial de 1000 pies/s
a. y = 3x - 7 durante un periodo de tiempo, t en segundos, es
- 3
b. y = a b x + 5
d = a b1 - 322t2 + 1000t.
1
4
c. y = x 12 2
2. Grafica lo siguiente: Usa esta fórmula para hallar lo siguiente:
a. y = x2 a. la distancia que ha viajado la bala a los 3 s
b. y = 2x b. la distancia que ha viajado la bala a los 5 s
3. Explica si es posible graficar y = 2 - x. De ser así, grafica. 8. La circunferencia, C, de un círculo es directamente pro-
4. Halla una función cuya gráfica sea la imagen espejo de porcional a su diámetro, d; en particular, C = pd. ¿Cómo
y = - x respecto a la recta y = x. se ve la gráfica de esta función?
5. Usa gráficas de una hoja de cálculo para estimar la solu- 9. En el libro The Number Devil: A Mathematical Adven-
ción del siguiente sistema de ecuaciones lineales: ture (El diablo de los números: una aventura matemá-
tica) de Hans Magnus Enzensberger, se analiza el
y = - 2x + 7 1 + 15
y = 3x - 5 número irracional , con aproximación decimal
2
6. Resuelve las ecuaciones siguientes: 1.618.
2 a. Halla el recíproco de este número hasta tres lugares
a. 2x - 13 = 23 b. x + 13 = 41 decimales.
3
c. 2x3 = - 16 d. 14x2 - 25 = 3 b. ¿Cómo se relaciona tu respuesta a la parte (a) con la
e. 1x - 721x + 32 = 0 f. x2 + 4x = 5 solución de la ecuación x2 - x - 1 = 0 ?
g. 2x2 + 3x = 5 10. Dos variables, x y y, tienen producto 1. Esboza la gráfica
de esta relación.

Evaluación 7-5B

1. Grafica cada una de las rectas siguientes: 4. ¿Cuál es la expresión algebraica de una función cuya grá-
- 3 fica es la imagen de espejo de y = - x + b, donde b es un
a. y = 2x - 5 b. y = a bx + 7
5 número real, respecto a la recta y = x?
c. y = 2x12 5. Usa gráficas para estimar la solución del siguiente sis-
2. Grafica lo siguiente: tema de ecuaciones lineales:
a. y = 2x2 b. y = 3x y = - 3x + 5
3. ¿Existe la gráfica de y = - 2 - x. De ser así, grafícala. y = 2x - 7

© Lopez Mateos Editores. ISBN 978-607-95583-2-1, obra completa, versión electrónica, ISBN 978-607-95583-3-8, volumen 1, versión electrónica. Ejemplar asignado a: Helecto Villarroel gutierrez -
helecto@gmail.com. Fecha: 27 de octubre de 2014. Prohibida su modificación, copia o distribución.
Sección 7-5 Uso de los números reales en ecuaciones 471

6. Resuelve las ecuaciones siguientes: 8. Escribe la expresión siguiente en la forma más simple:
2
a b12x2 , a b.
a. 3x - 12 = 23 b. x + 15 = 45 2 7p
3
3 100
c. 3x3 = - 24 d. 14x2 - 39 = 3
e. 1x - 721x + 22 = 0 f. 2x2 - 5 = 0 9. El diámetro, d, de un círculo es directamente proporcio-
g. 2x2 - 5x = - 3 C
nal a su circunferencia, C; en particular, d = . Grafica
7. Una versión de la fórmula para calcular la distancia que p
viaja una bala cuando se dispara con rapidez inicial de d como función de C.
1000 pies/s durante un periodo de tiempo, t en segundos, 10. La función cuya ecuación es y = x2 - 2x + 1 tiene
es d = - 16t2 + 1000t. Usa esta fórmula para hallar lo si- una gráfica que es una parábola. Halla los valores de x
guiente: donde la gráfica cruza el eje x.
a. la distancia que ha viajado la bala a los 2 s
b. la distancia que ha viajado la bala a los 6 s

Conexiones matemáticas 7-5

Comunicación Preguntas de repaso


1. Lee un capítulo de El diablo de los números: una aventura 9. John Wallis, matemático inglés, descubrió en 1650 la cu-
matemática de Hans Enzensberger (1997) y escribe un riosa razón siguiente:
párrafo para intercambiarlo con tus compañeros de clase
2#2#4#4#6#6#8Á
sobre el álgebra en este libro de fantasías matemáticas.
2. Debate con tus compañeros de clase la siguiente afirma- 1#3#3#5#5#7#7Á
ción, donde un grupo toma el lado “pro” y el otro el lado Para aproximar la razón, considera la sucesión siguiente:
“contra”: Un estudiante de educación media puede comprender
totalmente el concepto de número irracional para resolver una 2 2#2 2#2#4 2#2#4#4
, , , , Á
ecuación como x2 - 2 = 0. 1 1#3 1#3#3 1#3#3#5
Solución abierta Escribe el equivalente decimal de los términos dados.
3. Investiga una aproximación de p y explícala. 10. En 1914, en el Scientific American apareció la siguiente
4. Busca en Internet el argumento de Georg Cantor de frase mnemónica: “May I have a large container of coffee?”
que hay tantos números racionales como números natu- Considera el número de letras en cada palabra. ¿Qué nú-
rales hay. Escribe una explicación del argumento para mero muy conocido representa este mnemónico? (Uno
tus compañeros de clase. equivalente en español sería: “Mar e isla a veces contem-
plo la mañana”, Manuel López Mateos, 2011)
Aprendizaje colectivo 11. a. Halla 0.8 , 0.32 convirtiendo estos números a números
5. En un grupo analiza libros de texto de nivel 6–8 y determi - a
racionales de la forma , donde a y b son enteros y b Z 0,
na cómo tratan los números irracionales, si es que lo ha- b
cen. Escribe una argumentación acerca de si tu grupo cree, dividiendo los números racionales y regresando el co-
o no, que estos números se deben enseñar en este nivel. ciente a forma decimal.
6. En grupo, investiga al menos tres artículos sobre la ense- b. Halla 0.8 , 0.32 usando la división decimal y compara
ñanza de las matemáticas con hojas de cálculo y compara los tu respuesta con la de la parte (a).
resultados. a
12. Convierte 7.271 en un número racional de la forma ,
Preguntas del salón de clase b
7. Un estudiante argumenta que y = 11 + 122x no tiene
donde a y b son enteros y b Z 0.
gráfica pues 1 + 12 no se puede escribir como un nú- 13. Halla un decimal periódico, entre 0.21 y 0.2.
mero exacto. ¿Cómo le puedes ayudar? 14. Ordena lo siguiente de menor a mayor:
8. Una estudiante pregunta qué es más útil para enseñar
conceptos de álgebra: una hoja de cálculo o una calcula-
5 7
dora gráfica. ¿Cómo le respondes? 2.5, , 2.05, 2.15,
3 3

© Lopez Mateos Editores. ISBN 978-607-95583-2-1, obra completa, versión electrónica, ISBN 978-607-95583-3-8, volumen 1, versión electrónica. Ejemplar asignado a: Helecto Villarroel gutierrez -
helecto@gmail.com. Fecha: 27 de octubre de 2014. Prohibida su modificación, copia o distribución.
472 Decimales y números reales

Pregunta del National Assessment of Educational Progress a. 4 yardas y 5 yardas


(NAEP) (Evaluación Nacional del Progreso Educativo) b. 5 yardas y 6 yardas
El señor Hernán compró una alformbra cuadrada con c. 6 yardas y 7 yardas
área de 39 yardas cuadradas. ¿Entre qué pareja de núme- d. 7 yardas y 8 yardas
ros se encuentra la longitud de cada lado de la alfombra? e. 9 yardas y 10 yardas
NAEP, 2007, Grado 8

Sugerencia para resolver el problema preliminar


Piensa en decimales periódicos como una ruta posible para hallar una solución.

Resumen del capítulo

I. Decimales D. Las raíces cuadradas se pueden encontrar por


A. Todo número racional se puede representar medio del método del refinamiento o por el
como un decimal exacto o como uno periódico. método de Arquímedes.
a E. Los métodos de cálculo con decimales exactos
B. Un número racional , en su forma más simple
b o periódicos, se pueden verificar convirtiendo
cuyo denominador es de la forma 2m # 5n, donde los decimales a números racionales en la forma
m y n son números enteros, se puede expresar a
, donde a y b son enteros y b Z 0.
como decimal exacto. b
F. Se pueden obtener estimaciones para la aritmé-
C. Un decimal periódico es un decimal con un tica decimal siguiendo las mismas reglas genera-
conjunto de dígitos, llamado periodo, que se les que para otros conjuntos de números.
repite infinidad de veces. III. Radicales y exponentes racionales
D. Un número está en notación científica si se es- n
A. 2x o x1>n es la n-ésima raíz de x y n es el índice.
cribe como el producto de un número n que es
B. Las propiedades siguientes valen para todos los ra-
mayor o igual que 1 y menor que 10, y una po-
dicales si la expresión que los incluye tiene sentido:
tencia entera de 10. Si un número es negativo,
a. 2xy = 2x # 2y
n n n
sigue las reglas para un número positivo y
n
agrega el signo negativo. n x 1x
E. Una sucesión geométrica infinita con primer b. =
Ay 1y
n

término a y razón r, donde 0 6 r 6 1, tiene m


c. xm>n = a 1x b = 2xm, si m>n está en su
a n n
suma S = .
1 - r
II. Números reales forma más simple.
A. Un número irracional está representado por 夝 IV. La solución a una ecuación cuadrática de la forma
un decimal que no termina y no se repite. ax2 + bx + c = 0, donde a Z 0, es
B. El conjunto de números reales es el conjunto - b ; 2b2 - 4ac
de todos los decimales, a saber: la unión del x = . Para obtener soluciones en
2a
conjunto de los números racionales y el con-
números reales, b2 - 4ac Ú 0.
junto de los números irracionales.
C. Si a es cualquier número real no negativo, enton-
ces la raíz cuadrada principal de a, denotada por
1a, es un número no negativo b tal que bb =
b2 = a.

© Lopez Mateos Editores. ISBN 978-607-95583-2-1, obra completa, versión electrónica, ISBN 978-607-95583-3-8, volumen 1, versión electrónica. Ejemplar asignado a: Helecto Villarroel gutierrez -
helecto@gmail.com. Fecha: 27 de octubre de 2014. Prohibida su modificación, copia o distribución.
Revisión del capítulo 473

Revisión del capítulo

1. a. Sobre la recta numérica, halla los decimales que 1012


corresponden a los puntos A, B y C. d. 29.4 # -
10 4
b. Indica con D el punto que corresponde a 0.09 y e. 0.47 # 100012
con E el punto que corresponde a 0.15. 3
f. 9
0 0.1 0.2 5
12. a. Halla cinco decimales entre 0.1 y 0.11 y ordéna-
A B C los de mayor a menor.
b. Halla cuatro decimales entre 0 y 0.1 listados de
2. Escribe cada caso como un número racional en la menor a mayor de modo que cada decimal, co-
a
forma , donde a y b son enteros y b Z 0: menzando por el segundo, sea el doble de grande
b
que el anterior.
a. 32.012 b. 0.00103 c. Halla cuatro decimales entre 0.1 y 0.2 y lístalos
3. Da un criterio para determinar si una fracción se en orden creciente de modo que el primero esté
puede escribir como un decimal exacto sin tener a la mitad entre 0.1 y 0.2, el segundo a la mitad
que efectuar la división. Explica por qué es válido entre el primero y 0.2, el tercero a la mitad del
este criterio. segundo y 0.2, y de manera análoga el cuarto.
4. Una tabla tiene 442.4 cm de largo. ¿Cuántos estan- 13. Clasifica cada expresión como racional o irracional
tes se pueden cortar de ahí si cada estante ha de te- (supón que los patrones mostrados continúan).
ner 55.3 cm de largo? (No tomes en cuenta el ancho a. 2.19119911999119999119 Á
del corte.)
1
5. Escribe cada caso como decimal: b.
4 1 2 5 12
a. b. c. d. 4
7 8 3 8 c.
9
6. Escribe cada caso como una fracción en su forma
más simple: d. 0.0011001100110011 Á
a. 0.28 b. - 6.07 c. 0.3 d. 2.08 e. 0.001100011000011 Á
n
7. Redondea cada uno de los números siguientes como 14. Escribe cada caso en la forma a1b o a 2b, donde a
se especifica: y b son enteros positivos y b tiene el menor valor po-
a. 307.625 a la centésima más cercana sible:
b. 307.625 a la décima más cercana a. 1242 b. 1288
c. 307.625 a la unidad más cercana c. 1180 d. 2 3 162
d. 307.625 a la centena más cercana 15. Responde cada pregunta y explica tus respuestas:
8. Reescribe cada caso en notación científica: a. ¿El conjunto de los números irracionales es cerra-
a. 426,000 do bajo la suma?
b. 324 # 10 6
- b. ¿El conjunto de los números irracionales es cerra-
c. 0.00000237 do bajo la resta?
d. - 0.325 c. ¿El conjunto de los números irracionales es cerra-
9. Ordena los decimales siguientes de mayor a menor: do bajo la multiplicación?
1.4519, 1.4519, 1.4519, 1.4519, - 0.134, d. ¿El conjunto de los números irracionales es cerra-
- 0.13401, 0.13401 do bajo la división?
10. Las siguientes son sucesiones geométricas. Halla los 16. Halla una aproximación para 123 que sea correcta hasta
términos faltantes. tres lugares decimales sin usar las teclas yx o 1 .
a. 5, , 10 17. Aproxima 2 3 2 a dos lugares decimales por un mé-
b. 1, , , , 1>4 todo similar al del refinamiento.
11. Escribe cada caso en notación científica sin usar cal- 18. Grafica cada una de las rectas siguientes:
culadora:
b. y = a bx + 7
- 1
a. 1783411.56 a. y = - 2x + 5
5
347
b. c. y = - x12
108 19. Grafica lo siguiente:
c. 49.3 # 108 a. y = - x2
-
b. y = 3 x

© Lopez Mateos Editores. ISBN 978-607-95583-2-1, obra completa, versión electrónica, ISBN 978-607-95583-3-8, volumen 1, versión electrónica. Ejemplar asignado a: Helecto Villarroel gutierrez -
helecto@gmail.com. Fecha: 27 de octubre de 2014. Prohibida su modificación, copia o distribución.
474 Decimales y números reales

20. Usa gráficas para estimar la solución del siguiente c. x3 = - 1


sistema de ecuaciones lineales: d. 4x2 - 33 = 3
e. 12x - 7213x + 22 = 0
y = - 1x + 6
y = 2x - 7 f. x2 - x = 6
g. 2x2 - 3x + 1 = 0
21. Resuelve las ecuaciones siguientes para soluciones 22. ¿Cuál sería el tercer término de una sucesión geo-
en números reales: métrica cuyo primer término es p y cuya razón es
a. - 3x + 12 = 23 1
?
2 p
b. x + 18 = 42
3 23. Explica si crees, o no, que vas a encontrar en una
tienda un precio que sea un número irracional.

Bibliografía seleccionada

Behr, M., I. Wachsmuth, T. Post, and R. Lesh. “Order Hiebert, J. “Mathematical, Cognitive, and Instructional
and Equivalence of Rational Numbers: A Clinical Analyses of Decimal Fractions.” In Analysis of Arith-
Teaching Experiment.” Journal for Research in Math- metic for Mathematics Teaching, edited by G. Lein-
ematics Education 15 ( July 1984): 323–341. hardt, R. Putman, and R. Hattrup. Hillsdale, NJ:
Beigie, D. “Integrating Content to Create Problem- LEA, 1992.
Solving Opportunities.” Mathematics Teaching in the Irwin, K. “Using Everyday knowledge of Decimals to
Middle School 13 (February 2008): 352–360. Enhance Learning.” Journal of Research in Mathe-
Beigie, D. “Investigating Limits in Number Patterns.” matics Education 32 ( July 2001): 399–420.
Mathematics Teaching in the Middle School 7 (April Judd, W. “Instructional Games with Calculators.”
2002): 438–443. Mathematics Teaching in the Middle School 13 (Febru-
Boers-van Oosterum, M. “Understanding of Variables ary 2007): 312–314.
and Their Uses Acquired by Students in Traditional Leonard, J., and L. Campbell. “Using the Stock Market
and Computer-Intensive Algebra.” Ph.D. diss., Uni- for Relevance in Teaching Number Sense.” Mathe-
versity of Maryland College Park, 1990. matics Teaching in the Middle School 9 (February
Brown, M. “Place Value and Decimals.” In Children’s 2004): 294.
Understanding of Mathematics. London: John Murray, Lewis, L. “Irrational Numbers Can ‘In-Spiral’ You.”
1981, pp. 11–16. Mathematics Teaching in the Middle School 13 (April
Drum, R., and W. Petty, Jr. “2 Is Not the Same as 2007): 442–445.
2.0!” Mathematics Teaching in the Middle School 6 Martine, S., and J. Bay-Williams. “Investigating Stu-
(September 2000): 34–38. dents’ Conceptual Understanding of Decimal Frac-
Dunham, P., and T. Dick. “Research on Graphing Cal- tions Using Multiple Representations.” Mathematics
culators.” Mathematics Teacher 87 (September 1994): Teaching in the Middle School 8 ( January 2003):
440–445. 244–247.
Enzensberger, H. The Number Devil: A Mathematical Neuschwander, C. Sir Cumference and the Dragon of Pi:
Adventure. New York: Henry Holt and Company, A Math Adventure. Watertown, MA: Cambridge
1997. Publishing, Inc., 1999.
Flores, A. “On My Mind: The Finger and the Moon.” Oppenheimer, L., and R. Hunting. “Relating Fractions
Mathematics Teaching in the Middle School 13 (October and Decimals: Listening to Students Talk.” Mathe -
2007): 132. matics Teaching in the Middle School 4 (February 1999):
Glasgow, R., G. Ragan, W. Fields, R. Reys, and 318–321.
D. Wasman. “The Decimal Dilemma.” Teaching Reeder, S. “Are We Golden? Investigations with the
Children Mathematics 7 (October 2000): 89–93. Golden Ratio.” Mathematics Teaching in the Middle
Groves, S. “Calculators: A Learning Environment to School 13 (October 2007): 150–154.
Promote Number Sense.” Paper presented at the Reeves, A., and M. Beasley. “Sudorku.” Mathematics
annual meeting of the American Educational Re- Teaching in the Middle School 13 (August 2007):
search Association, New Orleans, April 1994. 30–31, 36.

© Lopez Mateos Editores. ISBN 978-607-95583-2-1, obra completa, versión electrónica, ISBN 978-607-95583-3-8, volumen 1, versión electrónica. Ejemplar asignado a: Helecto Villarroel gutierrez -
helecto@gmail.com. Fecha: 27 de octubre de 2014. Prohibida su modificación, copia o distribución.
Bibliografía seleccionada 475

Reys, B., and F. Arbaugh. “Clearing Up the Confusion 1990–2000 Mathematics Assessments of the National
over Calculator Use in Grades K–5.” Teaching Chil- Assessment of Educational Progress, edited by P. Kloo-
dren Mathematics 8 (October 2001): 90–94. sterman and F. Lester. Reston, VA: NCTM, 2004,
Reys, R., B. Reys, N. Nohda, and H. Emori. “Mental pp. 105–144.
Computation Performance and Strategy Use of Thompson, A., and S. Sproule. “Deciding When to Use
Japanese Students in Grades 2, 4, 6, and 8.” Journal Calculators.” Mathematics Teaching in the Middle
for Research in Mathematics Education 26 ( July 1995): School 6 (October 2000): 12–129.
304–326. Thompson, C., and V. Walker. “Connecting Decimals
Rojano, T. “Developing Algebraic Aspects of Problem and Other Mathematical Content.” Teaching Chil-
Solving within a Spreadsheet Environment.” In dren Mathematics 2 (April 1996): 496–502.
Approaches to Algebra: Perspectives for Research and Vinogradova, N. “Solving Quadratic Equations by Com-
Teaching, edited by Nadine Bednarz, Carolyn Kieran, pleting Squares.” Mathematics Teaching in the Middle
and Lesley Lee. Boston: Kluwer Academic Publish- School 12 (March 2007): 403–405.
ers, 1996. Wearne, D., and J. Hiebert. “A Cognitive Approach to
Shaw, K., and L. Aspinwall. “Mathematics Detective: Meaningful Mathematics Instruction: Testing a
Mission Possible: Exploring the Mysterious 1>7.” Local Theory Using Decimal Numbers.” Journal
Mathematics Teaching in the Middle School 11(No - for Research in Mathematics Education 19 (November
vember 2005): 178. 1988): 371–384.
Sheets, C. Effects of Computer Learning and Problem- Williams, S., and J. Copley. “Using Calculators to Dis-
solving Tools on the Development of Secondary School cover Patterns in Dividing Decimals.” Mathematics
Students’ Understanding of Mathematical Functions. Teaching in the Middle School 1 (April 1994): 72–75.
Ph.D. Diss., University of Maryland College Park, Yang, D., and R. Reys. “One Fraction Problem, Many
1993. Solution Paths.” Mathematics Teaching in the Middle
Sowder, J., D. Wearne, W. Martin, and M. Strutchens. School 7 (November 2001): 164–166.
“What Do 8th-Grade Students Know about Mathe-
matics?” In Results and Interpretations of the

© Lopez Mateos Editores. ISBN 978-607-95583-2-1, obra completa, versión electrónica, ISBN 978-607-95583-3-8, volumen 1, versión electrónica. Ejemplar asignado a: Helecto Villarroel gutierrez -
helecto@gmail.com. Fecha: 27 de octubre de 2014. Prohibida su modificación, copia o distribución.
Razonamiento proporcional,
porcentajes y aplicaciones
CAPÍTULO

Problema preliminar
Un vendedor vendió dos carros. En el primero obtuvo una ganancia del 10% y en el segundo
perdió el 10%. Vendió los carros a $9999 cada uno. ¿Cuál fue la cantidad de dinero de ganancia o
pérdida en las dos transacciones, o el vendedor quedó a mano?

476
© Lopez Mateos Editores. ISBN 978-607-95583-2-1, obra completa, versión electrónica, ISBN 978-607-95583-3-8, volumen 1, versión electrónica. Ejemplar asignado a: Helecto Villarroel gutierrez -
helecto@gmail.com. Fecha: 27 de octubre de 2014. Prohibida su modificación, copia o distribución.
Sección 8-1 Razones, proporciones y razonamiento proporcional 477

E l razonamiento proporcional es un concepto extremadamente importante que se en-


seña en los grados K–8 y que en muchos casos no recibe la atención necesaria. Como
lo señalaron Hoffer y Hoffer (1988), “El razonamiento proporcional generalmente se con-
sidera como una de las componentes importantes del pensamiento formal que se adquiere
en la adolecencia. . . . Cuando no se logra desarrollo en esta área en la adolescencia tem-
prana o media, se trunca la capacidad de estudiar una gran variedad de disciplinas en las
que se requiere razonamiento y comprensión cuantitativa, incluyendo álgebra, geometría,
algunos aspectos de biología, química y física” (p. 303). La proporcionalidad se relaciona
con la mayoría, si no es que con todos, los temas fundamentales de la educación media y
proporciona un contexto para estudiarlos.
Las personas enfrentan problemas de razonamiento proporcional en la vida cotidiana, por
ejemplo al modificar recetas o al determinar un plan de ahorro. Estos tipos de problemas
requieren que los estudiantes identifiquen las variables involucradas y después identifiquen
las relaciones entre estas variables. En los Principios y objetivos se da un ejemplo de cómo for-
mar razones para hacer comparaciones en situaciones que involucran pares de números:
Trabajar con proporciones es un objetivo fundamental de estos Principios para la educación me-
dia. Los estudiantes deben tener fluidez en formar razones para hacer comparaciones en situacio-
nes que involucren pares de números, como en el siguiente problema:
Si con tres paquetes de chocolate se hacen quince tazas de chocolate caliente, ¿cuántos paque-
tes se necesitan para hacer sesenta tazas? (p. 34)

Más aún, en los Puntos focales para el grado 7 hallamos lo siguiente:

Los estudiantes extienden su trabajo con razones para desarrollar comprensión de la proporciona-
lidad que aplican para resolver problemas de uno o varios pasos en numerosos contextos. (p. 19)

En este capítulo cubrimos razones y proporciones, razonamiento proporcional y porcen-


tajes, después incluimos una sección optativa sobre cálculos de intereses.

8-1 Razones, proporciones y razonamiento proporcional

Como se vio en las citas anteriores de los Principios y objetivos y los Puntos focales, las razones y
proporciones son una parte muy importante en los planes y programas de la educación media.
Las razones las encontramos en la vida cotidiana. Por ejemplo, puede haber una razón de 2 a 3
de diputados de un partido a diputados de otro partido en cierto comité legislativo, un amigo
recibió una infracción por exceso de velocidad al ir a 85 kilómetros por hora o los huevos cues-
a
tan $15.50 la docena. Cada caso ilustra una razón. Las razones se escriben o a : b y usual-
b
mente se usan para comparar cantidades.
1
Una razón de 1 : 3 de niños a niñas en una clase significa que los niños son de las niñas;
3
es decir, hay 1 niño por cada 3 niñas. Nota que también podríamos decir que la razón de niñas
a niños es de 3 : 1, o que hay 3 veces más niñas que niños. Las razones pueden representar com-
paraciones de la parte al todo o el todo a la parte. Por ejemplo, si la razón de niños a niñas en
una clase es de 1 : 3, entonces la razón de los niños (la parte) a toda la clase (el todo)es de 1 : 4.
1 b
Si hay b niños y g niñas, entonces g = 3b; esto es, = . Además, la razón de niños a toda
3 3b
b b b 1
la clase es = = = .
b + g b + 3b 4b 4

© Lopez Mateos Editores. ISBN 978-607-95583-2-1, obra completa, versión electrónica, ISBN 978-607-95583-3-8, volumen 1, versión electrónica. Ejemplar asignado a: Helecto Villarroel gutierrez -
helecto@gmail.com. Fecha: 27 de octubre de 2014. Prohibida su modificación, copia o distribución.
478 Razonamiento proporcional, porcentajes y aplicaciones

También podríamos decir que la razón de toda la clase (el total) a los niños (la parte) es de
4 : 1. Algunas razones comparan parte a parte, como la razón del número de niños a ni-
ñas o el número de alumnos a un maestro. Por ejemplo, una escuela podría decir que la
razón promedio de estudiantes a maestros no puede ser mayor que 24 : 1.
Nota que la razón de 1 : 3 de niños a niñas en una clase no nos dice cuántos niños y cuán-
tas niñas hay en la clase. Sólo nos dice el tamaño relativo de los grupos. Podría haber 2 ni-
ños y 6 niñas o 3 niños y 9 niñas o 4 niños y 12 niñas, o cualesquier otros números que
1
den una fracción equivalente a .
3

Ejemplo 8-1 Había 7 varones y 12 mujeres en una cafetería el lunes en la tarde, y en la sala de juegos con-








tigua había 14 varones y 24 mujeres.


a. Expresa como razón el número de varones a mujeres en la cafetería (parte a parte).
b. Expresa como razón el número de varones a mujeres en la sala de juegos (parte a parte).
c. Expresa como razón el número de varones en la sala de juegos al número de personas en
la sala de juegos (parte a todo).

7 14 7 14 7
Solución a. La razón es . b. La razón es ó . c. La razón es ó .
12 24 12 38 19

Proporciones
En un estudio de alumnos del sexto grado, conducido por Harel y sus colegas (1994), a los
niños se les mostró una figura de un envase con jugo de naranja y se les dijo que el jugo de
naranja se hace a partir de concentrado de naranja y agua. Se les mostraron dos vasos —uno
grande y uno pequeño— y se les dijo que ambos vasos se habían llenado con el jugo de na-
ranja del envase. Se les pidió que dijeran si el jugo de naranja de cada uno de los vasos sabía
igualmente a naranja o si uno sabía más a naranja que el otro. Cerca de la mitad de los alum-
nos dijo que el vaso más grande sabía más a naranja y alrededor de la mitad dijo que el vaso
pequeño sabía más a naranja. Estos alumnos estaban pensando sólo en una cantidad —sólo
el agua o sólo el concentrado de naranja. Por ejemplo, un estudiante dijo que el vaso grande
sabía más a naranja porque el vaso es más grande y le cabría más concentrado de naranja,
mientras que otro dijo que el vaso pequeño sabía más a naranja porque tenía menos agua
pero más naranja por onza.
Supón que la receta I de una bebida a base de naranja pide 2 latas de concentrado de naranja
por cada 3 latas de agua. Podemos decir que la razón de las latas de concentrado de naranja a
latas de agua es de 2 : 3. Representamos esto en la figura 8-1(a), donde N representa una lata
de concentrado de naranja y A representa una lata de agua. En la figura 8-1(b) y (c), continua-
mos el proceso de añadir 2 latas de concentrado de naranja por cada 3 latas de agua.

(a) N N (b) N N N N (c) N N N N N N


A A A A A A A A A A A A A A A A A A
Figura 8-1 (Receta I)

© Lopez Mateos Editores. ISBN 978-607-95583-2-1, obra completa, versión electrónica, ISBN 978-607-95583-3-8, volumen 1, versión electrónica. Ejemplar asignado a: Helecto Villarroel gutierrez -
helecto@gmail.com. Fecha: 27 de octubre de 2014. Prohibida su modificación, copia o distribución.
Sección 8-1 Razones, proporciones y razonamiento proporcional 479

De la figura 8-1 podemos desarrollar y continuar la tabla de razones, como se muestra en


la tabla 8-1.

Tabla 8-1

Latas de concentrado de naranja 2 4 6 8 10 12

Latas de agua 3 6 9 ? ? ?

En la tabla 8-1, las razones 2>3 y 4>6 son iguales. La igualdad 2>3 = 4>6 es una propor-
ción. En general, tenemos lo siguiente.

Definición
Una proporción es una igualdad entre dos razones. Es decir, dos razones están en proporción
cuando son iguales.

Si la receta II pide 4 latas de concentrado de naranja por cada 8 latas de agua, entonces la
razón de las latas de concentrado de naranja a latas de agua, para esta receta, es de 4 : 8. Ilus-
tramos esto en la figura 8-2(a).

(a) N N N N (b) N N N N
A A A A A A A A
A A A A A A A A
Figura 8-2 (Receta II)

¿Cuál de las dos recetas producirá una bebida que sepa más a naranja? En la figura 8-1(a),
vemos que en la receta I hay 2 latas de concentrado de naranja por cada 3 latas de agua. En la
figura 8-2(a), vemos que en la receta II hay 4 latas de concentrado de naranja por cada 8 latas
de agua. Para comparar las dos recetas necesitamos tener el mismo número de latas de con-
centrado de naranja o el mismo número de latas de agua, cualquiera de las dos. La figura 8-
1(b) muestra que para la receta I hay 4 latas de concentrado de naranja por cada 6 latas de
agua. En la receta II, para 4 latas de concentrado de naranja hay 8 latas de agua. La receta II
pide más agua para 4 latas de concentrado de naranja, de modo que sabe menos a naranja.
De manera alternativa podemos observar que en la figura 8-2(b), la receta II se puede dividir
para mostrar que hay 2 latas de concentrado de naranja por cada 4 latas de agua. Podemos
entonces comparar esto con la figura 8-1(a), que muestra 2 latas de concentrado de naranja
por cada 3 latas de agua y obtener la misma conclusión.
Del trabajo realizado en la sección 6.1, sabemos que 2>3 = 4>6 pues 2 # 6 = 3 # 4. Luego,
2>3 = 4>6 es una proporción. También que 2>3 Z 4>8 pues 2 # 8 Z 3 # 4 y esto no es una
proporción. En general, tenemos el siguiente teorema que se sigue de la propiedad de las
fracciones iguales desarrollada en la sección 6.1.

Teorema 8–1
Si a, b, c y d son todos números reales y b Z 0 y d Z 0, entonces
a c
= es una proporción si, y sólo si, ad = bc
b d

© Lopez Mateos Editores. ISBN 978-607-95583-2-1, obra completa, versión electrónica, ISBN 978-607-95583-3-8, volumen 1, versión electrónica. Ejemplar asignado a: Helecto Villarroel gutierrez -
helecto@gmail.com. Fecha: 27 de octubre de 2014. Prohibida su modificación, copia o distribución.
480 Razonamiento proporcional, porcentajes y aplicaciones

AHORA INTENTA ÉSTE 8-1 Justifica el teorema 8-1 multiplicando cada lado de la proporción por bd.

Los estudiantes de los primeros grados usualmente tienen problemas que son aditivos.
Considera el problema a continuación.
Ana y Beto teclean con la misma rapidez. Ana comenzó a teclear primero. Cuando Ana había captu-
rado 8 páginas, Beto había capturado 4. Cuando Beto haya capturado 10 páginas, ¿cuántas habrá cap-
turado Ana?
Éste es un ejemplo de relación aditiva. Los estudiantes deberían razonar que como las dos per-
sonas teclean con la misma rapidez, cuando Beto haya capturado 6 páginas adicionales, Ana ha-
brá capturado también 6 páginas más, es decir, ella habrá capturado 8 + 6, ó 14, páginas.
Ahora considera el siguiente problema:
Cali puede capturar 8 páginas por cada 4 páginas que captura Dina. Si Dina ha capturado 12 pági-
nas, ¿cuántas páginas ha capturado Cali?

Si los estudiantes usan un enfoque aditivo, concluirán que como Dina ha capturado 8 pági-
nas más que en la relación original, entonces Cali habrá capturado 8 páginas adicionales.
Sin embargo, el razonamiento correcto es que como Cali teclea el doble de rápido que
Dina, entonces capturará el doble de páginas que ésta. Por lo tanto, cuando Dina ha captu-
rado 12 páginas, Cali ha capturado 24. Esta relación entre razones es multiplicativa. Otra ma-
◆ Nota de nera de resolver este problema es construir la proporción =
8 x
, donde x es el número de pá-
investigación 8 8#3
4
24
12

El algoritmo del pro- ginas que captura Cali, y se despeja x. Como = # = , entonces x = 24 páginas.
4 4 3 12
ducto cruzado para
En el problema planteado en la cita de Principios y objetivos, falta un término en la proporción:
evaluar una propor-
paquetes : 3 x
ción (usando la igual- =
tazas : 15 60
dad de fracciones) es
(1) un algoritmo ex- Una manera de resolver la ecuación es multiplicar ambos lados por 60, como sigue:
tremadamente efi- 3 # x #
ciente pero mecánico 60 = 60
15 60
3#4
y sin significado, (2)
= x
usualmente se ma-
12 = x
linterpreta, (3) rara
vez los estudiantes lo Por lo tanto, se necesitan 12 paquetes de chocolate para hacer 60 tazas de chocolate caliente.
generan de manera 3 3#4 12
También pudimos resolverlo al notar que = = , de modo que x = 12. Otro
independiente, y (4) 15 15 # 4 60
a menudo se usa método de solución usa el teorema 8–1. También se le conoce como método de la multipli-
como un “medio de cación cruzada o método del producto cruzado. Esta ecuación es una proporción si, y sólo si,
evitar el razonamien-
to proporcional en 3 # 60 = 15x
lugar de facilitarlo” 180 = 15x
(Cramer and Post, 12 = x
1993; Post et al.
1988; Hart 1984; El método del producto cruzado (Teorema 8–1) es muy eficiente, pero nota los comenta-
Lesh et al. 1988). ◆ rios sobre este método señalados en la Nota de investigación.

© Lopez Mateos Editores. ISBN 978-607-95583-2-1, obra completa, versión electrónica, ISBN 978-607-95583-3-8, volumen 1, versión electrónica. Ejemplar asignado a: Helecto Villarroel gutierrez -
helecto@gmail.com. Fecha: 27 de octubre de 2014. Prohibida su modificación, copia o distribución.
Sección 8-1 Razones, proporciones y razonamiento proporcional 481

Ejemplo 8-2 Si hay 3 carros por cada 8 estudiantes en un bachillerato, ¿cuántos carros hay por 1200









estudiantes?

Solución Usamos la estrategia de construir una tabla, según se muestra en la tabla 8-2.

Tabla 8-2

Número de carros 3 x

Número de estudiantes 8 1200

La razón de carros a estudiantes es siempre la misma:


Carros : 3 x
=
Estudiantes : 8 1200
3 # 1200 = 8x
3600 = 8x
450 = x
Así, hay 450 carros.

En la página de muestra (p. 482) vemos el costo de renta de un carro por dos compañías di
1 2
ferentes. Las razones de la compañía QuéNegocio no forman una proporción pues Z .
20 35
Sin embargo, las razones de la compañía BuenosCarros sí forman una proporción porque
1 2 3 4
= = = . Para verificar las proporciones, en la página de muestra se usan pro-
20 40 60 80
ductos cruzados. Lee el ejemplo para ver que se prestó atención a las unidades. Resuelve el
Tema de plática al final de la página.
En la página de muestra vemos la frase “Decimos que estas propiedades varían propor-
cionalmente”. ¿Qué significa esto? Considera la tabla 8-3.
d 1 2 3 4
Las razones son todas iguales, esto es, = = = . Así, cada par de
Tabla 8-3 c 20 40 60 80
d 1
Días (d) 1 2 3 4 razones forma una proporción. En este caso, = para todos los valores de
c 20
20 40 60 80 c y d. Esto también se expresa diciendo que d es proporcional a c o que d varía
Costo (c)
1
proporcionalmente a c o que d varía directamente con c. En este caso, d = c para
20
1
todo c y d. El número es la constante de proporcionalidad. Por ejemplo, podemos decir
20
que la gasolina usada por un carro es proporcional a las millas recorridas o que las ganancias de la lo-
tería varían directamente con el número de boletos vendidos.

Definición
Si las variables x y y están relacionadas por la igualdad y = kx, ak =
b , entonces se dice que
y
x
y es proporcional a x y k es la constante de proporcionalidad entre y y x.

© Lopez Mateos Editores. ISBN 978-607-95583-2-1, obra completa, versión electrónica, ISBN 978-607-95583-3-8, volumen 1, versión electrónica. Ejemplar asignado a: Helecto Villarroel gutierrez -
helecto@gmail.com. Fecha: 27 de octubre de 2014. Prohibida su modificación, copia o distribución.
482 Razonamiento proporcional, porcentajes y aplicaciones

Página de un libro de texto COMPRENSIÓN DE PROPORCIONES

Lección 6-5

Comprensión
Idea clave Calentamiento
Una proporción es una Escribe=o≠.
declaración de que dos
razones son iguales. de proporciones
Vocabulario Aprende
• proporción
• productos cruzados ¿Qué es una proporción?
Una proporción establece que dos razones son iguales.
Tú puedes usar las dos primeras razones de Buenos Buenos
Carros para escribir una proporción: 1 día = 2 días . Carros
$20 $40 Días Costo
En una proporción, las unidades en las partes de
arriba deben ser iguales y las de abajo deben ser
iguales, en ambos lados de la igualdad. Aquí,
los días se encuentran arriba y el costo abajo.
Para BuenosCarros, las razones de días al costo
de renta de un carro son iguales. Decimos que
estas cantidades varían proporcionalmente. QuéNegocio
Las razones en QuéNegocio Ruedas NO son todas
iguales, de modo que estas cantidades NO varían Ruedas
proporcionalmente. Días Costo

En la proporción 1 = 2 , 1×40 y 20×2 son


20 40
productos cruzados. Los productos cruzados
de los términos de una proporción son iguales.

Ejemplo
Decide si las razones 3 8pies 9 pies
s y 24 s forman una proporción.
Lo que haces Por qué funciona
Mira las unidades. Multiplica ambos lados de
Las unidades son las mis- la proporción por 24×8 y
3 pies 9 pies
8s 24 s mas en la parte de arriba simplifica.
y en la parte de abajo.
Mira los productos cruzados.
Los productos productos
cruzados son iguales. cruzados

Como las unidades son las mismas y los productos cruzados son
iguales, las razones forman una proporción.

Tema de plática
¿Las razones 4 pies y 12 s forman una proporción? ¿Por qué sí o por qué no?
6 pies 18 s

Fuente: Scott Foresman-Addison Wesley Mathematics, Grade 6, 2008 (p. 316).

© Lopez Mateos Editores. ISBN 978-607-95583-2-1, obra completa, versión electrónica, ISBN 978-607-95583-3-8, volumen 1, versión electrónica. Ejemplar asignado a: Helecto Villarroel gutierrez -
helecto@gmail.com. Fecha: 27 de octubre de 2014. Prohibida su modificación, copia o distribución.
Sección 8-1 Razones, proporciones y razonamiento proporcional 483

O B S E R VA C I Ó N Una idea central en el razonamiento proporcional es que una relación


entre dos cantidades es tal que la razón de una cantidad a la otra permanece sin cambio
conforme los valores numéricos de ambas cantidades cambian.
a
Es importante recordar que en la razón a : b o , a y b no tienen por qué ser enteros. Por ejem-
b
7 3
plo, si en una ciudad, de la población se ejercita regularmente, entonces de la población
10 10
7 3
no se ejercita regularmente, y la razón de los que lo hacen a los que no es : . Esta razón
10 10
se puede escribir como 7 : 3.
Cuando trabajamos con proporciones, también es importante notar las unidades de
medición. Por ejemplo, si una tortuga viaja 5 pulg cada 10 s, ¿cuántos pies viajará en 50 s? Si
se ignoran las unidades de medición, podríamos formar la siguiente proporción:
5 x
=
10 50
En esta proporción no se listan las unidades. Una proporción con mayor información y que
con frecuencia previene errores es la siguiente:
5 pulg x pulg
=
10 s 50 s
25
Esto implica que x = 25. En consecuencia, como 12 pulg = 1 pie, la tortuga viaja pies, o
12
1
2 pies, o 2 pies 1 pulg.
12
Los Principios y objetivos señalan lo siguiente respecto al método de multiplicación cruzada
para resolver proporciones.

La instrucción para resolver proporciones deberá incluir métodos que tengan una fuerte base intui-
tiva. El llamado método de multiplicación cruzada se puede desarrollar con sentido si surge de ma-
nera natural en el trabajo del estudiante, pero también puede tener desafortunados efectos laterales
cuando los estudiantes no entienden de manera adecuada cuándo es apropiado usar el método.
Otros enfoques para resolver proporciones son a menudo más intuitivos y también bastante pode-
rosos. Por ejemplo, para decidir sobre cuál es una mejor compra —12 boletos por $15.00 ó 20 bole-
tos por $23.00— los estudiantes pueden usar una estrategia de escalamiento (hallar el costo de un
número común de boletos) o la estrategia de la razón unitaria (hallar el costo de un boleto). (p. 221)
La estrategia de escalamiento para resolver el problema incluye hallar el costo de un
número común de boletos. Como el MMC112, 202 = 60, podemos decidir hallar el costo de
60 boletos bajo cada plan.
En el primer plan, como 12 boletos cuestan $15, entonces 60 boletos cuestan $75.
En el segundo plan, como 20 boletos cuestan $23, entonces 60 boletos cuestan $69.
Por lo tanto, el segundo plan constituye una mejor compra.
La estrategia de la razón unitaria para resolver este problema incluye hallar el costo de
un boleto bajo cada plan y después comparar el costo unitario.
En el primer plan, como 12 boletos cuestan $15, entonces 1 boleto cuesta $1.25.
En el segundo plan, como 20 boletos cuestan $23, entonces 1 boleto cuesta $1.15.

© Lopez Mateos Editores. ISBN 978-607-95583-2-1, obra completa, versión electrónica, ISBN 978-607-95583-3-8, volumen 1, versión electrónica. Ejemplar asignado a: Helecto Villarroel gutierrez -
helecto@gmail.com. Fecha: 27 de octubre de 2014. Prohibida su modificación, copia o distribución.
484 Razonamiento proporcional, porcentajes y aplicaciones

Por lo tanto, el segundo plan constituye una mejor compra. Nota la sugerencia para el uso de
la estrategia de la razón unitaria en la siguiente Nota de investigación.

◆ Nota de El método de la razón unitaria es el mejor método para trabajar con problemas que incluyan
investigación razones y proporciones. El método de la razón unitaria se sugiere fuertemente como “anda-
miaje” para construir razonamientos proporcionales (Post et al., 1988). ◆

Ejemplo 8-3 Carlos, Pablo y Susana obtuvieron $2520 por pintar una casa. Carlos trabajó 30 h, Pablo








trabajó 50 h y Susana trabajó 60 h. Ellos se dividieron el dinero en proporción al número de


horas trabajadas. Si todos ganan el mismo salario por hora, ¿cuánto ganó cada uno?

Solución Sea x la razón unitaria o la razón de pago por hora. Entonces 30x denota la can-
tidad de dinero que recibió Carlos, y Pablo recibió 50x porque así, y sólo así, la razón de las
cantidades será igual que 30 : 50, como se requiere. De manera análoga, Susana recibió 60x.
Como la cantidad total de dinero recibido es 30x + 50x + 60x, tenemos
30x + 50x + 60x = 2520
140x = 2520
x = 18
Por lo tanto, Carlos recibió 30x = 30 # 18, ó $540
Pablo recibió 50x = 50 # 18, ó $900
Susana recibió 60x = 60 # 18, ó $1080
Dividiendo cada una de las cantidades entre 18, se demuestra que la proporción es la requerida.

15 3
Considera la proporción = . Como las razones de la proporción son fracciones
30 6
iguales y como fracciones iguales, distintas de cero, tienen recíprocos iguales, se sigue que
30 6
= . Nota además que las proporciones son verdaderas pues cada una resulta en
15 3
15 # 6 = 30 # 3. En general, tenemos la siguiente propiedad.

Teorema 8–2
a c a c b d
Para números racionales cualesquiera y , con a Z 0 y c Z 0, = si, y sólo si, = .
b d b d a c

15 3 15 30
Considera de nuevo = . Nota que = ; esto es, la razón de los numeradores
30 6 3 6
es igual a la razón de los denominadores correspondientes. En general, tenemos el teo-

rema siguiente.

Teorema 8–3
a c a c a b
Para números racionales cualesquiera y , con b, c, d Z 0, = si, y sólo si, = .
b d b d c d

© Lopez Mateos Editores. ISBN 978-607-95583-2-1, obra completa, versión electrónica, ISBN 978-607-95583-3-8, volumen 1, versión electrónica. Ejemplar asignado a: Helecto Villarroel gutierrez -
helecto@gmail.com. Fecha: 27 de octubre de 2014. Prohibida su modificación, copia o distribución.
Sección 8-1 Razones, proporciones y razonamiento proporcional 485

Dibujos a escala
Las razones y proporciones se usan en los dibujos a escala. Por ejemplo, si la escala es 1 : 300,
entonces la longitud de 1 cm en dicho dibujo representa 300 cm ó 3 m en tamaño real. La
escala es la razón del tamaño del dibujo al tamaño del objeto. El ejemplo siguiente muestra el
uso de los dibujos a escala.

En la figura 8-3 el plano de planta del piso principal de una casa está dibujado a escala de
Ejemplo 8-4
1 : 300. Halla las dimensiones de la sala en metros.








Cocina/Desayunador
Recámara principal

Baño
Sala
Estudio

Figura 8-3

Solución En la figura 8-3, las dimensiones de la sala medidas con una regla de centímetros
son aproximadamente 3.7 cm por 2.5 cm. Como la escala es de 1 : 300, 1 cm en el dibujo re-
presenta 300 cm, ó 3 m en tamaño real. Por lo tanto, 3.7 cm representan 3.7 # 3, u 11.1 m, y
2.5 cm representan 2.5 # 3, ó 7.5 m. Por lo tanto, las dimensiones de la sala son de aproxima-
damente 11.1 m por 7.5 m.

Evaluación 8-1A

1. Responde lo siguiente respecto al alfabeto en español: 3. Despeja x en cada una de las proporciones siguientes:
a. Determina la razón de las vocales a las consonantes. 12 18 x -10
a. = b. =
b. ¿Cuál es la razón de las consonantes a las vocales? x 45 7 21
c. ¿Cuál es la razón de las consonantes a las letras del al- 5 3x 1
fabeto español? c. = d. 3 es a 5 como x es a 15.
7 98 2
d. Escribe una palabra que tenga razón 2 : 3 de vocales a 4. Hay aproximadamente 2 lb de músculo por cada 5 lb de
consonantes. peso. ¿Aproximádamente cuánto peso muscular tiene
2. a. Si la razón de niños a niñas en un grupo es de 2 : 3, una persona de 90 lb?
¿cuál es la razón de los niños a todos los estudiantes del
5. ¿Cuál es mejor compra: 4 toronjas por 80¢ o 12 toronjas
grupo? ¿Por qué?
por $1.80?
b. Si la razón de niños a niñas en un grupo es m : n, ¿cuál es
1
la razón de los niños a todos los estudiantes del grupo? 6. En un mapa, pulg representan 5 mi. Si Nueva York y Aca-
3 3
c. Si del grupo son niñas, ¿cuál es la razón de niñas a xao distan 18 pulg en el mapa, ¿cuál es la distancia real
5
entre ellas?
niños?

© Lopez Mateos Editores. ISBN 978-607-95583-2-1, obra completa, versión electrónica, ISBN 978-607-95583-3-8, volumen 1, versión electrónica. Ejemplar asignado a: Helecto Villarroel gutierrez -
helecto@gmail.com. Fecha: 27 de octubre de 2014. Prohibida su modificación, copia o distribución.
486 Razonamiento proporcional, porcentajes y aplicaciones

7. David lee 40 páginas de un libro en 50 min. ¿Cuántas pági- a. La razón de rpm del engranaje mayor al engranaje
nas podrá leer en 80 min si lee a una velocidad constante? menor es de 4 : 6. Si el engranaje menor tiene 18
8. Dos números están a una razón de 3 : 4. Halla los números si dientes, ¿cuántos dientes tiene el engranaje mayor?
a. su suma es 98. b. El engranaje mayor gira a 200 rpm y tiene 60 dientes.
b. su producto es 768. ¿Cuántos dientes hay en el engranaje menor, que gira
9. Gabriel, Marcelo y Carmelita invirtieron en una compañía a 600 rpm?
a razón de 2 : 4 : 5, respectivamente. Si se dividen las ga- 16. Un jet Boeing 747 tiene aproximadamente 230 pies de largo
nancias de $82,000 proporcionalmente a su inversión, y una envergadura (la distancia entre los extremos de las alas)
¿cuánto recibirá cada uno aproximadamente? de 195 pies. Si un modelo a escala del avión tiene alrededor
1 1 de 40 cm de largo, ¿cuál es la envergadura del modelo?
10. Susana y Dora trabajaron 3 h y 4 h, respectivamente,
2 2 17. Juanita pesa 160 lb en la Tierra y 416 lb en Júpiter. Halla
en un proyecto de programación. Les pagaron $176 por el peso de Ana en Júpiter si ella pesa 120 lb en la Tierra.
el proyecto. ¿Cuánto ganó cada una si les pagan el 18. Una receta pide 1 cucharada de semillas de mostaza, 3
mismo salario por hora? 1 1
11. Esther logró 75 anotaciones en su práctica de tiro a gol. Si tazas de salsa de tomate, 1 de cebolla rebanada y 3
2 4
su tasa de acierto-falla es de 5 : 4, ¿cuántos tiros realizó? tazas de habas. Si se altera la cantidad de un ingre-
12. Expresa cada uno de los casos siguientes como una razón diente como se indica, ¿cómo hay que cambiar los otros
a
, donde a y b son números enteros: ingredientes para mantener las proporciones? Explica
b tu razonamiento.
1
a. : 1 a. 2 tazas de salsa de tomate
6
1 1 b. 1 tazas de cebolla rebanada
b. : 3
3 3 c. 1 tazas de habas
1 2 4
c. : 19. La resistencia eléctrica de un alambre, medida en ohms
1Æ2, es proporcional a la longitud del alambre. Si la re-
6 7
13. Usa los teoremas 8–2 y 8–3 para escribir otras tres pro-
sistencia eléctrica de un alambre de 5 pies es de 4.2 Æ ,
porciones que se sigan de la siguiente proporción:
¿cuál es la resistencia de 18 pies del mismo alambre?
12¢ 16¢ 20. En una fotografía de un padre y su hija, la altura de la
=
36 oz 48 oz hija es de 2.3 cm y la del padre es de 5.8 cm. Si el padre
14. La altura y el ancho del techo de una casa se ilustran en mide en realidad 188 cm, ¿cuánto mide la hija?
la figura. La pendiente de un techo es la razón de la al- 21. La cantidad de oro para joyería y para otros productos se
tura a la mitad del ancho. mide en quilates (K), donde 24K representa el oro puro.
a. Si la altura es de 10 pies y el ancho es de 28 pies, ¿cuál La marca 14K en una cadena indica que la razón entre la
masa del oro en la cadena y la masa de la cadena es de
14 : 24. Si un anillo de oro tiene la marca 18K y pesa 0.4
oz, ¿cuál es el valor del oro en el anillo si el oro puro está
Altura valuado en $300 la onza?
22. A Emma le pagan $8.00 la hora de mecanografía, y en la
siguiente tabla se muestran sus ingresos.
a. ¿Cuánto gana Emma por un trabajo de 40 h semanales?

Horas (h) 1 2 3 4 5

Salarios (s) 8 16 24 32 40
Ancho
b. ¿Cuál es la constante de proporcionalidad?
es la pendiente? 23. a. En el salón A del Centro Universitario hay 1 hombre
3 y 2 mujeres; en el salón B hay 2 hombres y 4 mujeres;
b. Si el ancho es de 16 pies y la pendiente es de , ¿cuál es
4 y en el salón C hay 5 hombres y 10 mujeres. Si todas
la altura?
las personas en los salones B y C van al salón A, ¿cuál
15. En la industria se usan razones de engranaje. Una razón es la razón de hombres a mujeres en el salón A?
de engranaje es la comparación del número de dientes en b. Demuestra la siguiente generalización a las propor-
dos engranajes. Cuando se combinan dos engranajes, las ciones usadas en (a):
revoluciones por minuto (rpm) son inversamente pro- a c e a c e a + c + e
porcionales al número de dientes; esto es, Si = = , entonces = = =
b d f b d f b + d + f
rpm en el engranaje mayor número de dientes en el engranaje menor
=
rpm en el engranaje menor número de dientes en el engranaje mayor

© Lopez Mateos Editores. ISBN 978-607-95583-2-1, obra completa, versión electrónica, ISBN 978-607-95583-3-8, volumen 1, versión electrónica. Ejemplar asignado a: Helecto Villarroel gutierrez -
helecto@gmail.com. Fecha: 27 de octubre de 2014. Prohibida su modificación, copia o distribución.
Sección 8-1 Razones, proporciones y razonamiento proporcional 487

Evaluación 8-1B

1. Responde lo siguiente respecto a las letras de la palabra b. ¿Cuál es la relación algebraica entre L y A?
Mississippi. c. Escribe A como función de L; esto es, expresa A en
a. Determina la razón de las vocales a las consonantes. términos de L.
b. ¿Cuál es la razón de las consonantes a las vocales? d. Escribe L como función de A; esto es, expresa L en
c. ¿Cuál es la razón de las consonantes a las letras de la términos de A.
palabra? 11. Halla tres conjuntos de valores de x y de y para lo siguiente:
2. Despeja x en cada una de las proporciones siguientes:
- 12 4 boletos x boletos
5 30 x =
a. = b. = $20 $y
x 42 8 32
7 3x 1 12. Si la renta es de $850 por 2 semanas, ¿cuánto es la renta
c. = d. 3 es a 8 como x es a 24
8 48 2 por 7 semanas?
3. En Acaxao hay 5 adultos que conducen por cada adoles- 13. Leonardo da Vinci, en su dibujo El hombre de Vitruvio,
cente que conduce. Si hay 12,345 conductores adultos en mostró que la medida de los brazos abiertos de un hom-
Acaxao, ¿cuántos conductores adolescentes hay? bre era igual a su altura. A continuación listamos otras
4. Una vela tiene 30 pulg de largo. Después de consumirse razones.
durante 12 min, la vela tiene 25 pulg de largo. ¿En Longitud de la mano 7
=
cuánto tiempo se consumirá la vela a la misma tasa? Longitud del pie 9
5. Un jardín rectangular tiene una razón ancho a largo de Distancia del codo a la punta de la mano 8
5 : 9. Si la distancia alrededor del jardín es de 2800 pies, =
¿cuáles son las dimensiones del jardín? Distancia del hombro al codo 5
6. Un saltamontes puede saltar 20 veces su longitud. Si la ha- Longitud de la mano 14
bilidad de los humanos para saltar fuera proporcional a la de =
Longitud del dedo gordo del pie 3
un saltamontes, ¿cuánto podría saltar una persona de 6 pies?
7. Jaime vio que después de trabajar durante 9 meses había Usando las razones anteriores, responde a lo siguiente:
ganado 6 días de vacaciones. A esta razón, ¿cuántos días a. Si la longitud del dedo gordo del pie es de 6 cm,
al año tiene vacaciones? ¿cuánto deberá medir la mano?
8. En una escuela la razón maestro-estudiante es de 1 : 30. Si b. Si la mano mide 21 cm, ¿cuánto mide el pie?
la escuela tiene 1200 estudiantes, ¿cuántos maestros adicio- c. Si la distancia del codo al final de la mano es de 20 pulg,
nales deben contratarse para reducir la razón a 1 : 20? ¿cuál es la distancia del hombro al codo?
9. A determinada hora del día, la razón de la altura de un 14. En un mapa de la ciudad, un parque rectangular tiene 4 pulg
objeto perpendicular a la superficie a la longitud de su de largo. Si la longitud real del largo y ancho del parque es
sombra es la misma en todos los objetos. Si un árbol de de 500 pies y 300 pies, respectivamente, ¿cuánto es el ancho
30 pies proyecta una sombra de 12 pies, ¿cuál es la altura del parque en el mapa?
de un árbol que proyecta una sombra de 14 pies? 15. El carro de Jaime viajará 240 mi con 15 gal de gasolina.
10. La siguiente tabla muestra posibles anchos A y las longitu- ¿Qué distancia espera recorrer con 3 gal de gasolina?
des correspondientes L de un rectángulo cuya área es de 10 16. Algunos modelos de vías férreas usan una escala O en sus
pies cuadrados réplicas de los trenes reales. La escala O usa una razón de
1 pulg/48 pulg. ¿Cuántos pies mide una locomotora real
Ancho (A) Longitud (L) Área si la réplica en la escala O mide 18 pulg de largo?
(pies) (pies) (pies cuadrados) 17. a. En una bandera de Estados Unidos, ¿cuál es la razón
de las estrellas a las barras?
0.5 20 0.5 # 20 = 10 b. ¿Cuál es la razón de las barras a las estrellas?
1 10 1 # 10 = 10 L 18. En una bandera de Estados Unidos, la razón del largo de
2 5 2#5 = 10 Área = 10 pie2 A la bandera a su ancho debe ser de 19 : 10.
2.5 4 2.5 # 4 = 10 a. Si una bandera tiene 9 12 pies de largo, ¿cuánto debe
4 2.5 4 # 2.5 = 10 tener de ancho?
5 2 5#2 = 10 b. La bandera que se colocó en la Luna medía 5 pies por
10 1 10 # 1 = 10 3 pies. ¿Esta razón forma una proporción con la
20 0.5 20 # 0.5 = 10 razón oficial longitud a ancho? ¿Por qué?
a. Usa los valores de la tabla y algunos valores adiciona- x a
19. Si es cierto que = , ¿qué otras proporciones sabes
les para graficar la longitud L en el eje vertical contra y b
el ancho A en el eje horizontal. que también son ciertas?

© Lopez Mateos Editores. ISBN 978-607-95583-2-1, obra completa, versión electrónica, ISBN 978-607-95583-3-8, volumen 1, versión electrónica. Ejemplar asignado a: Helecto Villarroel gutierrez -
helecto@gmail.com. Fecha: 27 de octubre de 2014. Prohibida su modificación, copia o distribución.
488 Razonamiento proporcional, porcentajes y aplicaciones

1 a c
20. Si cierta receta pide 1 t de harina y 2 t de leche, ¿cuánta 夝21. Demuestra que si = , a Z - b, y a Z b, entonces los
2 b d
leche llevará si sólo se dispone de 1 t de harina? siguiente siempre es verdadero:

a Sugerencia:
a + b c + d a c
a. = + 1 = + 1b
b d b d
a c a - b c - d
b. = c. =
a + b c + d a + b c + d

Conexiones matemáticas 8-1

Comunicación 10. Halla ejemplos de razones en periódicos.


1. Iris encontró algunos huesos de dinosaurio y una huella 11. La ley de Boyle dice que a una temperatura dada, el pro-
fósil. La longitud de la huella es de 40 cm, la longitud del ducto del volumen V de un gas y la presión P es una cons-
fémur es de 100 cm y la longitud del cuerpo es de 700 cm. tante c, como sigue:
a. ¿Cuál es la razón de la longitud de la huella a la longi- PV = c
tud del dinosaurio? a. Si a una temperatura dada, una presión de 48 lb>pulg2
b. Iris halló un nuevo rastro que supone fue hecho por la comprime cierto gas a un volumen de 960 pulg3, ¿qué
misma especie de dinosaurio. Si la huella tenía 30 cm presión sería necesaria para comprimir el gas a un vo-
de largo y si vale la misma razón de longitud de pie a lumen de 800 pulg3 a la misma temperatura?
longitud de cuerpo, ¿cuál es la longitud del dinosaurio? b. Halla otras tres situaciones del mundo real en que las
c. En la misma área, Iris también halló un fémur de variables estén relacionadas matemáticamente como lo
50 cm. ¿Crees que este fémur perteneció al mismo di- están en la ley de Boyle. En cada caso, describe cómo
nosaurio que dejó la huella de 30 cm hallada por Iris? se relacionan las variables usando razón y proporción.
¿Por qué sí o por qué no? 12. Investiga la razón áurea que usaron los griegos para dise-
2. Supón que una pizza de 10 pulg cuesta $4. Para que ob- ñar el Partenón. Escribe un reporte sobre esta razón e in-
tengas el precio x de una pizza de 14 pulg, ¿es correcto cluye el dibujo de un rectángulo áureo.
x 14
formar la proporción = ? (Supón que las razones de Aprendizaje colectivo
4 10
las áreas permanecen iguales.) ¿Por qué sí o por qué no? 13. En Los viajes de Gulliver de Jonathan Swift hallamos lo
3. Cuando usas una proporción para resolver un problema siguiente:
de dibujo a escala, ¿es posible expresar la proporción en
Los sastres tomaron mis medidas mientras yo yacía en tierra, uno
más de una manera? Explica.
parado en mi cuello y otro en mi rodilla, con una fuerte cuerda
4. Nelly dijo que le basta ver las razones 15 : 7 y 15 : 8 para
extendida, que cada uno sostenía por el extremo, mientras el ter-
saber que no forman una proporción. ¿Está en lo correcto?
cero medía la longitud de la cuerda con una regla de una pulgada
¿Por qué?
de largo. Después midieron mi pulgar derecho y nada más; por
5. Sol tenía fotografías de 4 pulg por 6 pulg, de 5 pulg por
medio de un cálculo matemático, de que el doble de la circunfe-
7 pulg y de 8 pulg por 10 pulg. ¿Varían proporcional-
rencia del pulgar es una vez la muñeca y así hasta el cuello y la
mente las dimensiones? Explica por qué.
cintura; y con ayuda de mi vieja camisa, que extendí en la tierra
a a + b
6. ¿Es posible que y formen una proporción? ¿Por qué? delante de ellos como patrón, me vistieron perfectamente.
b b
2 a. Explora las mediciones de tus compañeros para ver si
夝 7. En una unidad habitacional, 3 de los hombres están casa- te parecen bien las razones mencionadas para Gulliver.
3
dos con de las mujeres. ¿Cuál es la razón de las personas b. Supón que la distancia alrededor del pulgar de una per-
4 sona es de 9 cm. ¿Cuál es la distancia alrededor del
casadas a la población adulta total de la unidad? Explica cuello de esa persona?
cómo puedes obtener la razón sin saber en realidad el nú- c. ¿Qué razón podría usarse para comparar la altura de
mero de hombres o de mujeres. una persona con la extensión de sus brazos abiertos?
Solución abierta ¿Tiene algo que ver esta razón con el famoso dibujo de
da Vinci El hombre de Vitruvio?
8. Escribe un párrafo en donde uses correctamente los tér-
d. ¿Crees que haya una razón entre la longitud de los pies
minos razón y proporción.
y la estatura? Si es así, ¿cuál podría ser?
9. Lista tres situaciones del mundo real que involucren razón
e. Estima otras razones del cuerpo y después fíjate cuán
y proporción.
cerca estás de las medidas reales.

© Lopez Mateos Editores. ISBN 978-607-95583-2-1, obra completa, versión electrónica, ISBN 978-607-95583-3-8, volumen 1, versión electrónica. Ejemplar asignado a: Helecto Villarroel gutierrez -
helecto@gmail.com. Fecha: 27 de octubre de 2014. Prohibida su modificación, copia o distribución.
Sección 8-1 Razones, proporciones y razonamiento proporcional 489

Preguntas del salón de clases Tres hermanos, Roberto, Daniel y Marcos, recibieron un
14. Mary está trabajando con mediciones y escribe la si- regalo de 45,000 zeds de su padre. El dinero se reparte en-
guiente proporción: tre los hermanos en proporción al número de hijos que
tiene cada uno. Roberto tiene 2 hijos, Daniel tiene 3 hijos
12 pulg>1 pie = 5 pies>60 pulg. y Marcos tiene 4 hijos. ¿Cuántos zeds obtuvo Marcos?
¿Cómo la ayudarías? a. 5,000
15. Nora dijo que puede usar la división para ver si dos razones b. 10,000
forman una proporción; por ejemplo, 32 : 8 y 40 : 10 forman c. 15,000
una proporción porque 32 , 8 = 4 y 40 , 10 = 4. ¿Está d. 20,000
ella en lo correcto? ¿Por qué? TIMSS 2003, Grado 8
a a c Pregunta del National Assessment of Educational Progress
16. José reportó que si es una fracción propia y = , enton-
b b d (NAEP) (Evaluación Nacional del Progreso Educativo)
d b
ces = . Juana dijo que no le creía. ¿Cómo Sara tiene un trabajo de medio tiempo en un restaurante y
c - d a - b le pagan $5.50 por cada hora que trabaja. Ella hizo la ta-
respondes? bla siguiente para reflejar sus ganancias, pero necesita que
17. Ale mide 5 pies de altura y tiene una sombra de18 pulg de le ayudes a completarla.
largo. Al mismo tiempo, un árbol tiene una sombra de 15 a. Llena los registros faltantes de la tabla.
pies de largo. Ale construyó y resolvió la proporción si-
guiente:
Horas trabajadas Dinero ganado
5 pies 18 pulg
= , luego x = 54 pulg. 1 $5.50
15 pies x pulg
¿Cómo lo ayudarías? 4
18. Una amiga de Ana le dijo que la razón de niñas a niños en $38.50
su clase es de 5 : 6. Ana quedó muy sorprendida al pensar
que su clase tendría sólo 11 estudiantes. ¿Qué le dices? 3
7 $42.63
Preguntas del Third International Mathematics and 4
Science Study (TIMSS) (Tercer Estudio Internacional
sobre las Matemáticas y la Ciencia) b. Si Sara trabaja h horas, entonces, en términos de h,
Por cada botella de refresco que recolecta Federico, Ma- ¿cuánto ganará?
ría recolecta 3. Federico recolectó un total de 9 botellas. NAEP, 2007, Grado 8
¿Cuántas recolectó María?
a. 3 b. 12 c. 13 d. 27
TIMSS 2003, Grado 4

ROMPECABEZAS Micaela leyó que el brazo de la Estatua de la Libertad mide 42 pies de largo. A ella
le gustaría saber cuánto mide la nariz. ¿Cómo le sugieres proceder?

8-2 Porcentajes

Los porcentajes son muy útiles para transmitir información. Se escucha que hay un 60% de posi-
bilidad de lluvias o que nuestros ahorros producen un 6% de interés anual. Los porcentajes son
un tipo especial de fracciones, a saber, fracciones con denominador de 100. La expresión por
ciento viene de la frase en latín per centum. La raíz cent aparece en palabras como centuria (100
años), centímetro a
1
mb y ciempiés (100 patas). Un banco que paga 6% anual de interés
100
simple en una cuenta de ahorros, paga $6 por cada $100 en la cuenta por 1 año; esto es,

© Lopez Mateos Editores. ISBN 978-607-95583-2-1, obra completa, versión electrónica, ISBN 978-607-95583-3-8, volumen 1, versión electrónica. Ejemplar asignado a: Helecto Villarroel gutierrez -
helecto@gmail.com. Fecha: 27 de octubre de 2014. Prohibida su modificación, copia o distribución.
490 Razonamiento proporcional, porcentajes y aplicaciones

paga 6>100 de cualquier cantidad que esté en la cuenta por un año. El símbolo % indica por
ciento; 6% significa 6 de cada 100. Luego, para hallar el 6% de $400, determinamos
cuántos cientos hay en 400. Hay 4 cientos en 400, de modo que 6% de 400 es 6 # 4 = 24.
Por lo tanto, 6% de $400 = $24.

Definición de porcentaje
n
n% =
100

n
Así, n% de una cantidad es de esa cantidad. Por lo tanto, 1% es un centésimo del todo
100
200
y 100% representa la cantidad completa, mientras que 200% representa , o 2 veces, la
100
cantidad dada. Los porcentajes se pueden ilustrar usando una malla de cientos. Por ejem-
plo, ¿qué porcentaje de la malla está sombreado en la figura 8-4? Como 30 de los 100,
30
ó , de los cuadros está sombreado, decimos que el 30% de la malla está sombreado (o
100
Figura 8-4 de manera similar, el 70% de la malla no está sombreado).

AHORA INTENTA ÉSTE 8-2  Escribe la fracción simplificada y el porcentaje que representa la porción
sombreada en cada parte de la figura 8-5.

(a) (b) (c) (d)

Figura 8-5

n
Como n% = , para convertir un número a un porcentaje lo escribimos como una
100
fracción con denominador 100; el numerador da la cantidad del porcentaje. Por ejemplo,
3 3 # 25 75 3
= # = . Por lo tanto, = 75%.
4 4 25 100 4

Ejemplo 8-5 Escribe cada caso como un porcentaje:










a. 0.03 b. 0.3 c. 1.2 d. 0.00042


3 2 1
e. 1 f. g. h. 2
5 3 7

b =
0.03 3
Solución a. 0.03 = 100a = 3%
100 100

b =
0.3 33.3
b. 0.3 = 100a = 33.3%
100 100

b =
1.2 120
c. 1.2 = 100a = 120%
100 100

© Lopez Mateos Editores. ISBN 978-607-95583-2-1, obra completa, versión electrónica, ISBN 978-607-95583-3-8, volumen 1, versión electrónica. Ejemplar asignado a: Helecto Villarroel gutierrez -
helecto@gmail.com. Fecha: 27 de octubre de 2014. Prohibida su modificación, copia o distribución.
Sección 8-2 Porcentajes 491

b =
0.00042 0.042
d. 0.00042 = 100a = 0.042%
100 100

b =
1 100
e. 1 = 100a = 100%
100 100

a b
3

= 100 J K =
3 5 60
f. = 60%
5 100 100

a b a b
2 200

= 100 J K =
2 3 3 66.6
g. = = 66.6%
3 100 100 100

a2 b a b
1 1500

h. 2 = 100 J K =
1 7 7 1500 2
= %, ó 214 %
7 100 100 7 7

Un número también se puede convertir en porcentaje usando una proporción. Por ejemplo,
3
para escribir como porcentaje hallamos el valor de n en la siguiente proporción:
5
3 n
=
5 100

a b100 = n
3
5
n = 60
Por lo tanto,
3
= 60%
5
Otra manera de convertir un número a porcentaje es recordar que 1 = 100%. Por ejemplo,
3 3 3 3
= de 1 = # 1 = # 100% = 75%.
4 4 4 4

O B S E R VA C I Ó N El símbolo % es crucial para identificar el significado de un número.


1 1 1 1
Por ejemplo, y % son números diferentes: = 50%, lo cual no es igual a %. De
2 2 2 2
manera análoga, 0.01 es diferente de 0.01%, que es 0.0001.

En el ejemplo de la siguiente página de muestra sobre la obtención de porcentajes, se ve un


método algorítmico para convertir una proporción a un porcentaje. Completa el ejemplo.
En los Principios y objetivos, hallamos lo siguiente:
Así como con las fracciones y los decimales, las dificultades conceptuales deben atenderse cuida-
dosamente en la enseñanza. En particular, porcentajes menores que 1% y mayores que 100% a me-
nudo presentan dificultad. (p. 217)
A veces es útil convertir porcentajes a decimales. Esto se puede hacer escribiendo el por-
n
centaje como una fracción de la forma y luego convirtiendo la fracción en un decimal.
100

© Lopez Mateos Editores. ISBN 978-607-95583-2-1, obra completa, versión electrónica, ISBN 978-607-95583-3-8, volumen 1, versión electrónica. Ejemplar asignado a: Helecto Villarroel gutierrez -
helecto@gmail.com. Fecha: 27 de octubre de 2014. Prohibida su modificación, copia o distribución.
492 Razonamiento proporcional, porcentajes y aplicaciones

Página de un libro de texto O BT E N E R P O RC E N TA J ES

Objetivo
Aprende cómo . . .
Ex loración
hallar porcentajes

Obtener porcentajes
Mientras . . .
obtienes índices
de aprobación

Requerimientos Necesitas la tabla de frecuencia de la pregunta 2 en


“Preparando el escenario”.

Trata esto como una clase Supón que un grupo otorgó a dos películas
1 ó más estrellas según se muestra en las razones siguientes.
32

Película A Película B

Estima el índice de aprobación del público para cada película.


Explica el razonamiento que usaste.
Usando estimación, ¿puedes determinar cuál película tuvo
mayor índice de aprobación? Explica.

Un modelo de barra de porcentaje te puede ayudar a ver cómo establecer


una proporción para obtener el porcentaje exacto equivalente a una razón.

Ejemplo

Establece una proporción para obtener el índice de aprobación


19 .
para la película A, representado por la razón 30

Respuesta de muestreo

La parte sombreada representa a los Porcentaje Estudiantes


estudiantes a quienes gustó la película.
parte todo
Estudiantes
parte

Porcentaje todo

Gira la barra 90º para que las


partes queden sobre los todos. Proporción
parte parte
todo todo

Módulo 5 Recreación

Fuente: Math Thematics, Book 2 (Grade 7), McDougal Littell, 2008, (p. 362).

© Lopez Mateos Editores. ISBN 978-607-95583-2-1, obra completa, versión electrónica, ISBN 978-607-95583-3-8, volumen 1, versión electrónica. Ejemplar asignado a: Helecto Villarroel gutierrez -
helecto@gmail.com. Fecha: 27 de octubre de 2014. Prohibida su modificación, copia o distribución.
Sección 8-2 Porcentajes 493

Ejemplo 8-6 Escribe cada porcentaje siguiente como un decimal: 5


Solución a.5% = = 0.05 b.
100









a. 5% b. 6.3% c. 100% 6.3
2 1 6.3% = = 0.063
d. 250% e. % f. 33 % 100
3 3
100
c. 100% = = 1 d.
100
250
250% = = 2.5
100
2 1
33
2 3 0.6 1 3 33.3
e. % = = = 0.006 f. 33 % = = = 0.3
3 100 100 3 100 100

Otro enfoque para escribir un porcentaje como decimal es convertir primero 1% a decimal.
1
Como 1% = = 0.01, podemos concluir que 5% = 5 # 0.01 = 0.05 y que 6.3% =
100
6.3 # 0.01 = 0.063.

AHORA INTENTA ÉSTE 8-3


a. Investiga cómo maneja los porcentajes tu calculadora y di lo que hace cuando se presiona la tecla % .
1
b. Usa tu calculadora para transformar en un porcentaje.
3

Aplicaciones que incluyen porcentajes


En los puntos focales para el grado 7 hallamos lo siguiente:

Ellos (los estudiantes) usan razones y proporcionalidad para resolver una amplia variedad de
problemas de porcentaje, incluyendo problemas que incluyen descuentos, intereses, impuestos,
propinas e incremento o decremento porcentual. (p. 19)

Los problemas de aplicación que incluyen porcentajes usualmente toman una de las for-
mas siguientes:
1. Hallar un porcentaje de un número
2. Hallar qué porcentaje es un número de otro
3. Hallar un número cuando se conoce un porcentaje de ese número
Antes de considerar ejemplos que ilustren estas formas, recuerda lo que significa hallar una
2 2
fracción “de” un número. Por ejemplo, de 70 significa # 70. De manera análoga, para ha-
3 3
40 40 #
llar 40% de 70 tenemos de 70, lo cual significa 70, ó 0.40 # 70 = 28.
100 100

© Lopez Mateos Editores. ISBN 978-607-95583-2-1, obra completa, versión electrónica, ISBN 978-607-95583-3-8, volumen 1, versión electrónica. Ejemplar asignado a: Helecto Villarroel gutierrez -
helecto@gmail.com. Fecha: 27 de octubre de 2014. Prohibida su modificación, copia o distribución.
494 Razonamiento proporcional, porcentajes y aplicaciones

Una manera diferente de pensar acerca del 40% de 70 es considerar que 70 representa
100 partes (o el total) y 40% sólo requiere 40 de esas 100 partes. Por ejemplo, si
100 partes = 70

1 parte = a b, ó 0.7
70
100
40 partes = 4010.72, ó 28
Así, 40% de 70 = 28.

◆ Nota de En un estudio realizado a estudiantes japoneses de grados 4, 6 y 8, sólo un bajo porcentaje de


investigación estudiantes contestaron correctamente a “¿Cuál es el 100% de 48?” Se amplió la investigación
para determinar si este comportamiento era representativo del conocimiento sobre porcentajes
de los estudiantes japoneses. Ninguno de los estudiantes realizó comentarios para sugerir al-
guna liga, relación o similitud conceptual entre operaciones con fracciones y con decimales
(Reys et al., 1995). ◆

Como se vio en la Nota de investigación, los estudiantes japoneses tuvieron dificultades


para calcular el 100% de 48. Se puede usar la barra de porcentaje introducida en la página
de muestra (p. 492) como un modelo para comprender lo que significa el 100% de un
número, así como para comprender otros porcentajes. En la figura 8-6, considera la barra
de porcentajes que representa el 100% del total con 40% del total sombreado. Nota que
100% de la barra representa 70.
40%
0% 50% 100%

0 35 70

Figura 8-6
Tabla 8-4
Porcentaje Longitud Además, la mitad de la barra de porcentajes (50% denotado por el segmento punteado) re-
en la barra presenta la mitad de 70, ó 35. Así, sabemos que 40% de la barra (ó 40% de 70) es menor
0% 0 que 35. De hecho, si la parte superior de la barra se piensa como marcada por intervalos del
10% 1%, hay 100 intervalos marcando esos números completos de porcentajes. Si al mismo
20%
tiempo se considera la barra marcada por intervalos de 1, habría sólo 70 intervalos marca-
30%
dos en la parte inferior. ¿Dónde esperas que se alineen los dos conjuntos de intervalos?
40% ?
Supón que sabemos que, como en la tabla 8-4, 0% corresponde a 0; 50% corresponde a
50% 35
60% 35; y 100% corresponde a 70. ¿Qué porcentajes de 70 corresponden a 10%, 20%, 30%, y
70% así sucesivamente? Ahora, si hay 100 intervalos marcando porcentajes comparados con sólo
100 10
80% 70 intervalos marcando la longitud correspondiente, debe haber una razón de ,ó .
70 7
Así, 40% de 70 debe corresponder a 7; 20% a 2 # 7, ó 14; y así sucesivamente.Por lo tanto,
90%
100% 70
40% de 70 corresponde a 4 # 7, ó 28.
Los porcentajes pueden ser mayores que el 100%. Por ejemplo, si tu frecuencia cardiaca
en reposo se considera como la unidad de base, entonces ésta será el 100% de tu frecuencia
cardiaca en reposo. Al incrementar tu frecuencia cardiaca se tendrá una frecuencia mayor
que el 100%. Trabaja con la siguiente página de muestra para investigar porcentajes mayo-
res que el 100%.
© Lopez Mateos Editores. ISBN 978-607-95583-2-1, obra completa, versión electrónica, ISBN 978-607-95583-3-8, volumen 1, versión electrónica. Ejemplar asignado a: Helecto Villarroel gutierrez -
helecto@gmail.com. Fecha: 27 de octubre de 2014. Prohibida su modificación, copia o distribución.
Sección 8-2 Porcentajes 495

Página de un libro de texto


Porcentajes mayores que 100% Supón que la frecuencia cardiaca en reposo de
Juana es de 60 latidos por minuto y su frecuencia cardiaca en activo es de 96 latidos por
minuto. Una manera de comparar estas dos frecuencias es preguntando: “¿Qué porcentaje
de la frecuencia cardiaca en reposo de Juana es su frecuencia cardiaca en activo?”
Una manera de visualizar la comparación entre la frecuencia cardiaca
en activo de Juana y su frecuencia en reposo es con un modelo de
barra de porcentajes como el del ejemplo a continuación.
Ejemplo

Usa la frecuencia cardiaca en La frecuencia cardiaca en


reposo como el todo, o 100%. activo es mayor que la
frecuencia cardiaca en reposo,
de modo que el porcentaje es
mayor que 100%.
En reposo En activo
Frecuencia
cardiaca (latidos
por minuto)
Porcentaje

¿Cuántos latidos es el 50% de 60?


¿Cuántos latidos es el 100% de 60?

¿Cuántos latidos son el 150% de 60?


¿Cuántos latidos son el 200% de 60?

Usa el modelo de la barra de porcentajes y tus respuestas a la


pregunta 19, para estimar qué porcentaje de la frecuencia
cardiaca en reposo de Juana es su frecuencia cardiaca en activo.
Pregunta 21 Punto de
verificación La frecuencia cardiaca en reposo de Maura es de 80 latidos por
...verifica que
minuto y su frecuencia cardiaca en activo es de 140 latidos por minuto. Estima qué
puedes estimar porcentaje la frecuencia cardiaca en activo de Maura es de su frecuencia cardiaca en reposo.
porcentajes mayores
que el 100%.

Módulo 5 Recreación

Fuente: Math Thematics, New Edition, Book 2, McDougal Littell, 2008 (p. 378).

Ejemplo 8-7 Una casa que se vende por $92,000 requiere un 20% de enganche. ¿Cuál es el monto del








enganche?

Solución El enganche es el 20% de $92,000, ó 0.20 # $92,000 = $18,400. Por lo tanto, el


monto del enganche es de $18,400.

Ejemplo 8-8 Si Alberto tiene 45 respuestas correctas de un examen de 80 preguntas, ¿qué porcentaje de








sus respuestas son correctas?


© Lopez Mateos Editores. ISBN 978-607-95583-2-1, obra completa, versión electrónica, ISBN 978-607-95583-3-8, volumen 1, versión electrónica. Ejemplar asignado a: Helecto Villarroel gutierrez -
helecto@gmail.com. Fecha: 27 de octubre de 2014. Prohibida su modificación, copia o distribución.
496 Razonamiento proporcional, porcentajes y aplicaciones

45
Solución    Alberto tiene de las respuestas correctas. Para hallar el porcentaje de las respues-
80
45
tas correctas, necesitamos convertir a un porcentaje. Podemos hacerlo multiplicando la
80
fracción por 100 y añadiendo el símbolo % como sigue:
45 45
= 100 # %
80 80
= 56.25%
Así, el 56.25% de las respuestas fueron correctas.
Una solución alternativa usa proporción. Sea n el porcentaje de respuestas correctas y
procedemos como sigue:
45 n
=
80 100
45 #
100 = n
80
4500
n = = 56.25
80
Así, el 56.25% de las respuestas fueron correctas.

Ejemplo 8-9 Cuarenta y dos por ciento de los padres de los niños que asisten a la escuela pública de








cierta población trabajan en la universidad local. Si el número de padres empleados por la


universidad es de 168, ¿cuántos padres están en la escuela pública?

Solución  Sea n el número de padres en la escuela pública. Entonces, 42% de n es 168. Podemos


traducir esta información a una ecuación y despejar n.
42% de n = 168
42
n = 168
100
0.42n = 168
168
n = = 400
0.42
Hay 400 padres en la escuela pública.

42
El ejemplo 8-9 puede resolverse usando una proporción. Cuarenta y dos por ciento, ó , de
100
los padres están empleados en la universidad. Si n es el número total de padres, entonces
168
también representa la fracción de padres empleada ahí. Así,
n
42 168
=
100 n
42n = 100 # 168
16,800
n = = 400
42
© Lopez Mateos Editores. ISBN 978-607-95583-2-1, obra completa, versión electrónica, ISBN 978-607-95583-3-8, volumen 1, versión electrónica. Ejemplar asignado a: Helecto Villarroel gutierrez -
helecto@gmail.com. Fecha: 27 de octubre de 2014. Prohibida su modificación, copia o distribución.
Sección 8-2 Porcentajes 497

También podemos resolver el problema como sigue:


42% de n es 168
168
1% de n es
42

b
168
100% de n es 100a
42

b, ó 400
168
Por lo tanto, n = 100a
42

Ejemplo 8-10 Queta compró una bicicleta y un año después la vendió por 20% menos de lo que ella pagó.








Si la vendió en $144, ¿cuánto pagó por ella?

Solución    Buscamos el precio original, P, que Queta pagó por la bicicleta. Sabemos que la
vendió en $144 y que esto incluye una pérdida del 20%. Así, podemos plantear la ecuación:
$144 = P - pérdida de Queta
Como la pérdida de Queta es el 20% de P, procedemos como sigue:
$144 = P - 20% # P
$144 = P - 0.20P
$144 = 11 - 0.202P
$144 = 0.80P
$144
= P
0.80
$180 = P
Así, ella pagó $180 por la bicicleta.

Ejemplo 8-11 La Tienda del Vaquero anuncia un traje con un descuento del 10%, para un ahorro de $15.








Más adelante, el gerente anunció el traje con un 30% de descuento del precio original. ¿Cuál
es la cantidad del descuento actual?

Solución    Un 10% de descuento equivale a un ahorro de $15. Podríamos hallar la cantidad


del descuento actual si supiéramos el precio original. Así, hallar el precio original se con-
vierte en nuestro objetivo parcial. Como 10% de P es $15, tenemos lo siguiente:
10% # P = $15
0.10P = $15
P = $150
Para hallar el descuento actual, calculamos el 30% de $150. Como 0.30 # $150 = $45, la
cantidad del 30% de descuento es $45.
En la parte Revisar de la solución de problemas, verificamos la respuesta y buscamos otra
manera de resolver el problema. Un enfoque diferente lleva a una solución más eficiente y
confirma la respuesta. Si el 10% del precio es $15, entonces el 30% del precio es 3 veces
$15, ó $45.

© Lopez Mateos Editores. ISBN 978-607-95583-2-1, obra completa, versión electrónica, ISBN 978-607-95583-3-8, volumen 1, versión electrónica. Ejemplar asignado a: Helecto Villarroel gutierrez -
helecto@gmail.com. Fecha: 27 de octubre de 2014. Prohibida su modificación, copia o distribución.
498 Razonamiento proporcional, porcentajes y aplicaciones

AHORA INTENTA ÉSTE 8-4 En la tira cómica a continuación, calcula el porcentaje y número de
rebanadas para las partes con aceitunas, normal, y con cebolla y pimiento verde.

Pizza En 8 porciones quiero 3712% Odio cuando


3 normal y
La Rápida, con aceitunas,24 llaman maestros
¿diga? el resto con cebolla y de matemáticas.
Orden pimiento verde, y que la
circunferencia sea extra
crujiente.

En realidad,
que sea de 2pi

Matemática mental con porcentajes


La matemática mental puede ser útil cuando se trabaja con porcentajes. A continuación se
presentan dos técnicas:
1. Usar fracciones equivalentes
Conocer fracciones equivalentes para algunos porcentajes puede facilitar cálculos. La
tabla 8-5 presenta varias fracciones equivalentes.

Tabla 8-5

1 2
Porcentajes 25% 50% 75% 33 % 66 % 10% 1%
3 3
1 1 3 1 2 1 1
Fracción equivalente
4 2 4 3 3 10 100

Estas equivalencias se pueden usar en cálculos como el siguiente:

50% de $80 = a b80 = $40


1
2

66 % de 90 = a b90 = 60
2 2
3 3
2. Usar un porcentaje conocido
A veces podemos no conocer un porcentaje de algo, pero conocemos un porcentaje
cercano. Por ejemplo, para hallar el 55% de 62 podemos hacer lo siguiente:

50% de 62 = a b1622 = 31
1
2

5% de 62 = a b110%21622 = a b16.22 = 3.1


1 1
2 2
Sumando, vemos que el 55% de 62 es 31 + 3.1 = 34.1.

Estimación con porcentajes


Se pueden usar estimaciones con porcentajes para determinar si las respuestas son razona-
bles. A continuación se presentan dos ejemplos:
© Lopez Mateos Editores. ISBN 978-607-95583-2-1, obra completa, versión electrónica, ISBN 978-607-95583-3-8, volumen 1, versión electrónica. Ejemplar asignado a: Helecto Villarroel gutierrez -
helecto@gmail.com. Fecha: 27 de octubre de 2014. Prohibida su modificación, copia o distribución.
Sección 8-2 Porcentajes 499

1. Para estimar el 27% de 598, nota que el 27% de 598 es un poco mayor que el 25% de 598,
1
pero el 25% de 598 es aproximadamente igual al 25% de 600, ó de 600, ó 150. Aquí hemos
4
ajustado 27% hacia abajo y 598 hacia arriba, de modo que 150 debe ser un estimado razona-
ble. Un mejor estimado se puede obtener calculando el 30% de 600 y después restando
3% de 600 para obtener el 27% de 600, lo cual da 180 - 18, ó 162.
2. Para estimar el 148% de 500, nota que el 148% de 500 deberá ser ligeramente menor que
el 150% de 500. 150% de 500 es 1.515002 = 750. Así, el 148% de 500 deberá ser un poco
menor que 750.
Ejemplo 8-12 Laura quiere comprar una blusa cuyo precio original era de $26.50 pero que ahora en oferta
tiene el 40% de descuento. Ella tiene $17 en su cartera y se pregunta si lleva suficiente dinero.








¿Cómo puede hacerlo mentalmente? (Ignora el impuesto.)

Solución Es más fácil hallar mentalmente el 40% de $25 (que de $26.50) . Una manera es
hallar el 10% de $25, que es $2.50. Ahora, 40% es 4 veces eso, esto es, 4 # $2.50, ó $10. Así,
Laura estima que la blusa costará $26.50 - $10, ó $16.50. Como el descuento real es
mayor que $10 (40% de 26.50 es mayor que el 40% de 25), Laura tendrá que pagar menos
de $16.50 por la blusa y, por lo tanto, lleva suficiente dinero.

A veces puede no ser claro qué operaciones efectuar con el porcentaje. En el ejemplo si-
guiente se investiga esto.

Ejemplo 8-13 ¿Cuáles de las proposiciones siguientes son verdaderas y cuáles falsas? Explica tus respuestas.








a. Leonardo obtuvo un aumento del 10% al final de su primer año de trabajo y un 10% de
aumento después de otro año. Su aumento total fue del 20% de su salario original.
b. Juan y Diana pagaron el 45% de su cuenta de $620 en un primer almacén, y pagaron el 48%
de la cuenta de $380 en un segundo almacén. Ellos pagaron el 45% + 48% = 93% de la
cuenta total de $1000.
c. Vicente gastó el 25% de su salario en comida y el 40% en habitación. Vicente gastó
25% + 40% = 65% de su salario en comida y habitación.
d. En una ciudad de la frontera, el 65% de la población adulta trabaja en la ciudad,
25% trabaja al otro lado de la frontera y el 15% está desempleado.
e. En Ciudad SinBasura, la multa por actividades contaminantes es cierto porcentaje del
ingreso mensual. La multa por fumar en lugares públicos es del 40%, por llevar un carro
contaminante es del 50% y por tirar basura es del 30%. Don Cochinón cometió los tres
crímenes contaminantes el mismo día y pagó una multa del 120% de su salario mensual.
Solución a. En las aplicaciones el porcentaje tiene sentido sólo cuando representa parte de
una cantidad. Por ejemplo, el 10% de una cantidad más el 10% de la misma
cantidad es el 20% de esa cantidad. En el caso de Leonardo, el primer aumento
del 10% se calculó con base en su salario original y el segundo aumento del
10% se calculó sobre su nuevo salario. En consecuencia, los porcentajes no se
pueden sumar y la proposición es falsa. Recibió un aumento del 21%.
b. La última afirmación no tiene sentido; 45% de una cuenta más 48% de otra
cuenta no es 93% de la cuenta total pues las cuentas son diferentes.
c. Como los porcentajes son de la misma cantidad, la proposición es verdadera.
d. Como los porcentajes son de la misma cantidad, esto es, del número de adul-
tos, podemos sumarlos: 65% + 25% + 15% = 105%. Pero el 105% de los
habitantes se refiere a una cantidad mayor (en 5%) que la de los habitantes de
la ciudad, lo cual es imposible. Por lo tanto, la proposición es falsa.

© Lopez Mateos Editores. ISBN 978-607-95583-2-1, obra completa, versión electrónica, ISBN 978-607-95583-3-8, volumen 1, versión electrónica. Ejemplar asignado a: Helecto Villarroel gutierrez -
helecto@gmail.com. Fecha: 27 de octubre de 2014. Prohibida su modificación, copia o distribución.
500 Razonamiento proporcional, porcentajes y aplicaciones

e. De nuevo, los porcentajes son de la misma cantidad, esto es, el ingreso indivi-
dual mensual. Por lo tanto, podemos sumarlos. El 120% del ingreso mensual
es una multa severa, pero posible.

Evaluación 8-2A

1. Expresa cada caso como porcentaje: 11. Si un vestido que normalmente cuesta $35 está en oferta
5 por $28, ¿cuál es el porcentaje de descuento?
a. 7.89 b. 193.1 c.
6 12. En el año 2008 Miguel compró su casa por $159,000.
1 5 4 Recientemente, la casa fue valuada en $195,000. ¿Cuál es
d. e. f. el porcentaje de incremento en el valor aproximado a la uni-
8 8 5
2. Convierte cada porcentaje en decimales: dad porcentual más cercana?
1 13. Juan pesó 9 lb al nacer, y a los seis meses pesaba 18 lb.
a. 16% b. % ¿Cuál fue el porcentaje de incremento en el peso de Juan?
5
14. Dora compró un vestido marcado con el 20% de des-
2 1
c. 13 % d. % cuento. Si el precio normal era de $28.00, ¿cuál fue el
3 3 precio de venta?
3. Llena los espacios en blanco a fin de hallar otras expre- 15. Un boleto de avión cuesta $320 sin el impuesto. Si el im-
siones para el 4%: puesto es de 5%, ¿cuál es el costo total del boleto de avión?
a. ____ por cada 100 b. ____ por cada 50 16. Beto obtuvo 52 respuestas correctas en un examen de 80
c. 1 por cada ____ d. 8 por cada ____ preguntas. ¿Qué porcentaje de preguntas contestó equi-
e. 0.5 por cada ____ vocadamente?
4. Diferentes calculadoras calculan porcentajes de diversas 17. Un corredor de bienes raíces recibe el 4% de una venta
maneras. Para investigarlo, considera 5 # 6%. de $80,000. ¿Cuánto recibe?
a. Si se presiona la siguiente sucesión de teclas, ¿se pre- 2
18. Una encuesta reporta que 66 % de 1800 empleados favo-
senta en pantalla la respuesta correcta de 0.3? 3
recen un nuevo programa de seguros. ¿Cuántos emplea-
5 * 6 % = dos están a favor del nuevo programa?
b. Teclea 6 % * 5 = . ¿La respuesta es 0.3? 19. ¿Qué expresión representa el porcentaje más grande:
5. Responde a lo siguiente: 325 600
ó ? ¿Cómo puedes decirlo?
a. ¿Cuál es el 6% de 34? 500 1000
b. ¿Qué porcentaje es 17 de 34? 20. Un anuncio dice que si compras 10 artículos, obtienes el
c. ¿18 es el 30% de qué número? 20% de descuento en el precio total de la compra. Tú
d. ¿Cuál es el 7% de 49? necesitas 8 artículos que cuestan $9.50 cada uno.
6. a. Escribe una fracción que represente el 5% de x. a. ¿Cuánto costarían 8 artículos? ¿Y 10 artículos?
b. Si el 10% de una cantidad es a, ¿cuál es la cantidad en b. ¿Qué es más económico, comprar 8 artículos ó 10?
términos de a? 21. Se anuncian refrescos a 45¢ la lata o a $2.40 el paquete
7. Marcos tenía 84 cajas de caramelo para vender. Si vendió de seis latas. Si se van a comprar 6 latas, ¿qué porcentaje
el 75% de las cajas, ¿cuántas vendió? se ahorra comprando el paquete de seis latas?
8. Gael ganó $16,000 el año pasado y recibió un aumento 22. Roberto pagó $330 por una nueva bicicleta de montaña
del 6%. ¿Cuánto gana ella ahora? para venderla en su tienda. Quiere que el precio sea tal
9. Gael recibió un aumento del 7% el año pasado. Si su sala- que él pueda ofrecer un descuento del 10% y aún así ga-
rio es ahora de $27,285, ¿cuál era su salario el año pasado? nar el 20% del precio que pagó. ¿Con qué precio debe
10. José vendió 180 periódicos de 200. Beto vendió 85% de marcarse la bicicleta?
sus 260 periódicos. Rafa vendió 212 periódicos, 80% de los 23. Resuelve lo siguiente usando matemática mental:
que tenía. a. 15% de $22 b. 20% de $120
a. ¿Quién vendió más periódicos? ¿Cuántos? c. 5% de $38 d. 25% de $98
b. ¿Quién vendió el mayor porcentaje de sus periódi- 24. Para que las personas se beneficien sin riesgos del entre-
cos? ¿Qué porcentaje? . namiento cardiovascular, deben controlar sus ritmos car-
c. ¿Quién comenzó con el mayor número de periódi- diacos mientras se ejercitan. El máximo ritmo cardiaco
cos? ¿Cuántos? puede aproximarse restando tu edad de 220. Puedes lo-
grar un efecto seguro de entrenamiento si mantienes tu
© Lopez Mateos Editores. ISBN 978-607-95583-2-1, obra completa, versión electrónica, ISBN 978-607-95583-3-8, volumen 1, versión electrónica. Ejemplar asignado a: Helecto Villarroel gutierrez -
helecto@gmail.com. Fecha: 27 de octubre de 2014. Prohibida su modificación, copia o distribución.
Sección 8-2 Porcentajes 501

ritmo cardiaco entre el 60% y el 80% de ese número por b. Suponiendo que se requieran exactamente 815 años
lo menos durante 20 min, tres veces a la semana. para gastar los $2.57 billones, ¿qué porcentaje de di-
a. Determina el rango para tu edad. nero se gastó cada año?
b. En la cima de una colina, Tere detuvo su bicicleta y se 29. Si quieres gastar 25% de tu salario mensual en entrete-
tomó el pulso. Ella contó 41 latidos en 15 s. nimiento y 56% en renta, ¿podrían ser esas cantidades
(i) Expresa en forma decimal la cantidad de tiempo $500 y $950? ¿Por qué sí o por qué no?
en segundos entre latidos sucesivos. 30. Una organización tiene 100,000 miembros. Durante la
(ii) Expresa la cantidad en minutos. reunión anual se puede realizar un cambio en sus regla-
25. Un equipo de trabajo consta de 1 aprendiz, 1 cabo y un ma- mentos, el cual debe ser aprobado por la mayoría de los
estro carpintero. El equipo recibe un cheque por $4200 por asistentes a la reunión. La presidenta de la reunión no
un trabajo recién terminado. Un cabo gana el 200% de lo puede votar a menos que haya un empate, pero cuenta
que gana el aprendiz, y el maestro gana el 150% de lo que como asistente a la reunión.
gana el cabo. ¿Cuánto ganó cada persona del equipo? a. Con estas reglas, ¿cuál es el mínimo número reque-
26. a. En un grupo de nuevo ingreso de 500 estudiantes, rido en la reunión para cambiar el reglamento?
sólo 20 solicitaron la especialidad de matemáticas. b. Con base en tu respuesta a la parte (a), ¿qué porcen-
¿Qué porcentaje del grupo es? taje de los miembros puede cambiar el reglamento de
b. Cuando se repitió la encuesta al año siguiente, el 5% la organización?
de los que no estaban en matemáticas decidieron 31. La propina en un restaurante se ha fijado en un 15% de
cambiar a matemáticas. la cuenta.
(i) ¿Cuántos alumnos hay ahora en matemáticas? a. Si la cuenta es de $30, ¿cuál sería la propina?
(ii) ¿Qué porcentaje de la clase de primer ingreso re- b. Si el cliente que recibió la cuenta da una propina de la
presentan? mitad de la cuenta, ¿cuál es el porcentaje de la propina?
27. Doña Carmen ha recibido un aumento del 10% en cada c. Si el cliente que recibió la cuenta da una propina igual
uno de los últimos dos años. Si su salario anual este año a la cuenta, ¿cuál es el porcentaje de la propina?
es de $100,000, ¿cuál era su salario hace 2 años, redon- 32. Supón que la barra de porcentajes muestra el número de
deado al centavo más cercano? estudiantes de una escuela que no estan a favor de un
28. La revista USA Today (2005) reportó que el Congreso de código de vestido. ¿Cuántos estudiantes hay en la escuela?
Estados Unidos había aprobado un presupuesto de $2.57
billones para el año fiscal 2006. Añadió que se podría 0 374
Número de estudiantes
comprar un artículo de $100 cada segundo durante 815
años para gastar esa cantidad de dinero. Porcentaje
0% 50% 68% 100%
a. Decide si crees o no que estos reportajes concuerdan
entre sí.

Evaluación 8-2B

1. Expresa cada caso como porcentaje: d. 8 por cada ____


3 e. 0.5 por cada ____
a. 0.032 b. 0.2 c.
20 4. Responde a lo siguiente:
13 1 1 a. ¿63 es el 30% de qué número?
d. e. f. b. ¿Cuál es el 7% de 150?
8 6 40
2. Convierte cada porcentaje en decimales: c. ¿61.5 es qué porcentaje de 20.5?
1 2 d. ¿16 es el 40% de qué número?
a. 4 % b. % 5. Un carro usado costaba $1700. Un año después valía
2 7
$1400. ¿Cuál es el porcentaje de depreciación?
1
c. 125% d. % 6. En cierto parque se contaron un día 728 águilas. Cinco
4
años después se contaron 594. ¿Cuál fue el porcentaje de
3. Llena los espacios en blanco a fin de hallar otras expre-
merma en el número de águilas contadas?
siones para el 5%:
7. ¿Cuál es el precio de venta de una pelota si el precio nor-
a. ____ por cada 100
mal es de $6.80 y tiene un 25% de descuento?
b. ____ por cada 50
c. 1 por cada ____

© Lopez Mateos Editores. ISBN 978-607-95583-2-1, obra completa, versión electrónica, ISBN 978-607-95583-3-8, volumen 1, versión electrónica. Ejemplar asignado a: Helecto Villarroel gutierrez -
helecto@gmail.com. Fecha: 27 de octubre de 2014. Prohibida su modificación, copia o distribución.
502 Razonamiento proporcional, porcentajes y aplicaciones

1
8. Si un de taza de cereal para el desayuno tiene el 0.5% 20. Ordena estos números de menor a mayor.
4
de los requerimientos mínimos diarios de vitamina C, 65%, 3>5, 0.70, 50%, 2>3, 0.55
¿cuántas tazas deberías comer para obtener el requeri-
miento mínimo diario de vitamina C? 21. a. Se recomienda que no más del 30% de tu ingesta de ca-
9. a. ¿De qué manera un estimado del 10% de un número lorías provenga de grasa. Si consumes alrededor de 2400
te puede ayudar a estimar el 35% del número? calorías al día, ¿cuál es el máximo de calorías de grasa que
b. Calcula mentalmente 35% de $8.00. debes consumir?
10. Si 30 es el 150% de un número, ¿el número es mayor o b. Si una galleta contiene 140 calorías y 70 calorías de la ga-
menor que 30? ¿Por qué? lleta son de grasa, ¿puedes comer 3 galletas sin exceder la
11. ¿Cuál es el 40% del 50% de un número? cantidad recomendada de calorías grasas para el día?
12. Si sumas el 20% de un número al número mismo, ¿qué 22. Si compras una bicicleta nueva por $380 y el impuesto es
porcentaje del resultado deberías restar para obtener de de 9%, ¿cuál es la cuenta total?
nuevo el número original? 23. El número de especies vivas conocidas es alrededor de 1.7
13. El precio de un traje de $200 se redujo en 25%. ¿En qué millones. Alrededor de 4500 especies son mamíferos.
porcentaje debe incrementarse el precio del traje para vol- ¿Qué porcentaje de las especies vivientes conocidas son
ver al precio de $200? mamíferos?
14. El carro que se compró Elisa hace 1 año se depreció en 24. Hay 80 monedas en el cochinito de ahorro, de las cuales
$1116.88, que es el 12.13% del precio que ella pagó. el 20% son de 25 centavos. ¿Cuál es la mínima cantidad
¿Cuánto pagó por el carro, al centavo más cercano? de dinero que puede haber en el cochinito?
15. Resuelve lo siguiente usando matemática mental: 25. Un área para esquiar reporta que el 80% de sus pistas es-
a. 15% de $42 b. 20% de $280 tán abiertas. ¿Si hay 60 pistas abiertas, ¿cuántas pistas
c. 5% de $28 d. 25% de $84 hay en esa área de esquiar?
16. Si construimos un modelo de 10 * 10 con cubos, como 26. Una vendedora gana un salario semanal de $900 más una
se muestra en la figura siguiente, y pintamos todo el mo- comisión del 4% del total de las ventas. ¿Cuánto cobró
delo, ¿qué porcentaje de cubos tendrá lo siguiente? la vendedora en una semana si hubo un total de ventas de
a. Cuatro caras pintadas $1800?
b. Tres caras pintadas 27. Jaime compró dos camisas que originalmente estaban
c. Dos caras pintadas marcadas con $40 cada una. Una camisa tenía descuento
del 20% y la otra tenía descuento del 25%. El impuesto
de venta era del 4.5%. ¿Cuánto gastó en total?
28. Si los lados de un cuadrado se incrementan en un 80%,
¿se incrementa el área del cuadrado en un 80%
1A = s 22? De no ser así, ¿qué le pasa al área? Usa por-
centajes en tu solución.
29. De acuerdo con una encuesta de TV Guide, el 46% de las
personas en Estados Unidos dijo que no dejarían de ver te-
levisión por nada menos que un millón de dólares. Usa la
barra de porcentaje para la población de Estados Unidos
para estimar el número de personas que no dejarían de ver
17. Si el 70% de las niñas del grupo y el 40% de los niños televisión por nada menos que un millón de dólares.
querían un baile de fin de cursos, ¿es posible que sólo el
50% de los estudiantes del grupo quisieran un baile de Personas (millones)
0 151 302
fin de cursos? Explica tu respuesta.
18. Si el 70% de las niñas del grupo y el 60% de los niños Porcentaje
querían un baile de fin de cursos, ¿es posible que sólo el 0% 46% 50% 100%
50% de los estudiantes del grupo quisieran un baile de
fin de cursos? Explica tu respuesta.
19. Traza un segmento de recta de 3 pulg de largo y llámale X.
Este segmento representa el 50% de otro segmento Y.
Traza lo siguiente.
a. Un segmento que represente el 100% del segmento Y.
b. Un segmento que represente el 25% del segmento Y.
c. Un segmento que represente el 150% del segmento Y.

© Lopez Mateos Editores. ISBN 978-607-95583-2-1, obra completa, versión electrónica, ISBN 978-607-95583-3-8, volumen 1, versión electrónica. Ejemplar asignado a: Helecto Villarroel gutierrez -
helecto@gmail.com. Fecha: 27 de octubre de 2014. Prohibida su modificación, copia o distribución.
Sección 8-2 Porcentajes 503

Conexiones matemáticas  8-2

Comunicación Aprendizaje colectivo


1. Usa matemática mental para hallar el 11% de 850. Explica 13. Halla el porcentaje de estudiantes de tu grupo que practi-
tu método. quen las actividades siguientes:
2. ¿Es cierto que 0.4 = 0.4%? Explica.
3. ¿Qué significa alcanzar el 125% de tu objetivo de ahorro? a. Estudiar y hacer
4. ¿Es el 4% de 98 lo mismo que el 98% de 4? Explica. la tarea b. Ver la televisión
5. a. Si el 25% de un número es 55, ¿el número es mayor o
Número de Número de
menor que 55? Explica.
horas a la horas a la
b. Si el 150% de un número es 55, ¿el número es mayor
semana (h) Porcentaje semana (h) Porcentaje
o menor que 55? Explica.
6. ¿Puede el 35% de un número ser mayor que el 55% de h 6 1 h 6 1
otro número? Explica.
1 … h 6 3 1 … h 6 5
7. ¿Por qué una figura tiene mucho más área sombreada si
ambas muestran el 50%? 3 … h 6 5 5 … h 6 10
5 … h 6 10 h Ú 10
h Ú 10 Total

50% 50%
Total

c. ¿Tus totales suman 100% en cada tabla? ¿Por qué sí o por


8. ¿Por qué es posible tener un incremento de 150% en un qué no?
precio pero no un 150% de descuento en un precio? Preguntas del salón de clase
9. Dos cantidades iguales de dinero se invirtieron en dos ti-
14. Una estudiante pregunta que si el 90% significa 90 de
pos de acciones. El valor de la primera se incrementó en
cada 100, ¿cómo es posible una calificación de 90% en
un 15% el primer año y después disminuyó en 15% el se-
un examen que tiene sólo 20 preguntas? ¿Cómo le res-
gundo año. La segunda disminuyó en un 15% el primer
pondes?
año y se incrementó 15% el segundo año. ¿Fue una inver-
1
sión mejor que otra? Explica tu razonamiento. 15. Un estudiante dice que 3 % = 0.03 + 0.25 = 0.28. ¿Es
4
Solución abierta esto correcto? Por qué?
10. Describe una situación que plantee un problema cuya so- 16. Un estudiante reporta que es imposible subir un producto
lución incluya abajo descrito. Resuelve el problema. Si en 150% porque el 100% de algo es todo lo que hay.
una de las tareas es imposible, explica por qué. ¿Qué le respondes?
a. Suma de porcentajes 17. Una estudiante argumenta que un incremento salarial del
b. Resta de porcentajes p% seguido de un decremento del q% es equivalente a un
c. Multiplicación de porcentajes decremento del q% seguido de un incremento del p% de-
d. División de porcentajes bido a la propiedad conmutativa de la multiplicación.
e. Un porcentaje cuya representación decimal se eleve a ¿Qué le respondes?
la segunda potencia 18. Un estudiante argumenta que 0.01% = 0.01 pues en
f. Un porcentaje mayor que 100 0.01% el porcentaje ya se escribió como decimal. ¿Cómo
11. Busca en periódicos y revistas información dada en por- le respondes?
centajes.
Problemas de repaso
a. Con base en tus hallazgos, escribe un problema que in-
cluya ciencias sociales y matemáticas. 19. La podadora de Jaime requiere de 5 onzas fluidas de aceite
b. Escribe una solución clara al problema en (a). mezcladas con cada 2 gal de gasolina. Un contenedor
12. Escribe un problema de porcentaje cuya respuesta sea la tiene 12 gal de gasolina y Jaime añadió 34 onzas fluidas de
solución de la ecuación siguiente: aceite. ¿Es correcto? ¿Por qué?
37 115 20. Un cierto tono de verde requiere de 4 partes de azul por
a. = cada 5 partes de amarillo. Si tienes 25 l de amarillo,
100 x
p a ¿cuánto azul debes añadir?
b. =
100 x
© Lopez Mateos Editores. ISBN 978-607-95583-2-1, obra completa, versión electrónica, ISBN 978-607-95583-3-8, volumen 1, versión electrónica. Ejemplar asignado a: Helecto Villarroel gutierrez -
helecto@gmail.com. Fecha: 27 de octubre de 2014. Prohibida su modificación, copia o distribución.
504 Razonamiento proporcional, porcentajes y aplicaciones

21. Di si 4x>3 y 12x>9 siempre, a veces o nunca forman una Pregunta del National Assessment of Educational Progress
proporción. Explica. (NAEP) (Evaluación Nacional del Progreso Educativo)
22. Resuelve cada proporción.
x 21 27 36
a. = b. =
18 63 21 x
Tabla de impuestos
15 x 19 152
c. = d. =
33 22 x 4 Cantidad Cantidad
CEREAL vendida de impuesto
Preguntas del Third International Mathematics and
Science Study (TIMSS) (Tercer Estudio Internacional $6.00 $0.36
sobre las Matemáticas y la Ciencia)
6.20 0.37
Los expertos dicen que el 25% de los accidentes graves
6.40 0.38
de bicicleta incluyen heridas en la cabeza y que, de todas $4.95
las heridas en la cabeza, el 80% son mortales. ¿Qué por- 6.60 0.40
centaje de los accidentes graves de bicicleta incluyen 6.80 0.41
heridas en la cabeza mortales? 7.00 0.42
a. 16% b. 20%
c. 55% d. 105% 7.20 0.43
7.40 0.44
Leche
TIMSS, Grado 8, 1995
7.60 0.46
El año pasado había 1172 estudiantes en una secundaria.
Este año hay 15 por ciento más estudiantes que el año 7.80 0.47
pasado. ¿Aproximadamente cuántos estudiantes hay en 8.00 0.48
la secundaria este año? $2.45
a. 1800 b. 1600 Carlos compró el cereal y la leche mostrados. Usa la ta-
c. 1500 d. 1400 bla para hallar la cantidad total que gastó Carlos, inclu-
e. 1200 yendo impuestos.
TIMSS, Grado 8, 1995 Cantidad total gastada: _______
Ilustra cómo hallaste tu respuesta.
NAEP, Grado 4, 2007

ROMPECABEZAS  La corteza de cierta tarta de calabaza es el 25% de la tarta. ¿En qué porcentaje de-
berá reducirse la cantidad de corteza para que constituya el 20% de la tarta?

RINCÓN DE LA TECNOLOGÍA Podemos usar hojas de cálculo para resolver problemas acerca de
mezclas. Por ejemplo, considera el problema de hallar cuántos litros de agua deben añadirse a 5 L de
jugo de limón puro para cambiar la concentración de 100% a menos de 30% de jugo de limón.
Se prepararon seis mezclas de limonada comenzando con 5 L de jugo de limón puro y añadiendo agua
en incrementos de 2 L. En cada paso se calculó el porcentaje de jugo de limón en la mezcla. Los resulta-
dos del proceso se resumen en la hoja de cálculo de la figura 8-7. Las fórmulas usadas para obtener los
resultados en una columna particular se dan en el renglón 12.

© Lopez Mateos Editores. ISBN 978-607-95583-2-1, obra completa, versión electrónica, ISBN 978-607-95583-3-8, volumen 1, versión electrónica. Ejemplar asignado a: Helecto Villarroel gutierrez -
helecto@gmail.com. Fecha: 27 de octubre de 2014. Prohibida su modificación, copia o distribución.
Sección 8-3 Cálculo de interés 505

a. Explica cómo puede ayudar esta hoja de cálculo a los estudiantes para resolver el problema.
b. Explica las fórmulas en el renglón 12.

Litros de Litros de Total de litros % de jugo de


jugo de limón agua añadida en la mezcla limón en la mezcla
(L) (L) (L)
5 0 5 100.00
5 2 7 71.43
5 4 9 55.56
5 6 11 45.45
5 8 13 38.46
5 10 15 33.33
5 12 17 29.41

5 x 5+x 5 /(5 + x)*100


(donde x es un
múltiplo de 2)

Hoja 1 H
Listo

Figura 8-7

*8-3 Cálculo de interés

1
Cuando un banco anuncia una tasa de interés del 5 % en una cuenta de ahorros, el interés
2
es la cantidad de dinero que el banco te pagará por usar ese dinero. La cantidad original de-
positada o prestada es el capital. El porcentaje usado para determinar el interés es la tasa
de interés. Las tasas de interés están dadas para periodos específicos de tiempo, como años,
meses o días. El interés calculado sobre el principal original es el interés simple. Por ejem-
plo, supón que pedimos un préstamo de $5000 a una tasa de interés simple de 9% por 1
año. El interés que debemos por el préstamo por un año es el 9% de $5000, ó 0.09 # $5000.
En general, si un capital, C, se invierte a una tasa de interés anual r, entonces el interés sim-
ple después de 1 año es Cr # 1; después de t años es Crt. Así, si I representa el interés simple,
tenemos
I = Crt
La cantidad necesaria para pagar un préstamo de $5000 a un 9% de interés simple anual
por 1 año es de $5000 prestados más el interés sobre los $5000; esto es, 5000 + 5000 # 0.09,
ó $5450. En general, un importe (o saldo) A es igual al capital C más el interés I; esto es,
A = C + I = C + Crt = C11 + rt2

Ejemplo 8-14








1
Vera abrió una cuenta de ahorros que paga un interés simple a la tasa de 5 % al año. Si ella
4
deposita $2000 y no hace otros depósitos, halla el interés y el importe final para los siguien-
tes periodos de tiempo:
a. 1 año b. 90 días

© Lopez Mateos Editores. ISBN 978-607-95583-2-1, obra completa, versión electrónica, ISBN 978-607-95583-3-8, volumen 1, versión electrónica. Ejemplar asignado a: Helecto Villarroel gutierrez -
helecto@gmail.com. Fecha: 27 de octubre de 2014. Prohibida su modificación, copia o distribución.
506 Razonamiento proporcional, porcentajes y aplicaciones

Solución a. Para hallar el interés para 1 año, procedemos como sigue:


1
I = $2000 # 5 % # 1 = $2000 # 0.0525 # 1 = $105
4
Su importe al finalizar 1 año es de
$2000 + $105 = $2105
b. Cuando la tasa de interés es anual y el periodo de interés se da en días, pode-
mos representar el tiempo como una parte fraccional de un año dividiendo el
número de días entre 365. Así,
1 90
I = $2000 # 5 % #
4 365
90
= $2000 # 0.0525 # ⬟ $25.89
365
Por lo tanto,
A ⬟ $2000 + $25.89
A ⬟ $2025.89
Así, el importe de Vera después de 90 días es de aproximadamente $2025.89.

Ejemplo 8-15 Halla la tasa de interés anual si un capital de $10,000 se incrementa a $10,900 al final de 1 año.








Solución Sea x% el interés anual. Sabemos que x% de $10,000 es el incremento. Como el


incremento es $10,900 - $10,000 = $900, usamos la estrategia de plantear una ecuación en
x, como sigue:
x% de 10,000 = 900
x #
10,000 = 900
100
x = 9
Así, el interés es de 9%. También podemos resolver el problema mentalmente preguntándonos
“¿Qué porcentaje de 10,000 es 900?” Como 1% de 10,000 es 100, para obtener 900 tomamos
el 9% de10,000.

Interés compuesto
En las transacciones comerciales, a veces el interés se calcula diariamente (365 veces al año).
En el caso de los ahorros, el interés ganado se añade diariamente al principal, y cada día el
interés se gana sobre una cantidad diferente; esto es, se gana sobre el interés anterior junto
con el principal. Cuando el interés se calcula de esa manera, se llama interés compuesto.
Por lo general se compone en forma anual (una vez al año), semestral (dos veces al año),
trimestral (4 veces al año) o mensual (12 veces al año). Sin embargo, aun cuando el interés
se componga, se da como una tasa anual. Por ejemplo, si la tasa anual es de 6% compuesto
6
mensualmente, el interés mensual es del %, ó 0.5%. Si se compone diariamente, el in-
12
6
terés al día es del %. En general, la tasa de interés por periodo es la tasa de interés anual dividida
365
entre el número de periodos al año.

© Lopez Mateos Editores. ISBN 978-607-95583-2-1, obra completa, versión electrónica, ISBN 978-607-95583-3-8, volumen 1, versión electrónica. Ejemplar asignado a: Helecto Villarroel gutierrez -
helecto@gmail.com. Fecha: 27 de octubre de 2014. Prohibida su modificación, copia o distribución.
Sección 8-3 Cálculo de interés 507

Podemos usar una hoja de cálculo para comparar diversas tasas de interés compuesto.
Trabaja con la página de muestra (p. 508) y responde las preguntas de las secciones
INTÉNTALO y POR TI MISMO.
Si inviertes $100 al 8% compuesto trimestralmente, ¿cuánto tendrás en tu cuenta des-
1
pués de un año? La tasa de interés trimestral es del # 8%, ó 2%. Parece que tendríamos
4
que calcular 4 veces el interés. Pero podemos razonar como sigue. Si al principio de
cualquiera de los cuatro periodos tenemos x cantidad en la cuenta, al final de ese periodo
habrá
x + 2% de x = x + 0.02x
= x11 + 0.022
= x11.022 dólares
Por lo tanto, para hallar el importe final de cualquier periodo sólo necesitamos multiplicar la
cantidad inicial del periodo por 1.02. En la tabla 8-6, vemos que la cantidad al final del cuarto
periodo es $100 # 1.024. En una calculadora científica podemos hallar el importe usando
1 0 0 * 1 . 0 2 y 4 = . La calculadora presenta en pantalla 108.24322. Así,
x

el importe al final de 1 año es aproximadamente $108.24.


Tabla 8-6

Periodo Importe inicial Importe final


1 100 100 # 1.02
2 100 # 1.02 1100 1.0221.02, ó 100 # 1.022
#
3 100 # 1.022 1100 # 1.02221.02, ó 100 # 1.023
4 100 # 1.023 1100 # 1.02321.02, ó 100 # 1.024

Hallar el importe al final del periodo n-ésimo equivale a obtener el n-ésimo término de
una sucesión geométrica cuyo primer término es 100 # 1.02 (importe al final del primer pe-
riodo) y cuya razón es 1.02. Así, el importe al final del n-ésimo periodo está dado por
1100 # 1.02211.022n - 1 = 100 # 1.02n. Podemos generalizar este análisis. Si el capital es C y la
tasa de interés por periodo es r, entonces el importe (o saldo) S después de n periodos es
C11 + r211 + r2n - 1, o C11 + r2n. Por lo tanto, tenemos una fórmula para calcular el importe
al final del periodo n-ésimo, a saber
S = C11 + r2n

Ejemplo 8-16 Supón que depositas $1000 en una cuenta de ahorros que paga un 6% de interés compuesto








trimestralmente.
a. ¿Cuál es el saldo al final de 1 año?
b. ¿Cuál es el rendimiento efectivo anual de esta inversión, esto es, cuál es la tasa que se ha-
bría pagado si la cantidad se hubiera invertido usando interés simple?
1 0.06
Solución a. Una tasa de interés anual del 6% gana de 6%, o una tasa de interés de ,
4 4
en un trimestre. Como hay 4 periodos, tenemos lo siguiente:
0.06 4
A = 1000a1 + b ⬟ $1061.36
4
El saldo al final de 1 año es de aproximadamente $1061.36.

© Lopez Mateos Editores. ISBN 978-607-95583-2-1, obra completa, versión electrónica, ISBN 978-607-95583-3-8, volumen 1, versión electrónica. Ejemplar asignado a: Helecto Villarroel gutierrez -
helecto@gmail.com. Fecha: 27 de octubre de 2014. Prohibida su modificación, copia o distribución.
508 Razonamiento proporcional, porcentajes y aplicaciones

Página de un libro de texto TA S A S D E I N T E R É S COM P U ESTO

Tecnología

6TPEFVOBIPKBEFDÈMDVMPt*OUFSÏTDPNQVFTUP
1SPCMFNB {2VÏFTUSBUFHJBEFJOWFSTJØOQSPEVDJSÈVONBZPSJODSFNFOUP
FOUVJOWFSTJØOEFFOB×PTTJHBOBTFMEFJOUFSÏTDPNQVFTUP
BOVBMPTJHBOBTFMEFJOUFSÏTDPNQVFTUPNFOTVBMNFOUF
Una hoja de cálculo te puede ayudar a resolver este problema.

$PMPDBMBJOGPSNBDJØOTJHVJFOUFFOUV
IPKBEFDÈMDVMP TFHÞOTFNVFTUSB Año Importe Tasa anual Mes Importe Tasa mensual

$PMPDBMBTGØSNVMBTTJHVJFOUFT
&OMBDFMEB$ DPMPDB
&OMBDFMEB' DPMPDB
&OMBDFMEB" DPMPDB"  Año Importe Tasa anual Mes Importe Tasa mensual
&OMBDFMEB# DPMPDB# # $
&OMBDFMEB% DPMPDB% 
&OMBDFMEB& DPMPDB& & '

4FMFDDJPOBMBTDFMEBT"B'ZVTB
MBJOTUSVDDJØOEF-MFOBS"CBKP
Año Importe Tasa anual Mes Importe Tasa mensual

4PMVDJØO $100 al 4% de interés


compuesto anualmente es $116.99.
$100 al 3.75% de interés compuesto
mensualmente es sólo $116.16.

I N T É N TA LO
POR TI MISMO
¿Qué estrategia de inversión
producirá un mayor incremento en
¿Qué es más fácil, calcular el interés compuesto con una
tu inversión de $100 en 4 años: si
calculadora o usar una hoja de cálculo? Explica.
ganas el 5% de interés compuesto
anual o si ganas el 5% de interés ¿Por qué debes dividir la tasa de interés entre el número de
compuesto trimestralmente (4 veces periodos por componer?
al año)?
¿Por qué tienes que usar el símbolo “=” para colocar las fórmulas?

Fuente: Scott Foresman-Addison Wesley Middle School Math, Course 3, 2002 (p. 312).

© Lopez Mateos Editores. ISBN 978-607-95583-2-1, obra completa, versión electrónica, ISBN 978-607-95583-3-8, volumen 1, versión electrónica. Ejemplar asignado a: Helecto Villarroel gutierrez -
helecto@gmail.com. Fecha: 27 de octubre de 2014. Prohibida su modificación, copia o distribución.
Sección 8-3 Cálculo de interés 509

b. Como el interés ganado es $1061.36 - $1000.00 = $61.36, el rendimiento


efectivo anual se puede calcular usando la fórmula del interés simple, I = Crt.
61.36 = 1000 # r # 1
61.36
= r
1000
0.06136 = r
6.136% = r
El rendimiento efectivo anual es del 6.136%.

Ejemplo 8-17 Con el propósito de ahorrar para los estudios universitarios de sus hijos, una pareja deposita
$3000 en una cuenta que paga el 7% de interés anual compuesto diariamente. Halla el im-








porte en esta cuenta después de 8 años.

Solución El capital en el problema es de $3000, la tasa diaria r es de 0.07>365 y el número


de periodos de composición es de 8 # 365, ó 2920. Así, tenemos que
0.07 2920
A = $3000a1 + b ⬟ $5251.74
365
Por lo tanto, el importe en la cuenta es de aproximadamente $5251.74.

Evaluación 8-3A

Necesitarás una calculadora para solucionar la mayoría de los problemas.


1. Completa la siguiente tabla de interés compuesto.

Periodo Tasa de Importe Importe


Periodo de Tasa de tiempo interés por Número de de interés total
composición Capital anual (años) periodo periodos pagado contabilizado
a. Semestral $1000 6% 2
b. Trimestral $1000 8% 3
c. Mensual $1000 10% 5
d. Diario $1000 12% 4

2. Doña Rosa pidió un préstamo de $42,000 a un 8.75% de 5. Una compañía expande su línea para incluir más produc-
interés simple anual para comprar una casa. Si exacta- tos. Para ello pide un préstamo de $320,000 a un 13.5%
mente 1 año después pudo pagar el préstamo sin sanción, de interés anual simple por un periodo de 18 meses.
¿cuánto debía de intereses? ¿Cuánto de interés debe pagar la compañía?
3. Carolina pidió un préstamo de $125. Si la tasa de interés 6. A fin de ahorrar para su retiro, una pareja deposita
es de 1.5% al mes sobre saldos insolutos y ella no paga la $4000 en una cuenta que paga un 5.9% de interés com-
deuda durante 1 año, ¿cuánto debe de intereses al final puesto trimestralmente. ¿Cuál será el valor de su inver-
del año? sión después de 20 años?
4. El restaurante La Reina necesitará $50,000 en 5 años para 7. Una distribuidora de autos ofrece préstamos a una tasa
una ampliación. Para lograr este objetivo, la compañía de- de interés simple del 4.7%. Halla el interés que se carga
posita dinero en una cuenta que paga el 3% de interés anual a un cliente que solicita un préstamo de $7200 por 3
compuesto trimestralmente. Halla la cantidad que deberá años.
invertirse para obtener un total de $50,000 en 5 años.

© Lopez Mateos Editores. ISBN 978-607-95583-2-1, obra completa, versión electrónica, ISBN 978-607-95583-3-8, volumen 1, versión electrónica. Ejemplar asignado a: Helecto Villarroel gutierrez -
helecto@gmail.com. Fecha: 27 de octubre de 2014. Prohibida su modificación, copia o distribución.
510 Razonamiento proporcional, porcentajes y aplicaciones

8. Pedro y Graciela tienen tres planes de ahorro, con los 11. Se depositó en un banco la cantidad de $3000 a una tasa del
cuales se acumulan las siguientes cantidades de interés 2% compuesto trimestralmente durante 3 años. Después la
por 1 año: tasa se incrementó a 3% y se compuso trimestralmente du-
(i) Una libreta de ahorros que acumuló $53.90 sobre un rante los siguientes 3 años. Si no se retiró dinero, ¿cuál fue
capital de $980 el saldo al final de ese tiempo?
(ii) Un certificado de depósito que acumuló $55.20 so- 12. El Banco NuevaEra anuncia tasas de interés del 4% com-
bre un capital de $600 puestas diariamente, mientras que el banco PagoMás paga
(iii) Un certificado en mercado de dinero que acumuló el 5.2% de interés compuesto anualmente. ¿Qué banco
$158.40 sobre un capital de $1200 ofrece una mejor tasa para una cliente que planea dejar su
¿Cuál de estas cuentas pagó la mejor tasa de interés para dinero por exactamente 1 año?
el año? 13. Si se paga una tasa de interés simple sobre una cuenta de
9. Un emparedado cuesta $1.35 y el precio sigue creciendo a ahorros por cierto periodo de años y se retira suficiente
una tasa del 11% al año durante los siguientes 6 años. ¿Cuál dinero de la cuenta de modo que el capital se mantenga
será el precio de un emparedado al final de 6 años? fijo, pero no se retira más dinero que ése, ¿qué tipo de
10. Adriana y Javier depositaron $300 el 1º de enero en una sucesión representa el dinero ganado cada año?
cuenta de ahorro para vacaciones que paga 1.1% de interés
mensual, y retiran el dinero el 1º de diciembre del mismo
año. ¿Cuál es el rendimiento efectivo anual?

Evaluación 8-3B

Necesitarás una calculadora para solucionar la mayoría de los problemas.


1. Completa la siguiente tabla de interés compuesto.

Periodo Tasa de Importe Importe


Periodo de Tasa de tiempo interés por Número de de interés total
composición Capital anual (años) periodo periodos pagado contabilizado
a. Semestral $1000 4% 2
b. Trimestral $1000 6% 3
c. Mensual $1000 18% 5
d. Diario $1000 18% 4

2. Una persona cobró $28,500 de un préstamo de $25,000 7. Se compra un carro por $15,000. Si el carro se deprecia
que hizo hace 4 años. Si cargó interés simple, ¿cuál fue 10% de su valor cada año, ¿cuál es su valor al final del
la tasa? tercer año?
3. Si la colegiatura es de $10,000 este año, ¿de cuánto será 8. El interés anual compuesto que se paga sobre una cuenta
dentro de 10 años suponiendo una tasa de inflación de ahorros es del 4% por un periodo de n años y el inte-
constante de 9% al año? rés permanece en la cuenta; ¿qué tipo de sucesión repre-
4. Susana invirtió dinero en un banco que pagó 3.5% com- senta el importe de cada año?
puesto trimestralmente. Si ella tenía $4650 al final de 4 9. Si una compañía editorial firmó un acuerdo para permi-
años, ¿cuál fue su inversión inicial? tir que un libro de texto (publicado originalmente con
5. El número de árboles en una selva tropical decrece en un 500 páginas) aumente a 1000 páginas a través de varias
0.5% mensual. Si la selva tiene aproximadamente ediciones y que ese aumento se dé en alrededor del 10%
2.34 # 109 árboles, ¿cuántos árboles quedarán después de en el número de páginas de cada edición, ¿cuántas edi-
20 años? ciones podrán publicarse antes de alcanzar el límite con-
6. Un fondo de mercado de dinero paga 14% de interés tractual?
anual compuesto diariamente. ¿Cuál es el valor de 10. A Ana le cobran el 12% de interés anual compuesto
$10,000 invertido en este fondo después de 15 años? mensualmente sobre saldos insolutos de un préstamo de

© Lopez Mateos Editores. ISBN 978-607-95583-2-1, obra completa, versión electrónica, ISBN 978-607-95583-3-8, volumen 1, versión electrónica. Ejemplar asignado a: Helecto Villarroel gutierrez -
helecto@gmail.com. Fecha: 27 de octubre de 2014. Prohibida su modificación, copia o distribución.
Sección 8-3 Cálculo de interés 511

$2000. Ella no realizó pagos durante 2 años. Su amigo le 12. Ale invierte $1000 al 6% de interés anual compuesto
dijo que la cantidad que debe se ha más que duplicado. diariamente y Beti invierte $1000 al 7% de interés sim-
¿Es correcto esto? ¿Cuánto debe ella ahora? ple. ¿Después de cuántos años completos la inversión de
11. Si se espera que el precio de un carro nuevo se incre- Ale valdrá más que la inversión de Beti?
mente en 2.5% al año, ¿cuál sería el precio de un carro
nuevo dentro de 5 años si ahora cuesta $32,400?

Conexiones matemáticas 8-3

Comunicación Aprendizaje colectivo


1. Debido a una recesión, el valor de una casa nueva se de- 7. La Ley sobre Préstamos Transparentes, aprobada en 2003,
preció un 10% anual durante 3 años seguidos. Después, exige que las instituciones de préstamos coticen un Costo
durante los siguientes 3 años el valor de la casa se incre- Anual Total (CAT), una tasa en términos porcentuales
mentó en un 10% anual. ¿Creció o decreció el valor de la anuales que ayude a los consumidores a comparar los ver-
casa después de esos 6 años? Explica. daderos costos de los préstamos, sin importar cómo calcule
2. Determina el número de años (a la décima más cercana) cada institución de préstamos los intereses y los sume a los
que requeriría duplicar una cantidad de dinero si se depo- costos.
sitara a una tasa del 10% de interés compuesto anual. Ex- a. Llama a diferentes bancos y pregunta por el CAT de
plica tu razonamiento. algunos préstamos y cuál es el significado del CAT.
3. Cada año el valor de un carro se deprecia un 20% res- b. Con base en lo hallado en (a), escribe una definición
pecto al año anterior. Miguel asegura que después de 5 clara de CAT.
años el carro se depreciaría en un 100% y no valdría nada. c. Usa la información dada por tu tarjeta de crédito
¿Está Miguel en lo correcto? Explica por qué sí o por qué (quizá necesites llamar o ir al banco) y calcula el CAT
no. De no ser así, halla el porcentaje real de depreciación sobre disposiciones en efectivo. ¿Tu respuesta es igual
del carro en 5 años. que la respuesta del banco? Compara el CAT para di-
ferentes tarjetas de crédito.
Solución abierta
4. El efecto de la depreciación se puede calcular usando una Preguntas del salón de clase
fórmula similar a la del interés compuesto. 8. Jena dice que ella no entendió la diferencia entre interés
a. Supón que la depreciación es la misma cada mes. Escribe simple e interés compuesto. Ella dice que entre más alta sea
un problema que involucre depreciación y resuélvelo. la tasa de interés que pague el banco, más dinero obtendrá
b. Desarrolla una fórmula general para la depreciación sin importar el tipo de interés. ¿Cómo le respondes?
definiendo qué significa cada variable de la fórmula. 9. Nora leyó que a las mujeres les pagan 75¢ por cada dólar
5. Halla cuatro ciudades grandes en el mundo y una tasa apro- pagado a los hombres. Ella dice que esto significa que a
ximada de crecimiento de la población de estas ciudades. los hombres les pagan 25% más que a las mujeres. ¿Está
Estima la población de cada una de las cuatro ciudades den- ella en lo correcto? ¿Por qué?
tro de 25 años. 10. Una estudiante asegura que si el valor de un artículo se
6. Enuncia diversas situaciones que no impliquen dinero, en incrementa anualmente en un 100% anual de su valor
las cuales se use una fórmula parecida a la del interés com- del año anterior y si el precio original es p pesos, enton-
puesto. En cada caso, enuncia un problema relacionado y ces el valor después de n años será de p # 2n pesos. ¿Está
escribe la solución. en lo correcto la estudiante? ¿por qué sí o por qué no?

© Lopez Mateos Editores. ISBN 978-607-95583-2-1, obra completa, versión electrónica, ISBN 978-607-95583-3-8, volumen 1, versión electrónica. Ejemplar asignado a: Helecto Villarroel gutierrez -
helecto@gmail.com. Fecha: 27 de octubre de 2014. Prohibida su modificación, copia o distribución.
512 Razonamiento proporcional, porcentajes y aplicaciones

Sugerencia para resolver el problema preliminar


Plantea ecuaciones para hallar el valor de cada carro. Después compara el valor total
de los dos carros con la cantidad en que los vendió, para obtener la respuesta.

Resumen del capítulo

I. Razón y proporción a c a b
2. Si = , entonces = , donde c Z 0.
a b d c d
A. Una fracción es una razón.
b II. Porcentaje e interés
B. Una proporción es una igualdad entre dos ra- A. Porcentaje significa por ciento. El porcentaje se
zones. x
C. Si a, b, c y d son números reales y b Z 0 y escribe usando el símbolo %: x% = .
100
a c *B. El interés simple se calcula usando la fórmula
d Z 0, entonces = si, y sólo si, ad = bc.
b d I = Crt, donde I es el interés, C es el capital, r
D. Si las variables x y y están relacionadas por la igual- es la tasa de interés anual y t es el tiempo en
dad y = kx, a k = b , entonces se dice que y es
y años.
x *C. Cuando se calcula el interés compuesto usa-
proporcional a x y k es la constante de pro- mos la fórmula S = C11 + r2n, donde S es el
porcionalidad. saldo, C es el capital, r es la tasa de interés por
E. Propiedades de las proporciones periodo y n es el número de periodos.
a c b d
1. Si = , entonces = , donde a Z 0 y
b d a c
c Z 0.

Revisión del capítulo

1. Tomás echó una moneda 30 veces y obtuvo 17 águilas. 5. Una receta de ensalada de frutas es para 4 personas.
a. ¿Cuál es la razón de las águilas a los volados? Lleva 3 naranjas y 16 uvas. ¿Cuántas naranjas y uvas
b. ¿Cuál es la razón de las águilas a los soles? se necesitan para 11 personas?
c. ¿Cuál es la razón de los soles a las águilas? 6. Si la escala para dibujar una casa es de 1 cm = 2.5 m,
2. ¿Qué botella de jugo es mejor compra (costo por onza): ¿cuánto mide la casa si en el dibujo a escala mide 3 cm?
48 onzas fluidas por $3.05 ó 64 onzas fluidas por $3.60? 7. En el agua (H2O), la razón de peso del oxígeno al
3. El oro de dieciocho quilates contiene 18 partes peso del hidrógeno es de aproximadamente 8 : 1.
(gramos) de oro y 6 partes (gramos) de otros meta- ¿Cuántas onzas de hidrógeno hay en 1 lb de agua?
les. El nuevo anillo de Ana contiene 12 partes de 8. Para estimar el número de peces en un lago, los in-
oro y 3 partes de otros metales. ¿Es el anillo de 18 vestigadores usan la técnica de marcar y recapturar.
quilates de oro? ¿Por qué? Se capturan peces, se marcan y se liberan en el lago.
4. Despeja x en cada caso: Después de un tiempo se capturan peces y se cuenta
15 21 20 110 el número de peces marcados.
a. = b. =
12 x 35 x Sea T el número total de peces capturados, marca-
1 3 dos y liberados, n el número de peces recapturados en
2 2 una muestra y t el número de peces marcados en esa
c. = muestra. Finalmente, sea x el número de peces en el
1 x
3

© Lopez Mateos Editores. ISBN 978-607-95583-2-1, obra completa, versión electrónica, ISBN 978-607-95583-3-8, volumen 1, versión electrónica. Ejemplar asignado a: Helecto Villarroel gutierrez -
helecto@gmail.com. Fecha: 27 de octubre de 2014. Prohibida su modificación, copia o distribución.
Revisión del capítulo 513

lago. La hipótesis es que la razón entre los peces mar- 16. Convierte cada porcentaje a decimal:
cados y el número total de peces en cualquier mues- 2
a. 60% b. % c. 100%
tra es aproximadamente igual y de ahí los científicos 3
t T 17. Sandra recibió un dividendo igual a 11% del valor de
suponen que = . Supón que se capturaron, se
n x su inversión. Si su dividendo fue de $1020.80, ¿de
marcaron y se liberaron 173 peces. Después se re-
cuánto fue su inversión?
capturaron 68 peces y de ellos 21 estaban marcados.
18. De un cargamento de 150 computadoras se halló
Estima el número de peces en el lago.
que cinco estaban defectuosas. ¿Qué porcentaje de
9. Un fabricante produce el mismo tipo de procesador
las computadoras estaba defectuoso?
de computadora en dos plantas. En la primera planta
19. En un examen de matemáticas un estudiante falló 8
la razón de procesadores defectuosos a buenos es de
de 70 preguntas. ¿Qué porcentaje de preguntas, re-
15 : 100 y en la segunda planta esa razón es de
dondeado a la décima de porcentaje más cercana,
12 : 100. Una compradora de un gran número de pro-
contestó correctamente el estudiante?
cesadores sabe que algunos vienen de la primer planta
20. Actualmente una computadora portátil cuesta $3450.
y otros de la segunda. Sin embargo, no sabe cuántos
Esto es el 60% del costo de hace 4 años. ¿Cuál era el
vienen de cada una. La compradora quisiera saber la
costo de la computadora hace 4 años? Explica tu ra-
razón de procesadores defectuosos a procesadores
zonamiento.
buenos en cualquier orden de compra. ¿Puede ella de-
21. Si al comprar un traje nuevo te ofrecen descuentos
terminar esa razón? De ser así, explica cómo. Si no,
sucesivos del 5%, 10% ó 20% en el orden que
explica por qué no.
quieras, ¿qué orden debes elegir?
10. Supón que la razón de las longitudes de los lados de
22. Juana compró una bicicleta y la vendió en un 30%
dos cuadrados es de 1 : r. ¿Cuál es la razón de sus
áreas? 1A = s 2.2
más de lo que pagó por ella. La vendió en $104.
¿Cuánto pagó por ella?
11. Los Osos ganaron 18 juegos y perdieron 7.
23. La librería estudiantil vendía un libro en $89.95. Un
a. ¿Cuál es la razón de los juegos ganados a los juegos
estudiante encontró el libro en eBay por $62.00. Si
perdidos?
el estudiante compró el libro en eBay, ¿qué por-
b. ¿Cuál es la razón de los juegos ganados a los juegos
centaje del costo del libro de la librería ahorró?
jugados?
a 24. Cuando una tienda tenía una venta del 60% de des-
12. Expresa cada caso como una razón , donde a y b cuento, Dora tenía un cupón para un descuento adi-
b
son números completos: cional del 40% y pensó que podía obtener gratis el
1 2 3 vestido que quería. Si fueras el gerente de la tienda,
a. : 1 b. :
5 5 4 ¿cómo le explicarías la situación matemática?
13. La razón de niños a niñas en el grupo de la maestra 25. Explica qué caso de los siguientes tiene sentido
Chonita es de 3 a 5, la razón de niños a niñas en el como porcentaje matemático:
grupo de la maestra Berta es la misma, y tú sabes que a. p%
hay 15 niñas en el grupo de la maestra Berta. ¿Cuán- b. 12%
tos niños hay en la clase de la maestra Berta? c. 0.34%
14. Responde a lo siguiente: d. 11 + 0.32%
a. ¿Qué porcentaje es 6 de 24? 26. A una compañía se le ofreció un préstamo de
b. ¿Cuál es el 320% de 60? * $30,000 a una tasa de interés simple anual de 12.5%
c. ¿De qué número es 17 el 30%? por 4 años. Halla el interés simple del préstamo al
d. ¿Qué porcentaje es 0.2 de 1? final del año 4.
15. Convierte cada caso a porcentaje: *27. Un fondo paga el 14% anual de interés compuesto
1 3 cada trimestre. ¿Cuál es el valor de una inversión de
a. b. $10,000 después de 3 años?
8 40
c. 6.27 d. 0.0123
3
e.
2

© Lopez Mateos Editores. ISBN 978-607-95583-2-1, obra completa, versión electrónica, ISBN 978-607-95583-3-8, volumen 1, versión electrónica. Ejemplar asignado a: Helecto Villarroel gutierrez -
helecto@gmail.com. Fecha: 27 de octubre de 2014. Prohibida su modificación, copia o distribución.
514 Razonamiento proporcional, porcentajes y aplicaciones

Bibliografía seleccionada

Abrahamson, D., and C. Cigan. “A Design for Ratio Miller, J., and J. Fey. “Proportional Reasoning.” Mathe -
and Proportion Instruction.” Mathematics Teaching matics Teaching in the Middle School 5 ( January 2000):
in the Middle School 8 (May 2003): 493–501. 310–313.
Beckman, C., D. Thompson, and R. Austin. “Exploring Moss, J., and B. Caswell. “Building Percent Dolls: Con-
Proportional Reasoning Through Movies and Lit- necting Linear Measurement to Learning Ratio and
erature.” Mathematics Teaching in the Middle School Proportion.” Mathematics Teaching in the Middle
9 ( January 2004): 256–261. School 10 (September 2004): 68–74.
Billings, E. “Problems That Encourage Proportion Post, T., M. Behr, and R. Lesh. “Proportionality and
Sense.” Mathematics Teaching in the Middle School the Development of Pre-Algebra Understandings.”
5 ( January 2000): 310–313. In The Ideas of Algebra, K–12, edited by A. Coxford
Chapin, S., and N. Anderson. “Crossing the Bridge to and A. Shulte. Reston, VA: National Council of
Formal Proportional Reasoning.” Mathematics Teachers of Mathematics, 1988.
Teaching in the Middle School 8 (April 2003): 420–425. Reys, R., B. Reys, N. Nohda, and H. Emori. “Mental
Cramer, K., and T. Post. “Connecting Research to Computation Performance and Strategy Use of
Teaching Proportional Reasoning.” Mathematics Japanese Students in Grades 2, 4, 6, and 8.” Journal
Teacher 86 (May 1993): 404–407 for Research in Mathematics Education 26 ( July 1995):
Curcio, F., and N. Bezuk. “Understanding Rational 304–326.
Numbers and Proportions.” Curriculum and Evalua- Scaptura, C., J. Suh, and G. Mahaffey. “Masterpieces
tion Standards for School Mathematics Addenda Series to Mathematics: Using Art to Teach Fraction,
Grades 5–8. Reston, Va.: National Council of Teach- Decimal, and Percent Equivalents.” Mathematics
ers of Mathematics, 1994. Teaching in the Middle School 13 (August 2007):
Harel, G., M. Behr, R. Lesh, and T. Post. “Invariance of 24–28.
Ratio: The Case of Children’s Anticipatory Scheme Seeley, C., and J. Schielack. “A Look at the Development
for Constancy of Taste.” Journal of Research in of Ratios, Rates, and Proportionality.” Mathematics
Mathematics Education 25 ( July 1994): 324–345. Teaching in the Middle School 13 (October 2007):
Hart, K. Ratio: Children’s Strategies and Errors. Windsor, 140–142.
England: NFER-Nelson Pub. Co., 1984. Sharp, J., and B. Adams. “Using a Pattern Table to
Hoffer, A., and S. Hoffer. “Ratios and Proportional Solve Contextualized Proportion Problems.”
Thinking.” Teaching Mathematics in Grades K–8, Mathematics Teaching in the Middle School 8 (April
edited by T. Post. Boston, MA: Allyn & Bacon, 2003): 432–439.
1988, pp. 285–312. Sowder, J., D. Wearne, W. Martin, and M. Strutchens.
Horak, V. “A Science Application of Area and Ratio “What Do 8th-Grade Students Know about Mathe-
Concepts.” Mathematics Teaching in the Middle School matics?” In Results and Interpretations of the 1990–2000
11 (April 2006): 360–365. Mathematics Assessments of the National Assessment
Lanius, C., and S. Williams. “Proportionality: A Unify- of Educational Progress, edited by P. Kloosterman and
ing Theme for the Middle Grades.” Mathematics F. Lester. Reston, VA.: NCTM, 2004, pp. 105–144.
Teaching in the Middle School 8 (April 2003): 392–396. Sweeney, E., and R. Quinn. “Concentration: Connecting
Lembke, L., and B. Reys. “The Development of, and Fractions, Decimals, & Percents.” Mathematics Teach-
Interaction between, Intuitive and School-taught ing in the Middle School 5 ( January 2000): 324–328.
Ideas about Percent. ” Journal of Research in Mathe- Thompson, C., and W. Bush. “Improving Middle School
matics Education 25 (May 1994): 237–259. Teachers’ Reasoning about Proportional Reasoning.”
Lesh, R., T. Post, and M. Behr. “Proportional Reason- Mathematics Teaching in the Middle School 8 (April
ing.” In Number Concepts and Operations in the Middle 2003): 398–403.
Grades, edited by J. Hiebert and M. Behr. Reston, VA: VanDooren, W., D. DeBock. L. Verschaffel, and
National Council of Teachers of Mathematics, 1988. D. Janssens. “Improper Applications of Propor-
Lo, J., T. Watanabe, and J. Cai. “Developing Ratio Con- tional Reasoning.” Mathematics Teaching in the Mid-
cepts: An Asian Perspective.” Mathematics Teaching in dle School 9 (December 2003): 204–209.
the Middle School 9 (March 2004): 362–367. Watson, J., M. Shaughnessy, and M. Perlwitz. “Propor-
Martine, S., and J. Bay-Williams. “Using Literature to tional Reasoning: Lessons from Research in Data
Engage Students in Proportional Reasoning.” Mathe - and Chance.” Mathematics Teaching in the Middle
matics Teaching in the Middle School 9 (November School 10 (September 2004): 104–109.
2003): 142–148.

© Lopez Mateos Editores. ISBN 978-607-95583-2-1, obra completa, versión electrónica, ISBN 978-607-95583-3-8, volumen 1, versión electrónica. Ejemplar asignado a: Helecto Villarroel gutierrez -
helecto@gmail.com. Fecha: 27 de octubre de 2014. Prohibida su modificación, copia o distribución.
Agradecimientos
School book pages:
Scott Foresman Addison Wesley Math Grade 4, copyright © 2008 Pearson Education, Inc. or its affiliate(s); Scott Foresman Addison
Wesley Math Grade 5, copyright © 2008 Pearson Education, Inc. or its affiliate(s); Scott Foresman Addison Wesley Math Grade 3,
copyright © 2008 Pearson Education, Inc. or its affiliate(s); Scott Foresman Addison Wesley Math Grade 6, copyright © 2008
Pearson Education, Inc. or its affiliate(s). Used by permission. All rights reserved.
Prentice Hall Math Course 3, copyright © 2008 Pearson Education, Inc. or its affiliate(s); Prentice Hall Math Course 2, copyright
© 2008 Pearson Education, Inc. or its affiliate(s); Prentice Hall Connected Mathematics Grade 7, copyright © 2006 Pearson Edu-
cation, Inc. or its affiliate(s). Used by permission. All rights reserved.
enVisionMATH 2009, Grade 6, copyright © 2009 Pearson Education, Inc. or its affiliate(s).
Used by permission. All rights reserved.
McDougal Littell Math Thematics, copyright © 2008 McDougal Littell. All rights reserved. Used by permission of Holt
McDougal, a division of Houghton Mifflin Harcourt Publishing Company.

NAEP questions from the National Assessment of Educational Progress (NAEP).


TIMSS questions from Trends in International Mathematics and Science Study (TIMSS).
Excerpts from NCTM Curriculum Focal Points reprinted with permission from Curriculum Focal Points for Prekindergarten
through Grade 8 Mathematics: A Quest for Coherence, copyright 2006 by the National Council of Teachers of Mathematics
(NCTM). All rights reserved.
Excerpts from NCTM Standards reprinted with permission from Principals and Standards for School Mathematics, copyright
2000 by the National Council of Teachers of Mathematics (NCTM). All rights reserved. NCTM does not endorse the content
or validity of these alignments.
The Geometer’s Sketchpad and Dynamic Geometry are registred trademarks of Key Curriculum Press. Sketchpad is a trade-
mark of Key Curriculum Press.

p. 1 Chelsea Pingree, p. 2 Jerry Craft, p. 3 AP Wide World Photos, p. 5 Library of Congress, p. 5 © 1997 Carolina
Biological Supply Company. Burlington, NC. Used by permission. p. 31 Universal Press Syndicate, p. 31 The Granger
Collection, p. 45 The Image Works, p. 61 Craig McAteer/Shutterstock, p. 65 KING FEATURES SYNDICATE,
p. 66 Johnny Lott/Shutterstock, p. 72 KING FEATURES SYNDICATE, p. 78 Corbis, Bettmann, p. 84 © United Feature
Syndicate October 5, 1965, p. 85 KING FEATURES SYNDICATE, p. 110 PhotoCreate/Shutterstock, p. 118 © King
Features Syndicate September 12, 1990, p. 119 © 2005 Creators Syndicate. Used by permission of John L. Hart FLP, and
Creators Syndicate, Inc. January 21, 2005, p. 143 St. Andrews University MacTutor Archive, p. 145 PhotoCreate/
Shutterstock, p. 145 Chiyacat/Shutterstock, p. 163 KING FEATURES SYNDICATE, p. 165 From Portraits of Eminent
Mathematicians by David Eugene Smith published by Pictorial Mathematics, New York, 1936, p. 178 Calvin and Hobbes
© 1990 Bill Watterson, September 15, 1990. Distributed by Universal Press Syndicate, Inc. February 9, 1992, p. 194 Monkey
Business Images/Shutterstock, p. 196 © Stefano Bianchetti/CORBIS All Rights Reserved, p. 197 National Archives, Public
Programs, p. 212 Sheila Terry/Photo Researchers, Inc., p. 221 George Bernard/Photo Researchers, Inc., p. 248 Steve
Cole/Getty Images, Inc.-Photodisc., p. 273 Bryn Mawr College, p. 286 Photo Researchers, Inc., p. 294 Constance Reid,
p. 302 Bill Amend/Universal Press Syndicate, p. 307 © Sidney Harris/ScienceCartoonsPlus.com, p. 308 Culver Pictures,
Inc., p. 309 The Granger Collection, p. 310 Picture Desk, Inc./Kobal Collection, p. 340 Ron Chapple/www.indexopen.com,
p. 344 KING FEATURES SYNDICATE, p. 409 Image Source/Superstock Royalty Free, p. 410 Corbis/Bettmann,
p. 413 KING FEATURES SYINDICATE p. 451 The Granger Collection, p. 455 Corbis/Bettmann, p. 476 LLC. Vstock/
www.indexopen.com, p. 498 KING FEATURES SYNDICATE, p. 515 idesygn/Shutterstock, p. 517 Photo Researchers,
Inc., p. 519 The Granger Collection, New York, p. 529 © Newspaper Enterprise Association. September 26, 1998,
p. 540 © Creators Syndicate. Used by permission of John L. Hart FLP, and Creators Syndicate, Inc. March 17, 1992,
p. 564 Stan Lynde, p. 567 © 1998 Carole Cable; April 6, 1998. Used with permission., p. 592 EyeWire Collection/Getty
Images-Photodisc, p. 593 Hulton Archive, p. 594 © 1981 LaughingStock Licensing, Inc., October 20, 1981. All rights
reserved. HERMAN® is a registered trademark of LaughingStock Licensing, Inc., p. 596 © Thaves. Reprinted by permission.,
p. 648 Kobal Collection
515
© Lopez Mateos Editores. ISBN 978-607-95583-2-1, obra completa, versión electrónica, ISBN 978-607-95583-3-8, volumen 1, versión electrónica. Ejemplar asignado a: Helecto Villarroel gutierrez -
helecto@gmail.com. Fecha: 27 de octubre de 2014. Prohibida su modificación, copia o distribución.
Créditos de la edición en inglés
Executive Editor Anne Kelly
Acquisitions Editor Marnie Greenhut
Executive Project Manager Christine O’Brien
Senior Project Editor Joanne Dill
Assistant Editor Leah Goldberg
Editorial Assistant Leah Driska
Senior Managing Editor Karen Wernholm
Senior Production Supervisor Kathleen A. Manley
Senior Designer Barbara T. Atkinson
Executive Media Manager Peter Silvia
Software Development Eileen Moore (Math XL), Marty Wright (TestGen)
Executive Marketing Manager Becky Anderson
Marketing Assistant Katherine Minton
Senior Author Support/Technology Specialist Joe Vetere
Rights and Permissions Advisor Shannon Barbe
Manufacturing Manager Evelyn Beaton
Cover and Text Design Susan Raymond
Production Coordination, Composition, and Illustrations Pre-Press PMG

Cover image: From CRITICAL THINKING ACTIVITIES IN PATTERNS, IMAGERY, LOGIC,


GRADES 4–6, © 1988 by Pearson Education, Inc. or its affiliate(s). Used by permission. All rights reserved.

516

© Lopez Mateos Editores. ISBN 978-607-95583-2-1, obra completa, versión electrónica, ISBN 978-607-95583-3-8, volumen 1, versión electrónica. Ejemplar asignado a: Helecto Villarroel gutierrez -
helecto@gmail.com. Fecha: 27 de octubre de 2014. Prohibida su modificación, copia o distribución.
RESPUESTA S A LOS P RO B L E M A S

Aquí aparecen las respuestas a los ejercicios de la Evaluación A, a los problemas impares de
Conexiones matemáticas, a los problemas de Revisión del capítulo, a los problemas de
Ahora intenta éste, a los Rompecabezas, a las Actividades de laboratorio, a los Rincones
de la tecnología y a los Problemas preliminares del Volumen uno.

Capítulo 1 mediante problemas interesantes. Los estudiantes necesitan


saber cómo resolver problemas para poder avanzar en estos
Evaluación 1-1A problemas y, a su vez, aprender matemáticas. 3. Las respues-
1. (a) 4950 (b) 251,001 2. 10,248 3. 12 tas pueden variar desde estudiantes que no tienen idea de qué
4. 160 km 5. Marisolita, Trueno, Yolanda, Chocolata hacer e intentan adivinar y ver qué tan cerca están, hasta el
6. 45 7. $33.20, (uno de 10, uno de 5, cuatro de a peso, uso de conjeturas para empezar a aproximarse a la respuesta
nueve de 50¢, veinticuatro de 20¢ y cuarenta y nueve de exacta. Si se hacen conjeturas inteligentes y se aprende algo
10¢) 8. (a) (i) 541 * 72 (ii) divide 754 entre 12 de cada conjetura, entonces el estudiante se puede acercar a la
(b) (i) 257 * 14 (ii) divide 124 entre 75 9. $5,256,000 respuesta exacta.
10. 12 11. $24.50 12. 23 peldaños
Solución abierta
13. (a) 10,500 cuadrados (b) n2 + 5n cuadrados
14. ancho = 230 m; longitud = 310 m 15. 9°C 5. Las respuestas pueden variar; por ejemplo, cualesquiera
16. Alicia—invierno; Beti—verano; Carlos—primavera; de los problemas de esta sección son ejemplos de problemas
Daniel—otoño 17. A = 9. que podrían abordarse con la estrategia estudiada.

Conexiones matemáticas 1-1 Aprendizaje colectivo


Comunicación 7. Las respuestas pueden variar; por ejemplo, si el alcance
1. Las respuestas pueden variar; por ejemplo, las habilidades promedio en tu grupo es de 1.8 m, entonces serían necesarias
para resolver problemas pueden ayudar a los estudiantes a aproximadamente 22,000,000 de personas. 9. (a) Para un
enfrentar retos futuros en el trabajo, la vida cotidiana y la libro de 100 páginas, se necesitan 25 hojas de papel. (b) La
escuela. Las habilidades para resolver problemas permiten suma de los números de página sobre el mismo lado de la
atacar con seguridad nuevas tareas y problemas. Si falla un hoja es 101. (c) La suma de todos los números de página
primer intento, los buenos solucionadores de problemas se en un libro de 100 páginas es 5050.
pueden reponer con enfoques alternativos. Mucho de las (d) El caso general se da en la siguiente tabla.
matemáticas que se enseña a los estudiantes se introduce
Suma de dos números de
Número de Número de página sobre el Suma de todos los
hojas páginas de libro mismo lado de la hoja números de página
4#5
1 4 5 1 + 2 + 3 + 4 = = 10
2
8#9
2 8 9 1 + 2 + 3 + Á + 8 = = 36
2
12 # 13
3 12 13 1 + 2 + 3 + Á + 12 = = 78
2
o o o o
4n14n + 12
n 4n 4n + 1 1 + 2 + 3 + Á + 4n =
2
= 2n14n + 12

Preguntas del salón de clase incorrectas en este punto porque quizá nunca se molestaron
en verificar si la respuesta obtenida tenía sentido. Es en este paso
11. Las respuestas pueden variar; por ejemplo, es en el donde los estudiantes reflexionan sobre las matemáticas que
último paso donde los estudiantes examinan si su respuesta usaron y determinan si puede haber otras formas de resolver
es razonable y si cumple las condiciones originales del el problema. Además, en esta etapa se reflexiona sobre
problema. En ocasiones, los estudiantes llegan a respuestas conexiones con otros problemas o generalizaciones que se
© Lopez Mateos Editores. ISBN 978-607-95583-2-1, obra completa, versión electrónica, ISBN 978-607-95583-3-8, volumen 1, versión electrónica. Ejemplar asignado a: Helecto Villarroel gutierrez -
helecto@gmail.com. Fecha: 27 de octubre de 2014. Prohibida su modificación, copia o distribución.
R2 Respuestas a los problemas

resaltan en los estándares del NCTM. 13. Las respuestas segunda sucesión. La segunda se puede generar multiplican-
pueden variar; por ejemplo, si esos nueve números se van a do cada término de la primera sucesión por 10. 3. (a) Sí.
usar en un cuadrado mágico, entonces la suma en cada una de La diferencia entre los términos de la nueva sucesión es igual
las tres columnas debe ser el mismo número natural. Este que la anterior pues se sumó un número fijo a cada número de
número debe ser 1>3 de la suma de los nueve números. Sin la sucesión. (b) Sí. Si el número fijo es k, la diferencia entre
embargo, 1 + 3 + 4 + 5 + 6 + 7 + 8 + 9 + 10 = 53 y los términos de la segunda sucesión es k veces la diferencia
53>3 = 17 2>3, que no es un número natural. Por lo tanto, entre los términos de la primera sucesión. (c) Sí. La
estos números no se pueden usar en un cuadrado mágico. diferencia de la nueva sucesión es la suma de las diferencias
de las sucesiones originales.
Evaluación 1-2A
Solución abierta
1. (a) (b) (c)
5. Las respuestas pueden variar; por ejemplo, presentamos
dos patrones:
2. (a) 11, 13, 15; aritmética (b) 250, 300, 350; aritmética
(c) 96, 192, 384; geométrica (d) 106, 107, 108 geométrica
(e) 33, 37, 41; aritmética (f ) 63, 73, 83; ni una ni otra
3. (a) 199; 2n - 1 (b) 4950; 501n - 12
(c) 3 # 299; 3 # 2n - 1 (d) 10100; 10n (e) 405; 5 + 4n ó ,
9 + 41n - 12 (f ) 1003 = 1,000,000; n3 4. 2, 7, 12
5. (a) 2, 4, 8 (b) 169, 256, 169 La regla es elevar al 1 + 12 + 24 + 36 + 48
1 + 12(1 + 2 + 3 + 4)
12(10) + 1

cuadrado la suma de los dígitos del término anterior.


7. Las respuestas varían dependiendo de la sucesión.
(c) 4, 16, 37. Si un es el dígito de las unidades de an, tn es el
dígito de las decenas de an, y hn es el dígito de las centenas Aprendizaje colectivo
de an, entonces an = 1un - 122 + 1tn - 122 + 1hn - 122 (d) La
sucesión original se repetirá infinidad de veces. 9. (a) 81 (b) 40 (c) 3n - 1 (d) 1 + 3 + 32 + 33 +
6. (a) 30, 42, 56 (b) 10,100 (c) n1n + 12 o n2 + n . . . + 3n - 2; para n Ú 2.
7. (a) 41 (b) 4n + 1, ó 5 + 1n - 124 (c) 12n + 4
Preguntas del salón de clase
8. (a) 42 (b) 4n + 2 ó 6 + 1n - 124 9. 1200 estudiantes
10. vigésimo tercer año 11. (a) 3, 5, 9, 15, 23, 33 (b) 4, 6, 11. Las respuestas pueden variar; por ejemplo, se le pudo
10, 16, 24, 34 (c) 15, 17, 21, 27, 35, 45 12. (a) 299, 447, decir a Juanito que encontró un patrón que cumplen muchos
644 (b) 56, 72, 90 13. (a) 101 (b) 61 (c) 200 números, pero que debe tener cuidado al generalizar sus
(d) 11 14. (a) 3, 6, 11, 18, 27 (b) 4, 9, 14, 19, 24 conjeturas a todos los números. Se le debe alentar a buscar
(c) 9, 99, 999, 9999, 99999 (d) 5, 8, 11, 14, 17 más números para ver si puede encontrar un contraejemplo de
15. (a) Las respuestas pueden variar; por ejemplo si x = 5, su conjetura. Si comienza a investigar, hallará que 14 termina
5 + 5 en 4, pero no es divisible entre 4. 34 es otro número que
entonces Z 5 + 1. (b) Las respuestas pueden variar; termina en 4, pero que no es divisible entre 4. Estos contra-
5
por ejemplo, si x = 2, entonces 12 + 422 Z 2 + 16. ejemplos prueban que la conjetura de Juanito es falsa.
16. (a) 41 (b) n2 + 1n - 12 = n2 + n - 1 (c) sí, la 13. Se debe pedir al estudiante que muestre ejemplos de por
figura trigésimo quinta qué su afirmación es cierta. Luego, necesita explorar más
17. a3 = 8, a4 = 11, a5 = 14 18. El término 12 de la posibilidades para ver si puede hallar un contraejemplo.
5
suceción geométrica es mayor que el término 12 de la Considere 16 y 43. El numerador de la primera fracción es
sucesión aritmética. 19. (a) 1, 5, 9, 13, 17, 21, Á mayor que el numerador de la segunda, y el denominador de
(b) 4n + 1, o S + 1n - 124 20. Hay dos soluciones la primera fracción es mayor que el denominador de la
5
posibles: 64, 128, 256, ó -64, 128, -256. segunda. Sin embargo, 16 es menor que un medio y 43 es mayor
n1n - 12 3n2 - n n13n - 12 que un medio, de donde es claro que 16 5
no es mayor que 34.
21. (a) 51 (b) n2 + ,ó ,ó
2 2 2 Así, tenemos un contraejemplo que muestra que la conjetura es
Conexiones matemáticas 1-2 falsa. 15. Dos términos pueden conducir a más de una
sucesión. Por ejemplo, los términos 3, 6, Á conducen a 3, 6,
Comunicación 9, 12, Á , que es una sucesión aritmética con diferencia fija 3.
1. (a) Las respuestas pueden variar; por ejemplo, ambas También puede conducir a 3, 6, 9, 15, 24, 39, Á , en la cual
sucesiones comienzan igual pero la primera es aritmética cada término sucesivo se obtiene sumando los dos términos
con d = 2 y la segunda es geométrica con r = 2. (b) Las previos. Otra sucesión es 3, 6, 10, 15, 21, Á , en la cual la
respuestas pueden variar; por ejemplo, ambas son aritmé- regla es sumar 3 al primer término, luego sumar 4 al
ticas con d = 2, pero son diferentes en tanto que una genera segundo, luego sumar 5 al tercero y así sucesivamente. De
números pares y la otra genera números impares. (c) Las estos ejemplos, podemos ver que dos términos no bastan para
respuestas pueden variar; por ejemplo, ambas sucesiones son determinar una sucesión.
aritméticas con d = 5 en la primera sucesión y d = 50 en la
© Lopez Mateos Editores. ISBN 978-607-95583-2-1, obra completa, versión electrónica, ISBN 978-607-95583-3-8, volumen 1, versión electrónica. Ejemplar asignado a: Helecto Villarroel gutierrez -
helecto@gmail.com. Fecha: 27 de octubre de 2014. Prohibida su modificación, copia o distribución.
Respuestas a los problemas R3

Problemas de repaso 14. (a) no (b) sí (c) no 15. Si un número no es múlti-


plo de 4, entonces no es múltiplo de 8. (Contrapositiva)
17. 90 19. Necesitamos una tienda para 12 personas y una
16. (a) válida (b) válida (c) inválida 17. (a) Algunas
combinación de tiendas donde quepan 14 personas. Hay 10
estudiantes de primero son inteligentes. (b) Si estudio
maneras: 662, 653, 6332, 62222, 5522, 5333, 53222, 33332,
para el final, entonces buscaré un trabajo como
332222 y 2222222.
maestra. (c) Existen triángulos que son isósceles.
Evaluación 1-3A 18. (a) Si una figura es un cuadrado, entonces es un
1. (a) Proposición falsa. (b) Proposición falsa. (c) No es rectángulo. (b) Si un número es un entero, entonces es
proposición. (d) Proposición verdadera. (c) No es proposición. un número racional. (c) Si un polígono tiene exactamente
2. (a) Existe un número natural x tal que x + 8 = 11. tres lados, entonces es un triángulo. 19. (a) 3 # 2 Z 6 ó
(b) Existe un número natural x tal que x2 = 4. (c) Para todos 1 + 1 = 3 (b) No puedes pagarme ahora y no puedes
los números naturales x, x + 3 = 3 + x. (d) Para todos los pagarme después.
números naturales x, 5x + 4x = 9x. 3. (a) Para todo número Conexiones matemáticas 1-3
natural, x, x + 8 = 11. (b) Cada número natural x satisface
Comunicación
x2 = 4. (c) No existe número natural x tal que x + 3 =
1. Las órdenes, preguntas y opiniones no son proposiciones
3 + x. (d) No existe número natural x tal que 5x + 4x =
pues no pueden ser clasificadas como verdaderas o falsas.
9x. 4. (a) Este libro no tiene 500 páginas. (b) 3 # 5 Z 15
3. Una proposición compuesta puede estar formada por
(c) Algunos perros no tienen cuatro patas. (d) No hay
una combinación de dos o más proposiciones. Los conectivos
rectángulos que sean cuadrados. (e) Todos los rectángulos
tales como y, o, si Á entonces y no se usan para formar
son cuadrados. (f ) Algunos perros tienen pulgas.
proposiciones compuestas. 5. Dada la disyunción p o q, el
5. (a) V (b) V
uso inclusivo de “o” significa “p o q o ambos”. En lógica, usamos
6. (a) p ¬ p ¬ ( ¬ p)
el “o” inclusivo. Los abogados algunas veces usan la frase
“y/o” para aclarar el uso inclusivo del “o”. El uso excluyente
V F V
del “o” significa “ya sea p o q pero no ambos”. 7. Dr. No es
F V F un espía varón que no es pobre ni alto. 9. Las respuestas
(b) p ¬p p ¡ ¬p p ¿ ¬p pueden variar.
V F V F Aprendizaje colectivo
F V V F 11. Las respuestas pueden variar.
(c) sí (d) no 7. (a) q ¿ r (b) r ¡ ¬ q
(c) ¬ 1q ¿ r2 (d) ¬q 8. (a) falso (b) verdadero Preguntas del salón de clase
(c) falso (d) falso (e) falso 9. (a) falso (b) verdadero
(c) falso (d) falso (e) verdadero 10. (a) no (b) no 13. Cuando se escribe ¬ 1p ¿ q2, el signo de negación opera
en todo lo que está adentro del paréntesis; es decir, es la
p q ¬p ¬p ¿ q
11.
negación de la conjunción p ¿ q. Puedes hallar el valor
verdadero para p ¿ q y después negarla. Cuando escribimos
V V F F ¬ p ¿ q, el símbolo de negación opera sólo en la proposición
V F F F p y no en la conjunción.
F V V V 15. En el ejemplo,
F F V F Hipótesis: Todos los maestros tienen una altura mayor a 6 pies.
12. (a) p : q (b) ¬ p : q (c) p : ¬ q (d) p : q Kay es un maestro.
(e) ¬ q : ¬ p (f ) q 4 p 13. (a) Recíproca: Si Conclusión: Kay mide más de 6 pies de altura.
2x = 10, entonces x = 5. Inversa: Si x Z 5, entonces
2x Z 10. Contrapositiva: Si 2x Z 10, entonces x Z 5.
Personas que miden más de 6'
(b) Recíproca: Si no te gustan las matemáticas, entonces no
te gusta este libro. Inversa: Si te gusta este libro, entonces te Maestros
gustan las matemáticas. Contrapositiva: Si te gustan las Kay
matemáticas, entonces te gusta este libro. (c) Recíproca: Si
tienes caries, entonces no usas la pasta dental Ultra. Inversa:
Si usas la pasta dental Ultra, entonces no tienes caries.
Contrapositiva: Si no tienes caries, entonces usas la pasta Se puede dibujar un diagrama de Euler (Venn) para mostrar
dental Ultra. (d) Recíproca: Si tus calificaciones son altas, que todos los maestros pertenecen al conjunto de personas que
entonces eres bueno en lógica. Inversa: Si no eres bueno en miden más de 6 pies de altura. Kay pertenece al conjunto de
lógica, entonces tus calificaciones no son altas. maestros. Así, el argumento es válido aunque la hipótesis sea
Contrapositiva: Si tus calificaciones no son altas, entonces falsa.
no eres bueno en lógica.

© Lopez Mateos Editores. ISBN 978-607-95583-2-1, obra completa, versión electrónica, ISBN 978-607-95583-3-8, volumen 1, versión electrónica. Ejemplar asignado a: Helecto Villarroel gutierrez -
helecto@gmail.com. Fecha: 27 de octubre de 2014. Prohibida su modificación, copia o distribución.
R4 Respuestas a los problemas

Revisión del capítulo 28. (a) p q ¬ q ( p ¿ ¬ q) ( p ¿ q) ( p ¿ ¬ q) ¡ ( p ¿ q)

1. (a) 15, 21, 28 (b) 32, 27, 22 (c) 400, 200, 100 V V F F V V
(d) 21, 34, 55 (e) 17, 20, 23 (f ) 256, 1024, 4096 V F V V F V
(g) 16, 20, 24 (h) 125, 216, 343 2. (a) ninguna F V F F F F
(b) aritmética (c) geométrica (d) ninguna (e) aritmética F F V F F F
(b)
(f ) geométrica (g) aritmética (h) ninguna p q ¬ p ( p ¡ q) ( p ¡ q) ¿ ¬ p [( p ¡ q) ¿ ¬ p] : q
3. (a) 3n + 2 (b) n3 - 1 (c) 3n 4. (a) 1, 4, 7, 10, 13
(b) 2, 6, 12, 20, 30 (c) 3, 7, 11, 15, 19 5. (a) 10,100 V V F V F V
(b) 10,201 6. (a) F; por ejemplo, 3 + 3 = 6 y 6 no es V F F V F V
impar. (b) F; por ejemplo, 19 es impar y termina en 9. F V V V V V
(c) V; la suma de cualesquier dos números pares es par pues F F V F F V
2m + 2n = 21m + n2, donde m y n son números naturales.
29. (a) Chucho González adora a Juan Gabriel y el mole.
7. 16 3 2 13
(b) La estructura de la Estatua de la Libertad con el tiempo
5 10 11 8 se oxidará. (c) Albertina pasó el curso de matemáticas con
9 6 7 12 100. 30. Representa con las letras siguientes las proposicio -
4 15 14 1
nes dadas:
p: Tienes la piel delicada.
8. 26 9. $2.00 10. 21 postes 11. 128 partidas q: Te vas a asolear.
#
15 6 r: No vas al baile.
12. (a) 3 + 6 + 9 + 12 + 15 = ;3 + 6 + 9 + s: Tus padres quieren saber por qué no fuiste al baile.
2
18 # 7 Simbólicamente, p : q, q : r, r : s. Usando contrapositivas
12 + 15 + 18 = (b) 3 + 6 + 9 + 12 + Á +
2 tenemos: ¬ s : ¬ r, ¬ r : ¬ q, ¬ q : ¬ p. Por la regla de
3n = 3 # 1 + 3 # 2 + 3 # 3 + 3 # 4 + Á + 3n = la cadena, ¬ s : ¬ p; esto es, Si tus padres no quieren saber
n1n + 12
3 # 11 + 2 + 3 + 4 + Á + n2 = 3 a b
por qué no fuiste al baile, entonces no tienes la piel delicada.
2 31. (a) Válida, modus tollens (b) Válida, modus ponens
13n21n + 12
= , donde n es un número natural. Respuestas a Ahora intenta éste
2
13. 44,000,000 de vueltas 14. 20 estudiantes 15. 39 cajas 1-1. 11 piezas en 10 cortes; 1n + 12 piezas en n cortes
16. 48 triángulos 17. 9 h 18. 235 19. Habrá 96,000 hormi-
1-2. (a) 2500 (b) a b n 1-3. 120
a1 + an
gas en el séptimo día y 192,000 hormigas en el octavo día, de
2
modo que para entonces estará llena. 20. 4 preguntas 21. Sí;
1-4. 90 días 1-5. Las respuestas pueden variar; por ejemplo,
corta 10 cm y deja 80 cm. Luego corta 20 cm y deja 60 cm.
como cada persona debe pagar $130, Alberto puede pagar
22. 4 ó - 4 23. En la proposición (i) todos y cada uno de los
$42.50 a Beti y $40.00 a Carlos; Daniel podría pagar $70.00 a
estudiantes pasaron el examen final. En la proposición (ii) al me-
Carlos y así estarían parejos. 1-6. 23 pisos. 1-7. Las respuestas
nos un estudiante pasó el examen final y quizá todos los estudian-
pueden variar, por ejemplo, 132 ó 173
tes pasaron. 24. (a) sí (b) sií (c) no (d) sí 25. (a) Ninguna
mujer fuma. (b) 3 + 5 Z 8 (c) Algún mariachi no es ruidoso, + 932 + 873
o no todos los mariachis son ruidosos. (d) Beethoven escribió 1064 1046
alguna música que no es clásica. 26. Recíproca: Si alguno se 1-8. 83 1-9. Ale juega tenis; Beto juega beisbol; Cali
desmaya, tendremos un concierto de rock. Inverso: Si no te- juega baloncesto; Dani nada. 1-10. (a) Las respuestas
nemos un concierto de rock, entonces ninguno se desmayará. pueden variar; por ejemplo, los tres siguientes términos
Contrapositiva: Si ninguno se desmaya, entonces no podrían ser ^, ^, ~ . (b) El patrón podría ser un círculo,
tendremos un concierto de rock. dos triángulos, un círculo, dos triángulos, y así sucesivamente.
27. p 1-11. (a) Razonamiento inductivo (b) Funciona para varios
q ¬p ¬q p : ¬q q : ¬p
números. (c) Sí, si x = 11, entonces 112 + 11 + 11 no es
V V F F F F primo pues es divisible entre 11. 1-12. Como el segundo térmi-
V F F V V V no es 11, entonces 11 = a1 + d. Como el término 5 es 23, en-
F V V F V V tonces 23 = a1 + 4d. Despejando a1 e igualando las respuestas,
F F V V V V tenemos 11 - d = 23 - 4d, lo cual implica que d = 4. Para
encontrar el término 100, substituimos en a1 + 1n - 12d y el
término 100 es 7 + 1100 - 124 = 7 + 99 # 4 = 403.
Por lo tanto, p : ¬ q K q : ¬ p.

© Lopez Mateos Editores. ISBN 978-607-95583-2-1, obra completa, versión electrónica, ISBN 978-607-95583-3-8, volumen 1, versión electrónica. Ejemplar asignado a: Helecto Villarroel gutierrez -
helecto@gmail.com. Fecha: 27 de octubre de 2014. Prohibida su modificación, copia o distribución.
Respuestas a los problemas R5

1-13. (a) 4 (b) 7 (c) 12 (d) 20 (e) 33 (f ) La suma de los Si n personas están en cada lado, se necesitan 1n + 122 - 1
primeros n números de Fibonacci es uno menos que el número movimientos.
de Fibonacci que está dos lugares más adelante en la sucesión.
(g) F1 + F2 + F3 + F4 + Á + Fn = Fn + 2 - 1
1-14. (a) Después de 10 horas, hay 2 # 310 = 118,098 Sección 1-2
bacterias, y después de n horas, hay 2 # 3n bacterias. 312211; el patrón cuenta el número de veces que un número
(b) Después de 10 horas, hay 2 + 10 # 3 = 32 bacterias y aparece en la fila anterior. Por ejemplo, para hallar la sexta
después de n horas, hay 2 + n # 3 bacterias. Podemos ver que fila examinamos la quinta fila. Hay 3 unos, dos 2 y un 1, de
después de 10 horas, el crecimiento geométrico es mucho más modo que en la sexta fila están 312211. El patrón continúa
rápido que el crecimiento aritmético. En este caso, 118,098 usando esta regla.
versus 32. Esto es cierto, en general, cuando n 7 1.
1-15. (a) (b) 1 2 3 4 Respuesta a la Actividad de laboratorio
4 12 24 40 Sección 1-1
El número de movimientos es 2n - 1 con n monedas. Esto
se puede resolver usando la estrategia de resolver un proble-
ma más simple. Si hay una moneda, 1 movimiento es nece-
sario. Si hay dos monedas, 3 movimientos son necesarios.
(c) 4 12 24 40 60 84 112 Para tres monedas, el número de movimientos es 7. Para
8 12 16 20 24 28 cuatro monedas, el número de movimientos es 15.
4 4 4 4 4 A la razón de un movimiento por segundo, se llevará apro-
d) No, hallar diferencias para a100 y para an es muy difícil. ximadamente 584,942,417,418 años para mover 64 monedas.
Es más fácil hallar un patrón que incluya el número de
palillos horizontales y de palillos verticales, esto es, Respuesta al Problema preliminar
a100 = 101 # 100 + 101 # 100 = 20,200 y Es posible rotular correctamente cada plato si escoges el
an = 1n + 12n + 1n + 12n ó 231n + 12n4 ó 2n2 + 2n. plato rotulado MANZANAS Y NARANJAS. Si la fruta que
1-16. p q ¬ p ¬ q p ¡ q ¬ ( p ¡ q) ¬p ¿ ¬q tomas es una manzana, entonces debes colocar debajo del
plato el rótulo de MANZANAS. Como cada plato tiene in-
V V F F V F F correcta la etiqueta, debes mover el letrero de NARANJAS a
V F F V V F F donde está el letrero de MANZANAS y esto deja solo un
F V V F V F F plato para etiquetarlo con MANZANAS Y NARAN JAS. Si
F F V V F V V la pieza seleccionada fue naranja, entonces se puede usar un
razonamiento análogo. El letrero de NARANJAS debe ser
¬ (p ¡ q) K ¬ p ¿ ¬ q colocado en el plato y el letrero de MANZANA debe ser re-
1-17. p q p : q ¬ ( p : q) ¬ q p ¿ ¬ q movido, dejando sólo un lugar para el letrero de
MANZANAS Y NARANJAS.
V V V F F F Nota que si el plato etiquetado con MANZANAS fue
V F F V V V seleccionado y tomaste una naranja, entonces no sabrás
F V V F F F cual letrero le corresponde. Un razonamiento similar se
F F V F V F puede usar para mostrar que el plato etiquetado con NA-
RANJAS no se debe seleccionar primero.
¬ 1p : q2 K p ¿ ¬ q
1-18. p q p : q q : p ( p : q) ¿ (q : p) Capítulo 2
Evaluación 2-1A
V V V V V
V F F V F 1. (a) MCDXXIV; la doble barra sobre M representa
F V V F F 1000 # 1000 # 1000. (b) 46,032; el 4 en 46,032 representa
F F V V V 40,000 mientras que el 4 en 4632 representa sólo 4000.
(c) : el espacio en el último número indica que se mul-
Respuesta a los Rompecabezas tiplica por 60. (d) ; la representa 1000, mientras que
Sección 1-1 representa sólo 100. (e) representa tres grupos de 20 más
cero 1 y representa tres 5 y tres 1. 2. (a) MCML;
35 movimientos. Esto se puede resolver usando la estrategia MCMXLVIII (b) (c) ;
de examinar casos más sencillos y buscar un patrón. Si una
(d) ; 3. 1922 4. (a) CXXI (b) XLII
persona está en cada lado, se necesitan 3 movimientos. Si
dos personas están en cada lado, se necesitan 8 movimientos. 5. (a) ; ; LXXII; (b) 602; ; DCII;
Con 3 personas en cada lado, se necesitan 15 movimientos.

© Lopez Mateos Editores. ISBN 978-607-95583-2-1, obra completa, versión electrónica, ISBN 978-607-95583-3-8, volumen 1, versión electrónica. Ejemplar asignado a: Helecto Villarroel gutierrez -
helecto@gmail.com. Fecha: 27 de octubre de 2014. Prohibida su modificación, copia o distribución.
R6 Respuestas a los problemas

Se ha propuesto, junto con el sistema métrico, quitar las co-


(c) 1223; ; MCCXXIII; 6. (a) Cientos mas y simplemente usar espacios. (b) Las respuestas varían.
(b) Dieces 7. (a) 3,004,005 (b) 20,001 8. 811 ó
910 9. (a) 86 (b) 11 10. 2112cuatro 11. (a) 11, 10, Solución abierta
11, 100, 101, 110, 111, 1000, 1001, 1010, 1011, 1100, 1101, 5. 4; 1, 2, 4, 8; 1, 2, 4, 8, 16
1110, 11112dos (b) 11, 2, 3, 10, 11, 12, 13, 20, 21, 22, 23,
30, 31, 32, 332cuatro 12. 20 13. 2032cuatro = 2 # 43 + Preguntas del salón de clase
0 # 42 + 3 # 41 + 2 # 1 14. (a) 111dos (b) OOOdoce 7. Las respuestas pueden variar; por ejemplo, en base dos
15. (a) ODOdoce; OO1doce (b) 11111dos; 100001dos tenemos dos dígitos, en base cinco tenemos cinco dígitos y
(c) 554seis; 1000seis 16. (a) No hay numeral 4 en la base en base diez tenemos diez dígitos. Podría definirse una base
cuatro. (b) No hay numerales 6 ó 7 en la base cinco. negativa. 9. Es correcta. Sin embargo, los romanos usual-
17. 3 bloques, 1 losa, 1 barra, 2 unidades mente reservaban la barra para los números mayores que
18. 4000. Pues M es el símbolo especial para el número 1000, es
preferible escribir MI para 1001 en vez de II.
Evaluación 2-2A

1. (a) 5m, a, t, e, i, c, s6 (b) 5x ƒ x es un número natural


19. (a) 8 centavos pueden ser cambiados por 1 moneda de
donde x 7 206 ó 521, 22, 23, Á 6 2. (a) P = 5a, b, c, d6
cinco y 3 de centavo. Después del cambio, tenemos 2 monedas
(b) 51, 26 ( 51, 2, 3, 46(c) 50, 16 h 51, 2, 3, 46
de veinticinco, 10 de cinco y 3 de centavo. 10 monedas de cinco
(d) 0 僆 5 6 ó 0 僆 ⭋ 3. (a) Sí (b) Sí (c) No
se pueden cambiar por 2 de veinticinco. Después de este cambio
tenemos 4 monedas de veinticinco, 0 de cinco y 3 de centavo.
(b) Supón que tienes 73 centavos en cualquier combinación 4. (a) 720 (b) n1n - 121n - 22 # Á # 3 # 2 # 1
posible, por ejemplo, 10 monedas de cinco y 23 de un centavo. 5. (a) 24 (b) 6 (c) 12 6. A = C, E = H, I = J
Como 23 centavos se pueden cambiar por 4 monedas de cinco y 7. (a) 1100 - 100, ó 1000 con la aritmética (b) 501
3 de centavo tenemos 14 de cinco y 3 de centavo. 14 de cinco se (c) 11 (d) 100 (e) 5 8. A es el conjunto de todos los es-
pueden cambiar por dos de veinticinco y 4 de cinco. Después del tudiantes de secundaria que al menos tienen una calificación
segundo cambio, debemos tener 2 monedas de veinticinco, 4 de diferente de diez; es decir, es el conjunto de estudiantes cuyo
cinco y 3 de centavo. Obtenemos: 73 = 243cinco. promedio no es diez cerrado. 9. (a) 7 (b) 0
20. (a) 10 losa = 1 bloque; 10 losas en base diez = 1000 10. (a) n1D2 = 5 (b) C = D 11. (a) 僆 (b) 僆
(b) 20 losas = 1 bloque + 8 losas; 20 losas en base doce = (c) 僆 (d) 僆 12. (a) h (b) 8 (c) h (d) h
13. (a) Sí (b) No. A puede ser igual a B.
(c) Sí (d) No. Considera A = 516 y B = 51, 26.
1800doce 21. Los métodos varían. 100010dos
14. (a) Sean A = 51, 2, 36 y B = 51, 2, 3, 4, Á , 1006.
22. (a) 117 (b) 45 (c) 1331 23. 1 premio de $625,
2 premios de $125 y 1 de $25 24. (a) 8 semanas, 2
Como A ( B, n1A2 es menor que n1B2. Así, 3 6 100.
(b) Sean A = ⭋ y B = 51, 2, 36. Como A ( B, n1A2 = 0 es
días (b) 1 día, 5 horas 25. (a) 6 (b) 1 26. Sobre la
barra están 5, 50, 500 y 5000. Debajo de la barra están las
unidades, decenas, centenas y unidades de millar. Así se pre- menor que n1B2 = 3, así que 0 6 3. 15. 35 16. 81
sentan 1 # 5000, 1 # 500, 3 # 100, 1 # 50, 1 # 5 y 2 # 1 para un to-
Conexiones matemáticas 2-2
tal de 5857. El número 4869 se podría representar como
sigue: Comunicación
1. Un conjunto está bien definido cuando dado cualquier objeto
se puede decidir si pertenece o no al conjunto. Por ejemplo, el
conjunto de presidentes de Estados Unidos está bien definido
pero el conjunto de presidentes adinerados de Estados Unidos
27. Supón que tienes una pantalla de ocho dígitos sin nota- no está bien definido pues “adinerado” es una cuestión de
ción científica. 98,765,432 28. (a) Las respuestas varían; opinión. 3. Sí, ⭋ ( A para todos los conjuntos A ya que A con-
por ejemplo, resta 2020. (b) Las respuestas varían; por tiene al menos un elemento y ⭋ no contiene ninguno; también
ejemplo, resta 50. ⭋ 8 A. 5. Para mostrar que A h B, debemos poder encontrar
al menos un elemento del conjunto A que no pertenezca al con-
Conexiones matemáticas 2-1 junto B. 7. Si A y B son subconjuntos finitos, decimos que
n1A2 … n1B2 en el caso de que A sea un subconjunto (no nece-
Comunicación
sariamente propio) de B.
1. Las respuestas pueden variar. Benjamín no tiene razón. Solución abierta
El cero representa un lugar vacío en el sistema indoarábigo.
Se usa para diferenciar entre números como 54 y 504. Si el 9. (a) Sea A el conjunto de todos los números naturales diferen-
cero fuera nada, entonces lo podríamos eliminar sin cambiar tes del 1 con N como el conjunto universal. Entonces A = 516
nuestro sistema numérico. 3. (a) Esto es principalmente es finito. (b) Las respuestas varían. Sea A el conjunto de núme-
para facilitar la lectura con bloques agrupados por miles y ros naturales pares con N como conjunto universal. A es el con-
nombrarlos. junto de números impares y por lo tanto es infinito.
© Lopez Mateos Editores. ISBN 978-607-95583-2-1, obra completa, versión electrónica, ISBN 978-607-95583-3-8, volumen 1, versión electrónica. Ejemplar asignado a: Helecto Villarroel gutierrez -
helecto@gmail.com. Fecha: 27 de octubre de 2014. Prohibida su modificación, copia o distribución.
Respuestas a los problemas R7

Aprendizaje colectivo (c) U (d) U (e) U

64 19
11. (a) Hay 2 L 1.84 * 10 subconjuntos de A B A B A B

51, 2, 3, Á , 646. Si una computadora puede listar un C C C

subconjunto en aproximadamente 1 microsegundo (una mi-


llonésima de segundo), entonces tardará 6. (a) S ´ S = U (b) U = ⭋ (c) S ¨ S = ⭋
1 año (d) ⭋ ¨ S = ⭋ 7. (a) A - B = A (b) A - B = ⭋
1.84 * 1019 * 0.000001 s * L 580,000 años 8. Sí. Por definición, A - B es el conjunto de todos los ele-
31,536,000 s
en listar todos los subconjuntos. (b) Hay mentos que están en A y que no están en B. Si A - B es el
64 # 63 # 62 # Á # 2 # 1 L 1.27 * 1089 correspondencias uno conjunto vacío, entonces esto quiere decir que no hay
a uno entre los dos conjuntos. Así que tomará aproximada- elementos en A que no sean elementos en B, lo cual hace
mente que A sea un subconjunto de B. 9. Las respuestas pueden
1 año variar. (a) B - A (b) A ´ B (c) (A ¨ B) - C
1.27 * 1089 * 0.000001 s * L 4 * 1075 años 10. U
31,536,000 s
listar todas las correspondencias uno a uno entre los conjuntos. A B

Preguntas del salón de clase


13. Una manera de indicar el conjunto vacío es 5 6. 11. (a) Falsa U U

Cualquier cosa que encerremos dentro de las llaves es un


elemento del conjunto. Así 5⭋6 es un conjunto con un solo
A B A B
Þ
C C

elemento, el símbolo del conjunto vacío; así no es vacío. La


A < (B ù C) (A < B) ù C
dificultad aparece frecuentemente de la reticencia a conside-
(b) Falsa U U
rar al conjunto vacío como un elemento. 15. El conjunto
A = 51, 5166 tiene dos elementos, 1 y 516.
A B A B
Þ
C C

Problemas de repaso
12. (a) 1A ¨ B ¨ C2 8 1A ¨ B2 (b) 1A ´ B2 8
A 2 (B 2 C) (A 2 B) 2 C
17. Las respuestas pueden variar. El sistema métrico está ba-
sado en potencias de 10. Las longitudes más comunes y sus 1A ´ B ´ C2 13. (a) (i) 5 (ii) 2 (iii) 2 (iv) 3 (b) (i)
conversiones se dan a continuación. n + m (ii) El menor de los dos números entre m y n (iii) m
10 mm (milímetros) = 1 cm (centímetro) (iv) n 14. (a) El mayor es 15; el menor es 6. (b) El mayor
10 cm = 1 dm (decímetro) es 4; el menor es 0. 15. (a) El conjunto de jugadores de ba-
10 dm = 1 m (metro) loncesto universitario que miden más de 200 cm de
10 m = 1 dam (decámetro) altura (b) El conjunto de seres humanos que no son
10 dam = 1 hm (hectómetro) estudiantes universitarios o estudiantes universitarios que mi-
10 hm = 1 km (kilómetro) den menos o igual que 200 cm de altura (c) El conjunto de
El esquema de conversiones trabaja más como conversiones seres humanos que son jugadores de baloncesto universitario o
en base diez. 19. 1410 21. (a) alrededor de 121 semanas que son estudiantes universitarios que miden más de 200 cm
(b) Aproximadamente 3 años (c) Las respuestas pueden va- de altura (d) El conjunto de seres humanos que no son juga-
riar. (d) Las respuestas pueden variar. dores universitarios de baloncesto y que no son estudiantes
universitarios que miden más de 200 cm (e) El conjunto de
todos los estudiantes universitarios que miden más 200 cm y que
Evaluación 2-3A no son jugadores de baloncesto (f ) El conjunto de todos los
1. (a) A ó C (b) N (c) ⭋ 2. (a) Sí (b) Sí jugadores universitarios de baloncesto que miden menos o
(c) Sí (d) Sí 3. (a) Verdadero (b) Falso. Sean igual a 200 cm de altura. 16. 18 17. 4 18. (a) 20 (b) 10
A = 5a, b, c6 y B = 5a, b6. Entonces A - B = 5c6, pero 18. (a) 20 (b) 10 (c) 10 19. 3. Usando el siguiente diagrama
B - A = ⭋. (c) Falso. Sean U = 5a, b, c6, de Venn y el hecho de que el conjunto de personas que son O
A = 5a6 y B = 5b6. Entonces A ¨ B = ⭋ y
negativo es 100 - n1A ´ B ´ C2, vemos que la respuesta es 3.
A ¨ B = U. A = 5b, c6; B = 5a, c6, y A ¨ B = {c}.
U

A ¨ B Z A ¨ B. (d) Falso. Sean A = 5a, b6; B = 5b6.


A B
8 1 6
4

A ´ B = 5a, b6; (A ´ B) - A = ⭋ Z B. (e) Falso. Sean


27 7 3
44

A = 51, 2, 36, B = 53, 4, 56. Entonces (A - B) ´ A =


Rh

20. (a) Falso. Sean A = 5a, b, c6 y B = 51, 2, 36. (b) Falso.


51, 2, 36, pero 1A - B2 ´ 1B - A2 = 51, 26 ´ 54, 56 =
Sean A = 51, 2, 36 y B = 51, 2, 3, 46. (c) Verdadero. 21.
51, 2, 4, 56. 4. (a) A ¨ B = B (b) A ´ B = A
Guadalajara contra Santos, unam contra Pachuca, Atlas contra
5. (a) U (b) U
América, Cruz Azul contra Necaxa. 22. (a) A * B = 51x, a2,
A B A B
1x, b2, 1x, c2, 1y, a2, 1y, b2, 1y, c26 (b) B * A = 51a, x2, 1a, y2,
1b, x2, 1b, y2, 1c, x2, 1c, y26 23. (a) C = 5a6, D = 5b, c, d, e6
(b) C = 51, 26, D = 51, 2, 36 (c) C = D = 50, 16
C C

© Lopez Mateos Editores. ISBN 978-607-95583-2-1, obra completa, versión electrónica, ISBN 978-607-95583-3-8, volumen 1, versión electrónica. Ejemplar asignado a: Helecto Villarroel gutierrez -
helecto@gmail.com. Fecha: 27 de octubre de 2014. Prohibida su modificación, copia o distribución.
R8 Respuestas a los problemas

Conexiones matemáticas 2-3 3. (a) CMXCIX (b) (c) (d) 2341cinco


Comunicación (e) 11011dos 4. (a) 317 (b) 221 5. 2020tres
1. (a) Sí. 1A ¨ B2 8 1 A ´ B2 (b) No. Por ejemplo, sean 6. 1 bloque, 2 losas, 2 barras, 0 unidades
A = 51, 2, 36 y B = 546. Entonces 2 僆 A ´ B, pero 7. (a) (b)
2 僆 A ¨ B. 3. No. Sean A = 516 y B = 5a6. Entonces
A * B = 511, a26, pero B * A = 51a, 1)6. Éstos no son
iguales. 8. (a) 1010 (b) 58 (c) 231 9. (a) 10,000,000,023
(b) 10,000,000,001dos (c) 10,000,000,001cinco
Solución abierta
(d) 9,999,999,999 (e) 1,111,111,111dos (f ) OOOOOdoce
5. Las respuestas varían. 10. Las respuestas pueden variar. Vender lápices por unidades,
Aprendizaje colectivo docenas y gruesas es un ejemplo del uso de la base 12. 11. (a) El
sistema egipcio tenía siete símbolos. Era un sistema de muescas y de
7. Las respuestas varían. agrupación, y usaba la propiedad aditiva. No tenía un símbolo para el
Preguntas del salón de clase cero, pero no era muy importante pues no usaba el valor posicio-
nal. (b) El sistema babilonio sólo usó dos símbolos. Fue un sis-
9. El estudiante tiene razón. Si A = {1, 2}, B = {2, 3} tema con valor posicional (base 60) y fue aditivo sin las posiciones.
y C = {2, 4}, A ¨ B = A ¨ C pero B Z C. También, para Faltó un símbolo para el cero hasta alrededor de 300 A.C. (c) El
mostrar que la hipótesis implica que B = C, demostramos que sistema de numeración romano usó siete símbolos. Fue aditivo,
B 8 C y C 8 B. Para mostrar que B 8 C, sea x 僆 B, entonces substractivo y multiplicativo; no tuvo el símbolo para el 0. (d) El
x 僆 A ´ B y como A ´ B = A ´ C, x 僆 A ´ C. Por lo tanto, sistema indoarábigo usa 10 símbolos. Tiene valor posicional y un
x 僆 A o x 僆 C. If x 僆 C, entonces B 8 C. Si x 僆 A, entonces símbolo para el 0. 12. (a) 1003cinco (b) 10000000dos
como empezamos con x 僆 B, se sigue que x 僆 A ¨ B. Como (c) D8doce 13. (a) 4210014cinco (b) 10000001000dos
A ¨ B = A ¨ C, concluimos que x 僆 A ¨ C y por lo tanto (c) O0D018doce (d) 1100010ocho 14. ⭋, 5m6, 5a6,
x 僆 C. Así, B 8 C. Análogamente, empezando con x 僆 C, se 5t6, 5h6, 5m, a6, 5m, t6, 5m, h6, 5a, t6,5a, h6, 5t, h6, 5m, a, t6,
puede demostrar que x 僆 B y por lo tanto que C 8 B. 5m, a, h6, 5m, t, h6, 5a, t, h6, 5m,a,t,b6 15. (a) A ´ B = A
11. Aunque el producto cartesiano de conjuntos incluye todas (b) C ¨ D = 5l, e6 (c) D = 5u, n, i, v, r6
las parejas en las cuales cada elemento del primer conjunto es (d) A ¨ D = 5r, v6 (e) B ´ C = 5s, v, u6
la primera componente de una pareja con cada elemento del (f ) 1B ´ C2 ¨ D = 5l, e, a6 (g) 5i, n6 (h) 5e6
segundo conjunto, esto no necesariamente es una correspon- (i) 5 (j) 16
dencia uno a uno. Una correspondencia uno a uno implica 16.
que hay la misma cantidad de elementos en cada conjunto. U U

Esto no sucede en el producto cartesiano. Por ejemplo, consi-


dera los conjuntos A = 516 y B = 5a, b6.
A B A B

C C

Problemas de repaso
(a) A ¨ 1B ´ C2 (b) 1A ´ B2 ¨ C
13. El número “dos” existe en base dos pero no hay un solo
dígito que represente el “dos”. 15. (a) 5x ƒ 3 6 x 6 10 17. 5040 18. (a) Las respuestas pueden variar. (b) 6
donde x 僆 N6 (b) 515, 30, 456 17. (a) Éstos son todos t4e
los subconjuntos de 52, 3, 46. Hay 23 = 8 de dichos subcon- h4n
juntos. (b) Hay 8 subconjuntos que contienen el número 1. e4d
(c) Doce subconjuntos contienen 1 ó 2 (o ambos). Hay 4 19. No es cierto que A ¨ 1B ´ C2 = 1A ¨ B2 ´ C para
subconjuntos de 53, 46. Podemos formar subconjuntos que toda A, B y C, como se muestra en los siguientes diagramas.
contengan el 1 o el 2 o ambos añadiendo 1 a cada uno, 2 a U U

cada uno, o 1 y 2 a cada uno. Por el Principio Fundamental A B


Z A B

del Conteo, entonces hay 3 # 4 = 12 posibilidades. (No es di-


C C
fícil listarlos). (d) Cuatro subconjuntos no contienen el 1
ni el 2. (e) 16; B tiene 25 = 32 subconjuntos. La mitad A ¨ 1B ´ C2 1A ¨ B2 ´ C
contiene el 5 y la mitad no. (f ) Todo subconjunto de A es
un subconjunto de B. Los otros se pueden listar añadiendo 20. U (a) 36 (b) 6 (c) 5
H 2 E
el elemento 5 a cada subconjunto de A. Así hay el doble de 7
4
3
5
9

subconjuntos de B que de subconjuntos de A. A tiene 16 6


M

21. Las respuestas pueden variar. (a) B ´ 1C ¨ A2 (b) B - C


subconjuntos y B tiene 32 subconjuntos. 19. Las respuestas
22. (a) Falso. Considera los conjuntos 5a6 y 526. (b) Falso.
pueden variar. 21. 60
No es un subconjunto propio de sí mismo. (c) Falso. Considera
los conjuntos 5t, h, e6 y 5e, n, d6. Ellos tienen el mismo número
Revisión del capítulo

1. (a) dieces (b) miles (c) cientos 2. (a) 400,044 de elementos, pero no son iguales. (d) Falso. Esto es una corres-
(b) 117 (c) 1704 (d) 11 (e) 1448 pondencia uno a uno con el conjunto de los números naturales.
© Lopez Mateos Editores. ISBN 978-607-95583-2-1, obra completa, versión electrónica, ISBN 978-607-95583-3-8, volumen 1, versión electrónica. Ejemplar asignado a: Helecto Villarroel gutierrez -
helecto@gmail.com. Fecha: 27 de octubre de 2014. Prohibida su modificación, copia o distribución.
Respuestas a los problemas R9

(e) Falso. El conjunto 55, 10, 15, 20, Á 6 es un subconjunto 1 4 A


propio del conjunto de números naturales y es equivalente al con- 2 4 B como 1 2 3 4
junto de los números naturales, pues hay una correspondencia 3 4 C A B C D
uno a uno entre los conjuntos. (f ) Falso. Sean B = 51, 2, 36 y A 4 4 D
el conjunto de números naturales. (g) Verdadero (h) Falso. Las 24 correspondencias biunívocas son
Sean A = 51, 2, 36y B = 5a, b, c6. 23. (a) Como A ´ B es la 1 2 3 4 1 2 3 4 1 2 3 4 1 2 3 4
unión de conjuntos ajenos, A - B, B - A y A ¨ B, la ecuación A B C D B A C D C A B D D A B C
es verdadera. (b) Verdadera, pues A - B y B son ajenos así como A B D C B A D C C A D B D A C B
B - A y A; también, A ´ B = 1A - B2 ´ B = 1B - A2 ´ A. A C B D B C A D C B A D D B A C
24. (a) 17 (b) 34 (c) 0 (d) 17 25. 7 26. Las respuestas A C D B B C D A C B D A D B C A
pueden variar. La primera pregunta puede ser “¿Es un estado de A D B C B D A C C D A B D C A B
la República mexicana?” Si la respuesta es sí, La segunda pregunta A D C B B D C A C D B A D C B A
puede ser “¿Comienza con una vocal?” La tercera pregunta puede (c) Notamos que 24 = 4 # 3 # 2 = 4 # 3 # 2 # 1. También nota-
ser “¿Es ___?” Si la respuesta a la primera pregunta es no, la mos que tenemos cuatro maneras de colocar a las personas
segunda pregunta puede ser “¿Es la capital de un estado?” y la ter- en el carril 1. Después de escoger una, vemos que tenemos
cera pregunta puede ser como la anterior. 27. (a) Sean tres maneras de colocar a las personas en el carril 2, lo cual
A = 51, 2, 3, Á , 136 y B = 51, 2, 36; entonces B es un deja sólo dos maneras de escoger a la persona para el carril 3
subconjunto propio de A. Por lo tanto, B tiene menos elementos y, finalmente, una persona para el carril 4. Extrapolando
que A y así n1B2 es menor que n1A2; así, 3 6 13. (b) Sean todo esto, emitimos la conjetura de que hay
A = 51, 2, 3, Á , 126 y B = 51, 2, 3, Á , 96. B es un subcon- 5 # 4 # 3 # 2 # 1 = 120
junto propio de A, así que A tiene más elementos que B; por lo distintas correspondencias biunívocas entre un par de conjuntos
tanto, n1A2 es mayor que n1B2, 12 7 9. 28. 12 maneras de cinco elementos. 2-6. Si el evento M1 puede ocurrir de m1
maneras y si, después de ocurrido, el evento M2 puede ocurrir de
Respuestas a Ahora intenta éste m2 maneras y, después de ocurrido, el evento M3 puede ocurrir
de m3 maneras y así sucesivamente, donde los eventos M1, M2,
2-1. 3 bloques 12 losas 11 barras 17 unidades M3, Á , Mn pueden ocurrir, respectivamente, de m1, m2, m3,
= 3 bloques (1 bloque 2 losas) (10 barras 1 barra) Á , mn maneras, entonces el evento M1 seguido del evento M2
(10 unidades 7 unidades) seguido del evento M3, Á seguido del evento Mn, puede ocurrir
= 4 bloques 2 losas (1 losa 1 barra) (1 barra 7 unidades)
de m1 # m2 # m3 # Á # mn maneras.
= 4 bloques 3 losas 2 barras 7 unidades
2-7. (a) No, dos conjuntos pueden ser equivalentes sin ser
= 4327
iguales. Para verlo, considera el ejemplo siguiente:
A = 5a, b, c6
2-2. (a) (b) 203,034
B = 51, 2, 36
(c) Las respuestas pueden variar; por ejemplo, escribir nú-
meros grandes es molesto cuando el sistema es aditivo y no
Entonces,
usa valor posicional. Es difícil efectuar operaciones que in- a41
cluyan sumas, restas, multiplicaciones y divisiones debido a b42
la manera en que están representados los números. c43
2-3. (a) (b) 2 # 602 + 11 # 60 + 1 = es una correspondencia biunívoca entre A y B, y, por lo tanto,
7861. (c) Las respuestas pueden variar. El sistema indoará- A ' B. Sin embargo, A Z B. (b) Sí, si dos conjuntos son
bigo tiene un símbolo para el 0, lo cual es muy importante en iguales, entonces cada elemento se puede poner en correspon-
un sistema que use valor posicional. Como usa base sesenta, el dencia con él mismo, mostrando que son equivalentes. 2-8. El
sistema babilonio requiere el uso de varios símbolos para conjunto de los números naturales es N = 51, 2, 3, 4, 5, Á 6.
escribir números como 59. Si N fuera finito, entonces habría un elemento máximo q. Sin
2-4. (a) La ilustración indica divisiones sucesivas entre 5. embargo, q + 1 es un número natural mayor que q y segui-
Esto muestra que hay 164 cincos en 824 con residuo 4. A con- ría estando en N. Así, no hay un mayor elemento en N y no
tinuación vemos que hay 32 cincos en 164 con residuo 4. Este puede ser finito. 2-9. (a) Sí, por definición, A 8 B significa
proceso continúa hasta ver que hay un número 625 en 824 que cada elemento de A es un elemento de B. De manera aná-
con un 125, dos 25, cuatro 5 y 4 unidades. loga, A ( B significa que cada elemento de A es elemento de
(b) 5:728 B pero existe algún elemento de B que no es elemento de A.
5 :145 3 De aquí que, si A ( B, entonces es cierto que cada elemento
5 :29 0 de A está en B. En consecuencia, A 8 B. Nota que si la con-
5 :5 4 dición más fuerte A ( B se satisface, entonces la condición
1: 0 más débil A 8 B también debe satisfacerse. (b) No, para
Así, la respuesta es 10403cinco. 2-5. (a) y (b): Numera los verlo exhibimos el siguiente contraejemplo:
carriles como 1, 2, 3, 4 y nombra a las personas como A, B, A = 5a, b, c6
C, D. Luego representamos la correspondencia: B = 5a, b, c6
Entonces, A 8 B. Nota que A X B pues A = B.
© Lopez Mateos Editores. ISBN 978-607-95583-2-1, obra completa, versión electrónica, ISBN 978-607-95583-3-8, volumen 1, versión electrónica. Ejemplar asignado a: Helecto Villarroel gutierrez -
helecto@gmail.com. Fecha: 27 de octubre de 2014. Prohibida su modificación, copia o distribución.
R10 Respuestas a los problemas

2-10. (a) Sean dos representaciones del conjunto vacío ⭋ y U U

5 6. Supongamos que ⭋ h 5 6. Entonces existe un elemento


B C A B

de ⭋ que no está en 5 6. Esto no puede suceder, por lo tanto,


A A (B C) A (A B) C

⭋ 8 5 6. (b) Otra vez usa las dos representaciones, ⭋ y 5 6.


Del inciso (a) sabemos que ⭋ 8 5 6. Supón ⭋ ( 5 6. Si ⭋ es un
B C B C

subconjunto propio de5 6, entonces existe algún elemento en


5 6 que no está en ⭋. Como no existe tal elemento, así ⭋ X 5 6. De manera análoga, siempre es cierto que A ´ 1B ´ C2 =
2-11. (a) Suponiendo que una mayoría simple conforma una 1A ´ B2 ´ C. Los siguientes diagramas de Venn justifican ésta
coalición ganadora, vemos que cualquier subconjunto formado proposición.
de tres o más senadores es una coalición ganadora. Hay 16 de
dichos subconjuntos. Para verlo, sea 5A, B, C, D, E6 el conjunto
U U
B C A B

de los cinco senadores en el comité. Entonces las siguientes son A C A (B C)


está representada
A (A B) C
está representada
todas las posibles coaliciones ganadoras: por las regiones por las regiones

5A, B, C6 5A, B, D6 5A, B, E6 5A, C, D6


que contienen que contienen
B segmentos B C segmentos

5A, C, E6 5A, D, E6 5B, C, D6 5B, C, E6


5B, D, E6 5C, D, E6 5A, B, C, D6 5A, B, C, E6 Sin embargo, en general, A - 1B - C2 Z 1A - B2 - C. Para
5A, B, D, E6 5A, C, D, E6 5B, C, D, E6 5A, B, C, D, E6 verlo, consideren el siguiente contraejemplo:
De la lista vemos que hay cinco subconjuntos que contienen A = 51, 2, 3, 4, 56
exactamente cuatro miembros. También vemos que hay cinco B = 51, 2, 36
senadores en el comité. Para comprender por qué estos números C = 53, 46
Entonces, A - 1B - C2 = A - 51, 26 = 53, 4, 56, pero
son iguales, nota que crear un subconjunto de cuatro elementos
1A - B2 - C = 54, 56 - C = 556. Así, por la elección de A,
es equivalente a quitar un solo elemento del conjunto total. Esto
B y C, tenemos A - 1B - C2 Z 1A - B2 - C.
es, podemos dar una correspondencia biunívoca entre el
2-15. El siguiente diagrama de Venn muestra A ´ 1B ¨ C2 =
conjunto de los subconjuntos de cuatro elementos de
5A, B, C, D, E6 y el conjunto de los senadores asociando a cada
uno de los subconjuntos de cuatro elementos con el senador que 1A ´ B2 ¨ 1A ´ C2.
no está en ese subconjunto, según se muestra: U U

5A, B, C, D6 4 E
B C A B

5A, B, C, E6 4 D
A A A A C
A (B C) está (A B) (A C)

5A, B, D, E6 4 C representada por las

5A, C, D, E6 4 B
B C regiones que B C
contienen algún

5B, C, D, E6 4 A segmento

(b) Podemos dar una correspondencia biunívoca entre los Lo anterior debería llamarse propiedad distributiva de la unión
subconjuntos de tres elementos y los subconjuntos de dos elemen- sobre la intersección.
tos de 5A, B, C, D, E6 al asociar a cada conjunto de tres elementos 2-16. Tiene niños en la familia Tiene niñas en la familia

con el único subconjunto de dos elementos que contiene a los se-


nadores de comité que no están en el conjunto; por ejemplo, Cali

5A, B, C6 4 5D, E6. De la parte (a), sabemos que hay exactamen-


Elsa Ana
Beto
Hugo Diana
Félix
Iván Gloria

te 10 subconjuntos de tres elementos del comité; por lo tanto, Jorge

debe haber 10 subconjuntos del comité con dos elementos.


2-12. (a) 15 (b) 2n - 1
2-13. La fórmula es n1A ´ B2 = n1A2 + n1B2 - n1A ¨ B2.
Respuesta al Rompecabezas
Para justificarla, nota que en n1A ´ B2, los elementos de A ¨ B
se cuentan una sola vez. En n1A2 + n1B2, los elementos de Sección 2-2
A ¨ B se cuentan dos veces, una vez en A y otra en B. Así, substra-
er n1A ¨ B2 de n1A2 + n1B2 hace que el número sea igual a Considere el conjunto de estudiantes del maestro González.
n1A ´ B2. Por ejemplo, si A = 5a, b, c6 y B = 5c, d6, entonces Este conjunto tiene 224 - 1 subconjuntos no vacíos. El
A ´ B = 5a, b, c, d6 y n1A ´ B2 = 4. Sin embargo, n1A2 + conjunto de estudiantes de la maestra Salas tiene 225 - 1
n1B2 = 3 + 2 = 5 pues c es contado dos veces. Como A ¨ B = subconjuntos no vacíos. Usando el principio fundamental de
5c6, n1A ¨ B2 = 1 y n1A2 + n1B2 - n1A ¨ B2 = 4. conteo encontramos que 1224 - 12 # 1225 - 12, o aproxima-
2-14. Es siempre cierto que A ¨ 1B ¨ C2 = 1A ¨ B2 ¨ C. En damente 563 trillones de comités escolares se pueden formar
la siguiente figura vemos diagramas de Venn para cada lado de la conteniendo al menos un estudiante de cada clase. Este nú-
ecuación, como los diagramas de Venn describen el mismo con- mero es mayor que la población del mundo, la cual es de
junto, la ecuación siempre es verdadera. aproximadamente 6 mil millones. Por lo tanto, Linda tiene
razón.

© Lopez Mateos Editores. ISBN 978-607-95583-2-1, obra completa, versión electrónica, ISBN 978-607-95583-3-8, volumen 1, versión electrónica. Ejemplar asignado a: Helecto Villarroel gutierrez -
helecto@gmail.com. Fecha: 27 de octubre de 2014. Prohibida su modificación, copia o distribución.
Respuestas a los problemas R11

Respuesta a la actividad de laboratorio Capítulo 3


Sección 2-3
Evaluación 3-1A
1. A ´ B es el conjunto de bloques que son verdes, grandes,
o ambos verdes y grandes. B ´ A es el conjunto de todos los 1. Por ejemplo, sean A = 51, 26, B = 52, 36; entonces
bloques que son grandes, verdes, o ambos grandes y verdes. A ´ B = 51, 2, 36. Así, n1A2 = 2, n1B2 = 2, n1A ´ B2 = 3,
Estos conjuntos son iguales. pero n1A2 + n1B2 = 2 + 2 = 4 Z n1A ´ B2. 2. (a) ver-
2. A ¨ B es el conjunto de bloques que no son grandes y dadero (b) falso (c) verdadero 3. n1A ¨ B2 = 2
verdes. A ¨ B es el conjunto de los bloques pequeños que no 4. (a) 3, 4, 5, 6 (b) 3 5. (a) sí (b) sí (c) sí (d) No.
son verdes. Estos conjuntos no son iguales. 3 + 5 僆 V. (e) sí 6. (a) propiedad conmutativa de la suma
3. A ¨ B es el conjunto de bloques que no son verdes y (b) propiedad asociativa de la suma (c) propiedad conmu-
grandes. A ´ B es el conjunto de bloques que no son verdes o tativa de la suma (d) propiedad de la identidad aditiva (e)
no son grandes. Estos conjuntos son iguales. propiedad conmutativa de la suma (f ) propiedad asociativa
4. A - B es el conjunto de bloques verdes que no son gran- de la suma 7. No. Si k = 0, tenemos k = 0 + k, lo que
des. A ¨ B es el conjunto de bloques que son verdes y no son implica que k 6 k, lo cual es falso. 8. (a) (i) Para cua-
grandes. Estos conjuntos son iguales. lesquier números completos a y b, a 6 b si, y sólo si, existe
un número natural k tal que b - k = a, o equivalentemente
Respuesta al Problema preliminar si, y sólo si b - a es un número natural. (ii) Para cuales-
El enfoque del problema puede variar. Supongamos que A es quier números completos a y b, a 7 b si, y sólo si, existe un
el conjunto de todos los adultos, F el conjunto de todas las número natural k tal que a - k = b, o equivalentemente
mujeres y M el conjunto de todos los residentes de Missis- a - b es un número natural. (b) a Ú b si, y sólo si, a - b
sippi. El diagrama siguiente muestra las incógnitas de los nú- es un número entero. 9. (a) 33, 38, 43 (b) 56, 49, 42
meros de elementos (personas) en cada región. 10. (a) 9 (b) 8 (c) 3 (d) 6 u 8 (e) 5 (f ) 4 u 8
A F (g) 9 11. 0 12. (a) 8 1 6 (b) 17 10 15
c d e 3 5 7 12 14 16
a b f 13. (a) Queta es la más baja 4 9 2 13 18 11
g h
M
y Vera es la más alta. (b) Queta, 140 cm; Miriam, 142 cm;
Con esta clasificación, tenemos lo siguiente: Sandra, 148 cm; Vera, 152 cm 14. (a) 9 = 7 + x, ó
a – Hombres de Mississippi 9 = x + 7 (b) x = 6 + 3 ó x = 3 + 6 (c) 9 =
b – Mujeres de Mississippi x + 2ó9 = 2 + x 15. (a) 8 + 3 = 11, 3 + 8 = 11,
c – Hombres de Tennessee 11 - 3 = 8, 11 - 8 = 3 (b) 13 - 8 = 5, 13 - 5 = 8,
d – Mujeres de Tennessee 8 + 5 = 13, 5 + 8 = 13 16. (a) a Ú b
e – Niñas de Tennessee (b) b Ú c y a Ú b - c 17. (a) Quita
xx cinco x.
f – Niñas de Mississippi xx
xx 82553
g – Niños de Mississippi xx

h – Niños deTennessee Quedan tres x.

Además, con la información del problema tenemos (b) n + 5 = 8, así n = 3 (c)


la diferencia es

a + b = 24 (d) 825 5 18. (a) 4 (b) 8 (c) 5 (d) 9


b + d = 29 0 1 2 3 4 5 6 7 8

a + b + f + g = 44
e + f = 29 Conexiones matemáticas 3-1
d = 17
b + f = 26 Comunicación
c + h = 17
1. No, los conjuntos A y B no necesitan ser conjuntos ajenos.
Queremos a + b + c + d + e + f + g + h. Por ejemplo, supón que hay 11 estudiantes que toman álgebra
Combinándolos y biología. Esto lo representamos en el siguiente diagrama de
a + b + f + g = 44 Venn:
c + h = 17
A B
d = 17
11 11 19
Sabemos que
a + b + c + d + f + g + h = 44 + 17 + 17 = 78.
Del diagrama de Venn observamos que n1A ´ B2 = 41 y que
Si tenemos un valor para e, tenemos la solución. no necesariamente los 52 estudiantes toman álgebra o biología.
Con d = 17 y b + d = 29, obtenemos b = 12. 3. Una flecha que empieza en 0 y termina en 3 representa el
Con b = 12 y a + b = 24, obtenemos a = 12. mismo número que una flecha que empieza en 4 y termina en 7.
Con b = 12 y b + f = 26, obtenemos f = 14. Una manera de explicar ésto a los estudiantes es hacer modelos
Y con f = 14 y e + f = 29, obtenemos e = 15. físicos y mostrar que las longitudes son las mismas por compara-
Por lo tanto, el número total en el camión es 78 + 15 = 93. ción. 5. Es útil para los estudiantes aprender más de un método
para modelar la suma y resta si estos métodos se pueden genera-
lizar avolumen
© Lopez Mateos Editores. ISBN 978-607-95583-2-1, obra completa, versión electrónica, ISBN 978-607-95583-3-8, otros conjuntos de números
1, versión electrónica. diferentes
Ejemplar deHelecto
asignado a: los números
Villarroel gutierrez -
helecto@gmail.com. Fecha: 27 de octubre de 2014. Prohibida su modificación, copia o distribución.
R12 Respuestas a los problemas

completos. El modelo del sumando faltante es útil para todos 3. (a) una posibilidad: 863 (b) una posibilidad: 368
los conjuntos de números que se usan al resolver problemas + 752 + 257
de resta. El método de contar para calcular las sumas no es 1615 625
conveniente con el conjunto de fracciones o de números rea- 4. No, él puede escoger entre el pescado o la ensalada.
les. 7. (a) Cuando colocamos el 9 y el 4 juntos, obtenemos 5. sí, $124 6. 3428
la misma longitud que 13. (b) Si colocamos 9 y 4 juntos y + 5631
encima colocamos el 4 y el 9 juntos, ambos son iguales a 13. 9059
(c) Quitamos la longitud 9 de 13 y la longitud que queda es 7. (a) 93 93 + 3 96
- 137 + 32
igual a 4. (d) Quitamos la longitud 4 de 13 y la longitud : :
- 37 - 40
que queda es igual a 9. 9. Para que 0 sea el elemento identi-
dad de la resta, lo siguiente debe ser cierto: para cualquier 56
(b) 321 321 + 2 323 323 + 60 383
entero a, a - 0 = a = 0 - a. En este caso, a - 0 = a fun-
- 138 + 2) - 140 + 602
: : : :
ciona, pero 0 - a = a no es cierto. - 38 - 40 - 100
Solución abierta 283
8. (a) (i) 687 (ii) 359
11. Las respuestas pueden variar; por ejemplo, sean + 549 + 673
A = 5a, b6 y B = 5a, b, c, d6. Entonces 4 - 2 = 16 12
n1B - A2 = n 15c, d62 = 2. 12 12
Aprendizaje colectivo 11 9
1236 1032
13. (a) La tabla muestra que si sumamos cualesquier dos núme-
ros completos de un solo dígito, la respuesta es un número com- (b) El algoritmo funciona porque el lugar de las sumas parciales
pleto. (b) La tabla muestra que si sumamos cualesquier dos conserva el valor posicional. En el ejemplo es claramente fácil
números completos de un solo dígito el orden no importa; esto es, porque sólo se suman dos dígitos cada vez. Este proceso se
si a 僆 C y b 僆 C, a + b = b + a. Cada fila de respuestas tiene puede adaptar si más de dos números se suman.
una columna correspondiente con respuestas idénticas. (c) La 9. Las respuestas pueden variar; por ejemplo: (a) El estudiante
primera fila y la primera columna muestran que si sumamos a sumó 8 + 5 = 13 y escribió 13 sin reagrupar. Después sumó
cualquier dígito la identidad, 0, obtenemos de regreso el mismo 2 + 7 = 9 y escribió 9. (b) El estudiante sumó 8 + 5 = 13
dígito. (d) Las propiedades reducen el número de hechos que re- y en vez de escribir 3 y reagruparlo con el 1, escribió 1 y
cordar: por ejemplo, los 19 números en la primera fila y columna reagrupó con el 3. (c) El estudiante sólo indica las diferencias
se pueden aprender sabiendo que 0 es el elemento identidad de la en las unidades 19 - 52, en los diez 15 - 02 y en los cien
suma. La propiedad conmutativa también reduce el número de 13 - 22. El estudiante siempre resta el más pequeño del mayor
hechos; por ejemplo, si sabes 9 + 2, entonces sabes 2 + 9. no importando qué número esta “arriba” y cual “abajo”. (d) El
15. Las respuestas pueden variar. estudiante reagrupó 3 cientos como 2 cientos y 10 dieces, pero
Preguntas del salón de clase después no reagrupó los 10 dieces como 9 # 10 + 15.
10. Paso 1—valor posicional
17. Cualquier número puede ser representado por una flecha Paso 2—propiedad conmutativa y asociativa de la adición
dirigida con una longitud dada. En este caso, la flecha dirigida Paso 3—propiedad distributiva de la multiplicación sobre la adición
representa 3 unidades. Cualquier flecha con longitud de 3 uni-
Paso 4—tablas de sumar de dígitos
dades puede usarse para representar al 3, sin importar dónde
Paso 5—valor posicional
empieza el punto inicial. 19. Las respuestas pueden variar;
por ejemplo, todos los ejemplos que la estudiante muestra son 11. (a) 68 + 23 = 16 # 10 + 82 + 12 # 10 + 32
verdaderos, pero esto no prueba que la afirmación sea verdadera = 16 # 10 + 2 # 102 + 18 + 32
en general. Todo lo que se necesita es exhibir un contraejemplo = 16 + 2210 + 18 + 32
para demostrar que la afirmación no es verdadera. Si considera- = 8 # 10 + 11
mos 5 - 8, observamos que no existe ningún número completo = 8 # 10 + 11 # 10 + 12
que satisfaga la resta. Si el conjunto fuera cerrado, entonces si es- = 18 # 10 + 1 # 102 + 1
cogemos cualesquier dos elementos y los restamos uno de otro, = 18 + 12 # 10 + 1 = 9 # 10 + 1 = 91
la respuesta tendría que ser un número completo. En el caso (b) 174 + 285 = 11 # 100 + 7 # 10 + 42 + 12 # 100
5 - 8, esto no sucede. + 8 # 10 + 52
Evaluación 3-2A = 11 # 100 + 2 # 1002
+ 17 # 10 + 8 # 102 + 14 + 52
1. (a) 981 (b) 2025 2. 29 37
= 11 + 22 # 100 + 17 + 82 # 10 + 14 + 52
+ 421 1196 = 3 # 100 + 15 # 10 + 9
1402 + 3148 = 3 # 100 + 110 + 52 # 10 + 9
6369 = 3 # 100 + 1 # 100 + 5 # 10 + 9
66 = 13 + 12 # 100 + 5 # 10 + 9

© Lopez Mateos Editores. ISBN 978-607-95583-2-1, obra completa, versión electrónica, ISBN 978-607-95583-3-8, volumen 1, versión electrónica. Ejemplar asignado a: Helecto Villarroel gutierrez -
helecto@gmail.com. Fecha: 27 de octubre de 2014. Prohibida su modificación, copia o distribución.
Respuestas a los problemas R13

= 4 # 100 + 5 # 10 + 9 Conexiones matemáticas 3-2


= 459 Comunicación
(c) 2458 + 793 = 12 # 1000 + 4 # 100 + 5 # 10 + 82
+ 17 # 100 + 9 # 10 + 32
1. Las respuestas pueden variar. Este enfoque enfatiza el
= 2 # 1000 + 14 # 100 + 7 # 1002
significado del valor posicional de los dígitos y puede ser
+ 15 # 10 + 9 # 102 + 18 + 32
más fácil para los niños pequeños que usar el algoritmo con-
= 2 # 1000 + 14 + 72 # 100 +
vencional. También puede servir como transición hacia el
15 + 92 # 10 + 18 + 32
algoritmo convencional. 3. Las respuestas pueden variar.
Por ejemplo, podemos hablar de cómo sumar 1 y restar 1
= 2 # 1000 + 11 # 100 + 14 # 10 + 11
= 2 # 1000 + 110 + 12 # 100 +
tiene el efecto de sumar 0 al problema, lo cual nos lleva a un
110 + 42 # 10 + 11 # 10 + 12
problema equivalente. La razón de hacer el cambio al pro-
blema original es que los números son más faciles de traba-
= 2 # 1000 + 1 # 1000 + 1 # 100 jar. Le puedes decir que su técnica funciona bien y discutir
+ 1 # 100 + 4 # 10 + 1 # 10 + 1
= 12 + 12 # 1000 + 11 + 12 # 100
por qué funciona. 5. (a) Las respuestas pueden variar.
+ 14 + 12 # 10 + 1
(b) En el ejemplo, si a 10 le sumamos 4 obtenemos 14 unos
y no hacemos nada más; esto cambia el problema. Así, el 10
= 3 # 1000 + 2 # 100 + 5 # 10 + 1 hay que quitarlo para que el resultado sea sumar 0, lo cual
= 3251 no cambia el problema original. Cuando sumamos 1 decena
12. (a) 4 3 5 8 (b) 4 9 2 3 a 8 decenas cuando restamos abajo, esto balancea 1 decena
+ 3 8 6 4 + 9 8 9 7 que sumamos en el minuendo. Análogamente, continuamos
0 1 1 1 1 1 1 1 con el algoritmo sumando valores en el minuendo y después
7 1 1 2 3 7 1 0
sumando el valor correspondiente en la posición adecuada
8 2 2 2 1 4 8 2 0
del valor posicional en el sustraendo para neutralizar lo que
13. (a) 121cinco (b) 20cinco (c) 1010cinco (d) 14cinco sumamos arriba. 7. Por ejemplo, las palabras reagrupar e
(e) 1001dos (f ) 1010dos intercambiar reflejan de manera más precisa las acciones rea-
14. ⴙ 0 1 2 3 4 5 6 7 lizadas cuando se resuelven problemas de suma y de resta.
0 0 1 2 3 4 5 6 7 Las palabras prestar y llevar parecen reflejar mecanización y
1 1 2 3 4 5 6 7 10 no las ideas matemáticas usadas en el algoritmo.
2 2 3 4 5 6 7 10 11 Solución abierta
3 3 4 5 6 7 10 11 12 9. Se alienta a los estudiantes a usar la bibliografía al final
4 4 5 6 7 10 11 12 13 de cada capítulo para buscar artículos de educación
5 5 6 7 10 11 12 13 14 matemática para responder estas preguntas.
6 6 7 10 11 12 13 14 15 Preguntas del salón de clase
7 7 10 11 12 13 14 15 16 11. Primero podemos determinar cuál es la intención de
Base ocho Pepe y qué va hacer después y cuál es su respuesta. Después
15. (a) 9 h 33 min 25 s (b) 1 h 39 min 40 s podemos discutir diferentes maneras de verificar si la
16. La calculadora hacía dos veces la operación. respuesta es razonable y correcta usando esta técnica. Pode-
mos modelar el problema usando la base diez mediante
11
17. (a) 4 32 (b)312 bloques; comenzamos con 6 barras y 8 unidades y
97 6 preguntamos cómo podemos “quitar” 19. Varios modelos se
141
1 30
pueden usar para mostrar que necesitamos considerar tomar
+ 1418 22
9 de 18 en lugar de 9 - 8. 13. Beti está confundida
2826 4 330 respecto a cuál es la cantidad que hay que sumar para verificar
20 3 la resta. Ella debe trabajar con números más pequeños para
1 20 sentir lo que debe hacer; por ejemplo, 9 - 5 = 4 . Para verifi-
310cinco car sumamos 4 + 5 y observamos que regresamos al 9. Pue-
18. (a) 3 gruesas 10 docenas 9 unidades den usarse rectas numéricas o carriles coloreados para mostrar
(b) 6 gruesas 3 docenas 4 unidades que 9 - 5 = 4 y que 4 + 5 regresa a la longitud 9.

19. No existe el número 5 en base cinco: 22cinco + 33cinco = Problemas de repaso


110cinco. 20. (a) 230cinco (b) 20010tres 15. No, por ejemplo 2 + 3 no es un elemento del conjunto
- 22cinco - 2022tres 51, 2, 36.
203cinco 10211tres
21. (a) 1241cinco (b) 101dos (c) DODdoce (d) 4000cinco Evaluación 3-3A
22. (a) El método produce un palíndromo en cada caso:
1. (a) 5 (b) 4 (c) cualquier número completo 2. Cada
(i) 363 (ii) 9339 (iii) 5005. (b) por ejemplo, 89 ó 97
posible pareja con dos de los conjuntos es ajena. 3. (a) sí
© Lopez Mateos Editores. ISBN 978-607-95583-2-1, obra completa, versión electrónica, ISBN 978-607-95583-3-8, volumen 1, versión electrónica. Ejemplar asignado a: Helecto Villarroel gutierrez -
helecto@gmail.com. Fecha: 27 de octubre de 2014. Prohibida su modificación, copia o distribución.
R14 Respuestas a los problemas

(b) sí (c) sí 4. (a) No, 2 + 3 = 5. (b) sí (b) Sean a , c = x y b , c = y. Entonces a = cx y b = cy.


(c) No es cerrado para ninguno, 2 + 4 = 6 y 2 # 3 = 6. Por consiguiente,
5. (a) ac + ad + bc + bd (b) n # ^ + n # ~ a + b = cx + cy
(c) ab + ac - ac o ab 6. (a) 15 + 62 # 3 = 33 (b) no = c1x + y2
Ahora por definición de división, x + y = 1a + b2 , c.
se necesitan paréntesis (c) no se necesitan paréntesis
(d) 19 + 62 , 3 = 5 7. (a) y1x + y2 (b) x1y + 12
Cuando sustituimos x y y, las propiedades se siguen.
(c) ab1a + b2 8. (a) 6 (b) 0 (c) 4 9. 72
21. (a) 4 (b) 3 (c) 2 22. 5 meses. 23. 2; quedaron 3
10. (a) propiedad asociativa de la multiplicación (b) propiedad
24. (a) (1,36), (2,18), (3,12), (4,9), (6,6), (9,4), (12,3), (18,2),
conmutativa de la multiplicación (c) propiedad conmutativa de
(36,1) (b) 36
la multiplicación (d) propiedad de la identidad de la multiplica-
ción (e) propiedad de multiplicar por cero (f ) propiedad dis- 18
tributiva de la multiplicación sobre la suma 11. (a) propiedad de
cerradura de la multiplicación (b) propiedad de multiplicar por
cero (c) propiedad de la identidad de la multiplicación 2 4 6 8 12 18 36

12. (a) propiedad distributiva de la multiplicación sobre la suma (c) Los puntos de la parte (b) están sobre una curva, mien-
(b) 32 # 12 = 32110 + 22 tras que los puntos de la suma están sobre una recta.
= 32 # 10 + 32 # 2 25. Se da una posible respuesta, resultando en 4 # 3, ó 12,
= 320 + 64 combinaciones de color.
= 384 Interior
13. (a) 9110 - 22 = 9 # 10 - 9 # 2 = 90 - 18 = 72
Exterior
rojo
azul azul

(b) 2018 - 32 = 20 # 8 - 20 # 3 = 160 - 60 = 100


blanco
rojo

14. (a) 1a + b22 = 1a + b21a + b2 = 1a + b2a +


rojo azul
blanco

1a + b2b = a2 + ba + ab + b 2 = a2 + 2ab + b 2
rojo
verde azul
blanco
rojo
(b) El área del cuadrado con lado a + b se puede expresar blanco azul

como 1a + b2 # 1a + b2 y también como la suma de las áreas


blanco

de cuatro regiones: dos cuadrados, a2 y b2, y dos rectángulos 26. (a) 3 (b) 2 (c) 2 (d) 6 (e) 4 27. sí, 64 28.
ab y ba. De aquí, (a) Resta 18 de 45. (b) Divide 54 entre 9.
1a + b22 = a2 + b 2 + ab + ba = a2 + 2ab + b 2
(c) Suma 11 y 48. (d) Suma 6 y 8. 29. (a) A>l
a b
(b) f>3 (c) 60h (d) d>7
2
a a ab
a+b Conexiones matemáticas 3-3
b ba b2

Comunicación
15. El área del cuadrado completo es 1a + b22. El área del
cuadrado pequeño es 1a - b22. El área de cada rectángulo 1. El residuo es 1. El número puede escribirse como 10q + 6,
a * b es ab, así que el área de los cuatro rectángulos es 4ab. donde q es un número completo. 10q es divisible entre 5.
De aquí que el área del cuadrado completo menos el cua- Cuando 6 es dividido entre 5, el residuo es 1. 3. Las respues-
drado pequeño es 1a + b22 - 1a - b22 = 4ab. tas pueden variar; por ejemplo, podemos pensar 9 # 7 como
16. (a) 1ab2c = c1ab2 Propiedad conmutativa de la 7 # 9 y ver si eso ayuda. Y podemos pensar 9 # 7 como 9 # 6 + 9
multiplicación ó 54 + 9 = 63 ó 9 # 7 = 9 # 5 + 9 # 2 = 45 + 18 = 63.
= 1ca2b Propiedad asociativa de la 5. Esto sucede cuando x es 0 ó 1.
multiplicación Solución abierta
(b) 1a + b2c = c1a + b2 Propiedad conmutativa de la
multiplicación 7. Las respuestas pueden variar; por ejemplo, un taxista cobra
= c1b + a2 Propiedad conmutativa de la $3 iniciales y $2 por minuto durante 6 minutos. (Quizá los
suma precios no sean reales, pero éste es el tipo de problemas que
17. a. y1x - y2 b. 471101 - 12 (c) ab1b - a2 pueden sugerir los estudiantes).
18. (a) 40 = 8 # 5 (b) 326 = 2 # x 19. (a) 18 , 42 , Aprendizaje colectivo
2 Z 8 , 14 , 22 (b) 8 , 12 + 22 Z 18 , 22 + 18 , 22
20. (a) Supón que tenemos dos bolsas de canicas; en una bolsa 9. (a) Las respuestas pueden variar (b) Sabemos que si
hay a canicas y en la otra b. Queremos distribuir las canicas de ma- a , b = c, entonces a = bc, donde a, b y c son enteros y
nera equitativa entre c estudiantes. Entonces el número de canicas c Z 0. Por lo tanto, para hallar 35 , 5 vemos la tabla y ba-
que obtiene cada estudiante se puede hallar de dos maneras, como jamos hasta la fila 5, de ahí al 35, y después, para obtener el
sigue: colocando todas las canicas en una bolsa, tenemos a + b ca- otro factor, subimos al 7. Así, 35 , 5 = 7. (c) La única
nicas y cada estudiante recibe 1a + b2 , c. También podemos di- manera de que un producto de dos números sea impar es
vidir las canicas de la primera bolsa y luego dividir las canicas de la que los dos factores sean impares. Los ocho productos alre-
segunda bolsa. De esta manera, cada estudiante obtendría dedor de cada número impar contienen un factor par y por
1a , c2 + 1b , c2 canicas. lo tanto son pares.

© Lopez Mateos Editores. ISBN 978-607-95583-2-1, obra completa, versión electrónica, ISBN 978-607-95583-3-8, volumen 1, versión electrónica. Ejemplar asignado a: Helecto Villarroel gutierrez -
helecto@gmail.com. Fecha: 27 de octubre de 2014. Prohibida su modificación, copia o distribución.
Respuestas a los problemas R15

Preguntas del salón de clase 10 5

11. Podemos aconsejar a Susi que sustituya números en a y 15 # 21 = 315


b para ver si su afirmación es correcta. Por ejemplo, si a = 2
y b = 4, entonces 312 # 42 = 3 # 8 = 24 y 13 # 2213 # 42 = 20

6 # 12 = 72. Así, hemos encontrado un contraejemplo para


mostrarle a Susi que su afirmación es falsa. En este punto las
propiedades asociativas y distributivas pueden demostrarse.
13. En general, a , 1b - c2 Z 1a , b2 - 1a , c2. Por
1

ejemplo, 100 , 125 - 52 Z 1100 , 252 - 1100 , 52. De


(c) 4 3

2 43cinco # 23cinco = 2144cinco


hecho, el miembro derecho es 4 - 20, el cual no está defi-
nido en el conjunto de números completos. Sin embargo, la 3
propiedad distributiva de la división sobre la resta funciona
asegurándose de que cada expresión esté definida en el con- Para hallar 43cinco # 23cinco, contamos el número de losas,
junto de los números completos; esto es, barras y unidades; tenemos 4 # 2 losas, 2 # 3 + 3 # 4 barras y
1b - c2 , a = 1b , a2 - 1c , a2. 3 # 3 unidades. Recordamos que 5 unidades = 1 barra, 5
barras = 1 losa y 5 losas = 1 bloque, y tenemos
Problemas de repaso 43cinco # 23cinco = 2 bloques, 1 losa, 4 barras y 4
15. Por ejemplo, 50, 16 17. (a) El estudiante no reagrupó unidades = 2144cinco. 7. (a) 293 # 476 = 139,468
7 + 6 = 13 como 1 # 10 + 3 y no llevó el 1 a la columna de las (b) El lugar indica el valor posicional.
decenas. (b) El estudiante sumó 5 + 7 = 12 y sólo lo escribió (c) 363
sin considerar el valor posicional en la columna de las decenas. * 84
Análogamente, el estudiante sumó 3 + 4 = 7 y lo escribió en la 2904 18 * 3632
columna de las centenas. (c) El estudiante tomó la diferencia 1452 14 * 3632
absoluta entre 9 y 6 en lugar de reagrupar para tomar 9 de 16. 30492
(d) El estudiante no disminuyó el número en las decenas des- 8. : 17 * 63
pués de reagrupar. 8 126
4 252
Evaluación 3-4A 2 504
: 1 1008, y 63 + 1008 = 1071
1. (a) 426 (b) 327
* 783 * 941 9. (a) 1332 calorías (b) Juana, 330 calorías más
1278 327 (c) Mauricio, 96 calorías más 10. No 11. (a) 77
3408 1308 residuo 7 (b) 8 residuo 10 (c) 10 residuo 91 12. 3
2982 2943 13. 2 11
333558 307707 4 15
2. (a) (b) 0 7
7 2 8 3 0 6 6 19
6 6 31 87 2 9 0 0 1 2 12 31
6 0 2
8 2 80 83 2 4 7 1 20 02 4 4 14. (a) 32 23
4 3 2 3 4 4 * 69 * 96
728 # 94 = 68,432 #
306 24 = 7,344 2208 2208
(b) 110a + b2110c + d2 = 110b + a2110d + c2 implica
3. Las diagonales separan el valor posicional como se colocan los 100ac + 10bc + 10ad + bd = 100bd + 10ad + 10bc + ac ó
valores en el algoritmo tradicional. 4. (a) 519 (b) 615 (c) 10313 99ac = 99bd, lo cual implica ac = bd. Así, esto funciona cada
(d) 1012 ó 212 # 512 5. (a) 2100 porque 280 + 280 = vez que que el producto de los dígitos de las unidades sea
28011 + 12 = 280 # 2 = 281 (b) 2102 porque 2101 = 2100 # 2 igual al producto de los dígitos de las decenas. 15. 3 h
y 2102 = 2100 # 4 6. Los siguientes productos parciales, que 16. 1356, 2712 y 452 17. $142 18. (a) 5 fue
se obtienen por la propiedad distributiva de la multiplicación multiplicado por 6 para obtener 30; el 3 se reagrupó y
sobre la suma, se muestran en el modelo. después 3 fue multiplicado por 2 para obtener 6; el resultado
(a) 22 (b) 15 del “reagrupamiento” se sumó para obtener 9, lo cual fue in-
* 13 * 21 dicado. (b) Cuando bajamos el 1, el cociente de 0 no fue
6 13 * 22 5 11 * 52 indicado. 19. (a) valor posicional y propiedad distributiva
60 13 * 202 10 11 * 102 de la multiplicación sobre la suma; propiedad asociativa de la
20 110 * 22 100 120 * 52 multiplicación; definición de a n; propiedad de la identidad
200 110 * 202 200 120 * 102 para la suma y propiedad de la multiplicación con el cero, va-
286 315 lor posicional.

© Lopez Mateos Editores. ISBN 978-607-95583-2-1, obra completa, versión electrónica, ISBN 978-607-95583-3-8, volumen 1, versión electrónica. Ejemplar asignado a: Helecto Villarroel gutierrez -
helecto@gmail.com. Fecha: 27 de octubre de 2014. Prohibida su modificación, copia o distribución.
R16 Respuestas a los problemas

(b) 34 # 102 = 13 # 10 + 42102 valor posicional Preguntas del salón de clase


= 13 # 102102 + 4 # 102 propiedad 11. Evidentemente el estudiante no entiende el proceso de
distributiva de la la división larga. El método de las restas sucesivas puede
multiplicación ayudar a entender el error.
sobre la suma 6 冄 36
= 3110 # 1022 + 4 # 102 propiedad asocia- -6 1 seis
tiva de la
30
multiplicación
- 30 5 seises
= 3 # 103 + 4 # 102 definición de an
# 3 # 2 # 6 seises
= 3 10 + 4 10 + 0 10 propiedad de la
+ 0#1 identidad en la En lugar de sumar 1 y 5, el estudiante escribió 15. 13. Si el
suma y propie- número tiene tres dígitos y en las unidades el 0, tenemos
dad de la multipli- ab0 , 10 = ab ya que ab # 10 = ab0. Esto no funciona si el
cación por cero dígito 0 no está en las unidades. 15. (a) La primera ecua-
= 3400 valor posicional ción es verdadera porque 39 + 41 = 39 + 11 + 402 =
20. Se necesitan 58 autobuses, no todos llenos. 139 + 12 + 40 = 40 + 40. Ahora, 39 # 41 =
21. (a) 763 (b) 678 22. (a) nueve (b) seis 140 - 12140 + 12 = 402 - 12, y 402 - 1 Z 402. (b) Sí,
* 8 * 3 este patrón continúa pues los números considerados son de
6104 2034 la forma 1a - 121a + 12, que es igual a a2 - 1.
23. (a) (b) a = 5, b = 7
3 2 3 Problemas de repaso
3 22 13 22 4 17. (a) 1a + b + 22x (b) 13 + x21a + b2
0 1 10 4 1 1 2 19. (a) 36 = 4 # 9 (b) 112 = 2x (c) 48 = x # 6, ó
2 2 1 48 = 6x (d) x = 7 # 17

24. (a) 233cinco (b) 4cinco R1cinco (c) 1513seis Evaluación 3-5A

2. (a) 19 # 62 # 12 # 52 = 54 # 10 =
(d) 31cinco (e) 110dos (f) 1101110dos
1. (a) 160 (b) 120
Conexiones matemáticas 3-4 540 (b) 18 72 125 42 = 56 # 100 = 5600
# # # 3. (a) 605
(b) 963 4. (a) 36 (b) 120 (c) 46 (d) 97
Comunicación 5. 496 mi 6. (a) 28 + 2 = 30; 30 + 20 = 50;
1. Como 345 # 678 = 345 # 16 # 102 + 7 # 10 + 82, una expli- 50 + 3 = 53; así la respuesta es 2 + 20 + 3 = 25.
cación puede ser la siguiente: usando la propiedad distributiva de (b) 47 + 3 = 50; 50 + 10 = 60; 60 + 3 = 63; así la res-
la multiplicación sobre la suma, primero multiplicamos 345 por 6 puesta es 3 + 10 + 3 = 16.
y el resultado por 102; después multiplicamos 345 por 7 y el re- 7. (a) 86 + 37 = 180 + 302 + 17 + 62 = 123 sumando
sultado por 10; después multiplicamos 345 por 8. Sumamos to- las decenas y las unidades por separado. (b) 97 + 54 =
dos los números obtenidos previamente. 3. El resultado siempre 97 + 3 + 54 - 3 = 100 + 51 = 151 sumando 3 a 97 y
es 4. Sea x el número original. Las operaciones aparecen como luego restándolo de 54. (c) 230 + 60 + 70 + 44 +
sigue: 40 + 6 = 1230 + 702 + 160 + 402 + 144 + 62 = 450
C 12x23 + 24 D >6 - x = 4 usando números compatibles. 8. (a) 5300 (b) 100,000
(c) 120,000 (d) 2330 9. Las respuestas pueden variar; por
5. Las respuestas varían dependiendo de la elección de los
ejemplo: (a) 900 , 30 = 30 (b) 25,000 - 20,000 = 5000
estudiantes. Muchos estudiantes prefieren el algoritmo de la
(c) 30 # 30 = 900 (d) 2000 + 3000 + 6000 + 1000 =
multiplicación reticular porque multiplican los dígitos solos y
12,000 10. Las respuestas pueden variar; por ejemplo:
posteriormente suman. 7. abba = a # 103 + b # 102 +
(a) 2 + 3 + 5 = 10, de modo que 10,000 es un estimado
b 10 + a = a 1001 + b 110 = 11191 # a + 10 # b2. Sí,
# # #
inicial. 10,000 + 2000 1ajuste2 =12,000 como estimado fi-
abccba = a # 105 + b # 104 + c # 10 3 + c # 10 2 +
nal. (b) La suma de los dígitos de la izquierda es 22, de
b # 10 + a = a # 100001 + b # 10010 + c #
modo que 2200 + 270 al ajustar da 2470. 11. (a) El pri-
1100 = 111a # 9091 + b # 910 + c # 1002.
mer conjunto de números no está en cúmulos. El segundo
Aprendizaje colectivo conjunto está acumulado alrededor de 500, de modo que un
estimado es 2500. (b) Las estimaciones pueden variar.
12. (a) El rango es 600 120 # 302 a 1200 130 # 402. (b) El
9. Los argumentos varían dependiendo de los grupos.
rango es 700 1100 + 6002 a 900 1200 + 7002. (c) El rango
Algunos argumentarán que después de la suma se debe ver
es 230 1200 + 302 a 340 1300 + 402. 13. Las respuestas
la resta porque son operaciones inversas. Otros argumenta-
rán que después de la suma hay que ver la multiplicación
pueden variar; por ejemplo, 3300 - 100 - 300 - 400 -
porque la multiplicación es una suma repetida. Esto
500 = 2000. El estimado es alto pues las cantidades se
también pospone la resta hasta que los estudiantes estén
redondearon a la centésima más cercana y $8 se quitaron de
mejor preparados.
© Lopez Mateos Editores. ISBN 978-607-95583-2-1, obra completa, versión electrónica, ISBN 978-607-95583-3-8, volumen 1, versión electrónica. Ejemplar asignado a: Helecto Villarroel gutierrez -
helecto@gmail.com. Fecha: 27 de octubre de 2014. Prohibida su modificación, copia o distribución.
Respuestas a los problemas R17

la cuenta de cheques mientras que $13 se sumó a $3287. 11. Al usar primero la calculadora, ella no está aprendiendo la
14. Por ejemplo, 35 # 20 = 700 asientos ó 40 # 25 = 1000 habilidad de estimar, lo que es muy útil para tomar decisiones.
asientos; 700 será bajo y 1000 alto. 15. (a) Diferentes resulta- Uno de los usos importantes es determinar por estimación si una
dos pues los estimados de 800 y 220 están lejos uno de otro. respuesta es razonable. Por ejemplo, si el problema es 492 # 63,
(b) Misma respuesta pues 22 fue dividido entre 2 para obtener por estimación, la estudiante podrá saber si la respuesta se apro-
11 mientras que 32 se multiplicó por 2 para obtener 64. El re- xima a 30,000. Si obtiene una respuesta tal como 17,712 1492 #
sultado es multiplicar el cálculo original por 2>2, ó 1, lo cual 362 ó 59,346 1942 # 632, ella sabrá que hubo un error en el
no lo cambia. (c) La misma respuesta pues el primer número cálculo. (Tal vez pulsó la tecla equivocada en la calculadora.)
se multiplicó por 3 y el segundo número se dividió entre 3, lo
Problemas de repaso
que resultó en que multiplicamos por 1 el cálculo original, el
cual no cambia. 16. (a) falso (b) falso (c) falso
13. (a) 18 冄 623
(d) verdadero 17. La estrategia de cúmulos da 6 # 70,000, 34
ó 420,000. 18. (a) alto; 299 # 3 6 300 # 3 - 180 ; 10 (18’s) 18 冄 623
(b) bajo; 6,001 , 299 7 6000 , 300 (c) bajo; 443 -54
6,000 , 299 7 6000 , 300 (d) bajo; 10 # 99 es sólo 990 18 冄 443 83
19. Una posibilidad es que al hallar 110x + 522 podamos - 180 ; 10 (18’s) -72
escribir 110x + 522 = 100x2 + 50x + 50x + 25 = 263 11
100x2 + 100x + 25 = 100x1x + 1) + 25. Por ejemplo, en - 180 ; 10 (18’s)
652 tomamos 6 # 7 = 42 y agregamos 25 para obtener 4225. 83
- 18 ; 1 (18)
Conexiones matemáticas 3-5 65
Comunicación - 18 ; 1(18)
47
1. La matemática mental es el proceso de producir una
- 18 ; 1(18)
respuesta exacta a un cálculo sin usar ayudas externas. La esti-
29
mación de un cálculo es el proceso de formar una respuesta
- 18 ; 1 (18)
aproximada a un problema numérico. 3. Las respuestas pue-
11 34 (18’s)
den variar; por ejemplo, los estudiantes pueden sugerir que la
(b) 21 冄 493 23
matemática mental y la estimación son necesarias todos los
días con objeto de determinar rápidamente si el cálculo de los - 210 ; 10 (21’s) 21 冄 493
totales en varias operaciones fue efectuado correctamente. La 283 - 42
matemática mental y la estimación ayudan a los estudiantes a - 210 ; 10 (21’s)
73
saber si los resultados de la calculadora son razonables. La 73
- 21 ; 1 (21) - 63
NCTM (Consejo Nacional de Maestros de Matemáticas de
Estados Unidos) señala varios puntos importantes alrededor 52 10
del cálculo mental y la estimación en los Principios y objeti- - 21 ; 1 (21)
vos y Puntos focales. 31
- 21 ; 1 (21)
Solución abierta 10 23 (21’s)
5. Las respuestas pueden variar; por ejemplo, cuando determi-
(c) 97 冄 1000 10
namos el monto de la propina de un mesero en un restaurante.
- 970 ; 10 (97’s) 97 冄 1000
Aprendizaje colectivo 30 - 97
7. Las respuestas pueden variar, dependiendo del nivel del 30
grado y del libro de texto que se elija. - 0
30
Preguntas del salón de clase
Revisión del capítulo
9. Mane tuvo una buena idea y casi la tuvo correcta. Sólo
necesitamos indicar que ella necesitaba sumar el 2 al final en 1. (a) propiedad distributiva de la multiplicación sobre la
lugar de restar. El razonamiento de esto se debe explicar. adición (b) propiedad conmutativa de la adición
Por ejemplo, (c) propiedad de la identidad de la multiplicación
261 - 48 = 261 - 150 - 22 (d) propiedad distributiva de la multiplicación sobre la
= 261 - 50 + 2 suma (e) propiedad conmutativa de la multiplicación
= 1261 - 502 + 2 (f ) propiedad asociativa de la multiplicación
= 211 + 2 2. (a) 3 6 13 pues existe un número natural, digamos 10,
= 213 tal que 3 + 10 = 13 (b) 12 6 9 ó 9 6 12 pues existe un
número natural, digamos 3, tal que 9 + 3 = 12.
© Lopez Mateos Editores. ISBN 978-607-95583-2-1, obra completa, versión electrónica, ISBN 978-607-95583-3-8, volumen 1, versión electrónica. Ejemplar asignado a: Helecto Villarroel gutierrez -
helecto@gmail.com. Fecha: 27 de octubre de 2014. Prohibida su modificación, copia o distribución.
R18 Respuestas a los problemas

3. (a) 15, 14, 13, 12, 11 ó 10 (b) 10 (c) cualquier Respuestas a Ahora intenta éste
número completo (d) los números completos de 0 a 26 3-1. Por ejemplo, si los elementos de los conjuntos son a, b, c y
4. (a) 15a (b) 5x2 (c) xa + xb + xy a, d, entonces los elementos de la unión de los conjuntos
(d) 3x + 15 + xy + 5y o 1x + 5213 + y2 5. 40 latas son a, b, c, d. La unión tiene sólo 4 elementos mientras que
6. 12 maneras 7. 26 8. $60,000 para 80 personas es la los conjuntos originales tienen 3 y 2, respectivamente. La
más barata. 9. $214 10. $400 11. (a) Si n es el suma 3 y 2 es 5, no 4, el número de elementos de la unión.
número original, entonces cada una de las siguientes líneas 3-2. Las respuestas pueden variar; por ejemplo, los estudiantes
muestra los resultados de las instrucciones realizadas: cuando empiezan a contar, usualmente comienzan con 1 de
modo que algunas veces en la recta numérica comienzan con el
n
1. Debe señalarse que cuando trabajamos con los números com-
+ 17
n
pletos, el primer número es 0. Después, para representar el 3 en
21n + 172
= 2n + 34
la recta numérica trazamos una flecha (vector) de longitud 3
2n
+ 30
unidades. La longitud de la flecha en la figura 4-3 es 2 unidades.
4n
+ 60 3-3. (a) Cerrado; un número par más un número par es siempre
4n
+ 80 un número par. (b) No es cerrado; por ejemplo, 1 + 3 = 4 y 4
+ 20
n no es un elemento de F.
n 3-4. A n(A) = 3 B n(B) = 5 n(B) = 5 n(A) = 3
B A
(b) Las respuestas pueden variar; por ejemplo, las siguientes d e d e
a b a b
dos líneas pueden ser restar 65 y después dividir entre 4. c
f g
hi
f g
hi c
(c) Las respuestas pueden variar. 12. 1119 13. 60,074
14. (a) 5 residuo 243 (b) 91 residuo 10 (c) 120cinco residuo
2cinco (d) 11dos residuo 10dos 15. (a) 912 # 5 + 243 = 4803
a b a b
c d c d

(b) 11 # 91 + 10 = 1011 (c) 23cinco # 120cinco + 2cinco =


e f e f
g hi g hi

3312cinco (d) 11dos # 11dos + 10dos = 1011dos n(A) + n(B) = 3 + 5 = 8 = n(A B) n(B) + n(A) = 5 + 3 = 8 = n(B A)

16. (a) 119 # 194210 = 36,860 (b) 1379 # 1932100 = 3-5. Sea B 8 A. Si n1A2 = a y n1B2 = b, entonces
7,314,700 (c) 481 # 73 # 18 # 1252 = 1481 # 7321000 = a - b = n1A - B2. Por ejemplo, A = 5a, b, d, e6 y
35,113,000 (d) 374 # 893 # 1200 # 502 = 1374 # 8932 # B = 5a6, A - B = 5b, d, e6, n1A2 = 4, n1B2 = 1,
10,000 = 3,339,820,000 17. $395 18. $4380 n1A - B2 = 3. 3-6. (a) El conjunto de los números com-
19. 2600 cajas 20. $3842 21. $2.16 22. 36 bicicletas y pletos no es cerrado bajo la resta; por ejemplo, 2 - 5 no es un
18 triciclos 23. (a) 212cinco (b) 101dos (c) 1442cinco número completo. (b) En los números completos la resta no
(d) 101101dos 24. (a) por ejemplo, 126 + 242 + es asociativa; por ejemplo, 9 - 17 - 22 Z 19 - 72 - 2.
137 - 72 = 50 + 30 = 80 (b) por ejemplo, (c) La resta no es conmutativa para los números completos;
17 # 9214 # 252 = 63 # 100 = 6300 25. (a) 441 (b) 36 por ejemplo, 3 - 2 Z 2 - 3. (d) No hay elemento identidad
(c) 180 (d) 406 26. Las respuestas pueden variar; por para la resta de números completos; por ejemplo, 5 - 0 =
ejemplo, (a) 2300 + 300 (ajuste) = 2600, (b) 2600. 5 Z 0 - 5. 3-7. (a) 5 (b) 7 (c) 10 3-8. 182 + 61
27. 2400 # 4 = 9600 28. Las respuestas pueden variar; por puede estar representado por:
ejemplo, primero estimamos cuántas veces cabe 14 en 322. Al 1 bloque 8 losas 2 unidades
menos hay 20, de modo que usando el valor posicional indica- 6 losas 1 unidad
mos un 2 arriba del 2 en 32 para indicar que estamos quitando 1 bloque (10 losas + 4 losas) 3 unidades
20 conjuntos de 14. Al quitar 280 de 322, quedan 42. Después ó 1 bloque (1 bloque + 4 losas) 3 unidades
estimamos cuántas veces cabe 14 en 42. La respuesta es 3, así ó 2 bloques 4 losas 3 unidades
que escribimos 3 junto al 2 en el cociente para indicar que ó 243
quitamos 3 conjuntos de 14. Esto nos deja 0, de modo que no 3-9. (i) El método es válido porque restar y sumar el
hay más 14 que quitar. El 23 indica que 322 contiene 23 con- mismo número no cambia la suma original.
juntos de 14 y el 0 indica que no queda nada por quitar. (ii) 97 + 69 = 197 + 32 + 169 - 32
29. (a) 999 # 47 + 47 = 47 1999 + 12 = 47 # 1000 = = 100 + 66
47,000 (b) 43 # 59 + 41 # 43 = 43 # 159 + 412 = 166
= 43 # 100 = 4300 (c) 1003 # 79 - 3 # 79 = 79 # 3-10. (a) 1000cinco (b) 31cinco
11003 - 32 = 79 # 1000 = 79,000 (d) 1001 # 113 - 113 = 3-11. (a) ⴙ 0dos 1dos
113 # 11001 - 12 = 113 # 1000 = 113,000 (e) 101 # 35 = 0dos 0dos 1dos
1100 + 1235 = 100 # 35 + 1 # 35 = 3500 + 35 = 3535 1dos 1dos 10dos
(f) 98 # 35 = 1100 - 2235 = 100 # 35 - 2 # 35 =
3500 - 70 = 3430 30. (a) 3x3 + 4x2 + 7x + 8 + (b) (i) 1101dos (ii) 1111dos
+ 15x2 + 2x + 12 = 3x3 + 9x2 + 9x + 9 (b) Las res- -111dos + 111dos
puestas pueden variar. (c) Las respuestas pueden variar; 110dos 10110dos
por ejemplo, 34 # 102 = 13 # 10 + 42 # 102 = 3 # 103 + 3-12. Una explicación posible es como sigue: la primera camisa
4 # 102 + 0 # 10 + 0 = 3400 y 13x + 42x 2 = 3x 3 + 4x 2. se puede usar con cada uno de los 5 pantalones, lo cual da un to-
tal de 5 vestimentas diferentes. Además, la segunda camisa se
© Lopez Mateos Editores. ISBN 978-607-95583-2-1, obra completa, versión electrónica, ISBN 978-607-95583-3-8, volumen 1, versión electrónica. Ejemplar asignado a: Helecto Villarroel gutierrez -
helecto@gmail.com. Fecha: 27 de octubre de 2014. Prohibida su modificación, copia o distribución.
Respuestas a los problemas R19

puede usar con cada uno de los 5 pantalones, lo cual da un total de Sección 3-4
5 vestimentas diferentes de las anteriores. De manera análoga, la (a) 570,140 (b) 38 ó 39
3ª, 4ª, 5ª y 6ª camisas se pueden combinar con los 5 pantalones * 6 38 39
para obtener nuevas vestimentas. De esta manera, cada una de las
3,420,840 + 38 + 39
6 camisas se puede usar para obtener 5 nuevas vestimentas, lo
cual hace un total de 5 + 5 + 5 + 5 + 5 + 5 = 6 # 5 114 117
vestimentas. Sección 3-5
3-13. 91 miembros 3-14. (a) El conjunto de números com-
Hay 85 miembros en una asociación de padres y maestros;
pletos no es cerrado bajo la división; por ejemplo, 8 , 5 no es
para contactarse usan un árbol telefónico. Vamos a suponer
un número completo. También, 8 , 2 Z 2 , 8 y
18 , 42 , 2 Z 8 , 14 , 22 muestran que no es conmuta-
que cada llamada lleva 30 s y que todos están en casa y con-
testan el teléfono. La clave para resolver el problema es
tiva ni asociativa. (b) 1 no es la identidad para la división de
darse cuenta de que cuando alguien llama a la primera per-
números completos pues n , 1 = n para todos los números
sona de las dos que tiene que llamar, la primera persona no
completos n y 1 , n Z n excepto cuando n = 1.
3-15. 10 # 600 = 10116 # 1022
espera a que el que la llamó haga su segunda llamada para
= 16 # 1022101
hacer las llamadas que le corresponden. Una estrategia es
construir un modelo de este árbol telefónico y una tabla
= 6 # 11021012 para mantenerse informado del número de personas contac-
= 6 # 103 tadas. Un modelo para los primeros 3 min se da a continua-
= 6 # 103 + 0 # 102 + 0 # 10 + 0 # 1 ción. La tabla de la derecha en el modelo muestra el
= 6000 número de Personas llamadas en cada intervalo de 30 s y
20 # 300 = 12 # 101213 # 1022 también muestra el Número total de llamadas.
= 12 # 321101 # 1022
= 61101 # 1022
Total de
0s Personas números

= 6 # 103
Tiempo llamadas llamados
30 s
0 1 1
= 6 # 103 + 0 # 102 + 0 # 10 + 0 # 1 1 30 s 1 2

= 6000 90 s 1 min 2 4

3-16. 7 # 4589 = 714 # 103 + 5 # 102 + 8 # 10 + 92 2 90 s 3 7

= 17 # 42103 + 17 # 52102 + 17 # 82 # 150 2 min 5 12

10 + 7 # 9
150 s 8 20
3
3 min 13 33
= 28000 + 3500 + 560 + 63
= 32123 Si examinamos los números en la columna de las Personas llama-
3-17. Las respuestas pueden variar; por ejemplo, das, notamos una sucesión familiar, esto es, la sucesión de
(a) 40 + 160 = 200 y 29 + 31 = 60 así la suma es 260. Fibonacci donde cada número en la sucesión es la suma de los dos
(b) 3679 - 400 = 3279 y 3279 - 74 = 3205. números anteriores. Esto continúa, como se observa en el mo-
(c) 75 + 25 = 100 y 100 + 3 = 103. delo, porque el número de puntos en cada segmento es la suma
(d) 2500 - 500 = 2000 y 2000 - 200 = 1800. 3- de los puntos de los segmentos previos. Así, necesitamos conti-
18. Las respuestas pueden variar; por ejemplo, nuar la tabla para ver en qué tiempo el número total de personas
(a) 4 # 25 = 100 y 32 # 100 = 3200. (b) 123 # 3 = llamadas iguala o excede 85. Continuando la tabla, observamos
100 # 3 + 23 # 3 = 300 + 69 = 369. (c) 25 # 35 = que 8 + 13 = 21 personas más serían llamadas a los 210 s,
130 - 52130 + 52 = 302 - 52 = 900 - 25 = 875. dando un total de 33 + 21 = 54. A los 4 min habría
(d) 5075>25 = 5000>25 + 75>25 = 200 + 3 = 203. 13 + 21 = 34 personas más llamadas, dando un total de
3-19. Las respuestas pueden variar; por ejemplo, (a) Para es- 54 + 34 = 88. Así, podemos llamar a 85 personas en 4 min.
timar 4525 # 9, sabemos que 4525 # 10 = 45,250 y como sólo Podemos extender el problema tratando números diferentes
tenemos 9 conjuntos de 4525 podemos tomar la aproximación de miembros del grupo o generalizar el problema para encon-
de 5000 como el estimado y obtenemos 40,250. (b) Para es- trar el intervalo de tiempo requerido para llamar a n personas.
timar 3625>42, sabemos que la respuesta es cercana a Podemos también investigar cómo aparece la sucesión de Fi-
3600>40, ó 90. bonacci en otros contextos, tales como los conos de los pinos
o los hábitos reproductivos de algunos animales.
Respuestas a los Rompecabezas
Respuestas a las Actividades de laboratorio
Sección 3-1
Las respuestas pueden variar. Por ejemplo, Sección 3-2
1 2 3
Cualquiera de los dos ábacos se pueden usar. Los dos son
8 9 4
comparables. Algunos preferirán el ábaco chino porque
7 6 5
Sección 3-2 tiene dos cuentas arriba de la barra y cinco debajo de la
barra.
El número de placa es 10968.
© Lopez Mateos Editores. ISBN 978-607-95583-2-1, obra completa, versión electrónica, ISBN 978-607-95583-3-8, volumen 1, versión electrónica. Ejemplar asignado a: Helecto Villarroel gutierrez -
helecto@gmail.com. Fecha: 27 de octubre de 2014. Prohibida su modificación, copia o distribución.
R20 Respuestas a los problemas

Sección 3-3 Conexiones matemáticas 4-1


1. sí 2. 18 y 19 3. En general, los números pares parecen Comunicación
alcanzar más rápido el 1. 4. Las respuestas dependen de las
1. Los dos son correctos. Para el primer estudiante, x es el
elecciones. (Nota: La pregunta 1 es un problema famoso no
primero de los tres números naturales consecutivos. El se-
resuelto en matemáticas.)
gundo escogió a x como el segundo de los tres números na-
Sección 3-4 turales consecutivos.

1. (a) Calcula (b) Las respuestas pueden variar. Solución abierta


2. (a) Cuando una persona dice su edad mediante cartas listadas, 3. Las respuestas pueden variar.
la persona está dando la representación de su edad en base dos.
El número puede ser determinado sumado los números en la es- Preguntas del salón de clase
quina superior izquierda de las cartas nombradas. 5. El estudiante tiene razón. Si el tercer entero es x, los cinco
(b) La carta F tendría 32 en la esquina superior izquierda. enteros son x - 2, x - 1, x, x + 1, x + 2; su suma es 5x.
Cada uno de los números del 1 al 63 se podría escribir en base Para cualquier sucesión aritmética con diferencia d, los
dos de modo que puedas decir dónde poner los números en cinco primeros enteros son x - 2d, x - d, x, x + d,
las cartas. x + 2d. Su suma es 5x.
7. Sí, por ejemplo en 5A ƒ A ( C 6, A es un conjunto varia-
Solución al Problema preliminar ble; cualquier conjunto que sea un subconjunto propio de C,
Las respuestas pueden variar. A continuación se dará un el conjunto de números completos.
conjunto de respuestas que muestran cómo usar 5 cincos
agrupándolos con símbolos para obtener todos los números Evaluación 4-2A
del 1–10.
1. Si ^ n = 12, entonces ~ ~ + 12 = 18 y ~ = 3. En-
15 , 52 + 315 - 52 # 54 = 1 tonces n ~ ~ = n + 6 = 10, de modo que n = 4. Si
315 + 52 , 54 + 15 - 52 = 2
n = 4, entonces 4 + ^ = 12 y ^ = 8. Por lo tanto,
~ = 3, n = 4 y ^ = 8.
315 + 52 , 54 + 15 , 52 = 3 2. (a) 24 (b) 20 (c) 9 (d) 3 (e) 3
315 # 52 , 54 - 15 , 52 = 4 3. 22 4. 524 boletos para estudiantes 5. Sea x la cantidad
5 + 315 - 52 # 554 = 5
que recibió la menor. Entonces x + 3x + x + 14,000
= 486,000, ó 5x = 472,000. La menor recibió $94,400, la
315 # 52 , 54 + 15 , 52 = 6 mayor $283,200 y la de enmedio $108,400. 6. 41, 41 y 38
5 + 15 , 52 + 15 , 52 = 7 pulg. 7. Sea x el número de monedas de a cinco. Entonces
67 - x es el número de monedas de a diez. De modo que
5 + 315 + 5 + 52 , 54 = 8 10167 - x2 + 5x = 420, x = 50. Por lo tanto, 50 son de
355 - 15 + 524 , 5 = 9 cinco y 17 de diez. 8. Ricardo 4, Miriam 14 9. 625 10. 350
5 + 5 + 315 - 52 # 54 = 10
yd por 175 yd

Conexiones matemáticas 4-2


Capítulo 4 Comunicación
Evaluación 4-1A 1. Las dos son correctas. Para el primer estudiante, x es el prime-
ro de los tres números completos consecutivos. Para el segundo x
1. (a) 10 + 2d (b) 2n - 10 (c) 10n2 (d) n2 - 2n es el segundo de los tres números completos consecutivos.
71n + 32 - 14
2. (a) - n (b) 1 3. (a) 21n + 12 Solución abierta
7
(b) 1n + 222 - 21n + 12 4. (a) 200 + 250h (b) 175d 3. Las respuestas pueden variar. Por ejemplo: (a)
(c) 3x + 3 (d) q # 2n (e) 40 - 3t°F (f ) 4s + 15,000 1x + 121x + 22 = x2 + 3x + 2 (b) 2x + 3 = 2x - 1
(g) 3x + 5 (h) 3m 5. S = 20P 6. g = 5 + b (c) 3x + 1 = 2x + 1
7. 6n + 4 8. (a) P = 8t Preguntas del salón de clase
dólares (b) P = 15 + 101t - 12 dólares para t Ú 1
9. 5x + 1300 dólares 10. Las respuestas en dólares: 5. x = 0 es una solución. El estudiante está mal pues no se
(a) la más joven x, la mayor 3x, la de puede dividir entre 0. 7. Pues puede haber un error al plan-
3x tear la ecuación y así la ecuación no representa un modelo
enmedio dólares (b) la de enmedio y, la mayor 2y, la del problema.
2
2y z z Problemas de repaso
más joven c) la mayor z, la más joven , la de enmedio
3 3 2
9. x = 3y 11. Juan x, Julia 2x, Tina 6x

© Lopez Mateos Editores. ISBN 978-607-95583-2-1, obra completa, versión electrónica, ISBN 978-607-95583-3-8, volumen 1, versión electrónica. Ejemplar asignado a: Helecto Villarroel gutierrez -
helecto@gmail.com. Fecha: 27 de octubre de 2014. Prohibida su modificación, copia o distribución.
Respuestas a los problemas R21

Evaluación 4-3A Observa que como no podemos pintar 100 puntos en la grá-
fica solamente dibujamos un punto cada 10 minutos. Como
1. (a) Duplica el número de entrada. (b) Suma 6 al número de en la parte (a) suponemos un número completo de minutos,
entrada. 2. (a) Esto no es una función, ya que la entrada 1 está los puntos no están conectados; sin embargo, debido a la es-
asociada con dos salidas (a y d ). (b) Esto es una función. cala, si dibujamos 100 puntos, éstos se verían conectados.
3. (a) Las respuestas pueden variar. Por ejemplo: (b) 30 (b) La compañía cobra por cada parte de un minuto a ra-
1 zón de $0.50 por minuto (c) Los dos segmentos represen-
2
a
tan diferentes cargos por minuto. El que representa el ma-
3
4
b yor costo es el más empinado. (d) C1t2 = 0.50t si
5 0 … t … 60, C1t2 = 30 + 0.101t - 602 si t 7 60.
11. (a) 5n - 2 (b) 3n 12. (a) 30 (b) 65
13. (a) Los primeros tres son. (b) Sólo (ii) y (iii).
4. (a)
(c) Los primeros tres son. 14. (a) 2 # 1 + 2 # 7 = 16;
0 1
1 5
2 7 2 # 2 + 2 # 6 = 16; 2 # 6 + 2 # 2 = 16; 2 # 5 + 2 # 5 = 20
3 3
(b) 511, 92, 12, 82,13, 72, 14, 62, 15, 52, 16, 42, 17, 32,
18, 22, 19, 126 (c) El dominio es N * N, y el rango es el
4 9

(b) 510, 12, 11, 32, 12, 52, 13, 72, 14, 926
conjunto de todos los números mayores o iguales a 4.
15. (a) 50 carros (b) entre 6:00 a.m. y 6:30 a.m. (c) 0
(c) x f (x) (d) 10
f (x) (d) entre 8:30 a.m. y 9 a.m.., decreció en 100 carros
9 (e) Usamos segmentos porque los datos son continuos en
0 1 8
lugar de discretos. Por ejemplo, hay un número de carros a
7
1 3 6 las 5:20 a.m.16. (a) A122 = 192; A162 = 192; A132 = 240;
2 5 5
A152 = 240. Algunas de las alturas corresponden a la bola
4
3 7 3 yendo hacia arriba y otras hacia abajo.
2
4 9 1 (b) Altura como función del tiempo

0 x 300
1 2 3 4 5 6 7 8
250
5. (a) Ésta es una función. (b) Ésta es una función.
200
(c) Ésta es una función.
Pies

150
6. (a) i. A B ii. A B
1 100
1 1 1
2
2 3 2 2 50
4 3 3
3 0
5
4 6 4 4 1 2 3 4 5 6 7 8
7 Segundos
8
Con t = 4 s, la altura de la pelota es A142 = 256 pies por enci-
(b) La parte (i) es una función de A a B. Para cada elemento ma del piso. (c) 8 s (d) 0 … t … 8 (e) 0 … A1t2 … 256
de A, hay un único elemento en B. El rango de la función es 17. (a) i. 40
52, 4, 6, 86. ii. 49
7. (b) i. S1n2 = 2n1n + 12
ii. S1n2 = 1n + 122 + 1n + 22n ó 2n2 + 4n + 1
Dosis como función del peso
125
110 n1n + 12
95 18. (a) (b) 4n - 1 19. El recíproco es falso.
(mg)

2
Por ejemplo, el conjunto de pares ordenados 5(1, 2), (1, 3)6 no
80
65
50 representa una función, ya que el elemento 1 está relacionado
20 30 40
con dos diferentes segundas componentes. 20. Sólo (b) no
(lb)
representa una función; si x, la entrada, es cualquier número
8. (a) 8 dólares (b) 3n + 2 dólares 9. (a) L1n2 = 2n + completo, entonces y no es único, ya que puede ser cualquier
1n - 12, ó 3n - 1 (b) L1n2 = n2 + 1 número completo y tal que y 7 x - 2. 21. Sólo (b) no
10. (a) es. Para x = 1 hay cinco valores de y. 22. (a) niños: B,
H; niñas : A, C, D, G, I, J, E, F (b) 51A, B2,
Costo por minuto
$0.50

1A, C2, 1A, D2, 1C, A2, 1C, B2, 1C, D2, 1D, A2,1D, B2,
Costo total por las llamadas

1D, C2, 1F, G2, 1G, F2,1I, J2, ( J, I ), (E, H )6 (c) no


$0.40 $40.00

$0.30 $30.00

$0.20 $20.00

$0.10 $10.00

0 10 20 30 40 50 60 70 80 90 100 0 10 20 30 40 50 60 70 80 90 100
Número de minutos Número de minutos

© Lopez Mateos Editores. ISBN 978-607-95583-2-1, obra completa, versión electrónica, ISBN 978-607-95583-3-8, volumen 1, versión electrónica. Ejemplar asignado a: Helecto Villarroel gutierrez -
helecto@gmail.com. Fecha: 27 de octubre de 2014. Prohibida su modificación, copia o distribución.
R22 Respuestas a los problemas

Conexiones matemáticas 4-3 13. (a) Esto no es una función, ya que un estudiante puede
Comunicación tener dos especialidades. (b) Esto es una función. El rango
es el subconjunto de números naturales que incluye el número
1. Sí, pues cada elemento de A está relacionado con exacta- de páginas de cada libro de la biblioteca. (c) Esto es una fun-
mente un elemento de B 3. (a) Esto no es una función, ya ción. El rango es 56, 8, 10, 12, Á 6. (d) Esto es una función.
que un maestro de la universidad puede enseñar a más de un El rango es 50, 16. (e) Esto es una función. El rango es N.
grupo. (b) Esto sí es función (suponiendo que a cada maes- 14. (a) C1x2 = 200 + 551x - 12
tro le corresponde un grupo). (c) Esto no es función, ya
(b) Costo de la membresía como función del tiempo
que no todo senador está relacionado con una comisión. (No 900

todo senador preside un comité.) 800

700
Solución abierta

Costo (en dólares)


600
5. Las respuestas pueden variar. 7. Las respuestas pueden 500
variar. 9. Las respuestas pueden variar. 400

Aprendizaje colectivo 300

200
11. Las respuestas pueden variar.
100

Preguntas del salón de clase


1 2 3 4 5 6 7 8 9 10 11 12 13
Meses
13. No toda función es una sucesión. Considera, por ejem-
plo, la función cuyas entradas son estudiantes de la universi- (c) En el noveno mes, el costo excede $600. (d) en el mes
dad y cuyas salidas son los números de identificación de cada número 107 15. 5 16. (a) Sí, cada entrada tiene exac-
estudiante. 15. Una manera es mostrar que para una tamente una salida. (b) No, para x = 4, hay dos valores
salida dada y existe una única entrada. Si y = 3x + 5 y y es para y. (c) No, para x = 5, hay dos valores para y.
y - 5 17. (a) 14, 18, 22, 26 (b) La gráfica consiste en todos los
la salida, entonces la única entrada es . Otro enfoque
3 puntos sobre la recta y = 4x + 2 con x = 1, 2, 3, 4, Á .
es escribir la sucesión de salidas 5, 8, 11, 14, 17, Á . Enton- (c) y = 4x + 2 (d) La gráfica consiste en algunos puntos
ces al 0 le corresponde el 5, al 1 el 8, etc. sobre una línea recta pero no todos los puntos de ésta, y por
Problemas de repaso lo tanto no es una línea recta.
y
17. (a) 50 (b) 2 (c) 100 (d) 3 22
20
18
Número de cuadrados

Resumen del capítulo 16


14
12
1. S = 13P 2. La cantidad de niñas es 103 veces la can- 10
tidad de niños. 3. f = 3y 4. 10S - 10n 5. 26 8
6
6. (a) Si n es el número original, entonces cada uno de los si- 4
2
guientes renglones muestra el resultado de una instrucción:
0 1 2 3 4 5 x
n Número de cubos
n + 17 Respuestas a Ahora intenta éste
21n + 172 = 2n + 34
4-1. (a) Si quitamos los cuatro mosaicos blancos de las esqui-
2n + 30
nas, entonces los mosaicos blancos que quedan en un lado
4n + 60
igualan en número a los mosaicos sombreados, i.e., n. En los
4n + 80
cuatro lados tenemos 4n. sumando los cuatro mosaicos de las
n + 20
esquinas tenemos 4n + 4. (b) (i) 206 (ii) 2n + 6 4-2. (1)
n
al calcular primero 2 # 50 (2) escribiendo 70 # 50 = 50 # 70
(b) Las respuestas pueden variar; por ejemplo, los (3) no, los cálculos dentro de los paréntesis deben hacerse
siguientes dos renglones podrían decir “resta 65” y “divide primero 4-3. (a) n = 3, ^ = 9 (b) n = 4, ^ = 2
entre 4”. (c) Las respuestas pueden variar. 6s
7. (a) 12 (b) 29 (c) 3 (d) No hay solución (e) Todo 4-4. Para hallar s, tenemos = s 4-5. 1. libras que aumentó
6
número es una solución. 8. Pati 111, Juan 222 y Miguel 2. 4 + p = 8, p = 4 4-6. (1) Si Brenda entregó b periódicos
666 9. Los libros de ciencias 17 días, los otros libros entonces Abel entregó 3b periódicos y Carla 3b + 13. Por lo
3 días 10. Raquel 50, Dalia 150 y Jacobo 300 tanto,
11. (a) función (b) no es una función (c) función b + 3b + 3b + 13 = 496,
12. (a) rango = 53, 4, 5, 66 (b) rango = 514, 29, 44, 596 b = 69
(c) rango = 50, 1, 4, 9, 166 (d) rango = 55, 9, 156 a = 3b = 207
c = 3b + 13 = 220.

© Lopez Mateos Editores. ISBN 978-607-95583-2-1, obra completa, versión electrónica, ISBN 978-607-95583-3-8, volumen 1, versión electrónica. Ejemplar asignado a: Helecto Villarroel gutierrez -
helecto@gmail.com. Fecha: 27 de octubre de 2014. Prohibida su modificación, copia o distribución.
Respuestas a los problemas R23

(2) x + y = 8 (b)
z + y = 9
– – +
– +
– +
z + x = 7
Restando la tercera ecuación de la segunda, obtenemos
y - x = 2. Con y + x = 8 sumamos ambas ecuaciones – 2 cargas Suma 3 cargas positivas.
miembro a miembro y obtenemos 2y = 10; y = 5. Por lo Resultado neto: 1 carga positiva

tanto, x = 3 y z = 4. 4-7. (a) Es una función que va del (c)


conjunto de números naturales al 506, ya que para cada
número natural existe una única salida en 506. (b) Es una – – – +
– +
– –
función que va del conjunto de números naturales al 50, 16,
ya que por cada número natural existe una única salida en
50, 16. 4-8. (4) Ellos están en la misma línea recta.
– 3 cargas Suma 2 cargas positivas.
Resultado neto: 1 carga negativa
(5) Dibuja una recta que pase por los puntos y extiéndela.
(d)
El punto (6, 20) está en la recta. (6) No, no está en la recta.
4-9. 55, 10, 15, 20, 256 4-10. (a) niñas: A, C, D, F, G, I; – – – – – – – –
niños: B, J (b) E y H 4-11. (a) Para x 7 10, y no es
un número completo. El rango se obtiene al substituir
– 3 cargas Añade 2 cargas negativas.
x = 0, 1, 2, 3, Á , 10 en y = 10 - x. (b) Si x y y se Resultado neto: 5 cargas negativas
intercambian, obtenemos x = y + 10 ó y = x - 10, el cual
es diferente de y = x + 10. 5. (a) 5 + –3 –3
5

Respuesta al Rincón de la tecnología –3 –2 –1 0 1 2 3 4 5 6 7 8

Sección 4-3 (b) +3


–2 –2 + 3
Las respuestas pueden variar. Las gráficas son rectas parale-
–3 –2 –1
las porque cualesquier dos ecuaciones con valores diferentes 0 1 2 3 4 5 6 7 8

de b no tienen una solución común. (c) +2


–3 + 2
–3

Respuesta al Problema preliminar –3 –2 –1 0 1 2 3 4 5 6 7 8

Si el número que el estudiante escogió es x y la respuesta es (d) –2


–3 + –2
–3

a, entonces
–7 –6 –5 –4 –3 –2 –1 0 1 2 3 4 5 6 7

2# a + 5 b - 18 = a
6x + 4
6. (a) 3 + - 1 - 22 = 5 (b) - 3 + - 2 = - 5
(c) - 3 + - 1 - 2) = - 1 7. (a) 3 - 1 - 22 = x si, y sólo si,
2
6x - 4 = a
3 = - 2 + x. Entonces, x = 5. (b) - 3 - 2 = x si, y sólo
si, - 3 = 2 + x. Por lo tanto, x = - 5.(c) - 3 - 1 - 22 = x
6x = a + 4
a + 4
x = si, y sólo si, - 3 = - 2 + x. Por lo tanto, x = - 1.
6
8. (a) - 17 + 10 = - 7 puntos (b) - 10°C + 8°C = - 2°C
(c) 5000 pies + 1 - 100 pies2 = 4900 pies
Así, el maestro sumó 4 a cada respuesta y dividió la suma
9. (a) 1 - 452 + 1 - 552 + 1 - 1652 + 1 - 352 +
entre 6.
1 - 1002 + 75 + 25 + 400 (b) $400
10. (a) –(–1)
Capítulo 5 –4

Evaluación 5-1A –4 –3 –2 –1 0 1 2 3 4 5 6 7

(b) –(–3)
1. (a) - 2 (b) 5 (c) - m (d) 0 (e) m (f ) - a - b –4

o - 1a + b) 2. (a) 2 (b) m (c) 0 3. (a) 5


(b) 10 (c) - 5 (d) - 5
–10 –9 –8 –7 –6 –5 –4 –3 –2 –1 0 1 2

4. (a) 11. (a) - 4 - 2 = - 6; - 4 - 1 = - 5; - 4 - 0 = - 4;


- 4 - - 1 = - 3 (b) 3 - 1 = 2; 2 - 1 = 1; 1 - 1 = 0;
+ + + + + –+ –+ +
– + + 0 - 1 = - 1; - 1 - 1 = - 2; - 2 - 1 = - 3 12. (a) - 9
(b) 3 (c) 1 13. (a) (i) 55 - 60 (ii) 55 + 1 - 602
5 cargas + Añade 3 cargas negativas. (iii) - 5°F (b) (i) 200 - 220 (ii) 200 + 1 - 2202
(iii) $ - 20 14. (a) 10W–40 ó 10W–30 (b) 5W–30
Resultado neto: 2 cargas positivas

(c) 10W–40, 5W–30, ó 10W–30 (d) ninguno (e) 10W–30


ó 10W–40 15. (a) 1 + 4x (b) 2x + y 16. La

© Lopez Mateos Editores. ISBN 978-607-95583-2-1, obra completa, versión electrónica, ISBN 978-607-95583-3-8, volumen 1, versión electrónica. Ejemplar asignado a: Helecto Villarroel gutierrez -
helecto@gmail.com. Fecha: 27 de octubre de 2014. Prohibida su modificación, copia o distribución.
R24 Respuestas a los problemas

ecuación es válida si, y sólo si, c = 0 (a y b pueden ser enteros $80 en otra, teniendo un total de $84. Gastas $27 quitando
cualesquiera). Justificación: Se puede mostrar que a - 1b - $7 de la primera cuenta y $20 de la segunda. La primera
c2 = a - b + c para todos los enteros a, b y c. Así, la ecua- cuenta está sobregirada en $3; esto es, el saldo es - $3. El
ción original es válida si, y sólo si, 1a - b2 + - c = saldo en la segunda cuenta es $60. Después de transferir $3de
1a - b2 + c, lo cual, a su vez, es válido si, y sólo si, - c = c. la segunda a la primera, el saldo en la primera cuenta es $0 y
Esta última ecuación es verdadera si, y sólo si, c = 0 en la segunda $57; esto es, el saldo total es $57. 17. La
17. (a) E (b) C (c) E - 506 (d) ⭋ (e) ⭋ (f) E - figura se supone que ilustra el hecho de que un entero y su
18. (a) 0 (b) -101 (c) 1 (d) a - 1 (e) - 4 19. (a) opuesto son reflejos en un espejo. Como a puede ser
Todos los enteros negativos (b) Todos los enteros positi- negativo, la figura es correcta. Por ejemplo, los valores posi-
vos (c) Todos los enteros menores que - 1 (d) 2 ó - 2 bles para a y - a son a = - 1, - a = 1, y a = - 7, - a = 7. En
20. (a) 9 (b) 2 (c) 0 y 2 (d) el conjunto de los núme- este punto, el maestro puede recordar al alumno que el signo
ros completos 21. (a) 0 ó 12 (b) - 8 u 8 (c) Todo “ - ” en - a no significa que - a es negativo. Si a es positivo,
entero satisface esta propiedad. 22. (a) 89 (b) 19 - a es negativo, pero si a es negativo, - a es positivo.
23. El mayor valor posible: a - 1b - c2 - d, u 8; el menor
valor posible: a - b - 1c - d2, ó - 6 24. (a) d = - 3, tér- Evaluación 5-2A
minos siguientes: - 12, - 15 (b) d = - y, términos siguien-
1. 31 - 12 = - 3; 21 - 12 = - 2; 11 - 12 = - 1; 01 - 12 = 0;
tes: x - 2y, x - 3y 25. - 1 26. (a) verdadero
1 - 121 - 12 = 1, continuando el patrón de una sucesión arit-
(b) verdadero (c) verdadero 27. (a) - 4 (b) 3
mética con la diferencia fija de 1.
(c) - 5 28. (a) -18 (b) - 6 (c) 22 (d) - 18 (e) 23
2.
Conexiones matemáticas 5-1 –+ –+ +
– +
– +– +– + + + + + +
+
– +
– + +
Comunicación
0 cargas Quita 4 grupos de 2 cargas negativas.
1. Pudo haber manejado 12 mi en cualquier dirección a par- El resultado neto es de 8 cargas positivas.

tir del poste de la milla 68. Por lo tanto, su ubicación podría 3. 2(–4) = –8
ser el poste 68 - 12 = 56 mi o el poste 68 + 12 = 80 mi. –4 –4

3. Una manera es encontrar la diferencia del valor absoluto –8 –7 –6 –5 –4 –3 –2 –1 0 1 2 3 4 5 6 7


mayor y el valor absoluto menor. La suma tiene el mismo
signo que el del valor absoluto mayor. 4. (a) 1 - 321 - 32 = 9 (b) 1 - 522 = - 10
5. (a) No; si x 6 0, entonces - x es positivo. 5. (a) 41 - 202 ó - 80 (b) 1 - 421 - 202 u 80
(b) - 1a - b - c2 = - 1a + - b + - c2 (c) n1 - 202 ó - 20n (d) 1 - n21 - 202 ó 20n
= - a + - 1 - b2 + - 1 - c2 6. (a) 5 (b) - 11 (c) Imposible; debido a que
= -a + b + c = b + c - a 0 # k = - 5 no tiene solución entera. 7. (a) - 10
7. a 6 b si, y sólo si, existe un número completo distinto de (b) - 10 (c) No está definida (d) No está definida
cero c tal que a + c = b. - 8 6 - 7 ya que - 8 + 1 = - 7. (e) - 1 8. (a) - 30; - 30 , - 6 = 5; - 30 , 5 = - 6
(b) 20; 20 , - 5 = - 4; 20 , -4 = - 5 (c) 0,
Solución abierta 0 , - 3 = 0; la división entre 0 no está definida. (d) 0,
9. Las respuestas pueden variar; por ejemplo, los pisos por la división entre 0 no está definida. 9. (a) 14x2 , 4 = a
arriba de la superficie se podrían numerar como de costumbre si, y sólo si, 4x = 4a si, y sólo si, a = x. Así, 14x2 , 4 = x
1, 2, 3, 4, Á , n, el piso cero o a nivel de la superficie se podría (b) - xy , y = a si, y sólo si, - xy = ya = ay si, y sólo si,
llamar PB (Planta baja) y los pisos debajo de la superficie, o en a = - x 10. Todas las respuestas están en °C. (a) 32 +
el sótano, se podrían llamar 1S, 2S, 3S, 4S, Á , mS. El siste- 1 - 3230 ó 32 - 3 # 30 = - 58 (b) 0 + 1 - 421 - 252
ma se podría modelar con una recta numérica vertical con PB ó 4 # 25 = 100 (c) - 20 + 1 - 421 - 302 = 100
en lugar del 0 y 1S, 2S, 3S, Á , mS en lugar de los números (d) 25 + 31 - 202 = 25 - 3 # 20 = - 35 11. 108,000
negativos. 11. (a) Las respuestas pueden variar. Por ejem- acres 12. (a) - 11 - 5 + - 22 = - 11 - 72 = 7;
plo, f 1x2 = - ƒ x ƒ - 1. (b) Las respuestas pueden variar. 1 - 121 - 52 +( - 1)1 - 22 = 5 + 2 = 7 (b) - 31 - 3 + 22 =
Por ejemplo, f 1x2 = ƒ x ƒ . - 31 - 12 = 3; 1 - 321 - 32 + 1 - 32122 = 9 + - 6 = 3
13. (a) - 8 (b) 16 (c) - 1000 (d) 81 (e) 1 (f) - 1
Aprendizaje colectivo
(g) 12 (h) 0 14. (a) 0 (b) 0 (c) 9 15. (b) y (c)
13. Las respuestas pueden variar. siempre son positivas, (a) siempre es negativa. (d) y (e) no son
Preguntas del salón de clase ninguna 16. 1b2 = 1c2; 1d2 = 1e2 17. (a) Propiedad
conmutativa de la multiplicación (b) Propiedad de la cerra-
15. El algoritmo es correcto, y al estudiante hay que felici- dura de la multiplicación (c) Propiedad asociativa de la
tarlo por haberlo hallado. Una manera de alentar su creativi- multiplicación (d) Propiedad distributiva de la multiplicación
dad es referirse al procedimiento y nombrarlo por su nom- sobre la suma 18. (a) xy (b) 2xy (c) 3x - y (d) - x
bre; por ejemplo, “El método de resta de David”. En cuarto 19. (a) - 2x + 2y (b) x2 - xy (c) - x2 + xy
grado, la técnica se puede explicar usando un modelo de di- (d) - 2x - 2y + 2z
nero. Supongamos que tienes $4 en una cuenta de cheques y
© Lopez Mateos Editores. ISBN 978-607-95583-2-1, obra completa, versión electrónica, ISBN 978-607-95583-3-8, volumen 1, versión electrónica. Ejemplar asignado a: Helecto Villarroel gutierrez -
helecto@gmail.com. Fecha: 27 de octubre de 2014. Prohibida su modificación, copia o distribución.
Respuestas a los problemas R25

20. (a) - 2 (b) 2 (c) 0 (d) - 6 (e) - 36 (f ) 6 Aprendizaje colectivo


(g) Todos los enteros excepto 0 (h) Todos los enteros excepto 0
21. (a) - 5 (b) - 2 (c) No hay soluciones (d) - 2 13. (a) Las respuestas pueden variar. (b) Las respuestas
(e) - 2 y 2 (f ) - 2 ó 4 (g) - 1 (h) 1 ó - 3 pueden variar.
22. (a) 150 + 22150 - 22 = 502 - 22 = 2500 - 4 = Preguntas del salón de clase
2496 (b) 25 - 10,000 = - 9975 (c) x2 - y2 15. El estudiante tiene razón al decir que la deuda de $5 es
23. (a) 8x (b) x1y + 12 (c) x1x + y2 mayor que la deuda de $2. Sin embargo, lo que esto significa
(d) x13y + 2 - z2 (e) a3b1c + 12 - 14 en la recta real es que - 5 está más a la izquierda que - 2. El
(f ) 14 + a214 - a2 (g) 12x + 5y212x - 5y2 hecho de que - 5 esté más lejos a la izquierda que - 2 sobre
24. (a) 1a - b22 = 1a + - b21a + - b2 la recta real implica que - 5 6 - 2. 17. El procedimiento
= a1a + - b2 + - b1a + - b2 se puede justificar como sigue. Como para todos los enteros
= a2 + a1 - b2 + 1 - b2a + 1 - b21 - b2 c, - c = 1 - 12c, el hecho de realizar el opuesto de una expre-
= a2 - 2ab + b 2 sión algebraica es lo mismo que multiplicar la expresión
(b) (i) 982 = 1100 - 222 = 1002 - 212002 + 22 = por - 1. Sin embargo, en esta expresión x - 12x - 32, el sig-
10,000 - 400 + 4 = 9604 (ii) 992 = 1100 - 122 = no “ - ” denota la resta, no simplemente encontrar el opuesto.
1002 - 211002 + 12 = 10,000 - 200 + 1 = 9801 Si primero la expresión la escribimos como x + - 12x + - 32,
(iii) 9972 = 11000 - 322 = 10002 - 2130002 + 32 = entonces sucede - 12x + - 32 = - 112x + - 32, ó - 2x + 3.
1,000,000 - 6000 + 9 = 994,009 25. (a) 8, 11, d = 3, Ahora la expresión puede escribirse como x + - 2x + 3, la
el n-ésimo término es 3n - 13. (b) - 128, - 256, r = 2, cual el estudiante pudo obtener usando la regla de su padre.
el n-ésimo término es - 2n. (c) 27, - 28, r = - 2, el n- 19. El modelo muestra que - 21 - 32 = 6, pero esto no mues-
ésimo término es 2 # 1 - 22n - 1 ó - 1 - 22n. 26. 13,850 tra en general que la multiplicación entre enteros negativos
27. (a) - 9, - 6, - 1, 6, 15 (b) - 2, - 7, - 12, - 17, - 22 sea un entero positivo. Una demostración formal que demues-
(c) - 3, 3, - 9, 15, - 33 28. El primer término es 7 y el se- tre que la multiplicación entre dos números enteros negativos
gundo es 2 29. Después de17 min, la temperatura era de es un entero positivo para todos los enteros es deseable.
-108°C 30. (a) Si x Ú 0 y y … 0, entonces Problemas de repaso
- ƒ x ƒ # ƒ y ƒ = - x1 - y2 = xy. De manera análoga, la proposi-
ción es verdadera para x … 0 y y Ú 0. (b) Es verdadero 21. (a) 5 (b) - 7 (c) 0 23. 400 lb 25. (a) x = 3
si, y sólo si, x = 0. Si x Z 0, - x2 es negativo y x2 es ó - 3 (b) No hay x posible (c) x Ú 0 (d) x … 0
positivo y por tanto las expresiones no pueden ser
Evaluación 5-3A
iguales. (c) Si x y y son positivas y x 7 y, la proposición
es verdadera. 1. (a) verdadero (b) verdadero (c) verdadero (d) verdadero
(e) verdadero (f ) Falso; no hay ningún valor c 僆 E tal que
Conexiones matemáticas 5-2 30c = 6. 2. (a) sí (b) no (c) sí 3. (a) 2, 3, 4, 6, 11
Comunicación (b) 2, 3, 6, 9 (c) 2, 3, 5, 6, 10 4. (a) No, 17 ƒ 34,000 y
1. No; no es de la forma 1a - b21a + b2.
17ⱈ15, de modo que 17ⱈ34,015. (b) Sí, 17 ƒ 34,000 y 17 ƒ 51,

3. (a) 1 - 12a + a = 1 - 12a + 1 # a


de modo que 17 ƒ 34,051. (c) No, 19 ƒ 19,000 y 19ⱈ31, de
modo que 19ⱈ19,031. (d) Sí, 5 es un factor de 2 # 3 # 5 # 7.
= 1 - 1 + 12a (e) No, 5 ƒ 2 # 3 # 5 # 7 y 5ⱈ1, de modo que 5ⱈ12 # 3 # 5 # 72 + 1.
= 0#a 5. (a) Verdadero por el teorema 5–12 (b) Verdadero por
= 0 el teorema 5–13(b) (c) ninguno (d) Verdadero por el teo-
(b) La parte (a) implica que ( - 1) es el inverso aditivo de a. rema 5–13(b) (e) Verdadero por el teorema 5–12 6. (a)
Como - a es también el inverso aditivo de a, se sigue que Falso, 2|6 pero 12 + 52ⱈ16 + 52 (b) Verdadero. Como b ƒ a,
1 - 12a = - a.
existe c tal que a = bc. Entonces a3 = b3c3 = 1bc32b2, lo que
5. - 1a + b2 = 1 - 121a + b2 Problema 31b2 significa que b2 ƒ a3. (c) Verdadero. Como b ƒ a, existe c tal
= 1 - 12a + 1 - 12b Por la propiedad distributiva que a = bc. Entonces - a = - 1bc2 = b1 - c2, lo que implica que
= -a + -b Problema 31b2 b ƒ - a. También, - a = - 1bc2 = 1 - b2c, lo que implica - b ƒ - a.
7. (a) 5x + 3 6 - 20 si, y sólo si, 5x 6 - 23. Como 7. (a) 210 = 7 # 30 (b) 19 ƒ 1900 y 19 ƒ 38 (c) 6 ƒ 12 # 32 #
51 - 52 = - 25 6 - 23 y 51 - 42 = - 20 7 - 23, el mayor 22 # 3 # 174 y 6 ƒ 6 # 24 # 3 # 174 (d) 7 ƒ 4200 pero 7ⱈ22 8. (a)
entero es - 5. (b) No, no hay un entero mínimo. Como verdadero (b) falso 9. (a) 7 (b) 7 (c) 6 10. (a)
x es un entero, la desigualdad es equivalente a x … - 5 y no Cualquier dígito del 0 al 9 (b) 1, 4, 7 (c) 1, 3, 5, 7, 9
hay tal entero mínimo. (d) 7 (e) 7 11. 17 12. Cada lápiz costó 19¢. 13. (a) sí
Solución abierta (b) no (c) sí (d) no 14. (a) 12,343 + 4546 + 56 =
9. Las respuestas pueden variar. 11. Las respuestas va- 16,945; 4 + 1 + 2 = 7 tiene residuo 7 cuando dividimos
rían dependiendo del nivel o grado y la selección del libro entre 9, como también 1 + 6 + 9 + 4 + 5. (b) 987 +
publicado. 456 + 8765 = 10,208; 6 + 6 + 8 = 20 tiene residuo 2 al
dividir entre 9, como también 1 + 0 + 2 + 0 + 8 = 11.

© Lopez Mateos Editores. ISBN 978-607-95583-2-1, obra completa, versión electrónica, ISBN 978-607-95583-3-8, volumen 1, versión electrónica. Ejemplar asignado a: Helecto Villarroel gutierrez -
helecto@gmail.com. Fecha: 27 de octubre de 2014. Prohibida su modificación, copia o distribución.
R26 Respuestas a los problemas

(c) 10,034 + 3004 + 400 + 20 = 13,458; 8 + 7 + = a11001 # 1002 + b11001 # 102 + c1001
4 + 2 = 21 tiene residuo 3 cuando lo dividimos entre 9; = 1100123a100 + b10 + c14
también 1 + 3 + 4 + 5 + 8. (d) 1003 - 46 = 957; = 17 # 11 # 1323a100 + b10 + c14
4 - 1 = 3 tiene residuo 3 cuando lo dividimos entre 9, como
Así, si divides entre 1001, el cociente es abc.
también 9 + 5 + 7 = 21. (e) 345 # 56 = 19,320. 345 tiene
9. (a) Sea abcd un número. Restando el último dígito tene-
residuo 3 cuando lo dividimos entre 9; 56 tiene residuo 2
mos abc0. Así, el resultado es divisible entre 2, 5 y 10.
cuando lo dividimos entre 9; 3 # 2 = 6 tiene residuo 6 cuando
(b) Si restamos cd de abcd, obtenemos ab00. Este número es
lo dividimos entre 9. 1 + 9 + 3 + 2 + 0 = 15 tiene residuo
divisible entre 2, 4, 5, 10, 20, 25, 50 y 100.
(c) abcd - 1a + b + c + d2 = a # 103 + b # 102 + c # 10
6 cuando lo dividimos entre 9. (f ) Las respuestas pueden
variar. 15. (a) 1, 3 y 7 dividen a n. 1 divide a todo número.
+ d - a - b - c - d = a # 999 + b # 99 + c # 9 = 91a #
También, como n = 21 # d, d 僆 E, entonces
111 + b # 11 + c2. Así, el resultado es divisible entre 3 y 9.
n = 13 # 72d = 317d2 = 713d2, lo cual implica que n es divi-
sible entre 3 y entre 7. (b) 1, 2, 4 y 8 dividen a n. 1 divide a (d) • Considera un número de cuatro dígitos en base cinco,
todo número. Como, 16 ƒ n, entonces n = 16 # d, d 僆 E. Así, abcdcinco. Después restamos el último dígito y obte-
n = 12 # 82d = 218d2 = 812d2 = 414d2. Por lo tanto, n es di- nemos abc0cinco = 10cinco # abccinco, el cual es divisi-
visible entre 2, 4 y 8. ble entre 10cinco ó 5 en base diez.
16. (a) Sí, si 5 ƒ x y 5 ƒ y, entonces 5 ƒ 1x + y2. (b) Sí, si 5 ƒ y,
• abcdcinco - cdcinco = ab00cinco, el cual es divisible en-
entonces 5 ƒ 1 - y2. Si 5 ƒ x y 5 ƒ 1 - y2, entonces 5 ƒ 1x + 1 - y22 ó
tre 10cinco y 100cinco ó 5 y 25 en base diez.
• abcdcinco - a - b - c - d = 444cinco # a + 44cinco # b +
4cinco # c = 4cinco1111cinco # a + 11cinco # b + c2, el
5 ƒ x - y. (c) Sí, si 5 ƒ x, entonces 5 divide a todos los múlti-
plos de x y en particular 5 ƒ xy. 17. 6,868,395 es divisible en-
cual es divisible entre 2 y 4.
tre 15 porque es divisible entre 3 y 5. El último dígito es 5
y la suma de los dígitos es 45, el cual es divisible entre 3. Solución abierta
18. (a) Falso; 2 ƒ 4, pero 2ⱈ1 y 2ⱈ3. (b) Falso (mismo
11. (a) Las respuestas pueden variar; por ejemplo, por ins-
ejemplo que en (a)) (c) Falso; 12 ƒ 72, pero 12ⱈ8 y 12ⱈ9.
pección de los números dados observamos que todos los nú-
(d) Verdadero (e) Falso; si a = 5 y b = - 5, entonces a ƒ b
meros son múltiplos de 3. Como 3 divide a cada número,
y b ƒ a, pero a Z b.
entonces 3 divide a la suma de cualquiera de esos números.
19. Sea n = a104 + b103 + c102 + d10 + e.
Como 100 no es divisible entre 3, no hay una combinación
a104 = a110,0002 = a19999 + 12 = a 9999 + a
ganadora de los números dados que sume 100. (b) Las res-
b103 = b110002 = b1999 + 12 = b 999 + b
puestas pueden variar; por ejemplo, como muchos de estos
c102 = c11002 = c199 + 12 = c 99 + c múltiplos de 3 suman 99 (33 + 66, 45 + 51 + 3, etc.), la
d10 = d1102 = d19 + 12 = d 9 + d compañía podría colocar a lo sumo 1000 tarjetas con el nú-
Así, n = 1a 9999 + b 999 + c 99 + d 92 + 1a + b + mero 1 en la caja. Esto asegurará que no haya más de 1000
c + d + e2. Como 9 ƒ 9, 9 ƒ 99, 9 ƒ 999, 9 ƒ 9999, se sigue que si ganadores. También se podrían usar otros números.
9 ƒ 1a + b + c + d + e2 entonces 9 ƒ 31a 9999 + b 999 + c 99 Aprendizaje colectivo
+ d 92 + 1a + b + c + d + e24; esto es, 9 ƒ n. Si, por otro
lado, 9ⱈ1a + b + c + d + e2, se sigue que 9ⱈn. 13. Las respuestas pueden variar.
Conexiones matemáticas 5-3 Preguntas del salón de clase
Comunicación 15. Aunque estas dos expresiones se ven similares, no son
1. No, cualquier cantidad de timbres debe ser un múltiplo de iguales; a>b significa “a está dividido entre b,” una operación, y
3 (siendo la suma de un múltiplo de 6 y otro de 9, ambos son tiene una respuesta numérica si b Z 0; a ƒ b significa “a divide a
múltiplos de 3). b”, una relación que puede ser verdadera o falsa. Nota que si
3. (a) Sí, 4 ƒ 52,832, de modo que 4 divide a cualquier entero que a>b es un entero, entonces b ƒ a. 17. Sí, si a Z 0; la conclu-
multiplica a 52,832. Por lo tanto, 4 divide a 52,832 # 324,518, sión del estudiante es que a ƒ 0 y esto no es verdadero pues
que es el área. (b) Sí, 2 es un factor de 52,834 y 2 es un factor de a Z 0, la ecuación a # k = 0 tiene una solución única, k = 0.
324,514, de modo que 2 # 2 ó 4 es un factor de 52,834 # 324,514, 19. Se ha visto que para cualquier número n de cuatro dígitos
el cual es el área. 5. (a) No. Si 16|x, entonces x = 10n = 5 # 2n este puede ser escrito de la forma n = a103 +
para cualquier entero n. Por lo tanto x es divisible entre 5. (b) Sí, b102 + c10 + d = 1a999 + b99 + c92 + 1a + b + c + d2.
todos los múltiplos impares de 5 no son divisibles entre 10 pero La prueba de divisibilidad entre algún número g dependerá de
son divisibles entre 5. 7. 243; Sí; Considera cualquier nú- la suma de los dígitos a + b + c + d si, y sólo si,
mero n de la forma abcabc. Entonces tenemos lo siguiente: g ƒ 1a 999 + b 99 + c 92 independientemente de los valores de
a, b y c. Como los únicos números mayores que 1 que divi-
n = 1a1052 + 1b1042 + 1c1032 + 1a1022 den a 9, 99 y 999 son 3 y 9, la prueba de divisibilidad para
+ 1b1012 + c dividir la suma de los dígitos entre el número funciona sólo
= a1105 + 1022 + b1104 + 1012 + c1103 + 12 para 3 y 9. Un argumento similar funciona para cualquier
= a1100,000 + 1002 + b110,000 + 102 + c1001 número con m-dígitos.

© Lopez Mateos Editores. ISBN 978-607-95583-2-1, obra completa, versión electrónica, ISBN 978-607-95583-3-8, volumen 1, versión electrónica. Ejemplar asignado a: Helecto Villarroel gutierrez -
helecto@gmail.com. Fecha: 27 de octubre de 2014. Prohibida su modificación, copia o distribución.
Respuestas a los problemas R27

21. Es verdad que si un número es divisible entre 24 es divisible 3 ƒ n y 2 y 3 son primos, deben estar incluidos en la factoriza-
entre 6 y entre 4, y que, en general, un número divisible entre ab ción única:
es divisible entre ambos, a y b. El recíproco no es verdadero. Por 2 # 3 # p1 # Á # pm = n;
ejemplo, 12 es divisible entre 4 y entre 6 pero no entre 4 # 6, ó 24. 12 # 321p1 # p2 # Á # pm2 = n; por lo tanto,
Problemas de repaso 6 ƒ n.
23. (a) - 2 (b) No existen tales enteros. (c) No existen tales (b) Sí. Si a ƒ n, existe un entero c tal que ca = n. Si b ƒ n existe
enteros. (d) Todos los enteros (e) No existen tales un entero d tal que db = n. Por lo tanto,
enteros. (f ) Todos los enteros positivos 25. (a) 1ca21db2 = n2 Q 1dc21ab2 = n2 Q ab ƒ n2. 9. (a) 2, 3, 4, 6,
f 1 - 52 = 10 - 3 = 7 (b) - 10 (c) No, si - 2x - 3 = 2, 9, 12, 18 ó 36 (b) 1, 2, 4, 7, 14 ó 28 (c) 1 ó 17
entonces - 2x = 5, la cual no tiene solución en los enteros. (d) 1, 2, 3, 4, 6, 8, 9, 12, 16, 18, 24, 36, 48, 72 ó 144
(d) El conjunto de todos los enteros impares. 10. 90 11. 101, 103, 107, 109, 113, 127, 131, 137, 139,
149, 151, 157, 163, 167, 173, 179, 181, 191, 193, 197, 199
Evaluación 5-4A
12. 3, 5; 5, 7; 11, 13; 17, 19; 29, 31; 41, 43; 59, 61; 71, 73; 101,
1. 30 2. (a) primo (b) no es primo (c) no es primo 103; 107, 109; 137, 139; 149, 151; 179, 181; 191, 193; 197, 199
(d) primo (e) primo (f ) primo (g) primo
13. 1, 2, 3, 6, 7, 14, 21 14. Hay 16 factores de 1000. Los
(h) no es primo
otros 15 factores son: 1, 2, 4, 5, 8, 10, 20, 25, 40, 50, 100, 125,
3. (a) 504 (b) 2475
200, 250 y 500. 15. Hay una infinidad de números compues-
2 252 3 825 tos de la forma 1, 11, 111, 1111, 11111, 111111 Á pues cada
tercer número de la sucesión será divisible entre 3 (también,
2 126 3 275
cada dos de la sucesión es divisible entre 11). 16. Sí, porque
2 63 5 55 132 # 242132 # 232 = 34 # 27 17. (a) 3 # 5 # 7 # 11 # 13 es com-
3 21 5 11 puesto porque es divisible entre 3, 5, 7, 11 y 13.
(b) 13 # 4 # 5 # 6 # 7 # 82 + 2 = 2313 # 2 # 5 # 6 # 7 # 82 + 14 y,
3 7 por lo tanto, es compuesto. (c) 13 # 5 # 7 # 11 # 132 + 5 =
504 = 23 # 32 # 7 2475 = 32 # 52 # 11 5313 # 7 # 11 # 132 + 14 y por lo tanto es compuesto.
(d) 10! + 7 = 73110 # 9 # 8 # 6 # 5 # 4 # 3 # 2 # 12 + 14 de modo
(c) 11,250 que es compuesto. 18. 23 # 32 # 253 no es una factorización
2 5625 en primos porque 25 no es primo. La factorización en pri-
mos es 23 # 32 # 56. 19. (a) 235 # 335 # 740
3 1875 200 # 40 # 200
(b) 2 3 5 (c) 2 # 36 # 5110 (d) 2311 20. 73
3 625
Conexiones matemáticas 5-4
5 125
Comunicación
5 25
1. En cualquier conjunto de tres números consecutivos hay
5 5 un número que es divisible entre 3 y al menos uno de los
11,250 = 2 # 32 # 54 tres es divisible entre 2. Por lo tanto, el producto deberá ser
4. (a) 210 divisible entre 2 y entre 3 y, por lo mismo, entre 6.
2 105 3. No, no son correctos. Usar 3 y 4 está bien pues 1 es su
único divisor común. Pero 2 y 6 tienen a 2 como divisor co-
21 5 mún. Al usar este criterio sólo se asegura que el número sea
3 7 divisible entre 6. 5. Sean a = 2 # 3 # 5 # 7 y
b = 11 # 13 # 17 # 19. Entonces cada primo p menor o igual
que 19 aparece en la factorización en primos de a o de b
(b) Puedes multiplicar 2 # 3 # 7 # 5. 5. 73 pero no en ambas. Si p está en la factorización prima de a,
6. (a) 28 # 34 # 52 # 7 (b) 23 # 52 # 720 # 13 (c) 251 entonces p ƒ a pero pⱈb y, por lo tanto, pⱈa + b. Un argu -
(d) 7 # 11 # 13 7. (a) 1 entre 48, 2 entre 24, 3 entre 16 y mento similar vale si p está en la factorización en primos de
4 entre 12 (b) Sólo uno, 1 entre 47 8. (a) El Teorema b. 7. Supón que n es compuesto y d es su mínimo divisor
Fundamental de la Aritmética dice que n se puede escribir positivo distinto de 1. Necesitamos demostrar que d es primo.
como un producto de primos de manera única. Como 2 ƒ n y Si no es primo, entonces algún primo p menor que d dividirá a
d y por lo tanto dividirá a n, lo que contradice el hecho de que
d es el menor divisor de n mayor que 1.

© Lopez Mateos Editores. ISBN 978-607-95583-2-1, obra completa, versión electrónica, ISBN 978-607-95583-3-8, volumen 1, versión electrónica. Ejemplar asignado a: Helecto Villarroel gutierrez -
helecto@gmail.com. Fecha: 27 de octubre de 2014. Prohibida su modificación, copia o distribución.
R28 Respuestas a los problemas

Solución abierta Evaluación 5-5A

1. (a) D18 = 51, 2, 3, 6, 9, 186


9. (a) Las respuestas varían. (i) 25 (ii) 21 (b) 13, en
D10 = 51, 2, 5, 106
el intervalo 100–199 (c) (i) 8 (ii) 7 (d) Las respuestas
pueden variar. 10. (a) (i) 1 + 2 + 3 + 4 + 6 = 16, de
MDC118, 102 = 2
M18 = 518, 36, 54, 72, 90, Á 6
modo que 12 es abundante. (ii) 1 + 2 + 4 + 7 +
M10 = 510, 20, 30, 40, 50, 60, 70, 80, 90, Á 6
14 = 28, de modo que 28 es perfecto. (iii) 1 + 5 + 7 = 13,
de modo que 35 es deficiente. (b) Las respuestas pueden
MMC118, 102 = 90
(b) D24 = 51, 2, 3, 4, 6, 8, 12, 246
variar; por ejemplo, 10 y 14 son deficientes, 18 es
D36 = 51, 2, 3, 4, 6, 9, 12, 18, 366
abundante, y 496 y 8128 son perfectos.
Aprendizaje colectivo MDC124, 362 = 12
M24 = 524, 48, 72, 96, 120, 144, 168, Á 6
M36 = 536, 72, 108, 144, 180, Á 6
11. Los estudiantes deberán tener los primeros 23 números
primos para el número de losetas; esto es, 2, 3, 5, 7, 11, 13,
MMC124, 362 = 72
(c) D8 = 51, 2, 4, 86
17, 19, 23, 29, 31, 37, 41, 43, 47, 53, 59, 61, 67, 71, 73, 79,
D24 = 51, 2, 3, 4, 6, 8, 12, 246
83 losetas. Por lo tanto, el número de losetas es la suma de
D52 = 51, 2, 4, 13, 26, 526
los primeros 23 números primos, el cual es de 874 losetas.
Preguntas del salón de clase MDC18, 24, 522 = 4
13. Beto tiene casi la idea correcta pero se necesita un poco M8 = 58, 16, 24, 32, 40, 48, 56, 64, 72, 80, 88, 96, Á 6
de trabajo. Se deberá señalar que si un número no es divisi- M24 = 524, 48, 72, 96, 120, 144, 168, 192, 216, 240,
ble entre 2 y 3, entonces no puede ser divisible entre 6, de 264, 288, 312, Á 6
modo que no es necesario verificar el 6. También, si un nú- M52 = 552, 104, 156, 208, 260, 312, Á 6
mero no es divisible entre 2, entonces no puede ser divisible MMC18, 24, 522 = 312
entre 4 u 8, de modo que no se necesita verificar para 4 y 8. (d) D7 = 51, 76, D9 = 51, 96
Después, si un número no es divisible entre 5, entonces no MDC17, 92 = 1
puede ser divisible entre 10, por lo que no es necesario veri- MMC17, 92 = 7 # 9 = 63
ficar el 10. Así hemos recortado la lista de Beto en 2, 3 y 5, M7 = 57, 14, 21, 28, 35, 42, 49, 56, 63, Á 6
los cuales son todos primos. La criba de Eratóstenes la po- M9 = 59, 18, 27, 36, 45, 54, 63, Á 6
demos usar para motivar este concepto. Después necesitamos 2. (a) 132 = 22 # 3 # 11
explorar qué sucede cuando el número por verificar es mayor y 504 = 23 # 32 # 7
mostrar que sólo verificar 2, 3 y 5 no es suficiente. Por ejem- MDC1132, 5042 = 22 # 3 = 12
plo, verificar la divisibilidad entre 2, 3 y 5 no es suficiente MMC1132, 5042 = 23 # 32 # 7 # 11 = 5544
para verificar si 169 es primo. Podemos usar la criba para (b) 65 = 5 # 13
mostrar que si queremos determinar si un entero positivo n 1690 = 2 # 5 # 132
es primo, sólo necesitamos verificar los primos p tales que MDC165, 16902 = 5 # 13 = 65
p2 … n. 15. Sólo los cuadrados perfectos tienen un nú- MMC165, 16902 = 2 # 5 # 132 = 1690
mero impar de divisores. Los cuadrados perfectos menores (c) 96 = 25 # 3
que 1000 son 12, 22, 32, Á , 312. Por lo tanto, hay 31 900 = 22 # 32 # 52
cuadrados perfectos entre 1 y 1000 y por lo tanto hay 31 nú- 630 = 2 # 32 # 5 # 7
meros con un número impar de divisores. En consecuencia, MDC196, 900, 6302 = 2 # 3 = 6
hay 1000 - 31 = 969 números entre 1 y 1000 que tienen MMC196, 900, 6302 = 25 # 32 # 52 # 7 = 50,400
una cantidad par de divisores. 17. El estudiante tiene razón. (d) 108 = 22 # 33
En toda sucesión de seis números consecutivos mayores que 3, 360 = 23 # 32 # 5
sólo los números anterior y posterior a un múltiplo de 6 MDC1108, 3602 = 22 # 32 = 36
pueden ser primos. Por ejemplo, considera los números 17, MMC1108, 3602 = 23 # 33 # 5 = 1080
18, 19, 20, 21 y 22. 18, 20 y 22 son pares. 18 y 21 son múlti- 3. (a) MDC12924, 2202 = MDC1220, 642 =
plos de 3. Sólo 17 y 19, los números anterior y posterior a MDC164, 282 = MDC128, 82 = MDC18, 42 = MDC14, 02
18 (múltiplo de 6), pueden ser primos.Todos los demás son = 4 (b) MDC114595, 108562 = MDC110856, 37392
múltiplos de 2 ó 3. = MDC13739, 33782 = MDC13378, 3612 =
MDC1361, 1292 = MDC1129, 1032 =
Problemas de repaso
MDC1103, 262 = MDC126, 252 = MDC125, 12 = 1
19. (a) 2, 3, 6 (b) 2, 3, 5, 6, 9, 10 21. Sí, entre ocho 4. (a) 72 (b) 1440 (c) 630 (d) 9100 # 25100 ó
personas; cada uno obtiene $422. 3200 # 5200 ó 15200 5. (a) 220 # 2924>4, ó 160,820
(b) 14,595 # 10,856>1, ó 158,443,320
6. MDC16, 102 = 2, MMC16, 102 = 30

© Lopez Mateos Editores. ISBN 978-607-95583-2-1, obra completa, versión electrónica, ISBN 978-607-95583-3-8, volumen 1, versión electrónica. Ejemplar asignado a: Helecto Villarroel gutierrez -
helecto@gmail.com. Fecha: 27 de octubre de 2014. Prohibida su modificación, copia o distribución.
Respuestas a los problemas R29

7. (a) MMC115, 40, 602 = 120 min = 2 h, de modo que las Solución abierta
alarmas de los relojes sonarán juntas a las 8:00 a.m. (b) no
11. Las respuestas pueden variar. De la respuesta al
8. 24 9. 15 galletas 10. 36 min 11. Ellos deberán
problema 8 se sigue que MMC1a, b2 6 ab si, y sólo si,
pasar el punto de partida después de MMC112, 18, 162 =
MDC1a, b2 7 1; esto es, si, y sólo si, a y b tienen al menos
144 min. 12. (a) ab (b) MDC1a, a2 = a; MMC1a, a2 = a
un primo en común.
(c) MDC1a2, a2 = a, MMC1a2, a2 = a2 (d) MDC1a, b2 =
a; MMC1a, b2 = b 3. (a) Verdadero, si a y b son pares, Aprendizaje colectivo
entonces MDC1a, b2 Ú 2. (b) Verdadero, MDC1a, b2 = 2 13. Las respuestas pueden variar.
implica que a y b son pares. (c) Falso, el MDC podría ser
múltiplo de 2; por ejemplo, MDC18, 122 = 4. 14. (a) 15 Preguntas del salón de clase
(b) 1 15. 4 = 22. Como 97,219,988,751 es impar, no 15. No tiene sentido hablar del mínimo divisor común de
tiene factores primos de 2. En consecuencia, 1 es el único dos o más números ya que el mínimo divisor común de un
divisor común y son primos relativos. 16. El número 60 conjunto de números es siempre 1. De manera análoga, no
llamó 17. $48 18. Dos paquetes de platos, cuatro de va- tiene sentido hablar del máximo múltiplo común de un con-
sos y tres de servilletas junto de números ya que no hay un mayor múltiplo común
19. (a) Factores primos de 10 Factores primos de 15 pues el conjunto de múltiplos es infinito y por lo tanto no
hay un mayor elemento. 17. Para encontrar el MDC de
5
2 3 fracciones, uno debe encontrar el MMC de los denominado-
res. Así, el MMC de un conjunto de denominadores es el
Factores primos de 60
menor denominador común. 19. Hemos visto que
MDC1a, b2 = MDC1a - b, b2. Aplicando esto k veces, te-
(b) Factores primos de 8 Factores primos de 16
nemos MDC1a, b2 = MDC1a - kb, b2 para todos los ente-
2
ros k Ú 1. También MDC1a, b2 = MDC1 -a, b2. Por lo
tanto,
MDC12132, 5342 = MDC12132 - 4 # 534, 5342
3
Factores primos de 24
= MDC1 - 4, 5342
20. 1, 2, 22, 23, Á , 220 21. (a) 6x3y312x + 3y2 = MDC14, 5342
(b) 6x2y2z212x + 3y2z + 4x2yz22 22. (a) Siempre es = 2
verdadero; los divisores comunes de a y b son iguales que los Problemas de repaso
divisores comunes de ƒ a ƒ y b y son los mismos que los divi-
sores comunes de ƒ a ƒ y ƒ b ƒ . (b) Siempre es verdadero; 21. (a) 83,151; 83,451; 83,751 (b) 86,691 (c) 10,396
mismo razonamiento que en la parte (a) 23. Las respuestas pueden variar.
23. (a) MDC110!, 11!2 = 10! y MMC110!, 11!2 = 11! 30,030 = 2 # 3 # 5 # 7 # 11 # 13 25. 43
(b) MDC110!, 10! + 12 = 1 y MMC110!, 10! + 12 =
Evaluación 5-6A
10!110! + 12 24. 59 = 1,953,125 y 29 = 512
1. 2:00 p.m. 2. (a) 3 (b) 2 (c) 6 (d) 8 (e) 3
Conexiones matemáticas 5-5
(f ) 4 (g) No existe (h) 10 3. (a) 2 (b) 1
Comunicación (c) 2 (d) 4 (e) 1 (f ) 1 (g) 2 (h) 4
1. No, el conjunto de múltiplos comunes es infinito y por 4. (a)
lo tanto no puede haber un máximo múltiplo común. 䊝 1 2 3 4 5 6 7 8 9
3. No; por ejemplo, considera MDC12, 4, 102 = 2. 1 2 3 4 5 6 7 8 9 1
MMC12, 4, 102 = 20, y el MDC # MMC = 2 # 20 = 40, 2 3 4 5 6 7 8 9 1 2
mientras que abc = 2 # 4 # 10 = 80. 5. No. Sean a = 2 # 3,
3 4 5 6 7 8 9 1 2 3
b = 3 # 5, c = 5 # 7. Entonces MDC1a, b, c2 = 1, pero
4 5 6 7 8 9 1 2 3 4
MDC1a, b2 = 3 y MDC1b, c2 = 5. 7. El MMC es igual al
producto de los números si, y sólo si, los números no tienen facto- 5 6 7 8 9 1 2 3 4 5
res primos en común. Como 6 7 8 9 1 2 3 4 5 6
MDC1a, b2 # MMC1a, b2 = ab, MMC1a, b2 = ab si, y sólo 7 8 9 1 2 3 4 5 6 7
si, MDC1a, b2 = 1; esto es, a y b no tienen factores primos 8 9 1 2 3 4 5 6 7 8
en común. 9 1 2 3 4 5 6 7 8 9
9. • m = 6 y n = 9; entonces MDC16, 92 = 3 y
MMC16, 92 = 18. MDC115, 182 = 3. (b) 5 䊞 6 = 8, 2 䊞 5 = 6 (c) Todo problema de resta se
• m = 7 y n = 11; entonces MDC17, 112 = 1 y puede escribir como un problema de suma, el cual siempre
MMC17, 112 = 77. MDC118, 772 = 1. se puede resolver usando la tabla de sumar de 9 h.
• m = 8 y n = 16; entonces MDC18, 162 = 8 y
MMC18, 162 = 16. MDC124, 162 = 8.

© Lopez Mateos Editores. ISBN 978-607-95583-2-1, obra completa, versión electrónica, ISBN 978-607-95583-3-8, volumen 1, versión electrónica. Ejemplar asignado a: Helecto Villarroel gutierrez -
helecto@gmail.com. Fecha: 27 de octubre de 2014. Prohibida su modificación, copia o distribución.
R30 Respuestas a los problemas

5. (a) 䊟 1 2 3 4 5 6 7 8 9 Revisión del capítulo

1 1 2 3 4 5 6 7 8 9 1. (a) - 3 (b) a (c) - 1 (d) - x - y (e) x - y


2 2 4 6 8 1 3 5 7 9 (f ) x + y (g) 32 (h) 32 2. (a) - 7 (b) 8 (c) 8
3 3 6 9 3 6 9 3 6 9 (d) 0 (e) 8 (f) 15 3. (a) 3 (b) - 5 (c) Cualquier en-
tero excepto el 0 (d) Ningún entero funcionará. (e) - 41
4 4 8 3 7 2 6 1 5 9
(f ) Cualquier entero 4. 21 - 32 = - 6; 1( - 32 = - 3;
5 5 1 6 2 7 3 8 4 9
01 - 32 = 0; si el patrón continúa, entonces - 11 - 32 = 3;
6 6 3 9 6 3 9 6 3 9 - 21 - 32 = 6. 5. (a) 10 - 5 = 5 (b) 1 - 1 - 22 = 3
7 7 5 3 1 8 6 4 2 9 6. (a) - x (b) y - x (c) 3x - 1 (d) 2x2 (e) 0
8 8 7 6 5 4 3 2 1 9 (f ) - x2 - 6x - 9 (g) 4 - x2 7. (a) - 2x (b) x1x + 12
9 9 9 9 9 9 9 9 9 9 (c) 1x - 621x + 62 (d) 19y2 + 4x2213y - 2x213y + 2x2
(e) 511 + x2 (f ) 1x - y2x 8. (a) Falso; no es positivo para
(b) 3 s
4 5 = 6; 4 s 4 6 no está definida. (c) No, la divi- x = 0. (b) Falso, si un valor es positivo y otro negativo (c)
sión entre números diferentes de 9 no siempre es posible Falso, sean a = 2, b = - 5 (d) verdadero 9. (a) 1 , 2 Z
2 , 1 (b) 3 - 14 - 52 Z 13 - 42 - 5 (c) 1 , 2 no es
debido a que no en todas las filas (excepto la fila de la identi-
un entero. (d) 8 , 14 - 22 Z 18 , 42 - 18 , 22 10. (a)
dad) aparecen los números del 1 al 9. 6. (a) 3 (b) 2
(c) 1 (d) 1 (e) 1 (f) 4 7. (a) 2, 9, 16, 30 (b) 3, - 10 (b) - 299 (c) 289 (d) 0 (e) 3 ó - 3 (f ) x … 0; esto es,
10, 17, 24, 31 (c) 366 K 2 (mod 7); Miércoles 8. (a) 4
(b) 0 (c) 0 (d) 7 9. (a) x = 2k, k es un entero. 0, - 1, - 2, - 3, Á (g) x Ú 4 ó x … - 4; esto es,
(b) x - 1 = 2k implica x = 2k + 1, donde k es un entero. 5 Á - 6, - 5, - 46 ´ 54, 5, 6, 7, Á 6 (h) x = 11 ó - 9
(c) x - 3 = 5k implica x = 3 + 5k, donde k es un entero. 11. (a) - 1, 1, - 1, 1, - 1, 1 (b) 0, 2, 0, 4, 0, 6 (c) - 2, 4, - 8,
10. Martes a las 2:00 a.m. 11. C 16, - 32, 64 (d) - 5, - 8, - 11, - 14, - 17, - 20 12. (a) Geo-
métrica, radio - 1 (c) Geométrica, radio - 2 (d) Aritmética,
Conexiones matemáticas 5-6 diferencia - 3 13. (a) falso (b) falso (c) verdadero (d) Falso;
Comunicación 12, por ejemplo (e) Falso; 9, por ejemplo 14. (a) Falso; 7 ƒ 7
y 7ⱈ3, aunque 7 ƒ 3 # 7. (b) Falso; 3ⱈ13 + 42, pero 3 ƒ 3 y
1. (a) Sea abcd un número de cuatro dígitos, por ejemplo. En-
3ⱈ4. (c) verdadero (d) verdadero (e) Falso; 4ⱈ2 y
tonces abcd = a103 + b102 + c10 + d = 101100a +
10b + c2 + d. Nota que 101100a + 10b + c2 K 0 (mod 10); 4ⱈ22, pero 4 ƒ 44. 15. (a) Divisible entre 2, 3, 4, 5, 6, 8, 9, 11
por lo tanto, abcd K d (mod 10). (b) 5 (c) Sea abcd un (b) Divisible entre 3, 11 16. Si 10,007 es primo, 17ⱈ10,007.
número de cuatro dígitos, por ejemplo. Entonces abcd = Sabemos que 17 ƒ 17, de modo que 17ⱈ110,007 + 172 por el
a103 + b102 + c10 + d = 100110a + b2 + 1c10 + d2. teorema 5–13(b). 17. (a) 8724; 8754; 8784 (b) 41,856;
Nota que 100110a + b2 K 0 (mod 100); por lo tanto, 44,856; 47,856 (c) 87,174; 87,464; 87,754 18. (a) La
abcd = cd (mod 100). afirmación del estudiante es verdadera. Los ejemplos varían.
Solución abierta (b) Sea n un entero. Entonces n + 1n + 12 + 1n + 22 +
1n + 32 + 1n + 42 = 5n + 10 = 51n + 22. Así, la suma
3. Sobre un reloj de 12 h, 12 es la identidad aditiva, mien-
es divisible entre 5. 19. (a) compuesto (b) primo 20. Verifica
tras que para los enteros 0 es la identidad aditiva. Definimos
- 3 como la solución de la ecuación x + 3 = 12. Como la divisibilidad entre 3 y 8, 24 ƒ 4152. 21. No, son lo mismo
9 + 3 = 12, tenemos - 3 = 9. En general, si a es un número si los números son iguales. 22. MMC1a, b, c2 = MMC1m, c2,
ab
completo diferente a 12 en el reloj de 12 h, entonces - a = donde m = MMC1a, b2, MMC1a, b2 = y
12 - a y - 12 = 12. La última proposición es análoga a MDC1a, b2
mc
- 0 = 0 en los enteros. Las propiedades inversas aditivas ta- MMC1m, c2 = . Cada uno de estos MDC puede
MDC1m, c2
les como - 1a + b2 = - a + - b funcionan en el reloj. hallarse usando el algoritmo de Euclides. 23. El número no es
divisible entre 2, 3, 5, 7, 11 y 13 pues cada uno de estos
Preguntas del salón de clase
primos divide un producto en la suma 2 # 3 # 5 # 7 + 11 # 13
5. Daniel tiene razón en que si interpretamos 14 como 1 s 4 pero no el otro. El estudiante verificó que 17ⱈ353 y como
4, entonces 1 s4 4 = 4 y 4 7 3 pero 6 y 7 192 = 361 7 353, no se necesita verificar para otros primos.
no tienen significado en la aritmética del reloj. 7. En un reloj 24. (a) 4 (b) 73 25. (a) 24 # 53 # 74 # 13 # 29 (b) 77,562
de 5 h, el número 5 juega el papel del 0 en los enteros y es la 26. Las respuestas pueden variar; por ejemplo, 16. Para
identidad aditiva. Nota que 1 䊝 5 = 1, 2 䊝 5 = 2, obtener cinco divisores elevamos el primo (2) a la potencia
3 䊝 5 = 3, 4 䊝 5 = 4 y 5 䊝 5 = 5. Como la suma en el 15 - 12 . 27. 1, 2, 3, 4, 6, 8, 9, 12, 16, 18, 24, 36, 48, 72, 144
reloj de 5 h es conmutativa, cada una de estas sumas puede 28. (a) 22 # 43 (b) 25 # 32 (c) 22 # 5 # 13 (d) 3 # 37
ser invertida. 29. El MMC de todos los enteros positivos menores o iguales a
10 es 23 # 32 # 5 # 7, ó 2520. 30. $0.31 31. 9:30 a.m.

© Lopez Mateos Editores. ISBN 978-607-95583-2-1, obra completa, versión electrónica, ISBN 978-607-95583-3-8, volumen 1, versión electrónica. Ejemplar asignado a: Helecto Villarroel gutierrez -
helecto@gmail.com. Fecha: 27 de octubre de 2014. Prohibida su modificación, copia o distribución.
Respuestas a los problemas R31

32. Sabemos que el MDC1a, b2 # MMC1a, b2 = ab. Como No existe un solo entero i tal que para todos los enteros
MDC1a, b2 = 1, entonces MMC1a, b2 = ab. 33. 5 paquetes a - i = a e i - a = a (la primera ecuación implica que
34. 15 min 35. 71 capuchinos. Como 9869 = 71 # 139 y i = 0 pero 0 no satisface la segunda ecuación).
71 así como 139 son primos, ella vendió 71 capuchinos a $1.39 5-5. (a) 101 # 99 = 1100 + 121100 - 12 = 1002 - 12 =
cada uno. 36. (a) 210 # 310 10,000 - 1 = 9999 (b) 22 # 18 = 120 + 22120 - 22 =
(b) 12 # 172n = 2n # 17n (c) 974 pues 97 es primo 202 - 22 = 400 - 4 = 396 (c) 24 # 36 = 130 - 62
130 + 62 = 302 - 62 = 900 - 36 = 864
(d) 12324 # 12 # 323 # 12 # 1322 (d) 998 # 1002 = 11000 - 2211000 + 22 = 10002 - 22 =
= 212 # 23 # 33 # 22 # 132 1,000,000 - 4 = 999,996 5-6. a , 0 = x si, y sólo si,
= 217 # 33 # 132 0 # x = a y x es único. Como 0 # x = 0 para todos los enteros
(e) 23 # 3211 + 2 # 3 # 72 = 23 # 32 # 43 x, la ecuación no tiene solución si a Z 0. Si a = 0, entonces
(f ) 24 # 5613 # 5 + 12 = 28 # 56 37. Por el algoritmo de la para todos los enteros x, 0 # x = 0. Como la solución no es
división todo número primo mayor que 3 se puede escribir de única, 0 , 0 no está definida.
la forma 12q + r, donde r = 1, 5, 7 u 11 pues para cualquier 5-7. y y

otro valor de r, 12q + r es un número compuesto ya que 12 y r


compartirán un factor común, lo cual implica que no es primo.
38. n = a # 102 + b # 10 + c x x
n = a199 + 12 + b19 + 12 + c
n = 99a + 9b + c + b + a
Como 9 ƒ 99a y 9 ƒ 9b, entonces 9 ƒ 399a + 9b + 1a + b + c24
si, y sólo si, 9 ƒ 1a + b + c2. 39. Primero mostramos que en- (a) Una recta (b) Una recta
y y
tre tres enteros impares consecutivos cualesquiera, siempre hay
uno que es divisible entre 3. Para ello, supón que el primero de
la terna no es divisible entre 3. Entonces por el algoritmo de la
división ese entero puede escribirse de la forma 3n + 1 ó x x

3n + 2 para algún entero n. Entonces los tres enteros impares


consecutivos son 3n + 1, 3n + 3, 3n + 5 ó 3n + 2, 3n + 4,
3n + 6. En la primera terna, 3n + 3 es divisible entre 3, y en (c) Un ángulo recto (d) Un cuadrado
la segunda, 3n + 6 es divisible entre 3. Esto implica que si el
5-8. Si 5ⱈa y 5ⱈb, entonces 5ⱈ1a + b2 no es verdadera. Por
ejemplo, 5ⱈ 8 y 5ⱈ12 pero 5ƒ18 + 122. Por otro lado, 5ⱈ7 y
primer entero impar es mayor que 3 y no es divisible entre 3,
entonces el segundo o el tercero debe ser divisible entre 3, y 5ⱈ12 y 5ⱈ17 + 122. 5-9. Sí, es cierto. Si 3 ƒ x, entonces
por lo tanto, no pueden ser primos. 40. Viernes 41. mod 360. 3 divide a cualquier múltiplo de x, en particular 3 ƒ xy.
Cubriría toda el área rodeando el faro. 5-10. 1 + 2 + 5 + 0 + 6 + 5 = 19, de modo que debemos
Respuestas a Ahora intenta éste hallar números x y y tales que 9 ƒ 319 + 1x + y24. Cualesquier
dos números que sumen 8 ó 17 van a satisfacer esto. Por lo tanto, los
5-1. (a) sí (b) Depende de los valores de los números. espacios en blanco se pueden llenar con 8 y 9, ó 9 y 8, o las parejas
18, 02, 10, 82, 11, 72, 17, 12, 12, 62, 16, 22, 13, 52, 15, 32, 14, 42.
–2 8
(c) 6
5-11. (a) Las respuestas pueden variar. Por ejemplo, sólo hay
–4 –3 –2 –1 0 1 2 3 4 5 6
números cuadrados listados en la columna 3; 2 es el único número
5-2. (a) Como x … 0, ƒ x ƒ = - x y ƒ x ƒ + x = - x + x = 0. par que aparecerá en la columna 2; y la columna 2 contiene nú-
(b) Como x … 0, - ƒ x ƒ + x = - 1 - x2 + x = 2x. meros primos. Las potencias de 2 aparecen en columnas sucesi-
(c) Como x Ú 0, - ƒ x ƒ + x = - x + x = 0. vas. (b) No habrá más números en la columna 1 pues el 1 es el
5-3. (a) Las respuestas pueden variar. Por ejemplo, un ser- único número con sólo un factor positivo. Los otros números
vicio de mensajería trae tres cartas, una con un cheque de $23 y tienen al menos el mismo número y 1. (c) 49, 121, 169 (d) 64
las otras dos con cuentas de $13 y $12, respectivamente. ¿Eres (e) Los números cuadrados tienen un número impar de fac-
más pobre o más rico?, ¿en cuánto? (b) Las respuestas tores. Los factores se presentan en pares; por ejemplo, para 16
pueden variar. Por ejemplo, un servicio de mensajería te tenemos 1 y 16, 2 y 8, y 4 y 4. cuando listamos los factores, lista-
trae un cheque por $18 y se lleva una cuenta de $37 que es- mos sólo los factores distintos, de modo que 4 no se lista dos
taba dirigida a otra persona. ¿Eres más pobre o más rico?, veces, lo cual hace que el número de factores de16 sea un número
¿en cuánto? 5-4. (a) Sí, porque a - b = a + - b y la impar. Un razonamiento análogo es válido para todos los núme-
suma de dos enteros es un entero. (b) Ninguna de las ros cuadrados. 5-12. (a) Cuando obtenemos como respuesta un
propiedades es válida para los enteros pues: número completo, significa que el número con el que empezaste
a - b Z b - a 1si a Z b2 es divisible entre el número por el que dividiste. (b) 2261 es
1a - b2 - c Z a - 1b - c2 1si c Z 02
divisible entre 17 y 19 y por lo tanto la elección del color no
está determinada de manera única. 5-13. (a) 1, 2, 3, 6, 9

© Lopez Mateos Editores. ISBN 978-607-95583-2-1, obra completa, versión electrónica, ISBN 978-607-95583-3-8, volumen 1, versión electrónica. Ejemplar asignado a: Helecto Villarroel gutierrez -
helecto@gmail.com. Fecha: 27 de octubre de 2014. Prohibida su modificación, copia o distribución.
R32 Respuestas a los problemas

(b) 1, 2, 3, 4, 6, 8 (c) Sólo se pueden usar barras blancas para for- Sección 5-4
mar un tren de un solo color, para números primos si se deben
usar dos o más barras. (d) El número debe tener al menos 8 Suponiendo que las edades son números completos,
factores: 1, 2, 3, 5, 6, 10, 15, 30. 5-14. (a) No, pues los múlti- listamos la descomposición de 2450 en tres factores, cada
plos de 2 tienen a 2 como factor (b) Los múltiplos de 3: 53, 6, uno seguido por la suma de los tres factores.
9, 12, 15, Á 6 (c) Los múltiplos de 5: 55, 10, 15, 20, Á 6
(d) Los múltiplos de 7: 57, 14, 21, Á 6 (e) Sólo tenemos que
1, 1, 2450 2452 1, 2, 1225 1228
1, 5, 490 496 1, 7, 350 358
verificar la divisibilidad entre 2, 3, 5 y 7. 5-15. Se pueden usar
1, 10, 245 256 1, 14, 175 190
las barras de 1, 2, 3 y 6 a fin de construir los trenes para 24 y 30.
1, 35, 70 106 1, 25, 98 124
El mayor de éstos es 6, de modo que MDC124, 302 = 6.
1, 49, 50 100 2, 5, 245 252
5-16. (a) En el área de la izquierda están los factores de 24 que
2, 25, 49 76 2, 7, 175 184
no son factores de 40. En el centro o intersección de las áreas
2, 35, 35 72 5, 14, 35 54
están los factores de 24 y 40. En el área de la derecha están los
factores de 40 que no son factores de 24. (b) 8 5, 10, 49 64 5, 5, 98 108
(c) Factores de 36 Factores de 44 5, 7, 70 82 7, 7, 50 64
7, 10, 35 52
3
6 1 11
7, 14, 25 46
9 22
2
13
4 44
18 Las únicas sumas de tres factores que aparecen más de una
36
vez se indican en negritas. Si todas las sumas fueran diferen-
MDC (36,44) = 4 tes, Natalia sabría la respuesta pues le dijeron que la suma
5-17. Múltiplos de 8 Múltiplos de 12 era el doble de su edad. Como ella necesitó más información,
8 podemos concluir que su edad es 32 y las edades de las ami-
16
32 24 12
36
gas de Yoli son 5, 10 y 49 ó 7, 7 y 50. Que Natalia determi-
40 48
56
64
72 60
84.
nara la respuesta después que Yoli dijera que es por lo
96. ..
80
88.
.. menos un año menor que la mayor de sus tres amigas
..
eliminó 5, 10, 49 entre estas edades. Si la edad de Yoli era
MMC (8,12) = 24 48 o menos, Natalia seguiría necesitando más información.
Por lo tanto, las edades de las amigas deben ser 7, 7 y 50 y
5-18. (a) Sí (b) Sí, 12 es la identidad. (c) Sí Yoli tiene 49 años.

Respuestas a los Rompecabezas Sección 5-5


Si n es el ancho y m es el largo del rectángulo, entonces el
Sección 5-1 número de cuadrados que cruza la diagonal es
123 - 45 - 67 + 89 = 100 1n + m2 - MDC1n, m2 o 1n + m2 - 1.
Sección 5-6
Sección 5-2
No hay primos en la lista.
Página 280 Las respuestas pueden variar. 1 = 44>44; 2 =
14 # 42>14 + 42; 3 = 4 - 14>424; 4 = 314 - 42>44 + 4; Respuestas a las Actividades del laboratorio
5 = 4 + 414- 42; 6 = 4 + 314 + 42>44; 7 = 144>42 - 4; Sección 5-4
8 = 314 + 42>44 # 4; 9 = 4 + 4 + 4>4; 10 = 144 - 42>4 Sí, cada uno de los primos que están sobre la diagonal se pueden
Página 285 0 pues 1x - x2 = 0 obtener de la fórmula. El razonamiento es el siguiente:
Debido a la estructura geométrica de la espiral, la “distancia” del
Sección 5-3 centro del cuadrado (donde se localiza el 41) al siguiente
cuadrado sobre la diagonal a lo largo de la espiral son 2 pasos.
Página 295 La parte de la explicación que es incorrecta es
la división entre 1e - a - d2, la cual es igual a 0. Dividir
De ahí al siguiente en la diagonal a lo largo de la espiral, son 4
pasos y de ahí al siguiente son 6 pasos, y así sucesivamente. En
entre 0 es imposible. general, de cualquier punto sobre la diagonal al siguiente sobre
Página 300 El número es 381-65-4729. la diagonal a lo largo de la espiral son 2 pasos más de lo que to-
maría tratar de alcanzar el punto previo sobre la diagonal. Se
puede verificar que la fórmula n2 + n + 41 halla los primos
para 0 … n … 39 (no hay otra manera conocida de hacerlo).

© Lopez Mateos Editores. ISBN 978-607-95583-2-1, obra completa, versión electrónica, ISBN 978-607-95583-3-8, volumen 1, versión electrónica. Ejemplar asignado a: Helecto Villarroel gutierrez -
helecto@gmail.com. Fecha: 27 de octubre de 2014. Prohibida su modificación, copia o distribución.
Respuestas a los problemas R33

Para n = 0, obtenemos 41. Cada vez que n2 + n + 41 es Respuesta al Problema preliminar


conocida, el siguiente número resultado de la fórmula es
1n + 122 + 1n + 12 + 41 = 1n2 + n + 412 + 21n + 12.
La fórmula de diferencia de cuadrados la podemos usar para
resolver este problema. Reescribimos la expresión dada como
Por lo tanto, cada vez que n2 + n + 41 halla un primo con sigue para obtener una suma que es familiar en el capítulo 1:
0 … n … 38, el siguiente primo que se obtiene de la fórmula
está a 2n + 2 pasos y, por lo tanto, como se explicó anterior- 502 - 492 + 482 - 472 + Á + 22 - 12 =
mente, sobre la diagonal. Nota que la espiral se puede conti- 1502 - 4922 + 1482 - 4722 + Á + 122 - 12)
nuar con sólo primos en la diagonal llegando a n = 39, esto = 150 - 492150 + 492 + 148 - 472148 + 472 +
es, hasta obtener 392 + 39 + 41, ó 1601. Para n = 40, obte- Á + 14 - 3214 + 32 + 12 - 1212 + 12
nemos 1601 + 2139 + 12, ó 1681 = 412, que estará sobre = 1150 + 492 + 1148 + 472 + Á
la diagonal (con 2 # 40 pasos fuera del 1601) pero no es + 114 + 32 + 112 + 12
primo. = 50 + 49 + 48 + 47 + 46 + Á + 3 + 2 + 1
En este punto podemos usar el trabajo realizado con el pro-
Respuestas al Rincón de la tecnología blema de Gauss en el capítulo 1 para encontrar la suma de
Sección 5-1 los enteros que van de 1 a 50. 511502>2 = 1275 y, por lo
tanto, el valor de la expresión inicial es 1275.
1. Los registros de la columna A son 4 mientras que los re-
gistros de la columna B comienzan con 3 y decrecen en 1.
La suma de las columnas A y B se coloca en la columna C
comenzando con 7. Los registros en la columna C son los Capítulo 6
enteros en orden decreciente comenzando con 7. Los patro-
nes muestran que la suma de dos números positivos es posi- Evaluación 6-1A
tiva. La suma de un número positivo y uno negativo es
7 7
positiva si el número positivo es mayor que el valor absoluto 1. (a) La solución a 8x = 7 es . (b) Juana comió de la
del número negativo. La suma es 0 si ambos números tienen 8 8
el mismo valor absoluto. En caso contrario, la suma es pizza. (c) La razón de niños a niñas es 7 a 8.
negativa. Se pueden obtener los resultados análogos si la co- 1 1 2 1 7 2 4
2. (a) (b) (c) ó (d) 3. (a) (b)
lumna A se cambia a - 4. 6 4 6 3 12 3 6
2. (a) La gráfica debe aparecer como se muestra a 2 6 2 8 2
ó (c) ó (d) ó . El diagrama ilustra
continuación. 3 9 3 12 3
La Ley Fundamental de las Fracciones. 4. (a) no, partes
y
iguales no (b) sí (c) sí
4
5. (a) (b) (c)

2 3 3
x 8 9 6
24
9 3 12 1 4 1 8 1
6. (a) ó (b) ó (c) ó (d) ó
24 8 24 2 24 6 24 3
4 6 8 - 4 2 - 10 0 0 0
7. (a) , , (b) , - , (c) , ,
18 27 36 10 5 25 1 2 4
(b) Cuando x es menor que - 4, los valores y son negativos; -5
2a 3a 4a 52 3
cuando x = - 4, y = 0; cuando x es mayor que - 4, los valo- (d) , , 8. (a) (b) (c)
4 6 8 31 5 7
res y son positivos.
9. (a) indefinida (b) indefinida (c) 0 (d) no se puede
Sección 5-2 simplificar (e) no se puede simplificar
El producto de dos números negativos es un número posi- a - b 2x
10. (a) , a Z - b (b) , x Z 0, y Z 0
tivo. Los números en la columna C son cuadrados perfectos. 3 9y
11. (a) igual (b) igual 12. (a) no es igual (b) no es igual
Sección 5-5 36
13. 14.
1. La intersección está formada con los primeros doce múl- 48
tiplos de 12.
2. Sólo necesitas llenar hasta el 47. 3. (a) 180 32
(b) Necesitas usar una de las técnicas presentadas en la sec- 15. 16. (a) (b) - 36
3
ción para hallar MMC (6, 9, 12, 15). E F

© Lopez Mateos Editores. ISBN 978-607-95583-2-1, obra completa, versión electrónica, ISBN 978-607-95583-3-8, volumen 1, versión electrónica. Ejemplar asignado a: Helecto Villarroel gutierrez -
helecto@gmail.com. Fecha: 27 de octubre de 2014. Prohibida su modificación, copia o distribución.
R34 Respuestas a los problemas

11 11 11 2 1 2 1
17. (a) 7 (b) 7 (c) 6 18. (a) , , números racionales son y . En este caso, 7 .
13 16 22 3 2 3 2
- 1 - 19 - 17 Sira no tiene razón.
(b) , , 19. El término n-ésimo de la sucesión es
5 36 30 1
9. Como 36 pulg = 1 yarda, 1 pulg = yarda, x pulg es
. Por lo tanto, el término 1n + 12 es
n n + 1 36
. Necesitamos
n + 2 n + 3 1 x
n + 1 n de x yardas, esto es de yarda.
mostrar que 7 para n Ú 1. Por el teorema 6-4 36 36
n + 3 n + 2
esto es verdadero si, y sólo si, 1n + 121n + 22 7 n1n + 32,
Solución abierta
el cual es equivalente a n2 + 2n + n + 2 7 n2 + 3n, ó 2 7 0.
Como la última proposición es verdadera, la proposición 11. Las respuestas pueden ciertamente variar. Algunos edu-
n + 1 n cadores matemáticos argumentan que los enteros positivos y
7 es verdadera. Otro enfoque es usar el negativos son de más fácil comprensión para los estudiantes
n + 3 n + 2
que los números racionales. También argumentan que las
teorema 6–6 y observar que cada término de la sucesión co-
menzando por el segundo se puede obtener sumando los operaciones en los enteros son más fáciles que las operacio-
numeradores y los denominadores de los términos vecinos. nes en los números racionales.
20. Las respuestas pueden variar. A continuación tenemos Aprendizaje colectivo
las respuestas posibles.
- 22 - 23 13. (a) Las respuestas pueden variar; los estudiantes debe-
10 11
(a) , (b) , 21. 456 mi rán trabajar juntos para determinar las estaturas y ordenar a
21 21 27 27 las personas según su estatura. Por ejemplo, si las estaturas
6 3 6 3 52 56 58
22. (a) , ó de libra; ,ó de tonelada son 52", 56" y 58", las fracciones podrían ser , y
16 8 32,000 16,000 60 60 60
10 1 15 1 8 1 (b) Las respuestas varían dependiendo del inciso (a).
(b) ,ó (c) ó (d) ,ó
100 10 60 4 24 3
Preguntas del salón de clase
Conexiones matemáticas 6-1 15. Lo que hace el primer estudiante es correcto. Lo que
Comunicación hizo el segundo estudiante fue tratar el problema como si
fuera a b a b = , cuando en realidad, el problema es
3 1 5 1
1. Las respuestas pueden variar; por ejemplo, de una can- 5 3 3
4
tidad se puede encontrar dividiendo la cantidad entre 4 y 15
. Se cancelan factores no dígitos. 17. De acuerdo con la
3 53
tomando 3 de esas partes. Por lo tanto, de 4 es 3. Por otro propiedad de densidad de los números racionales, hay otro
4 número racional entre cualesquier dos números racionales.
3
lado, cuatro es 12 cuartos ó 3. 3. Las respuestas pueden 9999 999
4 es un número racional entre y 1.
variar; por ejemplo, el estudiante está pensando en la Ley 10000 1000
a a#c
Fundamental de las Fracciones que dice = # . Esto sólo 19. Los estudiantes probablemente estén pensando que más
b b c piezas significa más pizza. Una pizza (o círculo) se podría
2 6
funciona para la multiplicación. Z pues 2 # 7 Z 3 # 6. En cortar en 6 piezas; entonces cada pieza se podría cortar en 2
3 7
a a + c piezas más. Esto muestra que la cantidad de pizza no cambió
general, Z a menos que a = b o que c = 0. Un enfo- por estos últimos cortes, sólo cambió el número de piezas.
b b + c
a c
que alternativo es aplicar el teorema 6–6 a las fracciones y . 21. Las condiciones implican que a Z b y c Z d. También
b c
1 a c d b
5. La nueva fracción es igual a . El principio se puede ge- b Z 0 y a Z 0. = , si y sólo si, ad = bc. =
2 b d c - d a - b
neralizar como sigue:
si, y sólo si, ad - bd = bc - bd o ad = bc, lo cual es cierto.
a ar1 ar2 ar3 arn José tenía razón. 23. Daria no está en lo correcto pues el
= = = = Á =
b br1 br2 br3 brn entero no está dividido en 3 partes iguales.
a1r1 + r2 + r3 + Á + rn2 a 25. Observando la recta numérica podrás notar que
b1r1 + r2 + r3 + Á + rn2
= =
ex 2 - 6 x 6 f ( ex 2 - 6 x 6 f
b 1 1 3 3
2 1 2 2 2 2
7. (a) Supongamos que los números racionales son y .
16 4
1 2 Así, cualquier x en el primer conjunto también está en el se-
En este caso, 7 e Iris no tiene razón. (b) Supón que los gundo conjunto.
4 16

© Lopez Mateos Editores. ISBN 978-607-95583-2-1, obra completa, versión electrónica, ISBN 978-607-95583-3-8, volumen 1, versión electrónica. Ejemplar asignado a: Helecto Villarroel gutierrez -
helecto@gmail.com. Fecha: 27 de octubre de 2014. Prohibida su modificación, copia o distribución.
Respuestas a los problemas R35

Evaluación 6-2A simplificar si escogemos el menor denominador común. (b) No,


-4 -1 1 1 3
7 1 5y - 3x por ejemplo, + = , el cual no está en su forma más sim-
1. (a) ó1 (b) ó (c) 3 6 6
6 6 12 3 xy
- 3y + 5x + 14y2 ple. 5. Las respuestas pueden variar; por ejemplo, la maestra
71 23 - 23 2
(d) (e) ó 2 (f ) ó -7 está pensando cada rectángulo que dividió en 4 partes iguales
2 2 24 24 3 3
2x y como el todo dividido en cuartos, de modo que la figura
2 93 27 - 29 3
2. (a) 18 (b) - 2 3. (a) (b) sombreada representa 2 . Cristina está pensando que los tres
3 100 4 8 4
1 1 3 1 rectángulos representan un entero y 11 de 12 partes están
4. (a) , alto (b) , bajo (c) , bajo (d) , alto sombreadas. 7. Las respuestas pueden variar; por ejemplo,
3 6 4 2
5. (a) Castores (b) Patos (c) Osos 3 1 15 16 45 + 64 109 1
3 + 5 = + = = = 9 ó
6. Alrededor de 0 Alrededor de 1– Alrededor de 1
4 3 4 3 12 12 12
2

3 + 5 = 13 + 52 + a + b = 8
3 1 3 1 13 1
4
— 8
— 7 = 9 . En el se-
—1 7 12 —
10 8 4 3 4 3 12 12
1 2


100 5 gundo método, los números son más pequeños para trabajar.
—9
18 9. (a) Sí. Si a, b, c y d son enteros, b Z 0, d Z 0, entonces
a c ad - bc
1 3 - = es un número racional. (b) No; por
7. (a) , alto (b) 0, bajo (c) , alto (d) 1, alto b d bd
2 4 1 1 1 1
ejemplo, - Z - . (c) No; por ejemplo,
3 1 2 4 4 2
8. (a) 2 (b) 9. (a) (b) 0 10. (a) A
- a - b Z a - b - . (d) No. Si existe una
4 4 1 1 1 1 1 1
(b) E (c) T (d) E 11. Aproximadamente, 4 # 3 = 12. 2 4 8 2 4 8
1 7 5 identidad para la resta, ésta debe ser 0 pues sólo para el 0
12. 13. 6 yardas 14. 2 yardas 15. (a) Equipo 4, a a a
4 12 6 se cumple - 0 = . Sin embargo, en general, 0 - Z
11 11 1 3 b b b
76 lb (b) 3 lb 16. (a) + 僆Q a
16 16 2 4 - 0, y por lo tanto no existe la identidad. (e) No; como no
b
(c) a + b + + a + b
1 3 3 1 1 1 1 1 1 1 hay identidad, el inverso no puede ser definido.
(b) + = + = 1 1 1 1
2 4 4 2 2 3 4 2 3 4 11. (a) Decreciente ya que 7 7 7 7 Á , y si
2 3 4 5
3 7 6 7 8 sumamos 1 a cada término las desigualdades no cambian.
17. (a) , , 2 (b) , , , no es aritmética, 1 1 2 1 3 1
2 4 7 8 9 (b) Creciente pues = 1 - , = 1 - , = 1 - , Á ,
2 1 3 2 n n 2 2 3 3 4 4
- Z - 18. (a) (b)
3 2 4 3 4 n + 1 n 1
= 1 - . Así, en cada término de la sucesión
7 8 9 10 11 3 25 1 n + 1 n + 1
19. 1, , , , , , 2 20. (a) (i) (ii) ,ó2 restamos un número menor que 1 y, por lo tanto, nos acer-
6 6 6 6 6 4 12 12
1 -7 -1 camos más a 1. Formalmente tenemos:
(iii) 0 (b) (i) (ii) (iii) 21. (a) - 2
4 4 4 1 1
6
1 7 1 1 1 1 n + 1 n
(b) 0 (c) (d) 22. (a) (i) + # = + = -1 -1
2 4 4 3 4 4 12
7
16 1 1 1 1 1 25 1 n + 1 n
= (ii) + # = + = =
48 3 5 4 5 5 20 100 4 1 1
1 - 7 1 -
1 1 1 1 36 1 1 1 n + 1 n
(iii) + # = + = = (b) = +
6 5 6 6 30 180 5 n n + 1 1 1 1 1
(c) Decreciente pues 6 implica 6 2
3x + y3 1n + 12
2
1 5 21 1 n + 1 n 2
n
23. (a) (b) ó3 24. (a)
n1n + 12 6 6 2 x2y2 Solución abierta
az - by 2
2ab - b
(b) (c) 2 13. Las respuestas pueden variar; por ejemplo:
xy2z a - b2
2 1 b - a 2 4 1 150 49
(a) y , (b) , y (c) ,
Conexiones matemáticas 6-2 3 3 b 7 7 7 100 100

Comunicación Aprendizaje colectivo


1 1 7
1. Las respuestas pueden variar; por ejemplo, + ó no 15. Dependiendo de las personas entrevistadas, quizá un
3 4 12
maestro les responda así: Uso fracciones al determinar cali-
representa la cantidad recibida pues las fracciones no vienen
ficaciones finales. Por ejemplo, si un trabajo constituye
del “todo” del mismo tamaño 3. (a) No, pero es más fácil

© Lopez Mateos Editores. ISBN 978-607-95583-2-1, obra completa, versión electrónica, ISBN 978-607-95583-3-8, volumen 1, versión electrónica. Ejemplar asignado a: Helecto Villarroel gutierrez -
helecto@gmail.com. Fecha: 27 de octubre de 2014. Prohibida su modificación, copia o distribución.
R36 Respuestas a los problemas

1 1 1 3 3 8 4 10
2
de la calificación y otro trabajo constituye de la calificación,
3 (d) x Ú 1 23. (a) a b (b) a b (c) a b
2 4 3
necesito saber qué fracción de la calificación me falta determinar.
4 10 6 2
(d) a b 24. (a) 1010 (b) 1010 # a b =
Preguntas del salón de clase 5 5
1 1.44 # 1010 = 14.4 millardos
17. La figura de Queta muestra que de un entero dividido
3 1 1 1
3 25. (a) y (b) 2S = 1 + + 2 + Á + 63
en 3 partes combinado con de un entero dividido en 4 2 2 2
4
2S - S = 1 + a + 2 + Á + 63 b
4 1 1 1
partes da un total de de un entero dividido en 7 partes. 2
7 2 2
- a + 2 + Á + 1 b - 1
Para hacer lo que Queta trata de hacer, ella necesitaría usar el 1 1
mismo todo en lugar de usar 3 diferentes tipos de todos. 2 2 263 264
1 3 1 1
Cuando se suman + , el mismo todo debe usarse. = 1 - 64 Nota que 2S = 1 + S - 64 .
3 4 2 2
1 1
Problemas de repaso Por lo tanto, 2S - S = 1 + S - 64 - S = 1 - 64 .
2 2
2 13 25 a 1 2 6 2
19. (a) (b) (c) (d) , o a Z - 1 (c) 1 - n 26. (a) , , (b) 20
3 17 49 1 2 5 5 15
(e) simplificado 3 3 3 3 3 1
184 27. (a) , , , , (b) La razón común es .
2 4 8 16 32 2
(c) n Ú 10 28. (a) 3250, pues 3250 = 125250 = 2250 y
(f ) a + b (si a Z b). 21. (a) febrero (b)
365
4100 = 1222100 = 2200 (b) 1 - 32 75, pues 1 - 272 15 =
- -
Evaluación 6-3A - -
1 - 32 45 = 45 6 75 29. Supón x 6 y, i.e., y - x 7 0.
- 1 1
1 1 1 2#3 6 3 3
1. (a) , y (b) =
4 3 12 4 5 20 x + y x + y - 2x y - x
1 Entonces - x = = 7 0.
2. (a) (b) 3. (a) 2 2 2
5 x + y x + y
Por lo tanto, 7 x. Análogamente, 6 y.
2 2
b za 1 1
(b) (c) 2 4. (a) 10 (b) 8 5. (a) - 3 Conexiones matemáticas 6-3
a xy 2 3
3 y -1 21 7 6 3 Comunicación
(b) (c) (d) 6. (a) ó (b) ó 1
10 x 7 12 4 4 2 1. Las respuestas pueden variar; por ejemplo, de un nú-
-1 3 2
1 x
(c)
8
(d)
2
7. Las respuestas pueden variar; por ejemplo, mero x es equivalente a # x ó , mientras que dividir un
(a) 6 , 2 Z 2 , 6 (b) 18 , 42 , 2 Z 8 , 14 , 22
2 2
1 x x 2 2x
8. (a) 26 (b) 29 (c) 92 (d) 18 9. (a) 20 número x entre es equivalente a = # = = 2x.
2 1 1 1 1
(b) 16 (c) 1 10. (a) 18 (b) 25 11. (a) menor que 1 2
(b) menor que 1 (c) mayor que 2 12. (c) 13. 9600 Por lo tanto, no son lo mismo.
1 a b
estudiantes 14. 15. $240 16. $225 17. 32 canicas 3. Nunca menor que n. 0 6 6 1 implica 0 6 1 6 . La
6 b a
última desigualdad implica 0 6 n 6 n # a b . Por lo tanto,
1 1 1 b
18. (a) 13 (b) 313 (c) 511 (d) 519 (e) ó 2
3 1 52
- 2
5 a
n , a b = n # 7 n. 5. El segundo número debe ser el
1 10 1 3 2 9 a b
(f ) a5 19. (a) a b (b) a b (c) a b b a
2 2 3
20. (a) Falso; 23 # 24 Z 12 # 223 + 4.
recíproco del primero y debe ser un número positivo menor
(d) 1 (b) Falso;
que 1. 7. Las respuestas pueden variar; por ejemplo, Cali no
Z 12 # 22 3#2
Z 12 # 222 3.
3 #2
2 3 #2
3 # tiene razón pues 0 es un número racional y el inverso mul-
2 (c) Falso; 2
0 3 4
(d) Verdadero, pues ab Z 0. (e) Falso; 12 + 322 Z 22 + 32. tiplicativo de no existe. 9. José está mal. 2 # 3 =
1 5 5
(f ) Falso; 12 + 32-2 Z 2 + 2 . 21. (a) 5 (b) 6 ó - 6 a2 + b a3 + b Z a2 + b a3 + b 11. (a) Beto
1 1 3 4 4 3
2 3 5 5 5 5
(c) - 2 (d) - 4 22. (a) x … 4 (b) x … 1 (c) x Ú 2 corrió a 6 mph durante 3 h, esto es, 6 # 3, ó 18 millas,

© Lopez Mateos Editores. ISBN 978-607-95583-2-1, obra completa, versión electrónica, ISBN 978-607-95583-3-8, volumen 1, versión electrónica. Ejemplar asignado a: Helecto Villarroel gutierrez -
helecto@gmail.com. Fecha: 27 de octubre de 2014. Prohibida su modificación, copia o distribución.
Respuestas a los problemas R37

3 6 3 12 3 15 3
la distancia entre los pueblos. Nelson corrió de 18 millas, + = + = , ó . También podemos convertir
4 5 10 10 10 10 2
3#9 1 distancia 1312 cada número a una fracción impropia y multiplicar como se
esto es, = 13 millas. Su rapidez fue =
2 2 tiempo 3 5 3 3
muestra: # = .
27 1 2 5 2
= = 4 mph. (b) La rapidez de Beto es ahora de 5
2#3 2 Problemas de repaso
1 1 9 1
mph y de la distancia es # 18 = , ó 4 milla. El tiempo 25. 120
4 4 2 2
9
distancia 2 9 Revisión del capítulo
correspondiente es = = de una hora, ó
rapidez 5 10 1. (a) (b)
9 #
60 min., ó 54 minutos.
10 (c)
Solución abierta

13. (a) Las respuestas pueden variar; por ejemplo, si tienes un


3 1
pizarrón que mide 1 yd de largo, ¿cuántas veces cabe yd a
4 2
lo largo del pizarrón para que lo puedas dividir? (b) Las 10 15 20
respuestas pueden variar. (c) Las respuestas pueden variar. 2. Las respuestas pueden variar: por ejemplo , , .
12 18 24
Aprendizaje colectivo 6 ax 5 2
3. (a) (b) (c) 0 (d) (e) b (f ) (g) No se
7 b 9 27
15. Las respuestas dependen del tamaño de los ladrillos y
del tamaño de las juntas. Seguramente, las medidas de las puede reducir. (h) no se puede reducir 1
juntas se darán en fracciones de pulgada. El tamaño de los 4. (a) = (b) 7 (c) 7 (d) 6 5. (a) - 3,
3
ladrillos se pueden dar en pulgadas. Una alternativa es me- -5 6
- 1 7 3 -4 - 1 - 7
dirlos en centímetros. Toda la medición es aproximada y (b) 3 , (c) , (b) , 6. 2 , 1 , 0,
puede haber redondeo o estimación. 7 22 6 5 4 3 3 8
300 300
a b , ,a b
71 69 1 71 74
Preguntas del salón de clase , , 7. Sí. Por la definición
140 140 2 140 73
17. El estudiante está generalizando la propiedad distribu- de multiplicación y las leyes conmutativa y asociativa de la
tiva de la multiplicación sobre la suma a la propiedad distri- multiplicación, podemos hacer lo siguiente:
4#7#5
butiva de la multiplicación sobre la multiplicación. Esta
4#7# 5
última no es verdadera. 19. El estudiante está mal a menos = # #
5 8 14 5 8 14
que n = 0 ó p = m. La Ley Fundamental de las Fracciones
4#7#5
sólo es válida para la multiplicación. 21. Julieta está bien; = #
estrictamente hablando 3 R2 no es un número, y no se puede 8 14 # 5
4 7 #5
17
igualar a . Nota que del algoritmo de la división tenemos = #
5 8 14 5
17 = 5 # 3 + 2, de modo que cuando tratamos con los enteros
8. (a) 24, pues 18 # 92 es igual a a # 9 b # 8 = 3 # 8 = 24.
1 1
el resultado de la división es 3R2. Sin embargo, cuando nos 3 3
movemos con los números racionales procedemos como sigue: 5 11
5#3 + 2 #
(b) 66, pues 36 1 es igual a 36 # = 6 # 11 = 66.
17 = 6 6
5 11
5#3 2 9. 17 piezas, y sobran yd. 10. (a) 15 (b) 15
= + 6
5 5
2 (c) 4 11. Las respuestas pueden variar; ver sección 6.3.
= 3 + 76 78
5 12. Las respuestas pueden variar. 13. , , pero las
2 100 100
= 3 . respuestas pueden variar.
5
23. Franca probablemente no entiende la notación de los
1 3 14. 5 0 4 7 9 2 ⫻ 2 3 ⫽
números mixtos y pensó el problema como: 2 # es 1 240 6
2 5 15. 333 calorías 16. 752 veces 17. =
1 3 3 3
2 + # ó2 + ó 2 . Si la propiedad distributiva se usa 3 1000 25
2 5 10 10 18. No es razonable decir que la universidad ganó
correctamente, tenemos a 2 + b # = 2 # + # =
1 3 3 1 3 3 5 11
+ , ó , de sus juegos de baloncesto. La fracción
2 5 5 2 5 4 8 8
© Lopez Mateos Editores. ISBN 978-607-95583-2-1, obra completa, versión electrónica, ISBN 978-607-95583-3-8, volumen 1, versión electrónica. Ejemplar asignado a: Helecto Villarroel gutierrez -
helecto@gmail.com. Fecha: 27 de octubre de 2014. Prohibida su modificación, copia o distribución.
R38 Respuestas a los problemas

correcta no se puede determinar sin información adicional, 6-2. (a) 3


4
5 3 1
pero está entre y . 19. Los numeradores de los números 4
8 4 1
2
racionales son enteros y siguen las propiedades de los enteros; lo 3
2
mismo sucede con los denominadores. Así, tanto el numerador 2
3
como el denominador de la respuesta son enteros y podemos 4
aplicar otra propiedad de los enteros para determinar el signo de 3

la respuesta. 20. Debes mostrarle que la fracción dada puede 1 0 1 2


(b) 6-3. (1) (2) , ,
escribirse como un entero sobre otro entero. 22274 21 212 0 1
2 1 3
2 2 3 2 2 2
8 4
En este caso, el resultado es . 21. 112 bolsas 22.
aó 0, , 1b (3)
9 15 1 1
- 12 - 11 - 12 - 11 6-4. A continuación presentamos un dia-
- a b es un
2 4
23. es mayor que pues
10 9 10 9 grama de Venn para ilustrar la relación entre números naturales,
5 1 números completos, números enteros y números racionales:
número positivo. 24. (a) 3 (b) 3 25. (a) , ó 1
4 4
19 1 13 1 a3 Números racionales

(b) ,ó3 (c) - 100 (d) ,ó4 26. (a) 7 Enteros


6 6 3 3 x
Números completos
y8 3ax + b 15 - 2y2 Números
(b) 10
27. (a) 2 2
(b) naturales
x xy 3xy2
a - bx2y 3131
(c) 3 2
(d) 3 3

28. Las respuestas pueden
xyz 23 216
variar; por ejemplo, el problema es hallar cuántas piezas de
6-5. Si n fuera cero, entonces bn sería cero y la división en-
1 3 1
yd de cinta hay en 1 yd. Hay 3 piezas de longitud yd tre cero está indefinida. 6-6. El teorema 6–3 afirma que a,
2 4 2 a c
1 1 1 1 b y c son enteros y b 7 0; entonces 7 si, y sólo si, a 7 c.
y sobra yd. Este yd es de una pieza de yd. Por lo b b
4 4 2 2 La cuestión es investigar si el teorema es cierto si b 6 0.
1 1
tanto, hay 3 piezas de cinta de yd y 1 pieza que es de la Considera 2 7 1 y - 1 6 0. Ahora 2>1 - 12 6 1> 1 - 12, lo
2 2 cual contradice el teorema extendido para b 6 0.
1 1 1 9 5 2 3
pieza de yd ó 3 de pieza de yd. Estas “3 piezas y sobra 6-7. , , y 6-8. Si a 7 0 y c 7 b 7 0, entonces
2 2 2 16 8 3 4
1 1 a a a c
de yarda” y “3 piezas” son las respuestas correctas. 7 . 6-9. Considera dos números racionales y ,
4 2 b c b d
2ab a c
29. donde 6 . Por la propiedad de densidad entre los núme-
ab + bc + ac b d
ros racionales (Teorema 6–5) podemos hallar un número
Respuestas a Ahora intenta éste a
racional x, entre dos fracciones. Como 6 x1, existe un
6-1. (a) Un cuarto de 100¢ es 25¢, un cuarto de 60 min es b
a
15 min. (b) Las fracciones están definidas en relación con un número racional x2 entre y x1. Después podemos encon-
b
todo. En este caso hay 2 todos. En la figura 6-2(a), está sombrea- a
trar un número racional x3 entre y x2 y así sucesivamente. Este
1 b
do de un todo circular. En la figura 6-2(b), está sombreado
3 proceso se puede repetir indefinidamente y por lo tanto obte-
1 1 1
de un cuadrado pequeño. Para mostrar que 7 , las nemos una infinidad de números racionales x1, x2, x3, Á
2 3 2
a c a c
fracciones deben estar asociadas con el mismo todo, por ejem- entre y . 6-10. Como 6 con b 7 0 y d 7 0, el
1 1 b d b d
a a + c
plo, al comparar del círculo con del círculo. Es cierto teorema 6–4 implica que ad 6 bc. Ahora 6 si, y sólo
3 2 b b + d
que el área sombreada en el círculo es mayor que el área si, a1b + d2 6 b1a + c2, que es equivalente a ab + ad 6
1 1
sombreada en el cuadrado, pero esto no muestra que 7 . a a + c
3 2 ba + bc ó ad 6 bc. Por lo tanto 6 . De manera aná-
b b + d
loga podemos mostrar que

© Lopez Mateos Editores. ISBN 978-607-95583-2-1, obra completa, versión electrónica, ISBN 978-607-95583-3-8, volumen 1, versión electrónica. Ejemplar asignado a: Helecto Villarroel gutierrez -
helecto@gmail.com. Fecha: 27 de octubre de 2014. Prohibida su modificación, copia o distribución.
Respuestas a los problemas R39

a + c c 3 3 3 3 al guardia. Finalmente, él debió tener cuatro bolsas antes de


6 . 6-11. 2 + 5 = 2 + + 5 + =
b + d d 4 8 4 8 darle al guardia cualquier bolsa en el cuarto puente. Todo el
6 3 9 1 1 3 procedimiento se resume en la siguiente tabla.
7 + + = 7 + = 7 + 1 + = 8 . 6-12. es ma-
8 8 8 8 8 4
1 1 1 3 1 4 Bolsas Bolsas antes de Bolsas
yor que ; + = 1, de modo que + 7 1. es menor
2 2 2 4 2 6 después de darle al guardia antes de
3 1 Puente cruzar la bolsa extra cruzar
que 1, así que no puede ser la respuesta correcta para + .
4 2
1 1 Cuarto 1 2 4
6-13. (a) Doblar, ver página muestra. (b) (c)
4 8 Tercero 4 5 10
3 1 1 1 9 5
(d) Sombrear, ver página muestra. (e) (f ) , , , , Segundo 10 11 22
8 4 8 16 16 16
3 4 15 Primero 22 23 46
(g) , , (h) El producto de fracciones es el
12 15 24
Página 405 No. El deseo es imposible pues las fracciones
producto de los numeradores sobre el producto de los
de gatos a repartir no suman el total de gatos.
denominadores. 6-14. Las respuestas pueden variar; por
1 1 1 17 18
ejemplo, considera lo siguiente: Si Carlos tiene $10.00, x + x + x = x, pero la suma debe ser 1x, ó x.
2 3 9 18 18
¿cuántas barras de chocolate puede comprar si (a) el precio
de una barra es de $2.00? (b) el precio de una barra es
Respuestas a las Actividades del laboratorio
1
$ ? En el inciso (a) la respuesta es 10 , 2 ó 5.
2 Sección 6-1
1 1
En el inciso (b) la respuesta es 10 , , que es lo mismo que (a) Área de a = (b) Área de a = 1
2 4
1
encontrar s en 10. Como hay dos mitades en 1, en 1
2 Área de b = Área de b = 1
10 hay 20. Por lo tanto Carlos puede comprar 20 barras. 4
1 1
a a d a#d a#d Área de c = Área de c =
16 4
a c b b# c b c b c a d 1 1
6-15. , = = = = = # Área de d = Área de d =
b d c c d c#d 1 b c 8 2
d d c d c 1 1
Área de e = Área de e =
a a d a#d a#d 16 4
a , c c c b c b c b ad a c 1 1
6-16. = = # = = = = , Área de f = Área de f =
b , d b b d b#d 1 bc b d 8 2
d d b d b 1 1
Área de g = Área de g =
8 2
Respuestas a los Rompecabezas
Sección 6-1 Solución al Problema preliminar
A las 6:00, las manecillas del reloj forman una línea recta, Si x es la cantidad que se gastó en la pensión del caballo, en-
pero el segundero está en el 12. Después de eso, las maneci- tonces por un lado la pérdida de Blanca es 1270 + x2 - 540,
llas forman una línea recta aproximadamente cada 1 h, 5 1 1 1
min y 27 segundos. De modo que después de las 6:00 las ó x - 270, y por otro es # 270 + x, ó 135 + x.
2 4 4
manecillas de la hora y los minutos forman una línea recta a
Así,
las: 7:05:27, 8:10:55, 9:16:22, 10:21:49, 11:27:16, 12:32:44,
1
1:38:11, 2:43:38, 3:49:05, 4:54:33. 135 + x = x - 270
4
Sección 6-3 3
405 = x
4
Página 398 Observa que después de cruzar cada puente, el 4
príncipe se quedaba con la mitad de lo que llevaba menos x = # 405 = 540
3
una bolsa más de oro. Para determinar cuántas bolsas tenía
antes de cruzar el puente, podemos usar las operaciones in- La pérdida de Blanca es de 540 - 270, ó $270.
versas; esto es sumar 1 y multiplicar por 2. Al príncipe le
quedó una sola bolsa después de atravesar el cuarto puente.
Él debió haber tenido dos bolsas antes de darle la bolsa extra

© Lopez Mateos Editores. ISBN 978-607-95583-2-1, obra completa, versión electrónica, ISBN 978-607-95583-3-8, volumen 1, versión electrónica. Ejemplar asignado a: Helecto Villarroel gutierrez -
helecto@gmail.com. Fecha: 27 de octubre de 2014. Prohibida su modificación, copia o distribución.
R40 Respuestas a los problemas

Capítulo 7 Conexiones matemáticas 7-1

Evaluación 7-1A Comunicación

1. (a) 0 # 100 + 0 # 10 1 + 2 # 10 2 + 3 # 10 3
- - -
1. (a) 3s
1 2s
2 5s
3 6s
4 (b) 0s
1 0s
2 3s
3 2s
4 3. Un día no puede
1
(b) 2 # 102 + 0 # 101 + 6 # 100 + 0 # 10 1 + 6 # 10 2
- -
ser expresado como un decimal exacto pues no se pue-
# 2 # 1 # 0
(c) 3 10 + 1 10 + 2 10 + 0 10 + 1 10 + # -1
# -2 365
de escribir como un decimal exacto. 5. El diezmo significa
0 # 10 3 + 3 # 10 4 (d) 0 # 100 + 0 # 10 1 + 0 # 10 2 +
- - - -
el diez por ciento, ó 0.1. 7. Las respuestas pueden variar;
0 # 10 + 1 # 10 + 3 # 10 + 2 # 10
-3 -4 -5 -6
por ejemplo, una fracción puede ser escrita como un decimal
2. (a) 4356.78 (b) 4000.608 (c) 40,000.03 exacto si se puede escribir como una fracción con denomina-
(d) 0.2004007 3. (a) 536.0076 (b) 3.008 (c) 0.000436 dor que sea potencia de10. El denominador se puede escribir
(d) 5,000,000.2 4. (a) Treinta y cuatro centésimos como potencia de10 si sólo contiene factores de 2 y 5. Otros
(b) Veinte y treinta y cuatro centésimos (c) Dos y treinta factores pueden aparecer en el denominador si la fracción no
y cuatro milésimos (d) Treinta y cuatro millonésimos está en la forma más simple.
436 109 2516 629 - 316,027
5. (a) = (b) = (c) 28
1000 250 100 25 1000 Por ejemplo, en el denominador 35 tiene a 7 como factor,
35
281,902 140,951 - 43 - 6201 4
(d) = (e) (f ) pero en la forma más simple, , no tiene a 7 como factor.
10,000 5000 10 100 5
6. (a) sií (b) sií (c) sí (d) sí (e) sí (f) sí (g) no Solución abierta
(h) sií (i) no (j) sí 7. (a) 0.8 (b) 3.05 (c) 0.5 9. Las respuestas pueden variar, pero muchos países serán
(d) 0.03125 (e) 0.01152 (f ) 0.2128 (h) 0.08 (j) 0.4 similares al nuestro.
7 Aprendizaje colectivo
8. No termina; la fracción no termina cuando se escribe
60
como decimal. El denominador tiene un factor de 3 cuando está 11. Las respuestas pueden variar dependiendo de los obje-
en su forma más simple. 9. 3.56 10. Las respuestas pueden tos que se escojan. Por ejemplo, si una losa de 5 * 5 se usa
variar. Algunos de los números que están compuestos de poten- para representar 1, entonces una barra de 1 * 5 representa
cias de los números enteros 2 y 5 son 1, 5, 10, 25 y 50. Todos és- 0.1 y una unidad 1 * 1 representará 0.01.
tos dividen a 100 y se pueden escribir como un decimal de dos Preguntas del salón de clase
dígitos 0 y 1, pero hay otros. 11. (a) 13.492, 13.49199, 36 90
13.4919, 13.49183 (b) - 1.4053, - 1.45, - 1.453, - 1.493 13. El estudiante está equivocado. 0.36 = ; 0.9 =
100 100
12. (a) 0.014 pulg. (b) 365.24 días 13. (a) 90 36
y 7 , de modo que 0.9 7 0.36.
100 100
Evaluación 7-2A
32
Hay 32 cuadrados sombreados de 100, representados por 1. $231.24 2. (a) 8.2 1.9 6.4
(b) (i) Sí (ii) 19.05
100
3.7 5.5 7.3
de toda la cuadrícula, ó 0.32 de la cuadrícula. 14. 0.84
4.6 9.1 2.8
15. por ejemplo, 8.3401 16. (a) Un método para hacer
a (c) Sí; 8.25 3. 73.005 4. $27.746192, que nos lleva a la
esto es convertir ambos a la forma donde a, b 僆 E y b Z 0. cantidad de $27.75 en dólares canadienses 5. (a) Alrededor
b
Halla un número racional entre ellos cuyo denominador esté de $5.80 para calentar la casa por 1 día (b) 199 h (redondea-
en la forma más simple 2m # 5n. Convierte esta fracción a un do) 6. Aproximadamente 8.64 litros 7. 21.3 mph (redondeado
decimal exacto. (b) El proceso descrito en el inciso (a) se a la décima más cercana de mph) 8. (a) 5.4, 6.3, 7.2
puede usar para hallar decimales exactos mayores que el menor (b) 1.3, 1.5, 1.7 9. 1.12464 10. 0.2222 puede ser escrito
de los números y menores que el número original mayor. como la suma de una sucesión geométrica como sigue:
1 2 1 3
+ a b a b + a b a b + a b a b o como
17. (a) Si como 1 unidad usamos el bloque, entonces 0.613 lo 2 2 1 2 2
podemos representar con 6 losas, 1 barra y 3 cubos. 10 10 10 10 10 10 10
(b) No se puede poque el juego de cubos sólo puede represen- 0.2 + 10.2210.12 + 10.2210.122 + 10.2210.123.
tar a cuatro dígitos diferentes de cero. 18. Tres veintidós se
11. 0.06, 0.018, 0.0054, 0.00162
puede interpretar como trescientos veintidós, pero el prome-
12. (b) (a)
dio de bateo es 0.322. Un promedio de bateo no puede ser
0 0.5 1
mayor que 1.000. 19. Un significado puede ser como si-
gue: 3 # 60 + 1 # 6 1 + 4 # 6 2 + 5 # 6 3 en base 10. En base
- - -
13. No, el banco tiene $7.74 de más. 4. (a) 0.0000000032
seis, 10seis se usaría en lugar del 6. 20. Rhonda, Martha, (b) 3,200,000,000 (c) 0.42 (d) 620,000
Kathy, Molly, Emily 15. (a) 1.27 # 107 m (b) 4.486 # 109 km (c) 5 # 107 latas

© Lopez Mateos Editores. ISBN 978-607-95583-2-1, obra completa, versión electrónica, ISBN 978-607-95583-3-8, volumen 1, versión electrónica. Ejemplar asignado a: Helecto Villarroel gutierrez -
helecto@gmail.com. Fecha: 27 de octubre de 2014. Prohibida su modificación, copia o distribución.
Respuestas a los problemas R41

16. (a) 0.000000000753 g (b) 298,000 km> s Problemas de repaso


(c) 778,570,000 km 17. (a) 4.8 # 1028 (b) 4 # 107
15. 14.0479 = 1 # 101 + 4 # 100 + 0 # 10 + 4 # 10 2 +
-1 -
(c) 2 # 102 18. (a) 200 (b) 200 (c) 204 (d) 203.7
13 1
7 # 10 + 9 # 10
-3 -4
(e) 203.65 19. 19 mpg 20. 55 + 5 + 18, 78 dólares 17. Sí; por ejemplo, = = 0.5.
21. La estimación puede variar. Las respuestas exactas son 26 2
las siguientes: (a) 122.06 (b) 57.31 (c) 25.40 (d) 136.15 Evaluación 7-3A
-
22. Las respuestas pueden variar. 2.0 * 102 y 5.0 * 10 3
1. (a) 0.4 (b) 0.285714 (c) 0.27 (d) 0.06 (e) 0.026
23. 2.3 # 1 = 2.3; 8.7 # 9 = 78.3 24. 49,736.5281
(f) 0.01 (g) 0.83 (h) 0.076923 (i) 0.047619
25. Las respuestas pueden variar; por ejemplo, 40 # $8 = $320;
4 61 461
40a b = $10; de modo que su salario es de $330.
1 (j) 0.157894736842105263 2. (a) (b) (c)
9 99 330
4 5 - 211 -2
26. (a) 12 (b) 0.6 (c) 2b (d) 2b 27. (a), (b) y (d) (d) (e) (f) 3. 0.016 h
9 90 90
tienen igual cociente. 28. (a) 181.56 (b) 148.551 4. 1.454 7 1.454 7 1.454 7 - 1.454 = - 1.45
- - -
6 7 8
Conexiones matemáticas 7-2 5. Las respuestas pueden variar. , , , ó 0.857142,
7 8 9
Comunicación 0.875, 0.8 6. 0.01 7. La parte decimal que se repite está
1. Supón que se registra un depósito con $10.00 más que la canti- 22
determinada por el 7 en el denominador de . El denomi-
dad verdadera y se registra un cheque con $10.00 más que la 7
cantidad verdadera. El saldo será correcto. 3. Las respuestas pue- dor, 7, no es un factor de 10. 8. (a) 0.446355; seis dígitos
den variar; por ejemplo, muchas de las técnicas de estimación que
(b) 1.35775 La respuesta es un número racional y hay cua-
funcionan para la división de los números completos también fun-
tro dígitos que se repiten. 9. Sí. Los ceros y los nueves se
cionan para la división decimal. El algoritmo de la división larga es
pueden repetir. 10. (a) Las respuestas pueden variar; por
más eficaz cuando se usan buenos estimados. Además, los estima-
ejemplo, 3.221, 3.2211, 3.22111. (b) Las respuestas pueden
dos son importantes para determinar si la respuesta obtenida me-
variar; por ejemplo, 462.2425, 462.2426, 462.2427.
diante la división larga es razonable. Las técnicas de estimación
11. 0.472 12. (a) Las respuestas pueden variar; por ejem-
también se pueden usar para colocar el punto decimal en el
plo, 0.751, 0.752, 0.753. (b) Las respuestas pueden variar;
cociente cuando se dividen decimales. 5. Las respuestas pueden
por ejemplo, 0.334, 0.335, 0.336. 13. (a) 8 (b) 7
variar; por ejemplo, alinear los puntos decimales es como si usára-
1 1 1 1
mos el valor posicional. 14. (a) (i) (ii) (iii) (b) (c) 0.01
9 99 999 9999
Solución abierta 2 3 1 5 25
7. Las respuestas pueden variar; por ejemplo, la calculadora 15. (a) (b) ó (c) (d) (e) 10
9 9 3 9 9
se podría usar para explorar el resultado de colocar el punto 5 3 1 322 3122
decimal cuando se multiplica por una potencia de10 o de co- 16. (a) (b) o (c) (d)
99 999 333 99 999
locar el punto decimal cuando se multiplican dos decimales.
3 2009
9. Las respuestas pueden variar; sirve para apoyar. 17. 0.775 18. (a) (b)
10 990
Aprendizaje colectivo 19. 1.0; En esta representación los ceros se repiten pero no
11. Sigue el diagrama de flujo y juega el juego. añaden valor. 20. (a) No; por ejemplo, 0.3 + 0.6 = 1.
La respuesta podría ser sí, si consideramos 1.0 = 1 como un
Preguntas del salón de clase
decimal periódico. (b) Considera el valor posicional
13. Es evidente qué sucede cuando 0.5 lo elevamos a una cuando hallamos la suma. Aparecerán en la suma los bloques
1 10
potencia grande. Como a b =
1 1 que se repiten en los valores posicionales a la derecha después
y =
2 1024 220 de los dígitos del decimal que termina. (c) Ve la parte (a)
10 2
ca b d =
1 1 1 si se considera 0 como un bloque que se repite.
, entonces elevado a un entero
2 1,048,576 2 (d) Considera el ejemplo 0.20 + 0.03. La suma es 0.23,
positivo se acerca a 0. De hecho, cualquier número entre 0 y que no termina. Si 0.23 la convertimos a una fracción en su
1 cuando se eleva a un exponente suficientemente grande se forma más simple, el denominador tendrá otros factores di-
acercará a 0 tanto como queramos. A nivel de este curso, será ferentes a 2 y 5.
suficiente usar una calculadora para ver qué sucede cuando
Conexiones matemáticas 7-3
0.999 se eleva a una potencia grande. Usando la tecla x2 re-
petidamente, obtenemos: 0.998001, 0.9960058, 0.9920274, Comunicación
0.9841185, Á , cuyos valores aproximados son 0.9992, 2
1. (a) Matemáticamente, el costo es 66 ¢ pero en la prác-
0.9994, 0.9998,0.99916, Á , 0.9991024. Observamos que el 3
décimo término en la sucesión es menor que 0.5 y así, si se- tica el costo es 67¢. (b) Ver la parte (a). (c) El costo se
guimos elevando al cuadrado el resultado rápidamente se redondea hacia arriba. (d) La mayoría de las registradoras
acercará más y más a 0. no permiten decimales periódicos, de modo que las tiendas
de abarrotes no los usan. (e) Ver la parte (d).
© Lopez Mateos Editores. ISBN 978-607-95583-2-1, obra completa, versión electrónica, ISBN 978-607-95583-3-8, volumen 1, versión electrónica. Ejemplar asignado a: Helecto Villarroel gutierrez -
helecto@gmail.com. Fecha: 27 de octubre de 2014. Prohibida su modificación, copia o distribución.
R42 Respuestas a los problemas

3. Las respuestas pueden variar; por ejemplo, es más fácil positivos racionales. Suma 12 a cada uno de ellos y el resul-
calcular la suma de fracciones cuando tienen un denominador tado es una infinidad de números irracionales. 13. (a) R
1 5 (b) ⭋ (c) Q (d) ⭋ (e) R (f) R 14. (a) Q, R
común, como en + . Cuando hay decimales que no se
7 7 (b) N, I, Q, R (c) R, S (d) I, Q, R (e) Q, R
repiten, es más difícil sumar debido al alineamiento del lu- 15. (a) N, I, Q, R (b) Q, R (c) R, S (d) N, I, Q, R
gar posicional. Cuando los denominadores son diferentes y (e) Ninguno (f ) Q, R 16. (a) 64 (b) Ninguno
son decimales exactos es más fácil usar los decimales, (c) - 64 (d) Ninguno (e) Todos los números reales
2 1 mayores que cero (f) Ninguno 17. 6.4 pies 18. (a) 6 15
como en + = 0.40 + 0.25 = 0.65.
5 4 (b) 11 13 (c) 6 17 19. (a) - 3 2 3 2 (b) 2 253
(c) 5 2 -
3 2 (d) 3 20. (a) 5, 5 2 3 2, 52
3 4, 10
Solución abierta
5. (a) Cualquier entero n puede escribirse como un decimal 1 1 1
añadiendo .0 a la derecha. (b) (i) 0.6 (ii) 1 ó 1.0; 1 es la (b) Las respuestas pueden variar. 2, 2 4 , 2 4 , 2 4 , 1.
A2 A4 A8
forma más simple pues requiere menos símbolos. (iii) 1.06 3
10 11 12
(c) Se tendría que adaptar la multiplicación por la izquierda. 21. (a) 2 (b) 2 (c) 2 22. (a) 4 (b)
-4 2
d) Un decimal periódico puede escribirse como un número 5
a (c) (d) 23. (a) racional (b) racional
racional , donde b Z 0. Las fracciones se pueden multipli- 7 6
b
car y el producto se puede convertir a decimal. 7. La mayoría 24. (a) Sí, cuando a … 0. (b) (i) x Ú 12 ó x … - 12
7 (ii) todos los números reales x 25. Entre 18 y 19
preferiría como solución. n
3 + Propuesta1
Preguntas del salón de clase Propuesta1
26. Propuesta2 =
9. Las respuestas pueden variar. Una calculadora es una he- 2
rramienta que puede ayudar en los cálculos con los decima- Si la Propuesta1 es precisa, entonces sea x = Propuesta1 =
les. Puede ayudar a proporcionar las respuestas estimadas. Propuesta2.
n
Esto tiene que ver con el valor de las calculadoras como he- + x
x
rramienta. El valor de los decimales periódicos es un tema x =
diferente. Un crimen administrativo grave tiene que ver con 2
el redondeo de los centavos para depositar el dinero n
2x = + x
resultante en otra cuenta. Estas fracciones podrían ser el re- x
sultado de usar decimales periódicos y redondeo. 2x = n + x2
2

Dependiendo del redondeo y del contexto, el resultado de x2 = n


no usar dichos decimales nos puede llevar a serios x = 2n
problemas.
Conexiones matemáticas 7-4
Problemas de repaso
Comunicación
11. $22,761.95 13. 0.077. La regla dice que el punto se
coloca a cuatro lugares ó 0.0770. Como 0.077 y 0.0770 son 1. Los matemáticos probablemente intentaron ver que hay más
equivalentes, la regla funciona. números irracionales que racionales. Georg Cantor lo demostró.
3. Falso: 164 + 36 Z 164 + 136 5. No; 113 es un
Evaluación 7-4A número irracional. Así, cuando se expresa como decimal no ter-
-
4 1>3 25 1>3
mina ni se repite. 7. Nota que a b = a b y
1. Las respuestas pueden variar. Una respuesta es
0.232233222333 Á 2. (a) 16 (b) 2 (c) 3 25 4
3. 0.9, 0.998, 10.98, 0.98, 0.988, 0.9, 0.898 4. (a) Sí -1>4 1>4 1>4
25 1>3
a b = a b . Pues a b 6 a b ,
4 25 25
(b) No (c) No (d) Sí (e) Sí (f) Sí 5. (a) 15
25 4 4 4
(b) 13 (c) Imposible (d) 25 6. (a) 2.65 (b) 0.11 - -
4 1>4 4 1>3 25 1>3
7. (a) Falso; 12 + 0 (b) Falso; - 12 + 12 (c) Falso; tenemos a b 6 a b = a b .
12 # 12 (d) Verdadero; 12 - 12 = 0 8. Las 25 25 4
respuestas pueden variar; por ejemplo, 12, 13 y 15.
9. Las respuestas pueden variar. Por ejemplo, supón que el Solución abierta
siguiente patrón continúa: 0 .54544544454444 Á . 9. Las respuestas pueden variar; por ejemplo:
3
10. Las respuestas pueden variar; por ejemplo, (a) 11>2, p>6, 1 2>5, 0.505005000500005 Á , 13>10
0.5155155515555 Á . 11. La respuesta debe ser un nú- (b) 0.505005000500005 Á , 0.505105000500005 Á ,
mero irracional. Supón que fuera racional; entonces la dife- 0.505105100500005 Á , 0.505105100510005 Á , Á
rencia entre ese y otro número racional es racional. Enton-
Aprendizaje colectivo
ces tienes un número racional igual a un número irracional
24 5 12
—una contradicción. Por lo tanto, la suma no puede ser un 11. 3.72.4 = 3.710 = 3.7 5 = 2 3.712
número racional. 12. Hay una infinidad de números po-
© Lopez Mateos Editores. ISBN 978-607-95583-2-1, obra completa, versión electrónica, ISBN 978-607-95583-3-8, volumen 1, versión electrónica. Ejemplar asignado a: Helecto Villarroel gutierrez -
helecto@gmail.com. Fecha: 27 de octubre de 2014. Prohibida su modificación, copia o distribución.
Respuestas a los problemas R43

Preguntas del salón de clase (b) y

13. Para que un número sea racional, se debe poder es- 5

a
cribir de la forma con b Z 0 y a y b enteros. 12 no es un
b
entero. 15. La raíz cuadrada principal de 25 es 5 y no - 5. 0
x

Si bien es verdad que 1 - 522 = 25, - 5 no es la raíz cuadrada


−5 5

principal de 25; las raíces no tienen por qué ser iguales.


3. Puedes graficar la función pero los valores de x deben ser
17. El estudiante no está en lo correcto. Un ejemplo en
menores o iguales a 0. Un ejemplo de gráfica es la siguiente:
carpintería es encontrar la hipotenusa de un triángulo y
rectángulo. En general los resultados son estimados, pero si
5
las longitudes de los lados son 1 pulg y 1 pulg, la longitud
de la hipotenusa es 12 pulg. 19. Las respuestas pueden
variar. Hay anécdotas que apoyan al Sr. Brown. En este caso
x
se entienden menos los números decimales que los enteros. −5 0 5

Problemas de repaso

418 3 - 507 123 4. y = - x + b 5. La solución es (2.4, 2.2).


21. (a)
25
(b)
1000
(c)
100
(d)
1000 6. (a) 18 (b) 42 (c) - 2 (d) 12 ó - 12
-5
23.
3
25. (a) 208,000 (b) 0.00038 (e) 7 ó - 3 (f) - 5 ó 1 (g) 1 ó 7. (a) 2856 pies
12,500 2
(b) 4600 pies 8. una recta que pasa por el origen con
pendiente p 9. (a) 0.618 (b) Es una aproximación a una
Evaluación 7-5A solución de la ecuación. 10. y

1. (a) y

x x
0
−5 0 2 13 5

−5

Conexiones matemáticas 7-5


(b) y
Comunicación
1. Las respuestas varían dependiendo del capítulo leído.
5
Solución abierta
3. Las respuestas pueden variar. Hay muchas aproximacio-
x
0 5 62 10 nes de p.
3
Aprendizaje colectivo
5. Las respuestas varían dependiendo de la elección del libro
(c) y
de texto.
5
Preguntas del salón de clase
2.83
7. El estudiante no está en lo correcto. La ecuación represen-
ta una recta que pasa por los puntos (0, 0) y 11, 1 + 122.
x
−5 0 5

Problemas de repaso
2. (a) y

5 9. 2, 1.3, 1.7, 1.42 11. (a) 2.5 (b) 2.5 13. Las
respuestas pueden variar; por ejemplo, 1.21.

x
−5 0 5

© Lopez Mateos Editores. ISBN 978-607-95583-2-1, obra completa, versión electrónica, ISBN 978-607-95583-3-8, volumen 1, versión electrónica. Ejemplar asignado a: Helecto Villarroel gutierrez -
helecto@gmail.com. Fecha: 27 de octubre de 2014. Prohibida su modificación, copia o distribución.
R44 Respuestas a los problemas

Revisión del capítulo 19. (a) y

1. (a) A = 0.02, B = 0.05, C = 0.11 x


−5 0 5
32012 8003
(b) 0 0.1 0.2 2. (a) ó
A B D C E 1000 250
103 a −5
(b) 3. Una fracción en su forma más simple, ,
100,000 b
puede escribirse como un decimal exacto si, y sólo si, la facto- (b) y
rización en primos en el denominador contiene sólo los
5
primos 2 ó 5. 4. 8 estantes 5. (a) 0.571428 (b) 0.125
7 - 607 1 94
(c) 0.6 (d) 0.625 6. (a) (b) (c) (d)
25 100 3 45 x
0
7. (a) 307.63 (b) 307.6 (c) 308 (d) 300 −5 5

8. (a) 4.26 # 105 (b) 3.24 # 10 4 (c) 2.37 # 10 6


- -

(d) 3.25 # 10
- -1
9. 1.4519, 1.4519, 1.4519, 1.4519, - 11
20. a , b ó 14.3, 1.62 21. (a)
13 5
0.13401, - 0.134, - 0.13401. 10. (a) 5 12 ó - 5 12 (b) 36 (c) - 1
3 3 3
1 1 1 - 1 1 -1
(b) 4 , 4 , 4 ó 4 , 4 , 4 7 -2 1 1
14 116 164 14 116 164 (d) 3 ó - 3 (e) ó (f) - 2 ó 3 (g) ó 1 22.
2 3 2 p
11. (a) 1.78341156 # 106 (b) 3.47 # 10 6 (c) 4.93 # 109
-
23. No; las cajas registradoras no manejan números irracionales.
#
(d) 2.94 10 17 #
(e) 4.7 10 35
(f) 1.536 # 10 6
-

12. (a) Las respuestas pueden variar; por ejemplo: 0.105, Respuestas a Ahora intenta éste
0.104, 0.103, 0.102, 0.101 (b) Las respuestas pueden variar; 1 1
0.0005, 0.001, 0.002, 0.004 (c) 0.15, 0.175, 0.1875, 0.19375. 7-1. (a) ó 0.1 (b) ó 0.01
10 100
13. (a) Irracional (b) Irracional (c) Racional (d) Racional (c)
(e) Irracional (f) Irracional 14. (a) 11 12 (b) 12 12
(c) 615 (d) 32 3 6 15. (a) No; 12 + 1 - 122 es racio-
nal. (b) No; ve parte (a). (c) No; 12 # 12 es racional.
(d) No; 12> 12 es racional. 16. 4.796
7-2. En la tira cómica se reporta que el precio del tomate es
17. aproximadamente 1.26
.99¢, que es menor que 1¢. Este error se comete frecuente-
18. (a) y
mente en las tiendas cuando la cantidad es menor a 1 dólar;
5
por ejemplo, un objeto es marcado con .50¢ cuando en reali-
dad debería ser $0.50 ó 50¢. Nota que el empleado no elevó el
precio.
−5 0 2.5 5
x
3 3 # 53 375
7-3. (a) 2.5 (b) = 3 3 = = 0.375
8 2 #5 1000
(b) y
3 3#5 15 15
(c) = 2 = 2 2 = = 0.15
20 2 #5#5 2 #5 100
7-4. (a) 3.6 # 1000 = 3.6 # 103 = a 3 + b103 =
5 6
10
6 # 3
3 # 103 + 10 = 3 # 103 + 6 # 102 = 3 # 103 + 6 # 102 +
0 10 20 30 40 50
x 10
0 # 101 + 0 # 1 = 3600. Así, vemos que multiplicar por 1000
resulta en mover el punto decimal tres lugares a la derecha.
(b) En general, multiplicar por 10n, donde n es un entero
(c) y positivo, resulta en mover el punto decimal n lugares a la
5
derecha. 7-5. $1.19>32 oz es alrededor de $0.037>oz,
mientras que $1.43>48 oz es alrededor de $0.030>oz, así que
el frasco con 48 oz es una mejor compra. 7-6. Las respuestas
2
x
pueden variar; por ejemplo, al usar los dígitos de la izquierda el
0
−5 5
primer estimado es $2 + $0 + $6 + $4 + $5 = $17. A
continuación, ajustamos el estimado. Como $0.89 + $0.13
es alrededor de $1.00 y $0.75 + $0.05 es $0.80 y $0.80 +
$0.39 es alrededor de $1.20, el ajuste es $2.20 y el estimado
es de $19.20. 7-7. (a) La respuesta se redondea a
0.0000014.

© Lopez Mateos Editores. ISBN 978-607-95583-2-1, obra completa, versión electrónica, ISBN 978-607-95583-3-8, volumen 1, versión electrónica. Ejemplar asignado a: Helecto Villarroel gutierrez -
helecto@gmail.com. Fecha: 27 de octubre de 2014. Prohibida su modificación, copia o distribución.
Respuestas a los problemas R45

1 7–16. (a) x2 + 5x + 6 = 0
(b) La respuesta es 0.00000136. 7-8. (a) = 0.1 2
9 5 2
x2 + 5x + a b + 6 - a b
5
2 3 = 0
(b) (i) = 210.12 = 0.2 (ii) = 310.12 = 0.3 2 2
9 9 5 2
ax + b +
-1
5 8 = 0
(iii) = 510.12 = 0.5 (iv) = 810.12 = 0.8 2 4
9 9 5 2 1 2
a ax + b - a b = 0
7-9. (a) Si r = 1, el denominador de es 0. (b) Como 2 2
ax + - b ax + + b
1 - r 5 1 5 1
n se vuelve grande, también sucede con r n+1
cuando r 7 1. = 0
2 2 2 2
1x + 221x + 32 = 0
(c) Sea S = a + ar + ar 2 + Á + ar n
- rS = - 1ar + ar 2 + Á + ar n + ar n + 12 x = -2 ó -3
S - rS = a - ar n + 1
a11 - r n + 12 (b) 4 ó 3 (c) No hay una solución con un número real
S = cuando b2 - 4ac 6 0. En este curso no podemos realizar la
1 - r raíz cuadrada de números negativos.
235 129
7-10. 0.235 = , 2.345 = 7-11. Las respuestas pue-
999 55 Respuestas a los Rompecabezas
den variar. (a) 0.355; 0.365 (b) Con cada decimal periódico
(como 0.36), uno puede recorrer la mitad entre el número y los Sección 7–1
dos hallados para escribir más decimales periódicos, por ejemplo Dos decimales en base dos son 0.1dos y 0.01dos. Todo deci-
0.355 6 0.36 6 0.365, y continuar este proceso una y otra mal en base dos representa una fracción cuyo denomina-
vez. 7-12.(a) 113 L 3.6056 dor es una potencia de 2 y es un decimal exacto en base
n diez.
+ Propuesta1
Propuesta1
(b) Propuesta2 = Sección 7–4
2
7-13. (a) El enfoque funciona pues Marzo 14 es 3>14, los primeros tres dígitos de p. Muchas
1>2 1>2 1>2 escuelas tienen un evento especial a las 1:59 p.m. para cele-
2 2 1a = a a a1>2 b b = a a1>4 b brar los primeros seis dígitos de p (3.14159).

= a1>8 Solución al Rincón de la tecnología


8
= 1a
k
(b) Para n = 2 , donde k es un entero positivo. Como se mues- Sección 7-5
tra en la parte (a), aplicando de manera repetida la función raíz y

2k
cuadrada a a para obtener 2a.
5
7-14. Las rectas son simétricas con respecto a la recta y = x.
y
y 5 2x11
y5x
5
x
−5 0 5
(x21)
y5
2

x
−5 0 5

Solución al Problema preliminar


−5
Hay muchas respuestas. Entre ellas tenemos la siguiente:
.98765
7-15. Las rectas son paralelas; las pendientes son las mismas. + . 01234
y 0.99999, ó 1
5
y 5 2x12
y 5 2x11 y 5 2x Capítulo 8
y 5 2x21
y 5 2x22
x
Evaluación 8-1A
−5 0 5
1. (a) 5 : 21 (b) 21 : 5 (c) 21 : 26 (d) Las respuestas
pueden variar; por ejemplo, menor. 2. (a) 2 : 5. Como la
−5 razón es 2 : 3, hay 2x niños y 3x niñas; por lo tanto, la razón

© Lopez Mateos Editores. ISBN 978-607-95583-2-1, obra completa, versión electrónica, ISBN 978-607-95583-3-8, volumen 1, versión electrónica. Ejemplar asignado a: Helecto Villarroel gutierrez -
helecto@gmail.com. Fecha: 27 de octubre de 2014. Prohibida su modificación, copia o distribución.
R46 Respuestas a los problemas

2x 2 2 5 4
de niños a todos los estudiantes es = . las tres fracciones a su forma más simple, es decir , y , y
(b) m : 1m + n2 (c) 3 : 2
2x + 3x 5 3 7 5
observando que no hay dos iguales. 7. Las respuestas pueden
1 1 1
3. (a) 30 (b) - 3 (c) 23 (d) 10 4. 36 lb variar; por ejemplo, sea m el número de adultos hombres que
3 3 2 viven en el condominio y w el número de adultos mujeres. El
5. 12 toronjas por $1.80 6. 270 millas 7. 64 páginas número de hombres casados es igual al número de mujeres
8. (a) 42, 56 (b) 24 y 32 (ó - 24 y - 32) 2 3
9. $14,909.09; $29,818.18; $37,272.73 10. $77 y $99 casadas, de modo que m = w.
3 4
1 1 7 36 oz 48 oz La razón de personas casadas al total de personas adultas es
11. 135 12. (a) (b) (c) 13. = ;
6 1 12 12¢ 16¢
2m 3w 2m 2m 4m
12¢ 36 oz 16¢ 48 oz 5 + +
= ; = 14. (a) (b) 6 pies 3 4 3 3 3
16¢ 48 oz 12¢ 36 oz 7 = =
m + w m + w m + w
15. (a) 27 (b) 20 16. Aproximadamente 34 cm
2m 3w 8m
2 Como = , entonces w = . Así,
17. 312 lb 18. (a) cucharita de semillas de mostaza, 1 3 4 9
3
4m 4m 4m
1 2
taza de cebolla, 2 taza de habas (b) cucharita de semi- 3 3 3 12
6 3 = = =
1 m + w 8m 17m 17
llas de mostaza, 2 tazas salsa de tomate, 2 taza de habas m +
6 9 9
7 8
(c) cucharita de semillas de mostaza, 1 taza de salsa de que es la razón deseada.
13 13
21 Solución abierta
tomate, taza de cebolla 19. 15.12 Æ 20. alrededor
26 9. Las respuestas pueden variar. Por ejemplo, razones y pro-
de 74.6 cm 21. La razón entre la masa de oro en el anillo y la porciones se verán en el reporte del censo, los datos econó-
masa del anillo es 18>24. Si x es el número de onzas del oro micos de desempleados y la estadística del baloncesto.
puro en el anillo que pesa 0.4 onzas. tenemos 18>24 = x>0.4. 11. (a) 57.6 lb por pulgada cuadrada. (b) Las respuestas
Por lo tanto, x = 118 # 0.42>24, ó 0.3 oz. En consecuencia, pueden variar. Por ejemplo, distancia 1d2 =
el precio del oro en el anillo es de 0.3 # $300, ó $90. razón1r2 # tiempo 1t2.
22. (a) $320 (b) 8 23. (a) 1 : 2
a c e Aprendizaje colectivo
(b) Sea = = = r.
b d f 13. (a) Las respuestas pueden variar; muchas medidas se-
Entonces, a = br rán cercanas. (b) aproximadamente 36 cm (c) 1:1 ve a In-
c = dr ternet para observar el arte con las razones dadas (d) Las
e = fr respuestas pueden variar; por ejemplo, 1 : 6 es más probable.
Así, a + c + e = br + dr + fr (e) Las respuestas pueden variar.
a + c + e = r1b + d + f 2 Preguntas del salón de clase
a + c + e 15. Sí, ella está en lo correcto. Como cada razón es igual al
= r
b + d + f mismo número, entonces deben ser iguales entre ellas y por
Conexiones matemáticas 8-1 lo tanto forman una proporción. 17. Probablemente todos
pensaron que la altura del árbol era de 15 pies y trabajaron de
Comunicación
esa manera en la proporción. La proporción correcta compa-
1. (a) 40>700, ó 4>70, ó 2>35 (b) 525 cm (c) Para el rando el objeto con su sombra es
longitud de la huella 40 20 5 pies x pies
primer conjunto, = = ; = , así x = 50 pies.
longitud del fémur 100 50 3 15 pies
es decir, 50 cm de fémur corresponde a 20 cm de la huella. pies
2
Por lo tanto, no es probable que 50 cm de fémur sea de un
La clave es tener las mismas unidades para la sombra y la
animal que tenga 30 cm de huella. a Nota que .b
20 30 altura.
Z
50 50
Evaluación 8-2A
3. Sí, la proporción puede ser expresada en una variedad de
1
formas, pues lo siguiente es equivalente: 1. (a) 789% (b) 19,310%
(c) 83 % (d) 12.5%
3
a>b = c>d, a>c = b>d, b>a = d>c y c>a = d>b (e) 62.5% (f) 80% 2. (a) 0.16 (b) 0.002
Sin embargo, debemos mantener partes correspondientes en (c) 0.136 (d) 0.003 3. (a) 4 (b) 2 (c) 25
posiciones correctas. 5. No, las dimensiones no varían (d) 200 (e) 12.5 4. Depende de la calculadora.
proporcionalmente pues 4>6 Z 5>7 y 4>6 Z 8>10. Esto se 5x
5. (a) 2.04 (b) 50% (c) 60 (d) 3.43 6. (a)
puede ver verificando el producto cruzado o bien reduciendo 100
© Lopez Mateos Editores. ISBN 978-607-95583-2-1, obra completa, versión electrónica, ISBN 978-607-95583-3-8, volumen 1, versión electrónica. Ejemplar asignado a: Helecto Villarroel gutierrez -
helecto@gmail.com. Fecha: 27 de octubre de 2014. Prohibida su modificación, copia o distribución.
Respuestas a los problemas R47

(b) 10a 7. 63 cajas 8. $16,960 disminuye en q% y después la nueva cantidad aumenta en


p%, el salario final es de s a1 - b a1 + b . Como las
9. $25,500 10. (a) Beto vendió 221. (b) José vendió q p
90%. (c) Rafa empezó con 265. 11. 20% 12. apro- 100 100
ximadamente 23% de incremento 13. 100% dos expresiones son iguales por las propiedades conmutativa
14. $22.40 15. $336 16. 35% 17. $3200 y asociativa de la multiplicación, el estudiante tiene razón.
325 325 650
18. 1200 empleados 19. ; = = Problemas de repaso
500 500 1000
65 600 60 19. No, debió poner 30 onzas fluidas. 21. Siempre, pues
= 65%, mientras que = = 60%. 12x 3 # 4x 4x
100 1000 100 4x # 9 = 3 # 12x = 36x o bien = = .
9 3#3 3
20. (a) $76; $76 (b) Costaron lo mismo. 21. 11.1% ó aproxi-
madamente 11% 22. $440 23. (a) $3.30 (b) $24.00 Evaluación 8-3A
(c) $1.90 (d) $24.50 24. (a) Las respuestas pueden
variar. (b) (i) alrededor de 0.4 s entre latidos (ii) alrede- 1. Tasa Importe Cantidad
dor de 0.006 min entre latidos 25. El aprendiz ganó $700. de interés Número de del interés total en
El cabo ganó $1400. El maestro carpintero ganó $2100. por periodo periodos pagado la cuenta
26. (a) 4% (b) (i) 44 (ii) 8.8% 7. $82,644.63
28. (a) La respuesta es esencialmente verdadera. Gastar a 3% 4 $125.51 $1125.51
esa razón la cantidad será de $2.57 billones. (b) Aproximada- 2% 12 $268.24 $1268.24
damente 0.12% 29. No; los salarios respectivos serán apro- 10 5
% ó % ó 0.83% 60 $645.31 $1645.31
ximadamente $2000 y $1696.43. 30. (a) 3 (b) 0.003% 12 6
31. (a) $4.50 (b) 50% (c) 100% 32. 550 estudiantes 12
% 1460 $615.95 $1615.95
365
Conexiones matemáticas 8-2
Comunicación 2. $3675.00 3. $24.45 4. $43,059.50 5. $64,800
1. Las respuestas pueden variar; por ejemplo, 10% de 850 6. $12,905.80 7. $1015.20 8. (iii) paga a 13.2%
es 85 y 1% de 850 es 8.5, de modo que 11% de 850 es 93.5. 9. Aproximadamente $2.53 10. Aproximadamente
3. Significa que no sólo tienes el 100% de tus ahorros, 14.03% 11. $3483.81 12. El banco PagoMás ofrece
sino 25% más. Si tus ahorros eran de $100, en- tonces una mejor tasa. 13. Una sucesión aritmética con diferen-
ahora tienes $100 más $25 extra. 5. (a) Mayor, pues si el cia de 0 o una sucesión geométrica con razón de 1.
25% de x = 55, x debe ser 4 veces mayor que 55, ó
210. (b) Menor, pues si 150% de x = 55, x es sólo Conexiones matemáticas 8-3
2 2
de 55, ó 36 . 7. El todo en cada parte es diferente, de Comunicación
3 3
modo que el 50% de la cantidad mayor es mayor que el 50% 1. Sea a el valor original de la casa. Como se deprecia 10%
de la cantidad menor. Para que sean iguales, tendríamos que cada año durante 3 años, usando la depreciación compuesta
tener el mismo tamaño del todo para empezar. 9. Sea x la el precio después de 3 años será de a11 - 0.1023, ó a # 0.93.
cantidad invertida. El valor de la primera será 11.15x20.85 Debido a la depreciación compuesta, después de otros 3 años
después de 2 años. El valor de la segunda será 10.85x21.15. el valor de la casa será de a10.932 # 1.13, ó a10.93 # 1.132, lo cual
Como ambas son iguales a 11.15 # 0.852x, las inversiones son es aproximadamente igual a a # 0.9703. Como a # 0.9703 6 a,
igualmente buenas. el valor de la casa decreció en 6 años. El valor de la casa de-
creció en aproximadamente 3%. 3. No, los porcentajes
Solución abierta
no se pueden sumar porque cada porcentaje lo es de una
11. Las respuestas pueden variar. 13. Las respuestas pue- cantidad diferente. Después de 5 años, el auto se depreció
den variar. en un 67%.
Preguntas del salón de clase Solución abierta

A4B 1 5. Las respuestas pueden variar.


1 1 3
15. 3 % = 3% + % = + = 0.03 + 0.0025 = Aprendizaje colectivo
4 4 100 100
1 7. (a) Las respuestas pueden variar. (b) Las respuestas pue-
0.0325. Sabiendo que = 0.25, el estudiante se equivocó al
4 den variar. (c) Las respuestas pueden variar.
1
escribir % = 0.25. 17. Sea s el monto del salario. Después
4 Preguntas del salón de clase
de p% de aumento, el nuevo salario es s a 1 + b.
p
9. No, ella no tiene razón. Los hombres ganan 25¢ más por cada
100
Cuando esta cantidad disminuye en q%, el resultado es 25 1
75¢ que ganan las mujeres, así, el hombre tiene = 33 %
75 3
b a1 - b . Análogamente, si el salario inicial
p q
sa 1 + más o la mujer tiene 25% menos que el hombre.
100 100
© Lopez Mateos Editores. ISBN 978-607-95583-2-1, obra completa, versión electrónica, ISBN 978-607-95583-3-8, volumen 1, versión electrónica. Ejemplar asignado a: Helecto Villarroel gutierrez -
helecto@gmail.com. Fecha: 27 de octubre de 2014. Prohibida su modificación, copia o distribución.
R48 Respuestas a los problemas

Revisión del capítulo Sección 8-2


1. (a) 17 : 30 (b) 17 : 13 (c) 13 : 17 2. 64 onzas flui- Sea C = la cantidad de la corteza y P = la cantidad de pie;
das por $3.60 3. No, 18>6 Z 12>3. 4. (a) 16.8 (b) 192.5 x = el porcentaje que se reducirá la corteza.
1 C1100 - x2
= a b P.
(c) 1 5. 44 uvas y 8 naranjas 6. 7.5 m 20
4 C = 25% de P, de modo que
100 100
7. La razón de hidrógeno al total es de 1 : 9. Por lo tanto,
1 x 7 Por lo tanto, x = 20%.
= implica x = 1 onza. 8. 560 peces 9. No, el
9 16 9 Respuestas al Rincón de la tecnología
radio depende de cuántos procesadores vienen de cada planta.
10. 1 : r 2 11. (a) 18 : 7 (b) 18 : 25 12. (a) 1 : 5 Sección 8-2
(b) 8 : 15 13. 9 14. (a) 25% (b) 192 (c) 56.6
a. La columna D muestra que la mezcla al 30% se alcanza
(d) 20% 15. (a) 12.5% (b) 7.5% (c) 627%
después de que se mezclan 5 L de jugo de limón y 12 L de agua.
(d) 1.23% (e) 150% 16. (a) 0.60 (b) 0.006 (c) 1
5 #
17. $9280 18. 3.3% 19. 88.6% 20. $5750 b. 100 cambia la razón del jugo de limón al agua a
21. No hay diferencia, el descuento es siempre de 31.6%. 5 + x
22. $80 23. Aproximadamente 31% 24. Las respuestas pue- un porcentaje, donde x es el número de litros de agua añadi-
den variar; por ejemplo, si el vestido original costaba $100, dos y 5 + x es el total de litros en la mezcla.
entonces con el 60% de descuento el costo será de $ 40. Con
el cupón de 40% de descuento, el costo deberá ser $24. Este Respuesta al Problema preliminar
modelo puede ser utilizado para actualizar la lista de precios
Si el primer carro fue valuado en A dólares y el vendedor
del vestido. 25. Todos tienen significado matemático.
obtuvo el 10%, entonces tenemos A11.12 = $9999. Despe-
26. $15,000 27. Aproximadamente $15,110.69
jando A, tenemos A = $9090. Si el segundo carro fue
valuado en B dólares, entonces tenemos B10.92 = $9999.
Respuestas a Ahora intenta éste Despejando B, hallamos B = $11,110. Por lo tanto, el total
a c valuado por los dos carros fue de $9090 + $11,110 =
8-1. = $20,200. El vendedor recibió 21$99992 = $19,998 por los
b d
dos carros. Por lo tanto, el vendedor perdió
a # bd c # bd $20,200 - $19,998 = $202.
=
b 1 d 1
ad cb
=
1 1
ad = cb
ad = bc
11 3 13 1
8-2. (a) , 55% (b) , 60% (c) , 52% (d) , 20%
20 5 25 5
8-3. (a) Las respuestas pueden variar; por ejemplo, la ma-
yoría de las calculadoras convertirán la forma decimal de un
número a un porcentaje moviendo el punto decimal dos lu-
gares a la izquierda. Otras calculadoras colocarán el símbolo %
en la pantalla cuando se presione la tecla % .
1
(b) 33.3% 8-4. Aceitunas—37 %, 3 rebanadas;
2
1
Normal—12 %, 1 rebanada; Resto—50%, 4 rebanadas
2
Respuestas a los Rompecabezas
Sección 8-1
Recolecta datos sobre la proporción de la longitud del brazo
a la longitud de la nariz, midiendo los brazos y la nariz de los
los estudiantes en su clase. Encuentra el promedio de la
proporción, después úsala para encontrar la longitud espe-
rada de la nariz de la Estatua de la Libertad.

© Lopez Mateos Editores. ISBN 978-607-95583-2-1, obra completa, versión electrónica, ISBN 978-607-95583-3-8, volumen 1, versión electrónica. Ejemplar asignado a: Helecto Villarroel gutierrez -
helecto@gmail.com. Fecha: 27 de octubre de 2014. Prohibida su modificación, copia o distribución.
ÍNDICE

AAL. Ver ángulo, ángulo, lado, propiedad uso de restas repetidas para desarrollar, construcción de, 755–756
abaco, 141 166–167 correspondientes, 712, 780–781, 794
abscisa (coordenada x), 810 algoritmos para la multiplicación, 161–165 de un polígono, 700
acre, 858 multiplicación con factores de dos dígi- diedro, 691–692
acumulación, 182 tos, 164–165 exterior, 712
aditiva, propiedad multiplicación de retícula, 165 fracciones y, 342
en el sistema de numeración egipcio, 66 multiplicación por 10n, 162–164 giro, 942
en el sistema de numeración romano, 69 algoritmos para la resta, 132–133 homotecias y, 969
“Adivina mi regla”, 221 algún, 43 incluido, 756, 757–759
Adleman, Leonard, 310 Al-Jwârizmî, Muhammad ibn Musa, 196 inscrito, 792
agrupar altura interior, 686, 712
en el sistema de numeración de rayas, del cono, 734 llano, 689
65 del triángulo, 760, 784–785 medición del, 687–688
en el sistema de numeración egipcio, 65 altura de un paralelogramo, 860 obtuso, 689
números agradables, 182 altura inclinada, 897 opuestos por el vértice, 710
propiedad asociativa de la suma de nú- American Mathematical Society, 294 plano, 689
meros completos y, 115–116, 117 análisis de datos, 593 recto, 688, 689
ALA. Ver la propiedad ángulo, lado, cálculo de medianas, 633–634 suma de medidas
ángulo cálculo de medias, 631 de los ángulos exteriores de un n-
al azar, 520 comparación de conjuntos de datos, gono convexo, 717–719
álgebra 640–642 de los ángulos interiores de un polí-
descripción, 195–196 comprender la media como punto de gono convexo de n lados, 716–717
ecuaciones, 206–220 balance, 632–633 de un cuadrilátero, 704
fracciones en, 371–372 diagrama de caja, 638–639 de un triángulo, 713–715
funciones, 220–244 encontrar modas, 634–635 suplementario, 689, 711
historia del, 196 foráneos, 639–640 tipos de, 688–690
modelo del sumando faltante y, 120 medición de dispersión, 637–638 transversales y, 711–712
multiplicación y división de números medición de la tendencia central, ángulos adyacentes, 686
completos, 142 630–642 ángulos alternos externos, 712
suma y resta de números completos, 123 percentiles, 649–650 ángulos alternos internos, 712
variables, 197–205 selección del promedio adecuado, ángulos complementarios, 711
algoritmo de la marca para la suma, 132 635–636 ángulos congruentes, 700
algoritmo de retícula para la suma, 131 varianza, 642–649 construcción de, 755–756
algoritmo de sumas iguales, 133–135, 179 análisis de unidades, 840–841 verticales, 710
algoritmo expandido, 128 análisis dimensional (análisis de unidades), ángulos correspondientes, 712, 794
algoritmo para la suma de izquierda a de- 840–841 construcción de rectas paralelas y,
recha, 131 andamiaje, 166–167 780–781
algoritmo relajado, 132 ángulo agudo, 689 ángulos dihédricos, 691–692
algoritmos ángulo, ángulo, lado, propiedad (AAL), ángulos exteriores, 712
convencional, 127, 128 769 de un polígono convexo, 700
de intercambio, 179 ángulo, ángulo, semejanza de triángulos, suma de las medidas de los, de un n-
definición, 127 793 ágono convexo, 717–719
del cajero, 180 ángulo central de un círculo, 792, 868 ángulos interiores, 712
del producto cruzado, 480 ángulo de giro, 842 de polígonos, 700, 730
división, 151–152, 166–169 ángulo de incidencia, 961 suma de, de polígonos convexos,
de números racionales, 391–392 ángulo de reflexión, 961 716–717
de números racionales, alterna, ángulo incluido, 756, 757–759 ángulos planos, 689
392–393 ángulo inscrito, de un círculo, 792 ángulos verticales, 710
euclidiano, 319–321, 324 ángulo, lado, ángulo, propiedad (ALA), ángulos y rectas paralelas, propiedad, 712
expandido, 128 768–774 anillos, 273
para la multiplicación, 161–165 ángulo obtuso, 689 Annuities upon Lives (De Moivre), 648
para la resta, 132–133 ángulos, 686–690, 710–726 Ápice, 728
para la suma, 128–132 adyacentes, 686 Aplicaciones
sumandos iguales, 133–135, 179 agudo, 689 curvas normales, 648–649
algoritmos del producto cruzado, 480 alternos externos, 712 ecuaciones, 214–218
algoritmos para la división, 151–152, alternos internos, 712 homotecias, 971–973
166–169 caminos alrededor de estrellas, 719–720 porcentajes, 493–498
división corta, 168–169 central, 792 Apotema, 865
uso de cubos de base diez para desarro- complementarios, 711 aprendizaje colectivo, 3
llar, 167–168 congruentes, 700, 710, 748, 749 árbol de factores, 302–303, 304
I-1
© Lopez Mateos Editores. ISBN 978-607-95583-2-1, obra completa, versión electrónica, ISBN 978-607-95583-3-8, volumen 1, versión electrónica. Ejemplar asignado a: Helecto Villarroel gutierrez -
helecto@gmail.com. Fecha: 27 de octubre de 2014. Prohibida su modificación, copia o distribución.
I-2 ÍNDICE

Arco, 687, 749 babilonios funciones y, 224


longitud de, 849 fracciones y, 342 igualdad de fracciones con, 352
mayor, 749 funciones y, 221 máximo divisor común con, 317, 319
menor, 749 pi y, 679 multiplicar sin usar la tecla * con, 143
arco mayor, 749 sistema numérico, 67 notación científica con, 425, 426
arco menor, 749 balanceando con decimales permutaciones con, 577
Area, 858 en la división, 431 redondeo de decimales con, 431
Área, 854–876. Ver también área de super- en la resta, 430 resta repetida con, 151
ficie balanza, modelo de la, 206, 209–211 tecla de cambio de signo, 263
conversión de unidades de, 856–858 barra, en notación fraccionaria, 342 tecla de división entera, 288
de un círculo, 866–867 barra vertical, 637 tecla de factorial, 576
de un cuadrado, 450–451, 864 barras, 63–64 uso de, 461
de un papalote, 863–864 barras (huesos) de Napier, 165 calculadoras científicas
de un paralelogramo, 860 base exponentes racionales en, 455
de un polígono regular, 865–866 de un cilindro, 734 fracciones mixtas en, 367
de un rectángulo, 859–860 de un paralelogramo, 860 simplificación de fracciones en, 349–350
de un rombo, 864 de un prisma, 727 tasas de interés compuesto en, 507
de un sector, 868 de una pirámide, 728 calculadoras gráficas
de un trapecio, 864–865 base cinco, multiplicación en, 169, 171 desviación absoluta media en, 657
de un triángulo, 860–863 base cinco, sistema, 70–72 diagramas de caja en, 640
en un geotablero, 855–856 base cinco, suma en, 135–136 función RAND en, 564
fórmulas, 929 base diez, 63 gráficas de funciones, 231–232
medición de tierras, 858–859 base diez, bloques de notación de función en, 223
área de superficie, 894–906 para algoritmos de la resta, 132, 133 permutaciones en, 577
de la esfera, 900 para algoritmos de la suma, 128 caleidoscopio, 809
de prismas rectos, 894–896 para el algoritmo convencional de la di- cambio lineal, modelación del, 820
de una pirámide, 897–898 visión, 167–168 cancelación, propiedad en igualdades, 207,
del cilindro, 896–897 para el valor posicional, 63–64 211
del cono, 898–900 base doce, sistema de, 73–74 Cantor, Georg, 62, 78, 455
lateral, 895 base dos, multiplicación en, 171–172 capacidad, unidades de, 920–921
área, método de la suma, 855 base dos, sistema de, 72–73 capital, 505
área, modelos de base veinte, sistema de, 68 caras
fracciones y, 343 Bernoulli, Jakob, 519 de un poliedro, 726
la multiplicación, 145, 146 Bernoulli, teorema de, 519 laterales, 727, 728
probabilidad geométrica, 547–548 bicondicionales, 48 caras laterales
aristas de un poliedro, 726 bigotes, 639, 642 de un prisma, 727
Arithmetica (Diofanto), 196, 286 bimodal, 634 de una pirámide, 728
aritmética bisectores Cardano, Gerolamo, 249
del reloj, 329–332 ángulo, 781–782, 785–786 cardinales, números, 83–85
modular, 332–334 perpendiculares, 759, 760 cartesianos, sistemas coordenados,
aritmética del reloj, 329–332 bisectrices concurrentes, 786–787 esbozo, 810
división, 331 bisectrices de ángulos rectas y ecuaciones lineales en, 810–826
multiplicación, 330 construcción de, 781–782 catetos, de un triángulo, 877
resta, 331 propiedades de, 785–786 Cavalieri, Bonaventura, 912
suma, 330 Boole, George, 45, 197 Celsius, Anders, 922
aritmética modular, 332–334 Brahmagupta, 249 centímetro cuadrado, 856
Arquímedes, 848 centímetro cúbico, 908
Ars Conjectandi (El arte de la conjetura) cada, 43 centro
(Bernoulli), 519 caja (caja y bigotes), diagrama de, 638–639, de datos, 630
Ars Magna, 196 642 de gravedad, 799
asociación, 620–621 caja, diagramas de, 594 de giro, 942, 980
asociación negativa, 621 paralelos, 640–641 de rotación, 942
asociación positiva, 620–621 cajero, algoritmo del, 180 de un arco, 749
asociativas, propiedades de un círculo, 687, 748, 847
de la intersección de conjuntos, 96 calculadoras. Ver también calculadoras grá- de una esfera, 726
de la multiplicación, 207, 208 ficas; calculadoras científicas centroide, 799
de enteros, 273 algoritmo euclidiano con, 320 cero
de números completos, 147 aritmética modular con, 333 como lugar vacío, 64
de la suma, 207, 421 comparación de fracciones en, 446 división entre, 154–156
de enteros, 256 conversión de fracciones a decimales en el sistema de numeración maya, 68
de números completos, 115–116, 117 usando, 415 errores de redondeo y , 433
de la unión de conjuntos, 96 conversión de números de base diez a origen de la palabra, 112
de números reales, 455 números de diferentes bases propiedad de la identidad de la suma
asteriscos, para indicar foráneos, 639 usando, 72 para números completos, 116, 117
axiomas, 683–684 decimales periódicos con, 439, 441 cero absoluto, 922
división de decimales con, 414 Chudnovsky, David y Gregory, 848

© Lopez Mateos Editores. ISBN 978-607-95583-2-1, obra completa, versión electrónica, ISBN 978-607-95583-3-8, volumen 1, versión electrónica. Ejemplar asignado a: Helecto Villarroel gutierrez -
helecto@gmail.com. Fecha: 27 de octubre de 2014. Prohibida su modificación, copia o distribución.
ÍNDICE I-3

cilindro, 734 cia (LLL), 752–754 de círculos circunscritos a un triángulo,


área de la superficie del, 896–897 propiedades selectas del triángulo, 761–763
circular recto, 734, 913 759–760 de la mediatriz de un segmento, 760
recto, 734 teorema de la congruencia hipotenusa- de rectas paralelas, 780–781
volumen del, 911–913, 915 cateto (HC), 759 de rectas perpendiculares, 782–785
cilindro circular, 734 triángulo, 751–752 de reflexiones en papel punteado o geo-
cilindro circular oblicuo, 734 vía isometrías, 960–961 tablero, 958–959
cilindro oblicuo, 734 congruencia de triángulos, 751–752, de reflexiones mediante papel calca,
círculo, 687, 748, 749 759–760 955–958
ángulos de un, 792 ángulo, ángulo, lado (AAL), 769 de segmentos, 750
área de un, 866–867 ángulo lado ángulo (ALA), 768–774 de segmentos de recta, 750
circunferencia de un, 847–848 lado, ángulo, lado (LAL), 756, 757 de translaciones, 938–940
circunscrito alrededor de un triángulo, lado, lado, lado (LLL) hipotenusa- de un triángulo dados 3 lados, 754–755
761–763 cateto (HC), 752–754 incentro de un triángulo, 786–787
definición, 847 congruencia modular, 332 propiedades de las bisectrices de ángu-
fórmula de la distancia y ecuación de conjetura, 24–25 los, 785–786
un, 886–887 conjunción, 45 contar hacia atrás, 119
mayor, 900 conjunción, falacia, 540 conteo de combinaciones, 580
sector de un, 868 conjunto bien definido, 79 contradominio de una función, 224
círculo mayor, de una esfera, 900 conjunto de reemplazamiento de una va- contraejemplo, 24
circuncentro, 761 riable, 210 contrapositiva, 47, 48
circuncírculo, 761 conjunto finito, 84 conversión
circunferencia, 847–848 conjunto infinito, 84 de decimales periódicos a números ra-
circunradio, 761 conjunto vacío, 84 cionales, 442
circunscrito, 761 producto cartesiano y, 101 de fracciones a decimales, 413–415
Clarkson, Roland, 309 conjuntos, 78–93 de mediciones métricas de área, 857
clases, 602 ajenos, 94 de mediciones métricas de volumen,
clasificación de figuras por sus simetrías, complemento de un conjunto, 85–86 908–910
983 correspondencia biunívoca, 80–83 de mediciones métricas lineales, 843
clave de cifrado, 310 definición, 78 de medidas inglesas de volumen,
claves para descifrar, 310 desigualdades, 87–89 910–911
cociente, 150, 199 diagramas de Venn y, 85, 86, 94, 95, de números a porcentajes, 490–491
códigos, números primos y, 310 97–100 de números de base diez a distintas ba-
codominio de una función, 224 equivalentes, 83 ses numéricas, 72
combinaciones, 579–583 finitos, 84 de números racionales a decimales,
cometa, 702, 703 iguales, 80 413–415
área del, 863–864 infinitos, 84 de porcentajes a decimales, 491, 493
propiedades del, 772 lenguaje de, 78–80 de unidades de área, 856–858
Cómo mentir con Estadística (Huff), 664 nulo, 84 de unidades de medición, 840
Cómo plantear y resolver problemas (Polya), 3 números cardinales, 83–85 de unidades de medición lineal, 840,
comparación, modelo de la resta, números racionales, 342–362 843
119, 121 números reales, 454 coordenada x, 279, 618, 810
comparaciones parte a parte, 478 productos cartesianos, 100–101 coordenada y, 279, 618, 810
comparaciones parte a todo, 477 subconjuntos, 86–87 coplanares, 682
compás, 750, 756 número de, 89–90 correspondencia biunívoca (uno a uno),
complemento (evento), 523 universal, 85 80–83, 224, 226
complemento (conjunto), 85 vacío, 84 criba de Eratóstenes, 308, 309
de A respecto a B, 95 variables y, 197 cruce en x, 814
componentes, 100 cono, 734 cruce en y, 814
composición de dos funciones, área de superficie de un, 898–900 cruz de St. Andrew, 143
231–232 circular recto, 734, 913 cuadrado, 150, 702, 704
cómputo distribuido, 310 volumen de un, 913–916 área de, 450–451, 864
conceptos en planos, 682–686 en la jerarquía de polígonos, 703
conclusión, 46 cono circular oblicuo, 734 medición del ángulo interior, 730
condicionales, 46–52 constante, característica de, en una calcu- poliedros y, 731
congruencia. Ver también construcciones ladora, 143 propiedades, 773
geométricas constante de proporcionalidad, 481 cuadrados mágicos, problema de, 11–12
ángulo, lado, ángulo, propiedad (ALA), constantes, 197 cuadrantes, 279, 810
768–774 construcciones. Ver construcciones geomé- cuadriláteros
ángulos congruentes, 755–756 tricas clasificación, 704
aritmética del reloj, modular y, 250 construcciones euclidianas, 754 congruencia de, 774
de cuadriláteros y otras figuras, 774 construcciones geométricas, 749–750 embaldosado del plano con, 993–998
introducción, 747–749 con 2 lados y el ángulo incluido de un propiedades, 699, 702–703, 772–773
lado, ángulo, lado, propiedad (LAL), triángulo, 757–759 segmentos medios de, 798–799
756 de ángulos congruentes, 755–756 cuantificadores, 43–44
lado, lado, lado, condición de congruen- de bisectrices de ángulos, 781–782 existencial, 43

© Lopez Mateos Editores. ISBN 978-607-95583-2-1, obra completa, versión electrónica, ISBN 978-607-95583-3-8, volumen 1, versión electrónica. Ejemplar asignado a: Helecto Villarroel gutierrez -
helecto@gmail.com. Fecha: 27 de octubre de 2014. Prohibida su modificación, copia o distribución.
I-4 ÍNDICE

universal, 43 441–445 dispersión, 630, 636


cuartil inferior, 637 orden, 445–446 medición de, 637–638
cuartil superior, 637 decimales que no terminan, 439–450 Disraeli, Benjamin, 658
cuartiles, 642 decímetro cúbico, 908 distancia, 681
cubo, 729, 731, 732 De Moivre, Abraham, 648 alrededor de una figura plana, 845–847
truncado, 728 definición, en matemáticas, 710 medición de, 844
volumen de un, 915 demostraciones, 42, 713 propiedades de la, 844–845
cubos de base diez, 63–64 del teorema de Pitágoras, 877–878 distribución de probabilidad, 527
cubos de valor posicional, resta, 134 denominador, 342, 346 distribuciones normales, 647–648
cuerda, de un círculo, 749 denominadores comunes, 351, 354 disyunción, 45
cúmulos en diagramas de puntos, 598 densidad, propiedad de divide, definición, 285
curva cerrada, 698 números racionales y, 355–357 dividendo, 150
curva normal, 647 números reales y, 455 Divina Comedia (Dante), 517
aplicación de, 648–649 Descartes, René, 221, 732, 810 divisibilidad, 285–300
curvas, 697 descripción, 597 división
cerradas, 698 desde la izquierda con ajuste, 182 balanceo con decimales en la, 431
cóncavas, 698 desigualdad, 87–89, 210 corta, 168–169
convexas, 698 desigualdad del triángulo, 755, 845 de decimales, 414, 426–430
del copo de nieve, 808–809 deslizamientos. Ver traslaciones (desliz) de enteros, 275–276
simples, 698 desviación absoluta media (DAM), 630, de exponentes, 397
curvas cóncavas, 698 643–647 de números completos, 150–151
curvas convexas, 698 desviación común (standard), 630, 645–646 de números de tres dígitos, 170
curvas gaussianas, 648 diagonal de un polígono, 700 de números racionales, 388–393
diagramas de árbol en bases diferentes, 169–172
dado justo, 519, 569 para correspondencias biunívocas, en el reloj, 331
DAM. Ver desviación absoluta media 81–82 entre 0 y 1, 154–156
Dante, Alighieri, 517 para el modelo del producto cartesiano, entre un divisor de dos dígitos, 169
datos 146 estimación, 183–185
ajustar una recta a, 825–826 para espacios de muestra, 517 larga, 168
categóricos y numéricos, 595–596 para experimentos multietapa, 534–555 matemática mental, 181–182
continuos, 615 para subconjuntos, 89 números racionales en la, 342
discretos, 615 regla de multiplicación para probabili- relación con la multiplicación como
presentación de, 594–614 dades para, 537 operación inversa, 152–154
datos categóricos, 595-596 diagrama de Euler, 49 relación con la resta, 153
datos continuos, 615 diagrama ordenado de tallos y hojas, 599 división larga, 168
datos discretos, 615 diagramas divisores, 150, 285
datos numéricos, 595–596 construcción de, 6, 13–14 número de, 305–306
decágono, 699 de árbol (Ver diagramas de árbol) positivos, 300–301
decimales de Euler, 49 doblar papel
cálculo mental, 430–431 de flechas, 225–226 congruencia, 759
conversión de números racionales a, de Venn, 85, 86, 94, 95, 97–100 construcción de paralelas, 781
413–415 diagramas de dispersión, 594, 617–622 construcción de perpendiculares, 784,
conversión de porcentajes a, 491, 493 selección de, 623 697
división de, 414, 426–430 diagramas de flechas, funciones como, construcción de reflejos, 955
errores de redondeo, 433 225–226 investigación de simetrías, 981
esbozo, 410 diagramas de puntos, 594, 598–599 dobles, 118
estimación de cálculos con decimales diagramas de rectas, 594, 598–599, 600 doblez. Ver reflexiones (dobleces)
mediante redondeo, 432–433 diagramas de tallos y hojas , 594, 599–601 dodecaedro, 729, 731, 732
estimación de productos con, 438 lado a lado, 601–602 dominio, 224
exactos, 415–418 selección, 623
orden, 416–418 ordenados, 599
forma expandida, 413 diagramas de Venn, 85, 86, 94, 95 ecuaciones, 206–220
introducción a, 411–416 como herramienta para resolver proble- de rectas horizontales y verticales,
multiplicación de, 422–425, 438 mas, 97–100 810–811
notación científica, 425–426 diámetro, 749 de un círculo
notación fraccionaria, 413 dibujo en perspectiva, 971 con centro en 1h, k2, 887
operaciones con, 421–438 dibujo técnico y diseño, construcciones con centro en el origen, 886–887
periódicos, 439–446 geométricas y, 754 de una recta (Ver ecuaciones lineales)
que no terminan, 439–450 dibujos a escala, 485 equivalentes, 278
redondeo, 431 diferencia, 26, 199 escribir, 18, 202–203
resta de, 421, 423 diferencia común, 26 funciones como, 223–224
suma de, 421, 423 diferencia de conjuntos, 95–96 problemas de aplicación, 214–218
decimales periódicos, 439–446 dilatación, 968 propiedad de la suma y la resta de, 207
como fracciones, 443 Diophantus, 196, 286 resolver, 209–220
en forma a>b, donde a, b 僆 E, b Z 0, disección, 860 una ecuación con una incógnita,

© Lopez Mateos Editores. ISBN 978-607-95583-2-1, obra completa, versión electrónica, ISBN 978-607-95583-3-8, volumen 1, versión electrónica. Ejemplar asignado a: Helecto Villarroel gutierrez -
helecto@gmail.com. Fecha: 27 de octubre de 2014. Prohibida su modificación, copia o distribución.
ÍNDICE I-5

464–470 Eratóstenes, 308 examen de cálculos más sencillos , 161


con números completos, 213 Erlanger Programm (Klein), 936 existe, 43
traducción de palabras a, 215 errores de redondeo, 433 existe al menos, 43
uso de números reales en, 461–472 escala, 485 experimentalmente, 518
ecuaciones equivalentes, 278 en gráficas, 661–662 experimentos, 517. Ver también
ecuaciones lineales, 620, 811–815 escala de Celsius, 922 experimentos multietapas
definición, 815 escala de Fahrenheit, 922 de una etapa, 534
estimación de soluciones para, 463–464 escala de Kelvin, 922 experimentos de dos etapas, 534
forma ordenada pendiente, 814, 824 Escher, Maurits C., 990–991 experimentos unietapa, 534
método de eliminación, 822–823 esfera, 726, 727 experimentos multietapa
método de substitución, 821–822 área de superficie de, 900 con diagramas de árbol y probabilidades
sistemas de, 819–821 volumen de, 917–919 geométricas, 534–555
soluciones a, 823–825 espacio, 741 de dos etapas, 534
Efron, Bradley, 555 Espacio muestral, 517 eventos independientes, 538–543
ejercicios, problemas vs., 2 uniforme, 520 modelación de juegos, 543–547
ejes esperanza matemática, 569–572 modelos de área, 547–548
de rotación, 983 esperanzas probabilísticas, 516 exponenciación en el orden de las operacio-
en gráficas de rectas, 616 estadística, 593. Ver también análisis de da- nes, 276
x, 278, 279, 618, 810 tos; presentación de datos exponentes, 32
y, 278, 279, 618, 810 abusos con, 658–671 a a la m-ésima potencia, 394
El Código Da Vinci, 31 estimación. Ver también estimación com- números racionales y, 394–398
elemento identidad para la suma de enteros, putacional propiedades de la, 397, 456
256 con números racionales, 373–375, racionales, 455
elementos de un conjunto, 78 393–394 exponentes enteros negativos, 396
subconjuntos vs., 87 con porcentajes, 498–500 expresiones, 197–199
Elementos (Euclides), 319, 679, 912 de cálculos decimales mediante redon- expresiones algebraicas, 197–200
elipsis, 27, 79 deo, 432–433 extremo, 682
embaldosados del plano, 936, 990–1003 de productos con decimales, 438
dual de un embaldosado regular, 999 de raíces cuadradas, 453–454
regular, 992 de soluciones de ecuaciones lineales, factor de escala, 793, 968
semirregular, 993 463–464 factores comunes, 318
embaldosados con otras formas, estimación computacional, 182–183. Ver factores, 63, 146, 208, 285. Ver también
993–998 también estimación máximo factor común
embaldosados regulares, 992 definición, 178 factoriales, permutaciones y, 576–577
dual de, 999 multiplicación y división, 183–185 factorización, 149, 209, 302
empírico, 518 suma y resta, 182–183 factorización en primos, 302–305
encuestas, abuso/distorsión con, 659–660 estimación desde la izquierda, 182, 183, número de divisores y, 306
Enseñanza y Aprendizaje de Algoritmos 184 para hallar el máximo divisor común,
en las matemáticas escolares (Tea- estrategia de escalamiento, 483 316–317
ching and Learning Algorithms in estrategia de la razón unitaria, 483, 484 para hallar el mínimo múltiplo común,
School Mathematics), 127 estrategia de proponer y verificar, 14–15 322–323
enteros estrellas, caminos a su alrededor, 719–720 factorizar, 275
divisibilidad, 285–300 Estudio de una división regular del plano con Fairfax, Mary, 212
división de, 275–276 reptiles (Escher), 990–991 Fermat, Pierre de, 250, 286, 517
esbozo, 249–250 Euclides, 308, 319, 679, 683, 754, 912, Fibonacci (Leonardo di Pisa), 31, 64,
multiplicación de, 269–272 936 196, 202
propiedades de la multiplicación, Euler, Leonhard, 221, 732, 848 figura plana, distancia alrededor de,
272–275 evento seguro, 521 845–847
negativos, 250–251 evento imposible, 521 figuras conexas, 697
orden, 277–280 eventos, 517 figuras semejantes, 747–748, 793–809, 971
orden de las operaciones en, 264, 276 complementarios, 523 figuras simétricas, 982
positivo, 250–251 dependientes, 540, 541 figuras tridimensionales, simetrías de,
propiedades de la suma, 256–258 imposibles, 521 983–984
reglas de divisibilidad, 288–295 independientes, 538–543 Fisher, Ronald, 593
representaciones de, 250–251 mutuamente excluyentes, 522–523 flecha de deslizamiento, 937, 938
resta de, 258–264 no mutuamente excluyentes, 523–528 flecha de giro, 942
sistema de los números reales y, 454 seguros, 521 foráneos (outliers), 598, 639–640
suma de, 251–255 eventos complementarios, 523 forma expandida, 63
valor absoluto, 255–256 eventos dependientes, 540, 541 forma factorizada, 275
enteros impares, 285 eventos independientes, 538–543 forma pendiente-ordenada al origen de
enteros negativos, 249, 250–251 eventos mutuamente excluyentes, 522–523 una ecuación lineal, 814, 824
enteros pares, 285 eventos no mutuamente excluyentes, forma si-entonces, 46
entonces, en la forma si-entonces, 46 523–528 fórmula de diferencia de cuadrados, 275
equivalencia de una proposición, 48 Everest, George, 197 fórmula de Euler, 732
equivalente, igual vs., 83 Everest, Mary, 197 fórmula de la distancia, 883–886

© Lopez Mateos Editores. ISBN 978-607-95583-2-1, obra completa, versión electrónica, ISBN 978-607-95583-3-8, volumen 1, versión electrónica. Ejemplar asignado a: Helecto Villarroel gutierrez -
helecto@gmail.com. Fecha: 27 de octubre de 2014. Prohibida su modificación, copia o distribución.
I-6 ÍNDICE

ecuación de un círculo y la, 886–887 la, 697 heptágono, medición de los ángulos inte-
fórmulas orígenes de la, 679 riores, 730
de área, 929 planos perpendiculares y ángulos Herigone, Pierre, 687
de diferencia de cuadrados, 275 diédricos, 691–692 hexágono, 699
de Euler, 732 polígonos, 697–709 área de un, 865–866
de la distancia, 885 rectas perpendiculares, 691 congruente, 763
de la pendiente, 817 recta perpendicular a un plano, 691 embaldosados y, 991
del área de superficie, 898 geometría del movimiento, 936–937 medida de los ángulos interiores, 730
del volumen, 929 congruencia vía isometrías, 960–961 regular, 701
recursiva, 27 homotecias, 967–977 hipotenusa, 877
Fourier, Jean Joseph, 221 luz reflejada en una superficie, 961–962 hipótesis, 46
fracción impropia, 345 pendientes de rectas perpendiculares, histogramas, 594, 603–604
fracción propia, 345 944–946 abuso de, 661
fracciones, 64, 341. Ver también números reflexiones, 954–960 selección de, 623
racionales reflexiones deslizadas, 960 hoja de cálculo, 461
con denominadores distintos, orden, rotaciones, 942–944 desviación absoluta media en, 657
352, 354 simetrías, 978–990 estimado de soluciones de ecuaciones
con denominadores iguales, orden, 352 transformaciones, 937 lineales en, 463–464
conversión a decimales, 413–415 translaciones, 937–941 patrones recursivos y, 27
de referencia, 374 Geometría (Descartes), 221 problemas de mezclas en, 504–505
división de, 388–393 geotablero resolver una ecuación en una incógnita
en álgebra, 371–372 áreas en un, 855–856 en, 464–470
equivalentes o iguales, 345–349 construcción de reflexiones en un, sucesión de Fibonacci y, 32, 202
equivalentes para porcentajes, 498 958–959 tasas de interés compuesto y, 507, 508
escribir decimales periódicos como, 443 Germain, Sophie, 309, 310 trazo de otras funciones en, 462–463
expresión de probabilidades como, 526 giros. ver rotaciones (giros) variables en, 201–202
igualdad de, 351–352 grabados de Escher, 748 hojas, en los diagramas de tallos y hojas,
impropias, 345 grados celsius, 922 599
ley fundamental de las, 347, 349 grados de un ángulo, 687 homotecias, 967–977
propias, 345 graficación de pares ordenados, 227, 279 aplicaciones, 971–973
simplificación de, 349–351 gráficas, 594–595, 622 definición, 968
fracciones de referencia, 374 abuso con las, 660–663 hueco, en diagramas de punto, 598
fracciones equivalentes (iguales), 345–349 continuas, 662 Huff, Darrell, 664
búsqueda de, 348 de conjuntos, 810
fracciones mixtas, 365–367 funciones como, 227–230
multiplicación con, 386–387 para tablas de valores, 228 icosaedro, 729, 731, 732
fractales, 808–809 predicciones con, 619 icosaedro truncado, 731
frecuencia relativa, 518 gráficas circulares, 594, 606–607 identidad aditiva, 116
funciones, 220–244 abuso de, 663 identidad multiplicativa, 273
como diagramas de flechas, 225–226 engaños con, 663 de números racionales, 384
como ecuaciones, 223–224 selección de, 623 para números completos, 147
como gráficas, 227–230 gráficas continuas, abuso de, 662 identidad, propiedades
como máquinas, 221–223 gráficas de barra, 594, 598, 604–606 de la multiplicación de números
como reglas, 221, 222 apiladas, 605–606 completos, 147
como tablas y pares ordenados, 226–227 dobles, 605 de la suma de números completos, 116,
composición de, 231–232 engaños con, 661 117
definición, 223 porcentaje, 605 de los números reales, 455
relaciones, 232–235 selección de, 623 igual, equivalente vs., 83
sucesiones como, 230 gráfica de barras de porcentaje, 605 igualdad, 210
gráficas de barras dobles, 605 de fracciones, 351–352
Galileo, 912 gráficas de rectas, 594, 615–616 de pares ordenados, 100
galón, 910 abuso de, 661 propiedades de la, 206–207, 211
Galton, Francis, 593 quebradas, 621 propiedad de la suma, 369
Gardner, Martin, 994 selección de, 623 igualdad de conjuntos, definición, 80
Gauss, Carl Friedrich, 5, 250, 309, 648 gráficas de rectas quebradas, 621 imagen, 937, 938
generalizaciones, en razonamiento induc- gramo, 920 especular, 954
tivo, 24 Graunt, John, 593 imagen en el espejo, 954
Geographica, 308 gruesa, 73 implicaciones, 46
Geometer’s Sketchpad (GSP), 754 GSP. Ver Geometer’s Sketchpad importe (o saldo), 505
geometría. Ver también geometría del mo- In Artem Analyticam, 196
vimiento incentro, 786–787
ángulos (Ver ángulos) hacer 10, 118 incírculo, 787
conceptos básicos, 680–697 Harris, Sidney, 306 Indicaciones para el reporte de la evaluación e
conceptos planares, 682–686 HC. Ver teorema de congruencia hipote- instrucción para la educación en esta-
en tres dimensiones, 726–740 nusa-cateto dística (Marco estadístico), 593–594
etapas al desarrollar la comprensión de hectáreas, 858 índice, 451

© Lopez Mateos Editores. ISBN 978-607-95583-2-1, obra completa, versión electrónica, ISBN 978-607-95583-3-8, volumen 1, versión electrónica. Ejemplar asignado a: Helecto Villarroel gutierrez -
helecto@gmail.com. Fecha: 27 de octubre de 2014. Prohibida su modificación, copia o distribución.
ÍNDICE I-7

infinidad de primos, 308 ley de encadenamiento hacia adelante, 50 cálculo de, 631
input (entrada), 221 ley de los grandes números, 519 como punto de balance, 632–633
inscrito, 787 ley fundamental de las fracciones, 347, curva normal y, 647
intercambio, 130, 131 349, 391 definición, 631
intercambio, algoritmo de, 179, 180 leyes de De Morgan, 46 selección de, 635
interés, 505 Liber Abaci (Pisa), 31 media aritmética, 631
cálculo de, 505–512 libra, 920 mediana (estadística), 630, 636
compuesto, 506–509 límites de sumas finitas, 445 abuso de, 663–664
simple, 505 litro, 909 cálculo, 633–634
interés compuesto, 506–509 llevar, 130 selección, 635
interior de un ángulo, 686 LLL. Ver propiedad lado, lado, lado mediana (triángulo), 773–774, 799
intersección de conjuntos, 93–94 lógica. Ver razonamiento mediatriz, 759
propiedad asociativa de la, 96 lógicamente equivalente, 45 construcción de, 760
propiedad conmutativa de la, 96 longitud, arco, 849 medición
propiedad distributiva de la intersección Loomis, E., 877 área de superficie, 894–906
de conjuntos sobre la unión, 97 losas, 63–64 áreas de polígonos y círculos (Ver
intervalos en tablas de frecuencia, 600 Luo, pueblo de Kenya, sistema de base área)
inverso, 47 cinco y, 70–72 conversión de unidades de, 840
inverso aditivo, 256 ecuación de un círculo, 886–887
resta de números racionales e, 370 fórmula de la distancia, 883–887
unicidad del, 256–257 Mandelbrot, Benoît, 679, 808 indirecta, 800–801
inverso aditivo para números racionales, maquinas, funciones como, 221–223 lineal (Ver medida lineal)
propiedad del, 367 marcas, en tabla de frecuencias, 600 masa, 920–921
inverso multiplicativo de un número Marco estadístico, 596, 645 temperatura, 922–923
racional, 384 masa, 920 teorema de Pitágoras, 876–883
irracionalidad, de raíces cuadradas y de relaciones entre las unidades métricas volumen, 906–919, 920–921
otras raíces, 452–453 de capacidad, volumen y, 920–921 medición de la tendencia central,
isometría, 936, 937. Ver también reflexio- matemática mental, 178–179 630–642. Ver también media;
nes; rotaciones; translaciones con decimales, 430–431 mediana (estadística); moda
congruencia vía, 960–961 con números racionales, 393–394 medición de tierras, 858–859
reflexión deslizada, 960 con porcentajes, 498 medición lineal, 838–854
definición, 178 análisis dimensional, 840–841
división, 181–182 circunferencia de un círculo, 847–848
juego de atar cuerdas, 545–547 multiplicación, 181 conversión de unidades de medición,
juego de damas, 18 propiedad distributiva de la multiplica- 840
juego justo, 571 ción sobre la suma y, 149 distancia alrededor de una figura plana,
resta, 180 845–847
Kanada, Yasumasa, 450 suma, 179 longitud de arco, 849
Kerrich, John, 518 matemáticas, resolución de problemas y, propiedades de la distancia, 844–845
kilogramo, 920 3–22 sistema inglés, 839
kilómetro cuadrado, 856, 858 máximo divisor común (MDC), 315–321 sistema métrico, 841–844
Klein, Felix, 936 definición, 315, 318 mediciones indirectas, 800–801
Kolmogorov, Nikolaevich, 593 método de la calculadora, 317, 319 Mendel, Gregor, 593
método de las barras de colores, menor o igual que, 114
315–316 menor que, 87–89
La ciencia de compensar lo que falta y método del algoritmo euclidiano, 319– definición, 114
substraer los iguales, 196 321 en términos de la resta, 372–373
¿La dama o el tigre? (Smullyan), 44 método de la factorización en primos, para enteros, 277–278
La Thiende (La tienda) (Stevin), 410 316–317 relaciones en el conjunto de los núme-
lado, ángulo, lado (LAL), semejanza de método de la intersección de conjuntos, ros completos, 114
triángulos, 793 316 MEP. Ver máximo error posible
lado, lado, lado (LLL), semejanza de máximo error posible (MEP), 844 menos (operación), 95
triángulos, 793 máximo factor común (MFC), 317 Méré, Chevalier de, 517
lados maya, sistema de numeración, 68 Mersenne, Marin, 309
correspondientes, 794 mayas, numerales, 62, 68 método de división entre primos para ha-
de un ángulo, 686 mayor o igual que, 114 llar el mínimo múltiplo común,
de un polígono, 698 mayor que, 87–89 324–325
Lagrange, Joseph-Louis, 221 definición en términos de la resta, método de eliminación, 822–823
LAL. Ver lado, ángulo, lado, propiedad 372–373 método de intersección de conjuntos
Lambert, Johann, 848 para enteros, 277–278 para el máximo divisor común, 316
Laplace, Pierre de, 517, 648 relaciones en el conjunto de números para el mínimo múltiplo común, 322
Leibniz, Gottfried Wilhelm von, 143, completos, 114 método de invertir y multiplicar para divi-
221 MDC. Ver máximo divisor común dir números racionales, 391
lenguaje matemático, 3 media, 630 método de las barras de colores
Leonardo de Pisa (Fibonacci), 31, 64, abusos de, 663–664 para el máximo divisor común,
196, 202 aritmética, 631 315–316

© Lopez Mateos Editores. ISBN 978-607-95583-2-1, obra completa, versión electrónica, ISBN 978-607-95583-3-8, volumen 1, versión electrónica. Ejemplar asignado a: Helecto Villarroel gutierrez -
helecto@gmail.com. Fecha: 27 de octubre de 2014. Prohibida su modificación, copia o distribución.
I-8 ÍNDICE

para el mínimo múltiplo común, 322 119, 120, 121 modelo de la recta numérica para,
método de substitución, 821–822 modelo del algoritmo de Euclides 271–272
método del listado, 79 para hallar el máximo divisor común, modelo de la suma repetida, 143–146
método del producto máximo divisor co- 319–321 modelos de patrones para, 270
mún-mínimo múltiplo común, para hallar el mínimo múltiplo común, relacionada con la división como opera-
323–324 324 ción inversa, 152–154
método del rectángulo para el área, modelo de la recta numérica (medición) relacionada con la suma, 153
855–856 fracciones y, 343, 344–345 reloj, 330
método del rombo para el múltiplo común mínimo, retícula, 171
para bisecar un ángulo, 782 321–322 símbolos, 143, 206
para construir rectas paralelas, 780 para la división de números racionales, uso de números racionales en, 342
metro, 839, 841, 842 389, 390 multiplicación desde la izquierda, 181
metro cuadrado, 856 para la multiplicación, 143, 146, multiplicación, método del producto cru-
metro cúbico, 908 271–272 zado para la, 480–481
MFC. Ver máximo factor común para la resta, 259–260 multiplicación, propiedades
miligramo, 920 de números completos, 119, 121 de cero para números racionales, 385
milímetro cuadrado, 856 para la suma, 252–254 de desigualdad para números racionales,
milla, 839 de números completos, 113 385
cuadrada, 858 modelo de la resta repetida para la de igualdad, 206–207
MMC. Ver mínimo múltiplo común división, 151 de números racionales, 385
mínima expresión, 349 modelo de la suma repetida para la multi- de números completos, 147–150
mínima expresión de una fracción, 349 plicación, 143–146 de números racionales, 384–385
mínimo denominador común, 351 modelo del campo de carga en el sistema de numeración romano,
mínimo múltiplo común (MMC), 315, para la multiplicación, 270–271 69
321–325 para la resta, 258–259 propiedad asociativa de la multiplicación,
definición, 321 para la suma, 252 207
método de intersección de conjuntos, modelo del factor faltante para la división, de enteros, 273
322 150 de números completos, 147
método de la división entre primos, modelo del producto cartesiano de la mul- propiedad conmutativa de la multiplica-
324–325 tiplicación, 145–146 ción, 207
método de la factorización en primos, modelos de arreglos de enteros, 273
322–323 de la división, 153 de números completos, 147
método de la recta numérica, 321–322 de la multiplicación, 143, 145, 146 propiedad de la cerradura de la multi-
método de las barras de colores, 322 modelos de la resta del sumando faltante, plicación
método del algoritmo euclidiano, 324 119, 120–121, 180, 261–262 de enteros, 273
método del producto MDC-MMC, modelos de patrones para la de números completos, 147
323–324 multiplicación de enteros, 270 propiedad de la identidad de la multi-
minutos (ángulo), 687 modus ponens, 50 plicación
mira, 782, 784, 954, 955 modus tollens, 51 de números completos, 147
moda, 630, 636 Molecular and Microscopic Science (Fairfax), propiedad de la identidad multiplicativa,
abuso de, 663–664 212 273
en gráfica de puntos, 598 momios propiedad de la multiplicacion por cero
hallar la, 634–635 a favor, 564–568 de enteros, 273
selección, 635 cálculo de, 564–568 de números completos, 147
modelación de juegos, 543–547 definición, 565 propiedad distributiva de la multiplica-
modelo de barras de porcentaje, 492 en contra, 564–568 ción sobre la resta, 209
modelo de conjuntos (partición) moneda justa, 518 de enteros, 274–275
de la división, 150 Mouton, Gabriel, 841 de números completos, 148–150
fracciones y, 343 movimiento, 937 propiedad distributiva de la multiplica-
suma de números completos, 112 muestreo, 658–659 ción sobre la suma, 209
modelo de fichas multiplicación de enteros, 273
para la multiplicación, 270–271 base cinco, 169, 171 de números completos, 148–150
para la resta, 258 base dos, 171–172 propiedades de la multiplicación de en-
para la suma, 251–252 con fracciones mixtas, 386–387 teros, 272–275
modelo de la medición de decimales, 422–425, 438 multiplicación, regla para probabilidades
para la suma de números completos, de enteros, 269–272 para diagramas de árbol, 537
113 de exponentes, 397 multiplicación reticular, 165, 171
para la resta de números completos, de números completos, 142–146 múltiplo, 285
119, 121 definiciones, 145, 146
modelo de la partición para la división, de números racionales, 381–384 NAEP. Ver National Assessment of
150 en bases diferentes, 169–172 Educational Progress (Evaluación na-
modelo de patrones estimación, 183–185 cional del progreso educativo)
para la resta, 261 matemática mental, 181 NAEP. Ver National Assessment of Educatio-
para la suma, 254 modelo de campo de cargas, 270–271 nal Progress (Evaluación nacional
modelo de quitar elementos para la resta, modelo de fichas, 270–271 del progreso educativo)

© Lopez Mateos Editores. ISBN 978-607-95583-2-1, obra completa, versión electrónica, ISBN 978-607-95583-3-8, volumen 1, versión electrónica. Ejemplar asignado a: Helecto Villarroel gutierrez -
helecto@gmail.com. Fecha: 27 de octubre de 2014. Prohibida su modificación, copia o distribución.
ÍNDICE I-9

n-ágono, 699 numerales indoarábigos, 62, 63–65 números primos (primos), 26, 300–315
n factorial, 576 numerales romanos, 62 definición, 301, 304
Napier, John, 165 números. Ver también enteros determinar si un número es primo,
National Academy of Sciences, 294 cardinales, 83–85 306–308
preguntas de la National Assessment of completos (Ver números completos) infinidad de, 308
Educational Progress (NAEP) compuestos, 300–315 primos de Mersenne, 309–310
sobre área, 875 conversión a porcentaje, 490–491 Sophie Germain, 310
sobre área de superficie, 905 cuadrados, 35 números racionales, 454
sobre conjuntos, 93, 106 figurados, 34 como decimal exacto o periódico, 440,
sobre decimales, 437, 460 irracionales, 450, 454, 466 441
sobre decimales que no terminan, 450 naturales, 6, 85 conversión de decimales periódicos a,
sobre ecuaciones lineales, 831 negativos, 249, 250–251 442
sobre el múltiplo común mínimo, 328 primos, 26, 300–315 definición, 341
sobre el uso de números reales en ecua- racionales (Ver números racionales) densidad, 355–357
ciones, 472 reales (Ver números reales) división de, 388–393
sobre fracciones, 362, 404 transfinitos, 88 estimación con, 373–375
sobre fracciones mixtas, 366 triangulares, 34–36 extensión del concepto de exponente,
sobre funciones, 243 números agradables, agrupación en, 182 394–398
sobre geometría, 696, 726, 740 números compatibles, uso/construcción, fracciones, expectativas para, 341–345
sobre gráficas, 670–671 179, 181, 185 fracciones iguales o equivalentes,
sobre gráficas de barra, 614 con decimales, 430 345–349
sobre matemática mental, 188 números completos fracciones mixtas, 365–367, 368,
sobre medición de la tendencia central, álgebra elemental usando suma y resta 386–387
656–657 de números completos, 123 igualdad de fracciones, 351–352
sobre mediciones lineales, 854 algoritmos para la división, 151–152 menor que y mayor que para, 372–373
sobre multiplicación, 160, 176 cambiar un divisor decimal a, 428 multiplicación de, 381–384, 386–387
sobre notación decimal, 420 concepto de, 78 orden, 352–355
sobre números pares e impares, 299 conjunto de, 84, 111 propiedades de la multiplicación de,
sobre polígonos, 709 división de, 150–154 384–385
sobre porcentajes, 504 entre 0 y entre 1, 154–156 propiedades de la suma para, 367–369
sobre probabilidad, 533, 554, 587 estimación de, 178–179, 182–185 resta de, 369–372
sobre proporciones, 489 matemáticas mentales para, 178–182 simplificación de fracciones, 349–351
sobre resolución de problemas, 21 multiplicación de, 142–146, 152–154 suma de, 363–365
sobre resta de enteros, 269 por un decimal, 424 usos de, 342
sobre segmento de recta, 893 orden, 114 números reales, 450–460, 466
sobre semejanza, 808 orden de las operaciones, 156 en ecuaciones, 461–472
sobre sistemas de numeración, 77 propiedad distributiva de la multiplica- exponentes racionales, 455
sobre suma y resta, 141 ción sobre la suma y resta de , propiedades de los, 455
sobre variables, 205 148–150 propiedades de los exponentes, 456
sobre volumen, 928 propiedades de la multiplicación, radicales, 455
National Council of Teachers of Mathematics 147–150 raíces, 451–453
(NCTM), 62, 127 propiedades de la resta, 123 raíces cuadradas, 450–451, 452–454
NCTM. Ver National Council of Teachers of propiedades de la suma, 114–117 estimación, 453–454
Mathematics relación con los números reales, 454 irracionalidad de, 452–453
negación, 42–44 relación de la multiplicación y la sistema de los, 454–455
n-ésima, raíz, 451 división de, 152–154
n-ésimo, término, 27–28 resolución de ecuaciones con, 213
hallar para una sucesión geométrica, 34 resta de, 119–122 o incluyente, 45, 94
Newsweek, 659 suma, 112–114 objetos congruentes, 747–748
Newton, Isaac, 920 tablas de sumas, 118–119 observación, en razonamiento inductivo,
Nightingale, Florence, 593 números compuestos, 24
no asociación, 621 300–315 octaedro, 729, 730, 732
no coplanar, 683 números cuadrados, 35 octágono, 699
Noether, Emmy, 273 números de dos dígitos onza, 920
nonágono, 699 división, 169 operaciones, 286
notación científica, 425–426 mutliplicación, 171 con decimales, 421–438
notación constructora de conjuntos, 79 suma, 129 orden de las, 156
notación, división, 285 números de tres dígitos relación entre, 153
numerador, 342, 346 división de, 170 operaciones inversas, 119
numerales, 62 resta de, 134 relación entre multiplicación y división,
numerales arábigos, 62 números ficticios, 249 152–154
numerales babilonios, 62, 67 números figurados, 34 opuestos, 251
numerales egipcios, 62, 65–67 números irracionales, 450, 454, 466 orden de las operaciones, 156, 264
numerales griegos, 62 números naturales, 6, 85, 454 en enteros, 276
numerales hindús, 62 suma de, 7–8 ordenada (coordenada y), 810

© Lopez Mateos Editores. ISBN 978-607-95583-2-1, obra completa, versión electrónica, ISBN 978-607-95583-3-8, volumen 1, versión electrónica. Ejemplar asignado a: Helecto Villarroel gutierrez -
helecto@gmail.com. Fecha: 27 de octubre de 2014. Prohibida su modificación, copia o distribución.
I-10 ÍNDICE

origen, en sistema de coordenadas área de superficie de, 897–898 estimaciones con, 498–500
cartesiano, 810 recta, 728, 913 mayor que 100%, 494–495
Oughtred, William, 143 regular recta, 897, 899 matemática mental con, 498
output (salida), 221 volumen de, 913–916 potencias de 10, multiplicación por,
pirámide cuadrada, 728 162–164
pantógrafo, 801 pirámide pentagonal, 728 predicción mediante gráficas, 619
papel calca, construcción de reflexiones pirámide triangular, 728 prefijos, sistema métrico, 842
usando, 955–958 Pisano, Leonardo (Fibonacci), 31, 64, preimagen, 937, 938
papel punteado 196, 202 presentación de datos
áreas en, 855 pitagóricos, 729 diagrama de tallos y hojas de ambos la-
construcción de reflexiones en, 958–959 pixel, 279 dos, 601–602
paracaidismo, 466 plano coordenado, 279 diagramas de dispersión, 617–622
paralelogramo, 702, 704 plano de simetría, 983 diagramas de puntos, 598–599
área de, 860 planos, 680–681 diagramas de rectas, 615–616
en la jerarquía de polígonos, 703 intersección de, 685 diagramas de tallos y hojas, 599–601
propiedades del, 772 paralelos, 685 gráficas circulares, 606–607
paréntesis en el orden de las operaciones, perpendiculares, 691–692 gráficas de barras, 604–606
276 rectas perpendiculares a un, 691 histogramas, 603–604
pares ordenados, 100 relación con rectas, 686 pictogramas, 596–597
como relación, 232 semi-, 686, 692 selección de, 623
funciones como, 226–227 planos paralelos, 685 tablas de frecuencias agrupadas,
graficación de, 279 planos perpendiculares, 691–692 602–603
igualdad de, 100 planos que se intersecan, 685 primer cuartil, 637
partes congruentes, 700 Platón, 729, 750 primo de Sophie Germain, 310
partición del todo, 342, 343 poliedro, 726, 727 primos de Mersenne, 309–310
Pascal, Blaise, 517 convexo, 729 primos relativos, 317
patrón recursivo, 27 regular, 729–733 principio de Cavalieri, 911
patrones, 22–42 semirregular, 731 principio fundamental del conteo
números figurados, 34 truncado, 728 combinaciones y, 580
números triangulares, 34–36 poliedro convexo, 729 definición, 82
razonamiento inductivo, 24–26 poliedros regulares, 729–733 número de divisores y, 305–306
sucesiones aritméticas, 26–31 polígono convexo permutaciones y, 576
sucesión de fibonacci, 31–33 suma de las medidas de los ángulos inte- Principios y objetivos de las matemáticas
sucesiones geométricas, 32–34 riores de un, 716–717 escolares (Principles and Standards
variables y, 197 suma de las medidas de los ángulos ex- for School Mathematics) (PSSM)
patrones de multiplicación, 208 teriores de un, 717–719 sobre características del número, 300
Pearson, Karl, 518 polígono reticular, 871 sobre congruencia y semejanza, 747
pendiente, 813 polígonos, 697–709 sobre conjeturas, 24
de rectas perpendiculares, 944–946 ángulo de, 700 sobre decimales, 410
fórmula, 817 ángulo exterior de uno convexo, 700 sobre división de números racionales,
uso de triángulos semejantes para deter- área de, 854–876 388
minar, 815–819 regulares, 865–866 sobre el abuso de la estadística, 658
pentágono, 699 clasificación de, 699 sobre el método de la multiplicación
embaldosando con, 994–995 convexos, 716–719 cruzada, 483
medida del ángulo interior, 730 cuadriláteros (Ver cuadriláteros) sobre exploración de patrones, 22, 23
poliedro y, 731 diagonal de, 700 sobre fracciones, 341–342, 410
regular, 701 embaldosados regulares y, 992 sobre funciones, 220
pentomino, 998 jerarquía entre, 703–705 sobre geometría, 679–680
percentiles, 649–650 regular, 701, 865–866 sobre geometría del movimiento,
Perigal, Henry, 878 reticular, 871 936–937
perímetro, 845 segmentos y ángulos congruentes, 700 sobre gráficas, 595
periodo, 439 semejantes, 793-809 sobre habilidad computacional y algo-
permutaciones triángulos (Ver triángulos) ritmos convencionales, 127
con objetos similares, 578–579 polígonos de frecuencia, 594 sobre matemáticas mentales, 178
de objetos distintos, 575–578 polígonos equiangulares, 701 sobre mediciones, 838
de objetos en un conjunto, 577 polígonos equiláteros, 701 sobre mediciones de la tendencia cen-
peso, 920 polígonos regulares, 701 tral, 630
Philosophy and Fun of Algebra, 197 área de, 865–866 sobre números completos, 111
pi, 450, 679, 847–848 Polya, George, 2, 3, 214 sobre porcentajes, 410
Pick, G., 871 por, 143 sobre probabilidad, 516
pictogramas, 594, 596–597 porcentajes, 489–505 sobre propiedades de la suma, 115
pie, 839 aplicaciones, 493–498 sobre razonamiento algebraico, 195
pie cuadrado, 856 conversión de números a, 490–491 sobre razonamiento proporcional, 477
pie cúbico, 910 conversión a decimales, 491, 493 sobre recolección y análisis de datos,
pirámide, 728 definición, 490 593, 594

© Lopez Mateos Editores. ISBN 978-607-95583-2-1, obra completa, versión electrónica, ISBN 978-607-95583-3-8, volumen 1, versión electrónica. Ejemplar asignado a: Helecto Villarroel gutierrez -
helecto@gmail.com. Fecha: 27 de octubre de 2014. Prohibida su modificación, copia o distribución.
ÍNDICE I-11

sobre resolución de problemas, 2, 4 mixtas, 386 759–760


sobre resta de números completos, 119 para números completos, 148–150, 161, lado, ángulo, lado, propiedad (LAL),
sobre simulaciones, 557 164 756
sobre sistemas numéricos y relaciones para números racionales, 385 lado, lado, lado, condición de congruen-
entre números, 78 para números reales, 455 cia (LLL), 752–754
sobre tecnologías electrónicas, 461 razonamiento algebraico y, 209 triángulo, 751–752
sobre teoría de números, 249 propiedad distributiva de la multiplicación vía isometrías, 960–961
prisma, 727 sobre la resta proporciones, 478–484
oblicuo, 727 para enteros, 274–275 aplicación, 496
pentagonal, 727 para números completos, 148–150, 209 conversión de números a porcentajes
rectangular recto, 907 propiedad lado, ángulo, lado (LAL), 756, usando, 491
recto, 727 757 definición, 479
área de superficie de un, 894–896 propiedad lado, lado, lado (LLL), 752–754 dibujos a escala y, 485
truncado, 728 propiedad reflexiva de una relación, propiedades de, 796–798
volumen de, 911–913 233–234 proposiciones
prisma cuadrilátero recto, 727 propiedad transitiva, de una relación, 234 compuestas, 44–46
prisma hexagonal oblicuo, 727 propiedades definición, 42
prisma hexagonal recto, 727 asociatividad (Ver propiedades asociati- equivalencia de, 48
prisma triangular recto, 727 vas) implicación, 47
probabilidad teórica, 519 conmutatividad (Ver propiedades con- p si, y sólo si, q, 48
probabilidades mutativas) Ptolomeo, 342
combinaciones, 583 de exponentes, 397 pulgada, 839
condicionales, 568–569 de igualdad, 206 pulgada cuadrada, 856
de un evento con resultados igualmente propiedades de cancelación de la pulgada cúbica, 910
posibles, 519 igualdad, 207, 211 punto de referencia, 955
determinación de, 517–522 multiplicación, 206–207 punto decimal, 411
eventos complementarios, 523 suma, 206–207, 211 mover el, 427
eventos mutuamente excluyentes, de números reales, 455 punto fijo de una transformación, 965
522–523 de operaciones de conjuntos, 96–97 punto, para la multiplicación, 143
eventos no mutuamente excluyentes, distributividad (Ver bajo propiedad dis- puntos, 680–681
523–528 tributiva) colineales, 682
experimentos multietapas, 534–555 multiplicación (Ver propiedades de la no coplanares, 682
expresadas como fracción, 526 multiplicación) rectas que pasan por, 684–685
momios, 564–568 resta (Ver propiedades de la resta) puntos colineales, 682
números racionales y, 342 suma (Ver propiedades aditivas) Puntos focales en el currículo de matemáticas,
permutaciones, 575–579 propiedades conmutativas de las preescolares al grado 8, 62
teóricas, 519 de intersección de conjuntos, 96 sobre acopio y análisis de datos, 593,
uso de simulaciones, 555–564 de la multiplicación, 207, 208 594
valor esperado, 569–572 de enteros, 273 sobre algoritmos de la multiplicación y
probabilidades geométricas, 556 de números completos, 147 la división, 161
para experimentos multietapa, 536–555 de la suma, 207, 421 sobre área, 854
problema de Gauss, 6–7 de enteros, 256 sobre área de superficie, 894
problema de los conejos, 31 de números completos, 115 sobre área de un círculo, 866
problema del examen falso-verdadero, de números reales, 455 sobre decimales, 410
582–583 de unión de conjuntos, 96 sobre decimales que no terminan, 439
problema del tablero de ajedrez, 16, 18 propiedades de la cerradura sobre el orden en los números comple-
problemas, ejercicios vs., 2 de la multiplicación tos, 114
de enteros, 273 sobre el razonamiento algebraico,
producto, 146, 199, 208 de números completos, 147 195–196
parcial, 161, 164 de la suma sobre el teorema de Pitágoras, 878
reloj de doce horas, 330 de enteros, 256 sobre el volumen, 907
productos cartesianos, 100–101 de números completos, 114 sobre estimación, 178
definición, 101 de números reales, 455 sobre geometría del movimiento, 936
productos cruzados, 482 propiedades de la multiplicación por cero sobre la multiplicación y división de nú-
productos parciales, 161, 164 de enteros, 273 meros completos, 142
promedio, 630, 631 de números completos, 147, 148 sobre los números completos, 111
selección del más apropiado, 635–636 propiedades de la probabilidad, 524 sobre los números negativos, 251
propiedad de simetría, de relación, 234 propiedades de la resta sobre los números primos, 302
propiedad de substitución, 207 de ecuaciones, 207 sobre medición lineal, 843, 844
propiedad distributiva de la intersección de números completos, 123 sobre porcentajes, 493
de conjuntos sobre la unión, 97 en el sistema de numeración romano, sobre probabilidad, 516
propiedad distributiva de la multiplicación 69 sobre razonamiento proporcional, 477
sobre la suma propiedades del inverso de números rea- sobre resta de números completos, 119
para enteros, 273 les, 455 sobre sumas y restas multidígitos, 127
para la multiplicación de fracciones propiedades selectas del triángulo, sobre triángulos semejantes, 747

© Lopez Mateos Editores. ISBN 978-607-95583-2-1, obra completa, versión electrónica, ISBN 978-607-95583-3-8, volumen 1, versión electrónica. Ejemplar asignado a: Helecto Villarroel gutierrez -
helecto@gmail.com. Fecha: 27 de octubre de 2014. Prohibida su modificación, copia o distribución.
I-12 ÍNDICE

puntos no coplanares, 682 recta de deslizamiento, 937, 938 propiedades de las, 233–235
Pythagorean Proposition, The (Loomis), 877 recta de mejor ajuste, 621 propiedad reflexiva, 233–234
recta de tendencias, 825–826 propiedad de simetría, 234
es un diagrama de dispersión, 618 propiedad transitiva, 234
Quetelet, Adolph, 593 620–621 relaciones en un conjunto, 233
rectángulo, 702, 704 reloj de doce horas, sumas y productos,
área de un, 859–860 330
radián, 687 embaldosados, 995–997 reloj con cinco horas, 331–332
radicales, 455, 456 en la jerarquía de polígonos, 703 rendimiento anual efectivo, 507
radio, 748 propiedades del, 772 rep-tile, 748
radiolaria, 729 rectas concurrentes, 683 RSA, sistema, 310
raíces, 451–452 rectas coplanares, 682 residuo, 151
cuadradas (Ver raíces cuadradas) rectas horizontales, ecuaciones de, resolución de problemas, 3–22. Ver tam-
irracionalidad de, 452–453 810–811 bién razonamiento
raíces cuadradas, 450–451 rectas no coplanares, 683 búsqueda de un patrón, 6–7
estimación de, 453–454 rectas paralelas, 683 definición, 2
irracionalidad de, 452–453 construcción, 712–713, 780–781 diagramas de venn, 97–100
principal, 451 propiedades de ángulos y rectas parale- estrategia de proponer y verificar, 14–15
raíz cuadrada principal, 451 las, 712 examinar un caso más sencillo, 8, 9
RAND, función, 564 rectas perpendiculares, 691 examinar un problema relacionado, 7–8
rango, 630 construcción de, 782–785 hacer un diagrama, 13–14
definición, 637 pendientes de, 944–946 hacer una tabla, 8, 10
de una función, 224 rectas que se intersecan, 683 identificar un objetivo parcial, 11–12
intercuartil, 630, 634 rectas verticales, ecuaciones de, 810–811 introducción a, 2–3
uso del, 183 redes, 731–732, 733 patrones, 22–42
rango intercuartil (RIC), 630, 634, 637 redondeo, 183, 184 plantear una ecuación, 18
rayo, 682 decimales, 431 proceso de cuatro pasos, 4–5
razón, 477–478 estimación de cálculos con decimales proceso de polya para, 4–5, 214
números racionales y, 342 usando, 432–433 razonamiento directo, 18
sucesiones geométricas y, 32 fracciones, 373 razonamiento indirecto, 16, 18
razonamiento. Ver también resolución de reflexión de la luz en una superficie, razonamiento inductivo, 24–26
problemas 961–962 sucesiones aritméticas, 26–31
condicionales y bicondicionales, 46–52 reflexiones (dobleces), 954–960 trabajo regresivo, 16, 17
deductivo, 713 construcciones en papel punteado o en resta
directo, 18, 50 geotablero, 958–959 balanceo con decimales, 430
indirecto, 16, 18, 51, 452 construcción usando papel calca, de decimales, 423
inductivo, 24–26, 713 955–958 de enteros, 258–264
negación y cuantificadores, 42–44 definición, 955 de números completos, 119–122
proporcional, 477–489 deslizada, 960 de números de tres dígitos, 134
tablas de verdad y proposiciones com- en sistemas coordenados, 959–960 de números racionales, 369–372
puestas, 44–46 luz reflejada en una superficie, 961–962 definición, 261, 263
válido, 48–52 reflexiones deslizadas, 960 en bases distintas de diez, 135–136
razones. Ver también probabilidades regiones poligonales, 699 enfoque de sumar el opuesto, 262–264
dibujos a escala y, 485 regla de la cadena, 51 enfoque del sumando faltante, 261–262
proporciones y, 478–484 regla (para trazar), 750, 756 estimación, 183, 184
unidad, 840 reglas matemática mental, 180
razones unitarias, 840 cuerda, 451 mayor que o menor que en términos de,
reagrupamiento, 130 divisibilidad, 288–295 372–373
recíproco, 384 funciones como, 221, 222 modelo de campo de cargas, 258–259
recíproco de una proposición, 47, 207 para hacer una tabla, 228 modelo de la recta numérica, 259–260
recta, 680–681. Ver también rectas parale- reglas de divisibilidad, 288–295 modelo de las fichas, 258
las; rectas perpendiculares criterios de divisibilidad entre 2, 5, 6 y modelo del patrón, 261
concurrentes, 683 10, 289–290 propiedad distributiva de la multiplica-
coplanares, 682 criterios de divisibilidad para 3 y 9, ción sobre la resta, 148–150, 209
de ajuste, 619 292–293 para enteros, 274–275
de ajuste a datos, 825–826 criterios de divisibilidad para 4 y 8, relación con la división, 153
ecuación de la horizontal y la vertical, 290–291 relación con la suma, 121, 122, 153
810–811 criterios de divisibilidad para 11 y 6, reloj, 331
hallar la ecuación de una, 620 293–295 resta o substracción repetida para desa-
pendiente de una, 813, 815–819 relación aditiva, 480 rrollar el algoritmo convencional de
perpendicular a un plano, 691 relación de equivalencia, 234 la división, 166–167
puntos en una, 684–685 relación multiplicativa, 480 símbolo para, 113
que se intersecan, 683 relaciones, 232–233, 286 resultado, 517
reflejo en una, 954, 955 definición, 233 resultados igualmente posibles, 519
recta de ajuste, 619 funciones y, 232 RIC. Ver rango intercuartil

© Lopez Mateos Editores. ISBN 978-607-95583-2-1, obra completa, versión electrónica, ISBN 978-607-95583-3-8, volumen 1, versión electrónica. Ejemplar asignado a: Helecto Villarroel gutierrez -
helecto@gmail.com. Fecha: 27 de octubre de 2014. Prohibida su modificación, copia o distribución.
ÍNDICE I-13

Rice, Marjorie, 994 de figuras tridimensionales, 983–984 suan pan, 141


Rivest, Robert, 310 plano de, 983 subconjunto propio, 86
Robinson, Julia, 294 punto, 981 subconjuntos, 86–87
rombo, 702, 704 recta, 978–979, 982 elementos vs., 87
área del, 864 rotacional, 980–981 números de, en un conjunto, 89–90
construcciones y, 780 simetrías de giro, 980–981 propios, 86
en la jerarquía de polígonos, 703 simetrías respecto a una recta, 978–979, sucesión de Fibonacci, 31–33, 201–202
propiedades del, 773 982 sucesiones
Roper, Elmer, 659 simetrías rotacionales (de giros), aritméticas, 26–31, 230
rotaciones (giros), 942–944, 957 980–981 como funciones, 230
definición, 942 simulaciones de Fibonacci, 31–33
pendientes de rectas perpendiculares y, definición, 555 de números figurados, 34
944–946 uso de probabilidades en, 555–564 de números cuadrados, 35
Rudolff, Christoff, 451 Sir Cumference and the Dragon of Pi, 450 de números triangulares, 34–36
Russell, Bertrand, 680 sistema binario, 72–73 definición, 26
sistema coordenado geométricas, 32–34, 230
cartesiano, 810–826 sucesiones aritméticas, 26–29, 230
saldo, 505 extendido, 278–280 generalización de, 29–31
Scientific American, 994 reflexiones en el, 959–960 sucesiones geométricas, 32–34, 230
Scott Foresman-Addison Wesley Mathematics, traslaciones en el, 940–941 hallar el término n-ésimo de, 34
Grade 6, 262 sistema duodecimal, 73–74 sucesiones geométricas infinitas para con-
sector de un círculo, 775 sistema quintario, 70-72 vertir decimales periódicos en nú-
área de, 868 sistema de numeración, 62–77 meros racionales, 442
segmento (recta), 682 babilonio, 67 Sulbasutra (regla de las cuerdas), 451
congruente, 700 base cinco, 70–72 suma
segmento de recta, 682 base doce, 73–74 aprender las tablas de la, 118–119
congruente, 748, 749 base dos, 72–73 base cinco, 135–136
construcción, 750 definición, 62 de decimales, 421, 423
construcción de la mediatriz de un, 760 de muescas, 65 de enteros, 251–255
división en partes congruentes, 797 egipcio, 65–67 de fracciones mixtas, 368
homotecias y, 969 fracciones en el, 342 de números completos, 112–114
mediatriz, 783 indoarábigo, 63–65 de números de dos dígitos, 129
segmento unitario, 113 maya, 68 de números racionales
segmentos congruentes, 700 romano, 69–70 con denominadores iguales, 363
segmentos medios, 798–799 sistema de numeración indoarábigo, con denominadores distintos,
seguir contando, 118 63–65 364–365
segundos (ángulo), 687 sistema inglés de medición, 838, 839 de opuestos, para la resta, 262–264
semejanza, 972 unidades de área, 856, 858 en bases distintas de diez, 135–136
semicírculo, 749, 849 unidades de longitud, 839 en el reloj, 330
semiplanos, 686, 692 unidades de masa, 920 estimación, 182–183, 184
semivuelta, 943–944 unidades de volumen, 910–911 matemática mental, 179
separar y juntar, 179, 180, 430 sistema métrico, 838, 841–844 modelo de la recta numérica, 252–254
Shamir, Adi, 310 prefijos, 842 modelo de las fichas, 251–252
sí, y sólo si, 48 unidades de área, 856–857 modelo del campo de carga, 252
signo de resta, 250 conversión, 857 modelo del patrón, 254
signo más, 113 unidades de longitud, 839–844 relación con la multiplicación, 153
símbolo de menos, 113, 250 conversión, 843 relación con la resta, 121, 122, 153
símbolos unidades de masa, 920–921 resta de números racionales en términos
ángulo, 687 unidades de volumen, 908, 920–921 de la, 370
ángulo recto, 688 conversión, 908–910 símbolo para, 113
conjunto, 78 sistemas de conteo, 62 suma, 112, 199
en proposiciones compuestas, 44 Smullyan, Raymond, 44 reloj de doce horas, 330
entero negativo, 250 sociedad pitagórica, 451 suma, algoritmos para la, 128–132
menos, 113, 250 sólido, 726, 727, 733 de izquierda a derecha, 131
multiplicación, 143, 206 platónico, 729 de la marca, 132
para prefijos métricos, 842 sólidos platónicos, 729 relajado, 132
para unidades de masa, 920 soltura computacional, 127 retícula, 131
para unidades métricas de volumen, 909 soluciones suma, propiedades
pictograma, 597 de ecuaciones, 123 asociativa (agrupamiento), 207
porcentaje, 491 de ecuaciones lineales, 463–464 de enteros, 256
resta, 113 de sistemas de ecuaciones lineales, de números completos, 115–116, 117
suma, 113 823–825 cerradura
simetría respecto a un punto, 981 soroban, 141 de enteros, 256
simetrías, 936, 978–990 Stevin, Simon, 410 de números completos, 114
clasificación de figuras por, 983 Stockton, Frank, 44 conmutativa, 207

© Lopez Mateos Editores. ISBN 978-607-95583-2-1, obra completa, versión electrónica, ISBN 978-607-95583-3-8, volumen 1, versión electrónica. Ejemplar asignado a: Helecto Villarroel gutierrez -
helecto@gmail.com. Fecha: 27 de octubre de 2014. Prohibida su modificación, copia o distribución.
I-14 ÍNDICE

de enteros, 256 sobre decimales que no terminan, 449 triángulo isósceles, 702, 703, 728, 760
de números completos, 115 sobre enteros negativos, 285 triángulo obtusángulo, 701
de ecuaciones, 207 sobre fracciones, 362, 381, 404 triángulos 30°-60°-90°, 881–882
de enteros, 256–258 sobre funciones, 243–244 triángulos 45°-45°-90°, 880–881
de igualdad, 206–207, 211, 369 sobre geometría, 696, 726, 792 triángulos
de números completos, 114–117 sobre medición lineal, 854 acutángulos, 701
de números racionales, 367–369 sobre multiplicación y división, 160, alturas de, 760, 784–785
distributiva de la multiplicación sobre la 176 área de, 860–863
suma, 148–150, 209 sobre polígonos, 709 clasificación de, 701–702
para enteros, 273 sobre porcentajes, 504 construcción de, con 2 lados y el ángulo
para números racionales, 385 sobre probabilidades, 533, 554 incluido, 757–759
elemento identidad para la suma de en- sobre proporciones, 489 construcción de, dados 3 lados, 754–755
teros, 256 sobre rectas, 831 construcción del círculo circunscrito,
identidad (cero) para la suma de núme- sobre resolución de ecuaciones, 220 761–763
ros completos, 116, 117 sobre resolución de problemas, 21 de Pascal, 586escaleno, 701, 703
sumando, 112 sobre simetría, 989 esféricos, 715
faltante, 120 sobre sistemas de numeración, 77 equiláteros, 702, 703
método del, 180 sobre sucesiones, 41 incentro, 786–787
superficie lateral sobre suma de enteros, 269 isósceles, 702, 703, 728, 760
del cono, 734 sobre suma y resta, 126 mediana, 799
del cilindro, 734 sobre transformaciones, 953 medida del ángulo interior, 730
superficies cerradas simples, 726–728 sobre triángulos semejantes, 807 obtuso, 701
suplementos, 689 sobre variables, 205 para embaldosados, 997
sobre volumen, 927–928 poliedros y, 730–731
terna pitagórica, 892 propiedades, 699
tabla de números aleatorios, 557–561 tetraedro, 729, 730, 732 teorema de Pitágoras, 876–883
tablas TIMSS. Ver Tercer Estudio Internacional rectángulo (Ver triángulos rectángulos)
construcción de, 8, 10, 29 sobre las Matemáticas y la Ciencia segmentos medios, 798–799
de valores, 228 títulos de gráficas, 597 semejantes (See triángulos semejantes)
de verdad, 44–46 todo, 43 suma de las medidas de los ángulos inte-
funciones como, 226–227 todo-a-parte, comparación, 477 riores, 713–715
tablas, 121, 122 tonelada, 920 teorema de la congruencia Hipotenusa-
que relacionan la multiplicación y la di- tonelada métrica, 920 Cateto, 759
visión, 153 trabajo en grupo, 3 triángulos rectángulos, 701
tablas de frecuencia, 594, 597, 600 trabajo regresivo, 16, 17, 28 30°-60°-90°, 881–882
agrupadas, 602–603 traducción de frases a expresiones 45°-45°-90°, 880–881
tablas de sumar, 118–119 algebraicas, 197–200 especial, 880–882
Tales de Mileto, 800 traducción a ecuaciones, 215 triángulos semejantes, 793–809
tallos, en diagramas de tallos y hojas, 599 transformación identidad, 943 ángulo, semejanza de ángulo en, 793
tangente, 787 transformaciones, 937, 972. Ver también mediciones indirectas y, 800–801
tasas de interés, 505 rotaciones; traslaciones propiedades de proporción, 796–798
sucesiones geométricas y, 33 homotecias, 967–977 segmentos medios de, 798–799
tautología, 51 identidad, 943 semejanza lado ángulo, lado, de, 793
temperatura, 922–923 pendientes de rectas perpendiculares y, semejanza lado, lado, lado, de, 793
tendencia, 825–826 944–946 uso de, para determinar pendientes,
en gráfica de rectas, 616 transitividad, 51 815–819
teorema de congruencia hipotenusa-cateto transportador, 687
(HC), 759 transversal, 711–712
teorema de factorización única, 303 trapecio isósceles, 702, 703 último teorema de Fermat, 286
teorema de los segmentos medios, 798 propiedades del, 772 unidades (bloques de base diez), 63
teorema de Pitágoras, 452, 876–883 trapezoide, 702, 703, 704 unidades cúbicas, 906, 908, 910
fórmula de la distancia y, 883–886 área de, 864–865 unidades de medición
recíproco de, 882–883 en la jerarquía de polígonos, 703 área, 854
teorema fundamental de la aritmética, isósceles, 702, 703, 772 conversión, 856–858
303, 452 propiedades de, 772 lineal, 839–844
teoremas, 206, 683–684, 710 traslaciones (deslizamientos), 937–938 masa, 920–921
teoría de conjuntos, 78 construcciones de, 938–940 relaciones entre unidades métricas de
teoría de los números, 249–250 definición, 938 volumen, capacidad y masa,
Tercer Estudio Internacional sobre las representación coordenada de, 940–941 920–921
Matemáticas y la Ciencia (Third In- tren de un color, 305 temperatura, 922
ternational Mathematics and Science triángulo acutángulo, 701 volumen, 906, 908–911, 920–921
Study (TIMSS)), preguntas triángulo de Pascal, 586 unión de conjuntos, 94–95
sobre área, 875 triángulo equilátero, 702, 703 propiedad asociativa de la, 96
sobre congruencia, 768, 779 triángulo escaleno, 701, 703 propiedad conmutativa de la, 96
sobre decimales, 437 triángulo esférico, 715 propiedad distributiva de la intersección

© Lopez Mateos Editores. ISBN 978-607-95583-2-1, obra completa, versión electrónica, ISBN 978-607-95583-3-8, volumen 1, versión electrónica. Ejemplar asignado a: Helecto Villarroel gutierrez -
helecto@gmail.com. Fecha: 27 de octubre de 2014. Prohibida su modificación, copia o distribución.
ÍNDICE I-15

de conjuntos sobre la unión, 97


uno (1), división entre, 154–156
universo, 85
uso del rango, 183

valor absoluto, 255–256


definición, 255
valor decimal posicional, 417
valor esperado, 569–572
valor nominal, 63
valor posicional
decimal, 417
en el sistema de numeración
indoarábigo, 63
en el sistema de numeración babilonio,
67
estimación, 183
van Ceulen, Ludolph, 848
van Hiele, Dina y Pierre, 697
variables, 197–205
conjunto de reemplazo, 210
letras usuales para, 197
variación, 642–643
diagramas de caja, 638–639
desviación absoluta media, 643–647
desviación común (standard), 645–646
rango, 630, 634, 637
varianza, 630, 645
variación proporcional, 481, 482
varianza, 630, 645
vector, 937, 938
Venn, John, 85
vértice
de un ángulo, 686
de un cono, 734
de un poliedro, 726
de un polígono, 698
Viete, Francois, 196
volados, 518–519, 568–569, 571–572
volumen, 906–919
conversión de mediciones inglesas de,
910–911
conversión de mediciones métricas de,
908–910
de esferas, 917–919
de pirámides y conos, 913–916
de prismas rectangulares rectos, 907
de prismas y cilindros, 911–913
fórmulas, 929
relaciones entre unidades métricas de
capacidad, masa y, 920–921
von Koch, Helge, 808

Widman, Johannes, 113


Wiles, Andrew, 286

y, 44
yarda cuadrada, 856
yarda cúbica, 910

© Lopez Mateos Editores. ISBN 978-607-95583-2-1, obra completa, versión electrónica, ISBN 978-607-95583-3-8, volumen 1, versión electrónica. Ejemplar asignado a: Helecto Villarroel gutierrez -
helecto@gmail.com. Fecha: 27 de octubre de 2014. Prohibida su modificación, copia o distribución.
Glosario de símbolos
símbolo significado símbolo significado
= igual a 6 menor que
Ú mayor o igual que
an término n-ésimo de una sucesión
… menor o igual que
an a a la potencia n f1x2 f de x, valor de f en x
Sn suma de los primeros n términos de 1a, b2 par ordenado
la sucesión
1 g ⴰf 21x2
p ¡q
g1 f1x22
poq
-
a opuesto de a o el
¬p negación de p
inverso aditivo de a
p ¿q pyq
ƒaƒ valor absoluto de a
K logicamente equivalente
aƒb a divide a b
p:q p implica q
aⱈb a no divide a b
p4q p implica q y q implica p
1 raíz cuadrada principal
4cinco 4 base cinco
MDC máximo divisor común
O2Ddoce en base diez significa
11 # 122 + 2 # 121 + 10 # 1 MMC mínimo múltiplo común

5a, b, c6 conjunto que contiene los elementos 䊝 䊟, 䊞, s


4 operaciones aritméticas en reloj
a, b, y c a K b 1mod n2 a es congruente con b módulo n
5x ƒ Á 6
fracción “a sobre b” o razón 1b Z 02
notación constructora de conjuntos a
僆 es un elemento de b a , b
僆 no es un elemento de 3 3
5 fracción mixta 5 +
⭋o56 conjunto vacío 4 4

( es un subconjunto propio de a0 1, a Z 0
-n
8 es un subconjunto de a 1
,a Z 0
an
´ unión de conjuntos
⬟ aproximadamente igual
¨ intersección de conjuntos
L aproximadamente igual
U conjunto universal
0.18 decimal periódico 0.18181818 Á
A el complemento de A
2
n la raíz n-ésima
B - A diferencia de conjuntos o el
1
complemento de A respecto a B a n raíz n-ésima de a
m
n
A ' B el conjunto A es equivalente al an 2am
conjunto B % porcentaje
A * B producto cartesiano de los conjuntos
P1E2 probabilidad de un evento
AyB
n1C2 número cardinal del conjunto C n1E2 números de elementos en E
7 mayor que P1A2 probabilidad del
complemento de A

© Lopez Mateos Editores. ISBN 978-607-95583-2-1, obra completa, versión electrónica, ISBN 978-607-95583-3-8, volumen 1, versión electrónica. Ejemplar asignado a: Helecto Villarroel gutierrez -
helecto@gmail.com. Fecha: 27 de octubre de 2014. Prohibida su modificación, copia o distribución.
símbolo significado símbolo significado
n! n factorial, es igual a : ángulo recto o de 90°
n1n - 121n - 22 # Á # 3 # 2 # 1
⬵ es congruente al
para n Ú 1
µ arco menor que conecta A
0! cero factorial, que es igual a 1 AB
con B a lo largo de un círculo
nPr número de permutaciones de
n objetos escogiendo r cada vez ¬ arco mayor
ACB
nCr número de combinaciones de ^ ABC triángulo ABC
n objetos escogiendo r cada vez ' es similar a
P1B ƒ A2 probailidad condicional de que ocurra y = mx + b forma pendiente-ordenada al origen
el evento B dado que ocurrió A de una recta no vertical
E esperanza matemática
m pendiente de una recta
x la media aritmética de una sucesión b cruce en y u ordenada al origen
de números
k kilo (1000)
DAM desviación absoluta media
c centi (0.01)
n varianza
m mili (0.001)
s desviación común (standard)
km kilómetro
RIQ rango intercuartil
m metro
Qn cuartil n-ésimo
cm centímetro (0.01 m)
Dn decil n-ésimo
mm milímetro (0.001 m)
Pn percentil n-ésimo
p pi, la razón entre la circunferencia y el
A, B, C, Á puntos diámetro de un círculo
l, m, n rectas 100°C 100 grados Celsius
Í !
AB recta que pasa por A y B 32°F 32 grados Fahrenheit
!
AB rayo AB L litro
AB segmento AB g gramo
AB longitud del segmento AB V ovolumen
m ƒƒ n m es paralela a n S.A. área de superficie
– BAC ángulo BAC A¿ la imagen de A
° símbolo de grado 1x, y2 : una translación que mueve 1x, y2
¿ símbolo de minutos 1x + a, y + b2 a 1x + a, y + b2

– símbolo de segundos
m1–BAC2 medición de – BAC
⬜ es perpendicular a

© Lopez Mateos Editores. ISBN 978-607-95583-2-1, obra completa, versión electrónica, ISBN 978-607-95583-3-8, volumen 1, versión electrónica. Ejemplar asignado a: Helecto Villarroel gutierrez -
helecto@gmail.com. Fecha: 27 de octubre de 2014. Prohibida su modificación, copia o distribución.
V O L U M E N U N O

Formas ocultas
1. Un cubo
2. Un cilindro
3. Una pirámide cuadrada
4. Una estrella de 5 picos
5. Un prisma rectangular
6. Un cono

Al cubrir los contenidos de matemáticas de la currícula de la educación básica,


se convierte en el libro de texto ideal para la formación de maestros, pero no
sólo eso, también como el soporte adecuado para el proceso de actualización de
maestros de educación básica en servicio, para que, con un conocimiento sólido
de los contenidos académico de matemáticas, los maestros adquieran confianza
y seguridad en los cursos que imparten, mejoren su metodología y capacidad
didáctica y, finalmente, estén en óptimas condiciones para acoplarse a la inevi-
table evolución de planes y programas de estudio.

Matemáticas
un enfoque de resolución de problemas
para Maestros de Educación Básica

ISBN 978-607-95583-2-1
IS B N 9 78 -6 0 7 -9 5 5 8 3 -2-1. Ob r a c o m p le t a , ve r s i ó n e le ct r ó n ica .

9 786079 558321
IS B N 9 78 -6 0 7 -9 5 5 8 3 -3 -8 . Vo l u m e n u n o, ve r s i ó n e le ct r ó n ica .
López
editores

Mateos
www.lopezmateos.mx
© Lopez Mateos Editores. ISBN 978-607-95583-2-1, obra completa, versión electrónica, ISBN 978-607-95583-3-8, volumen 1, versión electrónica. Ejemplar asignado a: Helecto Villarroel gutierrez -
helecto@gmail.com. Fecha: 27 de octubre de 2014. Prohibida su modificación, copia o distribución.

Das könnte Ihnen auch gefallen